You are on page 1of 781

AGEMENT

ACCOUNTING

J. MADEGOWDA, Ph.D.,
Professor and Chairman,
Department of Post-Graduate Studies J .# I'

and Research in Business Administration,


Commerce and Tourism Administration,
Kuvempu University, Jnana Sahyadri,
Shankaraghatta - 577 451.
Shimoga District,
(Karnataka, India)

~
Gflimalaya GJlublishingGflouse
MUMBAI • DELHI • NAGPUR • BANGALORE • HVDERABAD
© Himalaya Publishing House, 2007
No part of this book shall be reproduced, reprinted or translated for any purpose wnatsoever
without prior permission of the Publisher in writing.

ISBN : 978-81-83189-72-9
First Edition: 2007

Published by Mrs. Meena Pandey


for HIMALAYA PUBLISHING HOUSE,
"Ramdoot", Dr. Bhalerao Marg, Girgaon,
Mumbai-400 004.
Phones: 2386 01 70 / 2386 38 63
Fax: 022 - 2387 71 78
E-Mail: himpub@vsnl.net.in
Website: www.himpub.com
Branch Offices
Delhi "Pooja Apartments", 4-8, Murari Lal Street,
Ansari Road, Darya Ganj,
New Delhi-110 002.
Phone: 2327 03 92, Fax: 2324 30 30
Fax: 011 - 2325 62 86
Nagpur Kundanlal Chandak Industrial Estate,
Ghat Road, Nagpur-440 018.
Phone: 272 12 16, Telefax: 0712-272 12 15 i

Bangalore No.16/1, (Old 12/1), 1st Floor ~


Next to Hotel Highlands, Madhava Nagar,
Race Course Road, Bangalore - 560 001.
Phone: 2228 15 41, 2238 54 61
Telefax:080-2228 66 11
Hyderabad No.2-2-1167/2H, 1st Floor, Near Railway Bridge,
Tilak Nagar, Main Road,

~
Hyderabad - 500 044.
Phone : 5550 17 45, Fax: 040-2756 00 41

DTPby Srt Siddhi Softtek


Bangalore

Printed by Geetanjali Press Pvt. Ltd.


Ghat Road, Nagpur - 18
CONTENTS

CHAFfER PAGE
No. CHAPTER No.

I FUNDAMENTALS OF MANAGEMENT ACCOUNTING 1- 19

II ANALYSIS AND INTERPRETATION OF FINANCIAL STATEMENTS 20 - 74

III ACCOUNTING RATIOS 75 - 232

IV FUNDS FLOW STATEMENT 233 - 363

V CASH FLOW STATEMENT 364 - 414

VI BUDGETARY CONTROL 415 - 553

VII STANDARD COSTING 554 - 655

VIII MARGINAL COSTING 656 - 751

IX MANAGERIAL REPORTING 752 - 768

BIBLIOGRAPHY 769 - 772

INDEX 773 - 775


"This page is Intentionally Left Blank"
Chapter - I

FUNDAMENTALS OF
MANAGEMENT ACCOUNTING

Objectives: The objectives of this chapter are:


• To Introduce you to the subject 'Management Accounting' with an emphasIs on ItS
emergence,
• To explain the Need for, and Meaning and Definitions of, Management Accounting,
• To describe the Nature, Scope, ObJectives, Utility and Functions of Management
Accounting, and
• To bring out clearly the differences that exist between Management Accounting on the
one hand, and Financial and Cost Accounting on the other.
Structure
Emergence of Management Accounting
• Meaning and Definitions of Management Accounting
• Need for Management Accounting
• Nature or Characteristics of Management Accounting
• Scope of Management Accounting
• Objectives of Management Accounting
• Functions of Management Accounting
• Advantages or Importance or Utility of Management Accounting
• Differences between Management Accounting and Financial Accounting
• Differences between Management Accounting and Cost Accounting
Limitations of Management Accounting
• Summary of the Chapter.
• Key Terms to Remember
• Notes and References

• Questions for Self-study


Management Accounting: 2

Emergence of Management Accounting


The persons at the helm. of affairs of all entItles including Trading Concerns,
Manufacturing Companies, Service Providing Organizations, etc., have to take a number of
decisions. For the purpose of taking decisions. they (managerial personnel. owners. directors of
the boards. executives. etc .. ) need a lot of relevant information. Because. these pieces of
information in the forms of facts, figures. etc., are essential for objective evaluation, as far as
possible, of each of the alternatives available so that right decisions can be taken. Hence, the
decision-makers (managerial personneL etc .. ) need an information system which is capable of
furnishing the precise and relevant information at the right time. Management Accounting serves
the managerial personnel by providing the required information and reports.
Of course, one can also find other two important kinds of Accounting viz., Financial
Accounting and Cost Accounting which are also functioning as information systems. However,
they are suffering from a few limitations and also the fact that their beneficiaries are different
and/or they are providing information to the same beneficiaries (viz., managerial personnel) but
for different purpose. The limitations, like this, of Financial Accounting and Cost Accounting
and their inability to serve the managerial personnel have led to the emergence of another branch
of Accounting viz., Management Accounting.
Though a few concepts and aspects of Management Accounting were known to the
managerial personnel in the first quarter of twentieth century, it developed substantially only
during last five decades. Even the term 'Management Accounting' was coined in 1950's as a
separate course of study by the British Team of Accountants which visited the United States of
America in 1950. The visit was sponsored by the Anglo-American Productivity Council for the
purpose of highlighting the utility of Accounting as an effective management technique. Since
then. it has d~veloped substantially.
Management Accounting - Meaning and Definitions
Management Accounting has been defined by many a number of experts and institutions
in different ways. But all the definitions highlight the fact that it (i.e .. Management Accounting)
aims at furnishing the relevant information to the managerIal personnel. And the managerial
personnel use these facts and figures for the purpose of taking a number of decisions. These
aspects become obvious from the f<?llowing definitions.
A team of British Industrial Accountants and Managers has defined Management
Accounting as the presentation of accounting information in such a way as to assist
management in the creation of policy and day-to-day operation of an undertaking. Batty
has defined Management Accounting as ... the term used to describe the accounting methods,
systems, and techniques which, coupled with special knowledge and ability, assist
management in its task of maximizing profits or minimizing losses.' In the opinion of Rose,
Management Accounting is the collection, analysis, diagnosis and interpretation of
accounting information in such a way as to help the management. Anthony has also opined
in the same way. In his view, Management Accounting is concerned with accounting
information that is useful to management. In the words of American Accounting Association,
Management Accounting includes the methods and concepts necessary for effective
planning, for choosing among alternative business actions and for control through the
evaluation and interpretation of performances. Broad and Carmichael have viewed
Fundamentals of Management Accounting: 3

Management Accounting as covering •.. all those services by which the accounting
department can assist the top management and other departments in the formulation of
policy, control of execution and appreciation of effectiveness."
An analysis of the above definitions bring out, amongst others, one important aspect, viz.,
functions of Management Accounting. Management Accounting deals with the internal
reporting. On the basis of the nature of these reports and their contents. and the parties who
receive these reports, it may be said that the Management Accounting deals primarily with the
furnishing of required and relevant data to the managerial personnel for the purpose of planning,
controlling and decision making. The type of accounting information required by the
management differs from one type of decision to another. and also from one level of management
to another. It is not necessarily confined to the Financial Accounting information but it is much
more than this depending upon the type, importance, complexity, etc., of the problem. Another
aspect which is to be clarified at this stage is about the terminology. The terminologies
Managerial Accounting, Management Accounting and Management Accountancy are used
synonymously to denote the same as there is no difference in the subject matter.
Need for Management Accounting
When the Financial Accounting is capable of furnishi11g the information eyen to the
internal parties or when there is no bar on the management to utilize the financial reports meant
for external parties, and when Cost Accounting is serving the internal parties by providing cost
details, a question normally crops up as to "what is and where is the need for another Accounting,
viz., Management Accounting"? Of course, these reasons are true. But what is to be noted here is
that both the Financial Accounting and the Cost Accounting lay emphasis on different objectives.
Management cannot base its decisions only on the information furnished by the Financial and
Cost Accounting. Therefore, there is a need for a system which utilizes and analyses the
abundant data (including the data generated by Financial and Cost Accounting) with the sole
objective of furnishing the relevant data to the management for the purpose of assisting it to take
a number of appropriate decisions. Management Accounting furnishes only those data which are
relevant to the decision under consideration and these relevant data include the data collected
from both financial and cost records and other sources. They also include both the quantitative
and the qualitative data. An illustration is enumerated below to highlight the need for
Management Accounting.
It is very well known that the price of a company's product is to be revised once or twice
in a year depending upon the policy of the company and the influencing factors. On the basis of
the financial reports, it is not possible to determine the extent to which the price is to be
increased. Because, it is very difficult to get an idea. from the financial reports, about the extent
to which the costs have increased. Therefore. Financial Accounting is not useful to the
management to decide about the extent of price revision. Cost books of accounts will be of much
help than the financial reports as the cost records provide some insight into the impact of hike in
the prices of input factors and their effect on the cost of sales, margin, etc. In spite of this
usefulness, even Cost Accounting is not of much use to the management in this regard. Because,
the management cannot increase the price equivalent to, or more than, the increase in the cost of
sales. It needs information about a number of other aspects such as, whether the competitors are
also going to increase .their prices, reactions of the customers to the proposed price increase, etc.
There is, therefore, a need for Management Accounting which 'collects info.rmation from different
Management Accounting: 4

sources, analyses them systematically and furnishes only the relevant information in the manner
which is most suitable, to the management and which uses the sophisticated tools and techniques,
including the tools and techniques derived from other disciplines, to present the reports to the
management in the most suitable and useful manner.
Since the managerial personnel are accountable to the owners of the company and since
their very continuation in the company depends upon the results produced by them which in turn
depends upon the quality of decisions and their implementation, the managerial personnel need a
system which furnishes the relevant information to them to take decisions. Therefore, the need
for Management Accounting which has been devised to serve the management through the
reports. On the basis of the analysis made hitherto, the specific reasons which unequivocally
bring out the need for Management Accounting are presented below.
Management Accounting - Why?
Increasing complexity of managerial decisions

Increase in size and problems of corporate entities


Different sets of information to different levels of
Need for management
Management Accounting
Management n~eds relevant and timely information to
take a number of decisions
Financial and Cost Accounting lay emphasis on other
objectives and fail to provide necessary
information to management
Nature or Characteristics of Management Accounting
The following are the important characteristics of Management Accounting and these
characteristics also explain the natllre of Management Accounting.
1. Cause and Effect Analysis: One of the important features of Management Accounting
is that it attempts to study the Causative Relationship (Cause and Effect) between
different variables. For instance, analysis of the reasons for the increased or reduced
profit of the company reveals the. factors responsible for the lower profit or factors
contributing to the higher profit. In the same way, reasons for the changes in Return on
Investment are analysed and unearthed by the Management Accountant.
2. Both Science and Art: Management Accounting possesses the features of both Science
and Art. Management Accounting aims at achieving accuracy, perfection and objectivity
at each of the works related to collection. analysis and interpretation of data, and in the
communication of the results of the same to the management. It is also concerned with
the quantification of problems, performance, etc. Hence, Management Accounting can
be considered as a Science. However, for the successful completion of his work, the
Management Accountant is expected to use some sort of subjective or personal
judgement which also influence the inferences, conclusion, etc., to a greater extent.
Management Accounting is, therefore, an Art in nature. Hence, Management Accounting
is both a Science and an Art.
Fundamentals of Management Accounting: 5

3. Information System: As Management Accounting is solely designed to provide


information to the management for the purpose of helping it to take various decisions, it
(i.e., Management Accounting) is basically an information system. It provides both the
monetary (quantitative) and non-monetary (qualitative) information, both the historical
and future-related or oriented information, both the actuals and the forecasts or
projections, both the' accounting and the non-accounting information, etc. For this
purpose, it makes use of the techniques such as Budgetary Control, Standard Costing,
Statistical Tools, etc.
4. Relevancy of Information: The most important feature of Management Accounting is
its emphasis on the relevancy of information to the decisions under the consideration of
management. If both the relevant and irrelevant data are supplied to the management, the
decision of the management may be intluenced more by the irrelevant lot than the
relevant few. Hence, the management may not be able to take the right decision. In this
background, this feature of Management Accounting assumes importance as it segregates
the data into relevant and irrelevant, and as it uses only the relevant data for evaluation
and communication to management.
5. Facilitator of Decision-Making: Management Accountant only provides the relevant
information and evaluates the alternative solutions to each of the problems, but he does
not take the decisions as he is not empowered to do so. Hence, he only facilitates the
decision making process by helping the decision-makers.
6. Flexibility but no Rigidity: In the case of Financial Accounting, certain Rules,
Principles, etc., are to be observed or followed strictly. But the Management Accountant
is not bound by such rigidity. He is free to use appropriate procedure, format, analysis,
etc., for reporting to the management. Usefulness of the report is more important than
the formalities of Reporting.
Functions of Management Accounting
The basic function of Management Accounting is -to serve the managerial personnel by
furnishing them the complete and relevant information whenever they are called for. By doing
so, Management Accounting aims at assisting management to take appropriate decisions and
discharging their responsibility satisfactorily. Hence, Management Accounting is called
Management-oriented Accounting. In order to accomplish this objective, Management
Accounting has to perform a number of functions. However, the following ,!re the important
functions of Management Accounting.
1. Provides Relevant Data: Management Accounting primarily aims at serving the people
who are internal to the business entity. That means, it furnishes the most pertinent facts
and figures to the managerial personnel and assists them to take various decisions. For
this purpose, it collects the data from different sources and presents the same to the
management whenever required and in the suitable form. Further., it provides information
and/or submits reports to different levels of management. Because, management at
different levels (top level, middle level and lower level) needs different information for
taking decisions at their le·vel.
2. Provides Qualitative Information: In order to assist management in its decision
making task, only the quantitative data are not sufficient. Management should be
Management Accounting: 6

provided even with the qualitative information if they are relevant to the decisIon under
consideration. For instance, in order to decide whether a plant is to be shut down or not
temporarily (due to trade recession), it i., not sufficient if the management report include.,
only the financial data. It should also include information about the impact on reputation.
loss of market to competitors. loss of experienc>ed employees, etc. Management
Accountant submits comprehensive reports which lI1clude both the quantitative and the
qualitative information.
3. Modification of Data: Both the FInancial and Cost Accounting generate voluminous
data about the performance and the financial position of business entities. In majority of
the cases, these data cannot be used as extracted from financial and/or co~t books of
accounts for managerial decisions. Because, the data is to be modified in such a way that
it becomes more useful for the management. The sales data can be classified according
to product, territory, customer class, cash sales, credit sales, etc. These details are easily
understandable and they are more useful to the management for taking decisions. This
way, Management Accounting classifies and modifies financial and other data according
to the requirements of management.
4. Analysis and Interpretation: Since the financial data lack communication, they are to
be analysed and interpreted properly to get an insight into the profitability, solvency,
liquidity, etc., of the company. Management Accountant. with the help of the tools of.
financial analysis, undertakes this task and presents the results with necessary comments,
conclusions, etc., to the management. Further, the managerial personnel may lack
technical knowledge about the financial information. Hence, the Management
Accountants analyse and interpret the financial data in simple way and report the same
using non-technical language.' Besides, the Management Accountants also undertake the
task of evaluating the alternatives and present the same to the management together with
their opinion.
5. Assists in Planning: One of the important functions of Management Accounting is to
prepare and submit the necessary reports to the management for the purpose of assisting
it in the process of planning for, anq forecasting, the future. Because, the management
has to formulate various policies, both short-term and long-term, for the future. To
formulate or design policies, the management needs various information, and these facts
and figures are furnished by the Management Accountant. Because, Management
Accounting uses different techniques such as Budgetary Control, Standard Costing,
Marginal Costing, Funds Flow Statement, etc., and helps the management in its planning
and forecasting activities.
6. Facilitates Overall Control: With the help of Standard Costing, Budgetary Control and
Responsibility Accounting, Management Accounting identifies the areas where the
control by the management is required. It is carried out through ~ process called
comparison - comparing the actuals with standards and budgets, and identifying the
variances. It also identifies the factors which are, and persons who are, responsible for the
poor performance. Based on this, the management takes necessary corrective measures.
Fundamentals of Management Accounting: 7

Scope of Management Accounting


As already stated, the primary objective of Management Accounting is to serve the
management in its functions of planning, directing and controlling. And to serve the
management, it (i.e., Management Accounting) makes use of tools and techniques of various
disciplines including Accounting. Any tool or technique which helps the Management
Accountant to discharge his duties and responsibility satisfactorily may be reckoned as falling
within the scope of Management Accounting. Consequently, it is a very difficult task to specify
the scope of a growing subject, like, Management Accounting. However, the following tools,
systems, techniques, etc., definitely fall within the scope of Management Accounting.
1. Financial Accounting: Management Accounting gathers maximum data from Financial
Accounting. Because, the annual reports (which include, amongst others, the financial
statements) provide voluminous data. Though Financial Accounting generates only the
historical data, they are useful for planning, directing and controlling purposes. Because,
in all these cases, historical financial data are used as the base and they act as guidelines
for the future. With the help of financial analysis tools, the Management Accountant
analyses and interprets the financial data to get a greater insight into the company's
performance, position, etc. Therefore, Financial Accounting is considered as falling
within the scope of Management Accounting.
2. Cost Accounting: Cost Accounting has been considered as an integral part of
Management Accounting due to two important reasons. One, as in the case of Financial
Accounting, Cost Accounting arso generates voluminous data about the primary activity
of the company, viz., costs of producing and selling the goods and services, and also the
revenue from the same. Two, it also provides a number of sophisticated analytical tools
like, Marginal Costing, Standard Costing, Budgetary Control, Inter-firm Comparison,
etc., which are widely being used by the Management Accountant to discharge:his duties
and responsibilities. Hence, Cost Accounting falls within the scope of Management
Accounting.
3. Tools and Techniques of Financial Analysis: Financial statements are analysed and
interpreted to get an idea about the company's performance and prospects. A number of
tools are used to accomplish this objective. The different tools are Comparative
Financial Statements, Common-size Financial Statements, Trend Ratios, Ratio Analysis,
and Statements of Changes in Financial Position. All these are, therefore, subject matter
of Management Accounting.
4. Statistical and Quantitative Techniques: Due to the complexity of the problems faced
by the management and the Management Accountants, a number of statistical tools and
mathematical programming like Sampling Techniques, Probability, Linear Programming,
Regression Analysis, etc., are used by the Management Accountants to present the
reports to the management more accurately and usefully. Therefore, the scope of
Management Accounting extends to include even th~se statistical and quantitative
techniques.
5. Management Reporting: Collection of facts and figures, analysis and classification of
the same, etc., are done with the sole objective of reporting to the management. The
reports are important as they identify both the achievements and failures, both the strong
Management Accounting : 8

and weak areas, both the positive and the negative aspects, etc., of the company.
Frequency, format, contents, etc., of the reports vary from one report to another.
Management reviews the performance of the company and that of each of the divisions
or departments on continuous basis and/or at regular intervals on the basis of these
reports and takes appropriate action and/or decision. The management reports which are
aho called internal reports are, therefore, subject ma~ter of Management Accounting.
Utility or Advantages or Importance of Management Accounting
As is known very well, planning, controlling, co-ordinating, organizing, motivating and
communicating are the six important managerial functions. Management Accounting helps the
managerial personnel to perform each one of these functions more effectively and profitably by
providing relevant information at the right time. For this purpose, the Management Accountant
collects the information from different sources, analyses them systematically to find out their
relevance to the decision under consideration and supplies only the relevant information to the
management to take proper decisions. The work of the management is, therefore, made easy by
the Management Accountant. Because, the Management Accountant will carry out a
comprehensive evaluation of all the possible and available alternatives, and suggest the best
alternative. This way, Management Accounting renders a very valuable service to the
management in all its fields of activity. It is because of this reason that Management Accounting
has rightly been interpreted as Accounting for Management, Management-oriented
Accounting, etc. In this background, the specific advantages or uses (utility) of Management
Accounting are presented below.
.
1. Analysis and Interpretation of Financial Data: Business activities and Accounting
Systems generate voluminous data and the managerial personnel may not be in a position
to understand them as they are highly technical in nature. Management Accountant
undertakes the responsibility of analysing and interpreting the financial data, and
presenting the same to the management in a simplified manner for the purpose of taking
necessary action.
2. Helps in Planning and Forecasting: Management is assisted, to a greater extent, by the
Management Accountant in its plans for different departments and operations. Because,
the Management Accountant provides the relevant information pertaining to different
products, markets, departments, etc. Consequently, it is possible for the management to
plan for each segment and also for the whole organization. Further, forecasting is made
easy by the use of Management Accounting tools along with statistical tools.
3. Helps in Decision Making Process: The management has to take a number of
decisions such as pricing, revision of prices, product diversification, dropping a product,
to sell or process further, replacement of existing capital assets, mechanization,
expansion, etc. Management Accountant identifies alternatives, collects all the necessary
details, evaluates the alternatives, identifies the optimal alternative, and prepares and
submits report to the management incorporating his analysis, and suggestion. The report
submitted by the Management Accountant helps the management in selecting a suitable
alternative and in taking appropriate decisions.
4. Measurement and Evaluation of Performance: With the help of Budgetary Control,
Standard Costing, Marginal Costing, etc., the Management Accountant is able to measure
Fundamentals of Management Accounting: 9

and evaluate the performance of departments, products, functions, etc., and also the
overall performance of the company. Standards andlor budgets set and actuals
accomplished are compared to find the deviations, ana to evaluate the performance.
Further, the Contribution, Plv Ratio, etc., are also used to assess the performance as
objectively as possible. Based on this exercise, a number of appropriate decisions are
taken by the management to improve the overall profitability.
5. Efficient Management Control: Since the Management Accountant makes use of the
Budgetary Control, Standard Costing, etc., it is possible to set the standards or targets and
compare them with the actuals accomplished. The comparison enables to identify the
areas wherein the company has failed to reach the targets. As the Management
Accountant, in his reports to different levels of management, draws the management's
attention to these areas, it helps the management to have management by exception and
to take immediate actions.
6. Customer Welfare: A part or whole of the benefits of cost control exercises in the form
of reduction in the cost of production and sales can be passed on by the company to its
customers in the form of reduction in prices. Further, the companies lay emphasis on the
quality of their products and the employees are made quality conscious. Hence, the
customers' interest is protected through the supply of quality goods at reasonable prices.
7. Maximization of Profitability: The emphasis of Management Accounting, and its tools
and techniques is on (a) improving efficiency of each and every individual and segment,
(b) cost control (c) maximization of revenue, etc. For this purpose, Budgetary Control,
Standard Costing, Marginal Costing, Responsibility Accounting, etc., are used, and every
segment or activity or product is expected to minimize its cost and contribute more
towards the overall contribution and profit of the company. Consequently, the overall
profitability will be improved.
Objectives of Management Accounting
The important objectives of Management Accounting are presented and analysed below
(for few more, see "Functions of Management Accounting").
1. To Analyse and Interpret the Financial Data: Financial Accounting generates
voluminous data and most of them are incapable of conveying any message to the
receivers including management. However, these pieces of information can be made to
convey certain message about various activitIes of the organization through a process
called 'analysis and interpretation' which is one of the objectives of Management
Accounting. Management Accountants classify the data on some bases, analyse them
and interpret the same in the right perspective. This helps the management to taKe
appropriate decisions.
2. To Report to the Management: The primary objective of Management Accounting is
to report to the different levels of management about the past performance, latest and/or
current position, and about the performance of different departments, products, etc.,
either regularly or at regular intervals depending upon the requirements. This helps the
management to take appropriate decision and/or to initiate suitable actions wherever
necessary.
Management Accounting: 10

3. To Help in Planning and Policy Formulation: Formulation of plans and policies by the
management is made easy if proper forecasts about production. sales, etc., are available.
Management Accounting helps management by presenting the statements of past
performance and making forecasts about the future.
4. To Facilitate Controlling: Management Accounting anTIS at nelping the management to
exercise its control over different activities. product~, departments. etc. ThiS is done
through its reports which contain the comparison hetween the 'budgets, standards,
targets, etc.: on the one hand and the 'actuals' on the other. The differences which are
called 'variances' are analysed further to identify the reasons for the same and this helps
the management to initiate controlling actions.
5. To Help the Management in its Decision Making Process: Decisions play.:! crucial
role in the success or otherwise of the organizations. The management which is
responsible and/or empowered to take the decisions must be very careful. And thl?
management is not in a position to take the right deciSion without the help of
Management Accountants. Because, the Man3gement Accountants identify the
alternatives, collect all the relevant information, evaluate each of the alternatives from the
view points of cost-benefits, pros and cons, etc.. ancl identify the most desirable
alternative. Incorporating all these, the reports will be prepared and submitted by the
M :magement Accountants to the management. Based on the5.·~ reports, the management
will take the appropriate decisions.
6. To Help in Organizing: The word 'organizing' refers to the establishment of
relationships among organizational individuals explaining unequivocally the authority-
responsibility rcJationship. Management Accounting intends to help the management
even in this matter with the help of 'Responsibility Accounting' wherein the entire
organization is divided into a number of ResponsibilIty Centres (which may take the form
of either the Cost Centres or Profit Centres or Inve<;tment Centres or a combination), and
making the heads of the centres accountable for the mutually agreed results. Of course,
adequate power will be delegated to them by the higher authority. This way, the
Management Accountant helps the management.
Management Accounting and Financial Accounting - Differences
Basically, Accounting is an Information System. Depending upon the party which the
Accounting aims to serve, it may be divided into Financial Accounting and Management
Accounting. Kenneth S. Most has rightly pointed out that Accounting is a service activity. Its
function is to provide quantitative information ... that is intended to be useful in making
economic decisions. in making reasoned choices among alternative courses of action.
Accounting, includes several branches, for example, Financial Accounting, Managerial
Accounting ... :< Therefore, a number of differences exist between Management Accounting and
Financial Accounting. They are summarized below.
Differences Between Management Accounting and Financial Accounting
Financial Accounting Management Accounting
1. Party to be Served: It aims at furnishing It aims at furnishing the information for use by
information to external parties (But, this the internal parties (viz., managerial
Fundamentals of Management Accounting: 11

information may also be utilized by the personnel).


internal parties).
'"\
I"'"
Utility: Financial Accounting and it-. Management Accounting and its reports are
reports are useful to the external parties useful to the lIlternal partIes viz .. management
<;uch as shareholders, debenture holder-;. for formulating plans. policies, and to take
creditors, financial institutions, govern- various decisions.
ments and their agencies, customers, etc.
Format of Reports: Financial Accounting No such rigidity is found in the case of
reports which include, amongst others, managerial reporting. Instead, there is a
Profit and Loss Account, Directors' flexibility in the format depending upon the
Report. Balance Sheet, and Auditors' requirements, suitability and convenience.
Report are to be prepared as per the format
specified by the relevant Provisions of the
Companies Act, 1956.
4. Time Period and Nature of Data: It There is no rigidity as to the time period. It
usually covers one year period just ended. depends upon the nature of the report. In a
The effect of transactions (on performance case, one hour may be covered and in another
and financial position) which have taken case, it may cover a period of more than one
place during an accounting year just ended year. These lay emphasis on the future and its
are covered. That means, these reports lay prediction on the basis of the past experience.
emphas: , on the historical data.
5. Unit of Study: Annual reports cover the Managerial reports concentrate on the detailed
overall performance of the company. study and assessment of performance of a
segment - di vision or product or activity, etc.

6. Principles: Generally Accepted Accounting GAAPs do not govern the preparation and
Principles (GAAPs) and practices govern I submission of managerial reports.
the financial or annual reports. Further,
Double Entry Principle is followed.
7. Contents of the Reports: Financial Managerial reports are prepared on the basis of
Accounting reports (also known as, Annual the requirements of the managerial personnel
Report.,) include the facts and figures as and/or managerial problem. The information
specified by the Provisions of the Companies which have a bearing on the decision under the
Act, 1956. They also include the information consideration of management are furnished and
which are commonly required by the external reported.
parties. Further, they include the information
which the management wishes to report.
8. Objectivity: Reports should be, as far as No such rigid objectivity is required in the case
possible, objective ones. That means, they of managerial reports. They may contain both
should contain the entries which are solidly the objective and subjective figures. Estimates,
supported by evidences such as documents. based on the past, also form parts of
deeds, invoices, vouchers, etc. They lay managerial reporting. They lay emphasis on
greater emphasis on the objectivity of data. relevancy and flexibility of data.
Management Accounting: 12

9. When to be Reported: It repots to the It reports very frequently to the managerial


external parties after the completion of the personnel depending upon the requirements -
accounting year (i.e., normally, once in a yearly, quarterly, monthly, bi-monthly, weekly,
year that too, at the end of the year). daily, hourly, erc.
10. Presentation of Reports: It is mandatory Managerial reports are not mandatory under the
under the Provisions of the Companies Act, Companies Act, 1956 bur they have been
1956 [Sections 201(1), 216, 217(1), prepared and submitted to the managerial
209(1 )). personnel because of necessity. In the absence
of these reports, management will not be in a
position to take correct decisions.
11. Auditing of Reports: Financial or annual Management reports are not subject to statutory
reports comprising of Final Accounts and audit. ./
others are subject to the statutory audit.
12. Publication: The annual reports which Management reports are circulated among the
include, amongst others, Profit and Loss managerial personnel and they are normally not
Account, Balance Sheet, Directors' Report, published.
and Audit Report, are to be prepared and
published for circulation among the
external parties.
13. Objective: The primary objective of Management Accounting aims at assisting the
Financial Accounting is to record the day- managerial personnel in their decision making
to-day transactions of the business with a process by redesigning accounting information
view to ascertain the profit earned, or loss suitably. Therefore, contents, format, timing,
incurred, by' the company during an frequency, etc., vary from one report to another
accounting period and also to prepare and also from one level of management to
the assets-liabilities statement. That means, another depending upon the nature of the
it aims at the preparation and presentatIOn problem.
of Profit and Loss Account, and
Balance Sheet to the parities like
shareholders, financial institutions,
bankers, management, etc., periodically
(i.e., once in a year).

From the above, it is obvious that both the Financial Accounting and the Managerial
Accounting intend to serve different parties and therefore, they differ to some extent from each
other. The most important differences between the two is clearly brought out by the opinion of
Sidney Davidson et aI., which is reproduced here, Financial Accounting typically refers to the
preparation of general purpose reports for use by the persons outside or external to a firm.4
Further, they have defined Managerial Accounting as typically referring to the preparation of
specific purpose reports for use by persons within or internal to a firm. 5

.~.
Fundamentals of Management Accounting: 13

In spite of the above differences, both function as complementary to each other.


Because, Managerial Accounting cannot exist without a suitable system of Financial
Accounting. Therefore, a sound Financial Accounting system is basically a must for
recording and collecting valuable information for the use of management.6 Further, it has
been said that Management Accounting has emanated from Financial Accounting as it (i.e.,
Financial Accounting) was designed earlier to furmsh the required information even to the
management. Financial Accounting provides the basic information to the Management
Accountants who analyse them in a different manner to suit to the requirements of managerial
personnel. Of course, Management Accountants also gather the additional information from
other sources for the purpose of reporting to the management.
Management Accounting and Cost Accounting - Differences
Though a few number of differences exist between Cost Accounting and Management
Accounting, the line of difference is very thin. Because, Cost Accounting, at present, comprises
of some of the advanced techniques and systems of Costing such as Marginal Costing, Standard
Costing, Budgetary Control, Responsibility Accounting, etc., and therefore, it (i.e., Cost
Accounting) tends to conform to Management Accounting. Consequently, no much difference
can be found between the two. Further, both serve the internal parties (viz., management).
However, a very few minor differences that exist between the two are presented below.
Differences between Management Accounting and Cost Accounting
Cost Accounting Management Accounting
1. Primary Objective: Cost Accounting Management Accounting aIms at the
aims at ascertaining the cost of goods and presentation of cost data, to the extent required,
services. It lays emphasis on the stage by wherever and whenever they are required
stage computation of costs. Further, it together with other relevant information to the
deals with cost control, matching of cost management for taking decisions and for
and revenue, and assessment of planning and co-ordinating the activities of the
performance of the operating activities. business enterprise.

2. Information Coverage: Cost reports deal Cost data form a part of managerial reports but
mainly with the costs - incurred or not the sole aspects. Because, managerial reports
budgeted and standards, variances, savings, cover various aspects such as costs, budgets, tax
etc. planning, projection, etc. Hence, the scope of
Management Accounting is broader.

3. Nature of Data: Normally, Cost Management Accounting uses and supplies not
Accounting lays emphasis on the past and only the quantitative but also the qualitative
therefore, the quantitative data are recorded information.
in cost books of accounts.

4. Governing Principles, Rules, etc: Cost No such rigidity IS there In the case of
Accounts and Reports are to be prepared as managerial reports. The procedure, format,
Management Accounting: 14

per certain rules. principles, procedures. etc., can be modified from time to time
etc.. as specified by the appropriate depending upon convenience and requirements.
authority (e.g., ICW AI) to the industry to
which tht c'ompany belongs to.

5. Time Factor: It lays more emphasis on It predicts the future on the basis of the past
the past and present. and less emphasis on events, present happenings and future
the future. That means. it reports about estimates. It also facilitates the formulation of
costs that have been incurred. plans and policies.

6. Utility of Reports: Though cost reports Management reports are useful only to the
are meant for management, they are useful management but not to both internal and
even to the external parties. external parties.

7. Inter-dependence: Cost Accountant Management AccountlOg. for its reports,


provides voluminous data to the extracts the maximum information from Cost
Management Accountant and therefore, Accounting reports. And therefore, Cost
acts as a source of information. Further, Accounting is a necessity for the smooth
Costing system can be installed in an functioning of Management Accounting.
organization without Management Because, Management Accounting cannot exist
Accounti'1g system. That means, even in the absence of a proper and systematic Cost
without Management Accounting system, Accounting system.
the Costing system can function.

8. Statutory Verification: Cost accounts Management reports are not subject to any
and reports, in many cases, are subject to statutory audit. Of course, there is a
statutory audit (i.e., Cost Audit). Hence, management audit. But, it is voluntary and
cost reports should be prepared, as far as internal. and it evaluates the managerial
possible, on objective manner. functions. decisions, etc. However,
management reports include both the objective
and the subjective data.

In spite of the above minor differences between the two, both work as complementary.
Because, Management Accountant will not be in a position to discharge his responsibility in the
absence of a sound Cost Accounting system. Because, Cost Accounting provides some of the
useful data to the Management Accountant who in turn utilizes them to appraise the management.
In the same way, Cost Accounting would be of not much use to the managerial personnel in the
absence of a proper Management Accounting system. Hence, both are complementary.
Fundamentals of Management Accounting: 15

Limitations of Management Accounting


Management Accounting is comparatively a new branch of Accounting. It is still in the
developing stage. Though it has made much progress during this short span of time, it suffers
from a few limitations. The important limItations are noted below. However, most of these
limitations from which Management Accounting is suffering can he overcome by convincing the
management about the need for, and sigmficance and utility of, Management Accounting.
1. Personal Bias and Lack of Objectivity: Managem;~nt Accountant, in his reports to
management. has to include the prediction about the future based on the past, present and
future. Therefore, the reports include butn the uctual and estimates, and both the
quantitative and the qualitative informatiun. Further, the managerial reports lay more
emphasis on relevancy rather than on objectiviry. The reports are, therefore, Influenced
by the personal judgements. Analysis and interpretation of financial information is also
influenced by the ability of the Management Accountants and therefore, there is ever)
likelihood of personal bias in analysis and interpretation which significantly influences
the decisions. Hence, it is said that they are more subjective than objective.
2. Intuitive Decision Making: Though the Management Accounting with the help of
advanced and sophisticated decision tools like, Marginal Costing, Mathematical
Programming, Statistical Tools, etc., is capable of serving and assisting management
with relevant data to take decisions, there is a tendency in management to prefer a short-
cut method of taking decisions incuitively. Consequently, the entire effort of
Management Accountants becomes useless and therefore, this type of attitude of
managerial personnel limits the utility of Management Accounting.
3. High Cost for Installation and Operation: In order to install a system of Management
Accounting, it is necessary to make elaborate arrangements. This requires a
comparatively high initial cost and operating expenses which majority of small-scale and
medium-scale organizations can ill-afford.
4. Limitations of Financial and Cost Accounting: Management Accounting bases its
reports on the data gathered even from financial and cost books of accounb. Hence, any
flaw, inaccuracy, etc., in the preparation of financial and cost books of accounts impaires
the quality of reports prepared and submitted by the Management Accountants. This has
resulted in a situation wherein some of the limitations of Management Accounting are
due to the dependence of Management Accountants on Financial Accounting and Cost
Accounting for the information. Hence, the limitations of Financial Accounting and
Cost Accounting can also be considered as the limitations of Management Accounting.
5. Wide Scope: It is very difficult to prescribe the boundary for Management Accounting.
Because, it requires the services of Cost Accounting, Financial Accounting, Quantitative
Techniques, Mathematical Programming, Production Engineering, Marketing Aspects,
etc. Therefore, it is very difficult to develop a Management Accounting Department
with the people who have full knowledge about all these di~l·irlines. This acts as one of
the limitations of Management Accounting.
6. Resistance: The successful installation and functioning of Management Accounting
system depends, to a greater extent, upon the co-operation of management. Because, it
involves the basic changes in the organizational set-up and the reporting system, and also
Management Accounting: 16

the attitude of management towards viewing a problem and solving it. The managerial
personnel may consider this as an unnecessary interference by the Management
Accountants in their activitIes and therefore, they may not extend their full co-operation
and may also resist the changes. Even the staff of Financial and Cost Accounting
Departments may resist the installation of Management Accounting System.
7. Not a Substitute for Management: Management Accounting which provides only
information but does not take decisions is not a substitute for management. Because, it
only helps the management, facilitates the work of management and provides invaluable
help to the management in its functions such as planning, controlling, communication,
etc., by furnishing the timely and relevant information. Hence, Management Accounting
performs only the service function. But it (i.e., Management Accounting) cannot act as
substitute for management and therefore, it cannot replace management.
Summary of the Chapter
In the present-day business environment which is characterised by continuous changes,
stiff competition, entry of new players into the market, etc., management has a very strenuous
task of taking a number of decisions which play crucial role in the success or otherwise of the
company. Since the management has to consider all the influencing factors and also all the
alternatives available before taking a right decision, it is in need of a reliable information system
which is capable of furnishing timely information and/or reports (to different levels of
management). Management Accounting fills this gap and this is how Management Accounting
emerged. After the introduction to Management Accounting, Need for, and Meaning and
Definitions of, Management Accollnt1l1g are presented followed by Characteristics, Scope,
Objectives, Functions and Advantages l)f Management Accounting. Differences between
Management Accounting, and Financial Accounting and Cost Accounting are presented in the
final stage followed by limitations of Management Accounting.
Key Terms to Remember
Management Accounting Management -oriented Accounting
Financial Accounting Cost Accounting
Notes and References
1. J. Batty, Manageme1lt Accountancy, London, The English Language Book Society and
Macdonald & Eyans, 1971, p.l.
2. Broad and CarmichaeL A Guide to Management Accounting, London, H.F.L Ltd., 1957,
p.2.
3. Kenneth S. Most, Accounting Theory, Ohio, Grid Inc, 1977, p.2.
4. Sidney Davidson, James S. Sch1l1der, Clyde P. Stickney and Roman L. WeiL Managerial
Accou1lting - All Introduction to Concepts, Methods and Uses, Illinois, The Dryden
Press, 1978, p.l.
5. Ibid, p.l.
6. Source not known.
Fundamentals of Management Accounting: 17

Questions for Self-study


01. Define Managerial Accounting and explain its managerial uses.
02. It has been said that 'Management Accounting has been evolved to meet the needs of
managerial personnel'. Do you agree? Why?
03. "Management Accounting has been evolved to meet the needs of management". Discuss
[Bangalore Uni, B.Com, May 2002 and Kuvempu Uni, B.Com, May 1999J
04. What is the role of Management Accountant in the management process?
[Bangalore Uni, B.Com, November 2003J
05. Explain the scope.of Management Accounting
IBallgalore Uni, B.Com, November 2001 and 2002)
06. Define Management Accounting. Discuss the functions and scope of Management
Accounting [Kuvempu Ulli, B.Com, May 1997J
07. Define Financial Accounting and Management Accounting. Discuss the scope and
functions of Management Accounting [Kuvempu Uni, B.Com, May 2000J
08. Define Management Accounting. Explain its functions.
[Kuvempu Uni, B,Com, May 2002 and November 2004J
09. Briefly explain the limitations of Financial Accounting and how Management
Accounting helps in removing them.
10. Discuss the role of Management Accountant in the decision-making process.
11. What do you understand by Management Accounting? Clearly explain the functions of
Management Accounting.
12. What is 'Accounting for Management'? What is the need for such a system of
Accounting?
13. Write a note on utility of Management Accounting.
14. Define Management Accounting and explain clearly its advantages.
15. Why is it said that Management Accounting is <l Management-oriented Accounting
System')
16. Comment on the statement that 'there are no externally imposed Generally Accepted
Accounting Principles for Management Accounting'.
17. Discuss the role of Management Accounting as an information system.
18. Define Management Accounting and explain the scope of Management Accounting.
19. Define Management Accounting and explain the factors which make the Management
Accounting a necessity.
20. 'Management Accounting provides immense help to the managerial personnel to take
various decisions'. Elucidate.
Management Accounting: 18

21. Explain briefly the functions of Management Accounting. Also explain briefly the tools
and techniques used in Management Accounting. [ICWA (lilt), December 1988J
22. What is Managerial Accounting? Discuss its usefulness for decision making and control.
[ICWA (lilt), December 1992J
23. Discuss the significance of Management Accounting for companies affected by
recession. [Mallgaiore Uni, MBA, November 1999J
24. 'The emphasis of Management Accounting differs from that of Financial Accounting'.
Discuss.
25. Define Management Accounting and explain how it differs from Financial Accounting.
26. Explain the meaning of Management Accounting and distinguish it from Cost
Accounting.
27. What are the objectives of Management Accounting? How do they differ from that of
Financial Accounting?
28. Distinguish between Management Accounting and Cost Accounting.
[ICWA (lilt), june 1989 alld December 1989J
29. Distinguish between Cost Accounting, Financial Accountll1g and Management
Accounting. [ICWA (lilt), jUlle 1992J
30. Differentiate between Financial Accounting and Management Accounting
[ICWA (lilt), JUlie 1993J
31. Write a note on 'Management Accounting as an extension of Cost Accounting'.
[ICWA (lilt), jUlle 1997J
32. 'Management Accounting is Financial Accounting belt at its elastic point'. How far do
you agree with this statement? [Ballgalore Uni, B.Com, November 2000J
33. Distinguish between Financial Accounting and Management Accounting.
[Ballgaiore Ulli, B.Com, May 2001, November 2002, and May 2003, alld Kuvempu
Uni, B.Com, November 2002J
34. Define the term "Management Accounting". What are the differences between
Management Accounting and Financial Accounting?
[Kuvempll Uni, B.C011l, May 1991J
35. What is Management Accounting? How does Management Accounting differ from
Financial Accounting? [Kllvempu Ulli, B.Com, October 1997J
36. Explain the meaning and functions of Management Accounting. Bring out the difference
between Management Accounting and Cost Accounting
[Kuvempu Ulli, B.Com, October 1998J
37. Describe fully the limitations of Financial Accounting and point out how Management
Accounting helps in overcoming them. [Kuvempu Uni, B.Com, October 1999J
Fundamentals of Management Accoul'tlng : 19

38. Define the term Management Accounting. How does it differ from Financial Accounting
and Cost Accounting? [Kllvempll Un;, B.Com, November 2000 and May 1992J
39. Discuss the scope and importance of Management Accounting. How does it differ from
Financial Accounting? [Kllvempll Uni, B.Com, November 2001 and 2003J
40. Critically evaluate the limitations of Management Accounting.
41. "Management Accounting a~"i~ts 111 the corpmale planning process··. Explam. Also
briefly describe the limitaUolb ()f Management Acclluntll1g. [ICWA. (lilt), June 1989J
42. State the lllmtations of Management Accounting and how they can be eliminated.
[ICWA (lilt), December 1990J
43. Attempt the following:
a. "Management Accounting is concerned with accounting information which is useful
to Management". Explain.
b. How does Management Accounting differ from Financial Accounting and Cost
Accounting? [ICWA (blt), June 1994J
44. What do you mean by Management Accounting? What are its utilities? Explain in
detail. Also state its limitations [Kllvempll Ulli, B.Com, May 1998J
45. Explain the meaning and functions of Management Accounting. State its limitations
[Kllvempll Uni, B.Com, May 2003J
46. Define Management Accounting. Explain its functions and limitations.
{Klivelllpu Uni, B.COIll, May 2004J
47. A few Short-answer Questions:
a. Define Management Accounting.
[Banga[ore Ulli, B.Com. November 2000 and 20()3, and May 2002J
b. State any two primary objectives of Management Accounting.
[Bangalore UIl;, B.COIll, May 2000 and 2001J
c. State any four functions of Management Accountant.
[Bangalore Uni, B.COIll, April2004J
d. State two main differences between Management Accounting and Cost Accounting.
[Banga/ore Uni, B.COIll, May 2001J
e. State any two limitations of Management Accounting.
[Bangalore Un;, B.Com, November 2001 and 2002J
Chapter - II

ANALYSIS AND INTERPRETATION


OF
FINANCIAL STATEMENTS

Objectives: The Important objectives of this chapter are:


To understand the Concepts, Nature, Objectives and limitations of Financial Statements,
To know the Meaning, Objectives and Essentials of Analysis and Interpretation,
• To study the different types of Analysis and Interpretation, and
• To discuss three methods of Analysis and Interpretation.
Structure
• Meaning and Concepts of Financial Statements
Nature of Financial Statements
Objectives of Financial Statements
• Limitations of Financial Statements
Meaning of Analysis and Interpretation
• Types of Analysis and Interpretation according to:
• Modus Operandi:
• Vertical AnalysIs
• HOrizontal Analysis
• Materials Used:
• External AnalysIs
• Internal Analysis
Methods or Tools of AnalysIs and Interpretation
• Comparative Financial Statements
• Common-size Statements
Trend Analysis
Illustrations
• Summary of the Chapter
Key Terms to Remember
• Notes and References
• Questions for Self-study
Analysis and Interpretation of Financial Statements: 21

Financial Statements - Meaning and Concepts


The following stages are involved in the Accounting work.
1. Analysis of each and every transaction to ascertain the amounts involved, and also the
accounts to be debited and credited;
2. Journalisation, posting to Ledger Accounts and the preparation of Trial Balance;
3. Preparation of Financial Statements (viz., Balance Sheet, and Profit and Loss Account or
Income Statement); and
4. Analysis and Interpretation of Financial Statements.
The emphasis of this chapter and the next three chapters is on the last aspect, viz ..
Analysis and Interpretation of Financial Statements. Therefore, it is necessary to understand the
concepts of Financial Statements. Concepts are essential ideas which permit the identification
and classification of phenomena or other ideas. 1 In this background, Financial Statements are
interpreted as those statements which show both the performance and the financial position.
SmIth and Ashburne define 'Financial Statements' as the end product of Financial Accounting
in a set of Financial Statements prepared by the Accountant of a business enterprise that
purport to reveal the financial position of the enterprise, the result of its recent activities
and an analysis of what has been done with earnings. Financial Statements, according to John
N. Myer, provide a summary of the accounts of a business enterprise, the Balance Sheet
reflecting the assets, liabilities and capital as on a certain date and the Income Statement
showing the results of operations during a certain period. These FinanCIal Statements which
are also called Final Accounts, form an integral part of Financial Reports, also called
Published Reports or Annual Reports. These FlI1ancIaI Statements include balance sheet,
income statement, funds statement, or any supporting statement or other presentation of
financial data derived from accounting records. 2 The contents of Financial Reports are
presented in the figure (page 22).
Balance Sheet has -been defined by Kohler as a statement of financial position of any
economic unit disclosing as at a given moment of time its assets, at cost, depreciated cost, or
other indicated value; its liabilities; and its ownership equities:~ Balance Sheet, in the words
of Howard and Upton, is a statement which reports the property's value owned by the
enterprise, and the claims of the creditors and owners against these properties. It shows
current assets, current liabilities, fixed assets, investments, deferred charges, long-term liabilities,
proprietors' fund, etc. In brief, it is a statement which shows the monetary values of various
assets held by the company and also its liabilities towards outsiders including the owners at the
end of the last day of an accounting year.
- Income Statement or Profit and Loss Account is a statement which shows. in summary
form, the revenues and expenses of a business entity for an accounting year. Profit and Loss
Account, according to Hary G. Guthmann, is the condensed and classified record of the gains
and losses causing changes in the owners' interest in the business for a period of time. It
shows the sales revenue, cost of sales, gross profit, operating expenses, operating profit, non-
trading income and expenses, net income or loss, appropriations, tax liability, etc. In brief, the
Profit and Loss Account presents information about the result or outcome of business operations
during an accounting year by matching expenses incurred during an accounting year against the
revenue earned in that year.
Management Accounting: 22

Composition of Financial Reports

Financial r Profit and Loss Account


Statements 4 Balance Sheet

Directors, {{eport

Composition
of Financial
Reports
--,1---7
'---_R_e_p_o_r_ts_'
E Chairman's Speech (delivered at the Annual General
Meeting)
Auditors' Report

Schedule of Fixed Assets


Schedule of Long-term Investments
Schedule of Inventories
Schedules ~ Schedule of Reserves
Schedule of Long-term Liabilities
Other Schedules

Depreciation Methods
Explanatory
Notes
E Inventory Valuation Methods
De .ails of Contingent Liabilities

These two basic Financial Statements are supported by a number of schedules, annexures,
supplementary statements, footnotes, etc., supplementing the data contained in the Balance Sheet
and the Income Statement. Therefore, all these, in addition to Profit and Loss Account, and
Balance Sheet, fall within the scope of Financial Statements.
Nature of Financial Statements
It is very well known that the Financial Statements basically refer to Balance Sheet, and
Profit :md Loss Account or Income Statement. Of course, these two basic statements are
supported by a number of schedules, annexures, supplementary statements, explanatory notes,
footnotes, etc. Therefore, all these form Financial Statements. They show, with supporting
figures, the profit earned or loss incurred during an accounting period and also the assets,
liabilities and capital at t1:e end of the last day of the accounting period. These statements are
prepared on the basis of the transactions (both busii1ess, financial and investment) that have taken
place during the accoullting period. Further, transactions of the previous yearls are also
considered to the extent of their relevance to the accounting period for which the Financial
Statements are being prepared. While recording their effects in the Financial Statements, the
Accountants observe certain well established Accounting Principles and exercise their
discretionary powers ·to select the best alternative accounting solution, whenever the alternatives
exist and the situation demands. It is, therefore, said that ... Financial Statements reflect a
AnalysIs and Interpretation of Financial Statements: 23

combination of recorded facts, accounting conventions, and personal judgements, and the
judgements and conventions applied affect them materially.4 It is, therefore, obvious that the
Financial Statements (and their contents) are influenced by ~hese factors which are di"cussed very
briefly in the following paragraphs.
1. Recorded Facts: It may be remembered here that the preparation of '~ina 'cial Statements
is made on the basis of Trial Balance which in turn is prepared on thl~ ba<,; of balances in
various Ledger Accounts. As is known, Ledger Accounts are prepa r ~J by posting Journal
Entries. Recorded facts here denote the figures recorded in jl)urnal Books, Ledger
Accounts and Trial Balance. The Financial Statements are preparerf on til'~ basis of entries
in these books of accounts. Any figure which does not find place ,n the books (If accounts
is usually not taken to the Financial Statements. For instance, depreciable fixed assets ale
recorded in the Financial Statements (more specifically, in Balance Sheet) at their
acquisition costs (sometimes, at net of depreciation to-date) but not at their current costs.
Because, these current cost figures are not recorded in the Journal Books, Ledger
Accounts and Trial Balance.
2. Generally Accepted Account;ng Principles (GAAPs): GAAPs are in the form of
guidelines and/or rules which are to be used as standards fOI recording business
transactions in the books of accounts and their fair presentatIOn in the Financial
Statements. Because, the FinanCial Statements have to be prepared in conformity with the
GAAPs. These (i.e.. GAAPs) include Principles, Concepu>, Conventions and
Assumptions or Postulates. Consequently. the figures in the Financial Statements are
influenced by the GAAPs. For instance, the guideline on 'inventory valuation' states that
tha year-end inventories are to be valued at lower of cost or net realizable value.
That means, the value of year-end inventory which appears in the Financial Statements IS
influenced by this principle. Like this, GAAPs influence almost all the items of Financial
Statements in one way or the other.
3. Personal Judgements: For a few number of important accounting problems,
Accountants find a number of alternative solutions, each being considered by the
competent authorities as based on sound principles. For instance, a number of methods
are available for computing the annual depreciation. The amount of depreciation (and
therefore, the cost, profit, written-down value of asset, etc.,) varies from one method to
another method. Since one (out of a number of alternative methods) i~ to be selected, the
opinion of the individuals also influences the Financial Statements.
From the above, it is clear that the Financial Statements reflect a combination of recorded
facts and figures, GAAPs and personal judgements and these three influence the financial data
substantially. Therefore, whenever the Financial Statements are considered and/or analysed, they
are to be dealt in the light of these three governing factors.
Objectives of Financial Statements
The following are the important objectives of FinanCial Statements.
1. To provide adequate information about the financial performance and the assets-liabilities
position of the entity;
2. To provide useful information which can gainfully be utilized to predict, compare and
evaluate the entity's earning capability;
Management Accounting: 24

3. To provide sufficient information which can be utilized by both the internal and the
external parties to predict, compare and evaluate the financial soundness of the entity. They
should also enable the parties to predict, compare and evaluate the potential funds flow in
terms ot both amount. time and associated uncertainty:
4. To provide required information to enable the users of Financial Statements to evaluate the
ability or performance of managerial personnel to utilize the company's resources for the
purpo~e of accomplishing the primary corporate objective;

5. To provide information primarily to those who have limited authority or resources to obtain
the required information. That means, to provide information to those who depend only on
the Financial Statements for information.
From the above, it is unequivocal that the Financial Statements aim primarily in
satisfying the informational requirements of external parties who gather much of the required data
from the Financial Statements. Of course, these statements are also used by the internal parties.
Limitations of Financial Statements
The Financial Statements are suffering from a number of limi.ations which are identified
and analysed below.
1. Fulfilment of Statutory Requirements: The Financial Statements which are normally
prepared in an absolute manner do not communicate much about the profitability,
solvency, stability, liquidity, etc., of the undertakings to the users of the statements.
Because, the statements include the figures which do not speak on their own. Due to this
reason, the Financial Statements are not assisting either the external parties or the internal
parties to take proper decisions though the very purpose of preparing and presenting these
statements is to assist them in this task. The Financial Statements are, therefore, criticized
<\s serving neither the internal nor the external parties except complying with the statutory
requirements.
2. Historical Data: Financial Statements comprise of only the effects on the items of
Income Statement or Balance Sheet or both, in brief, of variegated types of transactions -
business, financial and investment - that have taken place during an accounting year.
They also include the effects of the transactions of the previous year/so However, the
Financial Statements furnish only the historical data and therefore, one can assess the
performance and the financial position of the organization during the year or at the end of
the year for which the statements have been prepared. Therefore, they are of less
relevance for future Jec.isions.
3. Interim Reports: As is known, the Financial Statements are prepared annually.
Therefore, they are not capable of furnishing either the comprehensive or the correct
information about the profit, loss, assets, liabilities, etc. Because, actual profit can be
computed only after the winding up of business. The Financial Statements are, therefore,
more estimates in nature than actual as the costs of fixed assets and such other items are
depreciated or amortized over a period of estimated life.
4. Emphasis on only Quantitative Information: Financial Statements record only the
transactions which can be expressed in terms of money. And they do not show the non-
monetary facts or attributes which cannot accurately be expressed in terms of monetary
Analysis and Interpretation of Financial Statements: 25

unit even if they are very important. Quality of product, labour relations, attitude of
employees, etc., cannot accurately be measured in terms of money. Hence, they are not
accounted for in the Financial Statements. But, the evaluation of performance and the
financial position is incomplete in the absence of these qualitative factors.
5. Generally Accepted Accounting Principles: Since the preparation of Financial
Statements is governed by the GAAPs and since there are a number of diverse accounting
solutions to each of a few important problems, Financial Statements do not depict the
reality.
. ,

6. Personal Judgements: Accounting provides a number of avenues to the personnel in


charge of preparation of Financial Statements to influence the figures. For example,
method of depreciation, amortization period, valuation of invenmry, accounting treatment
of research and development costs, fictitious assets, etc., are decided by the management
and/or the Accountants and therefore, figures of Financial Statements are influenced by
the personal judgements.
7. Opportunities for Manipulation (Window Dressing): In the case of preparation of
Financial Statements, one can find a number of opportunities for manipulation by the
managementiFinancial Accountant wherein the Financial Statements can be prepared in
such a way which will suit the requirements of the management. It is because of the
presence of diverse accounting solutions to each of a few major items of Financial
Statements.
8. Failure to Recognize the Changes in the Price Level is another important limitation of
Financial Statements. Because, the Financial Statements are prepared on the basis of the
historical costs without recognizing the changes in the price level, and without
quantifying and incorporating the effects of changes in price level on the items of
Financial Statements.
9. As the companies are free to use different accounting treatments and procedures even for
identical problems, the Financial Statements cannot be used for inter-firm
comparison which is the base for the external parties to decide about purchasing, or
continuing to own, the shares of a company. Because, the financial results of the
companies are also influenced by the diverse accounting treatments.
Like the above, the Financial Statements are suffering from a number of limitations.
'Analysis and Interpretation' is, of course, not aiming at overcoming all the above limitations of
Financial Statements. But the most significant contribution of Analysis and Interpretation is its
ability to make the Financial Statements to speak about the weaknesses, strengths, ability to earn,
solvency, etc., of the company.
Analysis and Interpretation of Financial Statements
The facts and figures in the Financial Statements can be transformed into meaningful and
useful figures through a process called Analysis and Interpretation. The Analysis and
Interpretation of Financial Statements aims at finding out the importance and meaning of the
data in the Financial Statements so that proper evaluation about the past activities may be made
and a forecast may gainfully be made about the prospects of earnings, dividends, ability to pay
interest, etc. In other words, it is a process of establishing the meaningful relationship between
Management Accounting: 26

the items of Financial Statements with the objective of Identifying the financial and operational
strengths and weaknesses. This process involvl:'I both the Analysis and the Interpretation.
Analysis refers to the proper arrangement of data wherein the total figures in the
Financial Statements are regrouped into their distinct and different component parts. This
regrouping of data is necessitated due to the fact that the Financial Statements comprise of a
number of heterogenous and diverse accounting information. From these heterogenous figures,
no definite conclusion can be drawn or no specific idea can be formed. Therefore, it is necessary
to analyse the data. An example may be given here to highlight the fact that the regrouping is
actually a necessity. The amount of current assets in the Balance Sheet i-; split into debtors, cash,
year-end inventories, bills receivable, etc. Further, the amount of debtors may be analysed into
the amount due for less-than three months, for three to six months, and for more-than six months.
As the amount of bad debts is normally influenced by the latter category of debtor<;. this type of
regrouping assists the management to concentrate on the amount due for more-than ~IX months.
Interpretation refers to the comparison of various components and the examination of
their content so that useful and definite conclusions are drawn about the earning capacity,
efficiency, profitability, liquidity, solvency, trend, etc. Comparison is, therefore, a pre-requisite
for meaningful interpretation. In the words of F. Wood, to interpret means to put the meaning
of a statement in simple terms for the benefit of a person.
Though both . Analysis' and . Interpretation' appear to be distinct aspects, it is a very
strenuous task to draw a definite line of difference between them. Because, Analysis becomes
useless if the results of the Analysis are not properly IOterpreted to form an idea or opinion about
thl: performance of the organization. Further, Interpretation is impossible unless a proper
Analysis is made. Both the Analysis and the Interpretation are closely connected, inter-related
and are complementary to each other. Analysis is always followed by Interpretation and this
Interpretation is performed through a process called comparison. Therefore, whenever the word
Analysis is used, it implies both Analysis and Interpretation. From the analysis, it is clear that the
entire work associated with the Analysis and Interpretation of Financial Statements involves three
important steps or processes viz., (a) Analysis, (b) Comparison, and (c) Interpretation.
Moore and laedicke have defined Financial Analysis as ... a process of synthesis and
summarisation of financial and operative data with a view to getting an insight into the
operative activities of a business enterprise. Robert H. Wessel has defined Analysis and
Interpretation of Financial Statements as ... a technique of x-raying the financial position as
well as the progress of a company. Kennedy and MacMillan have opined ... by establishing
strategic relationships between the components of balance sheet and profit and loss account,
and other operative data, it unveils the meaning and significance of the various items
embodied in the Financial Statements. Metealt and Titard have defined Financial Statements'
analysis as a process of evaluating the relationship between component parts of a Financial
Statement to obtain a better understanding of a firm's position and performance. s These
definitions clearly show that both Analysi~ <lIld Interpretation are inter-related. Further, this type
of Analysis and Interpretation is intended 10 serve both the lI1ternal and the external parties in
their respective fields of decision making.
Analysis and Interpretation of Financial Statements: 27

Procedure for Analysis and Interpretation


For the successful completion of Analysis and Interpretation, it is necessary for the
analysts to follow the following procedure.
1. Define the objective of Financial Analysis,
2. Determine the information required, identify the other sources of information other than
the Financial Statements. Because, to analyse and interpret the financial data, the
analysts may need some more information than can be obtained from the Financial
Statements:
3. Arrange and analyse the data systematically:
4. Evaluate the data with the help of the techniques of financial analysis such as Ratios,
Trends, Funds Flow Statements, etc., and through a process of comparison;
5. Interpret the results to draw conclusion, inference, project and predict, comment, etc., and
6. Prepare and submit the report based on the findings of Analysis and Interpretation.
Types of Analysis and Interpretation
There are a number of bases on which Analysis and Interpretation can be classified into
two or more groups. Number of firms, party who is analysing and interpreting, number of years'
figures used, etc., become the base for classification of Analysis and Interpretation. However,
the modus operandi and the material or information used are the two common bases used-for
classification.
Types of Analysis and Interpretation

Vertical
Analysis

Modus
Operandi
Horizontal
Classification of Analysis
Analysis and
~
Interpretation
according to ...
External
Analysis

Materials
Used

Internal
Analysis
Managemert Accounting : 28

I. Classification According to Modus Operandi: On the basis of the modus operandi or the
method of operation followed for the Analysis wherein the number of years' Financial
Statements used for Analys.is dnd Interpretation is considered as the base, the Analysis is
classified into two tyf>es as Vertical Ana1ysis and Horizontal Analysis.
1. Vertical or Structural Analysis: When the analysis of Financial Statements of an
organization for only one accouming period is made, it is called Vertical Analysis. For
instance, analysing and interpreting the performance of ABC Company for the year
2004 with the help of the Profit and Loss Account of that company for the year ended
December 31, 2004 and the Ba' anee Sheet of that company as on that date is an example
to Vertical Analysis. A numrer of ratios establishing meaningful relationship between
the items of Financial Statemt:Jlts \.\. hich shed light on variegated aspects of the company
can be computed. Gross Profit Ratio, Net Profit Ratio, Operating Cost Ratio, Current
Ratio, Quick Ratio, Return on Investment, Stock Turnover Ratio, etc., can be computed.
But with the help of the~c computations, it is very difficult to draw any definite
conclusion such as whether the company has improved its performance, efficiency,
profitability, etc. Because, the performance in the current year is not compared with that
of the previous year/so Because of these reasons, Vertical Analysis is dubbed as static
analysis. However, Ratio Analysis and Common-size Financial Statements are the main
financial analysis tools employed under Vertical Analysis.
2. Horizontal Analysis: When the Financial Statements (and other relevant schedules,
footnotes, annexures, etc.,) of an organization for two or more years are analysed and
interpreted, it is called Horizontal Analysis. Since the data for more than one year are
used, it is possible, under this type, to compare the performance of a company during a
year with that of the previous ye~ll"/s. This type of comparison helps to identify the
trends in various indicators of performance. Hence, Horizontal Analysis can also be
called Trend Analysis. On the ba~i.., oj these trends, a definite conclusion can be drawn
about whether the organization has Improved its profitability, solvency, liquidity, etc.,
over the years. It is, therefore, called d Dynamic Analysis. For instance, comparison of
performance of an undertaking for till.: year 2004 wit~ that of 2003 is an example to
Horizontal Analysis. Comparative Financial Statements and Trend Analysis are the two
important financial analysis tools employed under Horizontal Analysis.
II. Classification According to Material or Information Used: Analysis and interpretation of
Financial Statements may be external or internal depending upon the materials or information
used or the persons who are interested in, or undertaking, the analysis.
1. External Analysis: The Financial Statements which form a significant and a compulsory
part of external or annual reports are prepared and presented to the outsiders including the
shareholders and the employees. Normally. the external people do not have easy access
to the detailed accounting records of the orgallizations. For the purpose of taking
decisions, the external people have to rely only on these statements. They use only the
figures in the Financial Statements and lither supplements in the annual reports for the
purpose of analysis and interpretation to form an idea and to take appropriate decisions.
Hence, this type of analysis by the people external to the organizations is called External
Analysis. Due to this reason of paucity or non-availability of detailed information, this
type of analysis serves only a very limited purpose.
Analysis and Interpretation of Fmancial Statements : 29

2. Internal Analysis: The Financial StatePlents of an organization are also analysed and
interpreted by the people who are inteLlal to the organization, and who have an easy
access to the detailed records, for the ;mrpose of assisting the managerial personnel to
take corrective mf(asures and appropnate decisions. Thi<; is conducted by the people
inside the firm and for the benefit of the company. Hence. it is called Internal Analysis.
As complete set of information is available easily to the analyst, he can analyse the
performance of the organization clearly stating the reasons for improvements or
decreasing trends in various indicators of performance.
From the above analysis pertaining to the types of Analysis and Interpretation, it can be
said that it is necessary to have both Internal Analysis and Horizontal Analysis as they are more
comprehensive in their approach to include even the External Analysis and Vertical Analysis
respectively.
Methods, Techniques or Tools of Analysis
, and Interpretation
In order to analyse and interpret the data in the FinanCial Statements, the analysts may
use anyone or more of the following five methods or tools. They are (1) Comparative Financial
Statements; (2) Common-size Statements; (3) Trend Analysis: (4) Ratio Analysis; and (5) Cash
Flow and Funds Flow Statements.
Comparative Financial Statements
Comparative Financial Statements (i.e., both the Comparative Profit and Loss Account,
and the Comparative Balance Sheet)6 are prepared by providing columns not only for the year
just ended (for the purpose of convenience, this year may be called. current year) but also for the
year preceding the year just ended. Besides, columns are also normally provided for changes
over the year - both absolute and relative. That means, on each side of the Financial Statement,
columns are provided for:
a Monetary values of different items as found in the Financial Statements of the current period,
b. Monetary values of different items as found in the Financial Statements of the previous year,
c. Changes in the monetary values of the items of Financial Statements (in absolute terms in the
form of increase or decrease) in the current period when compared to the previous period, and
d. Changes in the monetary values of the items of Financial Statements in relative terms
(ratio, percentage) in the current period when compared to the previous year.
The above points become clear from the format presented below.
Comparative Profit and Loss Account of ABC Company for the years ended
December 31, 2003 and 2004
Amount Amount of Amount Amount of
Increase (or %age of Increase (or %ageof
(Rs.) Decrease) Increase (or Rs. Decrease) Increase (0
Particulars Particulars
in 2004 Decrease) in 2004 Decrease)
2003 2004 in 2004 2003 2004 in 2004
Rs. Rs.
Management Accounting : 30

Comparative Balance Sheet of ABC Company as on December 31, 2003 and 2004
As on Amount of As on Amount of
Increase (or %age of Increase (or %age of
! Capital December 31.
Decrease) !1lI.:rease (or Assets and December 31.
Decrease) Increase (or
I and
(Rs.) Decrease) Properties (Rs.) Decrease)
Liabilities in 2004 in 2004
in 2004 in 2004
2003 2004 Rs. 2003 2004 Rs.

With the help of these statements, it is possible to find out not only the balances of
accounts as on different dates and summaries of different operational activities of different
periods but also the extent of their increase or decrease between these dates. The changes one can
observe in different items represent the outcome of operations, interactions amongst assets,
liabilities and capital. The figures in the Comparative Statement may conveniently be used for
identifying the direction of changes and also to study the trends in different indicators of
performance of an organization. On this basis, the management can easily identify the areas in
which the organization has improved its performance and the areas in which it has failed. The
It:asons for the failure can also be identified. Further, the reasons for the failure can be classified
into controllable and non-controllable. There is no justification if there is any increase in costs or
decrease in revenue due to the failure or negligence on the part of the concerned officials.
Therefore, it draws the attention of the management to take appropriate actions so that their
repetition in future can be avoided.
Illustration: 2.1
The following are the Profit and Loss Accounts and Balance Sheets (Rs. in lakh) of ABC
Company for two consecutive years. From these statements, prepare both the Comparative Profit
and Loss Account, and the Comparative Balance Sheet.
Profit and Loss Account of ABC Company for the years ended December 31, 2002 and 2003
2002 2003 2002 2003
Particulars Particulars
Rs. Rs. Rs. Rs.
40 To Material Cost 45 80 By Sales Revenue 95
20 To Conversion Cost 20 10 By Miscellaneous Income 06
10 To Operating Expenses 08
05 To Miscellaneous Expenses 07
15 To Net Profit c/d 21
90 101 90 101
AnalysIs and Interpretation of Financial Statements. 31

Balance Sheet of ABC Company as on December 31, 2002 and 2003


December December December December
31,2002 Capital and Liabilities 31,2003 31.2002 Assets and Properties 31, 2003
Rs. Rs. Rs. Rs.
80 Share Capital (Equity 25
-+I Land
. and Buildings 20
Shares of Rs. 10 each) 100 50 Plant and Machinery 75
I
20 Reserves and Surplus 26 10 ! Furniture and Fittings 10
I
20 Long-term Liabilities 20 55 I Current Asse[~ 50
20 Current Liabilities 14 00 i Mj<;ccllaneoll~ Assets 05
I i
~ i ~
140 160 I 140 i 160
I -'-------"--
I

Solution:
Comparative P & L Ale of ABC Company for the years ended December 31, 2002 and 2003
(Rs. in lakh)
, I
Amount of '!c·ag.: 01 A mount of 'ii-age of
Particulars
2002 I 2003 Incrca~e tncrea,.: I P~lI tkular,
2002 2003 Increa~ Increase
(Rs.) I (R,.) (Decrease) ill ( Decrea'>e) (R,.) (Decrea,t:) in (Decrease)
2(0) (R,.) in 20m
I (Rs.) 2003 ( R,.) in 2003
To Cost of By Sale,
Materials Revenue 80 1)5 15 IR.75
consumed 40 45 05 12.50
By Misce·
To Conver· Ilaneous
sinn Cost 20 20 00 O.O() Income 10 06 (4) (40.00)

To Operating
Expenses 10 08 (2) (20.00)

To MiscelIa·
neou,
Expenses 05 07 02 ·moo
I
Total expen·
ses (8) 75 80 05 6.67 I
To Profit c/d
(A· B) 15 21 06 40.00

Total of Total
Expenses Revenue
and Profit 90 101 II 12.22 (AJ 1)0 WI I II 12.22
Management Accounting : 32

Comparative Balance Sheet of ABC Company as on December 31, 2002 and 2003 (Rs. in lakh)
As on %age of As on Amount of %age of
Amount of
December 31, Increase December 31, Incre-ase Increase
Increase (or
Capital and (Rs.) (or Assets and (or Decre- (or
Decrease) in (Rs.)
Liabilities Decrea- Properties ase) in Decre-
2003
se) in 2003 ase) in
2002 2003 (Rs.) 2003 2002 2003 2003
(Rs.)
Share Land and
Capital Buildings 25 20 (5) (20.00)
(Equity
shares of Plant and
Rs.lO -, Machinery 50 75 25 50.00
each) , 80 100 20 25.00
I Furniture and
Reserves ! Fittings 10 10 0 0.00
and
Surplus 20 26 6 30.00 Current Assets 55 50 (5) (9.09)

Long-term Miscella-neous
Liabilities 20 20 0 0.00 Assets 0 , 5 5 -
Current
Liabilities 20 14 (6) (30.00)

Total of
Capital Total of Assets
and and
Liabilities 140 160 20 14.29 Properties 140 160 20 14.29

The various figures and the computations in the above two comparative statements are
self-explanatory requiring no explanation. Anyhow, one or two points may be explained to show
how the Comparative Financial Statements method can be utilized for analysis and interpretation.
The company has succeeded in increasing its sales revenue by Rs. 15 lakh from Rs. 80 lakh in
2002 to Rs. 95 lakh during 2003 registering an increase by 18.75%. This upward change in the
sales revenue is a welcome change. The cost of materials consumed in the manufacture of goods
and services has increased by Rs. 5 lakh from Rs. 40 lakh to Rs. 45 lakh accounting for an
increase by 12.5%. That means, the rate of increase in the cost of raw-materials consumed is
lower than the rate of increase in the sales revenue. Both the material cost and the sales revenue
move normally in the same direction and in the same proportion provided there is no change in
the price, rate of economies in costs, waste factor, etc. Since there is no proportionate increase in
the material cost, the increase in the sales revenue might have been influenced even by the
increase in the selling price. In the same way, less-than proportionate increase in the material
cost might have been the outcome of downward changes in both the prices of raw-materials and
waste of raw-materials in the production processes. The outcome of this greater-than
proportionate increase in revenue and less-than proportionate increase in cost factors is the
increase in the profit (by Rs. 6 lakh or by 40%). This way, each and every item in the
Comparative Financial Statements can be examined to find out the areas of strength and weakness
so that the probable reasons can be identified. Once the reasons are identified, remedial measures
will be taken by the management.
Analysis and Interpretation of Financial Statements: 33

This type of presentation is also useful to the external parties (as the analysis and
interpretation by them are in the same pattern as identified above) to guide them to form an idea
about the organization. This leads to actions in the form of decisions. Recogni?:ing the
importance of Comparative Financial Statements, the entire corporate sector has been providing
the data even for the previous year. This is also compulsory under the Provisions of the
Companies Act, 1956. This type of Comparative Financial Statements are normally prepared on
yearly basis. But, if the Financial Statements are prepared monthly, quarterly or bi-annually,
Comparative Financial Statements can be prepared accordingly (i.e., monthly, quarterly or bi-
annually) so that seasonal fluctuations can also be identified.
In order to use this method of analysis, it is necessary to use the same accounting method,
practice and policy from one period to another period. Otherwise, the figures having comparable
value cannot be obtained. For instance, if a company has used Straight-line Method for
computing depreciation on building during 2003, the company should use the same Straight-line
• Method even during 2004 for ascertaining the depreciation on building. Further, it is also
necessary to use the same accounting method or procedure for the purpose of both the Financial
Statements. For example, if the Straight-line Method is used for computing depreciation on
building for Balance Sheet purpose, the same method should be used even for Profit and Loss
Account purpose. That means, consistency in the accounting principles, policies, practices and
procedures should be maintained. Otherwise, the very purpose of Analysis and Interpretation will
be defeated.
GUidelines for the Interpretation of Comparative Financial Statements
At the time of interpreting the results of analysis of Comparative Balance Sheet, the
analyst or the interpreter is expected to study three important aspects viz., (a) Liquidity, (b) Long-
term Financial Position, and (c) Profitability (through Proprietors' Fund).
As is known very well, working capital required for meeting day-to-day operating
expenses represents the excess of current assets over current liabilities. For the purpose of
commenting on the liquidity position of an organization, one should concentrate on the working
capital at the end of both the years and also the changes. If the change is in the form of increase,
then it indicates the improvement in the liquidity position of the company. However, more
working capital than required is also not desirable.
In order to study the long-term financial position, it is necessary to concentrate on the
changes in the fixed assets, long-term liabilities and capital. Usually, a sound financial policy
expects the company to finance the fixed assets by the issue of long-term financial instruments
such as shares, debentures, bonds or borrowing long-term loans from financial institutions.
Hence, it is necessary to study whether the addition to fixed assets is financed by long-term funds.
Proprietors' fund includes not only the share capital but also the free reserves created out
of profit. Increase in the amount of reserves, surplus, retained earnings, etc., indicates the amount
of profit earned and retained in the company. Hence, to analyse the profitability, to some extent,
emphasis should be laid on the increase or decrease in the reserves, surplus, etc.
While interpreting the results of analysis of Comparative Income Statement or
Comparative Profit and Loss Account, the analyst or the interpreter is expected to study three
important aspects viz., (a) Sales Revenue, (b) Costs, and (c) Profit.
Management Accounting: 34

Initially, it is necessary to study the changes in both the revenue and the (manufacturing)
cost of goods sold. Because, the difference between the increase in the sales revenue and the
increase in the cost of goods sold denotes the increase or decrease in gross profit depending upon
whether the:
Increase in Sales Revenue > Increase in Cost of Goods Sold
or
Increase in Sales Revenue < Increase in Cost of Goods Sold
In the second stage, comparison of changes in gross profit with the changes in operating
expenses shall be made to find out the effect of these two changes on operating profit. Because,
Operating Profit = (Gross Profit - Operating Expenses). It is to be noted at thIS stage that the
aggregate of administrative, and selling and distribution overhead expenses represents the
operating expenses. If the increase in gross profit is higher than the increase in operating
expenses, then one can find the increase in the operating profit in the current year.
In the final stage, it is necessary to study the impact of non-trading or non-operating
activities on the net profit of the company. If the non-operating income (sllch as interest received,
profit on sale of fixed assets, etc.,) exceeds non-operating expenses (sllch a~ interest paid, loss on
sale of fixcd as:-.eIS, elc.,), the exces~ non-operating 1I1come over non-operating expenses
represents the non-operating profit WhICh when added to operating profit will result in net profit.
Because, Net Profit = (Operating Profit + Non-operating Profit) where, Non-operating Profit =
(Non-operating Income - Non-operating Expenses).
This way, the results of the analysis of Comparative Financial Statements should be
interpreted to form an opinion about the profitability, cost economies, financial position, etc., and
these are to be included in the reports to the management for proper action.
Common-size Financial Statements
It is a known fact that the amollnt of capital, liabilities, sales turnover, etc., vary from one
period to another 'Inc! from one entity to another. When they differ, it is not advisable to
comment on the basis of pcrrormance~. in absolute money units. of two similar companies.
Hence. there is a need for a cummon base. Under this method, individual items of Profit and
Loss Account, ;.ll1d Balance Sheet are reduced to a common base which is equivalent to one
hundred. The procedure for the preparation of Common-size Statements is summarized below.
1. Common-size Profit and Loss Account: When each of the items of Profit and Loss
Accounts or IllL'nnle Statement is expressed as a percentage of sales revenue or total revenue,
it is called COI11/llon-size Profit and Lo,>,; Account. In order to prepare the Common-size
Profit and Loss Account, from the given TraditIonal Profit and Loss Account. the following
procedure is to be followed.
I. Total sales revenue or total revenue I., to be taken as one hundred depending upon the
composition of revenue of an entity;
2. Each item of expense is expressed as a percentage of revenue and even the profit is
expressed as a percentage of revenue; and
3.
Analysis and Inter~etation "
of Financial Statements : 35

Each type of revenue (e.g., trading revenue, non-trading revenue, or sales revenue,
interest received, dividend received, profit on sale of land, etc.,) is also expressed as a
percentage of total revenue.
Illustration: 2.2
From the following Profit and Loss Account of ABC company, prepare the Common-size
Profit and Loss Account.

Amount Amount
Particulars Particulars
Rs. Rs.
To Material Cost 3,00,000 By Sales Revenue 8,00,000
To Conversion Cost 2,50,000
To Administrative and Distribution
Overhead Expenses 1,50,000
To Profit c/d 1,00,000
8,00,000 8,00,000
Solution:
Common-size Profit and Loss Account of ABC Company for the year ending ....

Particulars % age Particulars (1'0 age

To Material Cost By Sales Revenue 100.00 I


I
[ 100 x Rs. 3,00,000~ I
8,00,000 37.50
To COllversion Cost

[100 x Rs. ')~,50,000~


8,00,000 31.25
To Administrative and Distribution
Overhead Expenses

[ 100 x Rs. 1,50,000~


8,00,000 18.75 I
To Profit I II
I
[ 100 x Rs. 1,00,000
8,00,000
J 12.50 I
f-
100.00
i 100.00 II
f-J. ~

Management ACCOU_n:t:g_ : 36
F.rom the above, one can identify the relative importance of different components of sales
revenue. In order to extract the maximum advantage from this method of analysis, it is advisable
to compute similar ratios for the previous year/s also so that the reasons for the decline or
increase in the profit may easily be identified. Therefore this method may be called
Comparative Common-size Financial Statements Method. Further, the figures in the
Common-size Statement may gainfully be compared with that of another company facilitating the
inter-firm comparison.
2. Common-size Balance Sheet: A statement in which the total of either the assets side or the
liabilities side is taken as 100, and each of the items of assets, liabilities and capital is
expressed as a percentage of total assets or total of liabilities and capital is called Common-
size Balance Sheet. The procedure for preparing the Common-size Balance Sheet is similar
to that of Common-size Income Statement which is evident from the following:
1. Either the total of the assets side or the total of the capital and liabilities side (as both are
same) is taken as one hundred;
2. Each type of asset is then expressed as a percentage of total assets (or total of the capital
and liabilities); and
3. Each item in the 'capital and liabilities' side is also expressed as a percentage of total
assets (or total of the capital and liabilities).
Illustration: 2.3
From the following Balance Sheet, prepare a Common-size Balance Sheet.
Balance Sheet of ... Company as on December 31, 2003
Amount Amount
Capital and Liabilities Assets and Properties
Rs. Rs.
Capital 100,00,000 Fixed Assets 88,00,000
Liabilities 58,00,000 Current Assets 70,00,000
1,58,00,000 1,58,00,000

Solution:
Common-size Balance Sheet of ... Company as on December 31, 2003
Capital and Liabilities % age Assets and Properties % age

Capital 63.29 Fixed Assets 55.70

[ 100
Rs. 158 lakh
x Rs. 100 lakh J [ 100
Rs. 158lakh
x Rs. 88lakh J
Liabilities Current Assets

[ 100 x Rs. 58 1akh J 36.71 [ 100 x Rs. 70 lakh J 44.30


Rs.1581akh Rs. 1581akh
100.00 100.00
Analysis and Interpretation of Financial Statements: 37

As shown above, the Financial Statements are prepared on the basis of the percentages.
These statements are, therefore, called common-size statements or cent per cent statements or
component percentage statements. Normally, Comparative Common-size Statements are
prepared by providing separate columns for different years and therefore, it helps to study the
trends. In order to have a gainful comparison of performance of one year with that of another
year, it is necessary to ensure that the organization has followed the same Accounting practices
and policies consistently year after year. Otherwise, the purpose will be defeated and no proper
conclusion can be drawn.
Further, it is possible to compare the performance of an organization with that of another
organization as the totals (of sales, assets, and capital and liabilities) of both the organizations are
reduced to a common size, viz., one hundred. For this purpose, it is necessary to ensure that both
the companies are following uniform Accounting policies and practices as otherwise figures of
comparable nature and value cannot be obtained.
By studying these figures, it is possible to measure the extent to which the percentage of
profit to sales is influenced by the changes in its determinants and also the extent to which each
determinant has caused the change in the profit figure. In the same way, the changes in the
various items of Balance Sheet may be computed and a comparison of these may be made to
interpret the results to take decisions.
Trend Analysis or Trend Ratios or Trend Percentages
Trend Analysis is an example for Horizontal Analysis. The method of comparing the
past performance or data over a period of time with that of a base year is called Trend Analysis.
In this method, the Profit and Loss Account, and the Balance Sheet of an accounting year are
taken as the base. This base-year may be the earliest year involved in the comparison or the
latest year or any intervening year. For instance, assume that a study of the Financial Statements
of an organization for a period of fifteen years from 1989 to 2003 has been undertaken. In this
case, the base year may be either 1989 or 2003 or 1999 or any other year of the study period.
Normally, the earliest year in the study period is reckoned as the base-year.
Every item in the base-year's Financial Statements is taken as equivalent to 100. All the
corresponding figures in the Financial Statements of other years are expressed as percentages of
their value in the base-year's Financial Statements. For instance, assume that a company has
incurred Rs. 5 lakh of material cost during 2001 and it increased to Rs. 8 lakh during 2002 and
decreased to Rs. 4 lakh during 2003. This information may be presented as follows.
Trend Analysis of Material Cost

Year: 2001 2002 2003

Material Cost (Trend %ages): 100*


Note: * Year 2001 is taken as the base year and therefore, the cost of materials during that
period is taken as 100. Cost of materials consumed during 2002 is expressed as a
percentage of the same material cost in the base year, 2001.

100
# [ Rs. 5 lakh x Rs. 8 lakh
J 100
$ [ Rs. 5 lakh x Rs. 4 lakh
J
Management Accounting : 38

From the above, it may be concluded that, if the amount of an item in another statement
is less-than the amount in the base statement, the trend percentage will be lower than 100 (see,
material cost for 2003) and if the amount is higher than the base amount, the trend percentage will
be greater than 100 (e.g., the material cost %age for 2002). The trend ratios are computed by
dividing each amount in the statements by the corresponding item in the statement taken as the
base.
Illustration: 2.4
From the following Comparative and Condensed Income Statement, compute the trend
percentages.
Comparative Income Statement of ABC Company
Particulars 2000 2001 2002 2003
Cost of Materials Consumed (Rs.) 2,00,000 2,50,000 2,00,000 1,80,000
Labour Cost (Rs.) 1,50,000 1,50,000 2,00,000 1,25,000
Other Expenses (Rs.) 1,50,000 2,00,000 1,00,000 1,50,000
Total Cost of Sales (Rs.) 5,00,000 6,00,000 5,00,000 4,55,000
Profit (Rs.) 3,00,000 3,00,000 2,50,000 3,45,000
Sales Revenue (Rs.) 8,00,000 9,00,000 7,50,000 8,00,000

Solution:
Computation of Trend Percentages (Base Year: 2000 = 100)
Trend Percentages
Particulars
2000 2001 2002 2003
1 2
Material Cost 100.00 125.00 100.00 90.003
Labour Cost 100.00 100.00 133.33 83.33
Other Expenses 100.00 133.33 66.67 100.00
4 5
Total Cost of Sales 100.00 120.00 100.00 91.006
Profit 100.00 100.00 83.33 115.00
Sales Revenue 100.00 112.50 93.75 100.00

Working Notes:

(
L 100
1. Rs. 2,00,000 x Rs. 2,50,000
J 100
2. [ Rs. 2,00,000 x Rs. 2,00,000J

100 ] 100 ]
3. [ Rs. 2,00,000 x Rs. 1,80,000 4. [ Rs. 5,00,000 x Rs. 6,00,000
Analysis and Interpretation of Financial Statements : 39

100
5. [ Rs. 5,00,000 x Rs. 5,00,000
J 100
6. [ Rs. 5,00,000 x Rs. 4,55,000
J'
In the above illustration, only the broad categories of costs have been considered for the
purpose of showing the computational procedure of trend ratios. There may be hundreds of items
in the Profit and Loss Account, and the Balance Sheet. However, the procedure is same.
Trend ratios may also be computed by using the immediately preceding year as the base.
But this approach is not helpful in drawing conclusions as the base differs from year to year. The
efficiency in the cUlTent year is influenced favourably in the indicators, though not actually, by
the inefficiency of the base year. Trend percentages emphasize on changes in the financial and
operational data from year to year. With the help of the trend ratios, it is possible to Identify the
areas in which the organization has achieved improvement over the years and the areas in which
it has not succeeded.
Illustration: 2.5
The following are extracted from the annllal reports of ABC Company for the years
ended December 31, 2002 and 2003. Using the same, construct a Comparati ve Income Statement
and comment on the performance of the company.
Particulars 2002 (Rs.) 2003 (Rs.)
Cost of raw materials consumed 5,00,000 6,00,000
Wages and salaries 3,00,000 3,10,000
Production overhead expenses 1,50,000 1,70,000
Operating expenses 2,00,000 2,35,000
Sales revenue 15,00,000 20,00,000
Output and sales (units) 1,00,000 1,20,000

Solution:
Comparative Income Statement of ABC Company for the years ended
December 31,2002 and 2003
Amount of %age of
Particulars 2002 (Rs.) 2003 (Rs.) Increase Increase
(Decrease) (Rs.) (Decrease)
Cost of raw-materials 5,00,000 6,OO,OUO 1,00.000 20.00
Wages and salaries 3,00,000 3,10,000 10,000 3.33
Production overhead expenses 1,50,000 1,70,000 20,000 13.33
Manufacturing cost of goods sold 9,50,000 10,80,000 1,30,000 13.68
Sales revenue 15,00,000 20,00,000 5,00,000 33.33
Gross Profit 5,50,000 9,20,000 3,70,000 67.27
Less: Operating expenses 2,00,000 2,35,000 35,000 17.50
Profit 3,50,000 6,85,000 3,35,000 95.71
Management Accounting : 40

During the year 2003, the company earned a profit of Rs. 6,85,000 registering an increase
by Rs. 3,35,000 or 95.71 % over the profit earned during 2002. The increase in profit during 2003
was due to a number of reasons. Thty include: (a) increase in sales volume and selling price
which resulted in the increase in the sales revenue; and (b) economies achieved in the costs of
production and sales. The summary of the same is presented below.
Profit earned during 2002 Rs.3,50,000
Add: Increase in profit due to increase in sales revenue Rs.5,00,000
Less: Decrease in profit due to increase in:
Material cost 1,00,000
Labour cost 10,000
Production overhead expenses 20,000
Operating expenses 35,000 1,65,000 3,35,000
:.Profit earned during 2003 6,85,000
Illustration: 2.6
The following figures are extracted from the annual reports of a company (Rs. in lakh).
~"'1
Particulars 1999 2000 2001 2002 2003

Cost of material 200 220 200 250 150


Labour cost 150 140 175 150 200
Other conversion costs 200 150 175 140 150
Total Manufacturing cost of goods sold 550 510 550 540 500
Sales revenue 1000 1100 950 1000 1200
Gross profit 450 590 400 460 700
Operating expenses 200 220 180 200 300
Profit 250 370 220 260 400

From the above, (a) Prepare Common-size Comparative Income Statement, and (b) Calculate
trend ratios for each item taking 1999 as the base year.
Solution:
Common-size Comparative Income Statement of ... Company for the Years ended ...

Particulars 1999 2000 2001 2002 2003

Cost of material 20.00 20.00 21.05 25.00 12.50


Labour cost 15.00 12.73 18.42 15.00 16.67
Other conversion costs 20.00 13.63 18.42 14.00 12.50
Total Manufacturing cost of goods sold 55.00 46.36 57.89 54.00 41.67
Analysis and Interpretation of Financial Statements: 41

Sales revenue 100.00 100.00 100.00 100.00 100.00


Gross profit 45.00 53.64 42.11 46.00 58.33
Operating expenses 20.00 20.00 18.95 20.00 25.00
Profit 25.00 33.64 23.16 26.00 33.33
Note: Sales revenue is taken as equal to 100 and each item (both cost and profit) is expressed as
a %age of sales revenue.
Trend Ratios (Base year = 1999)
Particulars 1999 2000 2001 2002 2003
Cost of materials 100.00 110.00 100.00 125.00 75.00
Labour cost 100.00 93.33 116.67 100.00 133.33
Other conversion costs 100.00 75.00 87.50 70.00 75.00
Operating expenses 100.00 110.00 90.00 100.00 150.00
Sales revenue 100.00 110.00 95.00 100.00 120.00
Others: Manufacturing cost of goods sold 100.00 92.73 100.00 98.18 90.91
Gross Profit 100.00 131.11 88.89 102.22 155.56
Profit 100.00 148.00 88.00 104.00 160.00

I
Illustration: 2.6
Consider the following relating to ABC Company and make a horizontal analysis.
2002 2003
Profit (Rs.) 3,00,000 5,00,000
Sales Revenue (Rs.) 30,00,000 40,00,000
Solution:
From the above, Profit to Sales Ratio can be computed for both the years. We know that,

Profit to Sale~
Ratio f =
[Profit
Sales Revenue x 100
J
Therefore, Profit to sales} [ Rs. 3 lakh
Ratio = Rs. 30 lakh x 100
J= 10% (2002) and

Rs. 5 lakh
[ Rs. 40lakh x 100
J= 12.5% (2003)
Management Accounting: 42

Incremental Profit to 1 ( Increment Profit ]


Incremental Sales Ratiq[ = Incremental Sales Revenue x 100

Rs. 2 lakh (i.e., Rs. 5 lakh - Rs. 3 lakh)


[ Rs. 10 lakh (i.e., Rs. 40 lakh - Rs. 30 lakh)
x 100
J = 20%

From the above, it is obvious that the company has increased its sales revenue by Rs. 10
lakh from Rs. 30 lakh in 2002 to Rs. 40 lakh in 2003 registering an increase by 33 1/ 3%, The
increase in sales revenue has resulted in an increase in the profit also. But what is important is
the rate of increase. Consequent to the increase in the sales revenue. the amount of profit has
2
increased from Rs. 3 lakh to Rs. 5 lakh (i.e., by Rs. 2 lakh) accounting for an increase by 66 / 3%,
That means, the rate of increase in the profit is much higher than the rate of increase in the sales
revenue. In other words, on the incremental sales revenue, the company has earned 20% profit,
and due to these reasons, the company has improved its Profit to Sales Ratio from 10% in 2002 to
12.5% during 2003. This way, it is possible, under Horizontal Analysis, to identify the different
factors which have caused changes in cost, revenue, profit, profitability, liquidity, etc.
Illustration: 2.7
Develop proforma income statement for the months of July, August and September for a
company from the following information.
a. Sales are projected at Rs. 2,25,000, Rs. 2,40,000 and Rs. 2,15,000 for July, August and
September respectively.
b. Cost of goods is Rs. 50,000 plus 30% of selling price per month.
c. Selling expenses are 3% of sales.
d. Rent Rs. 7,500 per month. Administration expenses for July are expected to be Rs.
60,000 but are expected to rise 1% per month over the previous month's expenses.
e. The company has Rs. 3,00,000 of 8% loan, interest payable monthly.
f. Corporate tax rate is 70%. lCA (Fin), November 1981J
Solution:
Income Statement of ... Company for the Months of July, August and September
July August September
Particulars Rs. Rs.
Rs.
Fixed Cost 50,000 50,000 50,000
Variable Cost (30% of sales) 67,500 72,000 64,500
1,17,500 1,22,000 1,14,500
Sales Revenue 2,25,000 2,40,000 2,15,000
Gross Profit (a) 1.,07,500 1,18,000 1,00,500
Less: Selling Expenses (3% of sales) 6,750 7,200 6,450
Analysis and Interpretation of Financial Statements: 43

Rent 7,500 7,500 7,500


Administration Expenses I 60,000 60,600 61,206
Operating Expenses (b) 74,250 75,300 75,156
Operating Profit (a - b) 33,250 42,700 25,344
Less: Interest (8%)2 2,000 2,000 2,000
Profit before Tax 31,250 40,700 23,344
Less: Corporate tax (70%) 21,875 28,490 16,341
Profit after tax 9,375 12,210 7,003
Workmg Notes:
1. July, Rs. 60.000;
August, (Rs. 60,000 + 1% of 60,000) =(Rs. 60,000 + Rs. 600) =Rs. 60,600
September, (Rs. 60,600 + 1% of 60,600) =(Rs. 60,600 + Rs. 606) =Rs. 61,206
2. Annual interest =8% of Rs. 3,00,000 = Rs. 24,000
Monthly interest =(Rs. 24,000 + 12 months) =Rs. 2,000
Illustration: 2.8
From the following information, compute the trend percentages taking 1996 as the base
year.

Year: 1996 1997 1998 1999 2000 2001 2002 2003

Sales Revenue (Rs. 00,000): 30 35 28 36 40 25 60 42


Operating Profit (Rs. 00,000): 06 08 05 08 ·09 04 12 10
Solution:
Trend Percentage
Sales Revenue Operating Profit Trend Percentages
Year
(Rs. 00,000) (Rs. 00,000) Sales Revenue Operating Profit
1996 30 06 100.00 100.00
1997 35 08 116.67 133.33
1998 28 05 93.33 83.33
1999 36 08 120.00 133.33
2000 40 09 133.33 150.00
2001 25 04 83.33 66.67
2002 60 12 200.00 200.00
2003 42 10 140.00 166.67
Management Accounting : 44

Illustration: 2.9
Your are given the following trend and common-size %ages for MPM Company for 2002
and 2003.
Trend %ages Common-size %ages
Particulars
2002 2003 2002 2003
Net Sales Revenue 100 120 100 100
Less: Cost of goods sold 100 - - -
Gross Profit on Sales 100 - - 40
Less: Operating expenses 100 - 25 20
Operating Profit 100 - 10 20
(Rs.20,000)

From the above information, compute the missing common-size and trend %ages. Also
determine the amount of operating profit earned by the company during 2003.
Solution:
Comparative Income Statement of MPM Company
Amount Common-size
Trend %ages
Particulars (Rs.) %ages
2002 2003 2002 2003 2002 2003
Net sales revenue I 2,00,000 2,40,000 100 100 100 120.00
5
Less: Cost of goods sold 1,30,000 1,44,000 65 60 100 110.77
Gross Profit"' 70,000 96,000 35 40 100 137.14
2
Less: Operating Expenses 50,000 48,000 25 20 100 96.00
Operating Profit] 20,000 48.000 10 20 100 240.00

Working Notes:
1. Operating profit for 2002 comes to Rs. 20.000 and it is equal to 10% of Sales Revenue.
Therefore, Sales Revenue = (Rs. 20,000 + 10%) = Rs. 2,00,000.
Trend %agc for 2003 is 120. Therefore, Net Sales Revenue for 2003 = 120% of 2002
sales revenue = (Rs. 2,00.000 x 120%) = Rs. 2,40,000.
2. 25% and 20% of sales revenue respectively.
3. 10% and 20% of sales revenue respectively.
4. Gross Profit = Operating Expenses -I- Operating Profit
5. Cost of Goods Sold = Sales Revem'~ - Gross Profit
Analysis and Interpretation of Financial Statements: 45

Illustration: 2.10
From the following particulars relating to N.R Pura Ltd., prepare a Comparative Income
Statement.
2001 (Rs.) 2002 (Rs.)
Sales 58,000 65,200
Cost of Goods Sold 47,600 49,200
Administration Expenses 1,016 1,000
Selling Expenses 1,840 1,920
Non-operating Expenses 140 155
Non-operating Incomes 96 644
Sales Returns 2,000 1,200
Tax Rate 43.75% 43.75%
[Kuvempu Uni, B.Com, October 2002J
Solution:
Comparative Income Statement of N.R Pura Ltd., for the years ended
December 31, 2001 and 2002
Amount (Rs.) Amount of %age of
Increase Increase
Particulars
2001 2002 (Decrease) in (Decrease)
2002 (Rs.) in 2003
Sales Revenue 58,000 65,200 7,200 12.41
Less: Sales Returns 2,000 1,200 800* 40.00
Net Sales Revenue 56,000 64,000 8,000 14.29
Less: Cost of goods sold 47,600 49,200 1,600 3.36
Gross Profit 8,400 14,800 6,400 76.19
Less: Administration expenses 1,016 1,000 (16) 0.57)
Selling expenses 1,840 1,920 80 4.35
Operating Profit 5,544 11,880 6,336 114.29
Less: Non-operating expenses 140 155 15 10.71
5,404 11,725 6,321 116.97
Add: Non-operating income 96 644 548 570.83
Taxable Income 5,500 12,369 6,869 124.89
Management Accounting : 46

Lass: Tax liability 2,406 5,411 3,005 124.90


Profit after Tax 3,094 6,958 3,864 124.89
* Reduction in sales returns is a favourable one from the view point of the company
Illustration: 2.11
From the following Income Statement of Sri Vardhaman Co., Ltd., prepare a
Comparative Income Statement [amount Rs in lakhs]
Year Ended
1996 1997
Net Sales 6,850 7,210
Cost of Good Sold 4,190 4,630
Gross profit 2,660 2,580
Operating expenses:
Selling Expenses 940 910
General and Administrative Expenses 470 1,410 460 1,370
Operating profit 1,250 1,210
Add: Other income: Dividend 220 250
1,470 1,460
Less: Other deduction: Interest paid 220 220
Net profit before tax 1,250 1,240
Less: Income tax 620 620
Net profit after tax 630 620
[Mangalore Uni, B.Com, October 2002J
Solution:
Comparative Income Statement of Vardhaman Co., Ltd., for the years ended
December 31, 1996 and 1997
Amount (Rs.lakh) Amount of
Cffage of
Increase
Increase
Particulars (Decrease) in
1996 1997 (Decrease)
1997 (Rs.
in 1997
Lakh)
Sales Revenue 6,850 7,210 360 5.26
Less: Cost of goods sold 4,190 4,630 440 10.50
Gross Profit 2,660 2,580 [80] [3.01]
Analysis and Interpretation of Financial Statements: 47

Less: Operating expenses:


Selling 940 910 [30] [3.19]
General and administrative 470 460 [10] [2.13]
Operating profit 1,250 1,210 [40] [3.20]
Add: Other income: Dividend 220 250 30 13.64
1,470 1,460 [10] [0.68]
Less: Other deductions: Interest 220 220 0 0
Net profit before tax 1,250 1,240 [ 10] [0.80]
t'
Less: Income tax 620 620 0 0
Net profit after tax 630 620 [10] [ 1.59]

Illustration: 2.12
The Income Statements of Sri Maruthi Limited are given for the years1998 and 1999.
Convert them into Comparative Income Statement and interpret the changes with percentage (Rs.
000).
1998 1999
Net Sales 785 900
Cost of goods sold 450 500
Operating Expenses:
General and Administrative Expenses 70 72
Selling Expenses 80 90
Non-operating Expenses:
Interest paid 25 30
Income Tax 70 80
[Mangalore Uni., B.Com., May 2002J
Solution:
Comparative Income Statement of Maruthi Limited for the years ended
December 31, 1998 and 1999
Amount of
Amount (Rs. 000) %age of
Increase
Increase
Particulars (Decrease)
(Decrease)
1998 1999 in 1999
in 1999
(Rs.OOO)
Net Sales Revenue 785 900 115 14.65
Less: Cost of goods sold 450 500 50 11.11
Management Accounting: 48

Gross profit 335 400 65 19.40


Less: Operating expenses:
General and administrative 70 72 2 2.86
Selling 80 90 10 12.50
Operating profit 185 238 53 28.65
Less: Non-operating expenses:
Interest paid 25 30 5 20.00
Profit before tax 160 208 48 30.00
Less: Income tax 70 80 10 14.29
Profit after tax 90 128 38 42.22

From the above Comparative Income Statement, the following things, amongst others,
become very obvious:
01. During the year 1999, the company increased its revenue by Rs.l,15,000 which works out to
a 14.65% increase when compared to the revenue earned during 1998;
02. The rates of increase in different categories of expenses (except interest) during 1999 when
compared to 1998 were lower than the rate of increase in sales revenue. For example, the
cost of goods sold increased by 11.11 %; and
03. Therefore, the company succeeded in increasing its profit after tax by Rs.38,000 registering
an increase by 42.22% when compared to the profit earned during 1998.
Illustration: 2.13
Revenue and expense data of Colombo Ltd., for two years is as follows:
2001 2002
Sales Rs. 6,56,500 Rs. 8,16,000
Cost of goods sold 3,16,500 4,00,000
Selling expenses 1,30,000 2,00,000
Sales returns 8,000 16,000
General expenses 78,000 1,20,000
Miscellaneous incomes 6,500 8,400
Income tax paid 67,600 32,000
Financial expenses 2,900 4,400

In order to boost sales during 2002, Rs.70,000 was spent on advertisement. Prepare
Comparative Income Statement for the company and comment upon changes that have taken
place in 2002 over the year 2001. [Mangalore Uni, B.Com, October 2003J
Analysis and Interpretation of Financial Statements : 49

Solution:
Comparative Income Statement of Colombo Ltd., for the years ended
December 31,2001 and 2002
Amount (Rs.) Amount of %age of
Increase Increase
Particulars
(Decrease) (Decrease)
2001 2002
in 2002 (Rs.) in 2002

Sales revenue 6,56.500 8,16,000 1,59,500 24.30


Less: Sales returns 8,000 16,000 8,000 100.00
Net sales revenue 6,48,500 8,00,000 1,51,500 23.36
Less: Cost of goods sold 3,16,500 4,00,000 83,500 26.38

Gross profi t 3,32,000 4,00,000 68,000 20.45


Less: Operating expenses:
Selling 1,30,000 2,00,000 70,000 53.85
General 78,000 1,20,000 42,000 53.85
Advertisement - 70,000 70,000 -

Operating profit 1,24,000 10,000 [1,14,000] [91.94]


Less: Financial expenses 2,900 4,400 1,500 51.72
1,21,100 5,600 [1,15,500] [95.38]
Add: Miscellaneous income 6,500 8,400 1,900 29.23
Profit before tax 1,27,600 14,000 [1,13,600] [89.03]
Less: Income Tax 67,600 32,000 [35,600] [52.66]

Profit after tax 60,000 -18,000 [78,000] [130.00]

From the above Comparative Income Statement, the following conclusions, amongst
others, can be drawn:
01. The amount of net sales revenue increased by only Rs. 1,51,500 which works out to 23.36%.
Similarly, the miscellaneous income also increased by 29.23%;
02. However, the rates of increase in all the expenses during 2002 when compared to 2001 were
higher than the rate of increase in net sales revenue. For example, the cost of goods sold
increased by 26.38%, selling expenses by 53.85%, etc; and
Management Accounting : 50

03. Therefore, the company incurred a loss of Rs.18,OOO representing the reduction in the amount
of profit by Rs.78,000 (i.e, - Rs. I 8,000 - Rs.60,000) which works out to a reduction by 130%.
Illustration: 2.14
The following are the balance sheets of A Company Ltd. (Rs. in lakhs)
1.1.2003 31.12.2003
Assets:
Net fixed Assets 900 1,000
Cash 50 50
Marketable Securities 100
Debtors 100 400
Stock of Goods 600 80U
Total 1,750 2,250
Liabilities:
Equity Shares 500 500
Reserves 400 600
Long Term Debt 100 100
Creditors 200 250
Payables 450 650
Provisions 100 ISO
Total 1,750 2,250

From the above data, prerare a Comparative Balance Sheet with increase or decrease in
both amount and percentage. [Manga/ore Uni, B.Com, November 2004]
Solution:
Camparative Balance Sheet of A Company Ltd., as on January 1,2003 and December 31, 2003
Amount of
Amount (Rs.) %age of
Increase
Increase
Parllculars (Decrease)
(Decrease)
1.1.2003 31.12.2003 in 2003
in 2003
(Rs.lakh)
Capital and Liabilities:
Equity share capital 500 500 0 0
Reserves 400 600 200 50.00
Long term debt 100 100 0 0
Analysis and Interpretation of Financial Statements: 51

Creditors 200 250 50 25.00


Payables 450 650 200 44.44
Provisions 100 150 50 50.00
Total of Capital and Liabilities 1,750 2,250 500 28.57
Assets and Properties:
Net fixed assets 900 1,000 100 11.11
Marketable securities 100 0 (l00) (100.00)
Stock of goods 600 800 200 33.33
Debtors 100 400 300 300.00
Cash 50 50 0 0
Total of Assets and Properties 1,750 2,250 500 28.57

Illustration: 2.15
The Balance Sheets of Sri Gopal & Co., Ltd., for the years 2001 and 2002 are given below:
Liabilities: 31-12-2001 31-12-2002
Equity Share Capital 6,00,000 12,00,000
12% Preference Share Capital 5,00,000 9,00,000
Reserve Fund 4,00,000 5,00,000
Profit and Loss Alc 2,00,000 3,00,000
Long-term Loans 2,00,000 5,00,000
Creditors 1,00,000 3,00,000
20,00,000 37,00,000
Assets:
Buildings 6,00,000 12,00,000
Machinery 4,00,000 8,00,000
Investments 4,00,000 5,00,000
Accounts Receivables 1,00,000 4,00,000
Cash at Bank 50,000 1,50,000
S~ock 4,50,000 6,50,000
20,00,000 37,00,000
Management Accounting: 52

You are required to prepare Comparative Balance Sheets and comment on the financial
position. [Bangalore Uni, B.Com, October 2002]
Solution:
Comparative Balance Sheet of Sri Gopal & Co., Ltd., as on December 31, 2001 and 2002
Amount (Rs.) as on Amount of
%age of
December 31, Increase
Increase
Particulars (Decrease)
(Decrease
2001 2002 in 2002
) in 2002
(Rs.)
Capital and Liabilities:
Equity share capital 6,00,000 12,00,000 6,00,000 100.00
12% Preference share capital 5,00,000 9,00,000 4,00,000 80.00
Reserve fund 4,00,000 5,00,000 1,00,000 25.00
Profit and Loss Account 2,00,000 3,00,000 1,00,000 50.00
Proprietors' Fund 17,00,000 29,00,000 12,00,000 70.59
Long term loans 2,00,000 5,00,000 3,00,000 150.00
Creditors 1,00,000 3,00,000 2,00,000 200.00
Total of Capital and Liabilities 20,00,000 37,00,000 17,00,000 85.00
Assets and Properties:
Buildings 6,00,000 12,00,000 6,00,000 100.00
Machinery 4,00,000 8,00,000 4,00,000 100.00
Investments 4,00,000 5,00,000 1,00,000 25.00
Fixed Assets 14,00,000 25,00,000 11,00,000 78.57
Accounts receivable 1,00,000 4,00,000 3,00,000 300.00
Cash at Bank 50,000 1,50,000 1,00,000 200.00
Stock 4,50,000 6,50,000 2,00,000 44.44
Current Assets 6,00,000 12,00,000 6,00,000 100.00
Total of Assets and Properties 20,00,000 37,00,000 17,00,000 85.00
currenl~ Current Assetsa [6,00,000J l2,00,OOOJ [6,00,000J
Ratio . Current Liabilitie 1,00,000 3,00,000 2,00,000
=6:1 =4:1 = 3:1

A careful observation of the Comparative Balance Sheet reveals a number of things about
the company's financial position and policy. However, an attempt is made in the following
paragraphs to identify a few aspects.
Analysis and Interpretation of Financial Statements : 53

01. During the year 2002, the company acquired Rs.ll ,00,000 worth fixed assets. However,
it raised only Rs. 3,00,000 of long term loans. This shows that the major portion of the
fixed assets acquired was financed with the help of the proprietors' fund which increased
by Rs.12,00,000 during 2002.
02. During 2002, the current assets increased by only 100% as against 200% increase in the
current liabilities (creditors). Hence, the Current Ratio declined from 6:1 (on December
31. 2001) to 4:1 by December 31, 2002. Still, the company's liquidity position is very
sound as it is having RsA worth current assets for every Re.l of current liabilities.
03. The company transferred Rs.l,OO,OOO to Reserve Fund, out of the profit for 2003, and the
balance amount in the Profit and Loss Account increased by Rs.l ,00,000. Both are the
signs of good financial policy
Illustration: 2.16
Following are the Balance Sheets of a company for the years 2000 and 2001. Prepare a
comparative balance sheet and comment on the financial position of the company:
Liabilities 2000 Rs. 2001 Rs. Assets 2000 Rs. 2001 Rs.
Share capital 3,00,000 4,00,000 Land and Building 1,85,000 1,35,000
Reserves and surplus 1,65,000 1,11,000 Plant and Machinery 2,00,000 3,00,000
Debentures 1,00,000 1,50,000 Furniture 10,000 12,500
Long term loan 75,000 1,00,000 Other Fixed assets 12,500 15,000
Bills payable 25,000 22,500 Cash at Bank 10,000 40,000
Creditors 50,000 60,000 Bills receivable 75,000 46,000
Other current liabilities 2,500 5,000 Debtors 1,00,000 1,25,000
Stock 1,25,000 1,75,000
7,17,500 8,48,500 7,17,500 8,48,500
[Bangalore Uni, B.Com, April 2004]
Solution:
Comparative Balance Sheet of ..... Company as on December 31, 2000 and 2001
Amount of %age of
Amount (Rs.)
Increase Increase
Particulars
(Decrease) in (Decrease)
2000 2001 2001 (Rs.) in 2001
Assets:
Land and Building 1,85,000 1,35,000 (50,000) (27.03)
Plant and Machinery 2,00,000 3,00,000 1,00,000 50.00
Furniture 10,000 12,500 2,500 25.00
Uther Fixed Assets 12,500 15,000 2,500 20.00
Total Fixed Assets 4,07,500 4,62,500 55,000 13.50
Management Accounting : 54

Cash at Bank 10,000 40,000 30,000 300.00


Bills Receivable 75,000 46,000 (29,000) (38.67)
Debtors 1,00,000 1,25,000 25,000 25.00
Stock 1,25,000 1,75,000 50,000 40.00
Total Current Assets 3,10,000 3.86,000 76,000 24.52
Total Assets 7.17,500 8,48,500 1,31,000 18.26
Liabilities:
Equity Share Capital 3,00,000 4,00,000 1,00,000 33.33
Reserves and Surplus 1,65,000 1,11,000 (54,000) (32.73)
Proprietors' Fund 4,65,000 5,11,000 46,000 9.89
Debentures 1,00,000 1,50.000 50,000 50.00
Long-term Loans 75,000 1,00,000 25,000 33.33
6,40,000 7,61,000 1,21,000 18.91
Bills Payable 25,000 22,500 (2,500) (10.00)
Creditors 50,000 60,000 10,000 20.00
Other Current Liabilities 2,500 5,000 2,500 100.00
Total Current Liabilities 77,500 87,500 10,000 12.90
Total Liabilities 7,17,500 8,48,500 1.31,000 18.26

Some of the points which become very clear from the Comparative Balance Sheet are
presented below.
01. The rate of increase in current assets [24.52%] is higher than that of current liabilities
[12.9%). Hence, the Current Ratio increased from 4:1 [i.e. 3,10,000 + Rs. 77,500] to
4.41:1 [i.e., Rs. 3,86,000 + Rs.87,500). Consequently, the short-term liquidity
position of the company has further improved.
02. During the year 2001, the company acquired Rs.55,000 worth fixed assets which was
financed entirely by the additional long term debt [both debentures and long-term
loans]. This is also a sound policy as it did not affect the working capital.
Illustration: 2.17
Balance Sheet of RX Ltd., as on December 31, 2001 and 2002 is given below:
Liabilities: 31-12-2001 31-12-2002
Preference share capital Rs.4,OO,OOO
Equity share capital Rs.5,00,000 5,00,000
Reserves and surplus 1,35,000 1,71,000
12% Debentures 2,00,000
Analysis and Interpretation of Financial Statements : 55

Bank overdraft 50,000 80,000


Sundry creditors 1,50,000 1,25,000
Provision for taxation 75,000 1,20,000
Proposed dividend 1,00,000 1,50,000
12,10,000 15,46,000
Assets:
Fixed assets 7,00,000 10,00,000
Investments (at cost) 1,00,000 1,20,000
Stock 1,50,000 1,80,000
Debtors 2,36,000 2,44,000
Cash 24,000 2,000
12,10,000 15,46,000

You are required to prepare Comparative Balance Sheet and comment on the financial
position. [Bangalore Uni, B.Com, November 2003J
Solution:
Comparative Balance Sheet of RX Ltd., as on December 31, 2001 and 2002
Amount (Rs.) as on Amount of %age of
December 31, Increase Increase
Particulars
(Decrease) in (Decrease)
2001 2002 2002 (Rs.) in 2002
Current Assets:
Stock 1,50,000 1,80,000 30,000 20.00
Debtors 2,36,000 2,44,000 8,000 3.39
Cash 24,000 2,000 (22,000) (91.67)
Investments* 1,00,000 1,20,000 20,000 20.00
Total Current Assets 5,10,000 5,46,000 36,000 7.06
Current Liabilities:
Bank overdraft 50,000 80,000 30,000 60.00
Creditors 1,50,000 1,25,000 (25,000) (16.67)
Provision for taxation 75,000 1,20,000 45,000 60.00
Proposed dividend 1,00,000 1,50,000 50,000 50.00
Total Current Liabilities 3,75,000 4,75,000 1,00,000 26.67
Working Capital 1,35,000 71,000 (64,000) (47.41)
Management Accounting: 56

Fixed Assets:
Fixed Assets 7,00,000 10,00,000 3,00,000 42.86
Total Assets 12,10,000 15,46,000 3,36,000 27.77
Preference share capital 0 4,00,000 4,00.000 -
Equity share capital 5,00,000 5,00,000 0 0
Reserves and Surplus 1,35,000 1,71,000 36,000 26.67
... Proprietors' Fund 6,35,000 10,71,000 4,36,000 68.66
12% Debentures 2,00,000 0 (2,00,000) (l00.00)
8,35,000 10,71,000 2,36,000 28.26
Total Capital and Liabilities 12,10,000 15,46,000 3,36,000 27.77

Note: *Assumed to be a part of Current Assets meant for conversion into cash whenever the
need arises.
From the above, the following conclusions, amongst others, can be drawn:
01. The Current Ratio which was very low at 1.36: 1 [i.e, Rs.5,1O,000 + Rs. 3,75,000] on
December 31, 2001 has further worsened by the end of 2002 as it fell to 1.15: 1 [i.e.
Rs. 5,46,000 + Rs. 4,75,000]. This was due to the increase in the current liabilities at
higher rate [26.67%] than that of current assets [7.06%]. This aspect should,
therefore, be given preference by the company.
02. During 2002, the compary redeemed its debenture capital [Rs.2,00,000] and acquired
Rs. 3,00,000 worth fixed assets. Major portion of this was financed with the help of
the issue of preference shares.
Illustration: 2.18
From the following Profit and Loss Accounts and the Balance Sheets of Asva Ltd., for
the years ended 31st December 1995 and 1996, prepare a Comparative Income Statement and a
Comparative Balance Sheet.
Profit and Loss Account (Rs in lakh)
1995 1996 1995 1996
To Cost of By Net Sales 800 1,000
Goods Sold 600 750
To Operating
Expenses:
Administration 20 20
Selling 30 40
To Net Profit 150 190
800 1,000 800 1,000
Analysis and Interpretation of Financial Statements: 57

Balance Sheet as on 31 st December (Rs in lakh)


1995 1996 1995 1996
Bills payable 50 75 Cash 100 140
Creditors 150 200 Debtors 200 300
Tax payable 100 150 Stock 200 300
14% Debentures 100 150 Land 100 100
16% Preference Capital 300 300 Building 300 270
Equity Capital 400 400 Plant 300 270
Reserves 200 245 Furniture
. 100 140
1,300 1,520 1,300 1,520
[Kuvempu Uni, B.Com, October 1997J
Solution:
Comparative Income Statement of Asva Ltd., for the years ended
December 31, 1995 and 1996

Amount (Rs. Lakh) Amount of


%age of
Increase
Increase
Particulars (Decrease)
(Decrease)
1995 1996 in 1996
in 1996
(Rs.lakh)
Net Sales Revenue 800 1,000 200 25.00
Less: Cost of goods sold 600 750 150 25.00
Gross Profit 200 250 50 25.00
Less: Operating expenses:
Administration 20 20 0 0
Selling 30 40 10 33.33
Net Profit 150 190 40 26.67

Comparative Balance Sheet of Asva Ltd., as on December 31,1995 and 1996


Amount of
Amount (Rs. Lakh) %age of
Increase
Increase
Particulars (Decrease)
(Decrease)
1995 1996 in 1996 (Rs.
in 1996
Lakh)
Capital and Liabilities:
Equity share capital 400 400 0 0
\
~.

Management Accounting: 58

16% Preference capital 300 300 0 0


Reserves 200 245 45 22.50
Proprietors' Fund 900 945 45 5.00
14% Debentures 100 150 50 50.00
1,000 1,095 95 9.50
Bills payable 50 75 25 50.00
Creditors 150 200 50 33.33
Tax payable 100 150 50 50.00
Current Liabilities 300 425 125 41.67
Total of Capital and Liabilities 1,300 1,520 220 16.92
Assets:
Land 100 100 0 0
Buildings 300 270 (30) (l0.00)
Plant 300 270 (30) ( 10.00)
Furniture 100 140 40 40.00
Fixed Assets 800 780 (20) (2.50)
Stock 200 300 100 50.00
Debtors 200 300 100 50.00
Cash 100 140 40 40.00
Current Assets 500 740 240 48.00
Total Assets 1,300 1,520 220 16.92

Illustration: 2.19
The Profit and Loss Account of a company is given below (Rupees in lakhs):
1997 1998 1997 1998
To Cost of goods sold 600 750 By Net Sales 800 1,000
By Non-operating
Operating pxpenses :
Income 50 100
To Admn. Expenses 30 40
To Selling Expenses 40 50
-
To Non-operating
Expenses 30 40
To Net Profit 150 220
850 1,100 850 1,100
Analysis and Interpretation of Financial Statements : 59

You are required to prepare a common-size income statement and interpret the changes.
[Bangalore Uni, B.Com, May 2002J
Solution:
Common-Size Income Statement [on comparative basis] of .' ... Company
for the years ended December 31, 1997 and 199'j
%ages %ages
Particulars Particulars
1997 1998 1997 1998
To Cost of goods sold 70.59 68.18 By Net Sales 94.12 90.91
To Admni.expenses 3.53 3.64 By Non-operating Income 5.88 9.09
To Selling expenses 4.70 4.54 .
To Non-operating expenes 3.53 3.64
To Net profit 17.65 20.00
100.00 100.00 100.00 100.00

From the above, it is obvious that the share of non-operating income in the total income of the
company increased from 5.88% to 9.09%. This shows a marginal decline in the relative share of sales
revenue. The company suceeded in the control of both the cost of goods sold and the selling
expenses. However, both the administrative and non-operating expenses registered a marginal
increase. As a result of all these, the net profit increased from 17.65% in 1997 to 20% during 1998.
Illustration: 2.20
From the following information, prepare a Common size Statement and make a brief
comment.
Swastik Co., Ltd.,
Income Statement [for the year ended 31. 3 ....... ]
1997 1998 1999
Net sales Rs. 6,20,000 Rs. 4,40,000 Rs. 2,80,000
Less: Cost of goods sold 4,30,000 3,02,000 2,04,000
Gross profit 1,90,000 1,38,000 76,000
Less: Operating expenses 1,26,000 69,000 44,000
Net operating profit 64,000 69,000 32,000
Less: Interest expenses 6,000 3,500 2,500
Profit before tax 58,000 65,500 29,500
Less: Provision for tax 25,000 35,000 14,000
33,000 30,500 15,500
[Mangalore Uni, B.Com, October 2000J
Management Accounting: 60

Solution:
Common-size (comparative) Income Statement of Swastik Co., Ltd.,
for the years ended March 31,1997,1998 and 1999
%ages %ages
Particulars Particulars
1997 1998 1999 1997 1998 1999
To Cost of goods By Net sales 100.00 100.00 100.00
sold 69.35 68.64 72.86
To Gross profit 30.65 31.36 27.14
100.00 100.00 100.00 100.00 100.00 100.00
By Gross profit 30.65 31.36 27.14
To Operating
expenses 20.32 15.68 15.71
To Net operating
profit 10.33 15.68 11.43
30.65 31.36 27.14 30.65 31.36 27.14
By Net Operat-
ing Profit 10.33 15.68 11.43
To Interest 0.97 0.80 0.89
To Profit before
tax 9.36 14.88 10.54
10.33 15.68 11.43 10.33 15.68 11.43
By Profit befor
Tax 9.36 14.88 10.54
To Provision for
Tax 4.03 7.95 5.00
To Net Profit
after Tax 5.33 6.93 5.54
9.36 14.88 10.54 9.36 14.88 10.54
Note: In thIS solutIOn, the details such as the gross profit, operating profit, profit before tax and
profit after tax are shown instead of only the net profit after tax.
The following points become very obvious from the above Common-size Income
Statement.
01. The cost of goods sold constitutes the major portion of sales revenue and its share has moved
in both tile directions indicating both the decrease (efficiency of the compary) and increase
(inefficincy) in its relative share.
02. Consequently, the Gross Profit Ratio has moved in both the directions representing the
increase and decrease in the Gross Profit Ratio.
Analysis and Interpretation of Financial Statements: 61

03. The company has succeeded' in lowering the Operating Expenses Ratio.
04. As a result, there is an improvement in the Profit after Tax Ratio.
Illustration: 2.21
Income statement of Vinyas Ltd., is given for the years 1998 and 1999. Convert it into
common-size income statement and interpret the changes.
Income Statement for the year ending 1998 and 1999
1998 Rs. 1999 Rs.
Gross sales 7,25,000 8,15,000
Less: Sales return 25,000 15,000
Net Sales 7,00,000 8,00,000
Cost of Sales 5,95,000 6,15,000
Gross profit 1,05,000 1,85,000
Operating Expenses:
Selling and Distribution expenses 23,000 24,000
Administrative expenses 12,700 12,500
Total Expenses 35,700 36,500
Operating Income 69,300' 1,48,500
Other Incomes 1,200 8,050
70,500 1,56,550
Non-operating Expenses 1,750 1,940
Net Profit during the year 68,750 1,54,610
[Mangalore Uni, B.Com, April and October 2001J
Solution:
Common size (Comparative) Income Statement of Vinyas Ltd., for the
years ended December 31, 1998 and 1999 '
%ages %ages
Particulars Particulars
1998 1999 1998 1999
To Cost of sales 84.85 76.11 By Gross sales * 103.39 100.86
To Gross profit 14.97 22.89 Less: Sales Return 3.57 1.86
99.82 99.00 Net .sales 99.82 99.00
To S & D expenses 3.28 2.97 By Gross profit 14.97 22.89
To Administrative expenses 1.81 1.55
To Operating Income 9.88 18.37
14.97 22.89 14.97 22.89
Management Accounting : 62

To Non-operating expenses 0.25 0.24 By Operating Income 9.88 18.37


To Net profit 9.81 19.13 By Other Income 0.18 1.00
10.06 19.37 10.06
w
19.37
Note: All the items including gross sales revenue are expressed as percentages of agreegate of net
sales revenue and other incomes [Rs.7,00,000 + Rs. 1,200 = Rs.7,01,200 for 1998, and
Rs.8,00,000 + Rs.8,050 = Rs.8,08,050 for 1999].
It can be observed from the above that the company has succeeded in exercising control
over costs and also to increase its revenue. Therefore, the relative shares of all costs to 'net sales
revenue and other incomes' have reduced [for instance, Cost of Sales reduced form 84.85% to
76.11 %] and this has enabled the company to increase Gross Profit, Operating Profit and Net
Profit Ratios substantially [from 14.97% to 22.89%; 9.88% to 18.37% and from 9.81 % to 19.13%
respectively).
Illustration: 2.22
From the following data, prepare the common size balance sheet of Anitha Ltd.,
31-12-2001 31-12-2002
Share Capital Rs.6,00,000 Rs.6,00,000
Reserves 1,50,000 3,00,000
Loans 50,000 2,00,000
Sundry Creditors 2,00,000 1,00,000
Buildings 4,00,000 6,00,000
Plant and Machinery 3,00,000 4,00,000
Stock 2,00,000 1,00,000
Sundry Debtors 80,000 60,000
Cash at Bank 20,000 40,000
[Kuvempu Uni, B.Com, May 2003J
Solution:
Common-size Balance Sheet of Anitha Ltd., as on December 31, 2001 and 2002
%ages as on %ages as on
Liabilities December 31, Assets December 31,
2001 2002 2001 2002
Share captial 60.00 50.00 Buildings 40.00 50.00
Reserves 15.00 25.00 Plant and Machinery 30.00 33.33
Loans 5.00 16.67 Stock 20.00 8.33
Sundry creditors 20.00 8.33 Sundry Debtors 8.00 5.00
Cash at Bank 2.00 3.34
100.00 100.00 100.00 100.00
Analysis and Interpretation of Financial Statements: 63

Illustration: 2.23
From the following Balance Sheet, prepare a common-size statemendB<:i:~ance
.', . Sheet):
1993 1994
Assets:
Cash Rs.27,000 Rs.31,500
Debtors 2,20,000 2,11,000
Stock 1,00,000 1,26,000
Pre-paid expenses 11,000 21,000
Bills receivable 10,000 10,500
Fixed assets 6,35,000 6,50,000
Total 10,03,000 10,50,000
Liabilities:
Share capital 6,58,000 7,00,000
Long-term debts 2,25,000 2,00,000
Sundry creditors 42,000 50,000
Other liabilities 78,000 1,00,000
Total 10,03,000 10,50,000

[Kuvempu Uni., B.Com., May 2000J


Solution:
Common-size Balance Sheet of ..... Company as on December 31,1993 and 1994
%ages as on %ages as on
Capital and December 31, December 31,
Assets
Liabilities
1993 1994 1993 1994
Share Captial 65.60 66.67 Fixed Assets 63.31 61.90
Long-term Debts 22.43 19.05 Stock 9.97 12:00
Sundry Creditors 4.19 4.76 Debtors 21.93 20.10
Other Liabilities 7.78 9.52 Bills Receivable 1.00 1.00
I
Prepaid Expenses 1.10 2.00
Cash 2.69 3.00
100.00 100.00 100.00 100.00
Illustration: 2.24
The Balance Sheet of X Ltd., as on December 31, 2001 is given below:
Liabilities Rs. Assets Rs.
Preference Share Capital 1,20,000 Land and Building 80,000
Management Accounting: 64

Equity Share Capital 1,50,000 Plant and Machinery 3,34,000


Reserves and Surplus 14,000 Temporary Investments 1,000
Long term loans 1,15,000 Inventories 10,000
Bills payable 2,000 Book-Debts 4,000
Creditors 12,000 Prepaid Expenses 1,000
Outstanding Expenses 15,000 Cash and Bank balance 8,000
Proposed Dividend 10,000
4,38,000 4,38,000
Prepare a common-size Balance Sheet. [Bangalore Uni, B.Com, April 2004]
Solution:
Common-size Balance Sheet of X Ltd., as on December 31, 2001
Particulars Amount (Rs.) %age
Assets:
Fixed Assets: Land and Building 80,000 18.26
Plant & Machinery 3,34,000 76.26
Total Fixed Assets 4,14,000 94.52
Current Assets:
Temporary Investments 1,000 0.23
Inventories 10,000 2.28
Book Debts 4,000 0.91
Prepaid Expenses 1,000 0.23
Cash and Bank Balance 8,000 1.83
Total Current Assets 24,000 5.48
Total Assets 4,38,000 100.00
Share Capital and Reserves:
Preference share capital 1,20,000 27.39
Equity share capital 1,50,000 34.25
Reserves and surplus 14,000 3.19
Total Capital and Reserve 2,84,000 64.83
Long-term Loans 1,15,000 26.25
Total Capital and long term loans 3,99,000 91.08
Current Liabilities:
Bills payable 2,000 0.46
Sundry creditors 12,000 2.74
Analysis and Interpretation of Financial Statements: 65

Outstanding Expenses 15,000 3.44


Proposed Dividend 10,000 2.28
Total Current Liabilities 39,000 8.92
Total of Liabilites and Capital 4,38,000 100.00

Illustration: 2.25
Following are the Balance Sheets of Sheela Company Ltd., and Shanti Company Ltd., as
on 31-12-2001:
Assets: Sheel Co., Ltd., Shanti Co., Ltd.,
Land and Buildings Rs.l,60.000 Rs.2,40,000
Plant and Machinery 6,00,000 12,50,000
Investments 2,00,000 4,00,000
Stock 3,00,000 4,00,000
Sundry Debtors 2,00,000 2,40,000
Cash and Bank Balances 1,40,000 2,70,000
16,00,000 28,00,000
Liabilities:
Equity Share Capital 4,00,000 6,00,000
12% Debentures 2,00,000 4,00,000
10% Preference Share Capital 4,00,000 5,00,000
Reserves and surplus 2,00,000 2,40,000
Dividend provision 1,00,000 1,40,000
Sundry creditors 3,00,000 8,20,000
Bank overdraft 1,00,000
16,00,000° 28,00,000
Compare the financial position of the two companies with the help of common-size
balance sheets and comment. [Bangalore Uni, B.Com, May 2002J
Solution:
Common-size Balance Sheet of Sheela Companly Ltd., and Shanti Company Ltd.,
as on December 31, 2001
Amount (Rs.) %ages
Particulars
Sheela Shanti Sheela Shanti
Capital and Liabilities:
Equity Share Capital 4,00,000 6,00,000 25.00 21.43
10% Preference share capital 4,00,000 5,00,000 25.00 17.86
Management Accounting: 66

Reserves and surpluses 2,00,000 2,40,000 12.50 I 8.57


Total Shareholders' Fund 10,00,000 13,40,000 62.50 47.86
12% Debentures 2,00,000 4,00,000 12.50 14.29
Dividend Provisions 1,00,000 1,40,000 6.25 5.00
Current Liabilities:
Sundry Creditors 3,00,000 8,20,000 18.75 29.28
Bank Overdraft 0 1,00,000 0 3.57
Total Current Liabilities 3,00,000 9,20,000 18.75 32.85
Total Liabilities 16,00,000 28,00,000 100.00 100.00
Assets:
,
Land and Buildings 1,60,000 I 2,40,000 10.00 8.57
Plant and Machinery 6,00,000 12,50,000 37.50 44.64
Investments 2,00,000 4,00,000 12.50 .14.29
Total Fixed Assets 9,60,000 18,90,000 60.00 67.50
Stock 3,00,000 4,00,000 18.75 14.29
Sundry Debtors 2,00,000 2,40,000 12.50 8.57
Cash 1,40,000 2,70,000 8.75 9.64
Total Current Assets 6,40,000 9,10,000 40.00 32.50
Total Assets 16,00,000 28,00,000 100.00 100.00

Comparison of the Current Assets percentage with that of Current Liabilities reveals that
Sheela Company Ltd., is very sound as its Ratios are 40% and 18.75% respectively. This
indicates that the Current Ratio is more than 2: 1. However, in the case of Shanti Company Ltd.,
these Ratios come to 32.5% (Current Assets) and 32.85% (Current Liabilities) and therefore, the
Current Ratio is little lower than 1: 1. This shows that Sheela Company's short-term liquidity
position is very sound and that of Shanti is very poor.
In the case of Sheela Company, the Ratios of both Fixed Assets and the Proprietors' Fund
are, more or less, same implying the financing of fixed assets with Proprietors' Fund and Long-
term Debt which is a sound policy. However, in the case of Shanti Company, fixed :lssets
account for 67.5% of the total assets, and the aggregate of shareholders' fund and long-term debt
account for only 62.15%. That mean~ . .this company is using the short-term debts to finance a
part of its fixed assets which is not desirable.
Illustration: 2.26
The following are the Balance Sheets of APAR Corporation Limited (Rs in lakhJ.
Liabilities: 31.3.02 31.3.03
Share Capital 200 300
Analysis and Interpretation of Financial Statements: 67

10% Debentures 100 200


10% Preference Shares 200 250
Reserves 100 120
Tax Provision 50 70
Creditors 150 410
Bank Overdraft 0 50
800 1,400
Assets:
Land and Building 80 120
Plant and Machinery 300 625
Investments 100 200
Stock 150 200
Debtors 100 120
Cash 70 135
800 1,400

Compare the financial position of the company with the help of Common-size Balance
Sheets and comment. [Mangalore Uni, B.Com, May 2004J
Solution:
Common-size (comparative) Balance Sheet of APAR Corporation Limited
as on March 31, 2002 and 2003
Amount (Rs.lakhs) as %ages as on March
Particulars on March 31, 31,
2002 2003 2002 2003
Capital and Liabilities:
Share Capital 200 300 25.00 21.43
10% Preference Shares 200 250 25.00 17.86
Reserves 100 120 12.50 8.57
Proprietors' Fund 500 670 62.50 47.86
10% Debentures 100 200 12.50 14.28
600 870 75.00 62.14
Tax Provision 50 70 6.25 5.00
Creditors 150 410 18.75 29.29
Bank Overdraft 0 50 0 3.57
Management Accounting: 68

Current Liabilities 200 530 25.00 37.86


Total of Capital and Liabilities 800 1,400 100.00 100.00
Assets:
Land and Buildings 80 120 10.00 8.57
Plant and Machinery 300 625 37.50 44.64
Total Fixed Assets 380 745 47.50 53.21
Investments* 100 200 12.50 14.29
Stock 150 200 18.75 14.29
Debtors 100 120 12.50 8.57
Cash 70 135 8.75 9.64
Current Assets 420 655 52.50 46.79
Total Assets 800 1,400 100.00 100.00
Note: * Investments are assumed to be for working capital purpose.
It is obvious from the above that the relative share of current liabilities increased during
2002-03 (from 25% to 37.86%) and that of current assets declined (from 52.5% to 46.79%). As
a result, the short-term liquidity position has worsened during 2002-03. One of the reasons for
this is the substantial increase in the share of creditors.
During 2002-03, the company increased its fixed assets' share. On the other hand, the
share of capital and long-term loans/liabilities declined implying financing of fixed assets with
short-term financial arrangements.
Illustration: 2.27
From the following data, calculate the trend percentages:
2000 2001 2002
Sales Rs.6,00,000 Rs.8,00,000 Rs.10,00,000
Cost of goods sold 3,00,000 5,00,000 6,00,000
Selling expenses 1,00,000 1,50,000 2,00,000
Administrative expenses 50,000 60,000 80,000
Financial expenses 30,000 40,000 20,000
[Kuvempu Uni, B.Com, May 1997 and October 2004J
Solution:
For the purpose of calculating the trend percentages, the year 2000 is used as the base
year.
Analysis and Interpretation of Financial Statements: 69

Trend Ratios [Base Year = 2000]


Amount (Rs.OOO) Trend Percentages
Particulars
2000 2001 2002 2000 2001 2002
Sales Revenue 600 800 1,000 100 133.33 166.67
Less: Cost of goods sold 300 500 600 100 166.67 200.00
:. Gross Profit 300 300 400 100 100.00 133.33
Less: Selling Expenses 100 150 200 100 150.00 200.00
Admn. Expenses 50 60 80 100 120.00 160.00
:. Operating Profit 150 90 120 100 60.00 80.00
Less: Financial Expenses 30 40 20 100 133.33 66.67
Net Profit 120 50 100 100 41.67 83.33

Illustration: 2.28
From the following information, interpret the results of operations of a manufacturing
concern using Trend Ratios. Use 1995 as base. Amount in lakhs of Rupees for the year ended:
1995 1996 1997 1998
Net Sales 100.00 95.00 120.00 130.00
Cost of goods sold 60.00 58.90 69.60 72.80
Gross profit 40.00 36.10 50.40 57.20
Operating expenses 10.00 9.70 11.00 12.00
Net operating profit 30.00 26.40 ·39.40 45.20
[Bangalore Uni, B.Com, April 2001 and Mangalore Uni, B.Com., October 2000J
Solution:
Computation of Trend Ratios [Base Year = 1995]
Amount (Rs. Lakhs) Trend Percentages
Particulars
1995 1996 1997 1998 1995 1996 1997 1998
Sales Revenue 100.0 95.0 120.0 130.0 100 95.00 120.00 130.00
Less: Cost of goods
sold 60.0 58.9 69.6 72.8 100 98.17 116.00 121.33
Gross profit 40.0 36.1 50.4 57.2 100 90.25 126.00 143.00
Less: Operating
expenses 10.0 9.7 11.0 12.0 100 97.00 110.00 120.00
Net operating profit 30.0 26.4 39.4 45.2 100 88.00 131.33 150.67

From the above trend ratios, the following interpretations of the operating results can be
made:
Management Accounting : 70

01. For the year 1996, the sales trend ratio is less-than 100 indicating the reduction in the sales
revenue. However, the trend ratio of cost of goods sold is higher [98.17%] than that of sales
revenue (95%). Consequently, the trend percentage of gross profit is very low at 90.25.
Further, the operating expenses work out 97% which is again higher than the sales ratio of
95%. Therefore, the net operating profit for 1996 is only 88% of that for 1995.
02. However, for the years 1997 and 1998, the sales revenue trend ratios are higher than their
cost trend ratios. Hence, the gross profit and net operating profit trend ratios are more than
100 indicating higher amounts of profit than in the year 1995. During these two years, the
company not only increased its revenue but also succeeded in keeping the costs at lower
levels.
Illustration: 2.29
From the following figures of Om Swastik Co., Ltd., calculate the percentages taking
1995 as base and interpret them (amount Rs in lakhs).
Year Ended Sales Stock Profit Before Tax
1995 1,881 709 321
1996 2,340 781 435
1997 2,655 816 458
1998 3,021 944 527
1999 3,768 1,154 672
[Mangalore Uni, B.Com, October 2002]
Solution:
Computation of Trend Percentages [Base Year = 1995]
Amount (Rs.lakh) Trend Percentages
Year ended
Sales Stock PBT* Sales Stock PBT
1995 1,881 709 321 100.00 100.00 100.00
1996 2,340 781 435 124.40 110.16 135.51
1997 2,655 816 458 141.15 115.09 142.68
1998 3,021 944 527 160.61 133.15 164.17
1999 3,768 1,154 672 200.32 162.76 209.35
Note: * PBT = Profit before Tax
The trend ratios for all the three variables are more than 100. Hence, the monetary values
are higher in the subsequent years than in the base year 1995. For example, the trend ratio of
sales for 1999 is 200.32 which indicates that the amount of sales revenue earned during 1999 was
200.32% of the revenue earned during 1995.
Another important point is that the trend percentages of profit before tax are higher for
all the subsequent years than that of sales. For instance, the trend percentages for 1999 are
209.35 (gross profit) and 200.32 (sales). This indicates the ability of the company to exercise
control over costs.
Analysis and Interpretation of Financial Statements: 71

Comparison of sales and stock percentages reveals that the trend ratios for stock were
lower than for sales implying less-than proportionate increase in the stock when compared to
sales revenue.
Illustration: 2.30
The following data are available from the Profit & Loss Account of Synthetic Ltd.
Year Sales Wages Selling Expenses Gross profit
1999 Rs.l,24,000 Rs.43,000 Rs.I0,300 Rs.36,000
2000 1,31,000 43,000 11,600 38,000
2001 1,28,000 46,000 11,900 31,000
2002 1,33,000 48,000 11,100 32,000
2003 1,30,000 50,000 12,000 33,000
You are required to show: (a) Trend ratios of different items, and (b) Trend percentage
of relationship of Wages, Selling Expenses, Gross Profit to Sales.
[Mangalore Uni, B.Com, November 2004J
Solution:
Computation of Trend Ratios (Base Year = 1999)
Amount (Rs) Trend Ratios
Year Selling Gross Selling Gross
Sales Wages Sales Wages
Expenses Profit Expenses Profit
1999 1,24,000 43,000 10,300 36,000 100.00 100.00 100.00 100.00
2000 1,31,000 43,000 11,600 38,000 105.65 100.00 112.62 105.56
2001 1,28,000 46,000 11,900 31,000 103.23 106.98 115.53 86.11
2002 1,33,000 48,000 11,100 32,000 107.26 11l.63 107.77 88.89
2003 1,30,000 50,000 12,000 33,000 104.84 116.28 116.50 9l.67

(b) Sales Ratio is higher than 100 for all the subsequent years (when compared to the
base year 1999) indicating earning higher amounts of sales revenue than in the base year.
However, the Ratio has moved in both the directions - increase and decrease during the study
period. The trend ratios of wages for the last three years are higher than that of sales implying
greater than proportional increase in wages. Similarly, the trend ratios of selling expenses for all
the four subsequent years are higher than for sales revenue. This also indicates the greater than
proportional increase in selling expenses when compared to the sales revenue. As a result, the
Gross Profit Ratio is less-than 100 for the last three years. However, for the year 2000, it is
105.56% and one of the reasons for this is the non-increase in wages during 2000.
Summary of the Chapter
Though the Financial Statements disclose both the operating efficiency and the financial
position of the business concerns, they are suffering from a few limitations. These aspects,
besides the nature and objectives, of Financial Statements are discussed at the beginning followed
Management Accounting : 72

by the meaning of Analysis and Interpretation. Classification of Analysis and Interpretation of


Financial Statements on the basis of method of operation into Vertical Analysis and Horizontal
Analysis, and on the basis of information or materials used into External Analysis and Internal
Analysis are discussed in the next stage. In the last part of the chapter, three methods of Analysis
and Interpretation of Financial Statements viz., Comparative Financial Statements, Common-size
Financial Statements and Trend Analysis are discussed followed by an attempt to solve a few
problems.
Key Terms to Remember
Financial Statements Balance Sheet
Profit and Loss Account Analysis and Interpretation
Vertical Analysis Horizontal Analysis
External Analysis Internal Analysis
Comparative Financial Statements Comparative Balance Sheet
Comparative Profit and Loss Account Common-size Statements
Common-size Profit and Loss Account Common-size Balance Sheet
Trend Analysis Base Year
Notes and References
1. Sidney Davidson and Roman L. Weil (Eds), Handbook of Modern Accounting, New
York, McGraw-Hill Book Company, 2nd Ed., 1977, p.1.12.
2. Eric L. Kohler, A Dictionary for Accountants, New Delhi, Prentice-Hall of India Private
Limited, 5th Ed., 1978, p.208.
3. Ibid., p.93.
4. AICPA as quaoted in N.P. Srinivasan, Management Accounting, New Delhi, Sterling
Publishers Private Limited, 2nd Revised Ed., 1988, p. 19.
5. Metcalt and Titard as quoted in N. Vinayakam and LB. Sinha, Management Accounting
- Tools and Techniques, Bombay, Himalaya Publishing House, 1st Ed., 1988, p. 190.
6. Though' it is possible to prepare even other statements (other than the Financial
Statements) in a comparative form, only the Income Statement and the Balance Sheet are
usually prepared on a comparative basis as these contain the effects in summary form of
all the important business transactions.
Questions for Self-Study
01. Discuss the concept and nature of Financial Statements.
02. Critically examine the objectives of Financial Statements.
03. What are the limitations of Financial Statements? Explain how the Analysis and
Interpretation of Financial Statements help to overcome these limitations.
04. Explain the importance of Financial statements. [Bangalore Uni, B.Com, May 2002J
05. What are the limitations of Financial Statements? [Kuvempu Uni, B.Com, November 2004J
Analysis and Interpretation of Financial Statements: 73

06. Define Financial Statement AnalysIs and explain in what ways such an analysis will benefit
both the internal and the external parties.
07. Distinguish between Vertical and Horizontal Analysis of financial data.
08. Explain the meaning of Internal Analysis and distinguish it from Vertical Analysis.
09. Explain Static Analysis and state clearly how it differs from Dynamic Analysis.
10. Explain the different types of Financial Analysis. Distinguish b,etween Horizontal and
Vertical 'Analysis. [Mangalore Uni, MBA, January 1992J
11. Cntically examine the utility of Financial Statements in the light of their limitations
[Mangalore Uni, MBA, November 1992J
12. What is Financial Analysis? What are its objects?
[Mangalore Uni, MBA-Ill, January 1992J
13, Write a note..on "Vertical Analysis". [Kuvempu Un;' B.Com, October 1997J
14. Discuss the objects of, and steps in~ Financial Analysis? Differentiate between Horizontal'
Analysis and Vertical Analysis. ' [Kuvempu Uni, B.Com, October 19911J
15. Distinguish between Horizontal and Vertical Analysis
[l(uvempu Uni, B.Com, November 2000J
16. What are the Comparative Financi<d Statements? How are they prepared'!
[Kuvempu Uni., B.Com., November 2001J
17, Explain in brief the procedure for preparing the Comparative Financial Statements. What are
the benefits of such statements'!
18. What is meant by Common-size Financial Statements? How do you prepare them? What are
their uses to both the management personnel and the external parties?
19. Write a note on "Common-size Financial Statements".
[Kuvempu Uni, B.Com, May 1991 and November 2003J
20. What is a Common-size Statement? Illustrate a Common-size Balance sheet.
[Kuvempu Uni, B.Cam, October 1998J
21. Explain briefly the managerial uses of Trend Ratios. How are they coumputed?
22. What do you mean by Trend Analysis? What are its utilities?
[Kuvempu Uni, B.Com, May 1998J
23. What are Trend Ratios? Explain. [Kuvempu Uni, B.Com, October 1998]
24. Write a note on Trend Analysis.
[Kuvempll Uni., B.Com., October 1999, m,d May 2002 and 2003/
25. Examine the following Tools of Analysis and Interpretation of Financial Statements:
a. Trend Analysis
b. Common-size Statement [Kuvempll Uni, B.Com, November 2004J
26. What are the techniques of Financial Statement Analysis? Explain them briefly.
27. Discuss briefly the different techniques of Analysis and Interpretation of Financial
StatemYHts. [Ballgalore UIli., MBA, JUlie 1996/
Management Accounting : 74

28. "Decisions taken on the basis of Financial Statements may not be regarded as final and
accurate". Comment [Bangalore Uni., B.Com., May 2000 and 2001, and November 2002J
29. Discuss briefly the different techniques of Analysis and Interpretation of Financial
Statements. Also state their limitations on interpretations of Financial Statements.
[Kuvempu Uni, B.Com., May 1998 and 2001, and October 1999J
30. Discuss the various components of Interpretation. What are the different techniques used for
the Analysis and Interpretation of Financial Statements? Also state the limitations of
Financial Statement Analysis [Kuvemplt Uni., B.Com., May 2000J
31. What do you mean by Analysis and Interpretation of Financial Statements'! Explain briefly
the different techniques used for this purpose. [Kuvempu Uni., B.Com., November 2001]
32. A Few Short-answer Questions:
a. What is Trend Analysis?
[Bangalore Uni, B.Com, November 2000, 2001 and 2002, and May 2002J
b. Name any four tools of Management Accounting.
[Banga/ore Uni., B.Com., November 2000J
c. Give the meaning of Comparative Statements.
[Banga/ore Uni., B.Com., November 2000 and 2003, and April2004J
d. Give the meaning of Common-size Statement.
[Ba1lgaiore Uni., B.Com., November 2001 and May 2003J
Chapter - III

ACCOUNTING RATIOS
Objectives: Objectives of this chapter are:
To understand the Need for, and Importance of, Ratio Analysis,
To know the Meaning of Ratio, "Accounting Ratio and Ratio Analysis,
To learn the Benefits of, and Steps in, Ratio Analysis,
• To study the Modes of expression of Ratios and their interpretation,
To learn the procedure of computing various Ratios, understand their importance to evaluate
liquidity, solvency, and profitability of organizations, and
To solve a few problems on different kinds of Ratios and their application.
Structure
• Introduction - Ratio, Accounting Ratio and Ratio Analysis
• Steps in Ratio Analysis
Benefits and Objectives of Ratio Analysis
Modes of Expression of Ratios
• Interpretation of Ratios
• Classification of Ratios
o Source-wise Classification
Profit and Loss Account Ratios, • Balance Sheet RatiOS, • Inter-statement Ratios
o Importance-wise Classification
Primary Ratio, • Secondary Ratios, • Third Level RatiOS, • Fourth Level RCltios
o Functional or Purpose-wise Classification
Profitability Ratios, • Liquidity Ratios, • Activity Ratios, • Financial and Leverage Ratios
Profitability Ratios
o Gross Profit Ratio
o Operating Ratio
o Net Profit Ratio
o Net Income to Total Assets Ratio
o Return on Investment
o Return on Equity
o Earning per Share
Liquidity Ratios
o Current Ratio
o Liquid Ratio
o Absolute Liquidity Ratio
Activity or Turnover Ratios
o Stock Turnover Ratio
o Debtors' Turnover Ratio
o Creditors' Turnover Ratio
o Fixed Assets Turnover Ratio
o Working Capital Turnover Ratio
Solvency Ratios
o Debt-Equity Ratio
o Capital Gearing Ratio
o Proprietary Ratio
o Coverage Ratios
Advantages of Ratio Analysis
• Limitations of Ratio Analysis
Precautions
• Illustrations
• Summary of the Chapter
Key Terms to Remember
• Question~ for Self-study
Management Accounting: 76

Introduction - Ratio, Accounting Ratio, Ratio Analysis


Though a simple procedure of comparing the financial and operative data as under the
first three methods or tools of Analysis and Interpretation is possible and useful, the real picture
becomes clear only when the relation:-hip between the two figures having a cause and effect
relationship with each other is worked out and compared. Though the absolute figures are
valuable and form the base for analysis. they eonvey no meaning unless 'they are compared with
one anoth~r. For example, the information that Mr. P secured 95 conveys nothing. Because, no
conclusion can be drawn as to whether Mr. P is a good ancl intelligent student or not. It is because
of the reason that 95 itself does not convey anything. Instead. it raises a number of questions such
as (I) Is 95 the marks or percentage'! (2) Did he score 95 marks out of 100 marks or 600 marks?
etc. Suppose the answer to the above questions is '95 marks for 600 marks', then something can
be said definitely. The percentage of marks obtained to the nwrks for which the examination was
conducted comes to 15.83lk. He did not secure even ~5(1r which i~ the minimum percentage one
has to score to get the p.ass-class at the degree level. Therefore. one or the conclusions is that the
student is not intelligent. What is to be noted here is thatlJ5 by it:-.e1f conveys 'nothing but when it
is linked with another related aspect. it gi\'\~:- sunlL' hints about the intelligence of the student. In
the same way, the amount of profit earned by a company communicates no useful meaning but
when it is linked with either the sale~ or the capital employed. it conveys something definitely
about the protitability of the company.
A ratio refers to the establishmelll or relation:-hip between any two inter-related variables.
For example, both the amount of profit and the amount of :-ales revenue earned are inter-related as
one is influenced by another. Hence. a meaningful and useful relationship may be established
between these two. It is because of the reason that the amount, or the rate, of profit is influenced
by the sales revenue. While determining a ratio. it is desirable to divide the 'more favourable.
or desirable or significant one' by the 'Iess favourable or desirable or significant one'. For
instance, profit and sales revenue are two inter-related variables. A meaningful relationship can
be established bet\veen the two. Suppose. the sales revenue is divided by the amount of profit, the
result cannot be interpreted properly. But if the profit is divided by the sales revenue, it conveys
something definitely about the ability with which the company has earned the profit. That means,
though both sales and profit are important factors, profit is normally regarded as more important
than the amount of sales revenue.
Accounting Ratios :-how the inter-relationships that exist among various accounting data.
Accounting Ratios express the relationships, in mathematical terms. between two or more items
(of Financial Statements and others) which have a cause and effect relationship or which are
connected with each other in one way or the other. It is. therefore, obvious that no purpose will
be served by establishing the relationship between two entirely un-related figures.
Since the AnalY:-ls and Illlcrpretation of Financial Statements is made with the help 'of
ratios, it is called Ratio Anal)'sis. The ratio analysis is, therefore. an effective tool or a device to
diagnose the financial and operational diseases of business enterprIses. The Ratio Analysis of
Financial Statements stands for the process of arrangement of data, computation of ratios.
interpretation of the ratios so computed and projections through ratios.
Though the ratios are known and used from the time immemorial, the idea of using ratios
as a financial analysis tool was suggested by Alexander Wall, a German Scholar, way back in
Accounting Ratios : 77

1919. Subsequently, this has become a very usel'ul and powerful tool for analysis and
interpretation by both the internal and the external par!ie~ of hu~iness enterprises.
Steps in Ratio Analysis
The important steps involved in Ratiu Analysis are four in number which are presented
below.
1. Collect all the data required for computing the necessary ratios which in turn depends
upon the purpo~e of calculating the ratios:
2. With the help of the above information, compute the necessary ~ccounting ratios:
3. Compare the ratios so computed either with the ratios of the same company for the
previous yea'rls or with the standards set or with the ratios of its competing' I1rm/s or with
the rat.ios of its industry's average;. and
4. Interpret the ratios in the light of the comparison. draw inferences. and prepare and
submit reports to management.

Benefits or Advantages or Uses and Objectives of Ratio Analysis or Ratios


It is an irrefutable fact that the Ratio Analysis is an important. crfective and powerful tool
of financial analysis. As it is used to diagnose the financial and operational condition of the
organization, both the internal and the external parties will reap rruit~ or Ratio Analysis. In this
background, the important benefit~ of Ratio Analysi ... are presented IH.:rein LInder.
I. Accounting Ratios establish the' relationship between (WI) or more closely related
financial ligures. This enables the analysts to get a precise idea about the financial
significance of the data.
2. Ratio Analysis, as already stated, acb as an in ... trumcnt for diagnosing the operational and
tinancial health of the organizations. Henct'. i( help!'> to evaluate various aspects of the
business such as operational efficiency. profitahility. solvency. liquidity. etc.
3. Ratio Analysis helps the management in its planning and forecasting activities. Because,
with the help of the evaluation of past performance, trends in different variables can be
identitied which in turn help to forecast the future events. Based on thi!'>. plans for the
future can be formulated.
4. As the Ratio Analysis simplies the financial data, it becomes easy to understand the
FinaIwial Statements. - .
5. Ratio Analysis makes the financial data to narrate the story of the organization. Hence, it
is an effective means of communication.
6. Accounting Ratios help the managerial persons in their decision making task.
7. Ratio Analysis, through the evaluation of the performance, both segment-wise and
overall, helps management"in having effective control over different activities, operations,
segments, etc .. of the organization.
Management Accounting : 78
8. Accounting Ratios facilitate both the intra-firm and inter-firm comparisons. Because,
they provide the necessary ratios for comparison of performance of one department with
that of another or comparison of performance of one company with that of ~mother
company.
9. Ratio Analysis helps not only the internal parties but also the external parties such as
shareholders, creditors, customers, financial institutions, etc.
These are some of the important benefits or uses or advantages of Ratio Analysis. These
benefits of Ratio Analysis can also be used to understand the objectives of Ratio Analysis.
Because, the benefits which accrue to the parties (both internal and external) from the use of Ratio
Analysis can be interpreted to the effect that the Ratio Analysis keeps them as its objectives. In
this background, the above advantages are presented below as the objectives of Ratio Analysis
very briefly. The objectives of Ratio Analysis are as follows.
1. To establish the meaningful relationship between two or more closely .related financial
data.
2. To serve as an instrument of diagnosis to evaluate the various aspects of the business such
as operational efficiency, liquidity, long-term solvency, etc.
3. To help the management in its planning and forecasting activities.
4. To simplify the complex financial data and/or statement for tlie easy understanding of the
both the internal and the external parties.
5. To communicate to the interested parties about the performance of the company.
6. To facilitate the decisions ~y the management.
7. To help the management in having effective control over the activities of different
sections, departments, etc.
8. To enable both inter-firm and intra-firm comparisons.
9.. To help both the internal and external parties in their decisions.
Mode or Method of Expression of Ratios
Ratios can be expressed either as proportions or as rates or as percentages as presented
below.
1. A ratio may take the form of a pure ratio, simple ratio or proportion between two
variables. Here, the figures of the two items used for computing the ratio are expressed in
common denominator.. Examples are, Current Ratio = 3 : 2, Acid-Test Ratio = 1.2 : 1,
etc.
2. The ratio may also be expressed in the form of rate or times by dividing one item by
another. That means, quotient which is obtained by dividing one value by another is used
as a mode of expression. Examples are; Stock Turnover Ratio = 3 times, Average
Collection Period =20 days, etc. '
Accounting Ratios: 79

3. It (Le., Ratio) may also be expressed in the form of percentage which can be obtained by
mUltiplying the quotient by 100. For example, Net Profit Ratio = 30%, Return on Capital
= 12%, Operating Cost Ratio = 80%, etc.
Interpretation of Ratios
In order to reap the full benefits of Ratio Analysis, the ratios computed are to be
interpreted properly. Depending upon the scope of financial analysis (such as the period, area and
the number of firms) and also the available figures and ratios, anyone or more of the following
four methods of interpretation can be followed.
1. Interpretation of a Single Ratio: A single ratio may, by itself, convey some useful idea
about the company. For instance, Return on Investment (RoI) Ratio is capable of conveying
some useful hints about the company's protitability. To extract the maximum benetit. the
actual RoI accomplished by a company may be compared with its weighted average cost of
capital or it may be compared with the average Rol achieved by the industry.
2. Interpretation of a Group of Ratios: Interpretation of a single ratio will not be of much
help to the analysts as a number of ratios are to be used, compared and interpreted to get a
clear idea about the company's performance in a particular area. For instance, RoI Ratio
is a product of two ratios, viz., Profit Ratio and Capital Turnover Ratio. To find out the
factor responsible for increase or decrease in RoI Ratio, its determinant ratios (viz., Profit
Ratio and Capital Turnover Ratio) are to be computed. And to find out the reasons for
increase or decrease in Profit Ratio, a large number of cost ratios are to be computed and
interpreted. Therefore, interpretation of a group of inter-related ratios is usually
undertaken by the analysts - whether they are internal or external parties.
3. Interpretation of Trend Ratios: [n this ca"e, a single ratio or a group of inter-related ratios are
computed for a number of years and used for interpretation. Consequently, the analyst is in a
position to find out whether the corporate entity ha" improved its performance in the area during
the current year when compared to the immediately preceding year/so
4. Interpretation of Ratios for more than one Company: Besides the use of ratios for
assessing and inferring about a company's performance, the ratios are widely being used for
comparing the performance of one company with that of another company in the industry.
Classification of Ratios
A very large number of Accounting Ratios are used for the purpose of analysis and
interpretation of data in the Financial Statements and other supplementary statements. Thf!se
ratios may be categorised into two or more convenient and useful groups. A number of bases are
available for classification. A few important and widely used bases are summarized below.
Bases for Classification of' Ratios
Source-wise ClaSSIficatIon

Classification Importance-wise Classification


of Ratios Period-wise Classi fication
Functional or Purpose-wise Classification
Management Accounting : 80

Source-wise Classification
Under this method, Accounting Ratios are c1a~sified into three broad categories on the
basis of the source from which the accounting data arc extracted for computing the ratios. Each
category comprises of a number of ratios as presented below.
Source-wise Classification of Accounting Ratios
Gross Profit Ratio
Operating Ratio (and Efficiency Ratio)
Profit
and Net Profit Ratio
Loss Stock Turnover Ratio
Account
Ratios
Interest Coverage Ratio
Coverage
Ratios [
Fixed Dividend Coverage RatIO

,....---... Current Ratio


I - - - -... Liquid Ratio
Classification Balance I - - - -... Debt-Equity Ratio
of Accounting ... ~-~ Sheet ~- ........
Ratios I---':"'-.Proprietary Ratio
Ratios
I----.Capital Gearing Ratio
I - - - -... Total Assets to Net-worth Ratio

E
Return on Investment Ratio
Return on Proprietors' Fund Ratio
Net Profit to Total Assets Ratio
Inter-
statement ~Creditors' Turnover Ratio
Ratios
-L..... Debtors' Turnover Ratio
Earnings per Share
Price Earnings Ratio
Profit and Loss Account or Income Statement Ratios are those ratios which establish
the relationship between two items or two groups of items of the Profit and Loss Account or the
Income Statement. For example, Gross Profit Ratio. Plv Ratio. Operating Profit Ratio, Expense"
Ratio, etc.
Accounting Ratios: 81

The second category ratios are Balance Sheet Ratios which establish the relationship
between the two items or two groups of items of Balance Sheet. For example, Current Ratio,
Debt-Equity Ratio, Capital Gearing Ratio, Proprietary Ratio, Acid-test Ratio, etc.
The third type of ratios are the Inter-statement Ratios which deal with the relation<;hip
between the items of the Income Statement on the one hand and that of Balance Sheet on the
other. Debtors' Turnover Ratio, Assets Turnover Ratio, Return on Capital Employed, Working
Capital Turnover Ratio, etc., fall into this category.
Importance-wise Classification
The British Federation of Master Printers has used the significance or the importance of
the ratios as the base for the purpose of classification. Accordingly, the ratios have been
classified into four categories as presented in the next page.
The First Level Ratio is known as Primary Ratio as it indicates the overall performance
of an organization by establishing the relationship between the amoum 'Qf profit and one of its
determinants, viz., value of assets owned. As it is always expressed in terms of percentage, it
indicates the amount of profit earned for everyone hundred rupees of capital employed on as<;eh.
The Primary Ratio is further analysed into its constituent ratios which form the Socond
Level Ratios and which are known as Secondary or Explanatory or Supporting Ratio, rhese
ratios explain the reasons for the high or low Profit to Capital Employed Ratio.
The reasons for the difference In the profits may be further analysed and identified by
computing Third Level Ratios. The Third Le\C1 Ratios are in the form of general explanatory
ratios accounting for the differences in the Second Le\ eI Ratio-;.
Fourth Level Ratios are Specific Explanatory Ratios accounting for the differences in
the general explanatory ratios. These ratim pinpoint the specific reason!-. for the poor performance
or otherwise of an organization.
Period-wise Classification
On the basis of the period for which the ratios are computed, Accounting Ratio!-. may be
classified into two categories as structural ratios and trend ra~ios. The Structural Ratios are those
ratios which are computed by reckoning the financial data of only one period (say, year, six-
month period. quarter, month, etc). It is similar to Vertical Analysis. This type of ratio analysis
is, therefore, called Static Analysis. But in the case of Trend Ratios, financial data for more
than one period are considered and the ratios are computed for more than one period. Therefore,
it is similar to Horizontal Analysis. This type of ratio analysis is, therefore. called Dynamic
Analysis.
Management Accounting : 82

Importance-wise Classification of Accounting Ratios

Primary or Operating Profit to


~
First Level Assets Employed Ratio
Ratio

~
Secondary Operating Profit to Sales Ratio
or Second
Level Ratios Sales to Capital Employed Ratio

Factory Cost to Sales Ratio

Administration Cost to Sales Ratio


General
Accounting Explanatory or Selling and Distribution Cost to Sales
Ratios Third Level Ratio
Ratios
----- Sales to Fixed Assets Ratio

LC Sales to Current Assets Ratio

Direct Material Cost to Sales Ratio

Direct Labour Cost to Sales Ratio


Specific
Factory Overhead to Sales Ratio
Explanatory or
~
------+ Fourth Level
Ratios
Cost of Sales to Material Cost Ratio
'----+
Cost of Sales to Work-in-progress Ratio

Cost of Sales to Finished Goods Ratio

Cost of Sales to Debtors Ratio

Functional or Purpose-wise Classification


Any attempt to analyse and interpret the data by anybody is always made with a specific
purpose. It is, therefore, needless to state that a clear understanding of the purpose of analysis and
interpretation is necessary for taking up this work. The purpose of analysis and interpretation
differs from one party to another depending upon the type of economic link the party is having or
Accounting Ratios: 83

intends to have, with the company. For example, the financial institutions which have lent
money to the organizations are interested in finding out two things. One, whether the company is
able to pay the interest periodically, and two, whether the company is capable of paying back the
principal amount soon after the stipulated period. Hence, the financial analysis aims at computing
the ratios which shed light on these two things. On the other hand, the equity shareholders are
interested in finding out whether the company is capable of paying attractive dividend and
whether there is any scope for capital appreciation over the years. As a result, the analysis of
Financial Statements by the shareholders aims at computing some ratios which help to know these
two aspects.
From the above, it is obvious that each ratio performs a specific function in the form of
helping the analyst to get an insight about a particular thing. Functional or Purpose-wise
Classification of Accounting Ratios is, therefore, the most popular and useful method. On the
basis of the purpose the Accounting Ratios serve, they (i.e., Accounting Ratios) may be classified
into four categories as presented below.
Functional Classification of Ratios

Profitability Ratios

Accounting Liquidity Ratios


Ratios Activity Ratios
Financial and Leverage Ratios
The different Accounting Ratios presented above, classifying on different bases, are
discussed based on the Functional Classification of Accounting Ratios.
Profitability Ratios
With a very few exceptions, the entire corporate sector aims at maximizing the amount of
profit. Some of the public sector undertakings are also aiming at earning reasonable amount and
rate of profit. Even a few service oriented organizations are working on a 'no-profit, no-loss'
basis. Hence, a very few undertakings aim at some other objectives, other than the profit
maximization objective. Profit maximization is, therefore, one of the primary objectives of
business enterprises.
But profit figure in absolute terms does not· convey the proper meaning. Because, if we
rely on the absolute profit figures to draw conclusion, there is ample scope for a wrong
conclusion. Therefore, 'profitability is more important and relevant than the profit to assess the
earning capacity of the company. Profitability Ratios are, therefore, vital for many a number of
parties including the management and the shareholders. Profit should not be confused with
profitability though profit is one of the determinants of profitability. Profit represents the
absolute quantum of profit whereas profitability (profit + ability) refers to the ability of the firm to
earn profit. The Profitability Ratios are relative measurements. These ratios may be classified
into three categories.
Management Accounting : 84

Profitability Ratios

Gross Profit Ratio

Profitability Ratios
E Operating Ratio
Net Profit Ratio

Related to ...

Net Income to Total Assets Ratio


Return on Investment
Return on Equity

Earnlllgs per Share

Gross Profit Ratio

This Ratio estahlishes the relationship between Gross Profit (GP) on sales and Net Sales
in terms of percentage indicating the percentage of Gross Profit earned on sales. Gross Profit
which is '>ollldill1es referred to as Gross Margin is the difference between the net sales (i.e.,
Total Sales Revenue - Sales Return) and the cost of goods sold (i.e., Manufacturing Cost of
Goods Sold). That means.

Gross Profit = lNet Sales - Cost of Goods Sold1

Net Sales Revenue = [Total Sales Revenue - Sales Return I

Therefore. Gro<;s Profit Ratio = Gross Profit


[ Net Sales Revenue x
100 J
When thl" Ratio is subtracted frolll 100£;(. cost of goods sold as a (k,age of net sales is
obtained. That is. ';(age of Manufacturing Cost of Goods Sold to Sales = [lOOe;( - GP Ratiol. The
Gross Profit Ratio indicates the portion of saks revenue left for the recovery of administl ative,
and selling and distribution expenses and also towards the net protit. after the recovery of entire
factory cost of goods sold. Hence.

Sales Revenue = I Manufacturing Cost of Goods Sold + Gross Profit]

Gross} _ rAdministrative. and Selling)


Profit -land Distrihution ExpensesJ +
[operating
Profit
J
Accounting Ratios: 85

A high Gross Profit Ratio achieved by an organil.tltion. compared tl) that of the industry's
average. indicates the organization' '> succe~sful altempt to produce the product at relati vely lower
costs. With a view to analyse and identify the factors responsible for the variation in the gross
protit. it is necessary to study the effects of variou", elements of costs. and also the selling price
and sales volume on the company's profit. Further. in case of multi-product situation, it i~
advisable to compute the rate for each product separately. Because. the loss arising in one
product may be concealed by the high gross profit made in another.

Illustration: 3.1

Sangeetha Company is engaged in the production and salt: l)f a ..,ingle product viz .. Raga.
An excerpt from the performance result.., of the company for two years is pre!-.ented below. From
this, compute the Gross Profit RatIO for each year and Identify the probable reason'> for the
variations.

2002 2003
Sales revenue (Rs) 60.00.000 75,00,000
Cost of materials consumed (Rs.) 30.00,000 35.00.000
Labour cost (Rs) 20,00.000 20,00.000
Other conversion costs (Rs) 5,00.000 8.00.000
Gross profit (Rs) 5.00,000 12.00,000
Solution:

Gross ~ofit} = ( Gross Profit


Ratio Sales Revenue

Therefore, Gross Profit Ratio for.

2002 = [ Rs. 5 lakh


, R,>. 60 lukh
x 100J = 81/3'~
2003 _
,-
[R~. 12 lakh
Rs. 75 lakh x 1O(~ = 16%
The Gross Profit Ratio achieved during 2002 was a negligible one at 8'h% which the
company improved considerably to 16% during 2003. Still it is not a satisfactory rate as the
company has to recover two more elements of costs. viz., administrative, and selling and
distribution overhead expense.... Then only the company can find the profit element. Anyhow,
during the year 2003, it ha., improved it~ Profit Ratio .,ignificantly. Thl~ is evident from the fact
that with the 25% increa~e in the Sales Revenue, the company has IIlcreased its Gross Profit by
Management Accounting : 86

140%. This greater-than proportional increase in the profit is due to a number of reasons. While
analysing the reasons, it is assumed that the selling price of the product remained constant during
2003 at the same level as it was during 2002.
l. At 2002 Material Cost to Sales Ratio of 50%. [i.e., (Rs. 30 lakh-+ Rs. 60 lakh) X 100], the
material cost for 2003 would have been Rs. 37.5 lakh but for the economies achieved and
increased material productivity. the company has reduced the material cost by Rs. 2.5
lakh. That means. the increase in profit during 2003 due to the reduction in the material
cost comes to Rs. 2.5 lakh;
2. The total labour cost remained constant at Rs. 20 lakh during 2003 in spite of 25%
increase in the sales revenue. Consequently, the company has not incurred an additional
Rs. 5 lakh of labour cost [i.e., {(Rs. 20 lakh -+ Rs.60 lakh) x Rs.75 lakh} - Rs. 20 lakh]
and this has increased the profit during 2003 by Rs. 5 lakh;
3. Other conversion costs have increased at a higher rate than the rate of increase in the sales
revenue. At 2002 rate. they should have been at Rs. 6.25 lakh. But the company has
incurred Rs. 8lakh. Consequently, the profit for 2003 is reduced by Rs. 1.75 lakh;
4. The amount of profit for 2003 is increased by Rs. 1.25 lakh [i.e., {(Rs. 5 lakh -+ Rs. 60
lakh) x 75 lakh} - Rs. 5 lakh] due to the increase in the sales revenue.
The summary of the above analysis is presented in the form of a reconciliation statement
below.
statement sh owm~th e R econci'rlatIon
. 0f 2002 and 2003 P rofiIts
Amount
Particulars Rs.
Rs.
Profit earned during the year 2002 5,00,000
Add: Increase in profit in 2003 due to: decrease in material cost (l) 2,50,000
non-increase in labour cost (2) 5,00,000
increase in sales revenue (4) 1,25,000 8,75,000
13,75,000
Less: Decrease in profit in 2003 due to the greater-than proportional
increase in other conversion costs (3) 1,75,000
Therefore, Profit for the year 2003 12,00,000

Illustration: 3.2

XYZ Company is a multi-product concern engaged in the production and sale of two
products viz., x and Y. The operating results of the company pertaining to both the products for
2004 are presented below (Rs. in lakh). Using the same, compute the Gross Profit Ratio for each
product and also for the company as a whole.
Accounting Ratios : 87

X Y Total
Sales revenue 12 06 18
Manufacturing cost of goods sold 08 05 13
Gross profit 04 01 05

Solution:

Gross ~ofit} = ( Gross Profit


Ratio Sales Revenue x 100J

Therefore, Gross Profit Ratio for,

Product X, = [ Rs. 4 lakh


Rs.12lakh
x 100J = 33 '1,%

Product Y, =
Rs. Ilakh
[ Rs. 6 lakh
x 100J = 16 '1,%

XYZ Company, =[ Rs.5 lakh x 100J= 27.78%


Rs. 18lakh

It may be noted from the above that the overall Gross Profit Ratio has reduced
considerably due to the lower Gross Profit Ratio of Product Y. Though the company has earned
33 1/3% Gross Profit from Product x, this has been reduced to 27 .78% due to a lower Gross Profit
margin of only 162/3% from Product Y.

Operating Ratio

This Ratio takes into account the aggregate of manufacturing cost of goods sold and other
operating expenses on the one hand, and the net sales revenue on the other. Net Sales Revenue is
arrived at by deducting the Sales Return from the Gross Sales Revenue. Operating expenses
comprise of administrative overheads, and selling and distribution expenses. The Ratio, therefore,
establishes the relationship between the cost of sales and the sales revenue. It is always expressed
in percentage as in the case of Gross Profit Ratio. It is calculated by using the following formula.

operating}
Manufacturing
[ Cost of Goods Sold +
Operating
Expenses
J
Ratio = x 100 OR
( Sales
~evenue
J
Management Accounting : 88

operating} _ [cost of Sales


Ratio - Sales Revenue
x 100]
This Ratio recognizes both the direct and the indirect operating costs. and states them as a
percentage of sales. It indicates the portion of net sales absorbed by the cost of sales. As the high
Operating Ratio would leave a small margin to cover" or meet financial expenses such as interest.
dividends, tax, etc., it is less favourable. This Ratio may be used as an indicator of both the
profitability and the operational efficiency. Because. when the Operating Ratio is deducted from
100. the result represent" the Operating Profit Ratio which is nothing but expressing operating
profit as a percentage of sales. It may be noted at this stage that the difference between the Gross
Protit and the Operating Profit is the exclusion of operating expenses while computing the earlier
one. It may. therefore, be said that the difference between the Gross Profit Ratio and the
Operating Expenses Ratio represents the Operating Profit Ratio. The difference between 100and
Operating Profit Ratio is the Operating Cost Ratio or simply the Operating Ratio. The summary
of this analysis is presented below in the form of equations.

Gross Profit = [Operating Expenses + Operating Profit]


Gross Protit Ratio= IOperating Expenses Ratio + Operating Profit Ratio]
Operating Profit Ratio = [Gross Profit Ratio - Operating Expenses Ratio]
Operating Expenses Ratio = [Gross Profit Ratio - Operating Profit RatioJ
Operating Profit Ratio = [1 - Operating Ratio]
Illustration: 3.3

From the following extracts from the Income Statement of Extracts Company for the year
2004, compute both the Gross Profit Ratio and the Operating Ratio.

Sales Revenue Rs. 8 crore'


Manufacturing Cost of Goods Sold Rs. 4 crore
Selling and Administrative Overhead Expenses Rs. 2 crore

Solution:

Sales revenue Rs. 8 crore


Less: Manufacturing cost of goods sold 4
Gros~ profit 4
Less: Operating expenses 2
Operating profit 2
Accounting Ratios: 89

[ Gross Profit
1. Gross Profit Ratio = Sales Revenue x 100J ~ [Rs.4cmre
Rs. 8 crore x 100J =50%

. . ( Cost of Sales [RS. 6 crore x 100 J =75%


2. Operatmg Ratio = SIR
a es evenue X100J= Rs. 8 crore

3. Operating Profit Ratio = [100% - Operating Ratio] = [100% - 75%] = 25Ck

4. Operating Expenses}
-
[Operating Expenses ~_
X 100-
Rs. 2 crore x 100l = 25%
Ratio - Sales Revenue [ Rs. 8 crore )

5. Operating Profit Ratio = [Gross Profit Ratio - Operating Expenses Ratio]


= [50% - 25%] = 25%

6. Operating Ratio = [100% - Operating Profit Ratio] = [100% - 25%J = 75%

Further, Operating Cost = [Cost of Goods Sold + Operating Expenses]


Where, Cost of Goods Sold = [Material Cost + Conversion Costs], and

operating} _ /Administrative Selling and Distributionl


Expenses - l
Overheads + Overheads j
It is desirable to break-up the Ratio into various cost ratios. These ratios act as supporting
ratios to Operating ROatio explaining the reasons for the variation in the Operating Ratio during the
current year when compared to the previous year's performance. These supporting ratios may
also be used to unearth the reasons for the differer.ce in the operating ratios of two similar
undertakings. Each expense should, therefore, be linked to sales and expressed as a percentage of
sales. The four significant cost ratios are mentioned below.

1. Material CostL _ ICost of Materials Consumed x 100J


Ratio f -l Sales Revenue

2. conversion} = [ManufactUring Cost - Material Cost x 100J


Cost Ratio Sales Revenue

or

conversion} = [conversion Cost x 100 J


Cost Ratio Sales Revenue
Management Accounting : 90

3. Administration ExpensesL = rAdministration Expenses x 100J


Ratio J l Sales Revenue

~
e1ling and Distribution
4. Selling and Distribution} Expenses
Expenses Ratio

Illustration: 3.4
You are required to compute various accounting ratios from the figures extracted from the
income statement of PQR Company for the year 2004.

Cost of opening stock of raw-materials Rs.60,000


Labour cost 3,50,000
Purchases made during the year 6,00,000
Production overhead expenses 2,00,000
Closing stock of raw-materials 2,00,000
Administrative overhead expenses 1,25,000
Selling and distribution overhead expenses 40,000
Interest on debentures 10,000
Dividend received 20,000
Sales revenue 15,00,000
Solution:

With the help of the above information, a more meaningful income statement may be
prepared as shown below.

e
I ncome Statement 0 fPQRC ompany ~or the year enddD ecember 31, 2004
Amount
Particulars Rs.
Rs.
Cost of opening stock of raw-materials 60,000
Add: Purchases made during the year 6,00,000
Cost of raw-materials available for consumption 6,60,000
Less: Closing stock of raw-materials 2,00,000
Cost of raw-materials consumed 4,60,000
I Add: Conversion costs:
Accounting Ratios: 91

Labour cost 3,50,000


Production overhead expenses 2,00,000 5,50,000
Manufacturing cost of goods sold* 10,10,000
Sales revenue 15,00,000
Gross Profit 4,90,000
Less: Operating expenses:
Administrative overhead expenses 1,25,000
Selling and distribution overhead expenses 40,000 1,65,000
Operating Profit 3,25,000
Add: Non-operating income (dividend)
Less: Non-operating expenses (i.e., interest)
. 20,000
10,000 10,000
Profit before Tax 3,35,000

Note: *The company was able to sell all it produced during 2004 and it had no opening stock of
either the work-in-progress or the finished goods on January 1,2004.

1. Opera~ing1 _ r. Cost of Sales x 100


RatIo f - lSales Revenue
J
= [RS.1O,1O,000+RS.l,65,000 x 100J= [RS.l1.75,OOO x 1001 =78113%
Rs. 15,00,000 Rs. 15,00,000 )

This 78 113% is the aggregate of three expenses ratios as shown below.

Jr
(a) Material CostL_
Ratio f-
l ost of Material
Consumed
Sales Revenue
x 100 = Rs. 4,60,000 x 1001
lRs. 15,00,000 )
302/3%

(b) Conversion }
·
Cost RatIo =
~conversion
SIR
Cost
a es evenue x 100
] rRS. 5.50,000 l
= ~s. 15,00,000 x l00j = 36 2/3%

(c) Operating 1_ 'Operating Expenses


Expenses Rati9" -lSales Revenue x 100
]
=
Rs. 1.65,000 l
[ Rs. 15,00,000 x l00j= 11%

That means, Operating Ratio 78 113% = [Material Cost Ratio, 302/3% + Conversion Cost
Ratio, 36 2/3% + Operating Expenses Ratio, 11 %]. It may further be noted here that the 11 %
Operating Expenses Ratio is the sum of the following two ratios.
Management Accounting : 92

.. . E
(a) Ad mlnIstratlOn " R' Rs. 1,25,000
xpenses atlO = Rs. 15,00,000 x
100~) =
8 1/3 m
-10

(b) Selling and DistributionL _ [ Rs.40,000 100J = 2 2/3%


Expenses Ratio f - Rs. 15,00.000 x

2. Operating L _ [operating Profit


Profit Ratic.{ - Sales Revenue x 1
OoJ-- [RS. 3,25,000
Rs. 15,00,000 x
100 J-21
-
2/3O/t
0

= [100% - Operating Ratio] = [100% - 78 113%] = 21 2/3%

3. Gross L [ Gross Profit


Profit Ratiof = Sales Revenue
~
x lOOJ =
[ Rs. 4,90,000
Rs.15,00,000 x 100
J = 32 2/3%

4. In order to compute the Operating Expenses Ratio, one can use, in addition to the direct
formula of dividing the Operating Expenses by the Sales Revenue, the Gross Profit and
Operating Profit Ratios. Therefore,

Operating Expenses Ratio = [Gross Profit Ratio - Operating Profit Ratio]


=[322/3%-212/3%] = 11%
Note: Interest and dividend are not considered here as they are not the items of trading or
operating activities.

Net Profit Ratio

This Ratio establishes the relationship between the amount of net profit or net income and
the amount of sales revenue. Net Profit is arrived at by adding non-operating income (such as,
interest and dividend on investments, profit on sale of fixed assets, etc.,) to the operating profit
and deducting non-operating expenses (such as, loss on the sale of old assets, provision for legal
damages, etc.,) from such profits.

f ·t = [operating
Ne t Prol Pro fiIt
.
;- Non-operating] - [Non-operating]
Income Expenses

But the difficulty is in considering the amounts of interest and taxes. Because, there is no
single definition or concept to which all the concerned agree. Hence, a number of concepts and
formulae are found to compute the Net Profit Ratio. It depends upon the treatment of interest and
taxes while computing the amount of net profit (NP).
Accounting Ratios: 93

Net Profit before


1. Net Profit}= Taxes and Interest
Ratio [ Sales Revenue

~
rofit after Taxes
2. Net profi;L=
Ratio f
but before Interest
Sales Revenue x IO~
Net Profit after
3. Net P.rofiZL= Taxes and Interest x 100
J
RatIO f [ Sales Revenue

Illustration: 3.5

KMF Company has employed Rs. 10 crore on various assets and 60% of this capital is
being financed by the loan capital at an interest rate of 10% per annum. With the help of these
assets and other infrastructure, the company has produced 10 lakh units of product A which it has
sold at operating cost plus 50% profit on operating cost. In order to produce and sell these units.
the company has incurred variable manufacturing cost of Rs. 20 per unit, and variable
administrative and selling and distribution expenses of Rs. 5 per unit. Besides, the company has
incurred a fixed manufacturing cost of Rs. 1 crore and a fixed administrative, and selling and
distribution expenses of Rs. 40 lakh. The tax rate applicable to this type of organizations is 60%.
From these details, compute the Net Profit Ratio using different concepts of net profit.

Solution:

Before proceeding to compute the Net Profit Ratio, it is better to prepare an income
statement.

c ompu t af Ion 0 fS a Ies Revenue


Amount
Particulars Rs.
Rs.
Manufacturing Cost of Goods Sold:
Variable (10 lakh units x Rs. 20) 2,00,00,000
Fixed 1,00,00,000 3,00,00,000
Administrative, Selling and Distribution Expenses:
Variable (at Rs. 5 per unit) ,I 50,00,000
Fixed 40,00,000 90,00,000
Cost of Sales or Operating Cost 3,90,00,000
Management Accounting: 94

Add: Profit (50% of Cost) 1,95,00,000


Sales Revenue 5,85,00,000

I ncome statement 0 fKMFC omJ!anJ' f or theJ'ear ....


Amount
Particulars
Rs.
Sales Revenue 5.85,00,000
Less: Manufacturing Cost of Goods sold 3,00,00,000
Gross Profit 2.85,00,000
Less: Operating Expenses 90,00,000
Profit before Interest and Tax 1,95,00.000
Less: Interest (10% of 60% of Rs. 10 crore) 60,00,000
Profit before Tax (but after Interest) 1,35,00,000
Less: Tax (at 60%) 81,00,000
Profit after Tax and Interest 54,00,000
Profit after Tax but before Interest 1,14,00,000

Computation of Net Profit Ratios:

Net Profit before


1. Net Profi~ _ Taxes and Interest IRs. 5,85,00,000
1,95,00,000 x lOJ = 33 113%
Ratio f -( Sales Revenue lRs. )

~
et Profit after Taxes
2. Net Profi~ _ but before Interest
Ratio f - Sales Revenue x
1 ooJ = IRs. 1,14.00,000
~s. 5,85,00,000
10J) = 19.49%
x

The difference between 33 113% and 19.49% comes to 13.84%. That means, tax burden
comes to 13.84% of the sales revenue.

3. Net Profi}
Net Profit after
Taxes and Interest 0
J IRs. 54,00,000 x 10J = 9.23%
·
Ratto =( SIR
a es evenue x 1(1 = lRs. 5,85,00,000 )

The difference between 19.49% and 9.23% comes to 10.26% which is the percentage of
interest expense to sales revenue.
Accounting Ratios: 95

The Net Profit Ratio is widely used as a measure of over~Jl profitability and it is one of a
few ratios used by the proprietors as it indicates the portion of sales left to the proprietors after all
costs and charges have been deducted. A high Net Profit Ratio is desirable as it usually ensures a
higher return to the shareholders. Further, the company will be having its own internal resources
so that any expansion programme may be undertaken without 'lny financial problem. Besides, the
company will be capable of withstanding any unfavourable condition which may take place in the
future. A low Net Profit Ratio will have just opposite implicatIOns.

Profitability Ratios related to Investments

In order to carry out the production activities, it is necessary to invest huge amount of
capital on various types of assets especially on fixed assets v'hich are very essential to convert the
raw-materials into finished plOducts. Therefore, it is neces-;ary to raise the required fund from
outsiders who demand some return on their investment, besides the capital appreciation and
repayment. It, therefore, implies that the amount of capital so raised should be employed
judiciously and profitably. Hence, the ratios linking the amount of profit with the investments
made. The important ratios, as identified earlier, are Return on Total Assets, Return on
Investments and Return on Equity.

Return on Total Assets

This Ratio is also known as Profit to Assets Ratio and it establishes the relationship
between net profit and assets. As net profit and assets have a number of meanings, there are a
number of approaches to compute the Return on Assets Ratio. Usually, the following formula is
used to determine the Return on Total Assets Ratio.

t
Net Profit after
Return on Total} = Taxes and Interest
Assets Ratio Total Assets

The above Ratio tries to find out how efficient the company was in utilizing the funds to
generate or earn profit. But the above formula appears to be based on wrong procedure. Because,
in the denominator, both the funds raised from outsiders and also the funds generated internally
are considered. The funds raised from outsiders comprise" of both the share capital (both equity
share capital and preference share capital) and loan capital which bears fixed interest charges. On
the other hand, in the numerator, the net profit after ta'\:es and interest which represents the
profit available only to the shareholders (both equity and preference shareholders) is considered.
It is, therefore, necessary to consider net profit after taxes but before interest in the numerator.
Therefore,
Net Profit after Taxes
Return on Total} = but before Interest
Assets Ratio [ Total Assets
Management Accounting: 96

Sometimes, only the fixed assets or tangible assets are considered in the denominator.
Hence, the formulae are:

[
Net Profit after Taxes
Return on Total}
Assets Ratio = but before Interest
Tangible Assets
x too]

[ 11
Net Profit after Taxes
Return on Total} but before Interest
Assets Ratio = x
Fixed Assets

Return on Investment

This Ratio is also called Return on Capital Employed or Rate of Return. Though all
the three ratios discussed earlier reflect the profitability of an organization, all of them establish
the relationship between profit and sales. But there may be a situation wherein profit in terms of
s'jles may be adequate but sales with regard to capitai employed may be inadequate resulting in an
i-'-Iadequate profit to capital employed. The following illustration clarifies this point.

Illustration: 3.6

The following figures are extracted from the financial statements of two companies in the
same industry for a particular period. Using these details, comment on the Return on Investment.

Company P Company Q
Sales revenue (Rs. in lakh) 50 50
Profit 7 8
Capital employed 25 50

Solution:

Computation of a few ratios is a pre-requisite to comment on the Return on Investment.

1. Profit to
Ratio
Sale~
f -
_ [ Profit
Sales Revenue
x 100 J
Then-fore, Profit to Sales Ratio of:

Company P =[
Rs. 7lakh
-Rs. 50 lakh x 100 J= 14%
Accounting Ratios: 97

Company Q =[ Rs.8lakh
Rs. 50 lakh x 100
J= 16%
On the basis of this Ratio, it may be said that Company Q is more profitable than
Company P. But the conclusion 'Company Q is more profitable' may not be a correct one
as other aspects are not considered. Therefore, Capital Turnover Ratio is computed.

2. Sales to CapitalL _
Employed f -
r
Sales Revenue
19apital Employed
J
Therefore, Sales to Capital Employed Ratio of:

Company P =[
Rs. 50 lakh
Rs. 25 lakh
J=2 . tImes

Company Q =[
Rs. 50 lakh
Rs. 50 lakh
J=1 . tIme

From the point of view of Capital Turnover Ratio, Company P has achieved a good
performance of Rs. 2 sales revenue for every Re. 1 of capital employed as against only
Re. 1 of sales revenue for every Re. 1 of capital employed by Company Q. If these two
ratios are compared, there will be a confusion as to which company stands first in the
profitability. Both the ratios shown above concentrate on one aspect each of profitability
but not on both the aspects of profitability. Anyhow, what is apparent from the above is
that 'only satisfactory profit in terms of sales revenue is not sufficient but adequate sales
with reference to capital employed is also necessary'. Hence, there is a need for another
ratio which reckons all the determinants as listed above and that ratio is the Return on
Investment Ratio.

3. Return on InvestmentL = Profit to. sales] x Capita~


rSales to
Ratio [ J RatIO lEmployed Rati~

r Profit I r Sales Revenue* ] r Profit + I


= CSales Revenue] x lCapital Employed = l:apital Employed)

Note: * See Ou Pont Analysis which is explained at the end of the analysis of this Ratio:
+ Profit, here, is the Net Profit before Interest but after Taxes. Sometimes, net profit
before taxes and interest is also used;
$ Sales revenue in both the denominator and the numerator in the above equation gets
cancelled and therefore, only profit is divided by capital employed.
Management Accounting : 98

As the Return on Investment is always expressed in percentage, the above equation is to


be multiplied. by 100. Therefore,

Return on } [ Profit x 100 ]


Investment Ratio - Capital Employed

Therefore, Return on Investment Ratio of Company:


Rs. 7 lakh II [ Rs. 8 lakh ~
P = [ Rs. 25 lakh x 100j =28% ; Q = Rs. 50 lakh x 100j = 16%

This Ratio may also be computed using the two determinant ratios. That is, Return on Investment
Ratio = (Profit to Sales Ratio) x (Sales to Capital Employed Ratio). Therefore, in the case of
Company
P, Return on Investment Ratio = (14 x 2) = 28%
Q, Return on Investment Ratio = (16 x 1) = 16%

It is apparent from the above that Company P is more profitable than Company Q. The
abo.ve calculations and the analysis that followed signify the importance of Return on Investment
Ratio. This is the most important ratio indicating the earning power of the capital employed and
also reflecting the ability of the management to utilize the capital entrusted to it. It may be noted
here that there is no consensus among the experts as to what constitutes the capital employed.
Some experts opine that it represents the total of the assets side of the Balance Sheet; some others
feel that it denotes the shareholders' fund. Anyhow, two important approaches are identified here.
The amount of capital employed may be computed by using anyone of the following ways.

1. Capital Employed = [Net Fixed Assets + Working Capital]


= [Net Fixed Assets + (Current Assets - Current Liabilities)]
2. Capital Employed = [Share Capital (i.e., both Equity Share Capital and Preference Share
Capital) + Reserves and Surplus + Long-term Loan] - [Miscellaneous
Expenditure appearing on the assets side of the Balance Sheet + Non-
operating Assets]

Du Pont Analysis

As stated earlier, the Return on Investment Ratio is influenced by two determinants which
in turn are influenced by a large number of determinants. For example. one of the factors which
determines the return on investment is the amount of profit. This profit is influenced by a number
of factors. Further, it may be noted that the amount of sales revenue which is one of the
determinants of profit, is influenced by two important factors, viz., selling price and sales volume
which in turn are influenced by a large number of internal and external or controllable and non-
controllable factors. This analysis brings the point to the fore that the Return on Investment is
influenced by a very large number of factors and it is very difficult to list out all these factors.
Accounting Ratios: 99

Anyhow, a summary of the determinants is presented below in the chart and this chart is called
Du Pont Chart as this was applied for the first time by Du Pont Company of the United States of
America.
The Du Pont Chart

Net Profit to Sales to Capital


Sales Ratio = Employed Ratio =

[Net Sales Revenue] [Capital Employed]

Net Sales Revenue +/-


Surplus from, or deficit of, - [Total Cost of Sales] [Fixed Assets] +
*~
[Working Capital]
Non-operating Activities
I
~ +
Manufacturing Operating Cash, Bank, Bills Bank Overdraft,
Cost of Goods Receivable, Debtors, Creditors, Bills
Sold
+ Expenses
Inventories and other
-
Payable and other
Current Assets Current Liabilities

(Cost of Materialsl + rConversion Costs]


l Consumed j - ~

Direct Expenses and


Labour Cost + Manufacturing
Overhead Expenses

Return on Shareholders' Equity

This is also called Return on Net Worth Ratio or Return on Proprietors' Fund. This
is an important ratio as it shows the amount of profit available to the shareholders which
determines the rate of dividend. Of course, in case of preference share capital, the rate of
dividend is fixed and the preference shareholders will have a priority. The equity shareholders are
eligible to get dividend in the residual income. On the basis of the above analysis, two ratios
Management Accounting: 100

(showing Return on Equity) can be computed depending upon whether the equity comprises of all
shareholders' equity or only that of equity shareholders.

:r
Return on otal . } =
Net Profit after Tax and Interest
but before any Dividend x 100
J
Shareholders EqUity [ Total Shareholders' Equity

Total Shareholders' Equity comprises of preference share capital, equity share capital and
reserves and surplus. If there is any accumulated loss, it should be deducted from the aggregate of
these three items to arrive at the shareholders' equity. This ratio sheds light on how profitably the
company has employed and utilized the funds contributed by the shareholders.
Net Profit after Taxes, Interest
Return on Equity _ and Preference Dividend
Shareholders' EqUity} - [ Equity Shareholders' Equity

In the numerator, only the amount of profit available to the equity shareholders is
considered. And in the denominator, the aggregate of equity share capital, and reserves and
surplus less accumulated loss, if any is considered.
Illustration: 3.7
From the following, calculate the missing figures and the rate of return on total assets.
Also comment on the Rate of Return.

Company S Company T
Total assets (Rs. lakh) 3
Sales revenue earned during the year (Rs. lakh) 21
Gross profit margin 12% Rs. 1.68 lakh
Net profit on sales (%) 5 6
Turnover of total assets (times) 6 6

Solution:

1. Sales Revenue of Company S:


Turnover Of} (Sales Revenue'l . _ [Sales Revenue]
Total Assets - l
Total Assets ) .. 6 - Rs. 3 lakh

:. Sales Revenue = [6 x Rs. 3 lakh] = Rs. 18 lakh.


Accounting Ratios: 101

2. Total Assets of Company T: The above 'Turnover of Total Assets' Ratio formula is used here
also.

6- [RS. 21lakh
- Total Assets
J
[6 x Total Assets] = Rs. 21 lakh
:. Total Assets = [Rs. 21 lakh + 6J = Rs. 3.5 lakh

3. Gross Profit Percentage of Company T:

%age of } _
Gross Profit - lr Gross Profit x 100J
Sales Revenue
= RS. 1.68 lakh 100J = 801.
[ Rs. 21 lakh x -/0

4. Gross Profit Margin of Company S:

12 = rl Gross Profit
Rs. 18 lakh
x lOJ
)
't [12 x Rs. 18 lakh
:. Gross Pro f1 = 100)
I
Therefore, Gross Profit = Rs. 2.161akh

5. Net Profit of Company:

S = 5% of Sales Revenue Rs. 18 lakh = Rs. 0.9 lakh


T =6% of Sales Revenue Rs. 21 lakh = Rs. 1.26 lakh

6. Return on Total Assets of Company:


Rs.90,000
Rs. 3,00,000 x 100 J= 30%
x 100 J= 36%
Rs. 1,26,000
Rs. 3,50,000
Comments:
1. Turnover of Total Assets: Both the companies have turned over their assets to the extent of
six times during the year. In this regard, one can conclude that both the companies have used
their assets equally effectively, productively and profitably during the period.
2. Profitability:
Company S Company T
Gross Profit Ratio 12% 8%
Net Profit Ratio 5% 6%
Return on Total Assets 30% 36%
Management Accounting : 102

No doubt, the Gross Profit Ratio of S is higher than that of T; but Company T has
achieved a splendid economies in the operating expenses. It may also have managed the non-
trading activities more profitably. Consequently, it has been able to achieve a 6% Net Profit
to Sales Ratio. On the contrary, the high Gross Profit Ratio of Company S has been wiped
out by the heavy operating expenses resulting in low Net Profit to Sales Ratio of only 5%. As
a result, the overall profitability of Company T is higher than that of Company S from the
view point of rate of return on total assets.

Earning per Share

Since the Earning per Share is one of the determinants of dividend, it is necessary to
compute the Earning per Share. Hence, Earning per Share is determined by dividing the equity
earnings (i.e., earnings available for distribution among the equity shareholders which may be
computed by deducting preference dividend from net profit after interest and tax) by the number
of equity shares. Therefore,

Earning }
Per =
~ Net Profit after Interest,
Tax and Preference Dividend
J
Share Number of Equity Shares

Liquidity (Short-Term Solvency) Ratios

The profitability of an organization may be satisfactory and may be having a high rate of
return on the capital employed but these do not vouch the soundness of the financial position or
condition of the organization. In order to evaluate the financial health or soundness of an
organization, one has to use a number of ratios which are normally calculated with the help of the
data extracted from the Balance Sheet. Business undertakings usually wish to keep adequate
funds to meet their short-term financial obligations as well as to ensure that the day-to-day
business operations are not hampered due to th~ paucity of funds. At the same time, the
organizations should also realize that a very high degree of liquidity is also not desirable.
Because, the idle assets do not earn anything. It is, therefore, imperative to strike a right balance
between liquidity and lack of liquidity.

Liquidity here refers to the ability of the organization to generate cash internally from
business operations or to raise cash externally from the financial institutions so that it can meet all
its cash requirements and discharge all its current obligations. It is not an exaggeration but a fact
that liquidity is very essential for the very survival of the organization. Hence, there is a need for
evaluating the liquidity position to find out whether the company is capable of paying all its
current obligations. The ratios which are commonly used for this purpose are (a) Current Ratio,
(b) Liquid Ratio, and (c) Absolute Liquidity Ratio.
Accounting Ratios: 103

Current Ratio

The ability of a company to meet its short-term commitment is normally assessed by


comparing Current Assets with Current Liabilities. As the Working Capital is equivalent to the
difference between Current Assets and Current Liabilities, or as the Working Capital is the excess
of Current Assets over Current Liabilities, this Ratio is also called Working Capital Ratio. The
Current Ratio establishes the relationship between the Current Assets and the Current Liabilities.

· = [current Assets
Curren t Rat10
Current Liabilitie
J
Current Assets normally mean assets convertible and meant to be converted into cash
within a year's time. Current Assets usually include:
Cash in hand, Cash at Bank,
Debtors (net of Provision for Bills Receivable,
Bad and Doubtful Debts),
Prepaid Expenses,
Inventories (Raw-materials, Work-in-
progress and Finished Goods),
Marketable Securities, and Other Short-term High Quality
Investments

The following points are worth noting in this regard.


1. Debtors and Bills Receivable are also called Accounts Receivable and Notes Receivable
respectively;
2. Only those easily Marketable Securities which are meant for cash sales are required to be
considered as Current Assets; and
3. Prepaid Expenses are reckoned as Current Assets as they represent the payments because
of which the company need not pay in future on that score.

It may be noted here that the constituents of Current Assets are not restncted to the items
given above but it may also include some other items depending upon the purpose of their
acquisition and use. For example, furniture. If furniture is acquired by a firm dealing in purchase
and sale of furnishing materials, it forms a part of Current Assets. On the other hand, if it is
acquired by a hotel, the same is treated as a Non-current Asset.
Current Liabilities represent the liabilities which fall due for payment within a year.
Current Liabilities usually include:
Management Accounting: 104

Sundry Creditors (Accounts Payable), Bills Payable (Notes Payable),


Outstanding and Accrued Expenses, Income-tax Payable,
Unpaid or Unclaimed Dividends Part of Long-term Debt maturing during the
(but not Proposed Dividends), current year, and
Bank Overdraft
As a conventional rule or as a general rule of thumb, a Current Ratio of 2 : 1 is reckoned
as satisfactory. That means, it is necessary or desirable to have R:-.. 2 worth Current Assets for
every Re. 1 of Current Liabilities. This criterion of 100% margin of Current Assets over Current
Liabilities has generally been accepted as a necessity due to the following reasons.
1. It may be remembered here that the Current Liabilities are fixed in nature. But the
Current Assets are subject to some fluctuation or shrinkage in value due to the unexpected
or unforeseen fall in the market value of securities, and also due to inventories becoming
obsolete not having either the use value or the saleable value, bad debts, etc.
Consequently, the realizable value of Current Assets mayor may not be equal to their
book value. It is, therefore, necessary to keep more Current Assets so that the company
can meet its current obligations without any difficulty even when the realizable or market
value of Current Assets declines.
2. The aspect discussed above forces us to conclude further that the standard 2 : 1 Current
Ratio should not blindly be followed. In some cases, even with higher Current Ratio (i.e.,
Current Assets exceeding 200% of CUlTent Liabilities), the company may find it very
difficult to meet its current commitments. For example, the Current Assets may comprise
of a very high inventory (say, 80%) which is obsolete and therefore, full value cannot be
realized. Consequently, Current Liabilities cannot be met fully. Further, there may not be
any fund available for continuing the business operation.
3. Another reason behind fixing 2 : 1 Current Ratio is the fact that the Current Assets must
be sufficient enough to meet short-term commitments and also to ensure that the normal
business operations are not hampered due to the paucity of Working Capital after meeting
its current obligations.
Though the Current Ratio of 2 : 1 is reckoned as desirable and ideal in normal business
conditions, the actual ratio may be either equal to or more or less than 2 : 1. A company with a
comparatively higher Current Ratio will be capable of paying out the short-term liabilities as and
when they become due. An improvement in the ratio may be due to the improved financial
position and it may, therefore, imply improt'ement in the ability of the company to discharge its
obligations. At the same time, it may also be due to the piling up of slow-moving and non-
moving raw-material, finished goods, etc., and this will not improve the financial position. On the
other hand, a lower ratio indicates that the company may not be able to meet its obligations.
Consequently, a declining Current Ratio may indicate a deteriorating financial position of the
company. It (i.e., decline in the ratio) may also be due to the disposing-off of obsolete
inventories.
As the adequacy of Current Ratio relies heavily on the composition and quality of the
Current Assets, the composition of Current Assets should not be overlooked. A company with a
Accounting Ratios: 105

very high percentage of Current Assets in the form of cash is more liquid than the one with a very
high percentage of Current Assets in the form of slow and non-moving inventories, th6ugh both
are having the same Current Ratio. It is because of this reason that though all the Current
Assets are liquid, some (Current Assets) are more liquid. Cash is the most liquid asset
followed by receivables. Therefore, while interpreting the Current Ratio, it is necessary to look
into the composition of Current Assets.

From the above analysis, it is apparent that the Current Ratio is a very useful ratio in
assessing the liquidity position of the undertakings. It is also obvious from the above that it is
very difficult to interpret the result of the ratio. It is, therefore, necessary to have a
comprehensive look at the result before drawing any conclusion as to what might be the reasons
for the change and what this change indicates.

Quick Ratio

As identified earlier, inventory is also considered for the purpose of computing the
Current Ratio. Of the different Current Assets, inventory IS the least liquid asset. That means.
inventories require more time to become the most liquid asset (viz., cash). Further, the value of
inventories is subject to wide tluctuations. Besides, there IS every possibility of a lot of inventory
or a type of inventory becoming obsolete or losing market and therefore, there may not be any
scope for realizing anything. It may be noted that the Current Assets may comprise of this type of
inventories and therefore, it may not be possible to use this asset to discharge the liability. Since
inventories are also considered for computing the Current Ratio, the Current Ratio may give a
wrong picture about the short-term solvency of the company. The Current Ratio, therefore, fails
to serve as a realistic index of the short-term solvency or liquidity of the firm.

The Quick Ratio which is also known as. Liquid Ratio or Acid-test Ratio or Near
Money Ratio is, therefore, used as a complementary ratio to the Current Ratio. The ratio is
concerned with the establishment of relationship between the LiqUid Assets and the Liquid
Liabilities. Liquid Assets are those assds which can immediately. or at a short notice, be
converted into cash without loss or diminution in value. The Liquid Assets usuatly include all the
Current Assets except inventories and pre-paid expenses. Liquid Assets = rCurrent Assets] -
[Inventory + Prepaid Expenses]. The reasons as to why inventories are to be excluded for
computing Quick Ratio have already been stated earlier. In addition to inventories, pre-paid
expenses are also excluded from Quick Assets as they (i.e., pre-paid expenses) cannot. normally,
be converted into cash. Liquid Liabilities usually refer to Current Liabilities less Bank
Overdraft. Liquid Liabilities = [Current LiabilIties - Bank Overdraft]. The bank overdraft i<,
exCluded-as the overdraft tends to become some sort ot a permanent or regular mode of financing.
Thus, the Quick Ratio is calculated by using the following funnuia.

. k R' . -
Q UlC atlo -
~QQuick
UIC
or Liquid Assets
. k L' 'd L' b'l' .
or IqUi la I Itles
J
Management Accounting: 106

Sometimes, the Quick Ratio is also calculated by using Current Liabilities In the
denominator instead of Liquid Liabilities as presented below.

. .
QUick RatIo = ~C urrent
Quick Assets
L'labT .
I ItIes
J
Quick Ratio is designed to assess how well an organization is capable of meeting its
obligations without having to wait for much time to liquidate its assets. In normal business
conditions, a Quick Ratio of 1 : 1 is reckoned as satisfactory and therefore, an organization with
Quick Assets equivalent to 100% of its Quick Liabilities is said to be in a fairly good current
financial position.

While interpreting the result of an organization from the view point of this ratio, one has
to take all precautions which he has to take while interpreting the result of Current Ratio.
Because, a 1 : 1 or more of Quick Ratio does not necessarily mean or communicate sound
liquidity position or adequate Liquid Assets. At the same time, a Quick Ratio of less-than 1 : 1
does not necessarily imply poor liquidity position. Because. the conclusion is to be drawn after a
careful analysis of the composition of Quick Assets. In order to get an idea about the short-term
solvency position of an organization, one ha~ to use both the Current and Quick Ratios, and they
should be compared with the standard and also wIth the average ror the industry.

Absolute Liquidity Ratio (Super-Quick Ratio)

This ratio establishes the relationship between the Absolute Liquid Assets and Liquid
Liabilities. Both the debtors and the bills receivable are excluded as there is always an
uncertainty with respect to their realization. The Absolute Liquidity Ratio establishes the
relationship between the sum of cash and marketable securities on the one hand, and the total of
quick liabilities on the other.

. .. RatIO
Absolute LIqUidIty . = [caSh + Marketable .Securities
-----.---;-~.-----.-.-------
LIqUId LIabIlItIes
J
Illustration: 3.8

The details pertaining to MKM Company for the year 2005 are:
Current Ratio = 2_5 : 1: Liquid Ratio = 1.25 : 1; Inventory as on December 31. 2005 = Rs. 21
lakh; and each of bank overdraft and pre-paid expenses is equal to zero. Using these details, find
out the current liabilities of MKM Company as on December 31,2005.

Solution:

Current Ratio = [
Current Assets
Current Liabilities
J :.2.5 =
Current Assets
[ Current Liabilities
'I
J
Accounting Ratios: 107

Therefore, 2.5 Current Liabilities = Current Assets

. . Ratio
LiqUId . = [LiqUid . . . Asset~...
Current Ltabihtles*
J :.1.25 = ~ Liquid Assets
Current Liabilities
J
In this problem, Current Liabilities =Quick Liabilities (because, Bank Overdraft =0)

Therefore, 1.25 CUrrent Liabilities = Liquid Assets

That means, 2.50 Current Liabilities = Current Assets, and


1.25 Current Liabilities = Liquid Assets
Subtracting, l.25 Current Liabilities = Inventory

Because, Pre-paid expenses = 0, and


Current Assets - Liquid Assets =Inventory

. ...
:. Current LiabilIties = l
rInventOryJ
1.25 = [1.25
rRS. 21,00,000 ]
=Rs. 16,80,000
Activity or Turnover Ratios
Another important dimension of liquidity or the short-term financial position is the
computation of the rates at which different short-term assets are converted into cash and how
promptly the liabilities have been discharged. The important ratios used for this purpose are
Stock Turnover Ratio, Debtors' Turnover Ratio, and Creditors' Turnover Ratio. Other Turnover
Ratios are Fixed Assets Turnover Ratio and Working Capital Turnover Ratio.
Inventory Turnover Ratio
Inventory Turnover Ratio which is also called Stock Turnover Ratio or Stock Velocity
establishes the relationship between the Cost of Goods Sold· during a given P!!riod and the
Average of the Costs of Opening and Closing Stocks. As the computational procedure of the ratio
differs from trading concerns to manufacturing concerns, from seasonal industries to other
industries, it is necessary, as far as possible, to deal with it exhaustively and separately.
1. Trading Concerns
Trading concerns are those concerns which are engaged in the sale of goods and services
purchased from other manufacturers. As a result, there are no raw-materials and work-in-
progress. The entire inventory held by them (Le., companies) at any point of time comprises of
only the finished goods. In this situation, the Inventory Turnover Ratio can be calc\.!lated as
shown below.

.
Inventory Turnover Ratio = l
r Cost of Goods SOldl
Average Inventory J
Management Accounting: 108

Where,
Opening . Direct!
Cost of Goods Sold = Stock . + Purchases + Expense~ _ (ClOSing]
Stock
[

A verage· Inventory (
= openi~lg Inventory; Closing Inventory].

In case, the information about the Cost of Goods Sold is neither available nor can it be
computed with the available information, goods sold at selling price may be used. Hence,

r Sales R~venue !
Inventory Turnover Ratio = l Average Inventory j
2. Manufacturing Concerns
Manufacturing concerns acquire raw-materials and use them for the purpose of producing
goods and services which are finally sold to customers. As a result, the inventory of a
manufacturing company comprises of not only the finished goods but also the raw-materials and
the work-in-progress. It is, therefore. useful [0 hreak-up the Inventory Turnover Ratio into its
main constituent parts so that light may be thrown on the level of efficiency or otherwise at its
various points. The ratios which can be used are presc..:nted below.

.
In.ventory Turnover} _
RatiO (Raw-material) -
[ Cost of Raw-materials
Consumed during the year
Average Stock of
J
. _ . Raw-materials

Where.-

Co~t of Raw-mate;ials} = Purchase of Raw~


consumed during the
Period
l
rOpening Stock of
Raw-materials +
materials during
the Period
-l
rCIOsing Stock of!
Raw-materials J
Opening Stock of
Raw-m~terials
+
Closing Stock of
Raw-materials
I:J
A verage' Stock of } =
Raw-materials [
2

The Ratio indicates as to how fast the raw-materials have been consumed for production.
Further, it is possible to find out whether the inventory cOinprises of obsolete stock of raw-
materials or fast moving raw-materials. A proper understanding of the pattern of consumption of
raw-materials can be had by ascertaining the following.
Accounting Ratios: 109

1. The average cost of raw-materials con';umed dady;


2. The number of days for which the organizatIOn can continue Its production activitlc~ with
the help of the raw-materials held by it; and
3. The average number of days for which raw-materials have been kept 111 the stores
department before they are issued to the production departments.
The following formulae are used to get an idea about the above aspects

Rupee Volume of a Day's }


Consumption of Raw-materials
(i.e., the Average Cost of =
[ Cost of Raw-materials
Consumed dUring the year
J
Daily Material Consumption) 365 days

Days' Consumption of L Stock of Raw-materials


Stock of Raw-materialj"" = [
Rupee Volume of a Day's
rnn<;llmnrinn J
365 days
A verage Material HOlding}
(i.e., Turnover Period) = [
Raw-material Turnover
Ratio J
Inventory (Ra~-materiaIS)} 365 clays ]
= [ A verage Material Holding
Turnover Ratio

For the purpose of computlllg Inventory Turnover Ratio for Work-in-progress and
Finished Goods. the following formulae are used.

Inventory Turnover Rati} _


(Work-in-progress)
r
Cost of Completed Works
- l"Average work-in-progres~
I
Where,
Opening Stock of work-in-progress]
+ Manufacturing Cost incurred
Cost of Completed Work =[ during the Period) - (Closing
Stock of Work-in-progress)

Average } _
Opening Stock of + Closing Stock of J
_W_o_rk_-_i_n...!-p_r_o.:::.g_re_s_s____W_o_r_k_-i_n--'-p'--r_o..::g,-re_s_s
Work-in-progress - [ 2
Management Accounting: 110

Inventory Turnover Ratiol [ Cost of Goods Sold l


(Finished Goods) f - LA verage Stock of Finished Goodj

Where,
Cost of Goods Sold = [ (Opening Stock of Finished Goods +
Manufacturing Cost of Goods Produced
during the Period) - (Closing Stock of
J
Finished Goods)

Cost of Opening Cost of Closing


Stock of Stock of
Average Stock Of} _ Finished Goods + Finished Goods
Finished Goods -
2

In the case of seasonal industries (e.g., Sugar Industry), computation of Average


Inventory using the above formulae may fail to give the correct figures as both the opening and'
the closing stocks happen to be nominal and therefore, do not represent the usual stocks held
throughout the year. In this type of situation, Average Inventory may be computed by dividing
the 'aggregate of inventory at the beginning of the year and inventory at the end of every
month' by 13.
The Inventory Turnover Ratio (particularly, Finished Goods) indicates the velocity with
which the goods move. A low ratio, under normal business conditions, is an indicator of the
existence of slow moving, obsolete and shop-soiled goods of either low value or poor quality
goods which the company is unable to push through. A low ratio may, therefore, be an indicator
of dull business. On the other hand, a higher Inventory Turnover Ratio may be a reflector of good
management.
Illustration: 3.9
The summary of the Profit and Loss Account of Kala Company for the year ended March
31,2005 is given below.
Partic!llars (Rs.OOO) Particulars (Rs.OOO)
To Opening stock of: By Sales 1,200
Raw-materials 40 By Closing stock:
Finished goods 60 Raw-materials 80
To Purchases (materials) 500 Finished goods 120
To Manufacturing wages 400
To Manufacturing expenses 200
To Gross profit (balance) 200
1,400 1,400
Accounting Ratios: 111

From the above, you are required to calcubte (a) Raw-materials Turnover Ratio; (b) Rupee
Volume of a Day's Con~umption of Raw-material,: (c) Day ... · Consumption of Stock of Raw-
matelials; (d) Average Material Holding:: and (e) Inwntury Turnover Ratio (Finished Goods).
Solution:

1. Raw-materials} = k:ost of Raw-matenab consumed]


Turnover Ratio lAverage Stock. of Raw-materials

_ [RS. 40,0000 + Rs. 5,00,000 -


- , (RS. 40,000 ; R~. 80,000
80,000 R~.
J
l_ rRs. 4,60.00<.21_ 7 ')/3 .
,./ - LRs, 60,0()O ) - - times

2. Rupee Volume of a Day'~ _ L Cost lli Raw-


materials Con~lImed
J
Consumption of Raw-material~ -
e 365 days

= (RS.4,60,OOOI
365 days j _- R S.,~.
I ""6027 wort h raw-matena
. Is per d ay

3. Days' Consumption of }
Stock of Raw-materials
= ltock uf Raw-materials
Rupee Volume of a
J
Day's Consumption

-lrRs.
Rs. 60,000 J =
1,260.27
47.61 days

4. Average Inventory} - ~ 365 days J_ 65 days] = 47.61 days


Holding - Raw-materials
Turnover Ratio
- C7 2/ 3

5. Inventory Turnover
Ratio (Finished Goods)
}
=
[ Cost of Sales
Average Inventory
J
[RS' 60,000 + (Rs. 4,60,000 + Rs. 4,00,000 + Rs. 2'00'000)J
= - [Rs. 1,20,000]
[ (RS. 60,000 \RS. 1,20,000 J
= r Rs . 60,000 + Rs. 10,60,000 - Rs. 1,20,0091 = (RS. 10,00,000 J
l [ Rs. 1,~0,000 J J Rs. 90,000

= 11 1/9 times
Management Accounting: 112

Debtors' Turnover Ratio

One of the most commonly used methods of sales by the business organizations is the
credit sales wherein the companies sell their goods and service" to the customers on credit basis in
accordance with the credit policy formulated by them. The credit sales are recorcled in the books
of the selling company as Debtors. The Debtors are called Book Debts in Accounting language.
Further, goods are also sold to the customers who agree to give their acceptance for the bills in
consideration of sales payments. The bills so received from the customers are called Bills
Receivable or Notes Receivable. For the purpose of computing Debtors' Turnover Ratio, the
aggregate of Accounts Receivable and Bills Receivable is considered. It may be remembered
here that both the Accounts Receivable and the Bills Receivable are the items of Liquid Assets
which form a major part of Current Assets. The ability of an organization to meet Its current
obligations is intluenced by the ability with which it collects the amount of receivables. If the
organization fails to collect Debtors and Bilb Receivable a~ per ih LTedil policy, II ha~ no other go
but to make arrangements for short-term loan~ for meetll1g its current obligations. It IS, therefore,
necessary to establish the relationship between 'the aggregate of Debtors and Bills Receivable'
and 'sales'.

Debtors' Turnover Ratio, also known as Receivables Turnover Ratio or Debtors'


Velocity establishes the relationship between the net credit sales of the year and the average
receivables. The Ratio, therefore, indicates how well receivables are turning into cash. To put it
differently, it shows the number of times the receivables have rotated during the year in terms of
sales.

R D~bt?rs' Turn?ver
aho (I.e., ReceIvables
Turnover Ratio)
1 =
[Net Credit Sales during the year]
A verage Trade Debtors

In order to know the rate at which cash is generated by the turnover of receivables, the
Debtors' Turnover Ratio is to be supplemented by another ratio. viz., Average Collection Period.
Average Collection Period is computed by dividing the Average Trade Debtors (i.e., Average
Receivables) by the Daily Average Credit Sales as shown below. This ratio states unambiguously
the number of days' average credit sales tied lip in the amount owed by the buyers.

Average CoIlectioIiL = ( A verage Trade Debtors I OR


Period f ~aily Average Credit Salesj

A verage Collectio~ = A verage Trade Debtors or Receivables x 365J OR


Period f [ Net Credit Sales for the Year

A verage Collectio~ = (Number of Working Days in the year]


Period f l
Debtors' Turnover Ratio OR
Accounting Ratios. 113

A verage CollectiOI~
Period f
= [Number of WorkIng Day~
Net Credit Sale~
x Averaue
Trade
I
Debtorj
during the Year

[n this regard, the following points should be kept in mind.

Opening ClOSing]
(a) A verage Trade} Debtors + Debtors
Debtors = [ 2

I. 'Debtors' means the aggregate of both the Accounts Receivable and the Bills Receivable;
2. In the absence of opening balance of Debtors, only closing balance is used without
dividing it by 2; and
3. Debtors which do not arise from regular sales should be excluded. For example, a bill
receivable from the buyer of fixed assets.
(b) Net Credit Sales = [Sales Revenue - Sale" Return,; - Cash Sale" 1
If break-up of sales revenue Into cash sale., revenue and credit sales revenue is not
available. as in majority of the case.,. the total ;,ale" will usually be as,umed to be on
credit basis.

(c) Daily AVerage} _ [Net Credit Sales Revenue]


Credit Sales - 365 days

Amount of credit sales is to be divided by the same number of days for which sales
figures are considered.

A high Average Collection Period compared to the average period of credit extended by
the firm indicates the ineffective collection system and therefore. the need for proper inquiry so
that debt collection may be improved. In the ~ame way. a low Average CollectIOn Period will
imply the shorter time-lag between credit sales and cash collection.

In order to assess the performance of the company from the view point of debt collection.
it is necessary initially to compare the A verage Collection Period with the credit period offered or
extended by the company. Then. the company's Average Collection Period may be compared
with the industry's average.

Illustration: 3.10

From the following. compute the Debtor,' Turnover Ratio and the Average Collection
Period.
Management Accounting : 114

Sales revenue Rs. 25,00,000


Cash sales 3,00,000
Sales returns 2,00,000
Debtors as on December 31, 2004 8,00,000
2005 10,00,000
Bills receivable as on December 31. 2004 5,00,000
2005 7,00,000

Solution:

1. Net Credit Sales for the year 2005:

Sales Revenue earned during the year Rs. 25,00,000


Less: Sales Return~ 2,OU,(}OO

Cash Sales 3,00.000 5,00,000


:. , Net Credit Sales 20,00,000

Opening balance of Closing balance of


Debtors and Bills + Debtors and Bills
2. AVerage} Receivable Receivable
Trade =
Debtors 2

~
RS' 8 1akh + Rs. 5 lakh) + (Rs. 10 lakh + Rs. 7 lakh)j
= = Rs. 15,00,000
2

3. Daily Average Credit Sales:

= [credit Sales Revenue] =


365 days
rRs. 20,00,0001
l 365 days J =Rs. 5'479
(a) Debtors' Turnover}
Ratio
= r Net Credit Sales l _
rRs. 20,00,000l _ 1 1/3 times
tverage Trade Debtor~ -lRs. 15,00,000j -

(b) Average Collectiofil


Period S
= r Average Trade Debtors J-
(Rs. 15,00,000J - 273 77 d
l?aily Average Credit Sales - [ Rs.5,479 - . ays
OR
Accounting Ratios. 115

Average CollectionL
Period S
_
- [
Average Trade Debtors x 365J =
Net Credit Sales
r~RS.
15,00,000 x 365
lRs.2(),00,000 )
1
= 273.75 days OR

Average CollectionL _ [Number of Working Day~


Period S - Debtors' Turnover Ratioj = [ 365 days
11/3 times
J
=273.75days

Creditors' Turnover Ratio


It is also a common practice pursued by the organizations to effect the purchases on credit
basis. The credit purchases are recorded in the accounts of the buying-companies as Creditors
(in America, Accounts Payable). Sometimes, the buying-companies accept the bills or hundies
in consideration of purchase price. In the books of accounts of the buying-companies, such bills
or hundies are termed as Bills Payable (Notes Payable in America). Both the Creditors and the
Bills Payable are considered for the purpose of computing the Creditors' Turnover Ratio and also
the Average Credit Period enjoyed by the purchasing-company. This Ratio, also known as
Payables Turnover Ratio establishes the relationship between the net credit purchases and the
average trade creditors. Therefore,

Creditors' or payables} Net Credit Purchases l


Turnover Ratio = [ Average Trade Creditor~

This Ratio is supported by another ratio viz., Average Payment Period which is
calculated by dividing the Net Average Trade Creditors by the Average Net Credit Purchases per
Day. The result is the Average Payment Period in days. The Ratio indicates the promptness or
otherwise with which the payment is made to the suppliers in respect of credit purchases.
Therefore,

~
:\ verage Payment } = verage Trade creditors]
Period Daily Average Net Credit
Purchases

The Average Payment Period may also be computed by using the following formulae.

Average Payment }
Period
= r
l
Average Trade Creditors
Net Credit Purchases x
365]

Average Payment } _ [Number of Working Days]-


Period - Creditors' Turnover Ratio

While computing the above two ratios, the following points should be kept in mind.
Management Accounting: 116

Opening balance Closing balance]


(a) AVerage} of Creditors + of Creditors
Trade = [ 2
Creditors·

I. 'Creditors' means the aggregate of both the Accounts Payable and the Bills Payable;
2. In the absence of opening balance, only the closing balance i~ used without dividing it by
2:
3. Creditors which do not <In,,e from regular purcha..,e-; -;hould be excluded. For example, a
bill payable LO the seller of fixed a~set".

(b) Daily Average Net}


Credit Purchases .
= [Total Purchases - Purchase Returns - Cash Purchases
365 days
J
The amount of credit purchases should be divided by the same number of days for which
'purchase figures are considered.

(c) Net credit}


Purchase
(Total Purchase
= of Merchandise
Purchase
Returns
Cash
Purchases
J
The A veragc Payment Period may also be expressed in weeks and months. In that case,
52 weeks or 12 months is to be used in place of 365 days.
An unusual delay in payment will be reflected in the form of high Average Payment
Period and it affects the credit reputation of the firm adversely and therefore, the suppliers may be
reluctant to grant credit in future. A low ratio reflects the prompt payment to the suppliers
enhancing the reputation of the firm. On the other hand, a high ratio ..,igl11fie~ the delay in the
payment to the suppliers.
Illustration: 3.11
With the help of the informatIOn pre~ented below. you are required to calculate the
Creditors' Turnover Ratio and the Average Payment Period.
Tolal Purchase (including
Cash Purchases of Rs. 5 lakh and
Purchase Returns of Rs. 2 lakh) Rs. 22 lakh
Creditors as 011 December 31, 2004 06
2005 08
Bilb payable as on December 31. 2004 04
2005 07
Accounting Ratios: 117

Solution:

1. Average}
Opening balance
of Creditors +
Closing balance
of Creditors
J
Trade =[
Creditors 2

~ rRs. 61akh + Rs. 41akh): (Rs. 81akh + Rs. 71akh~ ~s. 251akhl
l j = l 2 j = Rs. 12,50,000

2. Total Credit Purchases and Daily Average Purchases:

Total Purchases Rs. 22,00,000


Less: Cash purchases Rs. 5,00,000
Purchase Returns 2,00,000 7,00,000
:. Credit Purchase 15,00,000

Daily AVerage} _ ,Net Credit purChases] = .[RS. 15,00,000J = Rs. 4,110


Credit Purchase -l
365 days 365 days

Therefore,

(a) Credi tors' } Net Credit Purchases J =


rlAverage [RS. 15,00,000J
Turnover = Trade Creditors Rs. 12,50,000 = 1.2 times
Ratio

(b) AVerage}
Payment
Average
Trade Creditors
=[ Daily Average
J RS. 12,50,000J
= [ .Rs. 4,110 = 304.14 days OR
Period Credit Purchases

A verage payment}
Period
= [Average Trade Creditors 6
Net Credit Purchases x 3 5
J

=[
Rs. 12,50,000 l
Rs. 15,00.000 x 365j.= 304.17 days OR

Average Payment }
Period
_ [Number of Working Days
- Creditors' Turnover Ratio
J--
[365 days
1.2 times
J= 304.17 days
Management Accounting : 118

Fixed Assets Turnover Ratio


As the organization employs capital on fixed assets for the purpose of equipping itself
with the required manufacturing facilities to produce goods and services which are saleable to the
customers to earn revenue, it is necessary to measure the degree of success achieved in this
bearing. This Ratio establishes the relationship between the amount of sales revenue and the
amount of capital employed on fixed assets.

Fixed Asset~J _ rNet ~ales Revenue~


Turnover Rah - l
FIxed Assets j

Though there is no standard norm for this ratio, a very high ratio indicates that the
organization is overtrading on its fixed assets. In the same way, a low ratio hints that the
organization has excessive investment on fixed assets.
Total Assets Turnover Ratio
This Ratio is calculated by dividing the amount of sales revenue by the amount of capital
employed on different types of assets.

Total Assets
Turnover Rati~
L [Net Sales
=Total Assets
J
As in the case of Fixed Assets Turnover Ratio, a high (Total Assets Turnover) Ratio
indicates the overtrading on assets, and a low Ratio signifies the excessive investment. The
standard acceptable norm for this Ratio is normally 2 times. It may be noted here that some
experts use cost of sales in place of net sales for the purpose of computing the above two ratios.
Working Capital Turnover ratio
This Ratio measures the efficiency with which the Working Capital is utilized. It is
computed by dividing the amount of sales revenue by the amount of Working Capital.

Working Capitall _ [ Net Sales "I


Turnover Ratio f Working Capita0

A high Working Capital Turnover Ratio indicates either the favourable turnover of
inventories and receivables and/or the inadequacy of Net Working Capital accompanied by low
turnover of inventories and receivables. A low ratio signifies either the excess of Net Working
Capital or slow turnover of inventories and receivables or both.
Financial Position and Leverage Ratios (Long-Term Solvency Ratios)
Be<;ides the ratios which shed light on profitability and liquidity of business undertakings,
one can find a number of accounting ratios which shed light on the composition of capital of the
business firms. As is known, the capital of any organization comprises of both the share capital
and the debt capital. And the share capital consists of both the equity share capital and the
Accounting Ratios: 119

preference share capitaL In the same way, the debt capital includes both the debenture capital and
other loans. The policies of the company about the composition or structure of its capital,
employment of this capital and the outcome of all these in the form of profit and return are
reflected by these ratios. Because, these ratios measure the relative importance and claims of the
shareholders and lenders in the organization.
While the Liquidity and the Turnover Ratios concentrate on evaluating the short-term
debt paying ability, the Financial Position and the Leverage Ratios lay emphasis on the long-term
financial prospects. In order to assess the long-term financial soundness, it is necessary to find
out whether the organization l~ able to meet its long-term financial commitments whenever they
become due and whether the organization is able to maintain or increase the market value of its
shares. In simple, it is very much essential to find out whether the company is capable of:
1. paying back the principal amount soon after the stipulated period as per the agreement;
and
2. paying the interest on principal amount promptly and periodically as per the terms and
conditions to which the company has agreed at the time of raising capital.
On the basis of the above, the ratios which shed light on the above aspects may broadly
be classified into two categories. They are Debt-Equity Ratios and Coverage Ratios. Debt-equity
Ratio, Capital Gearing Ratio, etc., deal with the relationship between the borrowed capital and the
owners' capital. In the same way, Interest Coverage Ratio, Fixed Dividend Coverage, etc., throw
light on the company's ability to pay interest, dividend, etc.
Debt-Equity Ratio
Various types of assets are acquired and installed by the organizations with the help of
both the owners' equity (i.e., internal equity) and the outside debt (i.e., external equity). The
proportion in which both the owners and other outsiders have provided funds to acquire the assets
is an important factor to be known as it has an impact on the long-term solvency position of the
organization. The cost of debt capital usually differs from that of share capitaL Further, cost of
debt capital (e.g., interest on debenture capital. interest on loans raised from the banks, etc.,) is
fixed and agreed to by both the parties (i.e., lender and borrower) at the time of entering into the
agreement. It is compulsory and the company has to pay the interest at the agreed rate to the
lender periodically irrespective of the amount of profit earned during the year. The debt capital is
to be paid back to lenders immediately after the stipulated period. Besides, the interest charge is a
deductible one from the income for the purpose of determining the amount of taxable income. By
using debt capital, the company is able to reduce its tax liability. The companies do not enjoy
this tax benefit in the case of share capitaL
Due to these advantages, the industrial undertakings prefer debt capital to share capitaL
But the corporate undertakings are also put to some difficulties if they use high degree of debt
capital. It is due to two important reasons. One is the compulsory payment of interest
periodically and the second one is the repayment of principal amount after the stipulated period.
Therefore. excess debt capital tends to cause insolvency. It is, therefore, imperative to have an
ideal proportion between outsiders' equity and owners' equity. With the object of measuring the
firm's obligations to the outsiders in relation to the funds provided by the owners, this Ratio
establishes the relationship between the creditors' claim on assets and the owners' claim. In other
Management Accountmg : 120

words, it studies the extent to which the assets of the company are financed by the outsider ... and
the owners. Debt-Equity Ratio which sheds light on the above aspect can be calculated by
dividing the 'Debt Ratio' by the 'Equity Ratio' as shown below:

Debt-EqUity} Debt Ratio J


rli~quity
Ratio = Ratio

t Debt
Total Assets
J Debt ~ J
=
l E9uit~
Total Assets
J = [ Total Assetsj x
[rotal Assets]
Equity =
[Debt
Equity

A Note on Proprietary Ratio


One of the Ratios used to determine the Debt-Equity Ratio is the Equity Ratio which is
also known as Proprietary Ratio or Capital Ratio or Net Worth to Total Assets Ratio wherein
proprietary fund IS called net wOl·th. Tim I~ calculated In the following manner.

. . [Share caPltalj
Propnetary RatiO = T o t,a I Asse
. ts

Usually, this Ratio is calculated by considering the owners' claim or equity instead of
only the amount of capital contributed by the owners or the shareholders. This may be computed
as given under.

. .
Propnetary RatiO =
[proprietors' Equity or
T o t,a I Asse t s
Fund~

This Ratio shows the relationship between shareholders' fund and total assets. The result
clearly shows the share of owners in the total assets of the company. When the Proprietary Ratio
is subtracted from one, the resultant figure represents the share of outsiders' claim on the assets of
the company. That means, Outsiders' Claim = [1 - Proprietary Ratio].
It may be noted here that the Proprietary Ratio is also one of the variants of Debt-Equity
Ratio. The only difference between the first Debt:Equity Ratio and this form (i.e., Proprietary Ratio)
of expressing Debt-Equity Ratio is that in the earlier case the Ratio is in the form of proportion,
whereas in the latter case, it is in the form of quotient or percentage. For example, in the earlier case,
the Ratio is in the form of 2 : 3 and in the latter case, it is In the form of either 0.6 time or 60%.
The components of Debt-Equity Ratio require some explanation. Debt includes all debts
whether they are in the form of long-term debts or short-term debts or both; whether they are in
the form of mortgages or bills or debentures or both. Short-term debts are also considered for the
purpose of computing the total debt a~ they assume the character of long-term debts through the
continuity of their flow into the firm. Further, they are not cost-free forms of capital.
Accounting Ratios: 121

On the other hand, equity (i.e., the claims of the owners) comprises of equity share
capital, preference share capital, capital reserves, retained earnings less losses and fictitious assets
(such as, preliminary expenses). It should be noted here that thOll[:h the Preference Share
Capital by classification and nomenclature is a part of equIty capital, the Redeemable Cumulative
Preference Share Capital is like the debenture capital. Many experts. therefore, consider the
Redeemable Cumulative Preference Share Capital in Debt. Further, the Controller of Capital
Issues includes Redeemable Preference Shares as a part of debt if they are redeemable within a
period of 12 years. Anyhow, it is the practice pursued by the corporate sector to reckon it as
owners' equity but not as external equity or as debt.
Another variation of Debt-Equity Ratio is by establishing the relationship between the
long-term debt and the shareholders' or proprietors' equity as presented below.

. . [Lon g -Term Debt ]


Debt-EqUity RatIo = P ropne
. tors ' EqUi't y

The Debt-Equity Ratio is also calculated by using the following formula.

Debt-Equity Ratio = [ Debt*. ] (Note: * Debt includes only long-term loans)


Debt* + EqUity

The first formula (viz., Debt-Equity Ratio = Debt + Equity) is widely used by both the
industrial undertakings and the parties having stake in the undertakings. The analysis and
interpretation here is, therefore, based on this formula. The importance of Debt-Equity Ratio is
very well reflected in the words of Weston and Brigham which is reproduced here: Debt-equity
ratio indicates to what extent the firm depends upon outsiders for its existence. For the
creditors, this ratio provides a margin of safety. For the owners, it is useful to measure the
extent to which they can gain the benefits of maintaining control over the firm with a limited
investment.
The Debt-Equity Ratio states unambiguously the amount of assets provided or financed
by the outsiders for everyone rupee of assets provided by the shareholders of the company. For
example, if Debt-Equity Ratio is 4 : 5, it indicates that the creditors have financed Rs. 4 of capital
for every Rs. 9 of capital employed for the purpose of acquiring assets. At the time of liquidation.
the creditors will have a priority over all the assets which are financed more by the shareholders.
Hence, it can be said that the lower Debt-Equity RatIo means the larger amount of buffer or
protection to the outsiders as the larger amount of assets are being financed by the shareholders.
On the other hand. owners prefer a higher Debt-Equity Ratio as it provides better return and
control with small capital contribution.
Though the borrowed capital bears a fixed interest rate, many a number of industrial
undertakings prefer and rely more and more on the borrowed capital. It is because of the reason
that the interest on borrowed capital is a permissible deduction from the profit before the income-
tax liability is determined. There is, therefore, an obvious advantage in raising loans in the light
of the above point and also due to the fact that the corporate tax rates are as high as 40 to 50%.
Management Accounting: 122

At the same time, one should also keep another important aspect that a very excessiv.e
external equity tends to cause insolvency. Because, it poses many a number of financial problems
to the organization with rc"pect to both the periodical payment of interest and also repayment of
loans. It is necessary, therefore. to have a balanced composillon of both the owners' equity and
the outsiders' equity which, of course. varie-; from one sItuation to another. Many financial
institutions in India (like IDBI. IFCI) specify a norm of 2 : I ratio for financing the private sector
undertakings and a norm of 1 : 1 ratio for public sector undertakings. But what is to be noted here
is the difference in the risks between ll1dustnes and therefore. most of the ll1dustne~ are now
developing their own norms so that these norms act as guideline~ to the finm in that particular
industry.
Capital Gearing Ratio

For the purpose of computing this Ratio, total capital i~ classified into two broad
categories as funds beanng fixed interest and/or fixed dividends: and other funds not bearing any
fixed interest or fixed dividend. The term Capital Gearing is Llsed to describe the relationship
between fixed interest and/or fixed dividend bearing securities, and the equity shareholders'
funds. Capital Gearing Ratio, therefore, establishes a meaningful relationship between the fund~
bearing fixed interest and/or fixed dividends on the one hand. and the equity shareholders' funds
on the other. It can be calculated as shown below.

. .}
CapIta} C?eanng
RatIO
=
( Funds bearing Fixed
Intere-;t and/or Fixed DivIdend
J
Equity Shareholders' Funds

It is implied that the numerator of the above equation include" both the debt (as defined
for computing Debt-Equity Ratio) and the preference ~hare capnal. Equity shareholders' funds
comprise of the equity (as described for computing the Debt-Equity Ratio) less preference share
capital. If an orgal1lzation i~ having large fund.., bearIng fixed interest and/or fixed dividends as
compared to the equity shareholders' funds. the organization is said to be highly geared. On the
other hand, the organization is said to be low geared. if reverse is the case (i.e .. if the fixed
interest and/or fixed dividend bearing fund~ are low than the equity o,hm:.eholders' funds). If both
the components are equal, the organization I~ said to be evenly geared.
A Note on 'Trading on Equity' or 'Leverage'
The importance of Capital Gearing Ratio lies in its capability to indicate the additional
residual benefits accruing to the equity shareholders. This can easily be understood with the help
of the following hypothetical illustration.

Total capital employed: Equity ~hare capital R:-,. 10 crore

Loan capital (interest rate to%) Rs. 40 crore


P"ofit before interest and tax Rs. to crore
Tax ral~ 40%
Accounting Ratios: 123

If the above data are re-ananged, a clear idea about the additional surplus benefits
accruing to the equity shareholders can be obtained.

Profit before interest and tax Rs. 10.00 crore


Less: Interest on borrowed capItal ( I uric ur R~. -J.O crore) R<,. 4.00 crore
Profit before tax Rs. 6.00 crore
Less: Tax liability (40% of Rs. 6 crore) Rs. 2.40 crore
Profit available to equity shareholders R~. 3.60 crore

Therefore, Rate of Profit available t~ _ [Rs. 3.6 crore x 10~ = 36%


Equity Shareholders f - l Rs. 10 crore J
Though the organization has earned only 20% 'pI~ofit before tax' on the total capital
employed [i.e., (Rs. 10 crore + Rs. 50 crore) x 100J. the organization is able to pay as much as
36% dividend to eqUIty shareholders. From this analysis, it is obvious that in case of high gearing
(in this case, it is 4 : I), some residual benefits accrue to the equity shareholders. This is due to
the fact that, when an organization earns a certain percentage of profit on total capital employed.
it pays only a fixed return against loans and preference share capital. The residual amount (after
meeting the above including tax) goes to the equity shareholders. That means, (1) If the equity
share capital is low, and 'loan and preference share capital' are high: and (2) If the rate of return
is higher than the interest rate (and the rate of diVIdend to preference shareholders). the equity
shareholders will receive more benefits. Such a situation is called the Trading on Equity or
Leverage. Even if an organization IS having an evenly geared ratIO. there may be a Tradlllg on
Equity or Leverage depending upon the overall profitability. tax rate. and rates of interest and
dividend to preference shareholders. The Trading on Equity .or Leverage is a necessary reward
for the risk taken by the equity shareholders. But. thiS reward i~ uncertain. Further. there is an
uncertainty about the dividend and they may also have to forego their capItal.
Interest Coverage Ratio
When an organization asks for loans, the lender wishes to know whether the organization
is capable of earning adequate amount of profit to pay the interest periodically. Interest Coverage
Ratio, also known as Debt Service Ratio or Fixed Charges Cover. establishes the relationship
between the amount of net profit before deduction of intcre:--l and tax. and the fixed interest
charges.

Net Profit before


Interest Interest and Income Tax
Coverage RatiO} = [ Fixed Interest Charges
For example, Profit before interest and tax = R".5 lakh
Interest on debentures and on other borrowings = Rs. 2 lakh
Management Accounting: 124

Therefore, Interest coverage} _ [RS. 5,00,000,_ 2 5' (. 2500.)


Ratio - Rs. 2,00,000J - . tImes I.e., 10.

The above computation clearly states that even if the future 'profit before interest and tax'
of the company is 112.5th (i.e., 40%) of the current year's profit, the company would be able to
pay the interest. In other words, the Interest payment to debenture holders and other lenders is
covered 2.5 times by the net profit before interest and ta for the year. It is, therefore, implied
that higher the cover, the more secure the debenture holders and other lenders would be with
respect to their periodical interest Income.
Fixed Dividend Coverage Ratio
The preference shareholders are entitled to receive dividend only after meeting the
debenture intere~t and other fixed charges and taxation liability. This Ratio, also known as Fixed
Dividend Cover, establishes the relationship between 'the amount of net profit after interest and
tax but before dividend', and 'the amount of preference dividend.' Fixed Dividend Coverage
Ratio Indicates how secure the dividends are for the preference shareholders.

Fixed Dividendl _
Net Profit after Interest
and Tax but before Dividend
J
Coverage RatioS - [ Preference Dividend

Debt-Service Coverage Ratio


This should not be confused with Interest Coverage Ratio. Because, Interest Coverage
Ratio is used to assess whether the organization's profit is adequate to pay the interest charge
periodically say, quarterly, half-yearly, yearly, etc., depending upon the agreement. On the other
hand, the Debt Service Coverage Ratio is used to evaluate the capability of the company not only
to pay the interest but also to repay the principal loan amount on time as and when it falls due for
payment. For this purpose, the relationship is established between (a,l the amount of profit which
can be utilized to pay the interest and to repay the principal loan amount, and (b) the aggregate of
interest charge and instalment of principal loan amount. Hence, the following formula is used to
compute the Debt-Service Coverage Ratio.

Net Profit before


L= Interest and Tax
Debt Service
Coverage RatioS

Interest +
Instalment towards
Principal Repayment
J
[ 1 - Tax Rate

Instalment towards repayment of loan amount i~ adjusted for tax effect. Because, this
repayment of loan capital is not deductible from profit for the purpose of corporate income tax.
Since the book costs and amortizations like depreciation, amortization of goodwill, prelimInary
Accounting Ratios: 125

expenses, etc., do not cause the outflow of cash, some experts prefer to consider cash profit III thl:
numerator and the aggregate of interest and instalment towards repayment of principal loan
amount in the denominator as presented below.

Cash Profit available for payment


Debt Service L =
of Interest and Loan Repayment
Coverage RatioS Instalment towards
Interest + Repayment of Principal

By computing the Debt Service Coverage Ratio, it is possible to find out whether the
profit (before interest, tax, etc.,) is adequate to pay the interest and to pay the instalment towards
principal amount. Hence, a high Debt Service Coverage Ratio is desirable.
Dividend Yield Ratio
When a person acquires the shares of an organization from the capital market hy paying
the prevailing market price (say. Rs. 18 per -;hare against the face value of Rs. 10), he naturally
wishes to compute the effective rate of return he receives on his investment. Hence, the
relationship is established between (a) dividend per share, and (b) market price per share. That
means, Dividend Yield Ratio is computed by dividing the dividend per share by the market price
of an equity as shown below.

Dividend }
Yield Ratio
=[ Dividen? per Share
Market Pnce per Share
x 100 1
j
It may be noted here that the dividend per share is computed by mUltiplying the face
value per share by the rate of dividend. For example, a company's share capital comprises of 10
lakh equity shares of Rs. 10 each fully paid and it declares 30% dividend for the year just ended.
At the end of the year, the company's shares were quoted at Rs. 24 on the stock exchange. Hence,

Dividend per share = [Rs. lOx 30%] = R~. 3

~ividend.
Yield RatiO
} = [ Rs.3 x 100J= 12.YY<:
Rs.24

From the above computations, it is unequivocal that though the company has declared
30% dividend, the effective dividend earning or Yield Ratio is only 12.5%.
Price-Earning Ratio
This Ratio establishes the relationship between the market price of an equity share and the
earning per equity share as stated below.
Management Accounting: 126

price-Earnmg} _ Market Price per Equity Share


Ratio - [ Earning per Share
J
A comparison of Price-Earning Ratios of two or more companies helps to find out
whether the equity shares of a company are undervalued or not.
Advantages of Ratio Analysis
The important advantages of Accounting Ratlo~ or Ratio Analysl~ are ~ummarized below.
I. Acc6unting Ratios enable the financial an~dy-;ts to summarile and simphfy the
voluminous financial data. Since the ACCllUlltlllg Ratio~ identify the significant
accounting data relationship,>. they provide greater insighb into the company' s
performance. Because. they provide greater clarity and Insight to the data.
2. Ratio Analysis is a valuable service to the managerial per",onnel in the satisfactory
discharge of their duties and responsibility such ~s planning. controlling. etc.
3. The Trend Ratios enable the analy,;ts to· find out whether the company has been
improving its performance or not ()\ er the year'>.
4. Since the ratios are capable of identifying the factors which are responsible for the failure
of the company, Ratio Analysis is of much help to the management to take necessary
cOlTective measures to improve the results in future.
5. Accounting Ratios help the financial analysts (both the internal and the external parties)
to assess the performance of the company from the view points of profitability. liquidity,
long-term solvency. etc.
6. By establishing the standards for each ratio. the management can compare the actuals
with the standards which act as the target'>. Thi, helps the company to improve its
performance in future by rectifying the mistakes which it has committed in the past.
7. Accounting ~atios help the companie" to undertake both the intra-firm and inter-firm
comparisons so that a healthy competition prevails not only between the firms but also
between the divisions within a firm.
8. Accounting Ratios are. therefore, important tools for both minimizing costs and
maximizing revenue and profits.
9. They help to formulate policies for future mcludlI1g the capital expenditure decisions.
Limitations of Ratio Analysis
Ratio Analysis plays a stupendous role in the process of analysis and interpretation of
data in the Financial Statements. It is one of the most widely used tools for analysis and
interpretation of both the operative and the financial data. It is these accounting ratios which help
the managerial personnel to diagnose the disease. monitor the performances and plan for the
future. Though so much is the pivotal role the Accounting Ratios are playing. they are not free
from certain limitations or drawbacks. The analysis and interpretation of Financtal Statements
through Ratio Analysis require a comprehensive and an intelligent understanding of their
Accounting Ratios: 127

limitations to reap the full benefits of R<Jtio Analysis. A few but the important limitations are
identified below.
1. As is known. the Ac;counting Ratios portray the relationship between two items or groups
of items of the financial and other statements. Prior to the interpretation of the rati<is. it is
necessary to study the factors. reasons. policies. etc .. which might have int1uenced the
two figures used a~ determinants (i.e .. both the numerator and the denominator) for
calculating the ratio. In the absence of thb thorough ';tudy. the re,;ults may be
misinterpreted. For instance. unsold stock of filll'-;hed goods at the end of the accounting
period. which IS one of the determll1anh llf lll\entury Turnuver Ratio. may be due to any
one or more uf the following rea~on~.
a. It may be due to the polIcy of the orgal1lzatlon. When the orgal1lzation
anticipates considerable rIse in the demand for 1[<, product at the beginning of the
ensuing year. it may take a deci~lon to step up the production at the end of the
current year and keep the extra Ul1lh as un~old stock to meet the additional
demand: and/or
b. It may be due to the inability of the "ale~ department to sell all the unlh that the
organization produced during the current year: and/or
c. It may be due to trade reces~ion I.e .. the period during which there will be a
reduction in the demand for the producb of the entire industry including the
organization for which the ratio is calculated and interpreted.
Consequently. the task of interpreting the ratios by mere reference to the
Financial Statements. from whH:h the figure,> are extracted and used for computing
ratios. become~ very difficult.
2. FinanCial Statements are usually prepared on the basis of the histoncal or the original
cost. That means. the effect of an ince~sant change 111 prices is ignored. As a result the
figures which represent the monetary values of transactions which have taken place at
different POlllts of time are used for computing the ratios. It may be remembered here
that the purchasing power of money varies from one point of tlllle to another. It.
therefore. cal b for the re-statement uf all the lIell1~ uf Financial Statemenh in terms of
purchasing power of money or replacement Clht or current cost at a particular point of
tllne which the majority of the organiLation~ are not dOIng at pre"ent.
3. Ratios are a~ accurate as the accuracy with which the Financial Statements and other
accounts or statements are prepared. If the correct monetary values are not assigned to
the assets, liabilities and other items. it is not possible to establish the correct. realistic,
meaningful and the useful relationship between two figures or groups of figures.
4. For most of the Accounting Ratios, no standards have been established. Hence. it is not
possible to compare the company's accounting ratios with the standards. Because, there
are no standards with which to compare the company's ratios indicating the company's
per~·ormance.

5. Due to the availability of diverse accounting principles and practices, and since the
companies are free to use any of these diverse accounting principles and practices, it is
Management Accounting: 128

very difficult to compare, with the help of the ratios, the performance of one company
with that of another.
Precautions
In order to have a comprehenSive analysis and interpretation, and to obtain the best result,
the following points are to be kept in mind.
1. Only the data which have a cause and effect relationship should be reckoned for the
purpose of establishing the relationship in the furm of ratio. By establishing relationship
between two entirely un-related factors, nothing can be achieved.
2. With a view to obtain maximum benefit from Ratio Analysis, it is always advisable to
have Horizontal Analysis instead of Vertical Analysis. By doing so, one can identify the
trends in the performance indicators. Further, it is possible to find out the area in which
the organization has improved its performance over the years and the area in which it has
suffered a set-back.
3. Ratios act as the symptoms indicating the probable reasons for the poor performance. It
should, therefore, be the primary duty of the managerial personnel to unearth the exact
reasons and to take the appropriate actions.
4. Each industry is having its own peculiar hallmarks. For example, in case of
manufacturing units, especially private undertakings, plofit maximization is one of the
primary objectives: whereas in the case of public utilIties, the service to the customers is
the primary objective and the profit objectIve is :-.ccondary. In this situation, it is not
rational to compare the profit ratio of a manufactUrIng firm in the private sector with that
of a public utility concern in the public "ector. This type of special features of the
industry should always be kept in mll1d while evaluating the performance of an
organization.
Illustration: 3.12
a. Average stock of a firm is Rs. 40,000. Its opening stock is Rs. 5,000 less than the closing
stock. Find out opening stock.
b. Gross profit ratio 20% on sales. Total gross profit Rs. 1,00,000. Cash sales Rs. 1,20,000.
A verage debtors Rs. 95,000. Calculate Debtors Turnover Ratio.
[Bangaiore Uni., B.Com., May 2000]
Solution:

a. AVerage} =
rl Opening Closingl
Stock 2+ Stock j
and Opening Stock = (Closing Stock - Rs. 5000)
Stock

.'. Rs. 40,000 = r


l 2
l
(Closing Stock - Rs. 5,000) + Closing Stock
-
Accounting Ratios: 129

= t 2 Closing Sto~k - Rs. 5,000 J


:. [Rs. 40.000 x 2] = [2 Closing Stock - Rs. 5,OOOJ
[Rs.80,000 + Rs.5,000J = 2 Closing Stock = Rs.85,OOO

ClOSing} _
Stock -
rl:
Rs. 85,OOOJ _
2 - Rs. 42.500

:.Opening Stock = Closing Stock - Rs.5,OOO = [Rs. 42,500 - Rs. 5.000] = Rs. 37,500

b. Gross Profit = Rs.I,OO,OOO and Gross Profit Ratio =20%

. sales} = [
Revenue
Gross Profit
Gross Profit Ratio
j = t RS.I,OO.OOoJ = Rs.5.00,OOO
20%

Credit Sales = [Total Sales - Cash Sales]


= [Rs.5,00,000 - Rs.I ,20,000] = Rs.3,80,000

.'. Debtors' Tur~oveJ-= I


r Net Credit Sales = [RS.3,80,OOO I = 4 times
RatIo [Average Debtors] Rs.95,000 J
Illustration: 3.13
a. Gross Profit Ratio of a firm is 25%. Gross profIt is Rs. LOO,OOO. Calculate the sales.
b. Average stock of a firm is Rs.50,OOO. Its opening stock is Rs.IO.OOO les:- than its C\o~ing
stock. Find out the opening and closing stock. [Bangalore Uni, B.COIll, May 2002J
Solution:

a. sales } _ r
Revenue
Gross Profit
(Gross Profit RatioJ
I r
= Rs.I ,00,0001 = Rs.4,OO.OOO
( 25% J

b. Opening Stock = [Closing Stock - Rs.I 0,000]

AVerage} = r Opening Stock2+ Closing Stockl


stock l j

Rs.50,000
_ rCClosing Stock - Rs.IO,OOO) + Closing Stock
- t ') Ij
Management Accounting: 130

:. [Rs.50,000 x 2 J = [2 Closing Stock - Rs.lO,OOO] = Rs.l.OO,OOO

. :.Closin g} = [RS.l,OO,ooo + RS.IO,OOO] = RS.55,000


<\to('k 2

:. Opening Stock = [Closing Stock - Rs.lO,OOO J


= [Rs.55,000 - Rs.1 0,000] = Rs.45,000
Illustration: 3.14
a. Gross profit on sales is 25 ck, cost of goods sold Rs.4,OO,OOO. Find out sales.
b. Average stock of a firm is R <;.1.00,000 and its opening stock is Rs.1 0,000 less than
clOSing stock. Calculate Ib openmg and clOSing stock.
IBallgaiore Ulli, U.Com, October 2002}
Solution:
a. Since the Gross Profit Ratio = 25(}t" the Cost of Goods Sold Ratio comes to 75%
[i.e., 1 - 25% = 100 - 25]

..
SaleS}
Revenue -
r Cost of Goods Sold
(Cost of Goods Sold Ratioj
I =[ Rs.4,OO,OOO
7591:
JJ = R 5 33 333
s.,' -

b. Opening Stock = [Closing Stock - R~.IO,OOOJ

AVerage} =
Stock
r
C
Opening Stock + Closing Stockl
2 J

R~. 1.00,000 = [ (Closing Stock - RS.I~'OOO) + Closing Stockj

:. rR~.1 ,00,000 x 2] = 12 Closing Stock - R~.IO,OOOJ = R~. 2,00.000

:. 2 Closing Stock = Rs.2, 10,000

ClOSing} =
Stock
tRS 2
. ,I
2
O'OOO~ = Rs.I.OS.O()()

:. Opening Stock = [Closing Stock - R~. 10,000]


= [Rs.1.05.0()() Ro,.IO,OOO] = Rs.95,000
Accounting Ratios: 131

Illustration: 3.15
a. Gross profit is 20% on sales, cost of goods sold is Rs.3.00.000. Find out sales.
b. Current Ratio 4.5; Acid Test Ratio 3; Inventory Rs.24,OOO. Find out total current
liabilities [Bangalore Ulli, B. Com, April 2003 J
Solution:
a. Since the Gross Profit Ratio =20%, the Cost of Goods Sold Ratio comes to SO%
[i.e., I - 20% = 100 - 20J

SaleS}
Revenue
r Cost of Goods Sold
= lCost of Goods Sold Ratioj
J _[Rs.3.00,000] _ '
- l 80% J - Rs.3,75,000

b. It is known that, [Current Assets -Inventory] = LlqL'ld A"sets.,


Hence, Inventory = [Current Assets - Liquid AssebJ
And. it is given that. the Current Ratio = 4.5 and Liquid Ratio = 3. Therefore,
Current Assets - Current Liabilities = [4.5 - I J = 3.5
Liquid Assets - Current Liabilities = [3 - I J = 2
Subtracting, Inventory - 0 = 1.5
Inventory = 1.5 = Rs.24,OOO

· [ Rs.24,OOO ~
T herefore. 1 = 1.5) = Rs.16,O ()() =
C 'b"
urrent Lia Iiitles.

Illustration: 3.16
a. Given: Current Ratio is 3.75; Working Capital is Rs.3,57.S00, Calculate the amount of
current assets and current liabilities,
b. Cost of goods sold is Rs.2,40,OOO; Stock Turnover 6 times. Opel1lng stock is Rs.6,OOO
more than closing stock. Calculate clo:-'lI1g :-.tocL
I Ballga(ore Ulli.. B. ( 0111 .. November 2003/
Solution:

.
a. Current RatIO = 3.75 = ~CCurrent Assets t3.75~
I lIes = -1-
urren t L'la b'I't' and

[Current Assets - Current LiabilitiesJ = Working Capital
:.[3.75 - I] = Rs. 3,57,50() = 2.75
Management Accounting: 132

:. 1 = lr Rs.3,S7,SOO
? 7'"
I
) = Rs.1,30,000 = Current Liabilities
Current Assets = [1,30,000 x 3.7S] = Rs.4,87,SOO.

b. Stock Turn-} = CCost of Goods SoldI = 6 = C Rs.2.40,000 I


over Ratio l Average Inventory J l Average Inventory J
:. Average} rRS.2,:0,000) = Rs.40,000
Inventory - t j

Average }= rOpening Stock + Closing Stockl = [(ClOSing Stock + 6 000) + Closing Stockl
Inventorv l 2 j 2 J
= Rs.40,000
:. [2 Closing Stock + Rs.6,000] = [Rs.40,OOO x 2J = Rs.80,OOO

:. ClOSing} =
Stock
rtRs.80,000 -
2
Rs.6,OO~
J
= r
t
Rs.7~,OOO I
j
= Rs.37,OOO

Illustration: 3.17
a. Current Ratio is 2.5, Liquid Ratio is 1.5. Working Capital is Rs.50,OOO. Ascertain current
assets and inventory
b. Turnover to Fixed Assets Ratio is I: 1.5: Value of goods sold is Rs.5,OO,000. Compute
the value of fixed assets. [Bangalore Uni., B.eom., November 2000J
Solution:
a. [Current Assets - Current Liabilities] = Working Capital = Rs.50.000
Since Current Ratio = 2.5. Current Assets = 2.5 and Current Liabilities = I
:. [2.S - 11 = Rs.50,000= 1.5

:. 1 =
rtRs.50,000
1.5
IJ = Rs. 33,333 =Current LiabilitIes

:. Current Assets = [2.5 x 33,333 J = Rs 83.333


Accounting Ratios: 133

LiqUid} =[ Liauid Assets J = rRs.83,333 - Inventory 'I =


Ratio l Current Liabilitiesj L Rs.33,333 ) I.S
:. [Rs.83,333 - Inventory] = [Rs.33,333 x I.S] =Rs.SO,OOO
:. Inventory = [Rs.83,333 - Rs.SO,OOO] = Rs.33,333

b. Turnover to}
Fixed Assets
r Sales Revenue"l
= CFixed Assets) = l
r 1
1 'i
J CRs.5,00,000]
J- [Fixed Asset0

:. Fixed Assets = [Rs.S,OO,OOO x 1.S] =Rs.7,SO,000


Illustration: 3.18
a. Gross profit of a firm is Rs. 3,20,000. Operating expenses are Rs. 1,00,000, Taxes
Rs.20,000, Owners' fund Rs.S,OO,OOO. Calculate return on proprietor's fund.
b. If closing stock is Rs.2,OO,000: total liquid assets are Rs.! 0,00,000 and liquid ratio is 2: I,
find out working capital.
c. Stock turnover is 5 times. Average stock is Rs.60,000. Rate of Gross Profit on sales is
20%. Calculate Sales and Gross Profit. [Bangaiore Uni, B.eom, April2004J
Solution:
a. Gross profit Rs.3,20,000
Less: Operating expenses 1,00,000
Profit before Tax 2,20,000
Less: Taxes 20,000
:. Profit after Tax 2,00,000

:. . on ~ = ~Profit
Return
Propnetors' Fund
P
ropnetors Fund
~
. after, Tax x 100 = tRS.2.00.000
Rs.S,OO,OOO
x 100] =40%
b. [Liquid Assets + Closing Stock] = Current Assets
[Rs.IO,OO,OOO + Rs.2,OO,OOO] = Rs.12,00,000 = Current Assets

· 'd} _
L Iqm GLiauid Assets ~ r
Rs.I0,00,000
:. 2 = LCurrent Llabilitiesj
J
R ~ti() - Current Liabilities

:.Current Liabilities = fRs.IO,OO,OOO + 2J = Rs. 5.00,000


Working Capital = [Current Assets - Current Liabilities I
= [Rs.12,00,000 - Rs.S,OO,OOOJ = Rs.7,00,000
Management Accounting: 134

c. Stock TUl:n-} = !cost of Goods Sold! :.5 = rCost of Good~ Sold!


over RatIo [ A vera.!:!e Stock J l RS.60,OOO J
:. Cost of Goods Sold = l Rs.60,000 x 5 J = Rs.3,00,OOO
Since Gross Profit Ratio = 20%, the Cost of Goods Sold Ratio comes to 80%
[i.e., I - 20% = 100 - 20J

:. Sales }
Revenue
= ~CostCost of Goods Sold
of Goods Sold Ratio
= J tRS.3.00.000j
80%
= Rs.3,75,000

:. Gross Profit = [Sales Revenue - Cost of Goods Sold]


= [Rs.3,75,000 - Rs.3,00,000] = Rs.75,000 [i.e., 20% of Sales]
Illustration: 3.19
From the following details, find o.ut (a) Cun'ent assets, (b) Current liabilities, (c) Liquid
assets, and (d) Inventory.
CU;Tent Ratio: 2.5; Liquidity Ratio: 1.5: Working Capital: Rs.60,OOO
[Kuvempu UIl;, B.Com, October 1998J
Solution:
It is given that, Current Ratio = 2.5 [that means, if Current Liabilities = .1. the Current
Assets = 2.5. And it is known that, [Current Assets - Current Liabilities] = Working Capital.
:. [2.5 - 1J = Rs.60,000 = 1.5
:. 1 = [Rs.60,OOO + 1.5] = Rs. 40,000 = Current Liabilities
:. Current Assets = [2.5 x Rs.40,000] = Rs.I ,00,000

LiqUidity}
R~lti()
r Liauid Assets l
= lCurrent Liabilitje~ :. 1.5 =
[ Liauid Assets J
Rs.40,OOO)

:. Liquid Assets = [Rs.40,000 x 1.51 = Rs.60,OOO


[Current Assets - Liquid Assets] = Inventory
:. [Rs. I ,00,000 - Rs.60,OOOJ = Rs.40,OOO = Inventory.

Illustration: 3.20
Find out the average collection period for following

a. Total gross sales Rs. I ,00.000


Accounting Ratios: 135

b. Cash Sales 20,000


c. Sales Returns 8.000
d. Debtors 9,000
e. Bills Receivable 2,000
f. Provision for Doubtful Debt 1,000
[Kllvempu Vni, B.Com, October 2000]
Solution:
Gross Sales «s. 1,00,000
Daily ~verage} =[
rrp(lIt ~alp<;
Rs.72.000.
360 days '"
J
Less: Sales Returns 8,000 = Rs.200
92,000 * Assumed 360 working days.
Less: Cash Sales 20,000

:.Credit Sales 72,000

Average ~ollectionl = ~(AVerage) Trade Debtor"


PerIod f Daily Average Credit Sabi
l
= [RS.9.000 +
Rs.200
RS.2.000~ = [RS.ll.000j
Rs.200
J

= 55 days.

Illustration: 3.21
The following information relates to Chirag Ltd:
10% Perference shares of R .,.10 each R <.;.50,00,000

Equity shares of Rs.IOO each RS.70,00,OOO


Profi t after tax @ 50% Rs.15,00,000
Depreciation Rs.6,00,OOO
Market Price per equity share Rs.200
Calculate: (a) EPS and (b) PER [Kllvempu Vni., B.eom., October 2002]

Solution:
Profit after tax Rs. 15,00,000 Number of Equity Shares: 70,000

Less: Preference Dividend 5,00,000

Equity Earnings 10,00,000

a. Earnings per}'
Equity Share
r EqUIty Earnings I C Rs.IO.00.000
=lNumber of Equity Share:) = l70,OOO Sharesj = Rs.14.29
J
Management Accounting: 136

b. price-Earnings}
Ratio
Market Price of an Equityj
= Equity Earnings per Share -
GRs.200
Rs.14.29
J = Rs.14 times

Illustration: 3.22
From the following information, calculate: (a) Earning per share, (b) Dividend yield on
Equity shares, and (c) Price earning ratio.
Profit after Tax at 60%: Rs. 3,00,000
Market price of Equity share: Rs.50
Depreciation: RsAO,OOO
Equity dividend paid at 20%
Equity capital of Rs. 10 shares: Rs. 4,00,000
9% Preference capital: Rs. 2,00,000
[Kuvempu Uni, B.Com., May 2000J
Solution:
Profit after tax Rs.3,00,000

Less: Preference dividend 18,000

Equity Earnings 2,82,000


Equity dividend= [20% x Rs.lO] =Rs.2
Number of equity shares = 40,000

a. Earnings per} [ Equity Earnings J [ Rs.2.82.000 J _


Equity Share = Number of Equity Sharesj = l 40,000 J - Rs.7.05

Dividend Yield}
b' .
on EqUity Shares
=~ividend per Equity Share
Market Price of an Equity
~
xlO =
t Rs.2
Rs.50
x 100
~ =4%

c. Price Earnings
.
RatIO
. }_
- ~Market
. Price
. of an EqUity]
EqUity Earnmgs per Share
-- t RS.50J -
Rs.7.05
- 7. 09 tImes
.

Illustration: 3.23
From the information given below, calculate (a) Earning per Share, and (b) Price Earning
Ratio.
4,000 Equity Shares of Rs.l 00 each.
Accounting Ratios: 137

4,000 10% Preference Shares of Rs.l 00 each


Net Profit before tax: Rs.~.?O,OOO
Income Tax Rate: 50%
Market Price of Equity Share: Rs.75 [Kuvempu Uni, B.Com, May 2001]
Solution:
Profit before tax Rs.2,80,OOO
Less: Income tax (50%) 1,40,000
1,40,000
Less: Preference Dividend [4,000
shares x Rs.100 x 10%] 40,000
Equity Earnings 1,00,000

a. Earnings per} _ r Equity Earnings ~_ Rs.l.00.000


= RS.25
Equity Share -lNumber of Equity Shares J - 4,000 Shares

b. PriceEarning } = rMarket Price of an EqUity] = [Rs.75] =3times


Ratio LEquity Earnings per Share L RS.25 j
Illustration: 3.24
The capital structure of XY Co., consisted of:
9% Preference share capital of Rs.l 00 each Rs.3,00,000
Equity shares of Rs. 10 each 8,00,000
6% Debentures of Rs.l 00 each 4,00,000
General Reserve 3,00,000
Sundry Creditors 2,00,000
Total 20,00,000
The Gross Profit was Rs. 5,64,000. The tax rate 50%. Market pric~ of Equity Shares is
Rs.40 each. The Dividend declared was 20%. Find out (a) Earnings per Share. and (b) Price
Earning Ratio. [Kuvempu Uni., B.Com., October 1998]
Solution:
Profit before interest and tax (Gross Profit) Rs. 5,64,000
Less: Debenture interest 24,000
"
5,40,000 ·r.

Less: Income tax (50%) 2,70;000


Management Accounting : 138

Profit after tax 2,70.000


Less: Preference Dividend 27,000
Equity Earnings 2,43,000
Number of Equity Shares = 80,000

a. Earnings per }= [ Equity Earnings ~ = r Rs.2.43.000 I = Rs. 3.0375


Equity Share Number of Equity Shares j C80,000 shares)

E~rning} =[Market Price of an Equity J= ~ = 13_. 17 tllnes


b. Price
RatIo

Illustration: 3.25
Equity Earnings per Shar~ t Rs.40
Rs.3.0375
.

From the following, compute the purchases made during the year and the Stock Turnover
Ratio.

Inventory (at cost price): at the beginning of the year Rs.28,000


at the end of the year 42.000
Sales revenue 2,40.000
Sales returns and allowance 12,000
Gross Profit 53.000

Solution:

a. Cost of Good __ Sold = [Sales Revenue - Gross Profit]


= I(Rs. 2.40.000 - Rs. 12.0()(}) - R,>. 53.0001 = IRs. 2.28,000 - Rs. 53,000J = Rs. 1,75,000

A verage Inventory = [ opening Stock 2+ Closing Stock J_ [RS. 28,000; Rs. 42,000J

= (RS. 7~,000J =Rs. 35,000


b. Cost of Goods Sold = [Opening Stock + Purchases - Closing Stockj
Therefore. Purchases = [Cost of Goods Sold - Opening Stock + Closing Stock I
= [Rs. 1.75,000 - Rs. 28.000 + Rs. -+2.000j = Rs. 1,89,000
:. Stock Turnovel} _ ~ost of Goods sOlj [RS. I ,75,00~_5 .
Ratio - Average Inventory
~
Rs. 35,000"::
~
1- -
tllnes
Accounting Ratios: 139

Illustration: 3.26

The capital of XYZ Ltd .. is as follows:

Equity shares of Rs. 10 each Rs. 8.00.000


9% Preference shares of Rs. 10 each Rs. 3.00,000
The following further information are available:
Profit after tax Rs.2,70.000
Equity dividend paid: 20%
Market price of equity shares: Rs. 40 each.

You are required to work out the following indices. Show your workings.

(1) Dividend yield on equity shares. (2) Cover for preference and equity dividend,
(3) Earnings per equity share. and (4) Price-earning ratio.

[ICWA (Fin), December 1988J


Solution:

Profit after tax Rs.2.70,000 Number of shares:


Less: Preference dividend 27.000 Equity 80.000
Divisible profit or Equity earnings 2,43.000 Preference 30,000

Equity Dividend = fRs. 10 x 20% J = Rs. 2 per share

1. Divi~end Yield onL _ [DiVidend ?er Equity Sh.are x 100J = rRR,,~.' :() >.: 10(~ = 5Ck
EqUity Shares f - Market Pnce 01 an EqUity ~, -+ 'J

2. Cover fo~' Pre~'el:enceL =


Profit available for
Paymel~t?f Dividend
J = [ R". 2.70.000 ,'. J
and Equity Dlvlden~ [ Dl\'Idend R". n.()()O + Rs. J .60.000"

= [R~. 2.70.00() + R-;, un.oool = J A-+ tlmcl.,

(* 20O/C of Rs. R.OO.OOO equity "hare capital)

3. Earnings per} [ Equity Earnings ] R" 2A3.0()O]


Equity Share - No. of Equity Share~ = xu.O()O = Rs.~.m7'i
[
• Management Accounting : 140

4. Price-Eamingl_ [Market Price of an Eguit~ _ [


Ratio f-
Equity Earnings per Sharj -
Rs. 40
Rs. 3.0375
J- -
13 17 .
_. tImes

Illustration: 3.27

The following is the condensed form of balance sheets of XYZ Limited for the three years
ended December 31,1982, December 31,1983 and December 31,1984 (Rs. in Lakhs).
Particulars 31.12.'82 31.l2.'83 31.12.'84
Current Assets:
Stock - Raw Materials 12 18 20
Finished Products and Process Stock 30 35 25
Stores and Spares 3 4 5
Debtors 40 50 50
Cash in hand and at Bank 5 10 20
Fixed Assets 90 110 120
180 227 240
Current Liabilities 20 32 30
Debentures, Secured 60 60 60
Unsecured Loans - Banks 15 40 45
Reserves and Surpluses 30 32.5 38.75
I
P & L Account before providing for Taxation and dividends 15 22.5 26.25
Equity Shares (Rs. 100 each) 20 20 20
10% Preference Shares (Rs. 100 each) 20 20 20
180 227 240
Sales 300 360 400
Gross Profi t (%) 15 18 20

The company earned the net profit before providing for income tax at 50 paise per rupee.
Equity shareholders to get dividends 50% more than preference shareholders. Show the
Appropriation Account and work out the following Ratios ann reworking Balance Sheet.
Accounting Ratios: 141

1. Acid test ratio; 2. Stock turnover ratio;


3. Earning per share by capital employed; 4. Ratio of fixed assets to shareholders funds; and
5. Return on capital employed [ICWA (Fin), December 1985J

Solution:

Profit and Loss Appropriation Statement of XYZ Ltd., for the Years ended December 31, ...
1982 1983 1984
Particulars
(Rs.) (Rs.) (Rs.) I
Profit before Tax and Di vidend 15,00,000 22,50,000 26,25,000
Less: Tax at 50% 7,50,000 11,25,000 13,12,500
Profit after Tax 7,50.000 11,25,000 13,12,500
Less: Preference Dividend (10%) 2,00,000 2,00,000 2,00,000
Equity Earnings 5,50,000 9,25,000 11,12,500
Less: Equity Dividends (15% i.e .. 50% higher than the rate
of preference dividend) 3,00,000 3,00,000 3,00,000
Retained Earnings 2,50,000 6,25,000 8,12,500

Restated Balance Sheet of XYZ Ltd., as on December 31, ...


1982 1983 1984
Particulars
(Rs.) (Rs.) (Rs.)
Current Liabilities 20.00,000 32,00,000 30,00,000
Add: Provision for Tax 7,50,000 11,25,000 13,12,500
Total Current Liabilities 27.50,000 43,25,000 43,12,500
Current Assets (all Assets - Fixed Assets) 90,00,000 117,00,000 120,00,000
Working Capital 62,50,000 73,75,000 76,87,500
Add: Fixed Assets 90,00,000 110,00,000 120,00,000
Capital Employed 152,50,000 183,75,000 196,87,500
Calculation of Shareholders' Fund:
Share capital: Equity 20,00,000 20,00,000 20,00,000
Preference 20,00,000 20,00,000 20,00,000
Reserves and Surplus 30,00,000 32,50,000 38,75,000
P & L Appropriation Alc balance 2,50,000 6,25,000 8,12,500
Shareholders' Fund 72,50,000 78,75,000 86,87,500
Management Accounting' 142

1982 1983 1984

1. Acid Test}_ roAs~ IRs. 45,00,000 J RS. 60,00,000 J RS. 70,00,000 J


Ratio - lQLs2j = U~s. 27 ,50,000 [ Rs. 43,2S,000 [ Rs. 43,12.S00

= 1.64 : 1 1.39: 1 1.62 : 1

2. Stock !urnove~= (COGS]1 = [RS. 255,00,000 Rs. 2?5.20,0001 RS. 320,00,000J


RutlO fl AI Rs. 4S,00,oooj [ Rs. ) 1,00,000j [ Rs. 53,50,000

= 5.67 time~ 5.79 times 5.98 times

3. Earnings} Equity Earnings] _ [RS. 5,50,0001 R\ 9 2S,0001 fRs.11,12,500)


per Share =Number of Equity - 20,000 j [ ",0,000 j l- 20,000 j
[
Shares
= Rs.27.S Rs.46.25 Rs.55.625
4. Fixed Assets to 1 [ Fixed Assets J
f
Shareholders' Fund = Shareholders' Fund

= [RS. 9~,OO,()OO
Rs. 7 _,SO,OOO
J RS. 120,00,0001
[ R~. 86.~n .SOO j

= 1.24 times 1.4 times 1.38 times

5. Return on Capital Employed =


Profit after Tax
Capital Employed
}= [ Rs. 7,SO,OOO XIOOJ [Rs. 11,25,000 X]()~ IRs. 13,12,500 XIOOJ
lRs.1S?'if).ono ~Z~.IS3,75,OOO J
lR<;. 196,87,500

= 4. 92 lft: 6.67o/c
Working Notes:
1. QAs. Quick Assets, in this case. include only Debtors and Cash
2. QLs, Quick Liabilities =Current Liabilities as there is no information about overdraft.
3. COGS, Cost of Goods Sold = [Sales - Gross Profit]
4. AI, Average Inventory. However. only closing balance is used for 1982
Illustration: 3.28

The following are the summarised Profit and Loss Account of Rajhans Products Ltd .. for
the year ending December 31. 1981 and the Balance Sheet as on that date:
Accounting Ratios: 143

Profit and Loss Account


I Particulars
! Rs. Particular~ Rs.
I To Opening Stock 9,950 By Sales 85.000
54,525 I
To Purchases By Closing Stock I 14,900
I
To Incidental Expenses 1.425 I
I I
I To Gross Profit 34.()()() : I
I
I
I
II l)l) .l)(JO i i
i
9lJ.9()O
I I
To Operating Expenses: i By Gro..,~ Profit I
34.000 I I
I

Selling and Distribution 3,000 By Non-operating Income:


Administration 15,000 Interest JOO I
I

Finance 1,500 19,500 Profit on Sale of Share 600 90() I


To Non-operating expenses:
I I

Loss on sale of assets 400 I


I
!
To Net profit 15,000 i
34,900 34.900
I
Balance Sheet
Liabilities Rs. ! Assets ! Rs.
I I

I Issued capital: 2,000 Equity Shares I Land and Buildings 15,000


of Rs. 10 each 20,()OO I! PI ant an d M ac h'mery I
I 8,000
i Reserves l),OOO
Stock-in-trade I 14,900
Ie
I un'ent L'Ia bT' I.ltle~ 13.000
I

Sundry Debtors 7.100


! Profit and Loss Alc 6,000
Cash and Bank Balances 3,000
I
48,000 48,000
I
You are required to calculate: (a) Current Ratio
(b) Operating Ratio (d) Return on Total Resources
I c I Stock Turnover Ratio (e) Turnover of Fixed Assets
[CS (Fin), December 1982]

Solution:

·
a. C urrent RatlO =[current Assets
C UITent L'IUb'I' .
J(
J Itles
= Rs. 25,000
Rs. I 3000
,
J = 1.92 : I
Management Accounting: 144

Current Assets: Current Liabilities: Rs. 13,000


Stock in Trade Rs.14,900
Sundry Debtors 7,100
-
Cash and Bank Balance 3,000
25,000

b. OperatingL =rCost of Goods Sold + Operating Expenses


Ratio f L Sales Revenue

Cost of Goods Sold:


Opening Stock Rs. 9,950 Operating Expenses:
Add: Purchases 54,525 Selling and Distribution Rs.3,000
64,475 Administration 15,000
Less: Closing Stock 14,900 Finance 1,500
49,575 19,500
Incidental Expenses ·1,425 51,000 Cost of Sales:
Sales: Rs. 85,000 Cost of Goods Sold Rs.51,000
Operating Expenses 19,500
70,500

. :.Operating Ratio=[::: ~~:~~~ x 100 J= 82.94%


Operating Profit Ratio =[1 - Operating Ratio] =(1 - 82.94%) =17.06%

c.
Stock }
Turnover
.
Raho
=~cost of Goods SOld'] ~ Rs.51,000 J
[RS. 51,000] =4.1 tImes
A verage Inventory = Rs. 9,950 2+ Rs. 14,900 = Rs. 12425
,
.

d. Return on TotalL = rrrofit before Financial Charges x 100


Resources f- L- Total Resources
J
Accounting Ratios: 145

e. Turnover of 1 [sales Revenue] [Rs.85,0001 .


Fixed Assetsf = Fixed Assets = Rs. 23,000J = 3.7 times

(Fixed Assets = Land and Buildings + Plant and Machinery = Rs. 15,000 + Rs. 8,000)

Illustration: 3.29

The following data have been extracted from the books of accollnt of Delhi
Manufacturers Ltd., for the year ended December 31, 1980.

Cash Rs.48,600 Ordinary Share Capital


(shares of Rs. 10 each) Rs. 10,00.000
Land and Building 8,00,000
P and L Alc Balance 2,17,000
Payment in advance 62,000
Proposed Ordinary Dividend
Stock 2,72,800
for 1980 (net) 86,250
Trade Creditors 4,05.750
Sundry. Debtors 5,23.000
General Reserve 1,00,000
Bills Receiv~ble 22,600
Plant and Machinery 5,44,000
Bank Overdraft 52,000
Secured ,Debentures
Trade Investments 20,000
(repayable, 1994) 2,50,000
Provision for Taxation 2,64,000
Advance payment of Tax 1,00,000
Net Sales 21,82,400
Bills Payable 18,000
Net Profit for the year 1980
before taxation and dividend 3,27,830
Note: The values of all fixed assets reflect current price levels and adequate depreciation has
been provided.

You are required to arrange the above items in the form of a Financial Statement and
work out the Return on Capital Employed and then w'ork out the following accounting ratios: (a)
Stock: Fixed Assets, (b) Current Assets: Current Liabilities, and (c) Sales: Debtors and Bills
Receiv~ble. [eS (F.in), June 1981]
Management Accounting : 146

Solution:
Financial Statement of Delhi Manufacturers Ltd., as on December 31, 1980
Amount I
Particulars Rs.
(Rs)
Share Capital (shares of Rs. to each) 10.00,000
General Reserve 1,00.000
Profit and Loss Account 2,17.000 13,17.000
Debentures (secured) 2,50.000
15,67,000
Fixed Asse~s: Land and Building 8.00.000
, Plant and Machinery 5,44.000 '13,44,000
Investments - Trade 20.000
Current Assets:
Cash 48,600
Payment in Advance I 62,000
Stock
I 2,72,800
Sundry Debtors
I 5.23.UO~ I
Bills Receivable I 22.600 I

r
Advance payment or Tax 1.00.000 :

I . 10.29.000
Ii Less: Current Liabilitie~: Trade Creditors 4.05.750
I Bank Cherdraft . 52.000 i
iI Provisiull ror Taxatioll 2.6..J.,llOU
!
B~lIs Payabk I X.UUO i·
Proposed Dividend H6.250 I 8,26,000 2,03,000
I 15,67.000
Note: a. Capital Employed = Rs. 15.67.000
b. Shareholders' Nct Worth = Rs. 13.17.000

l. Rcturn on Capitall = 'Profit (lx·rore FiilalH;iallntcrcst) 100 ]


Employed f l Capital Employed . x
Accounting Ratios: 147

_ fRs. 3,27.830 + Rs. 20,OOO*x 1O~ =22.2% (approximately)


- [ Rs. 15.67.000 )

Note: * Rate of Interest on Debenture is assumed to be 8%.


Therefore, Debenture Interest comes to Rs. 20,000 [Le., 8% of Rs. 2,50,000]

' dA
2 . Stoe k : Flxe - r.
Stock
ssets - L Fixed Assets
J--
(RS. 2.72.800
Rs. 13.44.000
J-02. I
-. .

3. Current Assets:
Current Liabilitiesf
1
=
r-
CUITent Assets
LCurrent Liabilities
J= [RS. 10,29.000
R, 8,26.000
J= 1.25: I

4. Sales: Debtors and Bills Receivable =

Net Sale!-- l r.
Rs. 21.82AOOJ'
( Debtors + Bills Retcivabl':) L Rs. 5,45.600

Illustration: 3.30

From the following annual accounts of JPF Ltd .• for the years 1978 and 1979. you are
required to state the several accounting ratios for the two years which will assist the management
of the company in measuring the efficiency of the company's operation and the possible reason!--
for the changes in the ratios.

1978 1979
Particulars
Rs. Rs.
Sales: Cash Sales 60.00n 6-l.000 I
Credit Sales 5.40.000 6.84.000 I
I
6.00.000 7,4g,OOO !
Cost of Sall!s 4.n.OOO 5.96.000
Gross Protit 1,28.000 1.52.000
Expenses: Warehousing and Transport 38.000 I 41(000
Administration 3X,OOO II 3~WOO I
I !
Selling 22.000 28.000 I
Debenture Interest -I 4.000
Net Profit 30,000 34.000
Management Accounting : 148

December 3 i ...
1978 1979
I
Rs. Rs.
Fixed Assets Less Depreciation 60,000 80,000
Stock 1,20,000 1,88,000
Debtors 1,00,000 1,64,000
Cash 20,000 '14,000
3,00,000 4,46,000
Share capital 1,50,000 1,50,000
Reserves 30,000 60,000
Profit and Loss Account 20,000 24,000
Debentures - 60,000
Current Liabilities 1,00,000 1,52,000
3,00,000 4,46,000

EM. Com., Karnatak University, 1980J


Solution:
1978 1979
Profitability

a. Gross Profit Ratio =


1
Gross Profit x 100 = 1
Rs. 1 28 000 x 100 = 21 1/3% [Rs. 1,52,000 x 1001 = 20.32%
[ Sales Revenue ) s. 6,00,000) ~s. 7,48,000 )

b. Net Profit Ratio =

[S:: ;:~~~:e x 100J = ~~·:.go~ x 100J:! 5% [Rs. 3i~7~:S~4,000 x 100J= 5.08%


(* Net Profit before Taxes and Interest)
Accounting Ratios: 149

c. Operating Ratio =

j J
Cost of Sales +
Other Operating
Expenses but
excluding Debenture
Interest
Sales Revenue
l
RS. 4,72,000 +
Rs.98,000
= Rs. 6,00,00( x 100
RS. 5,96,000 +

Rs.7,48,000

x100 =[RS. 5,70,000 x 1001 _ [ Rs.7,1O,000 x


l
Rs. 1,14,000 x 100 .

lOJ
Rs. 6,00,000 :J -
Rs. 7,48,000 )

=95% = 94.92%
d. Return on Capital Employed =

r Net Profit 100J ..:. fRs. 30,000 10~ = 15% ( Rs. 38,000 J
~apital Employel. x -LRs. 2,00,000 x ) Rs. 2,94,000 x 100 = 12.93%

[Note: Capital Employed = Total Assets - Current Liabilities]


Financial Position
a. Current Ratio =

rLCurrent
Current Assets ~
Liabilitiesj
= [RS. 2,40,000
Rs.l,OO,OOO
RS. 3,66,000J
( Rs. 1,52,000

= 2.4: 1 =2.41 : 1
[Note: Current Assets = Stock + Debtors + Cash]

b. Quick Ratio =
r Quick Assets ~ = r. Rs. 1,20,000J RS. 1,78,000J
LCurrent Liabilities j LRs. 1,00,000j ( Rs. 1,52,000

= 1.2: I =1.17:1

[Note: Quick Assets = Debtors + Cash]


c. Solvency Ratio =
Total Assets
[ External Liability
J rRs.3,00,0001 r.Rs.4,46,0001
= LRs. 1,00,00oj = 3: 1 LRs. 2,12,000j = 2.1 : 1
.
[Note: External Liability =Current Liability + Debentures]
Management Accounting : 150
d. Proprietary Ratio =
( Proprietors' Fund
l Total Assets
J Rs.2,00,0001
= ( Rs. 3,OO,oooj = 0.67 : I
r.Rs.2.34,0001
LRs. 4.46,000j = 0.525 : 1

[Note: Proprietors' Fund = Capital + Reserves + P & L Alc BalanceJ

Operational Efficiency

a. Inventory Turnover Ratio =


~ Cost of Sales ~ [RS. 4,72,0001 . r¥s. 5,96,0021 .
~verage Invento~ = Rs. l,20,OOOj = 3.93 times LRs. 1,88,000j = 3.17 times

(Value of inventories on January 1, 1978 is not given in the problem. Hence, only the
year-end inventory is used in the denominator. Though the value of average inventory for
1979 can be calculated, it is not used as only the value of year-end inventory is used for
1978. Hence. only the year-end inventory is used for both the years).

b. Working Capital Turnover Ratio =


~ Sales Revenue ~ [RS. 6,00,0001 . (Rs. 7.48,0001 .
LWorking Capital j = Rs. 1.40.00oj = 4.29 times lRs.2.l4,000j = 3.5 times

C. Fixed Assets Turnover Ratio =


Sales Revenue
[ Fixed Assets
J =
[RS. 6,00,0001 .
Rs. 60,000 j = 10 times
(Rs.7,48,0001
l Rs. 80,000 j
.
= 9.35 times

d. Total Capital Turnover Ratio =


Sales Revenue
[ Total Capital
J ([Rs.
Rs. 6,00,0001
= 2,00.000j
3'
=. times
(Rs.7,48,0001 .
[Rs. 2,94,000j = 2.544 times

[Note: Total Capital = Proprietors' Fund + Debenture Capital]

As a ba<;e for interpretation, three important ratios are selected for both the years. The
summary is presented below:

1978 1979
1. Net Protit to Sales Ratio (%) 5.000 5.080
2. Sales to Capital Employed Ratio (times) 3.000 2.544
3. Profit to Capital Employed Ratio (i.e .. Return on Capital Employed) (%) 15.000 12.925

From the above. it is obvious that the return on capital has reduced from 15% in 1978 to
12.925% in 1979. This decline is certainly not due to the proiit to sales ratio. Because, it ha-;
Accounting Ratios: 151

increased marginally from 5% to 5.0g%. The reason for the declined Return on Investment Ratio
is due to the decrease in the Capital Turnover Ratio which has reduced from 3 times during 1978
to 2.544 times during 1979. That means. the company has failed to generate adequate amount of
revenue for each rupee of capital employed. During the year 1979. the company employed an
additional capital of Rs. 94,000 registering an increa<;e by 47%. But the increase in the revenue
was only Rs. 1,48.000 which comes to an increase by only 24.67%. That means, the additional
capital has not judiciously been employed by the company during 1979. It is due to this reason
that the company's Return on Capital Employed Ratio has declined from 15% to 12.925%. Other
ratios may also be analysed in the same manner to tind out the extent to which each factor has
caused the variation.

Illustration: 3.31

Below presented are the Profit and Loss Account for the year ended March 31, 2005. and
the Balance Sheet as on that date of Sooryodhaya Company. You are required first to re-arrange
the data in the form of statements and secondly. to compute the following accounting ratios.

a. Gross profit ratio f. Acid test ratio


b. Operating ratio g. Stock turnover ratio
c. Net profit to capital employed ratio h. Debtors' turnover ratio
d. Return on total resources i. A verage collection period
e. Current ratio

Profit and Loss Account (Rs. 00,000)


Particulars Rs. Particulars Rs.
To Opening stock 3 By Sales revenue 20
To Purchases 12 By Closing stock 2
To Gross profit c/d 7
22 22
To Administrative expense 2 By Gross profit bid 7
To Selling and Distribution expenses 2 By Profit on sale of a part of building 3
To Loss an disposal of an old machine 1
To Net profit 5
\0 10
Management Accounting : 152

Balance Sheet (Rs. 00,000)


Liabilities Rs. Assets Rs.
Equity share capital (10,000 equity shares Land 2
of Rs. 100 each) 10
Building 5
General Reserve 5
Plant and Machinery 5
Profit and Loss Alc 2
Stock 2
Sundry Creditors 3
Debtors 5
Cash and Bank balance 1
20 20

Solution:

Income Statement of Sooryodhaya Company for the year ended March 31, 2005
Particulars Rs. Amount Rs.
Sales Revenue 20,00,000
Less: Cost of goods sold:
Opening stock 3,00,000
Add: Purchases 12,00,000
Cost of goods available for sale 15,00,000
Less: Closing stock 2,00,000 13,00,000
Gross Profit 7,00,000
Less: Administrative expenses 2,00,000
Selling and Distribution eXpenses 2,00,000 4,00,000
Operating Profit 3,00,000
Add: Net profit from Non-trading activities:
Profit on sale· of a part of building 3,00,000
.Less: Loss on sale of old machine 1,00,000 2,00,000
Net Profit 5,00,000
Accounting Ratios: 153

Financial Position of Sooryodhaya Company as on March 31, 2005


Particulars Rs. Amount Rs.
Total Assets:
Fixed Assets: Land 2,00,000
Building 5,00,000
Plant and Machinery 5,00,000 12,00,000
Current Assets: Stock 2,00,000
Debtors 5,00,000
Cash and Bank Balance 1,00,000 8,00,000
20,00,000
Capital and Liabilities:
Proprietors' Fund: Share Capital 10,00,000
General Reserve 5,00,000
Profit and Loss Alc 2,00,000 17,00;000
Current Liabilities: Sundry Creditors 3,00,000
20,00,000
Computation of Accounting Ratios

a. ~;~~~1. = [Gross Profit x 1O~ = CRs. 7,00,000 1 =35qf


x 100
RatioJ [Sales Revenue ) (Rs. 20.00,000 )

b. Operati!1g }
Ratio
_ [Cost of Sales
- , ~ales Revenue x 10)
01 -_ [RS. 13,00,000 + Rs. 4,00,000 1001
Rs. 20,00,000 x)
= 85%

Therefore, Operating Profit Ratio = [100 - 85] = 15%

1.
Operating = (Operating Profit x 100]
Profit Ratiof LSales Revenue
= rRs. 3:00,000 x
LRs. 20,00,000
10(0
)
= 15%
-

c. Net Profit to caPital}_


Employed Ratio
C
Net Profit
- ~apital Employed x 1
_r.Rs.Rs. 17,00,000
OJJ-l 5,00,000 OJ _ 29 41 qf
x I ) - . 0

[Note: Capital Employed = Proprietors' Fund (i.e., Rs. 17 lakh) OR


Capital Employed = (Sum of Fi,xed Assets and Working Capital)
(i.e.~ Rs. 12 lakh + Rs. 8 lakh - Rs. 3 \akhj
Management Accounting : 154

d. Return on Total} _ ( Net Profit


Resources - Total A:..sets x I UU
J (
=
Rs. S.OO.OOU J
Rs'. ")_().()(J.()UO x 100 = 25~

e. current} _ (cun.ent Assets


Ratio - Cun'ent Liabilities
J = rLI~s.
Rs. g.oo,ooo] = g: 3 =2.67 : I
3,00.000

f. Acid Test} (LiqUid Assets


Ratio - Liquid Liabilities
J rLR..,·3.00,OOOJ
Rs. 6.00.0001 = ") .
- .
I

g. Stock Turnover}
Ratio

h. Debtors'} (
T urnover =
Ratio
Net C red'It Sa Ies ...
Average Trade Debtors :~
J -. 00 ,OOOJ 4 .
. =(RRs.5,00,000
s. ?()
= tImes.

Note: + Assumed that the entire sale was on credit ba<;is as no other information is available:
* Since the opening balance is not given, only thc closing balance is consiQered without
dividing it by 2.

i. AVerage}
'II .
Co cctlon -
U Avera~e
. o· T"
lade D
e btors' ] - ~R~.
Rs 5.00,000::'J
") 0 00 0 00 - [-R s. -.
5 00, OOO~
Period - Daily Average Credit Sales - , 365 'da;s - Rs.5.479.45
= 91.25 days'
Illustration: 3.32
From the following information. prepare a summarised Balance Sheet a<; at 31 st March,
1990.
Stock Velocity: 6 Creditors Payment Period: 73 days
Fixed Assets Turnover Ratio: 4 The gross protit was Rs. 60,000
Capital TUl110ver Ratio: 2 Closing stock was Rs. 5.000 in excess of opening stock
Gross Profit: 20% Debt Collection Period: 2 months
All working should form part of your answer. leA (Fill), November 1990J
Accounting Ratios: 155

Solution:

a. Sales Revenue and Cost of Sales:


Gro»s Protit = Rs. 60.000 and Gross Profit Ratio = 20Ck

Gross Profit} [ Gross Profit J


·
RatlO =SIa es Revenue x 100

Gross Protit 1 rRs.60.0001


:. Sales Revenuc = ( Gross Pwiit Ratioj = l
20C'k j = Rs. 3.00.000

:. Cost of Good.; Sold = [Sale» Revenue - Gross Protit\ = IRs. 3.00.000 - Rs. 60.000J
= Rs. 2,40,000
b. Inventory:

Stuck 1. ( Cost of Goods Sold J


VelocitYf = Average Inventory

Inventory
r
:. Average }= Cost of Goods SOldJ = rRs. 2.40.000J = Rs. 40.000
L
Stock Velocity 6 L
It is known that, AVerage} = (opening Stock + Closing Stock
Inventory 2
J
Therefore. [Opening Stock + Closing Stock) = [2 x Average Invent-tJr) 1
[Opening Stock + (Opening Stock + Rs. 5.000)1 = (2 x Rs. 40,000) = Rs. HO.OOO
:. 2 Opening Stock = [Rs. 80.000 - Rs. 5.0001 = Rs. 75.000
:. Opening Stock = IRs. 75.000 + 2] = Rs. 37,500 and
Closing Stock = [Rs. 37.500 + Rs. 5,000J = Rs. 42.500

c. Fixed Assets:

Fixed Asset» 1.
rl)ales Revenue]
l
Turnover Ratil{ - Fixcd Assets

. _(RS.
.. 4 - . 3.00.000J .,
FIxed Assets
H x Fixed A»scl» 1= Rs. 3.00.000
:. Fixed Assets = [Rs. 3'()O.OOO + 4) = Rs. 75,000
d. Capital:

Capital Turnovel} _ [ Sales Revenue ~ . ") _ (RS. 3.00.000J


Ratio - Capital j .. - - Capital

:. 12 x Capital! = R.... 3.00.0()O


Hcnn'. Capital = [Rs. 3.00.000 + 21 = Rs. 1.50.000
Management Accounting: 156

e. Debtors

Debt Collection
Period
L_ r
f
Trade Debtors I
-lMonthly Credit Sales
J . 2 th -
.. . mon s -
~Trade Debtors
ffi.s. 3,00,OOQ)
L-12 months)
J
= (Trade Debtors
Rs: 25,000
J :. [2 x Rs. 25,000] = Trade Debtors
:. Trade Debtors = Rs. 50,000

f. Creditors

Average payment} _
Period
r
Trade Creditors
- Loaily Credit Purchase
2
J :. 73 days =.
Trade creditorJ'
(Rs.245,OOQ'l
( l 365 days -J

= (Trade creditors]
. Rs. 671.233
:. [73 days x Rs. 671.233] = Trade Creditors = Rs. 49,000

Working Notes:

1. Assumed to be based on only year-end balance of Trade Debtors


2. Assumed to be based on only year-end balance of Trade Creditors
g. Opening Stock + Purchases - Closing Stock = Cost of Sales.

:. Purchases = [Cost of Sales - Opening Stock + Closing Stock]


:. Purchases = [(Rs. 2,40,000 - Rs. 37,500 + Rs. 42,500)] =.Rs. 2,45,000

Balance Sheet as on March 31,1990


Amount Amount
Capital and Liabilities Assets and Properties
Rs. Rs.
Capital 1,50,000 Fixed Assets 75,000
Creditors 49.000 Debtors 50,000
Inventory 42,500
Cash (balance) 31.500
1,99,000 1,99,000
Accounting Ratios: 157

Illustration: 3.33

From the following information. prepare a summarised Balance She~t as at 31 st March,


1990. Your workings should form part of the answer.

1. Working Capital: Rs. 1.20,000 4. Fixed Asset: Proprietary Ratio = 0.75


2. Reserve and Surplus: Rs. 80,000 5. Current Ratio = 2.5
3. Bank Overdraft: Rs. 20,000 6. Liquid Ratio = 1.5
leA (Fin), May 1990J
Solution:

a. Current Assets and Current Liabilities:

• CurrentL
Ratio f
= r Current Assets
lCurrent Liabilities
J 2.5''
= :. 2.5 =[2 ·5J
1
and

Working Capital = (Current Assets - Current Liabilities)


Rs. 1,20,000 = (2.5 - 1); :. Rs. 1,20,000 = 1.5
:. I = (1,20,000 + 1.5) = Rs. 80,000 = Current Liability
:. Current Assets = (Rs. 80,000 x 2.5) = Rs. 2,00.000

b. Inventory:

Liquidl_ [ Quick Assets


Ratio f -
Quick Liabilities
J = 1.5: and

Quick Liabilitie:-. = (Current Liabilities - Bank Overdraft) = [Rs. 80,000 - Rs. 20,000]
= Rs. 60,000

:. 1.5 = [QUiCk Assets


Rs.60,000
J :. [1.5 x Rs. 60,000] = Quick Assets =Rs. 90,000

Inventory = Rs. 1,10,000 (Because, Inventory = Current Assets - Quick Assets)

c. Fixed Assets and Capital:

It is known that, [Proprietors' Fund + Long-term Loan + Current Liabilities] =


[Fixed Assets + Current Assets] .
Symbolically, (PF + LTL + CL) = (FA + CA)
:. (PF + 0 1 + Rs. 80,000) = (FA + Rs. 2,00,000)
[PF - FA] = [Rs. 2,00,000 - Rs. 80,000) = Rs. 1,20,000; and it is given that,
Management Accounting : 158

FA to PF Ratio =0.75.
Therefore, [PF - FA] = [1 - 0.751 = Rs. 1.20.000:
Therefore. 0.25 = 1.20.000 and 1 = [Rs. 1.10.000 + 0.25] =4JW.000
That means, PF = Rs. 4,80.000 and therefore. FAs = Rs. 3.60,000 (i.e., 4.80.000 x 0.75)
PF =Share Capital + Reserves and Surplus
:. Share Capital = fRs. 4,80,000 of PF - 80,000 of Reserve and Surplus] = Rs. 4.00,000
[Note: 1. Long-term liabilities are assumed to be zero]
Ba )ance Sheet of ... Com)lanv as on March 31, 1990
Amount Amount
Capital and Liabilities Assets and Properties
Rs. Rs.
Share Capital 4,00,000 Fi.xed Assets 3.60,000
Reserves and Surplus 80,000 Inventory 1.10.000
Quick Liabilities 60.000 Liquid Assets 90,000'
Bank Overdraft 20.000
5,60,000 5.60,000

Illustration: 3.34
PQR Company, an organization engaged in the production and sale of product A,
furnishes the following information with a request to complete its incompleted Balance Sheet.

Gross profit margin = 20% A wrage collection periud =30 days


Quick r~tio = 1.5 : 1 Total assets turnover =3 times
Stock turnover ratio = 12 times Other long-term debts (excluding debentures) to
proprietors' fund = 0.75 : 1

Baance
I SheetofPQRC ompanyason D ecemb er 31, 2005
Amount Amount
Liabilities Assets
Rs. Rs.
Equity share capital 3,00,000 Fixed assets 6.00.000
Retained earnings 2,00,000 Inventory -
Debentures 1,25,000 Trade debtors -
Other long-term loans - Cash -
Accounts payable 1.00,000 -
--- ---
Accounting Ratios: 159

Solution:

l. Long-term Debt to} _ (Other Long-term Debt


Proprietors' Fund -l
Proprietors' Fund
J
r.Long-term Debt]
0.75 = l Rs. 5.00.000'

[0.75 x Rs. 5.00.0001 = Long-term Debt


:. Long-term Debt = Rs. 3.75.00()

2. Total Assets .L = rSales Revenue] :. 3 = rSales Revenu~J.


Turnover Ratl<{ l Total Assets l
Rs. I 1.00.000-

:. Sales Revenue =(Rs. 11.00.000 x 3) =Rs. 33.00.000


3. Average CollectionL
Period f
=[ Trade Debtors.1
Credit Sales per Day"'
J :. 30 days = ~TrUde Debtors
TRs. 33.00.000 ~
l 365 days

x 3<D ..,,.,
Trad e De b tors = l[Rs. 33.00.000
365 days J = Rs. _.71._33

4. Sales Revenue Rs. 33.00.000


Less: Gross profit (20Ck) 6.60.000
Cost of goods sold 26,40.000

Stock Turnovel} = R=osl of Goods ~()Id, J Rs. 26.40,000 J


. , = b'r. ear-end
Ratio l Average Inventury .. I '- Inventory

=
IYear-end Inventory x 12] Rs. 26.40.000
:. Year-end Inventory = IRs. 26..+0.000 + 121 = Rs. 2.20,000
5.
Fixed Assets rts. 6,()O,rOO
Inventory 2.::') (JOO
Trade Debtors 2,7 ! .233
Total assets r::;,,;lud:ng cash IO.9L.:!3")
Total assets including cU'ib 1 1.'/;,000
:. Cash 8.767
Management Accounting: 160

Ba Iance Sheet 0 fPQRC ompany as on D ecem b er 31 , 2005


Capital and Liabilities Rs Assets and Properties Rs. .
Equity share capital 3,00.000 Fixed Assets 6,00,000
Retained earnings
. 2,00,000 Trade debtors 2,71,233
Debentures 1,25,000 Inventory 2,20,000
Long-term loans 3,75.000 Cash 8,767
Accounts payable 1,00,000
i
11,00.000 11,00,000
Note: 1. Proprietors' Fund = [Share Capital + Retained Earnings]
2. The sum of Liabilities side including the Long-term Loan
3. Assumed to be comprised of only the year-end balance
4. Assumed that the entire sale was on credit basis and there was no sales return
5. Since there is no information about the opening inventory. only the year end
inventory is taken
Illustration: 3.35

From the following figures and ratios. make out the Balance Sheet. and the Trading and
Profit and Loss Account with as many details as possible.

a. Share Capital: Rs.7,20.000 c. Bank Overdraft: Rs. 40,000


b. Working Capital: Rs. 2.52.000

There are no fictitious assets. In cun-ent assets, there is no asset other than stock. debtors
and cash. Closin~ stock is 20% higher than the opening stock.

1. Current Ratio =2.5 5. Stock turnover =4 times


2. Quick Ratio = 1.5 6. Average collection period = 36.5 days
3. Fixed assets/Proprietary fund =0.7 7. Net profit to paid up capital = lOo/s; ,
4. Gross profit ratio = 20% [M.Col11 (Fill), 1984]
Solution:
1. Working Capital = [Current Assets - Current Liabilities]
:. Rs. 2.52.000 = [2.5 - I] (Because. Current Ratio = 2.5)
:. 1.5 = Rs. 2.52.000
Accounting Ratios: 161

:. Current Assets;:: (2.5 x Rs. 1,68,000) = Rs. 4,20,000


Current Liabilities = (1 x Rs. 1,68,000) = Rs. 1,68,000

2. Quick Liabilities = [Current Liabilities - Bank Overdraft]


= [Rs. 1,68,000 - Rs. 40,000] = Rs. 1,28,000

3. QUiCk} _ r.
Quick Assets J
Ratio - lQuick Liabilities
. 15 -
.. . -
r Liquid Assets
~iquid Liabilitie~
~ = [LiqUid AssetsJ
Rs. 1,28,000

Quick Assets =(Rs-. 1,28,000 x 1.5) =Rs. 1,92,000 (i.e.,.Debtors + Cash)


4. Current Assets = [Quick Assets + Closing Stock]
Rs. 4,20,000 = [Rs. 1,92,000 + Closing Stock]
:. Closing Stock = (Rs. 4,20,000 - Rs. 1,92,000) = Rs. 2,28,000
. .
5. Closing Stock =Opening Stock + 20% of Opening Stock
Rs. 2,28,000 = 120% of Opening Stock
.
:. Opem~g Stock = lrRs. 120%
2,28,000J
= Rs. 1,90,000

6. TStock }
urnover
. =
[cost of Goods Sold
A verage I nventory
J
Ralto

Cost of Goods Sold


4 = [ ~s. 2,28,000; Rs. 1,90,000J
J _ [cost of Goods SOld]
- Rs. 2,09,000

:. Cost of Goods Sold =(Rs. 2,09,000 x 4) =Rs. 8,36,000


7. Gross Profit Ratio =20% of Sales. That mejll1S, it comes ro 25% of Cost of Goods Sold.
:. Gross Profit = (25% of Rs. 8,36,000) = R~. 2,(1),000

:. Sales Revenue =Cost of Goods Sold + Gross Profit


=Rs. 8,36,000 + Rs. 2,09,000 =Rs. 10.45.000
8. Net Profit = 10% of Paid up Capital =(10% of Rs. 7,20,000) = Rs. 72,000
9. Gross Profit Rs. 2.09,000
Less: Net Profit 72,000
:. Operating Expenses 1.37,000
Management Accounting: 162

10. Average Collection Period = [ Debtors*


S I'
a es
x 365 days J
[Note: :" Assumed to be only closing balance]·

36.5 days = (Debtor~ x 365 daysl


Rs. 10,45.000 j

Debtors = [
RS. 10,45.000 x 36.5
365
J= Rs. 1.04.500

II. Quick Assets = [Cash + Debtors]


Rs. 1.92.000 = ICa~h": R~. 1.0-l-.500J
Therefore. Cash =IR .... I.lJ2.00U - R~. l.04.500j = Rs. X7.500
12. Proprietary Ratio = [Fixed A~set:-. + Proprietary Fund] wid
Proprietors' Fund =[Fixed Assets + Working Capital]
I = [0.7 + Working Capital]
:. Working Capital = [1 - 0.7] = 0.3 (It is given that, Working Capital = Rs. 2.52,000)
0.3 = Rs. 2.52.000
.. I = [Rs. 2.52.01l0 + (D I = Rs. HAO.OOO
:. ProprIetary Fund =(I x R~. X.40.0()()) =Rs. X.40.000
Fixed Assets = to.7 x Rs. 8.40'<)00) = Rs. 5.8H.OOO
Reserves and Surplus = (Proprietors' Fund - Share Capital)
= IRs. HAO.OOO - Rs. 7,20.000] = Rs. 1,20.000
Trading. an,-!J~rofit and Loss .\ccount of ... Com any for the
Partil'ular~
! Amount i Particulars
Amount
(Rs.) I .__ _ (R~.) i
To Opening stock l.lJO.O()() I By Sales 10.45.000 I
To Purcha~cs (b ,-dance)
II By Clusing. :-.tock I
I 2.2H.000
i X.7-1-.00U 1

To Grm.... profit c/d I


I
2.09.000 I i
12.73.000 12.73,000
To Operating ex penses 1,37.000 By Gross protit bId 2,09.000
i
I To Net protit bld 72.000 I !

2.09.0()() i 2.09.000
Accounting Ratios: 163

Ba Iance Sheet 0 f ... Company as on ...


Capital and Liabilities Rs. Assets and Properties R~.

Share capital 7.20.000 Fixed Assets 5.88.000


Reserves and Surplus 1.20,000 Closing stock 1,28,000
Bank overdraft 40,000 Debtors 1,04,500
Other liabilities (balance) 1.18.000 Cash 87,500

IO.OX.OOO IO.m(OOO 1
I
Illustration: 3.36
The following data relate to the financial statements of RCK Limited for the year ended
December 31. 1979.

1. Current ratio 3 : I 6. Number of shares of Rs. 10 each 25,000


2. Acid test ratio 1.5 : 1 7. Earning per share Rs. 2
3. Quick assets Rs. 30,000 8. Gross profit to sales 20(/i

4. Inventory turnover 10 times 10. Creditors' velocity 27 times


5. Fixed Assets: Shareholders equity 0.875 : 1 II. Debt collection period 12 da)"
6. Tota1l~rerating expenses to sales 90% 12. Earning for the year as percentage of share
capital 25C;;

Closing stock is Rs. 6,()()0 lower than opening stock. There are no prepaid expenses,
long-term liabilities and intangible assets. Prepare company's Profit and Los" Account ror the
year ended December 31. 1979 and Balance Sheet on that date. Assume ~ ear to be of 360 day ....
I M. Com .. Allrlla(ak L·lliversity. 1980J
Solution:

I. Acid Test Ratio = lQuick Assets + Current Liahilitie"l


[Note: Assumed that the Bank Overdraft is equal to zero and therefore. the Current Liabilities
are equal to Liquid or Quick Liabilities].

1.5 = [ Rs. 30,O()() J :. Current Liabilities =[Rs. 30.000 + 1.51 =Rs. 10.000
Current Liabilities

2. ClIITcnt Ratio = [Current Assets + CU1Tcnt Liabilitic ... l


3 = (Current Assets + Rs. 20'()00)

:. Current Assets = (Rs. 20,000 x 3) = Rs. 60,000


Management Accounting : 164

3. Quick Assets = [Current Assets - Closing Stock]


Rs. 30,000 = eRs. 60,000 - Closing Stock)
:. Closing Stock = (Rs. 60,000 - Rs. 30,000) = Rs. 30,000
4. It is given in the problem that the Closing Stock is Rs. 6,000 lower than the Opening Stock.
:. Closing Stock = (Opening Stock - Rs. 6,000)
Rs. 30,000 = (Opening Stock - Rs. 6,000)
:. Opening Stock = (Rs. 30.000 + Rs. 6,000) = Rs. 36,000
5. Stock Tumove~ = (cost of Goods SOld] _I; ~
Ratio f Average Inventory
. 10
.. - l ~s.Cost of Goods Sold
36,000 ; Rs. j
30,OO~

rL
10 = Cost of Goods SOld]
Rs.33,000
:. Cost of Goods Sold =
(Rs .. 33,000 x 10) = Rs. 3,30,000
6. It is given in the problem that the percentage of Gross Profit to Sales comes to 20%. That
means, if the sales revenue is Rs. 100, the gross profit is equal to Rs. 20. Therefore, cost of
goods sold is equal to Rs. 80. The gross profit may be linked to the cost of goods sold and
it comes to 25% [(Rs. 20 + Rs. 80) x 100]. Therefore. gross profit comes to 25% of cost of
goods sold of Rs. 3,30,000 which is equal to Rs. 82.500. And, it is known that,

Sales Revenue = (Cost of Goods Sold + Gross Profit)


= (Rs. 3.30,000 + Rs. 82.500) = Rs. 4.12,500

7. Cost of Goods Sold'= [Opening Stock + Purchases - Closing Stock]


Rs. 3,30.000 = [Rs. 36.000 + Purchase~ - Rs. 30.0001
:. Purchases = [Rs. 3,30,000 - Rs. 36.000 + Rs. 30.000J = Rs. 3;24.000
8. Total Operating Expenses = 90% of Sales Revenue = 90% of Rs. 4,12.500 = Rs. 3,71,250
9. Operating Expenses (i.e., Administrative, and"Selling and Distribution Expenses)
= [Total Operating Expenses'- Cost of Goods Sold]
= [Rs. 3,71,250 - Rs. 3,30.000] = Rs. 41,250
10. Operating Profit = [Gross Profit - Operating Expenses] = [Rs. 82.500 - Rs. 41,250]
= Rs. 41.250
11. Net ProtIt = (Earnings per Share x Number of Shares) = (Rs. 2 x 25,000) = Rs. 50.000
Accounting Ratios: 165

12. [Operating Profit + Net Non-operating Profit] =Total Net Profit


(Rs. 41,250 + Net Non-operating Profit) = Rs. 50,000
:. Net Non-operating Profit = [Rs. 50,000 - Rs. 41,250] = Rs. 8,750

13. Debt Collection Period = [~:~~o~:: x 360 days] [Note: *Assumed to be the year-end balance]

12 days =[ Rs.Debtors
4,12,500 x 360 days
J :. [12 x Rs. 4,12,500] =[Debtors x 360 days]
RS. 4,12,500 x 12 ] rRs.49,50,0001
:. Debtors = [ 360 = l
360 J
= Rs. 13,750
14. Quick Assets = [Debtors + Cash and Bank balances]
Rs. 30,000 = [Rs. 13,750 + Cash and Bank balances]
:. Cash and Bank Balances = [Rs. 30,000 - Rs. 13,750] = Rs. 16,250

15. Creditors' }
Velocity
Net Credit Purchase
- [ Average Trade Creditors*
J [Note: * Assumed to be only the closing balance]

- [ Rs. 3,24,000]
27 - Creditors :. [27 x Creditors] = Rs. 3.24,000

· r Rs. 3,24,0001
:. Cre dItors =l 27 j =Rs. 12,000

16. Earnings for the year as %age of Share Capital =[p .~arni~gs.
al -up apIta
I x 100 ]

Note: Earnings =(Number


25 = r ~s. 50.000. x 1001 of Shares 25,000 x
LPmd-up Capital ) Earnings per Share
Rs.2)

apltaI = [RS. 50,000


:. Pm·d-up C· 25 x 100] =Rs. 2,00,000

That means, 25,000 shares of Rs. 10 each Rs. 8 paid (Because, Rs. 2,00,000 + 25,000 shares)

17. [Shareholders' Equity + Long-term Liabilities + Current Liabilities]


= [Fixed Assets + Current Assets]
Management Accounting: 166

:. [Shareholders' Equity + 0 + Rs. 20.000) = [Fixed Assets + Rs. 60,000)


:. [Shareholders' Equity - Fixed A~sets) ='\ R~. 60,000 - Rs. 20,000] = Rs. 40,000
It is given that, Fixed Assets: Shareholders' Equity is equal to 0.875 : I
:. [I - 0.875] = Rs. 40,000
0.125 = Rs. 40,000
Therefore, 1 =fRs. 40,000 + 0.12-5] = Rs. 3,20,000
That means, Shareholders' Equity = Rs. 3.20.000, and
Fixed Assets =Rs. 2,80,000 (Le., OJ.;7S x Rs. 3,20,000)
18. Shareholders' Equity = [paid-up Capital + Reserves + Protit)
Rs. 3.20,000 =[Rs. 2,00,000 + R"eserves + Rs. 50,000J
, ,

Reserves = [Rs. 3,20,000 - Rs. 2,00,000 - Rs. 50,000] = Rs. 70,000


P rotiIt and L. oss Account 0 f Rf:K Lt d., ~or the year ended December 31 , 1979
Amount Amount
Particulars Particulars
(Rs.) (Rs.)
To Opening stock 36,000 By Sales 4,12,500
T!} Purchases 3.24,000 By Closing stock 30,000
To Gross profit cld 82,500
4,42,500 4,42,500
To Operating expenses 41.250 By Gross profit bId 82,500
. To Net profit 50,000 By Net Non-operating protit 8,750
91,250 91,250

Balance Sheet of RCK Ltd. as on December 31, 1979


Capital and Liabilities Rs. Assets and Properties Rs.
Share capital (25,000 shares of Fixed Assets 2Jm,000
Rs. 10 each. Rs. 8 paid) 2,00,000
Current Assets:
Reserves 70,000
Closing stock 30.000
Profit and Loss Ale ' 50,000
. Debtors 13,750
Current Liabilities:
Cash and Bank Balance 16,250
Creditors 1:!,()OO
Other liabilities (balance) ~Ul()O

3.40,000 \ 3.40,000
I !
Accounting Ratios: 167

Illustration: 3.37
A~sume that a firm has owners' equity of Rs. 1.00,000. The ratios for the firm are:
Short-term deht to total debt = 0.4 Total assets turnover = 2 times
Total debt to owners' equity =0.6 Inventory turnover =8 times
Fixed assets to owners' equity = 0.6
Complete the following Balam:e Sheet. from the information given above.
Capital and Liabilities Rs. I Asseh Rs.
Short-term debt - Cash -
Long-term debt - Inventory -

Total Debt - Total Current Assets -


Owners' equity - Fixed Assets -
Total capital and Liabilities - Total Assets -

Solution:
Total Debt ~
I. Total Debt to Owners' Equity = (Total Debt + Owriers' Equity) :. 0.6 = [ Rs. I,OO,OOOJ
. .
.'. Total Debt = (Rs. 1,00,000 x 0.6) = Rs. 60,000
2. Short-term Debt} _ [Short-term Debt] :. 0.4 = [Short-term Debt]
to Total Debt - Total Debt Rs.60,000

:. Short-term Debt =(Rs. 60,000 x 0.4) =Rs. 24,000


3. Total Debt = (Long-term Deht + Short-term Debt)-
Rs. 60,000 = (Long-term Debt + R~. 24,000.,
.'. Long-term Deht =(Rs.60.000 - Rs.24'()OO) = Rs. 36,000

4. Fixed Assets to} =[ Fixed Assets ]


:.0.6 =
Fixed Assets ]
[ Rs. 1,00,000
Owners' Equity Owners' Equity

:. Fixed Assets = (Rs. 1,00,000 x 0.6) = Rs. 60,000

5. Total Assets = Total of the Liabilities side of the Balance Sheet


= Owner's Equity + Long-term Debt + Short-term Debt
= (Rs.1 ,00,000 + Rs.36,OOO + Rs.2-\"oOO) = Rs. t ,60,000
6. Total Assets 1 _ (Cost of Goods SOld*] . ., = [cost of sales]
Turnover Rati({ - l Total Assets .. - Rs. I ,c)().OOO
Management Accounting : 168

:. Cost of Sales = (Rs. 1,60,000 x 2) = Rs. 3,20,000


[Note: * In the numerator, cost price of goods sold is considered as the denominator is at
cost price]

7. Inventory }
Turnover Ratio
_
-
(Cost of Goods Sold
lAverage Inventory *
J Note: * Average inventory is assumed
to be based on only the closing
stock.
8-
-
[RS. 3,20,000
Closing Stock
J :. ClOSing}
Stock
= r.Rs. 3,~0,000J = Rs. 40,000.
l
8. Total Assets =[Fixed Assets + Cash + Inventory]
Rs. 1,60,000 = (Rs. 60,000 + Cash + Rs. 40,000)
:. Cash = [Rs. 1,60,000 - Rs. 60,000 - Rs. 40,000] = Rs. 60.000

Ba Iance Sheetof ... Company as on ...


Capital and Liabilities Rs. Assets and Properties Rs.
Short-term debt 24,000 Cash 60,000
Long-term debt 36,000 Inventory 40.000
Total Debt 60,000 Total current Assets 1,00,000
Owners' equity 1,00,000 Fixed Assets 60,000
Total Capital and Liabilities 1,60,000 Total Assets 1,60.000

Illustration: 3.38

Assuming the Current Ratio is 2. state in each of the following cases whether the Current
Ratio will improve or decline or will have no change.

a. Payment of a Current Liability d. Bills Receivable Dishonoured


b. Purchase of Fixed Assets e. Issue of New Shares
c. Cash collected from Customers lCA (Fin), May 1990]
Solution:

Current Ratio = 2. Let us, therefore. assume that Current Assets = Rs. 2.00,000. and
Current Liabilities = Rs. 1,00,000. By assuming these, the implications of each of the cases on
the Current Ratio is analysed below.

(a) Payment of a Current Liability of say. Rs. 20,000 improves the Current Ratio. Because. it
(i.e., payment of a Current Liability) lowers both the Current Assets and Current Liabilities hy
Accounting Ratios: 169

an equivCllent amount i.e .. Rs. 20,000 each. But the rate of reduction varies. Cun-ent Assets
deC\in~;by lOCK [(i.e., Rs. 20,000 + Rs. 2,00,000) x JO(>I and the Current Liabilities by 20 ck
[(i.e., Rs. 20,000 + Rs. l.OO,OOO) x 100J. Hence, the Current Ratio increases or improves.
This is evident from the following.

. [ Rs. 2,00,000 - Rs. 20,000 ] [RS. l.SO,OOOI


Cun-ent RatIO = Rs. 1,00,000 _ Rs. 20,000 = Rs. 80,000 j = 2.25 : I
(b) Purchase of Fixed Assets (assumed to be for Rs. lO,OOO on cash basis) lowers only the
Current Assets not followed by a similar type of r~dlll:tioll or increase in Current Liabilities
(of course, it will cause the increase in the Fixed ASS~h by Rs. 20,000). As a result. it (i.e.,
the purchase of tixed assets) will cause the reduction in the Current Ratio as evident from the
following.
.
Current RatIO =
[RS. 2,00,000 - Rs. 20,000J
Rs. 1,00,000
r.Rs. 1,00,000
= lRs.
1,80,00QI
j = 1.8 : I
(c) Cash Collected from Customers (of say, Rs. 20,00t» increases cash (one of the items of
Current Assets) and reduces the debtors (another item of Current Assets) by an equivalent
amount. Hence, no change takes place in the total Current Assets. Therefore. 'collection of
cash from the customers will have no change in the Current Ratio.
· rRs. 2,00,000 - Rs. 20,000 + Rs. 20.0001 ') I
C urrent Ratlo =t Rs. 1.00,000 j =- :

(d) Bills Receivable Dishonoured (of say, Rs. 20,000) increa<;es the debtors, one 'of the items of
Current Assets. and lowers the bills receivable, another item of Current Assets. by an
equivalent amount. Hence. it will not change the Current Ratio.
. R· r Rs. 2,00,000 + Rs. 20.000 - Rs. 20,000J 2 I
C urrem aho =l Rs. 1,00.000 = :

(e) Issue of New Shares (say, of Rs. 20.000 and for cash) increases the c:lsh and the share
capital, an item of non-working capital items. by an equivalent amount. But it will not affect
other items of either the Current Assets or the Current Liabilities. Therefore, issue of shares
for cash increases the Current Ratio.
· r Rs. 2,00,000 + Rs. 20,000 J ') ') I
C urrent Ratlo = l Rs. 1,00,000 =_.- :
Dlustration: 3.39
From the following information of M Engineering Co., complete the proforma Balance
Sheet if its sales are Rs. 16,00.000.
Sales to Net Worth =2.3 times Current Ratio = 2.9 times

Current Liabilities to Net Worth =42% Sales to Closing Inventory = 4.5 times

Total Liabilities to Net WOIth = 75% . A verage Collection Period = 64 days



Management Accounting : 170

Proforma Ualance Sheet


I -_ _ _ L_i_u_b_i_it_ic_.~_ _ ___t_--I-{-~.-I __A_!'>_sc_·t_s_____I--_RS~
Nct I
W0I11
., -Ii 1-' I A.
-IXt'( . ssets .,
I i
I ./ I Cush
I .) I Stock ?
.)
L~ Sundry Debtors

t I

Calculations are to be to the nearest rupee. leA (Fill), November 19911

Solution:
a. [ Sules J .. 00.0()(~
= .,_.-l''lrRs.Net16Worth
Net Worth J = 2.3: :.(Net-worth x 2.3) = Rs. 16.00.0()()

Net-worth =1Rs. 16.00.000 + 2.31 =Rs. ·6.95.652

b. [Total Liabilities]
Net Worth
5' (
= 7. lft = >.75; ..
r.Total
l
Liabilities]
Rs.6.95.652 =0.75

Total
,
Liabilities = (Rs. 6.95.652 x 0.75) = Rs. 5.21,739 ".

c. (current LiabilitieS] - 4?C'1, - 04?' :. ( Current Liabi1~i_CSJ


Net Worth - - 1(. - • -. Rs. 6.95.65_ = 0.42

CU~'ent Liabilities = (Rs. 6~95.652 x 0.42) = 2.92.174


Totul Liubilities Rs.5.21.739
Less: Current Liabilities Rs.2.92.174
,
.. Long-term Liabil'ities Rs. 2.29.565

,
··d. Current RatIO
.
=_.? ~ =
l (. Current As~ets
C urrcnt 1.IU
. bITIlles'
. -
J. :
[current ASSt'ts~
., I)')_. 174'
Rs. _.

:. Current Assets = (Rs. 2.92.174 x 2.9) = Rs. H.47.305


Accounting Ratios: 171

e. [ Sales J-
4 5'
Closing Inventory - . ,
. [
..
Rs. 16,00,000
Closing Inventory
J =. 45

[Closing Inventory x 4.5] = Rs. 16,00,000


Closing Inventory =(Rs. 16,00,000.+ 4.5) =Rs. 3,55.556

f. Average Collection Period =64 days = (Average) Trade Debtors


[ Daily A \'crage (Credit) Sales
J
64 = [ Trade Debtors'
(Rs. 16,00.000 + 365)
J'
Trade Debtors = [(Rs. 16,00.000 + 365) x 641 = 14383.56 x 64J = Rs. 2.80,548
Total Current Assets Rs. H.47,305
Less: Closing Stock Rs. 3.55.556
Trude Debtors Rs. 1.HO.548 6.36.104
:.Cash Rs.2, 11,20 I

p ro~orma BIa ance Shee t 0 fM Engmeermg 0 ..• c


Amount Amount
Liahi lities I !
Assets
I
I Net worth
i Rs.
6,95.652 Fixed Assets (balance)
Rs.
3.70,086
I
Long-term Liabi lities I 2,2lJ.565 Cash I 2.11,201
Current Liabilities 2,92,174 Stock 3.55.556
Sundry Debtors 2.80,548
I
12.17.391 12.17.391
I
Illustration: 3.40

From the following ratio~ and further information given below. prepare a Trading and
Protit and Loss Account amI a Balance Shcetllf \11'. Green.

Fixed A~sets/Capilal = 5/4 Slol' J.. Turnover Ratio = 10


Fixed Assets = Rs. 5.00,000 Fi \cd A~sets/Total CUITent Assets = 5/7
Capital/Liabilities = 1/2 1'Iiet Profit to Sales =2W/o
Net ProtitlCapitai = 115 Closing Stock = Rs. 50.000
Management Accounting : 172

Gross Profit Ratio = 25£K

Out of Current Assets, Sundry Debtor~ are R~. 6,00.000. The balance represents Cash
and Closing Stock. leA (lilt), May 1992J

Solution:
a. [ Fixed Assets
Capital
J =[~J
4 - - 1.-.
?5' :. [RS. 5,0.0,000
Capital
J = 1.25:

(Capital x 1.25) = Rs. 5,00,000; :. Capital = lRs. 5,00,000 -:- 1.25] = Rs. 4,00,000

b. Capital
· b'I'
GLJa
. J- [~] -
I ItJes - 2 - 0 .,
5' ~RS.L' 4,00,00!J
Ja
b'I' .
I Itles .. , ... (L',Jab'I'
-- 05' . x 05)
I ItJes . -- Rs. 400000
, ,

:. Liabilities = (Rs. 4,00,000 -:- 0.5) = Rs. 8,00,000

c. rNet Profit]= [~] = 0.2;


l Capital 5
r
Net Profit = 0.2
lRs. 4.00,00qj
I
Therefore. (Rs. 4,00,000 x 0.2) = Net Profit = Rs. 80,000

d. Net Profit to Sales '= 20%

. [Net Profit] _ ? =
.. Sales - 0._
rl Rs.Sales
80,00!~
J
(Sales x 0.2) = Rs. 80,000; :. Sales Revenue = (Rs. 80,000 -:- 0.2) = Rs. 4,00,000

e. (GrosS" ProfitJ _.
[ Sales :J = 25% = 0.25: :. Gross Pruitt = (Sales Revenue x 0.25);

Gross Profit = (Rs. 4,00,000 x 0.25) = Rs. 1,00.000

f. (Gross' Profit - Operating Expenses) = Operating (or Net) Profit


Hence, Operating Expenses = (Gross Profit - Net Profit)
= [Rs. 1,00,000 - Rs. 80,000] = Rs. 20,000

g. [ Fixed Assets J
[5 J .
lTotal Current Asset~ = -7- ; :. FIXed Assets = [Total Current Assets x (517)]

:, Current Assets = [ (517)


J= [
Fixed Assets Rs. 5 00 000
dl7) J= Rs. 7,00,000

Total Current Assets Rs.7,00,000


Accounting Ratios: 173
Less: Sundry Debtors 6.00.000
:. Cash and Closing Stock 1,00,000
Less: Closing Stock 50.000

:. Cash 50.000

h. (Saks Revenue - Gross Protit) = Cost of Goods Sold


IRs. 4.00,000 - Rs. 1.00.0(0) = Rs. 3.(JO.OOO = Cost of Goods Sold

Stock Turnover Ratio = enst of Goods SOI~


t
A verag~ I n\'l~n tor) . 10 -
.. -
[ R~. 3.00.000
Average Inventory
J
Thercforl'. -\\'crage Inventory = (Rs. 3,00.000 + 10) = Rs. 30.000

Rs. 30.000} =
Average t
Opening Stock; Closing Stock J
Inventory -

:. (Rs. 30.000 x::!) = [Opening Stock + Rs. 50.000 Closing Stockl


=
:. Opening Stock IRs. 60.000 - Rs. 50,0001 R~. 10.000 =
1. [Opening Stock + Purchases - Closing Stockl = Cost of Goods Sold
=
Rs. 10.000 + Purchases - Rs. 50.000 Rs. ],00.000
=
:. Purchases [Rs. 3,00,000 - Rs. 10,000 + Rs. 50'()OOI Rs. 3"+0,000 =
D r. T rad'mg. an d P roIiIt andLoss A cc()unt C r.
Amount' Amount
Particulars I Rs.
Particulars \
Rs.
To Olx'ning stock 10.000 By Sales 4,00.000
i
To Purchases 3.40.000 By CJo..,illg Stock 50,000
I To Gross Protit c/d 1,00,000
4.50,000 4,50,000
To Operating Expenses 20.000 By Gross Profit bid 1.00,000
To Net Protit 80,000
1.00.000
I 1,00.000
Management Ac.counting 0: 174

Balance Sheet
Amount " Amount
f °Capital and Liabilifies Assets and Properties
Rs. ·Rs.
Capital 4,00,000 Fixed Assets 5.00,000
Liabilities 8.00.000 Inventory 50.000 I

Debtors 6.00.000
Cash 50.000
I
12.00,000 12.00.000

Illustration: 3.41

A company having °a net working capital of Rs. 2.8 lakhs as on 30.6.1996 indicatos the
following financial ratios and pe~formallce ligures:

Cun-ent Ratio = 2.4 Gros~ profit 011 "ale~ = lock


Liquidity Ratio = 1.6 Credit allowed (months) = 1.5
Inventory turnover (on cost of sales) = 8

The company's tixed assets are equivalent to 90% of its net-worth (share capital plus
reserves) while reserves amounted to 40% of share capital. Prepare the Balance Sheet of the
company as on 30.6.1996 showing step by step calculations.
liCWA (Int), December 1996J
Solution:

u. Working Capital =(Current Assets - CUlTent Liabilities)


Rs. 2,80.000 =(2.4 - I) = 1.4 (because, Cun-ent Ratio =2.4 =2.4 : 1):
:. 1 =(Rs. 2.80.000 + 1.4) =Rs. 2,00,000 =Current Liabilities
Hence, Current Assets = (Rs. 2,00.000 x 2.4) = Rs. 4.80.000
b. Assuming that the Bank Overdraft and Pre-paid Expenses are zero, year-end Inventory can be
computed as shown below.

LiqUidity}
Ratio
- ( Liquid Assets
- Liquid Liabilities
J = r Current Assets - Inventory
lCurrent Liabilities(as. BOD =
"I
OU

16
. = lrRs. 4,80,000 - Inventoryl. .
Rs. 2.00.000 j . .. (1.6 x Rs.. 2.00.000) -- [. 00
Rs. 4,80. 0 - Inventory]
Accounting Ratios: 175

:. Inventory = (R!'>. 4.XO.OOO - Rs. 3.20.f)OO) = R..,. 1.60.000

c. Aswmin,g that the Inventory Turnover Ratio i, COJllIHI1(xl ha!'>t:d on only the Year-L'nd
II1\'el1lor). C(b\ or Sale.., can he computed a~ pre..,cllled hel< 1\\ .

Inventory. TunlOvel} = [Cll"t of sales] . X = [cust llf Sale!'> J'"


Ratl(l Inventory .. - Rs. I.W,()()O

Cost of Sales =(Rs. 1.60.000 x H) =Rs. 12.XO.O()()

Since the Gross Profit is given at 1Wk of Sales, Co..,t of Sales works out to HWIc of sales.
Therefore. Sales Revenue =(Rs. 12.HO.OOO + HO(/~ ) = R.'" I ().OO.OOO. and
Gross Prollt = R:-. ~.20.000 (i.e .. 2()li( 01 R!'>. 16.00.0(0)

d. As'mllling that all the Sales were made on credit ba!'>i!'>. the amount of Debtors can hc
determined as shown below.

~r~dit ~~I.~)~v~d } _ [<Average) Trad~ Dehtors ] Trade Dehtor!'> ~


(I.\:: .. A \(~I ae- c - Monthlv Credit Sale.., :. 1.5 months = [ {RS. 16.()o.OOn}
Collection Period) -
12 months

:. Trade Debtors = [1.5 x (Rs. 16,00.000 + 12)1 = [ 1.5 x 1,33.333) = R!'>. 2.00.000

ClIITent A..,sets R!'>. 4.XO.O(){J


Less: Inventory " .Rs. 1.60.000
Debtors 2.()O.OOO 3.60.000

:. Cash 1.20,000
e. Fixed Assets to Net Worth = l)()lk = 0.9
Net Worth + Current Liabilities = Fixed Assets + Current Assets
Net Worth - Fixed Assets =Current Asset!'> - Current Liabilities
:. :'-Jet Worth -- Fixed A!'>!'>et!'> = Working Capital
( 1 - 0.9) = Rs. 2.XO.OOO
0.1 = Rs. 2.HO.OOO

.. 1 =(Rs. 2.XO,nOo + 0.1) =Rs. 28.00.000 =Net Wnrth


:. Fixed Assets = (0.9 x Rs. 2X.00.OOO) = Rs. 25.20.000
Share Capital + Reserves = Net Worth
Share Capital + 40% of Share Capital = Net Worth
140% of Share Capital =Rs. 1X,OO.OOO
Management Accounting : 176

:. Share Capital =(Rs. 28,00.000 + 140%) =Rs. 20,00,000


:. Reserves~~ [Net Worth - Share Capital] = IRs. 28,00,000 - Rs. 20.00,000] = Rs. 8.00.000
.• Balance Sheet as on June 30,1996
Amount Amount
Capital and Liabilities Assets and Properties
R~. Rs.
Net Worth: Fixed Assets 25,20.000
Share Capital 20.00,000 Current Assets:
Reserves 8.00.000 Stock 1.60,000
Current Liabilities 2.00.000 Trade Debtors 2.00.000
Cash 1.20.000 I
30.00,000 30.00.000

Illustration: 3.42
The following are the ratios extracted from the Balance Sheet of a company as at December
31. 1990. Draw up the Balance Sheet of the firm.

Current Liabilities: 1.0 Gross Profit a~ percentage of Sales: 20%


Current Assets: 2.5 Debt Colleetion Period: 2 months
Working Capital: Rs. 3.00,000 Shareholders Capital: Rs. 5,00.000
Liquidity Ratio: 1.5 Reserve and Surplus: 2,50,000
Stock Turnover Ratio: 6 Fixed Assets Turnover is: 2
[ICWA (Fill), JUlle 1992J
Solution:
a. It is known that, Working Capital = [Current Assets - Current Liabilities]
Rs. 3,00.000 = (2.5 - 1)= 1.5
Hence. I = (Rs. 3.00.000 + =Rs. 2,00.000 =Cun'ent Liabilities
1.5) I" .'

:. Current Assets =(Rs. 2.00.000 x 2.5) =R~. 5,00.000

b. Assuming each of Bank Overdraft and Prepaid Expenses as equal to zero. value of Year-end
Inventory can be computed as shown below.

Liquidit;L _ ( Liquid Assets


Ratio f - Current Liabilities
J :. 1.5 =l
rRs. 5,00,000 - Closing Stoc~l
Rs. 2.00,000 :J
.
Therefore, (1.5 x Rs. 2,00,000) = [Rs. 5.00.000 - Closing Stock]
Accounting Ratios: 177

Closing Stock=(Rs. 5,00,000 - Rs. 3,00.000) =Rs. 2,00,000


Liquid AsSCh =(Rs. 5.00.000 - Rs. 2J)0.000) = Rs ..100.000

c. StockTurn~ver} _ rCost of Goods Sold


Ratio - l Closing. Stock
J (Assumed to be based on closing stock
instead of Average Inventory)

(Cost of Goods Soid"l


6= l Rs. 2.00.000 J
Therefore. Cost of Goods Sold = (Rs. 2,00.000 x 6) = Rs. 12.00.000
Gross Protit = 20% and therefore. Cost of Goods Sold Ratio = HOCk (i.e .. 1 - 20%)

Hence. Sales
RevcnuJ
1.. = r Cost of Goods Sold
lCost of Goods Sold Ratil~
I=-[ Rs. 12.00.000 J
XOC'k
=Rs. 15,00.000

Therefore. Gross Profit = Rs. 3,00.000 (i.e .. Rs. \5.00,000 - R~. 12.00.0()(l)

d. Fixed Assets Turnover (assumed to be based on Cost of Goods Sold instead of Sales
Revenue) .

.., = [cost of Go~ds


Sold rh. 12.00.000J :. 2 x Fixed Assets = Rs. 12.00.000
- FIxed AS!-.L·t~

:. Fixed Assets =(Rs. 12.00.000 + 2) = Rs. 6.00.000


e. Assuming that the Debt Collection Period was computed on the basis of only the closing
balance of Debtors and the entire Sales were on credit basis, it (i.e., closing balance of
Debtors) can be computed as shown below.

Debt Collection
Period. 2 monthif -
1.._ [. Closing Balance of Ikhlors
Average Monthly CrL'llil Sales
J . . ..
2_ [
-
Debtors "I
(Rs. 15.00.000112~

. -- (RS. 15.00.000
Hence. De b tOlS 12 x 2J -- Rs. ?-. 5() .(lOa

[Liquid Assets Rs. 3.00.000 - Rs. 2.50.000 Debtors] = Rs. 50.000 Cash (Balance)
Management Accounting : 178

Balance Sheet or ... Companv as on December 31,1990


Amount 1 . Amount
Capital and Liabilities I. R!-I. I
. Assets arid Properties
Rs.
Shareholdcrs Net-worth: I H \lo't! Assets 6.00.000
!
I

I Share Capital 5.00.000 ! CUlTcnt Assets: i


I
I

i
I Reserw and Surplus 2.S0.()(I() Stock 2.00.000
I
I Long-term Debt (b"f:,!\~ing amount) I . 1.50.00() Debtors 2.50.000 !
I Current Liabilitie< • I 2.00.000 Cash at Bank 50.000

I !i 11.00.000 11.00.000

Illustration: 3 ....3

From tlh' following information relating to Wise Limited, you arc required to prepare its
summarised Balancc Sheet. ''.

(a) Curi'cnt ratio 2.5 (g) SaleslDcbtors ratio 6.0


(b) Acid Test ratio l.S (h) ReseJ'\ es/Capital ratio 1.0
(L) Gross -Pn 'fit/Sales ratio 0.2 (i) Net worth/Long term loall ratio 20.0
I d I :"-Jct working capital/Net worth ratio 0.3 (j) Stock velodty 2 months
Ie) Sales/Ncr tixed assets ratio 2.0 (k) Paid up share capital Rs. )0 lakhs
(f) Sales/Net worth ratio 1.5
fC;l (1111), .Way 199()]
Solution:

(a) RC!-Icrve to C.lpital Ratio = .1 time

Reservcs
:. I = [ Capital Rs. IO lakh
J Hence. Resern!s =(R!-I. 10 lakh x I) = Rs. IO lakh

IShare Capital Rs. 10 lakh + Reserves Rs. Hilakh) =Net Worth Rs. 20 lakh.
(b) Net Working Cupital1<~
Net Worth Ratio f
=0 ~
..
= [ Net Working Capitul
Net Worth. Rs. 20 lakh
J
:. (~ctl Working Capitul =(Rs. 20 lakhs x 0.3) =Rs. () lukh
(c) Working Capirul =CUlTcnt AS!-It'ls - Current Liabilities
Rs. 6.00.000 =(2.5 - I) = 1.5
Accounting Ratios: 179

:. 1 = [Rs. 6.00.000.;- 1.5] = Rs. 4,00.000 = Current Liabilities.

:. Current Assets = (Rs. 4.00.000 x 2.5) = Rs. 10.00.000


(d) Assuming Bank Owrdraft = O. and Pre-paid Expenses =O. Liquid Assets and Closing Stock
can be computed as presented below.

L·Iqlll'd R' I 5=[


allo... Liquid Asseh J
CUI1"ent Liabilitie~. Rs . ..j. lakh

:. Liquid Assets = (Rs. 4 lakh x 1.5) = R~. 6 lakh


:. Stock = [Current Assets Rs. IO lakh - Liquid Assets Rs. 6 lakhJ = R~. 4 lakh
ee) Sales to N~t} = 1 5 = r, Sales Revenue
Worth RatIo . l,Net Worth. Rs. 20 lakh
J
Hence, Sales Revenue = (Rs. 20 lakh x 1.5) = Rs. 30 lakh
Gross Protit =(Rs. 30 lakh x 0.2) =Rs. 6lakh
Cost of Goods Sold = (Rs. 30 lakh x OJH = R~. 24 lakh

Sales to Debtors} _ (sales. Rs. 30 lakhJ :. [Debtor~ x 6 J = Rs. 30 lakh


Ratio. 6 - Debtors :. Debtors =[30 lakh + 61 = Rs. 5 lakh
Liquid Assets Rs.61akh
Less: Debtors Rs.5 Iakh
:. Ca-;h and other Current Assets Rs. I Iakh
(g) Sales to Net Fixed} _? _ rSales, Rs. 30 lakhl
Assets Ratio - - - LNet Fixed Assetsj
[2xNet Fixed Assets] = Rs. 30 lakh
Hence. Net Fixed Assets = (Rs. 30 lakh + 2) = Rs. 15 lakh

(h) Net W0l1h to LOng-term} -"0 _ rNet Wurth. Rs. 20 lakh]


Loan Ratio - - - L
Long Tl'rm Loan

[2xLong-term Loan I = Rs. 20 lakh


Hence. Long Term Loan = (Rs. 20 lakh + 2()1 = Rs. I lakh

lIalaoce Sheet of Wise Ltd., as 00 ..... .


Capital and Liabilities !I Amount Amount
Assets and propertie~:- - -Rs.
----J
I Rs.
.
Shareholders' Fund: Net Fixed As~ets IS.OO.OOO
Shure Capital 10.00.000 Current Assets:
i
Manage'ment Accounting: 180

Reserves 10,00.000 Stock 4.00.000


Long term Loan 1,00.000 Debtors 5,00.000
Current Liabilities 4.00.000 Cash and other Current Assets 1,00.000
25.00.000 25.00.000

Illustration: 3.44

Complete the following annual tinancial statements on the hasis of ratios give'n below,

I an dL oss Accoun t Iior th e_year en d ed30thJ une, 1990


Profit
To Cost of Goods Sold 6.00.000 I
By Sales I 20.00.000
To Operating Expenses - I
i

~
I
To Earnings before Int, and Tax - !

To Debenture Interest 10.000 By Earnings before Interest


To Income Tax - and Tax -
To Net Profit -
--

Balance Sheet as at 30th June, 1990


Rs. Rs.
Net Worth: Fixed Assets
Share Capital - Cun'cnt A~sets: i i
Reserws and Surplus - Cash -
I DC;( Dchentures - Stock
I -
Sundry Creditors 60.0UO .. Debtors I
I
35.000

I
Net profit to sales = 5% Inventory turnover (based on cost of goods sold) = ]5 times
Current Ratio = 1.5 Share capital 10 reserves = 4 : I
Return on net worth = 20~.f Rate of Income tax =50% lCA (lnt), November 1990J
Accounting Ratios: 181

Solution:
(a) Net Profit to Sales =5%;
:. Net Protit = 5% of Rs. 20,00,000 = Rs. 1,00.000
(b) Income Tax Rate = 50%
Net Protit (after Tax) =50% of Net Profit before Tax =Rs. 1,00,000
:. Net Profit (before Tax) = (Rs. 1,00,000 + 50%) = Rs. 2,00,000
Income Tax =50C'k of Rs. 2,00.000 =Rs. 1.00,000
(c) . Net Profit (after Tax) = Rs. 1,00,000
Add: Income Tax 1,00,000
Taxable Income 2,00,000
Add: Debenture Interest 10,000
:. Earning before Interest and Tax (EBIT) 2.10.000
(d) Sales Revenue Rs. 20.00,000
Less: Cost of Goods Sold 6.00,000
EBIT 2.10,000 H.IO,OOO
:. Operating Expenses 11.l)O,OOO

(e) Invento~ Turnover}


Ratio
= [cost of Goods Sold
(Average) Inventory
J= 15 =[ Rs. 6,00.000
Inventory
J
:. [15 x Inventory] = Rs. 6,00,000
:. Inventory =(Rs. 6,00,000 + 15) = Rs. 40,000
(f) Debenture Interest = Rs. 10,000 and the rate of interest is 10%. Therefore, the Debenture
Capital comes to Rs. 1,00,000 (i.e., Rs. 10,0000 + 10%)

(g) Return on 1
_? c. _ _ [Net ProHt
Net Worthf - _010 - 0.2 - Net Worth
J_ L
-
rRs. 1,00,0001
Net Worth j

:. Net-Worth = (Rs. 1,00,000 + 0.2) = Rs. 5.00,000


Net worth of Rs.5,00,000 comprises of Share Capital. and Reserves and Surplus in the ratio of 4
: I. Therefore, Share Capital = Rs. 4,00,000 Ii.e., (Rs. 5.00.000 + 5) x 4] and Reserves and
Surplus = Rs. 1,00.000 [i.e., (Rs. 5.00.000 + 5) x II.
Management Accounting: 182

· = I .5 = [current Assets
(h) C un'en t RattO ...
Current Liabilities
J [ = Current Assets
Rs. 60,000
J
:. Cun'ent Assets =(Rs. 60.000 x 1.5) = Rs. 90.000
Less: Debtors Rs.35.000
Stock 40.000 75.0UO
:. Cash 15,UOO
(i) Fixed Assets = [(Net Worth + Debenture + Creditors) - (Current Assets)]
= l(Rs. 5,00.000 + Rs. 1.00,000 + Rs. 60,000) - (Rs. 90,000)] = Rs. 5,70,000
Dr Profit and Loss Ale for the year ended June 30, 1990 Cr
Amount Amount
Parti cui ars Pat1iculars
Rs. Rs.
iTo Cost of goods sold 6.00.000 By Sales 20,00,000
I To opemti~g expenses 11,90,000
To Earnings before interest and tax 2.10,000
20,00,000 20.00,000
To Debenture Interest 10,000 By Earnings before Interest and
Tax 2,10,000
To Income tax 1,00.000 I
To Net Profit . 1.00.000 !

!. 2.10,000 I!
2,10,000 I
I :

Balance Sheet as at Jqne 30,1990


I
:
I
Liabilities
Amount
R~.
I Assets I Amount
R!'..
Net Worth: Fixed Assets 5.70,000
Share Capital 4,00.000 Current Assets:
Reserve and Surplus 1,00,000 Inventory 40,000
101k Debentures 1.00,000 I Debtors 35.0nO
Sundry Creditors 60,000 Cash 15,000
6.60.nOO 6,60,000
I I
Accounting Ratios: 183

Illustration: 3.45
The Directors of Bamcha Enterprises Ltd .. ask you to ascertain the following from the
following information.

a. Proprietors funds b. Fixed asset~

c. Closing debtors d. cto~ing creditor~

e. Closing stock f. Share capital


0
e' Cash and bank balance~

I. Inventory turnover ratio is 6 times; Year-end debtors are outstanding for 2 months; Year-
end creditors arc outstanding for 73 days.
2. Ratios of cost of goods sold to (a) Proprietors funds is 1: I; and (b) Fixed assets i!; 4: I.
3. Ratio of gross profit to ~ales is 1OCk.
4. Closing stock i~ greater than the opening stock by Rs. 10.000.
5. The gross profit for the year ended 31 s[ March. Rs. 1.20.000.
6. Reserves and surplus appearing in the balance sheet as at 31 st March. I99X total to Rs.
40,000. {('A (1111), May 1998J
Solution:

I. Gross Profit = Rs. 1,20.000 and the Gross Protit Ratio = 1()f/t. Therefore.
Sales Revelluc
Grms Profit
= [ Gross ... =
Prolll RatiO
J r
....
1.20.()()0~ = R~ 6 ()O oon
R....
~ (
~O !c .. . •

Cost of Gupds Sold = [Sale~ Revenue - Gro~" Profit!


= (Rs. 6.00.000 - R.... 1.20.(100) = R~. -LXO.OOO

2. Inventory Turnover} _
Ratio -
[cost of Goods SOld] _ _
A verage Stock - 6 -
[ Rs.4.80.000
A verage Stock
J
(Average Stock x 6) = Rs. -1-.80.000: :. Average Stock = (Rs. 4.80.000.;- 6) = Rs. 80.000
Average Stock = [(Opening Stock + Clo~ing Stock).;- 11 = Rs. 80.000
:. (2 x 80.0(0) = [Opening Stock + (Opening Stock + Rs. 10.0(0)]
[2 x Opening Stock) =[Rs. 1.60.000 - Rs. 1O.000j = Rs. 1.50.000
:. Opening Stock = [Rs. 1.50.000.;- 2) = Rs. 75.000
:. Closing Stock =[Rs. 75,000 + Rs. J(>.OOOI = Rs. 85.000
Cost of Goods Sold = [Opening Stock + Purcha~es - Clo~ing Stock[
Management Accounting : 184

:. Purchases = (Cost of Goods Sold + Closing Stock - Opening Stock)


= (Rs. 4,RO.000 + Rs. H5.000 - Rs. 75,0(0) = Rs. 4.90.000
(Assum~d to be on credit basis)

3. Average COllection} _ r. Year-end Debtors "1 INote: assumed all sales on credit basis]
Period (months) - ~onthly Credit SalesJ

:. :2 = year-end D~'htors
Rs.6.00.000J
[ ( 12 months
J :. ClOSing}
Debtors
= rRs. 6,00,000 x 2J
l 12 months
=Rs. 1,00.000

4. Average Payment } _ ( Closing Creditors ]

:.
Period (days) - Daily Credit Purchase

73 days = Lc[~~~~.~~~~~"J
365 days
ClOSing} -- ( 7 3- x (4.90.000
Creditors 365 J J-
- Rs. 98,000

5. It is given that the Cost of Goods Sold to Proprietors' Fund is 2 : I. Therefore,

? - [ Rs. 4.80,000 ] :. (2 x Proprietors' Fund) = Rs. 4,80.000


- - Proprietors' Fund :. Proprietors' Fund = (Rs. 4,80,000 + 2) = Rs. 2,40,000

[Proprietors' Fund, Rs. 2,40,000 - Reserves. Rs. 40.000] = Share Capital, Rs. 2.00,000
6. Cost of Goods Sold to} _ . _ _ [RS. 4,80,000 ]
Fixed Assets - 4. I - 4 - Fixed Assets

:. (4 x Fixed Assets) = Rs. 4,80.000


:. Fixed Assets = (Rs. 4,80,000 + 4) = Rs. 1,20,000

t ., as on M arc h 31 , 1998
a ance Sh eetofBh arucha E nterprlses Ld
HI
Amount Amount
Capital and Liabilities Assets and Liabilities
Rs. Rs.
Proprietors' Fund: Fixed Assets 1.20,000
Share Capital 2,00,000 Current Assets:
Reserves and Surplus 40,000 Stock 85,000
Trade Creditors 98,000 Debtors 1,00,000
Cash and Bank Balance
(Balancing figure) 33.000
3,38,000 3,38,000
Accounting Ratios: 185

Illustration: 3.46

From the followi.ng information and ratios, prepare the Profit and Loss Account for the
year ended 31st March, 1994 and the Balance Sheet as on that date of MIs Stan and Co., an export
company.

Current Assets to Stock = 3:2 Fixed Assets Turnover Ratio = 1.20


Current Ratio = 3.00 Total Liabilities to Net Worth = 2.75
Acid Test Ratio = 1.00 Net Working Capital = Rs. 10 lakhs
Financial Leverage =2.20 Net Profit to Sales = 10%
Earnings per Share (each of Rs. 10) = Rs.IO.00 Variable Cost =60%:
Book value per Share =RsAO.OO Long Term Loan interest = 12%
A verage Collection Period Taxation = Nil
(Assume 360 days in the year) = 30 days Stock Turnover Ratio = 5.00
lCA (Int), November 1994}
Solution:

(a) Working Capital =(Current Assets - Current Liabilities); and Current Ratio =3
:. Working Capital, Rs. 10 lakh = (3 - I) = 2 :. 1 = (10 lakh 7- 2) = Rs. 5 lakh
That means, Current Liabilities = Rs. 5,00.000
Current Assets =(3 x Current Liabilities) = (3 x Rs. 5 lakh) = Rs. 15 lakh

(b) Current Assets }


to Stock
_ . .
- 3 . 2,
(Current Assets]
L Stock = [2] _lr
2 -
RS . 15 ,00,000J·
Stock

(Stock x 3) = (Current Assets, Rs. 15 lakh x 2)

Stock = [ Rs: 15 ~akh x 2 J = Rs. 10 lakh


(Current Assets. Rs. 15 lakh - Stock, Rs. 10 lakh) = Rs. 5 Iakh, Liquid Assets .

(c) Assuming that the SU)ck Turnover Ratio is computed on the basis of Sales (instead of Cost of
Goods Sold) and Closing Stock (instead of Average Stock), Sales Revenue may be
ascertained as detailed below.

Stock Turnove~ _ .[ Sales Revenue J


Ratio,S f - Stock, Rs. 10 lakh

Sales Revenue = (5 x Rs. 10 lakh) = RS',50 lakh


Management Accounting: 186

Variable Cost = (60~ of Rs. 50 lakh) = Rs . .30 lakh


Contribution = (Sales Revenue - Variable Co~t) = (Rs. 50 lakh - Rs. 30 lakh) = Rs. 20lakh

(d) Net Profit (i.e., Net Profit before Tax) to Sales = 1Olk- of Rs. 50 lakh = Rs. 5 lakh
Profit before
Financial } = Interest and Tax, PBIT
Leverage. 2.2 (Profit before Tax,
PBT), Rs. 5 lakh

:. PBIT = (2.2 x Rs. 5 lakh)= Rs. 11 lakh


:. Interest (on Long-term Loan) = [PBIT - PBT] = rRs. 11 lakh - Rs. 5 lakhJ = Rs. 61akh
Long-term Loan = (Rs. 6 lakh Interest + 12%, Interest Rate) = Rs. 50 lakh
(e) Average CollectiOl~ _ r Trade Debtors] (Note: Assumed all Sal~s were on credit
Period f .- lDaiIy Credit Sales basis)

:. Trade Debtors = [ 30 x 50lakh]


360 = Rs. 4.16.667

Total Current Assets Rs. 15.00,000


Less: Stock 10,00,000
Debtors 4.16.667 Rs. 14.16,667
:. Other Current Assets Rs.83.333
(t) Total Liabili'ties to Net-Worth = 2.75
275:';; rLong-term Liabilities, Rs. 50 lakh + Current Liabilities Rs. 5 lakhJ
. L Net Worth -- .

(Net Worth x 2.75) = Ri.· -55 lakh


Net Worth = (Rs. 551akh + 2.75) = Rs. 20 lakh

(g) .' I Fixed Assets 1_ (Sales Revenue, Rs. 50 lakhl


Turnover Ratio, 1.20f - Fixed Assets j
(1.20 x Fixed Assets) = Rs. 50 lakh
:. Fixed Assets =(Rs. 50 lakh + 1.20) =Rs. 41,66,667 lakh
Accounting Ratios : 187

(h) [Contribution - Fixed Cost (excluding Interest)] = PBIT (Profit before Interest and Tax)
:: Fixed Cost (excluding Interest) = (Contribution - PBIT)
=(Rs. 20 lakh - Rs. 11 lakh) =Rs. 9lakh

(i) Number of Shares = [ Net Worth


Book Value per Share
J - [RS. 20,00,000
- Rs. 40
J =50,000
(j) [Net Worth, Rs. 20 lakh] - [Share Capital, (50,000 x Rs. 10)] = [Rs. 20 lakh - Rs. 5 lakh]
= Rs. 15 lakh, Reserves arid Surplus

·Dr I an d Loss Ajcof Mfs Stan & C0., Iior the year end ed Marc h31 , 1994
Profit Cr
Amount Amount
Particulars Particulars
Rs. Rs.
To Variable Cost of Sales 30,00,000 By Sales Revenue 50,00,000
To Contribution cld 20,00,000
50,00,000' 50,00,000
To Fixed Cost 9,00,000 By Contribution bId 20,00,000
To Interest 6,00,000
To Net Profit 5,00,000
20,00,000 20,00,000

Balance Sheet of MIs Stan & Co., as on March 31, 1994


Amount Amount
Capital and Liabilities Assets and Properties
Rs. Rs.
Share Capital (50,000 share of Fixed Assets 41,66,667
Rs. 10 each) 5,00,000
Current Assets:
Reserves and Surplus 15,00,000
Stock 10,00,000
Long-term Liabilities 50,00.000
Debtors 4,16,667
Current Liabilities 5,00,000
Others 83,333
Miscellaneous Assets (balance) 18.33.333
75,00,000 75.00,000 j
Management Accounting : 188

Illustration: 3.47

Calculate the Average Collection Period form the following details by adopting 360 days
to an year:

Average Inventory = Rs. 3,60,000 Gross Profit Ratio = 10%


Debtors = Rs. 2,30,000 Credit Sales to Total Sales = 20%
Inventory Turnover Ratio = 6 leA (lilt), May 1993J

Solution:

(a)' Inventory 1_
[Cost of Goods Sold
• Turnover RatiJ" - l Average Inventory
J- -
6 = [cost of Goods SOldl
Rs. 3.60.000 J

:. Cost of Goods Sold =(6 x Rs. 3,60,000) = Rs.21,60,000


Add: Gross Profit [( 10 + 90) x 21.60.000] 2.40,000
:. Sales Revenue 24,00,000
Credit Sales =[20% of Rs. 24,00,000] = Rs. 4,80,000
Daily Credit Sales = (Rs. 4,80,000 + 360) = Rs. 1,333.33

(b) Average COllection} _ [ Trade Debtors


Period - Daily Credit Sales
J_ -
rRs.2,30,0001
lRs.l,333.33J = 172.5 days
Illustration: 3.48
Mr. T. Munim is made an offer by the promoters of Svargiya Enterprises Ltd., to invest in the
product of the company by purchasing a substantial portion of the share capital. He is promised
good returns by way of dividends and capital appreciation. Mr. Munim desires that you compute
the following ratios for the financial analysis. Working should form part of your answer: (a)
Return on investment ratio, (b) Net profit ratio, (c) Stock turnover ratio, (d) Current ratio, and
(e) Debt equity ratio. The figures given to him are as under (Rs. ·OOOs).
Sales 16,000 Share capital 5,000
Raw materials consumed 7,800 Reserves and Surplus 1,500
Consumables 800 Secured term loans 12.000
Direct labour 750 Unsecured term loans ·1,500
Other direct expenses 480 Trade creditors 3,350
Administrative expenses 1,200 Investments 400
Selling expenses 260 Inventories 6,000
Accounting Ratios: 189

Interest 1,440 Receivable 3,700


Fixed assets 14,000 Cash at hand and bank 100
Income tax 50lK ProvisIOns 650
Depreciation 700 Other current liabilities 200
lCA (Inl), November 1988J
Solution
Income Statement
Amount
Particulars
(Rs.OOO)
Cost of Raw material consumed 7,800
Consumables 800
Direct labour 750
Oth~r direct expenses 480
Direct Costs 9,830
Ind.irect Expenses:
Administrative expenses 1,200
Selling expenses 260
Depreciation 2.160
Total Operating Cost 11,990
Sales Revenue 16,000
Operating Profit 4,010
Less: Interest 1,440
Profit before Tax 2.570
Less: Income Tax (at 50%) 1,285
Net profit (after Tax) 1.285

Balance Sheet
Amount Amount
Capital and Liabilities Assets and Properties
(Rs.OOO) (Rs.OOO)
Share Capital 5,000 Fixed Assets 14,000
Reserves and Surplus Investments
1,500 400
Current Assets:
I Long Term Loans: 6,500 Inventories 6,000
I Secured Loans 12,000 Receivables 3,700
I Unsecured Loans 1,500
Cash and Bank 100
Management Accounting: 190

Current Liabilities: 13,500 9,800


Trade Creditors 3,350
Provisions 650
Others 200
4,200
24,200 24,200

Capital Employed = (Fixed Assets + Investments + Working Capital)


= [Rs. 1,40,00,000 + Rs. 4,00,000 + (R~. 98,00,000 - Rs. 42,00,000))
= [Rs. 1,44,00,000 + Rs. 56,00,000] = Rs. 2,00,00,000
(a) Investment
Return on } = [ EBIT x 100 ] = [ Rs. 40, 10,000 x 100] =20.05%
Ratio Capital Employed 200,00,000

(b) Net p.rofitL= ( EBIT x 1001 = [RS. 40,10,000 x IOOJ= 25.0625%


RatIo f LSales Revenue ) Rs. 1,60,00,000

(c) StoCk}
T urnover =
[ Cost of Goods Sold
.
] RS. 98,30,00~I 638 .
= [ Rs. 60,00,00oj =. times
Ratio Average or Closmg Inventory

(d) current}
Ratio
(Current Assets
= LCurrent Liabilities =
J [RS.98,00,000J
Rs. 42,00,000 =2.33 : 1
(e) Debt-EqUity} _ (LOn~-term) Debt] _ rRs.1,35,00,00Q) _ 2 077 . 1
Ratio - Equity - L Rs. 65,00,000 J -. .
Illustration: 3.49
The Balance Sheet of Y Ltd., stood as follows as on:
Liabilities 31-3-95 31-3-94 Assets 31-3-95 31-3-94
Capital 250 250 Fixed Assets 400 300
Reserves 116 100 Less: Depreciation 140 100
Loans 100 120 260 200
Creditors and other Investment 40 30
. Current Liabilities 129 25
Stock 120 100
Accounting Ratios: 191

Debtors 70 50
Cash/Bank 20 20
Other Current Assets 25 25
Misc. Expenditure 60 70
. 595 495 595 495
\

You are given the following informatio~ for the year 1994-95:
Sales 600 Provision for Tax 60
PBIT 150 Proposed Dividend 50
Interest 24
All the figures given above are rupees in lakhs. From the above particulars, calculate for
the year 1994-95:

(a) Return on Capital Employed Ratio. (d) Current Ratio.


(b) Stock Turnover Ratio. (e) Proprietary Ratio.
(c) Return on Net Worth Ratio. leA (lilt), November 1995J
Solution:
(a) 31-3-1994 31-3-1995
Total assets (Rs. lakh) RS.495 595
Less: Current Liabilities 25 129
Capital Employed 470 466

:. Average}
Capital =( R,. 470 lakh ~ Rs. 466lakh ] = (RS. 9361akhJ = Rs. 4681akh
2
Employed

( Profit before "\


I
I
1 - 32 05%
t
Return on Capitai'l Interest and Tax rRs. 150 lakh x 100
Employed Ratio f Average Capital X 100 ! = lRs. 468 lakh ) - .
I

Employed

(b) Based on Sales Revenue,

TStoCk} rSales Revenue] Rs. 600 lakh lJ= ( Rs. 600 lakh J
uRrn~ver
atto
= LA verage Stock =[ Rs. 100 lak\+ Rs. 120 lakh Rs. 110 lakh

= 5.45 times
Management Accounting: 192

(c) 31-3-1994 31-3-1995


Capital (Rs.lakh) 250 250
Reserves 100 116
350 366
Less: Miscellaneous Expenses 70 60
;
:: Proprietors' Fund or Net worth 280 306

:. ::~e } =[ Rs. 28~ lakh; Rs. 3061akh J = [RS. 5~61akhJ =Rs. 2931akh
Profit after Tax = (pBIT Rs. 150 lakh - Interest Rs. 24 lakh - Tax Rs. 60 lakh)
=Rs. 661akh
:. Return on1 _ [Profit after Tax
Net ~orthf - Net Worth x toO
J r. Rs 66 lakh
= l Rs."293 lakh
~
x IOOj =22.53%
(d) Current Assets as on 31-3-1995

Stock Rs. 120 lakh Current Liabilities Rs. 129lakh


(on 31-3-1995)
Debto~s 701akh
Cash 201akh
Others 251akh
235lakh

. rlCurrent Assets J
:. Current Ratio = Current Liabilities =
[RS. 235 lakhJ
Rs. 129 lakh
.
= 1.82 times

(e) Proprietary
Ratio
1
f
= r. Proprietary Fund ")
l!otaI Assets (excluding Mis.exl1 =
Rs. 306 lakh .
[ Rs. 595 lakh - Rs. 60 lakh
J
RS. 306 lakh J
= ( Rs. 535 lakh = 0.572 time
Accounting Ratios: 193

Illustration: 3.50
The following are the summarised Profit and Loss Account for the year ending 31.3.1999
and the Balance Sheet of Krishna Company Limited as on 31.3.1999.
Trading, and Profit and Loss Account
To Opening Stock Rs.lO,OOO By Sales Rs.l,OO,OOO
To Purchases 55,000 By Closing Stock 15,000
To Gross Protit c/d 50,000
1,15,000 1.15.000
To Administrative Expenses 15.000 By Gross Profit bid 50,000
To Interest 3.000
To Selling Expenses 12.000
To Net Profit 20,000
50,000 50,000

Balance Sheet
Capital Rs.I.OO,OOO Land and Building Rs.50.000
Profit and Loss Nc 20,000 Plant and Machinery 30,000
Creditors 25,000 Stock 15,000
Bills Payable 15,000 Sundry Debtors 15,000
Bills Receivable 12,500
Cash in hand and Bank 17,500
Furniture 20,000
1,60,000 1,60,000

Additional Information: Average Debrors Rs.12.500; Average Credit Purchases Rs.40,OOO.


You are required to calculate: (a) Stock Turnover Ratio, (b) Debtors Turnover Ratio.
(c) Sales to Working Capital. (d) Return on Shareholders Investment. (e) Current Ratio, (f) Acid
Test Ratio, (g) Net Profit Ratio and (h) Operating Ratio. {Mangalore Uni., B.Com., May 2000J
Solution:
1. Cost of Goods Sold = [Sales - Gross Protit] = fRs.I.OO,OOO - Rs.50,OOOl = Rs.50.000

2. A;:::e } ~ t
Opening Stock + Closing Stock
2
J~ [ Rs.IO.OOO J
~ Rs.15.000 ~ Rs.12.500
Management Accounting : 194

3. Sundry Debtors Rs.IS,OOO 4. Creditors Rs.2S,OOO


Bills Receivable 12.500 Bills Payable IS,OOO
Cash 17,SOO Current Liabilities 40.000
Liquid Assets 4S.000 Current Assets 60,000
Stock IS,OOO Working Capital 20,000
Current Assets 60,000 6. Cost of Goods Sold Rs.SO,OOO

S. Capital Rs.I,OO.OOO Add: Operating expenses:


Profit and Los~ Alc balance 20,000 Administration 15,000
Selling 12,000
Shareholders' Investment 1.20,000 :. Operating Cost 77,000

a. Stock Turn} = (Cost of Goods Sold) = Rs.SO.OOO = 4 times


over Ratio l Average Stock j Rs.12,s00

b Debtors' Turn}_
over Ratio -
Net Credit Sales
Average Debtors
j =tRS.I.OO.OOO~ =
Rs.12,SOO
.
8 tImes

c. Sales to W01'k'In~ =
Capital Raito
o} Sales Revenue
. .
WorkIng CapItal
j =
t RS.I.OO.OOO~
Rs.20,OOO
.
= S times
.

d Return on Share} = ~
Net Profit x 100 = ~ ~ ~
Rs.20.000 x 100 = 16.67%
holders Investment Shareholders Investment Rs.I.20.000

e. current} -_ [ Current .Assets


'"
~ -_ ~ RS.60.000j _
-lSI
.
Ratio Current Lmbllttles Rs.40,000 . .

f. A~~t~~st ~ cu~e::~~~:~:;es j ~
} t::::::} 1125: I

g Net Profit} =
Rntin
t Net Profit
Sales Revenue
~ U
x 100 = Rs.20.000 x 100
Rs.I.OO.OOO
~ =20%

h operatina}
. e = ~operating
Cost X
~ =~Rs.77.000
100 ~
x 100 = 77%
Ratto Sales Revenue Rs.I.OO.OOO
Accounting Ratios : 195

Illustration: 3.51
Rearrange the following statements in a form suitable for analysis and calculate:
(a) Liquidity ratio, (b) Current ratio. (c) Debt Equity ratio, (d) Capital Gearing ratio, and
(e) Capital Turnover ratio.
Condensed Balance Sheet
1995 Rs. 1996 Rs. 1995 Rs. 1996 Rs.
Creditors 20,000 16,000 Bank 15.380 26.020
Bills Payable 12,750 6.500 Debtors 11.260 11,710
Debentures 1.00,000 1.00.000 Stock 56.160 49.460
Reserves and Profits 67,250 84.500 Fixed Assets Less Depr 2.17.200 2.19.810
Paid-up Capital 1,00,000 1.00.000
3.00,000 3.07.000 3,00,000 3,07,000
Sales 1,80,000 1.95,000
[Kllvempu Ulli., B.Com.• May 1999J
Solution:
Restated Balance Sheets of ..... Company as on December 31,1995 and
I . P urpose
1996 f'or A nalysis
Amount IRs) as on
Rs. as on December 31,
Particulars December 31,
•...... 11)95 1996 1995 1996
Paid-up Captial 1,00,000 1,00,000
Reserves and Profits 67,250 84,500
:. Proprietors' Fund 1.67,250 1,84,500
Debentures 1,00,000 1,00,000
:. Capital employed 2.67.250 2,84,500
Fixed Assets [Less:Depreciation] 2.17,200 2,19,810
Bank Balance 15.380 26,020
Debtors 11.260 11,710
Liquid Assets 26.640 37,730
Stock 56,160 49,460
Current Assets 82.800 87.190
Creditors 20,000 16,000 16,000
Bills Payable 17. 7';n 6,500
- - 6.500
Liquid or Current Liabilities 32.750 22.500
Working capital 50,050 64.690
Capital employed 2.67.250 2,84.500
Management Accounting: 196

a. LiqUid} = ( Liquid Assets J (Rs.26.640] ( Rs.37.730]


Ratio l
Liqu~d Liabilitiesj - l
Rs.32,750j l Rs.22,500J
=0.81:1 1.68: 1

b. current} _ ( Current Assets] ( Rs.82.800 J [ Rs.87.190]


Ratio - lCurrent Liabilitie~ - l Rs.32,750j l Rs.22.500 j
= 2.53:1 3.88:1

c. Debt EqUity
Ratio
. t
_
} -
Debt:1: ~
Equity * =
( Rs.1.32.750
[
J CRs.I.22.500 J
Rs.I,67.250J l Rs.I,84,500 J
=0.79:1 0.66:1
[*Debt = Debenture + Current Liabilities; Equity = Proprietors' Fund]

t
Funds Bearingj
d. Capital _ Fixed charge * _ CRs.l.00.000] CRs.I.OO.OOi)
Gearing RatiJ - Equity share- - l Rs.I,67,250J l Rs.l,84,50~
holders' Fund
=0.6:1 0.54:1
[* Funds bearing fixed charges = Debentures, in this case 1

e. Capital L-
Tllrnover RatioJ -
(
l
Sales Revenue
Capital Employed
= CRs.l.80.000] r Rs.I.95.000!
l Rs.2,67,250j l Rs.2.84,50~
=0.67 time =0.69 time
Illustration: 3.52
The following balances as at 31 st March 2000 are' extracted from the books of Pramod
Private Ltd. ~

Dr. Cr.
Equity share capital Rs.5,00,000
10% Preference share capital 2,50,000
Unclaimed dividends 10,000
Proposed dividends 50,000
Land and Buildings Rs.7,OO,000
Plant and machinery 10,25,000
Vechicles 75,000 •
Accounting Ratios: 197

Furniture 1,00,000
Depreciation on Assets (up to 31.3.2000) 4,00,000
Bank overdraft (unsecured) 3,00,000
Long term secured loan from bank 4,00,000
Fixed Deposits (repayable within one year) 5,00,000
Stock in Trade 6,00,000
Sundry Debtors 2,50,000
Cash on hand 5,000
Bank balance 20,000
Preliminary expenses 50,000
Profit and Loss Account 1.00.000
Provision for taxation 1040,000
Sundry creditors 2,00,000
Trade investments 25,000
28,50,000 28,50,000

You are required to prepare a Balance Sheet in the suitable form for analytical purposes
and calculate the following ratios: (a) Current ratio, (b) Proprietary ratio, (c) Dept Equity ratio,
(d) Fixed assets ratio, and (e) Acid Test ratio. [Kuvempu Uni., B.Com., October 2004]
Solution:
Restated Balance Sheet of Pramod Privated Ltd., as on March 31. 2000
~or AnalYllca
If IPo rpose
Particulars Rs. Amount (Rs.)
Equity share capital 5,00,000
10% Preference Share Capital 2.50,000
Profit and Loss Account credit balance 1,00,000
8,50,000'
Less: Preliminary expenses 50,000
Shareholders' Fund 8,00,000
Long term loan 4,00,000
:. Total capital employed 12,00,000
Land and Buildings 7,00,000
Plant and Machinery 10,25,000
Vehicles 75,000
Furniture 1,00,000
Gross Fixed Assets 19,00,000
Less: Depreciation up to March 31, 2000 4,00,000
Management Accounting: 198

Net Fixed Assets 15,00.000


Trade investments 25,000
Current Assets:
Sundry Debtors 2,50,000
Cash on hand 5,000
Bank balance 20,000
Liquid Assets 2,75,000
Stock in trade 6,00,000
8.75,000
Current Liabilities:
Unclaimed Dividend Rs.IO,OOO
Proposed Dividend 50.000
Fixed Deposits 5,00,000
Provision for taxation 1,40,000
Sundry Creditors 2,00,000
Liquid Liabilities 9,00,000
Bank overdraft 3,00,000 12,00,000
.' Working Capital -3,25,000
Total Capital employed 12,00,000

a. current} CCurrent Assets J lRs.8.75.000 J


Ratio = l
lCurrent Liabilitiesj = Rs.12,00,000j = 0.73 : 1

b. proPrietary}
.
R ::tho
=
G Shareholders' Fund
Total (Tan~ible)Assets*
=
GRS.8.00.000
Rs.24,00,000
j =0.33:1

[*Total Tangible Assets.= Net Fixed Assets Rs.15,00,000 + Investments Rs.25,000 +


Current Assets Rs.8,75,000] .

c. Debt E~Uity} _ [ De.bt J :. . [Rs.16.00.000 J _ .


RatIO l
- EqUity j - l Rs.8,00,000 J- 2 . ~
[Debt = Long-term Loans + Current Liabilities; and Equtiy = Proprietors' Fund]
This Ratio is also computed by considering only the long-term debt in the numerator.
Therefore.
Accounting Ratios: 199

Debt EqUity} = (RS.4.00.000J


Ratio Rs.8,OO,oooj= 0.5 : 1

d. Fixed Assets} _ [ Fixed Assets J _CRs.15.00.000 J_ .


Ratio -l
Capital Employedj -l Rs.12.00.000 j - 1.25 . 1
J=
Acid Test}
Rptin
II1ustration : 3.53
-l
[Liquid Assets
Liquid Liabilitiesj
t RS.2.75.000J
Rs.9,OO.000
= 0.31 : 1

.The following data are extracted from the published accounts of two companies in an
industry:
PQR Ltd .. ABC Ltd.,
Sales Rs. 32,OO.UOO Rs. 30,00,000
Net Profit (after tax) 1.23.000 1,58,000
Equity Capital (Rs.1O per share, fully paid up) 10.00,000 8,00,000
General Resetve 2,32,000 6,42.000
Creditors 3,82,000 5,49,000
Bank Credit (short-term) 60,000 2.00,000
Fixed Assets 15,99,000 15.90,000
Inventories 3,31,000 8,09,000
Other current assets 5,44.000 4,52,000
Long term debt 8,00.000 6,60.000
You are requested to calculate and c,omment on ratios showing liquidity and profitability.
[Kuvempu Uni., B.Com., May 1991]
Solution:
PQR Ltd., ABC Ltd.,
Liquid Assets (other Current Assets) Rs.5.44.000 Rs.4,52,OOO
Inventories 3.31,000 8,09,000
Current Assets 8,75,000 12,61,000
Creditors 3,82,000 5,49,000
Bank credit (Short-term) 60,000 2,00,000
Current or Liquid Liabilities 4,42.000 7,49,000
Share capital 10,00.000 8,00,000
General Reserve 2,32.000 6,42,000
Shareholders' Fund 12,32,000 14,42.000
Management Accounting : 200

Liquidity Ratios

a. current} _ C
Current Assets J = Rs.8.75.000 J (Rs.12.61.000]
Ratio - ~urrent Liabilitie0 Rs.4,42,OOO J [ Rs.7,49,000 J
= 1.98: I 1.68: 1

b LiqUid} [Liquid Assets J CRs.5.44.00i) [ Rs.4.52.000]


Ratio -lLiquid Liabilitie0 = LRS.4.42.00~ l Rs.7,49.000J
= 1.23 : I 0.60: I
Profitability Ratios

J= [L
t J CRs.l.58.000
Net Profit
a. Net profit} = after Tax
Ratio Sales x 1OJ
Rs.l.23.000
Rs.32,00,000 x 1O~ = l.Rs.30,00,000 XI~
Revenue
= 3.84% =5.27%

Net Profit
b Return on} _af_t_e_rT_a_x__ x 100 = (Rs.1.23.000 x
Shareholders' IO~ r.Rs.1.58.000 x 1O~
Fund Shareholders'
Fund
lRs.12,32,000 J lRs.14,42,000 J
=9.98% = 10.96%
As far as the liquidity is concerned PQR Ltd., is having satisfactory position when
compared to ABC Ltd., and also the standards. However, from the view point of profitability,
ABC Ltd., is more profitable when compared to PQR Ltd.
Illustration: 3.54
The Gross Profit of X Ltd., for the year 1998 is Rs.80,000. This is one-fourth of the
year's sales. Out of total sales, three-fourth is on credit. The stock turnover is 10 times and
average collection period is 15 days (assume 360 days). Total assets turnover is 4 times and the
long term-debt to equity is 50%. Shareholders' equity is Rs.40,000. The current ratio is 2:1.
Find out: Creditors, Long-term debt, Cash in hand, Debtors, Closing stock, and Fixed assets. And
also prepare Balance Sheet of X Ltd. [Ballgaiore Uni., B.Com., November 2000J
Accounting Ratios: 201

Solution:
01. Gross Protit = Rs.80.000 = 25% of Sales
02. Hence, Sales Revenue - [Rs.80,000 + 25% J = Rs.3,20,OOO [comprising of Rs.2.40.000 credit
sales and Rs.80,00.0 cash sales].
03. Cost of Goods Sold = [Sales Revenue - Gross Profit]
= [Rs.3,20,OOO - Rs.80.000 J = Rs.2.40.000

04. Stock TUrn}


over Ratio =
Cost of Goods Sold
A verage Stock
:. \0 =
Rs.2.40.000 J
[ Average Stock

:. AVerage}
Stock = lrRs.2,40,OO~
\0
.
) = Rs.24,OOO [assumed Lo be closmg stock]

05 Average COllee} = A v~rage Trade Debt~rsJ :.15 = [Average Trade Debtors]


-tion Period Dmly Average CredIt Rs.2,4Q,000
Sales 360 days

Average
..l nh }
T r:llIe e tors
__ ~5 x 2,40,000
360
j_ Rs. 10,000 [assumcd to be clo.sing balanceJ

06 Total Assets}-
Turnover Ratio
rl Sales Revenuci
Total Assets J
:. 4 = (Rs.3.20.000
l
Total Asset0
J

:. T 0 ta,
Ass~ts
I} = tRs. 3.20,000]
4
=Rs.XO,OOO

07. Long-term Debt =50% of equity; and Shareholders' equity =RsAO,OOO


:.Long-term Debt =[Rs.40,OOO x 50%J =Rs.20,OOO
Management Accounting : ,202

os. Total of Capital and Liabilities =SO,OOO [i.e., total assets] 'I • • • .'. ~

Less: Shareholders' Equity 40,000


Total Debt, 40~000 , ,
Less: Long-term Debt 20,000
:. Current Liability 20,000 = Creditors

current}
Current Assets , rcurren~ Assetsl
Ratio = Current Liabilities :. 2 = l" Rs. 20,000 ,J '
:. Current Assets =[Rs.20,00Q x 2] = Rs.4O,000
Less: Debtors Rs. 10,000
Inventory 24,000 34,000
:. Cash 6,000
09. Total Assets Rs. SO,OOO '
Less: Currerit . Assets" 40,000
', ..

:. Fixed Assets 40,000

, ,
Bahmce Sheet of X Ltd., as on"December 31,1998 "
Amount Amount
Capital and Liabilities Assets
Rs. Rs.
",

Shareholders' Equity _40,000 Fixed Assets " " "" 40,000


,
Long-term Debt 20,000 Current Assets: Inventory 24,000
Creditors 20,000 Debtors 10,000
, , Cash' 6,000
SO,OOO SO,ooO
mustration : 3.55
Given the following particulars:
Debtors' Velocity: 3 months Fixed assets turnover ratio: S times
Creditors' Velocity: 2 months Gross profit turnover raito: 25% .
Stock Velocity: S times
Gross Profit during the year amounted to Rs.SO,OOO. There is no long-term loan or
overdraft. Reserves and surplus amounted to Rs.2S,000. Liquid assets are Rs.97,333. Closing
stock is Rs.2.000 more than the opening stock. Bills receivable and Bills payable are Rs.5,000
Accounting Ratios : 203
and 2,000 respectively. Find out: (a) Sales (b) Sundry.Debtors (c) Closing Stock (d) Sundry
Creditors (e) Fixed Assets (0 Proprietors Fund. Also make out the Balance Sheet with as many
details as possible. [Bangalore Uni, B.Com,. Apri12001]
Solution:
01.. Gross Profit Ratio =25% and Gross Profit = Rs.80,000. Therefore,

Sales} Gross Profit] CRs.80.000 J .


Revenue = Gross Profit RatioJ = l 25% J = Rs.3,20,000 (a)

Average Trade Debtors verage Trade Debtor]

c
02. Debtors'}
Velocity Monthly Credit Sale Rs.3,20,OOO
= :.3 = 12 months

Average Trade
:. Debtors (assumed,
~ = [ 3 x RS.3'20'OO~ =Rs. 80,000
Closing Balance) 12.

:. [Rs. 80,000 - Rs.5,OOO Bills Receivable] = Rs.75,OOO Sundry Debtors (b)

01. [Sales Revenue - Gross Profit] =Cost of Goods Sold


[Rs.3,20,OOO - Rs.80,OOO] = Rs.2,40,000 = Cost of Goods Sold

over Ratio
r
Stock Tum} = Cost of Goods Sold
( AveraJ!e Stock
I
J 8
C
Rs.2,40,OOO
l
:. = Average Stock
J
J'

Stock - (
r
Average } _ Rs.2,40,OOO
8
I_ _(Opening Stock + Closing Stock)
:J - Rs.30,000 - l 2, j

= fpenin g Stock + Qp:nng Stock + RS.2,o1 =[ 2 Opening StO;k + Rs. 2,()()O J


:. [30,000 x 2] - Rs.2,OOO =2 Opening Stock = [Rs.60,000 - Rs.2,OOO] = Rs.58,OOO
:. Opening Stock =Rs. 29,000 [i.e., Rs.58,ooO + 2]
Closing Stock = Rs.31,OOO [i.e. Rs.29,000 + 2,000] (c)
, "

04. [Opening
, Stock + Purcha~es - Closing Stock]
I =Cost of.
Goods S,oId '
:. Purchase =[Cost of Goods Sold + Closing Stock - Opening Stock)
Management Accounting : 204

= IRs.2.40.000 + Rs.3I.000 - Rs.2lJ.()OO I = R~.2A2.0{)()

Creditors
, . '} ~A Wrage Trade credilorsJ
~ _veraoe
(A _.::.e_Trade
_ _creditors]
_ __

Vc10dty Monthly Credit Pun:h4lses Rs.2.42.000


= :. 2= 12 m()nth~

A Vl'ru~e Trude}
Creditors =
r
eX 2,42.000
12
Ij = R~.40.333 (dosing balance)
ITrade Creditors RsAO,333 - Rs.2,OOO Bills Payable] = Rs.38.333 Sundry Creditors (d)

05. Fixed Asscts l


Turnover Ratic{
r.
=l
Sales Revenue
Fixed Assets
J
j
r Rs.3,20,OOO
:.8 = [Fixed Assets)
l
:. Fixed Assets =IRs.3,2().O()O + XI =R~.4(J.OOO
·06. Liquid Assets R~.97.333

Add: Inventory 31.000


:. ClIITent Assets 1,2!U33
Less: Inventory Rs.3I.000
Sundry Debtors 75.000
Bills Receivable 5.000 1.11,000
:. Cash 17.333

07. Current Assets Rs.I,2M,333


Fixed Assets 40,000
:. Total Assets 1.68.333 (Total of Capital and
Liabilities)
Less: Bills payable Rs.2.000
Sundry creditors 3M.333 40.333
:. Proprietors Fund 1.28,000 (f)
Less: Reserves and Surplus 2X.()OO
.'

:. Share Capital 1.00,000

as on ••.•.
Capital and Liabilities Amount Rs. Assets Amount Rs.
Accounting Ratios: 205

I Share CapItal I.()()JJ()() I Fixed A""L'h 4().OIlO


I
I
I Re,>en e" and "urplu<., 2HJJ()() I
I
('UITl'lll :\'>"L'h: i
! Current Liabilities: IIl\ clIlllry 3 1.0110 I.I,
Sundry l-reditors
Bill'> payable
lx.mJ
2.0011
Sundry Dchtor,>
Bill '> Recei vahle
75.000
5.000
Ca~h 17,333
1,6X,J33 1.61U33

Illustration: 3.56
The following information i" given to you.
a. Current Ratio 2.5
b. Liquidity Ratio 1.5
c. Net Working Capital Rs.3.00.000
d. Stock Turnover Ratio: 6 times (Cost of sales IClosing stock)
e. Gross Profit Ratio 20%
r. Fixed A"sets TUrnllVl'r Ralio .2 time'>
g. A wrage Debt Collectioll Period 2 1l1ol1tll~

h. Fixed Assets: Shan:holdl'l'\ \jet \\orth I: I


I. Reserve: Share Capital 0.): I
Draw up a Balance Sheet Irolll the above illlormation.
I Bllllgll/ore VIIi, IJ.Com, Novem/Jer 20(IJ lllui Oct(J/Jer 20(}2 J
Solution:
01. Current Ratio = ICurrent As<.,ets + Current Liahilitie'>l = 2.5
Working Capital = ICurrent A"scts - Currellt Liabilitie.,1

:. Rs.3.00.000 = 12.5 - II = 1.5


.'. I = I Rs.3.00.000 + 1.51 = Rs.2.()0.OOO = Current Liabi lilies

:. Current Assets = IRs.2.00.000 x 2.51 =R,>.5,OO,OOO


02. LiqUid} [ Liquid Assets J =l
r Liquid Assets I
J
Ratio = Current Liahilitiesj :. 1.5 Rs.2.00,()O()

:. Liquid Assets = I Rs.2.00.000 x 1.51 = Rs.3,{)O,(}OO

ICurrent A"sets - Liquid A"~L'hl = Inventory


I Rs.5,O{),OOO - Rs.3,OO.OOOI = Inventory = Rs.2.00,OOO
Management Accounting : 206

03. Stock Turn} r


Cost of Sales!
over Ratio =lClosinll Stock.)
6
[Cost of Sales]
l
:. = Rs.2,OO,OOO j
::Cost of Sales = [Rs.2,00,000 x 6] = Rs.12,00,000
02. Gross Profit Ratio = 20%. Therefore, Cost of Sales Ratio =80%

Sales } ( Cost of Sales


Revenue = Cost of Sales Rati~ =
J
[RS.12,00,000!
80%) = Rs.1S,OO,OOO
, . : '

OS. Fixed Assets 1- [Sales Revenu~ . _


Turnover RatioJ - Fixed Assets 2-
rR~.IS,OO,OOO!
J .. l FIxed Assets]
:. Fixed } = ~RS.1S'00'000~ = Rs.7,SO,000
Assets 2

06. Average D e b tAverage


· Trade Debtors
Collection perioJ [ Monthly Credit Sales
J ' tA verage
. Trade
, Debtors]
Rs.1S,00,000
= :.2 = 12 months

:. Average Trade} =
. ..,' .,.' .. -::' Debtors· .
r
2
l··
x ~s.IS,OO,OOO I
12 )
= Rs.2,SO,000 (assumed to be the
closing balance]
07. Fixed Assets = Rs.7,SO,000 and Fixed Assets to Shareholders' Net Worth = 1 : 1. TIierefore,
Shareholders' Net Worth = Rs.7,SO,000.
08. Reserve to Share Capital = O.S:I, and Reserve + Share Capital = Shareholders' Net WoI1h =
O.S + 1 = I.S = Rs.7 ,SO,OOO. Therefore,
Reserve = [(O.S + 1.5) x Rs.7,50,000] = Rs.2,50,OOO and
Share Capital = [(1+1.5) x Rs.7,50,OOO] = Rs.5,00,000
09. Current Assets Rs.5,OO,OOO
Less: Inventory Rs.2,OO,OOO
Debtors 2,SO,000 4.50,000
:. Cash 50,000

as on .....
Amount Amount
Capital and Liabilities Assets
Rs. Rs.
Accounting Ratios: 207

Share Capital ' 5,00,000 Fixed Assets ' 7,50,000


Reserves 2.50,000 Current Assets:
Long-term Debt (balance) 3,00,000 Inventory ,,2.00,000
Current Liability 2.00,000 Debtors 2,50,000
Cash 50,000
12,50,000 ' 12,50,000

mustration : 3.57
Following is an incomplete Balance Sheet given to you:

Liabilities
Amount
Rs.
Assets ,Amount
Rs.
Equity capital 3,00,000 Fixed Assets --
Retained earnings 3,00,000 Inventories --
Creditors -- Debtors --
Cash . '-'"

-- --
You are given the, additional information:
a. Total Debt is two-thirds of Net Worth
b. Turnover of Total Asset is 1.8
c. 30 days sales are in the form of Debtors
d. Turnover of Inventory is 2
e. Cost of goods sold in the year is Rs.9,OO,OOO
f. Acid Test Ratio is 1 : 1
Complete the Balance Sheet by using the additional information.
{Bangaiore Uni, B.Com, April 2004]
Solution:
01. Equity Share Capital Rs.3,OO,OOO
Retained earnings 3,00,000
:.Proprietors' Fund 6,00,000 (Net worth)

Total Debt = [(2/3) x Net worth] = [213 x Rs.6,00.000] = Rs.4,OO,OOO = Creditors


02. Proprietors' Fund Rs;6,OO,OOO
Creditors (=Current Liability) 4,00,000
Management Accounting : 208"

:. Total of Capital and Liabilities 10.00.000 = Total Assets

Turnover
Total Assets
Of} [sales Revenue]
=" Total Assets J
. )-l_r
.. I.M
Sales Revenuc]
Rs. 10.00.000 J
:. Sales Revenue = [Rs. I 0.00,000 x 1.8 J= Rs.18,OO.OOO
Less: Co~t of goods sold 9.00.000
:.Gross Profit 9.00.000
I "
03. Debtors = 30 days' sales = I month sales
J(
~ = RS.18.00.000~
Sales Revenue
= Number . balance]
"= Rs. I.SO.OOO [assumed to be c10smg
of Months 12 "

04. Stock TU~-} = CCost of Goods SoldJ :.2 = r Rs .• 9.00.000 J


over Ratio L Average Stock. j lAvemge Stocl~J

Average }" "__ [ Rs.9.00.000


= Rs. 4.S0.000 lassumed to be c1osinl!. stockl
Stock 2 ~.

os. Acid Test} CLiquid Asscts J r


l
Liquid Assets]
J
Rat!o = LCurrent Liabilities j :. I = Rs.4.0().O()O

:. Liqufd Assets =IRs.4.00.000 x I J =RsA.OO.O()()


Less: Debtors = 1.50. 000
:.Cash 2.50.000
06. Total Assets ' Rs.IO.OO.OOO
Le~s: Inventory Rs.4.50.()OO
Debtors 1,50.000 -I:'"

Cash 2.S0.000 H.5 () .non


:. Fixed Assets 1.50.000

any as on .••••
Capital and Liabilities Amuunl Rs. Assds Amuunt Rs.
Accounting Ratios : 209

Equity capital 3,00,000 Fixed Assets . 1,50,000


Retained earnings 3,00,000 Inventories 4.50,000
Creditors 4,00,000 Debtors 1.50,000
Cash 2.50,000
10,00,000 10,00,000

Illustration: 3.58
,
State with reasons whether the following transactions result in increase or decrease of
working capital or do not affect the working capital:
a. Redemption of debentures worth Rs.40,000
b. Investments worth Rs.50,000 were sold for Rs.45,000
c. Goodwill written off Rs. 15,000
d. Building purchased by issue of equity shares Rs.2,00,000
e. Expenses paid in advance Rs.5,000
[Ballga/ore Ulli., B.Com., Apri/200Ij
Solution:
a.Redemption of Debentures worth Rs.40,OOO reduces the Working Capital to the extent of
Rs.40,000. Because, redemption of Debentures requires the payment of cash which is an item
of Current Assets and therefore, the amount of Working Capital a~ Working Capital =
rCun'ent Assets - Current Liabilities). Though the redemption of debentures reduces the
Debenture Capital, it does not lower the Current Liabilities as Dehenture Capital is 'not an
item of CUITent Liabilities.
b. Sale of "Investments worth Rs.50,OOO for Rs.45,OOO increases the amount of Working
Capital to the extent of sales proceeds viI. .. Rs.45,OOO. Because, the sale of investments
hrings in cash which is an item of Current Assets [which is one of the determinants of
Working Capital as Working capital = [Current As<;ets - Current Liabilitiesl. Hence. the sale
of investments brings in cash. increases the ca~h balancc and thc amount of Current A~~ds.
and therefore, it increase!. the amount of Wor"-ing' <.. 'apital.
c. Rs.15,OOO Goodwill written-off has no effect on Working Capital as it is only book
adjustment. Hence. there is no outflow or inllow of funds. .
d. Purchase of Rs.2,OO,OOO worth Building by the Issue of Equity Shares has no cffect on
Working Capital as it affects only items of Non-currcnt Assets <Building) and NOIH:urrent
Liabilities [Equity Share Capital).
e. Expenses I)aid in Advance of Rs.5,OOO docs not alter the amount of Working Capital.
Because. it lowers cash [one of the items 01" CllITcnt A~sets 1 and il1l:reases prl'-paid expcnsc~
[another itcm of Current Assets I. That means. this transaction lowcrs tilt: ca~h halance hy
Rs.5,000 [and thcrefore, the Current Assets by Rs.SJ)()OI and incrcases thc prl'J1aid c:\pcnscs
Management Accounting: 210
by Rs.5,OOO [and therefore, the Current Assets by Rs.5,OOO]. Hence, the net effect on
Working Capital is zero.
Illustration: 3.59
State with reasons whether the following transactions result in increase or decrease of
working capital or do not affect the working capital.
a. Bills Receivable Rs.40,000 dicounted for Rs.39.000
b. Fixed Assets purchased by issue of shares for Rs.3,OO,OOO
c. Advace Income Tax paid Rs, 10,000.
d. Goodwill written off Rs.5,000. [Bangalore Uni, B.Com., April2002J
Solution:
a. Discounting of Rs.40,000 Bills Receivable for Rs.39,000 reduces the amount of Working
Capital by Rs.I,OOO. Because, this transaction lowers the amount of Bills Receivable [and
therefore. the Current Assets as Bills Receivable is an item of Current Assets] by Rs.40,OOO
and increases the amount of cash balance [and therefore, the Current Asstes as cash is an "item
of Current Assets] by only Rs.39,000. .
b. Purchase of Rs.3,00,000 worth Fixed Assets by the Issue of Shares does not ~m~Ct the
Working Capital as it affects only the items of Non-current Assets [Fixed Assets] and Non-
current Liabilities [Share Capital]. It increases only the Share Capital and Fixed Assets by
Rs.3,OO,OOO each.
c. Rs.I0,000 Income Tax paid in Advance has no effect on the amount of Working Capital as
it lowers cash balance by Rs.1O.000 increasing the Pre-paid Income Tax [another item of
Current Assets] by an equivalent amount viz., Rs.1O.000. Hence, the net effect on Working
Capital is zero.
d. Rs.5,000 Goodwill written-off has no effect on Working Capital as it is only a book
adjustment causing neither the outflow nor the inflow of fund.
Illustration: 3.60
State with reasons whether the following transactions result in the increase or decrease of
working capital or do not affect the working capital.
a. Bills Receivable Rs.lO,OOO was discounted for Rs.9,OOO
b. 10% debentures Rs.80,OOO redeemed at 5% premium
c. Buildings purchased for Rs.3,00,000 by issue of equity shares of the same amount.
d. Preliminary expenses written off Rs.5,OOO [Bangalore Uni., B.Com., October 2002J
Accounting Ratios: 211

Solution:
a Discounting of Rs.I0,OOO Bills Receivable for Rs.9,OOO decreases the amount of Working
Capital by Rs.I,OOO. Because, this transaction lowers the Bills Receivable [an item of
Current Assets] by Rs.IO,OOO and increases the amount of cash balance [another item of
Current Assets] by only Rs.9,000. Hence, it reduces the Working Capital by Rs. 1,000.
b. Redemption of Rs.80,OOO 10% Debentures at 5% premium reduces the amount of
Workj.ng Capital by Rs.84,000 [Le., Rs.80,000 Debenture Capital redeemed + Rs.4,OOO
Premium on redemption (i.e., 5% of Rs.80,000)] as it [Le., transaction] causes the outflow of
cash. Of course, it also reduces the amount of Debenture Capital, but it [Debenture Capital] is
an item of Non-current Liabilities.
c. Purchase of Rs.3,OO,OOO worth Buildings by the issue of Equity Shares has no effect on
the Working Capital. Because, the transaction affects only the Non-current Account items
viz., Buildings [an item of Fixed Assets] and Equity Share Capital [a part of Share Capital].
d. Rs.5,000 Preliminary Expenses written-off does not affect the amount of Working Capital
as it is only a book adjustment. The transaction does not cause either the outflow or the
inflow of fund or cash. Hence, it has no effect on Working Capital
mustration: 3.61
Rubber Industries Ltd., has the following capital structure:
9% preference shares of Rs. 100 each Rs. 10,00,000
Ordinary shares of Rs. 10 each 40,00,000
50,00,000
The following information are available for financial year just concluded:
Profit after Taxation: Rs. 22,00,000; Ordinary Dividend paid: 20%: and Market price of
Ordinary Share: Rs. 20. You are required to find out (a) The Dividend Yield on the Ordinary
Shares, (b) The cover for the Preference and Ordinary Dividends, (c) The Earning Yield, and
(d) The PIE Ratio.
[ICWA (Fin), December 1992J

Solution:
Profit after Tax Rs. 22,00,000 Number of Shares: Equity: 4,00,000
Less: Preference Dividend Preference: 10,000
(9% of Rs. 10,00,000) 90,000
Equity Earnings 21,10,000

(a) Dividend Yield o~ _


Ordinary Shares f -
r
Ordinary Dividend Paid
l!vtarket Price of an Ordinary Share x
IO~ _ r20% of Rs. 10
) - l
Rs. 20
x 1001
~
Management Accounting: 212

= [RS. 2 XJO(~ = 10%


Rs.20 )

(b) Cover tor the}


Preference
and Ordinary -
(
Divisihle Profit
Preference and
J =
Rs. 22.00,000
( Rs. 90,000 + (20% of Rs. 40J)9,c)OO)
J
Dividends Ordinary Dividend

= ( Rs. 22,00,000 l = s. 22,OO,0()(j =2 .472. times


.
~s. 90.000 + Rs. X,O(J,OO~ IT
Rs. X,90,OOO

~arning
(c) Y.iel;
EgUitY Earnings per Share J
~
(I.e., Earmng
for Ordinary = Market Price of an x 100
Shares) Ordinary Share

= [< Rs. 21, 10,000 + 4,00,0(0) x Ion]


Rs.20

rRs.5.275 I ~
=l Rs. 20 x O(~ = 26.375Ck
(d) pricC-Eurnin~}
. (P,Jr..::
RaLlo J c)
_ (Market Price of an Ordinary Share] _ ( Rs. 20
- E ' E
.qUlty '
.armngs per.SI mre - I~'so ..
5 ')75
.:.. '.
J_ ~ .~.
7l) ' •.
lImes

Illustration: 3.62
The following extracts of rinandal information relate to Curious I.ld .. (R,. lakh).
Balance Sheet as al 3 I st December. 1995 1994
Share Capital 10 10
Reserves and Surplus 30 10
Loan funds 60 70
100 90
Fixed As.sels (Nel) 30 30
Current A '>sels:
Slock 30 20
()chltll':-' 30 30
Cash allli Bunk Balunce JO 20
Accounting Ratios: 213

Other Current A~~els 30 IO

100 HO
Less: Current Liabilities ~ --1!L
Net 70 60
Total Assets 100 90
Sales (Rs. lakhs) 270 300

(a~ Calculate, for the two ycars, Debt-Equity Ratio. Quick Ratio, and Working Capital
Turnover Ratio; and
(b) Find the Sales Volume that should have been generated in 1995 if the Company were to
have maintained its Working Capital Turnover Ratio.
leA (Int), November 1996J
Solution:
(a) Debt-Equity Ratio (based}
on Long-Term Loan)
r
Long-Term Deht ~
= ~hareholdcrs' Fun(~

Dcbt E~uity} 1994 = [RS. ~O,Oo,oooJ = 3.5 : I; IlJ95 = [R~. 60,00,000J = 1.5 : I
Rallo Rs. _0,00,000 Rs. 40,00.000

"k R' t' - [ Quick or Liquid Asscts


Q UIe alo- Q' " dLla
Ulck or LlqUl ' b'I"
I Itles
J (NoLe: Assumed that other Current Assets
include only the Pre-paid Expenses)

QUic.'k} 19l)4 = (RS. ~O.OO,{)ooJ = "_.5 '. I,' [RS. 40.00,OooJ


R at 10 () 00,o()()
Rs. _, 1995 = R~. 30.00.000 = 1.33 : I

Working Capitall _ ( Sales Revenue


Turnover Ratio f - Working Capital
J
Work·mg C aplta
. 1T R' Il)l)4 rR~. 3,00,OO.()0(~_ 5 '
urnover atlO, ;I = lRs.60,OO,oooj- tImes

1995 = (Rs. 2.70.00.0()(~_ 3 H6' ,


lRs. 70,OO.Oooj-· . times

(b) De~ired Salcs Revenue that should haw heen carned in 1995 to maintain it'> Working
Capilal Turnover Ratio (5 timc~) can be compLltL'd hy L1'>ing the ,>ame formula.
Workin u Capital L
f
Turnllve7- Ratio. 5 =
( De'>ired Sale,> Rc\ cnLlc
R~. 70.00.000. Wurking Capital
J
:. Desired Sales Revenue = (5 x Rs. 70.00,(100) = R,>. 3.50.00.000
Management Accounting : 214
Dlustration: 3.63
Agro Industries Ltd., have sl1bmitted the following projections. The ~et profit has been
arrived after charging depreciation of Rs. 17.68 lakh every year. You are required to work out
yearly Debt Service Coverage Ratio (DSCR) and the Average DSCR (figures "in lakh}.

Interest on Term Loan Repayment of Term


Year Net Profit for the year
durin~ the year Loan in the year
1 21.67 19.14 10.70
2 34.77 17.64 18.00
3 36.01 15.12 18.00
4 19.20 12.60 18.00
5" 18.61 10.08 18.00
6 18.40 7.56 18.00
7 18.33 5.04 18.00
8 16.41 nil 18.00
[ICWA (Fin), June 1994J
Solution:
Debt Service 1 Profit before Depreciation and Interest J
rLAggregate
Coverage RatioS = of Interest and Loan Repayment

c"omputatIon 0 fD ebt Servlce c overa2e R"


atIo
Profit before Aggregate of Loan
Debt Service
Year Depreciation and Interest Repayment and Interest
Coverage Ratio
(Rs.lakh) (Rs. Jakh)
1 58.49 29.84 1.960
2 70.09 35.64 1.967
3 68.81 33.12 2.078
4 49.48. 30.60 1.617
5 46.37 28.08 1.651
6 43.64 25.56 1.707
7 41.05 23.04 1.782
8 34.09 18.00 1.893
Total 412.02 223.88
Average 51.5025 27.985 1.840
Accounting Ratios: 215
Profit before Depreciation = [Net Profit for the year + Interest + Depreciation];
Aggregate of Loan Repayment and Interest = [Repayment of Term Loan in the year +. Interest].
mustration: 3.64

Summarised Balance Sheet, and Profit and Loss Account of a company is given below.
Determine the following ratios and comment on the health of the company basing your arguments
on the industry averages given in brackets. Assume income tax at 50%.
Inventory turnover (10); Investment turnover (1.5);
Sales margin (3.5%); Profit/Assets employed (4.0%);
ProfitlNet worth (11.5%); Average realization time (45 days); and
DebtlEquity (3 : 2).
Balance Sheet
Liabilities Rs. crore Assets Rs. crore
Equity 96.8 Net Block 48.4

Secured Loans 17.6 Stocks 66.0

Creditors 13.2 Debtors 22.0

Overdraft 17.6 Bank Balance 17.6

Income Tax due 8.8

154.0 154.0

Profit and Loss Account


Particulars Rs. crore Particulars Rs. crore
Materials 83.6 Sales 220.0
Manpower 52.8

Energy 8.0

Factory Expenses 13.2

Depreciation 4.8

Selling and Distribution 22.0

Administration 18.0

Interest 1.6

Profit 16.0
220.0 220.0

[ICWA (Fin), June 1995]


Management Accounting: 216

Solution:
Profit '<tfter Tax = [Rs. 16 erore - (50% of Rs. 16 erore Tax)]
=l Rs. 16 crore - Rs. 8 crore J=Rs. 8 erore
Proprietors' Fund = Rs. 96.8 erore, Share Capital

(a) InventOry} _ r.Sales RevenueJ = Rs. 220 crore]


Turnover Ratio - LClosing Stock ( Rs. 66 crore =3;33 times
(Sales-based)

(b) Investmenll _ (Sales Revenu~1 (RS. 220 croreJ 143'


Turnover f - Total Assets J = Rs. 154 crore =. times

(e) sales.l = [Profit after Tax x 1001 = r Rs. 8 erore x 100J = 3.64% .
Marglr§" l
Sales Revenue :.1 LRs. 220 crore

(d) Profit to Assetsl = (Profit afterTax X'IOO] (RS. 8 erore ]


Employed fTotal Assets - Rs. 154 crore x 100 =5.19%
(e) Profit to Net } _ r Profit after Tax I o~ _ ( Rs. 8 crore
x x 100] = 8.26%
WOIth - LProprietors' Fund ) - Rs. 96.8 crore

(t) Average } Debtors ]" ( Rs. 22 crore ~


Realisation
Time (days)
( Daily Credit Sales
If- s
. ;~~ croreJj

Rs. 22 crore ] (1 ]
= ( Rs. 220 crore x 365 days = W x 365 = 36.5 days

(g) Debt-EquitYl _ rDebt (S~cured~ _ (RS. 17.6 erore ] _ ? .


Ratio f - l EqUIty J - Rs. 96.8 crore - 0.18_ . 1

Comparative Statement

SI. Accounting Industry Company


Comments
No. Ratio Averages Actuals
I. Inventory Slow and non-moving goods; high inventory
Turnover (times) 10.00 3.33 holding period.

")
Investment Failed to generate higher rcycnue per rupee
Turnover (times) , 1.50 1.43 of investment.

3. Sales Margin (%) 3.50 3.64 Net Profit to Sales Ratio is slightly higher
Accounting Ratios: 217

4. Profit to Assets
Employed (%) 4.00 5.19 Profit to Assets Employed is also higher

5. Profit to Net
Worth (%) 11.50 8.26 Return on Equity is low

6. Average
Realisation Time Efficient in collecting the amount from its
(days) 45.00 36.50 customers

7. Debt-Equity
Ratio 3:2 0.182: 1 Lower debt component; losing tax benefits
Illustration: 3.65
From the following particulars, estimate four critical ratios affecting working capital and
comment on their significance.
Balance Sheet (Rs. in Lakh)
Equity 120 Net block 500
Secured loans 240 Stocks 80
.Sundry creditors 150 Debtors 220
Bank overdraft 250
Taxes collected but not remitted 40
800 800
Sales 500
Purchases 350
[ICWA (Fin), December 1995J
Solution:
With the available information. the following ratios which affect the Working Capital can
be computed: (a) Current Ratio. (b) Quick Ratio, (c) Average Collection Period, and (d) Average
Payment Period.
. (current Assets )
(a) Current RatIO = Current Liabilities
I __ 0 68') . I
RS. 300 lakh
[ Rs. 440 lakhJ
= . _.

(b) Quick Ratio = ( ,?uick ~s~~t~


QUIck LlUblhtles
J= (Rs. 220 lakhl
lRs. 190 lakh)
= 1.158: 1
. (c) Average. CollectioJi'l _
Penod f -
[ Debtor~
lMonthl y Credit Sale~
l _
-
~RS' 220 lakh]
(Rs. 500 lakh)
12 months
Management Accounting: 218

- [RS. 220 lakh x 12J = [0.44 x 12) = 5.28 months


- Rs. 500 lakh
Assumed that the entire sale was on credit basis. In the same way. for computing the Average··
Payment Period. it is assumed that all the purchases were made on credit basis.

(t) Average Payment }


. Period
= t Creditors
Monthly Credit
Purchases
J = ~RS' 1501akh]
(Rs. 350 lakh]
12 months

_ [RS. 150 lakh


- Rs. 350 lakh x
12J _
-
RS. 1800 lakh
( Rs. 350 lakh
J5It h
=."t mont s

From the above, it is unequivocal that the short-term liquidity of the firm is not
satisfactory. The firm is having more Current Liabilities than the value of Current Assets. Hence,
the Current Ratio is less-than one. Of course. the tirm is having a Quick Ratio of 1.158. But it is
due to exclusion of Bank Overdraft (which is the largest item of short-term obligation) from the
short-term liabilities while computing the Quick Ratio. The Average Collection Period is very
long at 5.28 months and consequently, it takes about 5.14 months to pay its purchase bill.

Illustration: 3.66
The Balance Sheet of XYZ company is given below. You are requested to calculate (a)
Debt equity ratio, (b) Current ratio. (c) Interest coverage ratio and comment.
Liabilities Rs. Lakh Assets Rs.lakh
Equity 250 Fixed Assets 400
General reserves 280 Stock 460
P&L Nr.: (halance) 30 Debtors 460
Secured loans - long term 300 Cash in hand 10
Secured loans - short term 360 Investments 50
Creditors 150 Misc. expenditure not written off 20
Other liabilities 30
1.400 1.400
Accounting Ratios: 219
Additional Information:
(a) From the Profit and Loss Account. Rs. 90 lakh was transferred to General Reserve during
the year.
(b) Interest cost amounted to Rs. 120 lakh.
(c) Taxation @ 40%. [ICWA (Fin), June 1996J

Solution:
Proprietors' Fund = Debt (only long-term) = Rs. 300 lakh
Equity Capital Rs. 250lakh
Reserves Rs. 280 lakh
P&L Alc (bal) Rs. 30lakh
Rs. 560lakh
(a) Debt-Equity Ratio = (Debt + Equity) = (Rs. 300 lakh + Rs. 560 lakh) =0.536 time. The
share of long-term debt in the total (long-term) capital of the company is only about 35%.
The company can increase the debt capital to about 50% to reap the benefits of tax
advantage.
(b) Current Assets = Current Liabilities =
Stock Rs.4601akh Short term secured loan Rs. 360 lakh
Debtors RS.4601akh Creditors Rs. I50lakh
Cash Rs. 10lakh Others Rs. 30lakh
Rs. 930 lakh Rs.540lakh

currentL _ [current Assets


Ratio S - Current Liabilities
J =
[RS. 930 lak~1
Rs. 540 lakhJ = 1.722: I
The Current Ratio is slightly lower than the standard and the share of inventory is about
50% which may affect the liquidity. Hence. steps should be taken to improve the
liquidity position.
Rs.lakh
(c) Profit Retained (balance) 30
Add: Profit transferred to General Reserve 90
Protit after tax 120

Add: Tax (40tk ) (RS. 120 lakh x 40tk


60'7('
J 80

Taxable income 200


Management Accounting : 220
Add: Interest 120
:. Protit before interest and tax 320

}~
Profit before ~
Interest
·
Coverage
. ~ =
Interest and Tax
Interest
( Rs. 320 lakh
= Rs. 12() Ia kh
'
J 2 67 .
=. tImes
RatIO
Interest Coverage Ratio is satisfaetory
Summary of the Chapter
Ratio Analysis is an effective tool of tinancial analysis and it diagnoses the operational
and financial strengths and weaknesses of an organization. This .diagnosis becomes the base for
the steps to be initiated and the decisions to be taken. In this background, meaning, objectives,
and utility of Ratio Analysis are discussed followed by modes of expression and interpretation of
Ratios. Thereafter, classification of Accounting Ratios is discussed. Ratios which enable to
evaluate the Profitability, Liquidity and Solvency are discussed followed by the limitations and
advantages of Accounting Ratios. A number of problems are also solved at the end.

Key Terms to Remember


Ratio Accounting Ratio Ratio Analysis
Proportion Percentage Internal Analysis
External Analysis Vertical Analysis Horizontal Analysis
Gros'> Protit Ratio Operating Ratio Net Profit Ratio
Rol Return on Capital Employed Earnings per Share
Current Ratio. Liquid Ratio Inventory Turnover Ratio
Debturs' Turnover Ratio Creditors' Turnuver Ratio Debt-equity Ratio
Proprietary Ratio Capital Gearing Ratio Trading on Equity
Interest Coverage Ratio.
Questions for Self-study
01. Define 'Accounting Ratio' and distinguish it from 'Ratio Analysis'
02. What is Ratio'? Explain the relevance of Accounting Ratios.
03. Analyse the role of RUlio Analysis in Financial Decision making.
[Ballga/ore Ulli., B.COIll., November 2000 and April 2004]
04. What arc the limitations of Ratio Analysis? /Kllvelll/Jll Uni., B.Com., May 1998/
05. Examine the following tools of Analysis and Interpretation of Financial Statements:
(a) Trend Analysis (b) Ratio Analy~is. [Kllvelllpu Uni., B.Com., November 2000J
Accounting Ratios: 221

06. Explain the folluwing concepts of Ratio Analysi~: (a) Liquidity. (b) Solvency. and
(c) Protitability. [Mallga/ore Vni., B.Com., April200lj
07. Explain the need for analysis of Financial Statements. [Mangaiore Vni., B.Com., May 2003}
08. Explain the meaning. objectives and importance of Financial Statements.
[Mallgaiore Vlli., 8.Com., November 2004}
09. Define Current Ratiu and explain its roie in the evaluation of short-term financial solvency of
a company.
10. State the significance of Acid Test Ratio. [Ballgaiore Vni., B.COIll., May 200l}
II. State the signi ticance of Current ·Ratio. [Ballgaiore VIIi., B.COIll., May 2003}
12. What is the significance of Short-term Solvency Ratio'!
[Ballgaiore Vlli., 8.Com., April2()04/
13. State the im.portance of working capital to a business concern.
[KIIVel1lpll VIIi., 8.Com., November 20()3j
14. What are tht! advantages of adequate working capital'!
/KlIvempll Clli., 8.('0111., November 20()4/
15. Explain the role of Acounting Ratios in the evaluation of profit earning capability III' a
company.
16. What is Debt-Equity Ratio'! How is it differs from Capital Gearing Ratio'?
17. Define Accounting R,ltio and critically evaluate the limitations of Ratio Analysis.
18. What are the limitations of Ratio Analysis'!
[Kuvemp" Vlli, 8.COI1l., May 1991, 1997 alld 2001, alUl November 2002}
19. Explain the Gross Profit and Stock Turnover Ratios. /KlIvempll VIIi., 8.COIll., May 1998}
20.' What is Debt-Service Ratio'? Explain how the 1I1!ere~t-C(}verage Ratio differs from Dividend
Coverage Ratio.
21. What is meant by Trading on EquiLy·.>
22. Deline Debt-Equity Ratio and explain thL' components and importance of Debt-Equity Ratio.
23. Write a note on Du Pont Analy ... i~.
24. What is Proprietary Ratio'?
25. What is Operating Ratio'! Distinguish between Operating Expenses Ratio. Operating Cost
Ratio and Operating Protit Ratio.
26. Descrihe Gross Protit Ratio. Operating Protit Ratio and Net Profit Ratio. and briefly explain
their role in the evaluation of Protitability of a business concern. What similarities and
dissimilarities do you tind between these ratios on the one hand and the Return on
Investment on the other'!
Management Accounting : 222
27. Examine the role of Return on Investment Ratio as a measure of overall profitability of a
concern.
28. Discuss the relevance of Ratio Analysis for inter-firm comparison.
29. What do you mean by 'Projection through Ratios'? Explain the role of Accounting Ratios in
the preparation of 'Projected Financial Statements'.
30. What do you understand by Capital Gearing Ratio'? Explain the advantages and disadvantages
from the view po~nt of companies having low gearing and high gearing ratios.
31. Define Turnover Ratio and explain the significance of Inventory Turnover Ratio.
32. Describe Debtors' Turnover Ratio and explain the inferences which may be drawn from its
use.
33. Define' Average Collection Period' and distinguish it from' Average Payment Period' .
34. Define Creditors' Turnover Ratio and explain its relationship with Average Payment Period.
35. Define Quick Ratio and explain how does it differ from Current Ratio? Which is more useful
to evaluate the company's capability to meet its short-term obligations?
36. Write a note on Absolute Liquidity Ratio as a measure of short-term solvency.
37. What is Proprietors' Fund and why is it called residual sum?
38. Discuss the importance of Accounting Ratios for inter-tirm and intra-firm comparisons.
39~· From the view point of (a) an investor, (b) a creditor, and (c) a financial controller of a
company, how would you analyse the financial position of a company.
40. Write short notes on (a) Window Dressing, and (b) Return on Capital Employed Ratio
[Kuvempu Uni., B.Com., October 1999J
4l. Short-answer Questions
a. State the key steps involved in Ratio Analysis. [BangaloT:e Uni., B.Com., May 2000J
b. What is Price-Earning Ratio? [Bangalore Uni., B.Com., May 2000J
c. What do you mean by Net Working Capital?
[Bangalore Uni., B.Com., November 2001 and May 2002J
d. What is Activity Ratio? [Bangalore Uni., B.Com., November 2001 and May 2002J
e. State any four components of Current Assets.
[Bangalore Uni., B.Com., May and November 2002J
f. Define Ratio Analysis. [Bangalore Uni., B.Com., May 2002J
g. What is meant by Return on Capital Employed? [Bangalore UIli., B.Com., May 2003J
h. What are Solvency Ratios? [Bangalore Uni., B.Com., November 2003J
i. How do you calculate Earnings per Share? [Ballgalore Uni., B.Com., April2004J
Accounting Ratios: 223

J. Mention the signiticance of Return on Investment.


{Mangalore Uni., B.Com., October 2003J
42. Calculate the following for the years' 1990 and 1991 using tigures made available.
(a) Return on Capital Employed (e) Inventory Turnover Ratio
(b) Current Ratio (t) Earnings per Share
(c) Debt/Equity Ratio (g) Dividend Cover.
(d) Fixed Assets Turnover Ratio
Balance Sheet (as at 31st December,) (Rs. in lakhs)
1989 1990 1991

Share Capital: Shares of Rs. 10 each 800 1.000 1,000


Reserves and surplus 700 800 1.000
Secured Term Loans 800 2,000 2,400
Cash Credits from Banks 800 1.000 1.500
Sundry Creditors 1.200 900 UOO
4,300 5.700 7,000

Assets: Fixed Assets: Gross Block 2,800 3,000 4,000

Less: Depreciation 920 1,400 2,000


1,880 1,600 2,000
Stocks 1,520 2,400 2,800

"
Debtors 480 500 900

Other Current Assets 420 1,200 1,300


2,420 4,100 5.000

Total Assets 4,300 5,700 7,000

Ex t ractsfrom P rofit
I and L oss Accounts (~or years en d e d31D ecem ber) Rs.m
' l kahs
1990 1991
Sales 4,800 7,200
Profit before depreciation and interest on term loans 1,500 2,400
Depreciation 480 600
Interest on term loans 420 600
Management Accounting : 224

I Tax
Dividends
300
100
600
150
1 1

lCA (Fin), May 1991]


43. 'Given the· following information for ABC company at the end of the current year,
determine (i) balances for the income statement and (ii) the balance sheet.
Net sales = Rs. 1,00,000 Debt assets tatio =0.6
Debtors' turnover ratio (based on net sales) = 2 Net profit margin = 5%
t
Inventory turnover ratio = 1.25 Gross profit margin =25%
Fixed assets turnover ratio = 0.8 Return on investment = 2%
~

Income Statement for .the_year endiDJ~ on 31st December, •••


Sales Rs. 1,00,000
Cost of goods sold
Gross profit
Other expenses
Earnings before tax
Taxes @ 50%
Earnings after tax
I
Balance Sheet as on 31st December
Liabilities Rs. Assets Rs.
Equity - Net fixed assets -
Long-term debt - Inventory -
Short-term debt 50,000 Debtors -
Cash -
Total - Total - .. ,
>.

44. Selected financial figures for Tilak and Co., for three years are given below
Year 1 Year 2 Year 3
GP Ratio (%) 30 25 20
Stock Turnover (times) 20 25 16
Opening Stock (Rs.) 30,000 ·27,000 50,000
Accounting Ratios: 225
Closing Stock (Rs.) 40.000 37.800 60.0()()
Income-Tax Rate (%) 50C'k

Administrative expenses in year I amounted to 10% of sales and the annual increase was IOlk
over the previous year. ·Prepare a statement of profits in a comparative form for all the three
years. and comment on the reasons for decrease in profitability.
[CS (Fin), December 1990J
45. Given bel~w are summarised accounts of Alok Ltd .• for the years I and 2.
Balance Sheet
(Rs.. in lakh!\)
Liabilities: Year 1 Year 2
Share Capital 250 250 I
General reserve 100 172
Debentures 180 150
I
I
Term loan 30 I! 30
I I
I Creditors 70 i
56
II I
630 658 I
Assets:
I I
I

I
I
Fixed Assets (at cost) 500 500
Less: Accumulated depreciation 80 115
Net fixed assets 420 385
Cash 55 85
I
Debtors 65
I 75 ,
Inventories 90 I 113
I
I
658

Income Statement (Rs.. in lakhs)


Year I
I
Year 2
I
Net sales 350
!
450 I
1

Less: Cost of material 90 !I I


I
IIJ I
I
i i !
I
Wages 70 70 i 160 I 183 !
- - I
Gross profit 190 267
Less: Selling. general and administration costs 50 60
Management Accounting : 226

Earnings before depreciation, interest and tax 140 207


Less: Depreciation 30 35
Earnings before intcrl.'<;t and tax 110 172
Less: Interest 25 27
Earnings before tax 85 145
Less: Tax 15 48
Earnings aftcr tax 70 97
Less: Dividend . 25 25
Retained earnings 45 72

Compute the (a) liquidity, (b) lewragl.:, (c) activity and (d) protltability ratios, and comment.
rCA (Fin), May 1980J

46. The following figures are extracted from the books of ABC Ltd., and XYZ Ltd., (Rs. in
lakhs)
ABC Ltd XYZ Ltd
Sales 800 1,000
Net Profit 60 50
Capital Employed 406 250
Calculate the primary ratio by establishing two secondary ratios of which the primary ratio is
composed of and also give your judgement on the performance of both the companies.
res (Fill), June 1986J
47. From the following information. prepare the projected Trading, and Profit and Loss Account
for the next financial year ending Decemher 31, 1985 and the projected Balance Sheet as on
that date:
Ratio of Gross profit = 25% Proprietary ratio (fixed
assets to capital emplo) cd) = XOCk
Net protit to equity capital = lork
Capital Gearing ratio (Preference
Stock turnowr ratio =5 times Shares and Debentures to Equity) = 30%
Average Debt 'Collection
General Reserve and Profit and
period = 2 months
Loss to Issued equity capital =25%
Creditors velocity = 3 months
Preference share capital to Debentures =2
Current I1ltio = 2
Cost of sales consists of 40% for materials and balance for wages and overheads. Gross profit
Rs.6.00,OOO. Working notes should be clearly shown. rCA (Fill), May 1985J
Accounting Ratios: 227

48. Using the following data, complete the balance sheet given below.
Gross profit (20% of sales) = Rs. 60,000 Inventory turnover (to cost of sales) = 8 times
Shareholders' equity =Rs. 50.000 Average collection period
Credit sales to total sales =80%
(a 360-day year) = 18 days
Current ratio = 1.6
Total assets turnover = J times
Long-term debt to equity =40%
Creditors .. Cash ..
Long-term debt .. Debtors ..
,
Shareholders equity .. Inventory ..
Fixed Asset~ ..
.. ..
IICWA (Fill), JlIlle 1979J
49. Mr. Desai intends to supply goods on credit to A Ltd .. and B Ltd. The relevant financial data
relating to the companies for the year ended 30th June. 1980 arc a~ under.
A Ltd B Ltd
Terms of payment (stated) 3 months 3 months
Stock Rs. 8,00.000 . Rs. 1.00,000
Debtors 1.70.000 1.40.000
Cash 30.000 60,000
Trade Creditors' 3,00.000 1.60.000
Bank overdraft 40,000 30,000
Creditors for expenses 60,000 10.000
Total Purchases 9,30.000 6.60.000
Cash Purchases 30,000 :20.000
Advise with reasons. ac; to whkh of the eompanie~ he ..,hould prefer to deal with.
ICA (Fin), November 1970J

50. From the following details. prepare a statement of proprietary fund with as many details
as possible:
(a) Stock velocity =6 (d) Gross profit turnover ratio =20Ck
(b) Capital turnover ratio =2 (e) Debtors velocity =2 months
(c) Fixed assets turnover ratio = 4 (f) Creditors velocity = 73 days
Management Accounting : 228

The gross profit was Rs. 60.000; reserves and surplus amounted to ~s. 20,000; closing stock
was Rs. 5.000 in excess of opening stock. ICA (Fin), 1970}

51. Light and Sound Manufacturers furnish the following information (Rupees in thousands).
1981 1982 1981 1982
Cash at Bank 24 36 Buildings (Net) 192 208
Bills receivable 122 I3X Land 32 32
Stock in trade 178 160 Current liabilities 138 114
Expenses prepaid 8 12 Equity shares 380 380
Furniture and Fixtures 38 60 General reserve 76 152
Profit and Loss Ale
Net sales 640 658 General & admn. Expenses 54 64
Cost of goods sold 460 464 Income Tax 14 IO

You are required to compute the following ratios:


(a) Current ratio (c) Net sales to receivable ratio
(b) Inventory turnover ratio (d) Net income per'share.
[MBA, University of Mysore, June 1985}
52. From the following information, prepare a summarised balance sheet as on 31 st March:
l. Working Capital = Rs. 1.20,000 4. Asset (tixed) to Proprietary. Ratio = 0.75
2. Reserves and surplus = Rs. 80.000 S. Current Ratio = 2.5
3. Bank overdraft = Rs. 20.000 6. Liquid Ratio = 1.5
ICA (Fill), Ntwelllber /98/J
53. (a) Supposing an Industry's average 'return on investmem' (RoJ) is X.5£K. M/~. Surewell
Pliannaceutical Ltd's net profit aner taxes is Rs. 460 and its total a!'.sets amount to Rs.
5.125. Calculate its Rol. Should this return on investment rate be acceptable to Mis.
SU!"l~\\ ell PharmacclIIical Ltd.'!

(b) Calculate the break-even point in quahtity as well as rupee volume on the basi~ of
following data:
I. Selling prkc per unit = Rs. 1.20 3. Variable cost per unit = Re. (UW
., Total Fixed l'Ust = Rs. 30.000 4. Contribution per unit = Re. 0.40
[CS (Fill), December 1988J
Accounting Ratios : 229

54. Given below are the summarised income statement and balance sheet of PQR Ltd.
Income Statement for the Current Year Ended on 31st December (Rs. in thousands)
Sales 1,600
Less: Cost of goods sold 1,310
Gross margin 290
Less: Selling and administrative expenses 40
Net operating income (EBIT) 250
Less: Interest expense 45
Earnings before tax 205
Tax paid ~Q

Net income after tax 123


Earnings per share (EPS) = Rs. 3.0
Balance Sheet as on 31st December
Liabilities (Rs, 000) Assets (R<;.OOO)
Paid-up capital (40.000 shares Net fixed assets 800
of Rs. 10 each fully paid) 400
Inventory I 400
Retained earnings 120
Debtors 175
Debentures 700
Marketable securities 75
Creditors 180
Cash 50
Bills payable 20
Other current liabilities 80
Total liabilities 1.500 Total assets 1.500

Price per share Rs. 15.00. Industry's average ratios are:


Current ratio = 2.4 Debt to assets =40%
Quick ratio = 1.5 Time interest earned =6
Sales to inventory = 8.0 Profit margin = 7%
A verage collection period =36 days Price to earning ratio = 15
Price per sharelbook value of the share = 1.6 Return to total assets = I 1%
Management Accounting : 230
1. PQR Ltd., would like to borrow Rs. 5,00,000 from a bank for less than a year. Evaluate
the firm's current financial position by calculating ratios that you feel would be useful for
the bank's evaluation.
2. What problem areas are suggested by your ratio analysis? What are the possible reasons
for them'!
3. Do you think the bank should grant the loan?
4. If PQR Ltd's inventory utilisation ratio (sales to inventory) and average collection period
were reduced to industry average, what amount of funds would be generated?
lCA (Fin), May 1981J
55. From the following information pertaining to M/s Sukanya & Co., Ltd., prepare its Trading,
and Profit and Loss Account for the year ended March 31, 1992 and a summarised Balance
Sheet as at that date.
Current ratio: 2.5 Sales: Rs. 7,30,000
Quick ratio: 1.3 Working Capital: Rs.I,20,000
Proprietary Ratio (Fixed Assets/Proprietary Fund): 0.6 Bank Overdraft: Rs. 15,000
Gross profit to sales ratio: 10% Share Capital: Rs. 2,50,000
Debtors' velocity: 40 days Net Profit: 10% of proprietary funds
lCA (lnt), November 1993J
56. The Balance Sheet of Major Ltd., a~ on March 31. 1989 is as under.
Liabilities (Rs.) Assets (Rs.)
Share capital (2,000 Equity Fixed Assets (at cost) 5,00,000
Shares of Rs. 100 each
Less: Depreciation 1,60,000
fully paid) 2,00,000
3,40,000
Current Assets:
7.5% Preference shares 1,00,000
Stock 60,000
General reserve 60,000
Debtors 80,000 .
12% Debentures 60,000
Bank 20,000
Current Liabilities:
Sundry Creditors 80,000
5,00,000 5,00.000

The company wishes to forecast Balance Sheet as on March 31, 1990. The following
additional particulars are avai lable.
Accounting Ratios: 231
(a) Fixed assets costing Rs. 1,00,000 have been installed on 1.4.1989 but the payment will be
made on 31.3.1990.
(b) The fixed assets turnover ratio on the busis of gross value of fixed assets would be 1.5.
(c) The stock turnover ratio would be 14.4 (l:alculated on the basis of average stock).
(d) The break-up of cost and protlt would be as follows. The profit is subject to interest and
taxation at 50%.
Materials 40% Oftice and Selling expense
Labour Depreciation
Manufacturing expenses IO~ Profit
(e) Debtors would be 1/9 of sales
(f) Creditors would be 1/5 of material consumed.

(g) In March 1990. a dividend at 10% on equity capital would be paid.


(h) 12% Debenture for Rs. 25,000 have been issued on 1.4.1989.
Prepare the forecast Balance Sheet as on March 31. 1990 and show the following resultant
ratios: (a) Current Ratio, (b) Fixed AssetslNet Worth Ratio and (c) Debt-Equity Ratio.
lCA (Fin), November 1989]
Answers
42. (a) 33.44,*, and 43.90%; (b) 2.16: I and 1.92: I; (c) 1.11 and 1.20; (d) 3 and 3.36 times; (e)
1.68 and 1.85 times. (f) Rs. 3 and Rs. 6; (g) 3 and 4 times.

43. COGS = Rs.75.000; Other Expenses Rs.15,OOO; EBT=Rs.IO.000; EAT = Rs.5.000; Equity =
R~.lIlOO.OOO: Long-term Debt = Rs.l . 00"OOO: Fixed Assets = Rs.93,750: Inventory =
Rs.60.000; Debtors = Rs.50.000; Cash = Rs.46,250 (bal); Total of Balance Sheet =
Rs.2.50.000.
44. Compute Average Stock. Cost of Goods Sold li.e .. COGS = (Stock Turnover Ratio x Average
stock)], Sales li.e .. Sales = COOS/(lOO - OP Ratio)]. and OP (i.e .. GP = Sales x GP Ratio).
Sales = Rs.lO. Rs.IO.8 and Rs. 11 lakh; AOHEs = Rs. 1. Rs. 1.1 and 1.21 lakh: PBT = Rs.2,
Rs.1.6 and Rs.0.99Iakh; and PAT = Rs. 1. Rs.0.8 and Rs.O.495 lakh.
45. (a) CR = 3: 1 and 4.87: 1; QR = 1.71 : 1 and 2.86: 1;
(0) D-E Ratio = 0.8 and 0.56; Interest Coverage = 4.4 and 6.4 times;
(c) STR = 1.78 and 1.61 times: DT'R = 5.38 and 6 times; Collection Period = 67 and 60 days;
(d) GP = 54.3 and 59.3%; Operating Profit Ratio = 31.4 and 38.2%; NP = 20 and 21.5%;
Return on Total Assets = 15.1 and 18.8%-; Return on Capital Employed = 16.9 and 20.6%.
46. Rol = 15 and 20%: Profit to Sales = 7.5 and 5%: Capital Turnover = 2 and 4 times.
47. Sales = Rs.24 lukh; Net Profit = Rs.l.06AOO: Fixed Assets = Rs.15.2 lakh; Debtors = RsA
lakh; Stock = Rs.3.6 lakh: Equity Capital = Rs.IO.64 lakh; Preference Share Capital = Rs.3.8
Management Accounting : 232

lakh; Debentures = Rs.1.9 lakh; General Reserve = Rs. 1,59,600; Creditors = Rs.1.8 lakh;
,Bank-OD =Rs.2 lakh; Total of Balance Sheet = Rs.22.8 lakh.
48. Creditors = Rs.30.000: Long-term Debt = Rs.20,000: Equity Capital = Rs.50,000; Cash =
Rs.6.000: Debtors =Rs. I 2,000; Inventory = Rs.30.000; Fixed Assets = Rs.52,000.
49. Current Ratio = 2.5 : 1 and 1.5 : I; Quick Ratio = 0.56 : 1 and 1.18: 1; Average Payment
Period = 122 and 91 days. Hence, B is to be preferred.
50. Fixed Assets = Rs.60.000; Working Capital = Rs.60,000; Proprietors' Fund = Rs. I ,20,000;
Share Capilal = Rs.l,OO,OOO; Reserves and Surplus = Rs.20,000.
51. (a) 2.41 : I and 3.04 : 1; (b) 2.58 and 2.9 times; (c) 5.25 and 4.77 times; (d) Rs.2.95 and
Rs.3.16 (assumed shares of Rs. 10 each).
52. Capital = Rs.4 lakh; Creditors = Rs.60,000; Fixed Assets = Rs.3.6 lakh; Stock = Rs.80,000;
Other Current Assets = Rs.l.2 lakh; Total of Balance sheet =Rs.5.6 lakh.
53. (a) RoI =8.98; Yes, acceptable; (b) BEQ =75,000 units; BESR =Rs.90,000.
54. (1) Compute Liquidity and Turnover Ratios, and compare them with the industry's averages
to draw conclusion;
(2) Low Inventory Turnover Ratio, accumulation of obsolete and/or non-moving stocks;
(3) Yes;
(4) Rs.2.l6.600 (approximately).
, . ,
55. Gross Profit Rs. 73.000; Net Profit Rs. 30.000; and Balance Sheet total Rs. 3,80,000.
56: Balance Sheet total Rs. 5.69.300; Current Ratio 1.56: 1; Fixed Assets to Net Worth 1.06: 1;
and Debt-Equity Ratio 0.23: 1 or 0.19: 1.
Chapter-IV

FUNDS FLOW STATEMENT

Objectives: The objectives of this chapter are:


To understand the Concepts of Fund, Flow of Funds, and Funds Flow Statement,
To learn the Importance and Managerial Uses of Funds Flow Statement,
To know the Sources and Applications of Funds,
To understand the Procedure of Preparing the Funds Flow Statement, and
To solve, in the light of the theoretical knowledge, the problems on Funds Flow Statement.
SlTucture:
Introduction
Concept of Fund
Statement of Changes in Financial Position - Funds Flow Statement
Importance or Managerial Uses of Funds Flow Statement
Business Transactions and Flow of Funds
o Current Asset Vs Current Asset
o Current Liability Vs Current Liability
o Current Assets Vs Current Liabilities
o Non-current Items
Sources and Applications of Fund
o Funds from Operation
o Issue of Shares and Debentures
o Increas~ in Long-term Liabilities
o Sale of Fixed Assets and Long-term Investments
o Repayment of Long-term Loans, and Redemption of Preference Shares
and Debentures .
o Acquisition of Fixed Assets
Preparation of Funds Flow Statement
o Schedule of Changes in Working Capital
A Note on Treatment of Proposed Dividend
A Note on Provision for Taxation
o Statement of Sources and Application of Funds
Limitations of Funds Flow Statement
Illustrations
• Summary of the Chapter
• Key Terms to Remember
• Questions for Self-study
Management Accounting : 234

Introduction
The two important Financial Statements. viz.. Protit and Loss Account. and Balance
Sheet which form a part of annual reports and which are prepared on the basis of conventional
accounting principles and practices have been criticised by many - both individuals and
institutions - on a number of grounds. The important criticisms, amongst others, are identified
below:
. I. Balance Sheet depicts the effects on assets and liabilities of variegated types of
transactions that have taken place during an accounting year and therefore. it shows the
position of various assets. liabilities and owners' equity at the end of the last day of the
accounting year. However, the Balance Sheet does not effectively depict the major
tinancial and investment transactions which intluence the tinancial position. If at all one
wants to draw inferences about the transactions. he has to compare the Balance Sheet of
two periods. By comparing an asset at the end of the current year with that of the
previous year, changes in the amount may be ascertained. Still it is not possible to know
why and how the changes have occurred.
2. There are some transactions which influence the financial position but do not tind place in
the Financial Statements. For instance. the transactions pertaining to the raising of loan
during an accounting year and paying back the same in the same accounting year.
3. Further. the Balance Sheet is criticised as a static one showing no movement of funds.
Because, it is prepared on the basis of the company's assets and liabilities at the end of a
particular day, usually at the end of the last day of the accounting year. It is no
exaggeration that the success or otherwise of an organization usually depends upon the
availability of funds and the efficiency with which the available funds are being utilized.
4. As identified earlier. Financial Statements portray the effects of business activities. These
activities lead to the movement in assets. liabilities and capital. This movement brings
about the changes. These changes form the most significant facts which the conventional
Financial Statements fail to retlect in a meaningful manner. The order in which the
changes take place is presented below.

Activities - Financial, Cause movement in Assets. Movement Changes


Investment and Business --+ Liabilities and Capital --+ brings changes f-+ form Bases

Financial Statements fail to


reflect these changes properly
Funds Flow Statement: 235

The Statement of Changes in Financial Position or the Funds Flow Statement acts as a
supplementary statement but not as a substitute for the Financial Statements. And it aims at
getting over the criticisms listed above.
Concept of Fund
The word fund has different connotations or meanings and these meanings vary between
two extremes of cash and all financial resources. Anyhow, important and useful approaches are
three in number as present~d below.
1. The word fund is widely used to represent the Working Capital which is the difference
between Current Assets and Current Liabilities (Le., Net Current Assets). It may be noted
here that cash and bank balances are the items of Current Assets.
2. The word fund is used in a narrower sense also to denote cash - both on hand and at
bank. The Funds Flow Statement, therefore. furnishes information about all significant
changes in cash.
3. The word fund is also used in a broader sense to represent all financial resources
including the Working Capital.
Concept of Flow
The word Flow refers to movement bringing about the changes in the form of either
inflow or outflow. Hence, the movement of funds is referred to as flow of funds. Therefore, the
flow of funds which takes the form of either the inflow of funds or the outflow of funds causes the
change in the position of fund.
Statement of Changes in Financial Position
The title of the Statement has undergone a number of changes over the years from where-
got and where-gone statement to the funds statement to statement of sources and
applications of funds to the newly recommended title of statement of changes in financial
position. Anyhow, depending upon the meaning attached to the word 'fund', different statements
.may be prepared as stated below.
1. Statement of Changes in Financial Position (Working Capital basis) popularly known as
'Funds Flow Statement';
2. Statement of Changes in Financial Position (Cash basis) popularly known as 'Cash Flow
Statement'; and
3. Statement of Changes in Financial Position (All Financial Resources basis).
In this work. the discussion is confined to the tirst two, viz .. Funds Flow Statement and
Cash Flow Statement. This Chapter deals with the Funds Flow State"ment and the next Chapter
discusses the Cash Flow Statement.
Funds Flow Statement
The Accountants differ in their opinion as to what constitutes the word fund. It has been
interpreted in different ways including as cash, short-term monetary assets, net monetary
assets, working capital and all financial resources. The word is used in both the narrower sense
Management Accounting : 236

as synonymous with cash and in the broader sense as inclusive of all financial resources. The
absence of a generally acceptable definition of 'fund' is one of the reasons as to why the Fund
Analysis has not made much headway. Anyhow, the most widely accepted view of 'fund' is the
one relating to 'working capital' wherein 'fund' is used to denote the Working Capital which is
the difference between Current Assets and Current Liabilities. As the Current Assets comprise of
cash also. cash is a part of the 'fund.' In this background, Funds Flow Statement has been defined
by Almand Coleman as .•. a statement summarising the significant financial changes which
have occurred between the beginning and the end of company's accounting period. In the
opinion of Smith and Brown. a Funds Flow Statement is prepared in summary form to
indicate changes accruing in terms of financial condition between different balance sheet
dates. The Funds Flow Statement, according to Robert Anthony, describes the sources from
which additional funds were derived and the uses to which these funds were put.
Significance or Utility or Managerial Uses of Funds Flow Statement
The following are the,important uses which the management can derive from the Funds
Flow Statement.
1. The Fun~s Flow Statement acts as a supplementary statement to the traditional Financial
Statements. viz., Balance Sheet, and Profit and Loss Account; -
2. The Funds Flow Statement furnishes the information about the sources from which the
company has mobilized the resources or fund during the year;
3. It also presents the details which spell out clearly the manner in which the mobilized fund
has been utilized or employed during the year;
4. It guides the management to evolve proper Dividend and Retention Policies, and also
about the issue of bonus shares;
5. Through the Funds Flow Statement, the management is able to plan for tpe retirement of
long.,term and other debts;
6. It helps to know how the changes in Working Capital took place and the factors which
caused the change in Working Capital;
7. It also sheds light on the efficiency of management in the Working Capital management;
8. The Statement is most useful to the lending authorities for the sanction of loans to the
company;
9. It guides the management about the allocation of resources and the priorities in the
allocation of resources;
lO. It presents in brief the financial consequences of transactions - both operational, financial
and investment.
Business Transactions and the Flow of Fund
It may be noted at this stage of analysis that for the purpose of Funds Flow Statement, the
items of Balance Sheet are classified into two broad categories viz .. Items of Current Accounts
and Items of Non-current Accounts. Items of Current Accounts include the Current Assets and
Funds Flow Statement: 237
Current Liabilifies. And the Non-current Accounts comprise of Non-current Assets and Non-
current Liabilities. The table presented below gives the different items under each category.
Items of Current and Non-current Assets and Liabilities

Curr~nt Alount Items

~
Current Assets Current Liabilities
Cash in Hand Bills Payable
Cash at Bank (including Fixed Trade or Sundry Creditors
Deposits)
Outstanding Expenses
Bills R-eceivable
Trade ~r Sundry Debtors Cash CreditlBank Overdraft

Inventory - Raw-materials, Work- Short-term Loans


in-progress, Finished Goods, Incomes Received in Advance
Stores, etc
Pre-paid Expenses Long-term Loans (or part) which
fall due for repaymertt within a
Outstanding Incomes year
Short-term Loans and Advances
Provision for Doubtful Debts
Temporary Investments, etc
Provision for Discount on Debtors

Non-current Account Items


t
Non-current Assets Non-current Liabilities
Land and Buildings Equity Share Capital
Plant and Machinery Preference Share Capital
Vehicles
Reserves and Surplus
Furniture and Fittings
Debenture Capital
Goodwill
Long-term Loans, etc
Patents, Trade Marks, Copy rights,
Preliminary Expenses
Profit and Loss Account (deticiency), etc
Management Accounting : 238
The word 'fund' is used to denote Working Capital. Funds flow, therefore, refers to the
changes in the Fund (Le., Working Capital) by the transactions - operational, financial and
investment. Though the effect of all the transactions on the Funds are considered, it should be
remembered here that not all the transactions cause the flow of Funds. There are certain
transactions which do not cause the tlow of Funds. For instance, when a transaction increases the
items of both the Current Assets and the Current Liabilities by an equivalent amount, then though
there will be an increase in the totals of both the Current Assets and the Current Liabilities, its
impact on Working Capital is zero. It is therefore necessary to analyse the different transactions
to find out whether they cause any change in the Fund or not. Hence, the analysis of transactions
is made under four heads viz.,
1. Transactions affecting the items of only Current Assets (Current Assets Vs Current
Assets),
2. Transactions Involving the items of only Current Liabilities (Current Liability Vs Current
Liability),
3. Transactions involving the items of Current Assets on the one hand, and the Currerit
Liabilities on other (Current Assets Vs Current Liabilities). and
4. Transactions affecting only the Non-current Items (Non-current Items).
1. Current Asset Vs Current Asset
The transactions which convert one item of Current Assets into another item of Current
Assets will have no effect on Fund. An illustration helps to understand this point. Assume that
the Current Assets of XYZ Company as on March 31, 2004 comprised of Rs. I lakh, Rs. 2 lakh
and Rs.0.5 lakh of Debtors, Stocks of Raw-materials and Bills Receivable respectively. During
the accounting year 2004-05, the company succeeded in collecting the entire amount due from the
customers and also collected 75% of Bil-Is Receivable. The total amount of Current Assets which
stood at Rs. 3.5 lakh on March 31, 2004 remained constant at the same level of Rs. 3.5 lakh even
on March 31, 2005. During this one-year period, the company con~cted the entire amount of
Debtors and 75% of the Bills Receivable. This had an impact of converting Debtors and a part of
Bills Receivable into another item of Current Assets, viz., cash. In other words, the reduction of
Rs. llakh in Debtors and Rs. 37,500 in Bills Receivable are followed bY.Rs. 1,37,500 increase in
cash balance of the company. The composition of the Current Assets, therefore. on March 31,
2005 comes to Rs. 2 lakh of Raw-materials, Rs. 12,500 of Bills Receivable and Rs. 1,37,500 of
Cash.
From the above, it is obvious that the transactions which convert one or more items of
Current Assets into another item or other items of Current Assets by an equivalent amount do not
cause any change in the Fund. Though these transactions do not cause any change in the Working
Capital which is used to represent the Fund. the transactions cause the flow (either inflow or
outflow or both) of Funds. Hence, they are to be accounted properly in the Funds Flow
Statement.
2. Current Liability Vs Current Liability
The transactions which replace one of the items 'of Current Liabilities by another item of
Current Liabilities will not cause any change in the Fund. An example may be cited to
Funds Flow Statement : 239

substantiate this point. Assume that a company is finding difficulty in meeting its current
obligations. It is only during this accounting year, smt of fifty years of its existence, that the
company is in tight short-term liquidity. Its banker, therefore. extended the overdraft facility to
the company. The Current Liabilities of the company as on April I. 20.04 comprised of Rs. I lakh
of Bank Overdraft, Rs. 5 lakh of Creqitors and Rs. 3 lakh of Bills Payable. During the year 2004-
05. the company has taken Rs. 5 lakh of overdraft facility and utilized the same for the purpose of
paying-off the Creditors fully.
Consequent to the above, Creditors of Rs. 5 lakh as on April 1, 2004 has been replaced by
an increase in Bank Overdraft by Rs. 5 lakh. Hence. the Current Liabilities as on March 31, 2005
comprise of Rs. 3 lakh of Bills Payable and Rs. 6 lakh of Bank Overdraft. But what is to be taken
cognizance of is the fact that the total Cunent Liabilities remained constant at Rs. 9 lakh as it was
on April I, 2004. That means. the transactions which convert one or more items of Current
Liabilities into another or other items of Current Liabilities will not cause any change in the Fund.
Still, they are to be accounted properly in the Funds Flow Statement as they cause the flow of
Funds.
3. Current Assets Vs Current Liabilities
The transactions which increase or decrease Current Assets causing a corresponding
increase or decrease in Current Liabilities by an equivalent amount will not cause any change in
the Fund. A simple illustration substantiates this point. Assume that a company hac; purchased
Rs. 5 la~h worth raw-materials on a three-month credit basis. This transaction increases both the
Stock (an item of Current Assets) and the Trade Creditors (an item of Current Liabilities) by Rs. 5
lakh each. The difference between the increase in the Current Assets and the increase in the
Current Liabilities is equal to zero. Therefore, the impact of this transaction on Funds is zero. As
a result. the amount of Funds will remain constant. Hence. the transactions which increase or
decrease both the Current Assets and Current Liabilities uniformly will not result in the change in
the Fund. In spite of this, they are accounted in the Funds Flow Statement as these transactions
cause t,he flow of Funds.
4. Non-Current Items
The transactions which relate to only the non-current items do not cause any change in the
Fund. It is because of the reason that they affect neither the Current Assets nor the Current
Liabilities - the two determinants of Working Capita\. For instance. a company acquiring a piece
of land for a price of Rs. 12 crore by issuing 1.2 crore equity shares of Rs. IO each. No doubt,
thi .. transaction affects both the assets and the capital. but it is not having any impact on Working
Capital as the items which this transaction affects are non-current items. Still they are to be
accounted in the Funds Flow Statement. This type of transactions are viewed in a different
manner. In the above example, issue of shares to the owner of land is to be understood as issued
for cash (i.e .. cash inflow) which the company utilizes for acquisition of land (i.e., cac;h outflow).
Therefore. this transaction is considered as causing both inflow and outflow of Funds by an
equivalent amount.
On the other hand, the following types of transactions cause the flow of funds (i.e.•
changes in Working Capital).
Management Accounting : 240

1. The transactions which increase the Current Assets and which (Le., the same transactions)
do not cause an equivalent increase in Current Liabilities, and vice versa; and
2. The transactions which decrease the Current Assets and which (Le., the same
transactions) do not cause an equivalent decrease in Current Liabilities, and vice versa.
On the basis of the above analysis, it may be said that the transactions between the items
of:
1. Current Assets and Fixed Assets (e.g .• purchase of fixed assets such as land, building,
macqine, etc., on cash basis or the sale of fixe.d assets either on cash basis or on short-
term credit basis);
2. Current Assets, and 'Capital and Long-term Liabilities' (e.g., issue of Shares and
Debentures to the general public for cash or raising long-term loans or repayment of loans
or redemption of Redeemable Preference Share Capital and Debenture Capital);
3. Current Liabilities and Fixed Assets (e.g., purchase of fixed assets on short-term loan
basis or sale of fixed assets on short-term credit basis); and
4. Current Liabilities and 'Capital and Long-term Liabilities' (e.g., conversion of short-
term loan into long-term loan and vice versa) cause the flow of Funds.
These transactions alter or change the Working Capital. The change in the Working
Capital may be in the form of increase in the Working Capital (positive) or decrease in the
Working Capital (negative). The increase' and the decrease are called inflow and outflow.
Whenever there is a change and if this change is in the form of an increase in the Working
Capital, it amounts to inflow of fund and the transaction responsible for this type of change is said
to be a source of fund. In the same analogy, when a change takes the form of decrease in the
Working Capital, it will imply outflow' of Fund and the transaction which causes this type of
change is consid~red as an item of use of fund. In order to study the flow of Fund. it is necessary
to analyse each and every transaction having a bearing upon the Working Capital. Of course, this
is a difficult task as the number of transactions are very large. Therefore. flow of Fund is
analysed only on a 'summary basis.
The Funds Flow Statement, also known as Statement of Sources and Applications of
Funds or Where Got and Where Gone Statement or Statement of Funds Supplied and
Applied or Statement of Funds Received and Disbursed or Funds Movement Statement or
Inflow-Outflow of Funds Statement, has been defined as a report on movement of funds
explaining where from working capital originates and where into the same goes dut-hlg an
accounting period. In simple, it is a statement showing the sources and uses of Funds.
Sources and Applications of Fund
As the Funds Flow Statement shows the sources from which the Funds have flown into
the organization and the manner or the way in which or the purposes for which Funds have been
used, it is necessary to study the different sources and uses of Fund. The sources and applications
of Fund are identified below followed by a brief explanation.
Funds Flow Statement: 241

Sources and Uses of Funds


Sources of Funds Uses of Funds
I
'Funds from Operation ... ~ Operating Losses (Funds depleted by
Operation)
Contribution from Shareholders---+
1+ Repayment of Long-Term Loans
Increase in Long-Term Liabilities
and Issue of Debentures . FUNDS .. ~ Redemption of Preference Shares and
Debentures
Sale of Fixed Assets and Long-
Term Investments ~ Acquisition of Fixed Assets and Long-
Term Investment
Gifts, Damages awarded in Legal
Action, etc ~ Payment of Cash Dividends

Income from Investments

It may be observed here that the items of Current Assets and Current Liabilities are not
incorporated in the above list. It may, further, be noted that, if the total of the Funds received
exceeds the total of the Funds applied, the difference is the excess of inflow which represents the
increase in the Working Capital. Similarly, if the reverse is the case (i.e., if the total of the Funds
applied exceeds the total of the Funds received). the difference represents the shortage in Funds
representing the decrease in the Working Capital.
(1) Funds from Operation
The Funds from Operation constitute a major source of Funds. The sale of goods and
services brings in revenue in the form of either cash or trade debtors or bills receivable. Whatever
may be the form of revenue, it has an impact on the Working Capital as all the form~ in which the
sales revenue is earned are the items of Current Assets and therefore. the items of Working
Capital. The sales revenue may. therefore, be reckoned as causing an inflow of Funds. In the
same way, the company incurs a number of items of expenses for the purpose of producing, and
selling goods and services causing an outflow of FlIIlu~. The difference between the sales revenlll'
and the cost of sales (i.e., the difference between the inflow and the outtlow of Funds) represents
the result or the outcome of the business or operational or trading activitico., that the company
carried out during the year and it will be in the form of either protit or loss depending upon the
quantum of revenue and cost. If the amount of revenue exceeds the amount of expenses incurred
during the year, it will result in profit which may be considered as an inflow of Funds. This is the
amount of profit generated by the operational activities. But Funds from Operation or the Funds
Depleted by Operation are not necessarily equivalent to the amount of profit or loss as reported by
the conventional Profit and Loss Account. It is. because of the reason that the profit or loss of
Profit and Loss Account is arrived at after considering a number of items which do not fall into
the category of trading or operational activities and which do not represent current drains on
Funds. A simple illustration may be used here to substantiate the fact that the Profit and Loss
Management Accounting : 242
Account includes, amongst others, some items which do not cause either the inflow or the outflow
of Funds.
Though the amount of depreciation represents the gradual recovery of the amount· of
capital already employed on fixed assets. it is viewed. for all practical and accounting purposes, as
a provision for the reduction in the value of the a<;set due to wear and tear. Further, though it is
charged to the Profit and Loss Account while ascertaining the result (in the form of profit or loss)
of the operation. it does not cause the outflow of cash or fund. Of course. acquisition of a fixed
a<;set causes an outflow of funds but not the depreciation. Depreciation is simply the spreading of
the capital or the acquisition cost of the asset over the expected useful life of the same on some
basis. Hence. it is only a non-cash expense involving no ca<;h outflow. In brief. it can be said that
though the depreciation is to be considered for, and charged to, Profit and Loss Account, the same
should not be reckoned for ascertaining the Funds. from Operation. It is, therefore, necessary to
take note of this while making some adjustments to profit or loss as revealed by the Profit and
Loss Account for the purpose of computing Funds from Operation. For instance, assume that a
company has reported Rs. 1.2 lakh profit after deducting all attributable expenses for the year
including Rs. 30.000 of depreciation on its fixed assets. In this case. it is necessary to add-back
Rs. 30.000 depreciation which has been debited to the Profit and Loss Account at the time of
computing Rs. 1.2 lakh profit. Therefore,
Profit as per Profit and Loss Account Rs. 1.20.000 •
Add-back depreciation which has been debited to Profit and Loss Account Rs. 30,000
:. Funds from Operation Rs. 1.50,000

From the above. it is obvious that the expenses which are chargeable against the revenue
of the period are of two types. viz .. cash and non-cash expense~. Non-cash expenses do not cause
the outflow of Funds though they are chargeable against the revenue to ascellain the profit or loss.
In order to anive at the Funds from Operation to be included in the Funds Flow Statement. "it is
necessary to re-compute the profit or loss figure (to represent the Funds from Operation) as
detailed below. Three different approaches are available to anive at the Funds from Operation.
They are (a) Conversion Method: (b).Restatement Method; and (c) Retained Profits Method.
Conversion Method

When the cunent year's.Protit and Loss Account is given. take the figure of net profit and
to this. make some adjustments as detailed below.
Computation of' Funds f'rom Operation
Amount !
Particulars Rs.
Rs.
Net Profit (a<; per Protit and Loss Account) a
Add: I. Non-cash Charges like Depreciation bl
Amount of Intangible Assets (like Goodwill. Patents, etc.,
written off) b2
Amount of Discount on Issue of Debentures writtl!n off b3
Funds Flow Statement: 243

Amount of Preliminary Expenses written off b4


2. Loss on Sale of Non-current Assets cl
Premium on Redemption of Debentures d
(The above adjustments are necessary to ascertain the
funds generated by Operation)
3. Amount of Profit transferred to General Reserve Account, dl
Provision for Taxation, Provision for Proposed Dividend, etc d2
(Note that the above three items are the items which are b+c+d
debited to the Profit and Loss Account. To arrive at the
Funds from Operation, these items are to be added back
to Net Profit) a+b+c+d
Less: I. Profit on Sale of Non-current Assets e

2. Profit on Redemption of Debentures and Preference Shares f


3. Interest received on Advances to Subsidiaries or Allied g
Companies
i
4. Dividend on Shares and Debentures
e+f+g+i
(Note that these are the items which are credited to the Profit
and Lo,ss Account and therefore, subtract these from the
profit to arrive at the Funds from Operation)
:.Funds from Operation la + (b + c + d) - (e + f +'g)J F

An illustration may be used to understand clearly the procedure pertaining to the


cOl1version of profit into Funds.
Illustration: 4.1

The following is the abridged Income Statement of M Company for the accounting year
just ended.
Cost of raw-materials consumed: Rs. Rs.
Opening stock 25.000
Add: Purchases made during the year 10.00,000
Cost of materials available for consumption 10,25,000
Less: Cost of unused materials 1,00,000 9,25,000
Conversion costs:
Wages and salaries 2.00.000
Management Accounting : 244
Depreciation . 3,00,000
Rent 50,000
Light and heat 10,000
Repairs and maintenance 50,000 6,10,000
Administrative, and selling and distribution expenses:
Salary 1,00,000
Rent 60,00q
Sales promotion 70,000
Carriages outwards 1,00,000 3,30,000
Other expenses:
Loss on sale of machine 10,000
Goodwill written-off 10,000
Patents written-off 5,000
Discount on issue of shares and debentures written-off 25,000 50,000
19,15.000
Sales Revenue 25.00.000
Profit on sale of building 1,25.000 26.25.000
Profit 7.10.000

From the above, compute using Conversion Method. the amount of funds generated by
the operating activities during the year just ended.
Solution:

Statement Showing the Funds from Operation


Amount
Particulars Rs.
Rs.
Protit as per Profit and Loss Account . 7.10,000
Add: Non-cash Expenses - Depreciation 3,00,000
.
Non-operating Expenses written off:
Loss on sale of Machine 10,000
Goodwill 10,000
Patents
. 5,000
Funds Flow Statement: 245

Discount on issue of Shares and Debentures 25,000


3,50,000
10,60,000
Less: Profit on sale of building 1,25,000
Therefore, Funds from Operation 9,35,000

This is how the Funds from Operation is ascertained with the help of the profit and other
infOJ:mation.
Restatement Method

Under this method, the conventional Profit and Loss Account is restated by considering
only those trading expenses and revenues which cause movement of Funds. The illustration taken
earlier [Illustration: 4.1] is used here also to show how the Funds from Operation is ascertained by
restating the income statement.
Restated or Adjusted Income Statement
Amount
Particulars Rs.
Rs.
Sales revenue 25,00,000
Less: Cost of Materials consumed 9,25,000
Conversion Costs:
Wages and Salary 2,00,000
Rent 50,000
Light and Heat 10,000
Repairs and Maintenance 50,000
Administrative, and Selling and Distribution Expenses 3,30,000 15,65,000
Therefore, Funds from Operation 9,35,000

Retained Profit Method


Retained profit may be in the form of either the Profit and Loss Account balance or the
General Reserves or in some other forms. The retained profit figures are used with a view to
ascertain the amount of profit earned during the year and retained in the company. Under this
approach, the retained profit figures in the Balance Sheet and also some of the items of Profit and
Loss Account are used to ascertain the Funds from Operation. For instance, assume that the
Balance Shee~ of a company shows an opening General Reserve balance of Rs. 8 lakh and a
closing balance of Rs. 10 lakh. Before transferring the net profit to the General Reserve Account,
Management Accounting : 246

the company has paid a dividend of 10% to equity shareholders on their capital of Rs. 30 lakh; 8%
and 9% to debentureholders and preference shareholders on their capital contributions of Rs. 10
lakh and Rs. 20 lakh respectively. While ascertaining the profit figure, the company has provided
for Rs. 3 lakh depreciation and other Rs. 25,000 of goodwill, discount on issue of shares and
debentures. etc., has been written-off. Using this. the amount of Funds· from Operation can be
computed as shown below.
Statement Showing the Computation of Funds from Operation
Amount
Particulars Rs.
Rs.
Balance in General Reserve Account at the:
End of the period 10,00,000
Beginning of the period 8,00,000
:. Profit earned and transferred to Reserve Account during the year 2,00,000
Add: Depreciation 3,00,000
Goodwill. etc., written-off 25,000
Dividend or Interest on: Equity Shares 3,00,000
Preference Shares 1,80,000
Debentures 80,000 8,85,000
Therefore, Funds from Operation 10,85,000.

This way the amount of Funds from Operation can be ascertained by following anyone of
the above three methods. The specific approach to be followed in a given situation rests on the
type of information available. After a careful examinatiQn of the problem, one has to decide the
method to be followed to ascertain the amount of Funds from Operation. Further, irrespective of
the method to be used, it is necessary to ensure that no transaction which involves non-cash
expenses and no transaction pertaining to non-trading activity are considered while computing the
Funds from Operation.
(2) Issue of Shares and Debentures
The actual amount of cash received by the issue of shares and debentures would
constitute inflow of Funds. The shares and debentures may be issued at PM, at premium or at
discount. But what is important is the actual amount of cash received. Tile inflow of Fund is to
the extent of consideration (i.e., including premium or net of discount).
Illustration: 4.2
From the facts and figures given below for a company, find out the amount of cash
inflows resulted from the issue of shares and debentures to the general public. The company
issued:
Funds Flow Statement : 247

a. 1.00.000 Equity Shares at their face value of Rs. 100 each;


b. 2.00,000 8% Preference Shares of Rs. 100 each at 25Cf( premium: and
c. 1,00,000 10% Debentures of Rs. 1.000 each at a discount of 25%.
All the shares and debentures were subscribed for by the general public and the company
was able to collect the total amount from the general public without fail.
Solution:
Amount of Cash Inflows from the Issue of Shares and Debentures
Amount
Particulars
Rs.
Amount of Cash Inflows from the issue of:
1,00,000 Equity Shares Rs. 100 each at par (i.e .. Rs. 100) 1,00,00.000
2,00,000 8% Preference Shares of Rs. 100 each at 25<k premium
(i.e., at Rs. 125) 2,50,00.000
1.00.000 10% Debentures of Rs. 1.000 each at 25% discount (i.e .. at Rs. 750) 7,50,00,000
:. Cash Inflows or Funds from the Issue of Shares and Debentures 11,00,00,000

However, it should be noted that the following types of issues will not result in the inflow
of Funds or cash.
a. Issue of Shares, out of Reserves, in the form of Bonus Shares does not cause the
inflow of Fund a'i the company pays dividend in the form of shares but not in the form of
cash. Consequently, it will not result in the inflow of Funds as it will not affect any of the
items of Working Capital. Because, the issue of bonus shares lowers the accumulated
profit or reserves and increases the share capital. It may be remembered here that both
the accumulated profit or reserves and the share capital are not the items of Working
Capital.
b. Issue of Shares and Debentures for consideration other than those which go into the
Working Capital. A simple illustration helps to understand the point. Assume that a
company has purchased a machine for a price of Rs. 10 lakh by issuing 80,000 equity
shares of Rs. 10 each issued at a premium of 25%. An analysis of this illustration reveals
that this transaction affects both the Equity Share Capital and the Machine Accounts -
both are not the items of either the Current Assets or the Current Liabilities or the
Working Capital. The outcome of this is that the transaction will not cause any inflow of
Funds. It is because of the reason that this transaction affects only Non-current Items. As
this type of issue will not cause the flow of Funds. it should not be treated as causing an
inflow of Funds while preparing the Funds Flow Statement. In the same analogy, the
issue of shares for the supply of technical know-how. underwriting. etc .. should not be
treated as inflow of Funds.
Management Accounting: 248
c. Issue of Shares in Exchange for Loan or Debentures, etc. For instance, assume that a
cOl:npany has decided to discharge both the loan capital and the debenture capital by
issuing equity shares. The company is having a loan capital of Rs. 15 lakh and a
debenture capital of another Ra. 30 lakh. The company's present equity share capital
comprises of 2,00,000 equity shares of Rs.1O each issued at par. Now the company has
decided to discharge the entire long-term loan capital by issuing equity shares of Rs. 10
each at a premium of Rs. 5 each. From this, it is obvious that the company has issued
another 3 lakh equity shares of Rs. 10 each at 50% premium. After the issue, both the
loan capital of Rs. 15 lakh and the debenture capital of Rs. 30 lakh will be wiped out
fully; and equity share capital will record an increase of Rs. 30 lakh (3,00,000 equity
shares x Rs. 10) and there will be a creation of a new account called 'Share Premium
Account' with a credit balance of Rs. 15 lakh (3,00,000 equity shares x Rs. 5). What is to
be taken cognizance of from the above is that, this financial transaction has affected
neither the Current Liabilities nor the Current Assets but only the Non-current Items.
Therefore, it has not caused any inflow of Funds.
(3) Increase in Long-Term Liabilities
It is the experience of the entire corporate world that the corporate undenakings are
neither able nor desire to carry on their activities with the help of only the equity capital. They
prefer to achieve a right balance between loan capital and share capital. One of the important
advantages of loan capital is that the interest charge is, a deductible item for ascertaining the
taxable income. Therefore, the companies wish to raise even the loan capital. Whenever the
loans are raised either in the form of secured or unsecured loans, it will cause an inflow of Funds.
But, as identified under the head 'Issue of Shares and Debentures' above, the IQans obtained in the
form of supplies or services of a non-current nature will not constitute the inflow of Funds.
(4) Sale of Fixed Assets and Long-Term Investments
The amount of revenue realized from the sale of fixed assets and long-term inv~stments
would be the inflow of Funds to the extent of actual sale proceeds. That means, the inflow of
Funds includes even the profit on the sale of fixed assets and investments. This is the reason as to
why the profit (loss) on the sale of fixed assets and long-term investments is subtracted (added
back) from (to) the Profit and Loss Account's profit figure while arriving at the Funds from
Operation. A simple illustration may be t~ken to understand this point.
Illustration: 4.3
The Balance Sheet of a company for the year 2005 includes, amongst other items',
information about machines. The statement states that the value of machinery held by the
company increased from Rs. 80,000 on January 1. 2005 to Rs. 2,00,000 by December 31, 2005.
During the year 2005. the company sold a machine for a price of Rs. 10,000. The written-down-
value of the, machine sold wa'i Rs. 12,000. For the current year, the company charged Rs. 10,000
depreciation to the Profit and Loss Account. Analyse the transactions to find out which of these
cause the flow of funds and which do not.
Funds Flow Statement : 249
Solution:
Dr MachineA/c Cr
Amount Amount
Particulars Particulars
Rs. Rs.
To Balance bId 80,000 By Cash (sale of machine) 10,000
To Cash (balancing figure; assumed By Profit and Loss Alc
to be cash purchases) 1.42.000 (Loss on sale of machine) 2,000
By Profit and Loss Alc
(Depreciation for the year) 10.000
By Balance cld 2,00,000
2,22,000 2,22,000

From the above Account, the following points become very clear.

a. Rs. 10,000 depreciation which has already been debited to Profit and Loss Account is to
be added back to profit or loss of Profit and Loss Account for the purpose of ascertaining
the Funds from Operation. Because, it is a non-cash expenditure and it does not involve
cash outflow;
b. Rs. 10,000 sale proceeds received is to be considered as cash inflow or' as a source of
Funds in the Funds Flow Statement;
c. Rs. 2,000 loss on the machine sold has already been debited to the Profit and Loss
Account and therefore, it is to be added back to the profit of Profit and Loss Account
while ascertaining the Funds from Operation as the loss on sale of machine does not fall
into the category of operating activities. It is a non-operating activity and therefore, it
should be added back to the profit for ascertaining the Funds from Operation: and
d. Rs. 1.42,000 of cash purchases should be considered as cash outflow in the Funds Flow
Statement.
(5) Repayment of Long-term Loans, and Redemption of Preference Shares and Debentures
Except the non-refundable capital (such as, equity capital), all other capitals (both the
Debenture Capital, Redeemable Preference Share Capital and other long-term loan capital) are to
be returned to the parties from whom the entity raised the same. Repayment of some capital takes
place as per the agreement and some others, as and when the company finds adequate financial
resources. Repayment of capital - either the share capital or the loan capital - would. therefore.
entail an outflow of Funds. In order to determine the amount of fund flowing out. it is necessary
to consider even the premium or discount on redemption. It should also be noted here that if any
capital or long-term loan is discharged by other forms of shares or securities, it will not cause any
flow of Fund. This point becomes very clear from the following illustration.
Management Accounting : 250

Illustration: 4.4
Assume that the capital structure of a company is as follows.
Equity Share Capital: 10 lakh Shares of Rs. 10 each =Rs. 100 lakh
Redeemable Preference Shares: 10,000 Shares of Rs. 100 each = Rs. 10 lakh
Redeemable Debentures: 1,000 Debentures of Rs. 1,000 each = Rs. 10 lakh
Convertible Debentures: 1,000 Debentures of Rs. 1,000 each = Rs. 10 lakh
The management has taken a major decision of discharging some of its liability and
altering the composition of capital. The salient features of the decision are summarized below.
(a) The company has decided to redeem all the Preference Shares at 10% premium;
(b) All the Redeemable Debentures are to be redeemed at par;
(c) The management has also decided to convert the Convertible Debentures by issuing 50
equity shares of Rs. 10 each at 100% premium for each debenture held.
Determine the outflow of funds.
Solution:
Redemption of Preference Shares [10,000 Shares x (Rs. 100 + 10% of Rs. 100)J Rs. 11,00,000
Redemption of Debentures (1,000 Debentures x Rs. 1,000) 10,00,000
.'. Outflow of Funds 21,00,000

Note that the decision .pertaining to the conversion of debentures into equity shares will
not cause any flow of Fund as it is between two Non-current Items. It may be noted here that Rs.
10 lakh of Convertible Debenture Capital is to be replaced by Rs. 5 lakh increase in Equity Share
Capital (i.e., 1,000 convertible debentures x 50 equity shares x Rs. (0) and Share Premium
Account of Rs. 5 lakh (50,000 equity shares x Rs. 10 premium).
(6) Acquisition of Fixed Assets
Fixed assets are very essenfial for the purpose of carrying out any productive activity.
The acquisition of fixed assets involves a very huge capital outlay. The impact of acquisition of
these assets on the flow of Fund depends upon the agreement between the buying-company and
the selling-company. If purchase of tixed assets like land, machine, etc., is made on cash basis, it
reduces the cash balance and therefore. reduces the Current Assets. In other words, it results in an
outflow of cash which is a part of Fund. Even the acquisition of these fixed assets on short-term
credit basis increases the Current Liabilities resulting in the reduction in the Working Capital
which is used to repre~ent Funds. This type of investment transactions would therefore entail the
use of Funds. .
Funds Flow Statement: 251

mustration: 4.5
A company acquired Rs. I crore worth automatic machine as a part of its mechanization
and automation plan envisaged by the management. The supplier agreed to receive the price of
the machine as follows.
Rs. 50 lakh cash immediately;
Rs. 20 lakh is to be paid within three months:
Another Rs. 20 lakh to be paid in the form of 2,00,000 equity shares of Rs. 10 each to be
issued at par; and
The rest Rs. 10 lakh to be paid within a period of 5 years.
Find out the outflow of funds.
Solution:

(a) Purchac;e price of the machine: Rs. 100 lakh (c) Outflow of Funds associated with the
investment transaction:
(b) Mode of Payment:
Cash (down-payment) Rs. 50 lakh Down-payment (cash) Rs. 50 lakh

Short-term loan Short-term loan Rs. 20 lakh


Rs. 20 lakh
Equity shares Therefore, Outflow of Fund Rs. 70lakh
Rs. 20 lakh
Long-term loan Rs. 10 lakh
Rs. 100lakh
Preparation of Funds Flow Statement
While preparing the Funds Flow Statement, individual items of Current Assets and
Current Liabilities are not shown separately but they are consolidated in a separate statement
called Schedule of Changes in Working Capital and only the net change in the Working Capital
during an Jccounting year is taken to the Funds Flow Statement. The Funds Flow Statement,
therefore, comprises of two parts, viz., Schedule of Changes in Working Capital and Statement
of Sources and Uses of Funds.
(1) Schedule of Changes in Working Capital
This schl!dule is prepared to find out the amounts of Working Capital at the end of two
years [one being the end of the previous year (i.e .. opening balance for the current year); and other
being the end of the current year (i.e., closing balance)] and abo the increase or decrease in the'
individual items of Current Assets and Current Liabilities,
An increase in the amount of an item of Cunent As!>ets in the cunent year as compared to
the previous year (Le., at the end of the 'previous year) represents an increase In the Working
Capital. Similarly, a decrease in the amount of an item of Current A~sets in the current year when
compared to the previous year would represent a reduction in the Working Capital. In the same
Management Accounting : 252

way, different items of Current Assets and Current Liabilities are analyzed and presented in the
scheduler In this regard, the following points are worth mentioning.
(a) The increao;e in the items of Current Assets increases the Working Capital. A simple
illustration helps to verify this conclusion.
As on December 31, ... Changes in Working
Particulars
2004 (Rs.) 2005 (Rs.) Capital (Rs.)
Current Assets:
Stocks 2,00,000 3,00,000 1,00,000
Debtors 1.00.000 1.50.000 . 50,000
Cash 3,00,000 5,00,000 2,00,000
6.00.000 9,50,000 3,50,000
Current Liabilities 4,00,000 4,00,000 0
Working Capital 2.00,000 5,50,000 3,50,000

It may be noted from the above comparative figures that the increase in the Working
Capital comes to Rs. 3.5 lakh (i.e .. Rs. 5.5 lakh - Rs. 2 lakh). This increase is due to Rs. 1 lakh
increase in stocks, Rs. 50.000 increase in debtors and Rs. 2 lakh increase in cash. That means, the
increase in Current Assets if other things remain constant, increases the Working Capital by an
equivalent amount as shown above.
(b) The increase in the items of Current Liabilities decreases the Working Capital.
Another example may be taken to verify whether this conclusion is right or not.
As on December 31, ... Changes in Working
Particulars
2004 (Rs.) 2005 (Rs.) Capital (Rs.)
Current Liabilities: I

Trade Creditors 2,00,000 3,00,000 1,00,000.


Bills Payable 1.00,000 1,50,000 50,000
Bank Overdraft 3,00,000 4,00,000 1.00,000
6,00,000 8,50,000 2,50,000
Current Assets , 9,00,000 9,00,000 0
Working Capital 3,00,000 50,000 -2,50,000

From the above. it is apparent that the increase in the 1l1:111'i of Current Liabilities
decreases the amount of Working Capital, if other things remain c( 'ilstant. by an equivalent
amount. The amount pf Working Capital has decreased from Rs. 3 lakh 1m December 31, 2004 to
Rs. 50,000 by December 31. 2005. It is due to the fact that the items of Current Liabilities have
Funds Flow Statement: 253
registered the upward trends resulting in the decrease in the gap between the total of Current
Assets and Current Liabilities.
(c) The decrease in the items of Current Assets decreases the Working Capital. Assume
that the amounts of Current Assets and Current Liabilities were Rs. 10 lakh and Rs. 8 lakh
respectively on December 31, 2004. By December 31. 2005, though the Current Liabilities
remained at Rs. 8 lakh, the amount of Current Assets decreased to Rs. 9 lakh.
It can be seen from the above that, the decrease in the Current Assets by Rs. 1 lakh from
Rs. 10 lakh on December 31, 2004 to Rs. 9 lakh by December 31, 2005 has caused the reduction
in the Working Capital (i.e., Fund) by Rs. 1 lakh (i.e., Rs. 2 lakh on December 31,2004 to Rs. 1
lakh by December 31, 2005). That means, the reduction in the amount of Current Assets reduces
the Working Capital.
(d) The decrease in the items, of Current Liabilities increases the Working Capital. For
instance, the reduction in the Current Liabilities from Rs. 5 lakh to Rs. 3 lakh increases the
Working Capital by Rs. 2 lakh if the amount of Current Assets remains constant.
A Note on Treatment of Proposed Dividend
Strictly speaking, proposed dividend is not an item of Current Liability. Because. unless
the dividend is declared, it is not a liability. Since the company is going to make, formal
declaration (in the General Meeting of Shareholders) of what it has proposed, the proposed
dividend may be considered as being declared. Therefore, two alternative treatments are
available. One, as Current Liability and two. as an Appropriation of Profit (payment of which is
left to the discretion of management).
If it is treated as an appropriation (i.e., not as a Current Liability), it will not appear in the
Schedule of Changes in Working Capital. The proposed dividend for the current year will be
added back to 'profit' for the current year to find out the Funds from Operation. If the profit for
the current year, given in the problem, is before provision for dividend, then there is no need for
adding back to profit (to arrive at the Funds from Operation) as the same (i.e .. proposed dividend)
was not deducted to arrive at the profit figure. The proposed dividend for the previous year is
shown as application of fund in the Funds Flow Statement assuming that the previous year's
proposed dividend is paid during the current year.
If the proposed dividend is reckoned as an item of Current Liability. then the proposed
dividend (both for the current and the previous years) will appear in the Schedule of Changes in
Working Capital. Therefore, dividend paid during the current year (either against the proposed
dividend for the previous or the current year) will not appear in the Funds Flow Statement as
application of Funds. Further, no adjustment to profit is required to arrive at the Funds from
Operation. [Alternatively, proposed dividend for the current year may be added back to profit to
arrive at Funds from Operation and showing the same (proposed dividend for the year) in the
Funds Flow Statement a..<; being paid during the same current year].
In the case of interim dividend for the current year, it is to be added back to profit to
arrive at Funds from Operation and it is to be shown as the application of Fund. If Funds from
Operation are computed on the basis of the profit before provision for interim dividend, then no
adjustment is required (to be made to profit) to arrive at Funds from Operation.
Management Accounting : 254
A Note on Provision for Taxation
As far as the Provision for Taxation is concerned. the following procedure is followed.
I. Provision for Taxation made during the year is to be charged to the debit side of ,Adjusted
Profit and Loss Account. This provision may be computed by preparing a separate
account called Provision for Taxation Account. The opening balance is credited to this
Account and the closing balance to the debit side. The amount of tax paid is also debited
to this Account. The balancing figure (on the credit side) represents the provision made
during the year. And it should be transferred to the debit side of Adjusted Profit and Loss
Account to arrive at the Funds from Operation.
2. The amount of tax paid during the year will be shown in the Funds Flow Statement as the
application of Funds.
Schedule of Changes in Working Capital
Changes in Working
At the end of ...
CaJital
Particulars
Previous Current Increase Decrease
Year (Rs.) Year (Rs.) (Rs.) (Rs.)
Current Assets .
Cash on Hand and at Bank r

Debtors
I
Bills Receivable
Stock
Prepaid Expenses
Accrued Income
Total Current Assets (a) A B
Current Liabilities
Ban-k Overdraft
Creditors
Bills Payable
Outstanding Expenses
Unclaimed or Unpaid Dividend
Total Current Liabilities (b) C D
Working Capital (a - b) A-C=E B-D=F
Total Increase and Decrease in Working Capital X Y
Increase or Decrease in Working Capital Y-X* X-y*
Total
Note: * Any .one of these two will appear in the Schedule and the Statement
If F is greater than E. there was be an increase in the Working Capital. On the other hand.
if E is greater than F. there was a reduction in the Working Capital. Only the increase or decrease
in the Working Capital is taken to the Funds Flow Statement. An increase in the Working Capital
Funds Flow Statement : 255

is reckoned a~ the application of Funds and the decrease ill the Working' Capital as th~ ~ource"of
Funds.
(2) Statement of Sources and Application of Funds
This is the most important part of Funds Flow Statement. This statement is prepared on
the basis of other items of Ba1ance Sheets (Le., the items other than those which are considered
for preparing the Schedule of Changes in Working Capital). This statement is, therefore, prepared
with the help of the changes in the fixed assets, share capital and long-term liabilitie'i which can
be ascertained on the basis of the values of these items presented in the Balance Sheets. Further.
if any other information is available, it should also be considered if it results in the fluw of Fund
There is no standard format for the preparation of the Funds Flow Statement. It may be prepared
in the form of either an account or a statement. But what is important IS the proper presentation of '
all the important financial events of the year. A format of each type is presented below.
(a) Statement Form of Funds Flow Statement

Statement of Sources and Uses of Funds of ABC Ltd., for the year ended ...

Particulars I Amollnt-l
-i-- -B~ __--i
Sources of Funds
I
Funds from Operation"',
I!.sue of Shares and Debentures
Long-term Loan
Sale of Investments. Fixed Assets. etc:
Non· trading Income I
Decrease in Working Capital (as per the Schedule of Changes in Working!
Capital)'"
Total Inflow
)--------1
Applications of F'unds
Fund depleted by Operation*
Redemption of Preference Shares and Debentures
Repayment of Loan
Purchase ot Investments, Fixed Assets. etc
Non-trading Expenses
Increase in Working Capital (as per the Schedule of Changes in Working
Capitalt

I
Total Outflow
-------------------------------------
(b) Account Form of Funds Flow Statement
Management Accounting : 256

Statement of Sources and Uses of Funds of ABC Ltd., for the year ended ••.
Amount Amount
Sources Applications
Rs. Rs.
Funds from Operation* Funds depleted by Operation*
Issue of Shares and Debentures Redemption of Preference Shares
Long-term Loans and Debentures
Sale of Investment. Fixed Assets. etc Repayment of Loan
Non-trading Income Purchase of Investments, Fixed
Decrease in Working Capital+ Assets. etc
Non-trading Expenses
Increase in Working Capital+

Note: * Anyone of these will find the place in the Statement


+ Anyone of these will find the place in the Statement
Limitations of Funds Flow Statement
The following are some of the limitations of Funds Flow Statement. These limitations are
to be kept in mind while drawing any conclusion about the operating results and financial
soundness on the basis of the Funds Flow Statement.
1. Funds Flow Statement is being criticised as the one which re-arranges the financial data
extracted from the Financial Statements;
2. The Funds Flow Statement is also being criticized as not furnishing anything new and/or
original over and above the conventional Financial Statements:
3. Funds Flow Statement is also based on the historical data ali it bases its preparation on the
conventional Financial Statements; and
4. Some of the transactions affecting only the Non-current Items are, sometimes, not
considered while preparing the Funds Flow Statement.
Illustration: 4.6
Calculate Funds from Operations from the following details.
Profit and Loss Account for the year ended 31-12-1998

To Salaries Rs.30,000 By Gross Profit Rs.80,000

To Rent 5,000 By Profit on sale of Machinery 10,000

To Loss on Sale of Furniture 5,500 By Interest and Dividend 5,500

To Sundry Expenses -- 10,000


To Goodwm 8,000
To Preliminary Expenses 2,000
Funds Flow Statement : 257

To Depreciation 13,000

To Net Profit 22,000

95,500 95,500
[Mangalore Uni., B.Com, October 1999J
Solution:
Gross Profit Rs.80,000

Less: Salaries Rs.30,000

Rent 5,000
Sundry Expenses 10,000 45,000

:. Fund from Operation 35,000

Illustration: 4.7
From the following Profit and Loss Account of Sreyus Co .. Ltd .. calculate the Funds from
Operation.
Profit and Loss Account for the year ended 31 st December ] 999

To Salaries Rs. 56,000 By Gross Profit Rs. 3.00,000

To Rent 24,000 By Profit on sale of Machinery 24.000


To Printing and Stationery 12,000 By Interest on Investment 16,000

To Depreciation of Plant 50,000

To Goodwill written off 20.000


To Loss on sale of Plant 10,000

To Provision for Taxation 30,000

To Proposed Dividend 28,000

To Net Profit taken to Balance


Sheet 1,10.000

3,40.000 3,40,000

[Mangalore Uni., B.COIll, October 2001J


Management Accounting : 258
Solution:
Gross Profit Rs. 3.00.000
Less: Salaries Rs.56,000

Rent 24,000

Printing and Stationery 12,000 92,000

:. Fund from Operation 2.08,000

Alternatively,

Net Profit [as per Profit and Loss Account] - Rs. 1,10,000

Add back : Depreciation on Plant Rs. 50.000

Goodwill written off 20.000

Loss on sale of Plant 10.000

Provision for Taxation 30.000

Proposed Di vidend 28.000 1.38.000

2,48,000

Less: Protit on sale of Furniture Rs.24,000


Interest on Investment 16.000 40.000

:. Fund from Operation 2,08.000


- - .. - - -

Illustration: 4.8
From the following. calculate funds from operation.
Net Loss Rs.l.00,oon
Depreciation on Furniture 1,50,000
Amortization of Goodwill 1.00.000
Loss on sale of Plant 50.000
Profit on !oale of Land 80.000
Provision for likely Bad Deht -J IWOO
[KlIvempll U";,, B.COIII., Ot·tober 1999 alld May 2001J
Funds Flow Statement: 259

Solution:
Net loss Rs.l,OO.OOO
Add Back: Depreciation on Furniture Rs.l,50,000
Amortization of Goodwill 1,00,000
Loss on sale of Plant 50,000 3,00.000
[- Rs. 1,00,000 + Rs. 3,00,000] 2,00,000
Less: Profit on sale of Land 80,000
:. Fund from Operation 1,20,000
Illustration: 4.9
From the following Profit and Loss Account of Swaraj and Co., Ltd., calculate the funds
from operations.
Profit and Loss Account for the year elided 31 ,( December 1999
I
Particulars Rs. Particulars I Rs. I
I I
To Salaries 5,000 By Gross Profit I 50.000 I
! By Profit on sale uf
To Rent 3,000
Buildings:
i I
To Depreciation on Plant 5,000 Book value R,. 10.000 I I
To Printing and Stationery 3,000 Selling price __ ~.000 5.000 I
!
To Preliminary Expenses I I
written off 2,000 !
I

To Goodwill written off 3,000 Ii

4,000
i
To Provision for Tax
I
To Proposed Dividend 6.000 II I
I
I

To Net Profit taken to Balance Sheet 24,000


I

I
I
I
I
I
55.000 I
i -5 000 !I
).
I I .. - --~-.- - _ . j
Note: Assume that Provision for Tax and Proposed Dividend arc currenr Iiabilitie ....
[Mallgaiore Ulli, B.eom., October 2fJ02/
Management Accounting: 260
Solution:
Gross Profit Rs.50,000
Less: Salaries Rs. 5,000
Rent 3,000
Printing and Stationery 3,000 11,000
------------------
:. Fund from Operation 39,000

Other items are either book cost items or non-operating items. Hence, the same are not
considered while computing the Fund from Operation.
lliustration: 4.10
Birla Cements presents the following information and you are required to calculate funds
from operation:
Profit and Loss Account
To Operation Expenses Rs.l,OO,OOO By Gross Profit Rs. 2,00,000
To Depreciation 40,000 By Gain on sale of Plant 20,000
To Loss on sale of Building 10,000
To Advertisement Suspense Nc 5,000
To Discount (allowed to customers) 500
To Qiscount on issue of shares written off 500
To Goodwill written off 12,000
To Net Protit 52,000
2,20,000 2,20,000

[Manga/ore Uni., B.Com., April 2001]


Solution:
The following items of Profit and Loss Account are not considered for computing the
Fund from Operation.
Book Costs: Depreciation
Discount on issue of shares written off
Goodwill written off
Advertisement Suspense Account
Non-operating Items: Loss on sale of Building
Gain on sale of Plant
Funds Flow Statement: 261

Hence, the Fund from Operation is calculated by considering only operating activities
which caused the flow offund as below.
Gross Profit Rs.2,OO,000
Less: Operating expenses Rs.l,OO,OOO
Discount (allowed to customers) 500 1,00,500
:. Fund from Operation 99,500

lllustration: 4.11
From the following particulars, ascertain the Fund from Operations
1-1-1995 31-12-1995
PIL Appropriation Alc Rs.45,OOO Rs.60,OOO
General Reserve 15,000 18,000
Goodwill 15,000 12.000
Preliminary expenses 8,000 5,000
Provision for Depreciation on Plant 8,000 12,000
[Kuvempu Uni., B.Com., May 1994J
Solution:
Profit and Loss Appropriation Account balance on:
December 31, 1995: Rs.60,000
January I, 1995: 45,000
:. Profit earned during 1995 and retained in the company Rs.15,OOO
Add Back*: Profit transferred to General Reserve 3,000
Goodwill written off 3,000
Preliminary expenses written off 3,000
Depreciation 4,000
13,000
:. Fund from Operation 28,000

[* These are the non-operating items and/or items which do not cause the outflow of fund. These
have reduced the amount of profit. But, as is known very well, they do not lower the amount
of Fund from Operations]
Management Accounting : 262

Illustration: 4.12
From the following Profit and Loss Account, compute Funds from the Operation.
Depreciation Rs.1.800 Gross Profit Rs.17,000
Discount on issue of shares 400 Profit on sale of plant 8,000
Loss on sale of Machinery 800
Goodwill written off 4,400
Preliminary Expenses written off 1,300
Sundry Expenses 5,000
Net Profit 11,300
25,000 25,000

[Kuvempu Uni, B.Com, May 1997J


Solution:
Gross Profit Rs.17,000
Less: Sundry expenses 5,000
:. Fund from Operation 12,000
Other items of Profit and Loss Account are either book costs causing no outflow of fund
or non-operating items. Hence, they are not considered.
Illustration: 4.13
Prepare a Statement of Changes in Working Capital, from the following Balance Sheets
of Sri Vardhaman and Company Limited.
Liabilities 1998 1999 Asse~s 1998 1999
Equity Share Capital 10.00,000 10,00,000 Fixed Assets 12,00,000 14,00,000
Long term
Debenture~ 7,40,000 9,00,000
Investment 4,00,000 2,00,000
Tax Payable 1,54,000 86,000 Work in progress 1.60,000 1,80,000
Accounts Payable 1,92,000 3.84,000 Stock in Trade 3,00,000 4,50,000

Interest Payable 74,000 90.000 Accounts


Receivable 1,40,000 2.80.000
Dividend Payable 1,00,000 70.000 Cash 60.000 20,000

22,60,000 25.30,000 22,60,000 25,30,000

[Mallgaiore Uni, B.Com., October 2001J


Funds Flow Statement: 263

Solution:
Statement of Changes in Working Capital
-
Amount (R~.) as on Changes in Working
I Particular~ December 31. Capital (Rs.)
I 1998 1999 Increase Decrease
Current Assets:
Work-in-Progress 1,60,000 1,80,000 20,000 -
Stock-in-trade 3,00,000 4,50.000 1,50,000 -
Accounts Receivable 1,40,000 2,80,000 1,40,000 -
Cash 60.000 20,000 - 40.000
(a) 6,60,000 9,30,000
Currerit Liabilities:
Tax Payable 1,54,000 86,000 68,000 -
Accounts Payable 1,92,000 3,84,000 - 1,92,000
Interest Payable 74.000 90,000 - 16,000
I
Dividend Payable 1.00.000 I 70,000 30,000 -
(b)
r
5.~0.OOO I 6.30.000
i
i
:. Working Capital (a'- b) IAO.OOO I 3.00.000
r

Changes in Working Capital I 4.08.000 2,48.000


!
. Net increase in Working Capital 1.60,000 - - 1,60,000
1--
3,00.000 3.00.000 4.08.000 4,08.000

Illustration: 4.14
From the following Trial Balance of A Ltd .. you are required to prepare a schedule of
changes in working capital.
Trial Balance
1998 1999
Particulars
Dr. (Rs) Cr. (Rs) Dr. (Rs) Cr. (Rs)
I
r
Capital -- 80,000 -- I 85,000
I Mortgage
i -- -- -- I 5.000
I
I

I Land and Buildings 50.000 j -- 50,000


I
--
I PI am and Machinery .
_ _ _ _ _ _ _ _ _ _ ---1.____
24.0~ _ ..
-
II
I
34.000
I --
Management Accounting : 264

Stock 9.000 -- 7.000 --


Debtors 16.500 -- 19,500 --
Cash at Bank 4.000 -- 9.000 --
Profit and Loss Alc. -- 14.500 -- 24,500
Creditors -- 9,000 -- 5.000
1,03,500 1,03,500 1,19,500 1,19,500

[Bangalore Uni., B.Com., May 2000}


Solution:
Schedule of Changes in Working Capital
Amount (Rs.) as on Changes in Working
Particulars December 31, Capital (Rs.)
1998 1999 Increase Decrease
Current Assets:
Stock 9,000 7,000 - 2,000
Debtors 16,500 19,500 3,000 -
Cash at Bank 4,000 9.000 5,000 -
(a) 29,500 35,500
Current Liabilities:
Creditors 9,000 5,000 4,000 -
(b) 9,000 5,000
:. Working Capital (a - b) 20,500 30,500
Changes in Working Capital 12,000 2,000
Net increase in Working Capital 10,000 10,000
30,500 39,500 12,000 12,000

Illustration: 4.15
The tinancial position of Mis AB Finn on 1st January 2000 and on 31st December 2000
was as follows:
1.1.2000 31.12.2000
Assets:
Cash in hand Rs.4.000 Rs.3.600
Funds Flow Statement. 265

Debtors and Stock 60,000 60AI)0 ~.:,;.

Land and Building1i 70,000 85,()00


!

1 Machinery 80,000 86,000


2,14,000 2,35,()OO
Liabilities: .

Current Liabilities Rs.36,000 R!'..4l,OOO
-~

Mrs. 'A' Loan 20,000


Loan From Bank 30,000 25,000
Capital of: A 70,000 70,000
B 78,000 79,000
2,14,000 2,35,000

During the year, the partners withdrew Rs.26,000 for personal use. The provision for
depreciation against machinery as on 1.1.2000 and 31.12.2000 was Rs. 27,000 and Rs.36,000.
You are required to prepare Funds Flow Statement. [Kuvempu Uni, B.Com, May 2001J
Solution:
• There is no information about either the amount of depreciation on Building or the purchase
of Land and/or Building. Hence, the increase in its value of Rs. 15,000 is considered as on
account of purchases.
- • Provision for depreciation on: December 31, 2000: Rs. 36,000
January l, 2000: 27,000
:. Depreciation on Machinery for 2000, 9,000
• Machinery on January 1,2000 Rs. 80,000
Less: Deereciation for 2000 9,000
71,000
Machinery on December 31, 2000 86,000
:. Purchases 15,000
• Total capital on January 1,2000 1,48,000
Less: Amount withdrawn during 2000 26,000
1,22,000
But, the total capital on December 31, 2000 \,49,000
:. Prottl earned during: 2000 27,000
Management Accounting : 266

• Profit earned during 2000 Rs.27,000


Add back: Depreciation on Machinery 9.000
:. Funds from Operation 36,000

Schedule of Changes in Working Capital


Changes in Working
Amount (Rs.) as on
Particulars Capital (Rs.)
1.1.2000 31.12.2000 Increa'ie I Decrease
Current Assets:
Cash in hand 4.000 3.600 - 400
Debtors and Stock 60,000 60.400 400 -
I
I
I
II (a) i 64.()()()
i
6-UJO()
I
I Current Liabilities (b) I 36,()OO
i 41.0(J0 - 5,000
I Working Capital I
I
2H,OOO 23.000 -

Changes in Working Capital 400 5,400


:.Net decrease in Working Capital· 5,000 5.000.
28.000 28,000 5.400 5.400
• Both the loan from Mr.A and from Bank are considered as non-current liability items.
Funds Flow Statement of MIs AB for the year ended December 31, 2000
Amount Amount
Sources Applications
(Rs) (Rs)
Funds from Operation 36,000 Purchase of Land andlor Building 15,000
Loan from Mr. A 20,000 Purchase of Machinery 15,000
Net decrease in Working Capital 5,000 Amount withdrawn by the Partners 26.000
Repayment of Bank Loan 5,000
61.000 61.000

Illustration: 4.16
From the following details, calculate fund from operations and show the relevant items
only in the Fund Flow Statement as on 31-12-2000. The current year's profit of X Ltd., is Rs.
80,000 after incorporating the following:
Funds Flow Statement: 267
Depreciation Rs.45,000
Goodwill written off 12,000
Loss on sale of Furniture 5,000
Profit on sale of Investments 15,000
Tax provision 35,000
Dividends paid 40,000
Preliminary expenses 5,000
Transfer to General Reserve 25,000

Other information: Machinery was sold on 1-1-2000 at its book value of Rs.40,000, Book value
of investment sold was Rs.35,OOO. Increase in working capital was "
Rs.2,77,000. . [Bangaiore Uni., B.Com., April 2004]
Solution:
Funds from Operations
Amount
Particulars
(Rs)
Profit for the year 2000 80,000
Add back: Depreciation Rs.45,000.
Goodwill written off 12,000
Loss on sale of Furniture 5,000
Tax provision 35,000
Dividends 40,000
Preliminary expenses written off 5,000
Transfer to General Reserve 25,000 1,67,000
2.47,000
Less: Profit on sale of Investments 15,000

. :. Funds from Operation 2,32,000


Management Accounting : 268

.'unds Flow Statement for the year ended December 31, 2000
I

Sources
Amount
II Applications
Amount .
(R~) (Rs)
.,
Funds from Opl!ration 2.32.000 Dividend paid 40.000
Increase in Working
Sale of Investment'" 50,000
Capital 2.77.000

ISale of Machinery I 40.000


I
:.: Book value of Investments sold Rs.35.000
Add: Profit on sale of Investments 15,000
I'" , I

:., Sales value of Investmel1ts 50.000.

From the inllowing particulars. prepare a Funds Flow Statement for the year ending 31st .. '
December j 999. j • • . ~

; ·u. /\oct prolit hef6re writlllg off goodwill Rs.21.500


b. 'Depreciation written off on fixed a<;sets 3,500
c. Goodwill to be written off from profit -5,000
d. Di vidend paid 7,000
e. Shares issued for cash 10,000
f. Purchusc of Machinery 20.000
g. IIH:rease in working capital ~,OOO

[Manga/ore Uni, B.Com, May 2000}


Solution:
.'
Net profit Rs.21,500
Add back: Depreciation 3.500

:. Funds from Operation 25.000


Funds Flow Statement of ...•. Co., for the year ended December"31, 1999
Amount Amount
Sources Applications
(R,,) (Rs)

Funds from Operation 25.000 Dividend paid 7,000

Shares issued for ca<;h 10,000 Purchase of Machinery 20,000


.....

Funds Flow Statement: 269

I Increase in Working Capital 8J)00


,I
35,000

Illustration: 4.18
A statement of retained earnings of a company is as follows:
Balance as on 1.1 .2002 Rs.3.25.600.
Add: Net profit after tax 6.-1-X,-I-XO
Tax refund 25,470
9.99.550
,Less: Loss on sale of plant 14.-\.60
Goodwill written off 95.370
Dividend paid 4,70.350 5,HO.IHO
Balance of retaint;d earnings as on 31.12.02 4.19,370
Additional information:
a. Plam with written down value Rs.54,360 was sold in 2002.
b. Net profit of the year was after the depreciation of Rs.6~U50.
c. Plant costing Rs.I.60.000 was purchased and the payment was made in equal amount of
(kh l'lllurcs.

d. Rs. -: 2.XOO debentures have been redeemed during 2002.


Preparl' a statement of sources and applicat!0n of Fund.
[Mallga/ore Uni, B.Com, May 2003J
Solution:
Net profit after lax (for 2002) Rs.6.48.4g0
Add back: Depreciation 68.250
... Funds from Operation 7,16,730
Statement of Sources and Applications of Fund of ..... Company for the
year ended December 31, 2002

Sources
' , Amount I Applications
Amount
(Rs) I (Rs)

Funds from Operation 7,16,730 Dividend paid 4,70,350


Tax refund 25.470 Redemption of Debentures 72,800
Management Accounting : 270

Sale of plant* 39,900 5,43,150


Net increase in Working
Capital (balance) 2,38,950,
7,82,100 7,82,100

* Written-down-value of plant sold Rs.54,360


Less: !toss on sale of plant 14,460
, -'

:. ,Sales value of plant 39,900


* Purchase of Rs.I ,60.000 worth Plant by issuing deb~ntures is ,not taken to the Funds
Flow Statement [as it is neither a.,source nor a use of Fund].
IlIustration:- 4.19
Prepare a Fund Statement from the following.
2001 2002
Cash, Rs.2.000 Rs.2.500
Bills Receivable 2.400 2,700
Stock 3.100 3.200
Other Current Assets 800 700
Fixed Assets 5.000 5,800
Accumulated Depreciation 2.100 2,500
Bills Payable 1,400 1.300
Equity Capital 5.000 5.300
Retained Profits 2.800 3.700

Other information:
",: ..,
a. Fixed Assets costing Rs.I.200 were purchased for cash.
b. Fixed Assets cost Rs.400. accumulated depreciation Rs.150, was sold for Rs.2oo.
c. Depreciation/or the year 2.QQ2 was Rs.550..
d. Dividend paid during 2002 was Rs.300
e. Reported profit for 2002 was Rs.l.200. [Manga/ore Uni., B.Com., May 2003J
Funds Flow Statement: 271

Solution:
Computation of Funds from Operation:
Retainedprofits on December 31. 2002 Rs.3.700
2001 2.800
:. Protit earned during 2002 and retained Rs.900
Add back: Depreciation 550
Dividends JOO
Loss on sale of fixed assets* 50
----
l)()()

.'. FU,nds from Operation I.XOO


"'Cost of fixed assets sold R'i . ..J.()()
Less: Accumulated Depreciation 150
Written-down-value of asset sold 25()
Selling Price 200

:. Loss on sale of tixed assets 50

Schedule of Changes in Working Capital


Amou11l (R.,.) I
I
Changes in Working
I
Particulars December 3 I. Capital (Rs.)
200l I 2002 Increase Decrease
I

I Current Assets: I I :

II -
CU'ih 2.000
I
2.500
i 500
Hi lis recei vable 2 ...J.00 I 2.7()0 I 300 -
Stock 3,100 '::\.200 100 -
Other Current Assets xoo 7()O 100

,aJ 1~-8-.3-0-0----1~-9-.l-0-0___l
Cun-ent Liabilities
I
lAOO ,
Bills payable 1.300
I 100
I
- , I
I
(b) lAOO 1.300
Working Capital (a - b) 6.900 7.800
Changes in Working Capital 1,000 100
Management Accounting : 272

:.Net increase in Working Cap itall 900 900


7.800 7.800 1,000 1,000
!
Funds Flow Statement of ..... Co., for the year ended December 31, 2002
Amount
. Amount
Sources Applications
(Rs) (Rs)
Funds from Operation 1.800 Purchase of Fixed Assets 1.200

Sale of Fixed Assets 200 Payment of Divined 300

Increase in Equity Capital 300 ·Net increase in Working


Capital 900

Increase in Long-term
Liabilities * 100
2.400 2.400

* One or few entries not given


Toral Capital and
Total Assets Difference
Liabilities
on 31.3.2001 Rs.13.300 11,300 2,000
31.3.2002 14,900 12.800 2,100
:. Increase during 2002 100
Illustration: 4.20
The summarised Balance Sheet of Pragathi Ltd:. as at 31-12-1999 and 2000 are given
below:
Liabilities 1999 2000 Assets 1999 2000
Share Capital 4,50,000 4.50.000 Fixed Assets 4,00,000 3,20,000
General Reserve 3,00,000 3,10,000 Investment (fixed) 50.000 60,000
Pand L Ale 56.000 68.000 Stock 2.40.000 2,10,000
Creditors 1.68.000 1.34.000 Debtors 2.10.000 4,55.,000
Provision for Tax . 75.000 10.000 Bank 1049.000 1,97.000
Mortgage Loan 2.70.000
10,49,000 12,42,000 .10,49,000 12,42,000
Funds Flow Statement: 273

Additional information:
a. Investmems costing Rs.8,000 were sold during year 2000 for Rs.8,500.
b. Provisio~ for tax made during the year was Rs.9.000.
c. During the year. part of the fixed assets costing Rs.10,000 was sold for Rs.12.000 and
the profit was included in the Profit and Loss A/c ..
d. Dividend paid during the year amounted to Rs.40.000.
You are' required to prepare a schedule of changes in working capital and a sources and
application of funds. I Banga/ore Uni, B.Com., May 2000 and November 2001J
Solution:
Computation of Funds from Operation:
Profit and Loss Alc balance on:
31.12.2000 Rs.68,000
31.12.1999 56.000
:. Pr~fit earned during 2000 and retained 12.000
Add back: Transfer to General Reserve Rs.IO,OOO
Provision for Tax 9,000
Dividend 40.000
, Depreciation 70,000 1.29,000
1,41.000
Less: Profit on sale of: Investments Rs.500
Fixed Assets 2.000 2.500
:. Fund, i'rom Operation 1.38,500

Dr Investment Ale Cr

Particulars
Amount
Particulars Amount I
(Rs) (Rs) I
i
To Balance bId 50.000 By Cash 8.500 I
I
To Profit and Loss Alc (profit on sale) d500
To Cash (balance, Purchases) 18,000 By Balance cld 60,000
,
68,500 68,500
Management Accounting : 274

Dr. Provision for Taxation Ale Cr

Amount Amount
Particulars Particulars
(Rs) (Rs)

To Cash (balance, tax paid) 74,000 By Balance bId 75,000


By Profit and Loss Alc 9,000
To Balance cld 10,000
84,000 84,000

Dr Fixed Assets Ale Cr

Amount Amount
Particulars Particulars
(Rs) (Rs)

To Balance bId I ~oo.OOO By Cash (sale) 12,000


I
To Profit and Loss Alc By Cash (balance, assumed
(Protit) " 2.000
to be depreciation) 70.000
By Balance cld 3,20,000
I

i 4,02.000 4,02,000

Schedule of Changes in Working Capital


Amount (R~.) as on Changes in Working

I Particulars December 31,


IY99 2000
Capital (Rs.)
Increase Decrease
I
! Current A~sels: ~

I
I
I
Stock 2,40.000 I 2. 1(),OOO I - 30.000
Debtor~ I 2.IO.OO() I 4.55.000
I
2,45.000 -
Bank Balance 1.49.000 I 1.97.000 4X.OOO -
(a) 5.99.000 g.62.000
Current Liabilities:
Creditors 1,68,090 1,34,000 34,000 -
(b) 1,68,000 1,34,000
Funds Flow Statement: 275

:. Working Capital (a - b) 4,31,000 7,28.000


Changes in Working Capital 3,27,000 30,000
Net increase in Working Capital 2,97,000 2,97,000
7,28,000 7,28',000 3,27,000 3,27,000

Funds Flow Statement of Pragathi Ltd., for the year ended December 31, 2000
Amount· Amount
Sources Applications
(Rs) (Rs)

Funds from Operation 1,38,500 Tax paid 74.000


Mortgage loan obtained 2,70,000 Purchase of Investments 18,000

Sale of Fixed Assets 12,000 Dividend 40,000

Sale of Investments 8.500 Net increa~e in Working Capital 2,97,000

4,29,000 4.29,000

Illustration: 4.21
The comparative balance sheets of Rayal Ltd" as on 31st December 200 I and 31 st
December 2002 are given below in condensed form:
31.12.2001 31.12.202
Liabilities:
Equity share capital Rs. I ,00,000 Rs. I ,50,000
Premium on shares 10,000
Retained earnings 70.250 85.300
Mortgage Debentures 50.000
Accounts payable 28.000 4X,000
2,48.250 2.93.300
Assets:
Fixed assets • U1,000 2,00,000
Less: Accumulated depreciation 60.000 40,000
92.000 \,60,000
Cash 2X.100 20,000
Management Accounting : 276

Accounts Receivable 30,800 21.100


Prepaid expenses 3.950 3,000
Inventory 93,400 89,200
2,48.250 2.93,300
Additional information:
a. Net income for the year R~.17.050.
b. Depreciation for the year Rs.1 0.000.
c. Interim dividend paid during the year Rs.12,OOO.
d. An addition to the building was made during the year at a cost of Rs.78.000 and fully
depreciated equipment costing Rs.30,000 was discarded, no salvage being realized.
Prepare: (a) A schedule of changes in working capital, and (b) A statement of source and
application of funds. [Bangalore Un;' B.Com., April 2003]
Solution:
Computation of Funds from Operation
Re.tai"ncd earnings on December 31. 2002 : Rs.85.300
200 I : 70,250

~\ Profit earned during 2002 and retained 15,050


Add back: Depredation Rs.IO.OOO
Interim Dividend 12.000 22.000

:. Funds from Operation 37.050

'Dr Fixed Assets Ale C'r

Particulars
Amount
. Particulars I Amount
(Rs) (Rs)

To Balam:e bId 1.51.000 I By Accumulated Depreciation


Ale (balance) 30.000
To Bank Ale (Purchase
of Building) 78.000 By Balance cld 2.00:000
2,30,000 2.30,000
Funds Flow.Statement : 277

Dr Accumulated Depreciation. \/c Cr

Amount Amount
Particulars
(Rs) i Particulars
(Rs)
To Fixed Assets Alc 30,000 By Balance bid 60.000
By Balance cld 40.000 By Profit and Loss Ale 10,000
70,000 70,000

Schedule of Changes in Working Capital


Amount (Rs.) as on Changes in Working
Particulars December 31. Capital (Rs.)
2001 2002 Increase Decrease
Current Assets :
Cash 28,100 20,000 - 8.100
Accounts Receivable 30,800 21,100 9.700
Prepaid expenses 3.950 3,000 950
Inventory 93,400 89,200 - 4,200
(a) 1,56,250 1,33,300
Current Liabilities:
Accounts Payable 28,000 48,000 - 20,000
(b) 28,000 48.000
:. Working Capital (a - b) 1,28,250 85,300
Changes in Working Capital 0 42,950
Net Decrease in Working Capital 42,950 42.950
1.28,250 1,28.250 42,950 42.950

Funds Flow statement of Rayal Ltd., for the year ended December 31, 2002
Amount Amount
SOl~rces Applications
(Rs) (Rs)

Funds frC?m Operation 37,050 Redemption of Mortgage


Debentures 50.000
Management Accounting : 278

Equity Share Capital 50.000


Share Premium 10,000 Purchase of Building 78,000

Net decrease in Working Payment of Interim


Capital 42.950 Dividend 12,000

1,40,000 1,40,000

Illustration: 4.22
Balance Sheets of Young India Ltd.,
Assets: 2000 ' 2001
Cash at Bank Rs.2,500 Rs.2,700
Sundry Debtors 87,490 73,360
Stock 1,11,040 97,370
Plant and Machinery 1,12,950 1,16,200
Land and Buildings 1,48,500 1,44,250
Good will 20,000
. 4,62,480 4,53,880
Liabilities:
Sundry Creditors 39,500 41,135
Bills payable 33,780 11,525
Bank overdraft 59,510
Provision for Taxation 40,000 50.000
Reserves 50,000 50,000
Profit and Loss Alc 39,690 41,220
Share Capital 2,00,000 2,60,000
4,62,480 4,5j,880
Additional information:
a. During the year 200 I, an interim dividend of Rs. 26,000 was paid.
b. The assets of another company were purchased for Rs.60,000 payable in fully paid
shares of the company. The assets consisted of Stock Rs.22,000, Machinery
Rs.18,OOO and Goodwill Rs.20,000.
c. Purchase of plant for cash Rs.5,600 was made during the year 2001.
Funds Flow Statement: 279

d. Tax paid during 2001 was Rs.25,000.


e. The Net profit for the year 2001 before tax was Rs.62.530.
Prepare a Fund Flow Statement and a statement showing changes in Working Capital.
[Ballga/ore Ulli., B.Com .• April 2001 and May 2002 alld
Klivempu Ulli., B.Com., October 2004J
Solution:
Profit and Loss Alc balance on December 31, 200 I: Rs.41,220
2000: 39,690
:. Profit earned during 200 I and retained 1,530
Add: Interim dividend 26,000
Depreciation on: Plant and Machinery* Rs.20,350
Land and Buildings+ 4.250 24,600 50,600
. :. Funds from Operation 52,130

Note: *Plant and Machinery on January 1,2001 Rs.l,12,950


Add: Plant purchased for cash 5,600
Purchase of Machinery of another wmpany 18,000
1.36,550
Plant and Machinery on December 31. 200 I 1.16,200
:. Depreciation for 2001 20.350

+ [Rs.l ,48.500 - Rs.l.44,250] =Rs.4.250


• Provision for Taxation is treated in this illustration as an item of current liabilities. Hence, it
does not require any adjustment while computing Fund from Operation.
• Since, the balance in the Profit and Loss Alc, both at the beginning and at the end of the year
is given, the amount of net profit of Rs.62,530 given in the problem is not relevant.
Schedule of Changes in Working Capital
Amount (Rs.) as on Changes in Working
Particulars December 3 I , Capital (Rs.)
2000 2001 Increase Decrease
Current Assets:
Cash at Bank 2.500 2.700 200 -
Management Accounting : 280

Sundry Debtors 87,490 73.360 - 14,130


Stock 1.11.040 97.370 - 13,670
(a) 2.01.030 1.73,430
Current Liabilities:
Sundry Creditors 39,500 41.135 - 1,635
Bills Payable 33.780 11.525 22,255 -
Bank Overdraft 59,510 0 59.510 -
Provision for Taxation 40,000 50,000 - 10,000
(b) 1,72.790. 1,02,660
:. Working Capital (a - b) 28,240 70,770
Changes in Working Capital 81,965 39,435
Net increase in Working Capital 42,530 42,530
70,770 70,770 81,965 81,965

- Fund Flow Statement of Young India Ltd., for the.year ended December 31, 2001
Amount Amount
Sources Applications
(Rs) (Rs)

Funds from Operation 52.130 Purchase of Plant 5,600

Issue of Shares (for Stock Interim Dividend 26,000


purchased) 22.000
Net increase in Working
Capital 42,530

74,130 74,130

Illustration: 4.23
From the following Balance Sheets of Omega Ltd., prepare a statement of changes in
working capital and Funds Flow Statement.
1982 (Rs.) 1983 (Rs.)
Capital and Liabilities
Equity Share Capital 3.00.000 4,00,000
8% Redeemable preference shares 1,50.000 I,oo,oq~

Capital reserve 0 20.000


General reserve 40.000 50.000
Funds Flow Statement: 281

Protit and Loss Account 30.000 48,000


Proposed dividends 42.000 50.000
Sundry creditors 25.000 47.000
Bills payable 20.000 16,000
Liability for expenses 30,000 36.000
Provision.for taxation 40.000 50,000
6,77,000 8,17.000
Assets
GoodwilI 1,00,000 80,000
Land and buildings 2,00,000 1,70,000
Plant and machinery 80.000 2,00,000
Investments 20,000 30,000
Sundry debtors • 1,40,000 1,70,000
Stock 77,000 1,09,000
Bills receivable 20,000 30,000
Cash in hand 15,000 10,000
Cash at bank 10,000 8,000
Preliminary expenses 15,000 10,000
6,77,000 8,17,000

1. A piece of land ha'i been sold out in 1983 and the profit on sale has been credited to
capital reserve:
2. A machine was sold for Rs. 10.000. The written down value of the machine was Rs.
12.000. Depreciation of Rs. 10.000 was charged on plant account in 1983.
3. The; investments are trade investments. Rs. 3.()()() is received by way of dividends which
included Rs. LOOO from pre-acquisition profit. It has been credited to investment
account;
4. An interim dividend of Rs. 20,000 has been paid in 1983.
(M.Com., University of Mysore, June 1984)
Management Accounting : 282

Solution:
Statement of Changes in Working Capital
Changes in Working
Particulars
At the end of ...
Capital (Rs.) I
1982 (Rs.) 1983 (Rs.) Increa<;e Decrease
Current Assets
Sundry debtors IAO,OOO 1,70,000 30.000 0
Stock 77.000 I.OY.OOO 32.000 oi
Bills receivable 20,000 30,0()() 10,000 0
Cash in hand 15,000 10,000 0 5,000
Cash at bank 10,000 8.000 0 2,000
Total Cun'ent Assets (a) 2,62;000 3.27,000
Current Liabilities
Sundry creditors 25.000 47.000 0 22,000
Bills payable 20,000 16,000 4,000 0
Liability for expenses 30~000 36,000 0 6,000
Total CUITent liabilities (b) 75.000 99,000
Working Capital (a - n) 1.87.000 2,28,000
Changes in Working Capital 76.000 35.000
Increase in Working Capital 41.000
Total 76.000 76,000

Statement of Sources and Uses of Fund of Omega Ltd., for the year ended December 31,
1983
Amount Amount
Sources Applications
Rs. Rs.
Funds from Operatioll 1.85.000 Redemption of preference shares 50,000
I
Equity share capllal 1.00,000 I Payment of (last year's Proppsedl
Sale of Land 50,000 I Dividend 42,000 I
Payment of Tax (provision of the
Sale of Machine 10,000
last year) 40,000
Dividend on investment
Purchase of plant 1,42.000
(credited to Investment Account) 1,000
Purchase of investment 11,000
Funds Flow Statement : 283

Payme nt of interim dividend 20,000


Increas e in Workirig Capital 41,000

3,46,000 3,46,000
Working Notes:,

Calculation of Funds from Operation:

Closing balance of Profit and Loss Account Rs.48,000


Less: Opening balance of Profit and Loss Account 30,000
Profit earned during the year 1983 18,000
Add: Goodwill written-off 20,000
Preliminary expenses written-off 5,000
Depreciation 10,000
Provision for taxation 50,000
Proposed final dividend 50,000
Loss on sale of plant 2,000
Transfer to general reserve ·10,000
Interim dividend 20,000 1,67,000
Therefore, Funds from Operation 1,85,000
Dr Land Ale Cr
Amount Amount
Particulars Particulars
To Balance bId
. Rs.
2,00,DOO By Cas h (sale of land, balancing figure)
Rs .
50,000
To Capital Reserve (profit on sale) 20,000 By Bal'ance cld 1,70,000
2,20,000 2,20,00q

Dr Plant Ale Cr
Amount I Amount
Partieu lars Particular"
Rs. I Rs.
To Balance bId 80,000 By Cas h tsale of plant) 10,000
To Purchase (balance) By Profit and Loss Account (loss) 2,000
1,42,000
By Protit and Loss Ale (depreciation) 10,000
By BahlI1C~ cld (closing balance) 2,00,000
2,22,000 2,22,000
Management Accounting: 284

Dr Investment Ale Cr
Amount Amount
Particulars Particulars
Rs. Rs.
To Balance bid 20,000 By Ca<;h (dividend received out of pre-
acquisition profit) 1,000
! To Purchases (balance) 11,000
! By Balance dd 30,000
! 31,000 31,000
I
Dr Provision for Taxation Ale Cr
i Amollnt Amount
Particulars Particulars
Rs. Rs.
To Bank 40,000* By Balance bid 40,000
To Balance cld 50,000 By Adjusted Protit and Loss Ale
(provision for the year) 50,000

90,000 I 90,000
* Assumed that the previolls year's tax of Rs. 40,000 has been paid during 1983.
Illustration: 4.24
Prepare a Funds Flow Statement from the following Balance Sheet of ABC Company.

1-1-1984 31-12-1984
Cash Rs.40,OOO- Rs.44,400
Accounts receivable 10,000 20,700
Inventories 15,000 15,000
Land 4.000 4,000
Buildings 20,000 16,000
Equipment 15,000 17,000
Accumulated depreciation (5,000) (2.800)
Patents 1.000 900
Total Assets 1,00,000 1,15,200
Current liabilities 30.000 32,000
Bonds payable 22,000 22,000
Bonds payable discount (2,000) (1,800)
Capital stock 35,000 43,500
-l".,
Funds Flow Statement: 285

Retained earnings 5.000 19.500


Total Equities 1.00.000 1.15,200

Additional information:
(a) Income for the period was Rs. 10.000
(b) A building that cost Rs. 4.000 and which had a book vaille of Rs. 1.000 was sold
forRs.IAOO
(c) The depreciation charge for the period was Rs. 800.
(d) There was an issue of Rs. 5,000 of common stock.
(e) Cash dividends of Rs. 2,000 and stock dividends of Rs. 3.500 were declared.
[M.Col1l., Ulliversity of Mysore, 1985J
Solution:
1. Computation of Profit on Sale of Building
Acquisition cost Rs. 4.000
Less: Depreciation to the date of sale
(i.e., Acquisition cost - Undepreciated Book Value = Rs. 4,000 - Rs. 1,000) = 3,000
:. U.ndepreciated Book Value of the building sold 1.000
Sale!> Price 1.400
:. Profit on sale of building 400
This Rs. 400 profit on sale of building has been credited to the Protit and Loss Account
and Rs. 10,000 protit for the year has been arrived at after crediting this Rs. 400 proHt. Since
the proHt is not an item of trading profit, it should be deducted from the profit figure to alTive
at Funds from Operation. Rs. 10400 sales price of the building should be considered as an item
of source of Funds in the Funds Flow Statement.
2. Calculation of Funds from Operation
Income for the period (as given in the problem) Rs.I0,000
Add: Expenses not causing the outflow of Funds:
Depreciation Rs.800
Amortization of: Patents Rs.100
Discount (bonds) 200 300 1,100
11,100
Less: Profit on the sale of building (Working Note 1) 400
:. Funds from Operation 10,700
Management Accounting : 286

, W ork'109 C apl'ta I
Sch ed u IeofCh angesm
Amount (in Rs) Changes in Working
as on ... Capital
Particulars
January December Increase Decrease
L 1984 31, 1984 (Rs) (Rs)
Current Assets:
Cash 40.000 44.400 4.400 0
Accounts receivable 10.000 20.700 10.700 0
Inventories 15.000 15.000 0 0
65,000 80,100
Less: Current Liabilities 30,000 32,000 0 2,000
Working Capital 35.000 48.100
Total Increase and Decrease in Working Capital 15,100 2.000
Increase in Working Capital 13.100
I
15.100 15.100
I

3. The value of equipment increased from Rs. 15.000 ori January I. 1984 to Rs. 17.000 by
December 31. 1984 and this increase of Rs. 2.000 is assumed to be the purchase price of the
new equipments acquired during the year. It involves the outflow of cash which i~ a part of
Fund. It. therefore, appears on the 'uses' side of Funds Flow Statement.

e ecem ber 31 , 1984


Fun ds FIow Statement 0fABCC ompany ~or th e year enddD
Amount Amount
Sources of Funds Application of Funds
Rs. Rs.
Funds from Operation (Working Purchase of equipment
Note 2) . 10.700 (Working Note 3) 2.000
I
Sale of building 1,400 Equity dividend (cash) 2,000
Issue of equity shares 5.000 Increase in Working Capital
13.100
17,100 17.100
Funds Flow Statement· 287

Illustration: 4.25

The Balance Sheet of X Ltd., as at March 31. 1988 and 1989 are given below.

31-3-88 31-3-89
Share Capital Rs. 4,00,000 Rs. 5,00,000
Capital Reserve~ 0 20.000
General Reserves I.~m.ooo 2, 10.000
Profit and Los~ Account 70,000 l)O,OOO
Debentures 3.00.000 2.00.000
Current Liabilities 1,30.000 1.20.00()
Provision for Income Tax XO.OOO 60.000
Proposed Dividend 40.000 50.0QO
---
12.00.000 12.50.000
Fixed A'i'>et:-. at cu:-.t 10.00.000 10.00.000
Less: DeprL'CiatlOn 2.60.000 3. I ()'oOO
7AO.OOO 6.90.000
Trade Investment 1.10.000 90.000
Current A:-.:-.ets 3.2(),OOO 4,50,000
Preliminary Expen'ies 30.000 20.000
12,0(),OOO 12,50,000

During the year ended 31 st March, 1989, the company:


I. sold one machine for Rs. 40.000, the cost of which was Rs. 80,000 and the depreciation
provided on it was Rs. 30,000,
2. provided Rs. 1,00.000 as depreciation,
3. redeemed the debentures at Rs. 105,
4. sold some trade investments at a profit which was credited to Capital Reserve,
5. decided to write off Fixed Assets (fully depreciated) costing Rs. 20,000. and
6. decided to value opening stock at cost which was valued previously at cost less 10%. The
opening stock according to book was Rs. 63.000. The closing :-.tock was correctly valued
at cost.
Management Accounting : 288

,- Prepare the Statement of Sources and Application of Funds for the year ended 31 st
March . .1989 showing the changes in the Working Capital. (All working notes are to form part of
your answer). ICA (Fill), November 1989J
Solution:
. Calculation of Loss on Machine Sold:

Acquisition cost of the machine sold Rs.80,000


Less: - Depreciation to date 30.000
Written down value 50,000
Selling price 40.000
Loss on sale of machine 10.000

-Dr Fixed Assets Ale Cr


Amount Amount
Particulars Parlil:ulars
Rs. Rs.
To Balance bid 7,40.000 By Cash (sale of machine) 40.000
To Cash (Purchase. balance) 1,00.000 By Profit and Loss Alc (loss) 10.000
By P & L Alc (Depreciation for the year) 1.00.000
By Balance c/d 6.90,000
8,40.000 8.40.000

Calculation of Funds from Operation:

Profit earned during the year and retained in the company (Le .• the
difference betweenJhe closing and opening balance of P&L Alc) Rs.20,000
Add: Proposed dividend Rs. 50.000
Transfer to general reserve 30.000
Premium on redemption of debentures (i.e., 5% of Rs. 1.00.0(0) 5.000
Preliminary expenses written-off 10.000
Provision for tax 60,000
Depreciation 1,00,000
Loss on sale of machine 10.000
2.65.000
2,85.000
Less: Under-valuation of opening stock [Le .. (Rs. 63.000/90%) - Rs. 63.000] 7.000
Therefore. Funds from Operation
2.78.000
Funds Flow Statement: 289

1. Instead of valuing the opening stock at co'>t of Rs. 70.000 (i.e .. 63.000/90%). it was
valued at cost less lOst. Hence. opening "tock wa~ undervalued by Rs. 7.000 which
resulted in the overstatement of protlt by Rs. 7.000 and under valuation of Current Assets
on March 31. 1988 by Rs. 7.000. Hence. the need for adjustment.
2. As far as the decision to write-off fully depreciated tixed assets costing Rs. 20.000 is
concerned. it has no effect as the amount has fully been depreciated.
3. Selling price of trade investments sold comes to Rs. 40.000 (i.e., cost price of investment
sold Rs. 20.000 + Profit on sale of investment of Rs. 20.000 which was transferred to
capital reserve).
Dr Provision for Taxation Ale Cr
Amount Amount
Particulars Particulars
Rs. Rs.
To Bank (assumed to be paid) 80.000 By Balance bId 80.000
To Balance cld 60,000 By Adjusted Profit and Loss Alc
(provision for the year) 60.000

1.40.000 1.40.000

Sehe d ueo , W or k'mg C apl'ta I


I fCh anges m
!I Amount (Rs.) a~ on I Change~
in Working
Particulars March 31 ... f
Capital (Rs.)
1988 11)89 i Increa~c Decrease
Current Assets 3.27.000* 4.50.001l ! 1.23.UOO i f
0
I I
I
I
(a) 3.27.000 4.50.000 I
Current Liabilities 1.30.000 1,20.000 10.000 0
(b) 1.30.000 1.20.000 i i
I
:. Working Capital (a - b) 1.97.000 3.30.000 !
I
Changes in Working Capital 1.33.000 I
I
0
I I
I 1.33.000
:. Net increase in Working Capital

. I I 1.33.000 1.33.000
Note: * includes Rs. 7.000 lIlcrease in the value of Current Assets due to restatement of opening
stock at its cost price .

•'unds I-low Statement of X Ltd., for the year ended March 31. 1989

A,;~unt Use~
i Amount I
S()urct:~ I, I,

~F-UI-ld-s-f-r-o-m-O-p-er-a-t1-'o-n----+--2-,7-g-,U-0-0-+-R-e-dcmplion of Debenture~
1 Rs.
!
I
1.05,000
Issue of shares I 1.00.000 I Payment uf dividend (previous year) I 40,000
Management Accounting: 290

Sale of trade investment 40.000 Acquisition of fixed assets 1.00,000


Sale of tixed assets 40.000 Payment of tax 80.000 I
Increase in Working Capital 1.33,000
4,58,000 4,58.000

Illustration: 4.26
Fair Deals Ltd .. presents the Balance Sheet as at 31-12-1983 and 31-12-1984
31.12.19~D 31.12.1984
Share Capital: Shares of Rs. 100 each Rs. 20,00.000 Rs. 25.00.000
Reserves and Surplus 8,00,000 8,70.000
13.5% debentures (convertible) 10.00,000 8,00,000
Public deposits 3,00,000 2.50.000
Current Iiabilitie!'> and provision!'> 6.20.000 7.10,000
Proposed dividend 2.00.0()O 2.50.000
..J.9.20.000 53.80.000
As!'>cts: Fixed Assets: Cost 25.00.000 30.00.000
Depreciation till date nJm.ooo 8.20.000
I ~{'20.000 21.80.000
Trade In\'estments 12.50.()OO 13.50.000
Current Assets: Marketable investment!'> 60.000 30,000
Inventories 4.10.000 5.20.000
Book debts 5.30.000 5,05.000
Cash and Bank balance 1,20.000 1.40.000
Preliminary expenses 1.00.000 50.000
Capital work-in-progre!'>s . 6.30.000 6.05.000
..J.l).20.000 53.80.000

You are infonned that during 1984:


I. Rs. 2.00.000 of debentures were converted into shares at par.
2. Rs. l.On.OOo shares were issued to the shareholders a" bonus shares fully paid. out of reserves.
3. Rs. 1.00.000 shares were issued to a vendor of tixed assets who had supplied a machine
costing Rs. 1.20.000.
Funds Flow Statement: 291

4. A machine costing Rs. 50.000, book value Rs. 30,000 a<; on 31 st December. 1983 was
disposed of for Rs. 20,000.
You are required to prepare the Funds Flow Statement of the ~ompany for 1984
indicating therein the Working Capital in the beginning and at the end of the year.
ICS (Fin), June 1985J
Solution:
1. Conversion of debentures (Rs. 2.00,000) into share~ does not cause the tlow of Funds and
therefore, not taken to the Funds Statement.
2. Issue of bonus shares (Rs. 2,00,000) out of Reserves has no effect on the tlow of Funds
and therefore, does not figure in the Funds Statement.
3. Acquisition of tixed assets (Rs.1.20.000) by issuing (Rs.I.(JO.O()()) "hares at Rs 20.000
premium may be considered a<; causing both int10w (Rs.I,20.000 due to issue) and outtlow
(Rs. 1.20,000 due to acquisition) of Funds and therefore: appear~ in the Funds Statement.
4. Loss on sale of machine:
Acquisition Cost Rs.50.000
Less: Depreciation to date 20,000
W ri tten-down-val ue 30,000
Selling price 20,000
Loss 10,000

5. Calculation of Depreciation:
Depreciation up to 31-12-83 Rs. 6,80.000
Less: Depreciation on machine sold 20,000
6.60,000
Depreciation up to 31-12-84 8,20.000
:. Depreciation for 1984 1.60.000

Dr Fixed Assets Ale Cr


Amount i ! Amount
Particular" I Particular...
Rs. Rs.
To Balance bid (including capital By lkprcciation Ale:, (accum) i 20.00n
WIP) By Cash (selling price) 20.000
31.30,000
I
To Purchases: By Profit and Loss Ale:, (iOS~) 10.000
Shares 1.20.000 By Balance'cld (including closing WIP) 36,05.000
Cash (balance) 4,05,000 5,25,000
36.55.000 36.55.000
Management Accounting: 292

Computation of Funds from Operation


Net Profit earned during the year (1984) and retained in the company
[Le., difference between opening (Rs. 8,00,000 - Rs. 2,00,000
utilized for bonus issue = Rs. 6,00,000) and closing balance (Rs.
H,70,(00) of Reserves and Surplus) Rs.2,70,000
Add: Proposed dividend for 1')84 Rs.2,50,000
Depreciation for 1984 1,60,000

Preliminary expenses written-off 50,000


Loss on sale of machine 10,000 4,70,000
7,40,000

Less:Premium on issue of shares (a<;sumed to have been taken to P & L Nc) 20,000

.'. Funds from Operation 7.20,000


, W or k'109 C apl'ta I
Sch ed uIeofCh anges 10
Amount as on Changes in Working
December 31, ... Capital
Particulars
1983 1984 Increase Decrease

CUITent Assets:
Rs. I
Rs. Rs. Rs.
I
Marketahle Securities flO 000 i 30000 I Oi 30000
Inventories -J..IO.OOO I 5,20,00U I
1.1 (WOO
I
I
Books Debt~ I 5 30000 ' 505000 . 0 25,000 I
Cash and Bank balance I 1,20,000 1,40,000 20.000
01
! 11.20,000 11,95.000
Less: Cun'ent Liabilities 6,20.000 7.10,000 0 90,000
:. Working Capital 5,00,000 4,85,000
Total changes in Working Capital 1,30,000 ].45,000
I
Net decrease in Working Capital I I
I 15,000
1,45,000 I I
I
I
. 1.45,000 I

Funds Flow Statement


Amount Amount
Sources Uses
Rs. Rs.
Funds from Operation 7,20,000 Repayment of public deposits 50,000
Opening balance of Working Capital 5.00,000 , Payment of dividend (llJX3) I
2,00,000
Is<;uc of ~hare~ (for Fixed Asset~) 1.20.000 i Acqui~ition of Iixl.'d a~seh 5.25,000 I
Funds Flow Statement: 293

Sale of machine -20,OOl)jAC~!t~i-~iw;l~l;r trade InVeSLments I.OO,ODU


I Closing balance of Working Capital 4,85,000
I
13,60.000 I 13,60,000

Illustration: 4.27
From the following Balance Sheet of ABC Ltd., pn:pare Funds Flow Statement:

(Rs.OOO's) (Rs. OOO's)


Liabilities Assets
1985 1986 1985 1986
Equity share capital 300 400 Goodwill 115 90
Preference shares 150 100 Land and Building 200 170
General reserve 40 70 Plant and Machinery 80 200
Profit and Loss Alc 30 48 Debtors 160 200
Proposed dividend 42 50 Stock 77 109
Creditors 55 83 Bills receivable 20 30
Bills payable 20 16 I Cash in hand 15 10
Provision for taxation 40 50 I Cash at bank 10 8
677 817 677 817

Additional information:
(a) Dividend provi'sion made during 1985 has been paid during 1986.
(b) Depreciation: Rs. tO,OOO on plant and machinery;
Rs. 20,000 on land and buildings.
(c) An interim di vidend of Rs. 20,000 has been paid in 1986.
(d) Income-tax Rs. 35,000 has been paid during 1986. res (Fin), June 1987J
Solution:
Dr Ad·
(JUS t ed PTO fit
I an d L oss A ccount Cr
Amount Amount
Particulars Particulars
Rs. Rs.
To Goodwill written-off 25,000 By Balance bid 30,000
To Depreciation 30,000 By Funds from Operation (balance) 2.18.000
To Provision for taxation (2) 45.000
To Proposed dividend (1986) 50.000
To Transfer to general reserve 30,000
Management Accounting : 294

To Interim,dividend 20,000
To Balance cld (to Balance Sheet) 48,000
2,48,000 2,48,000

Dr Provision for TaxatiQn Ale Cr


Amount Amount
Particulars Particulars
Rs. Rs.
To Bank Alc 35,000, By Balance bId (opening balance) 40,000
By Balance c/d (closing balance) 50,000 By ProtIt & Loss Adjustment Alc
(Provision for the year) 45,000
85,000 85,000

Dr L an d an dB UI'Id'10KAIe Cr
Amount Amount
PaI1iculars Particulars
Rs. Rs.
To Balance bId 2.00.000 By P & LAIc (Depreciation) 20,000
By Bank (Sates, balance) 10,000
By Balance cld 1,70,000
2,00,000 2.00,000

Dr PI ant an d M aeh'mery AIe Cr


Amount Amount
Particulars Particulars
Rs. Rs.
To Balance bId 80,000 By Protit and Loss Ale (Dep) 10,000
To Bank (balance, purchase) 1,30,000 By Balance cld 2,00,000
2,10,000 2,10,000

Se h ed ueo . W orking C apia


I fCh angesm "t I
Amount (Rs.) as on Changes in Working
Particulars December 31 ... Cagital (Rs)
1985 1986 Increase Decrease
Current Ass'ets
Debtors 1,60,000 2,00,000 40,000 0
Stock 77.000 1,09,000 32.000 aI
Bills Receivable 20,000 ' 30.000 10.000 0
Cash in hand 15.000 10,000 0 5,000
Funds Flow Statement: 295

Cash at bank 10.000 g.OOO 0 2.000


I
(a) 2,82,000 3,57,000

Current Liabilities
Creditors 55.000 83.000 0 28.000
Bills payable 20.0()O I 16.000 4.000 0
(b) 75.000 99.000
:. Working Capital (a - b) 2.07.000 I 2.Sx'00O
I 86.000 35,000
Total Increase and Decrease in Working Capital I I
I
Net increase in Working Capital I
I I
51,000
86,000 86,000
I !
Funds Flow Statement of ABC Ltd., for the year ended December 31, 1986
Amount
Panicular~
Rs.
Sources of Funds:
Funds from Operation 2,18,000
Sale of land and building 10.000 I
I

Issue of equity shares 1.00,000 I


3,28,000
Application of Funds:
Purchase of plant and machinery 1,30,000
Redemption of preference shares 50,000
Payment of dividend, Final for 1985 42,000
Interim for 1986 20.000
Payment of tax 35,000
Increase in Working Capital 51,000
3,28,000
Management Accounting : 296

Illustration: 4.28
From the following details relating to the accounts of Husmundi and Co .. Ltd .• prepare
Statement of Sources and Application of Funds.

31.12.86 31.12.85 31.12.86 31.12.85


Liabilities Assets
Rs. Rs. Rs. Rs.
Share Capital 4.00.000 3,00,000 Goodwill 90,000 1.00,000
Reserves 1.00,000 80,000 Plant and Machinery 4.29,250 2,98,000
Profit and Loss Account 50,000 30,000 Debenture Discount 5,000 8,000
Debenture 1,00,000 1,50,000 Prepaid Expenses 5,750 4.000
Income Tax Provision 40,000 50,000 Investments 60,000 1,00.000
Trade Creditors 70,000 90,000 Sundry Debtors 1,10,000 1,60,000
Proposed Dividend 40,000 30,000 Stock 80,000 50,000
Cash and Bank 20.000 10,000
8,00,000 7,30,000 8,00,000 7,30,000

l. 15% depreciation has been charged in the accounts on plant and machinery.
2. Old machines costing Rs. 50,000 (WDV Rs. 20,000) have been sold for Rs. 35,000.
3. A machine costing Rs. 10,000 (WDV Rs. 3,000) has been discharged.
4. Rs. 10,000 profit has been earned by sale of investments.
5. Debentures have been redeemed at 5% premium.
6. Rs. 45,000 income tax has been paid and adjusted against Income Tax Provision Account.
lCA (Fin), November 1987}
Solution:
Dr Provision for Taxation Ale Cr
Amount Amount
Particulars Particulars
Rs. Rs.
To Bank Ale 45,000 By Balance bid (opening balance) 50,000
To Balance cld 40.000 By Adjusted P&L Ale (provision for t~e year) 35,000
85,000 85.000

Dr PIant an dMaeh'mery AIe Cr


Amount Amount
Particulars Particulars
Rs. Rs.
To Balance bid 2.98,000 By Bank (sale of machine) 35,000
To P&L Ale (profit on sale of By P&L Ale (WDV of machine
Funds Flow Statement: 297

machine) l5.000 discharged) 3,000


To Bank (balance. Purchase) 2,30,000 By P&L Alc (depreciation)* 75.750
By Balance cld 4,29.250
5,43,000 5,43,000
Note: *15% of (Rs. 2,98.000 - Rs. 20,000 - Rs. 3.000) = 15% of Rs. 2,75,000 = Rs.41,250
+15% on cost of new 'acquisition made = IYk of Rs. 2,30.000 = Rs.34,500
Rs.75,750

Closing balance of Plant Rs.4.29.150


Less: WDV of existing machine
(2,75,000 - 41 ,250) 2,33,750
WDV of new machine 1,95,500 This represents 85% of cost of new
machines acquired.

Therefore, Co,t of New MacHines =~ ,~;~o~ =Rs. 2,30,000


1. SeIling price of investments = Rs. 50,000
2. Premium on redemption of debenture = Rs. 2,500 and Total cash outflow = Rs. 52,500.
Calculation of Funds from Operation:
Profit earned during the year and retained in the company
(difference between closing and opening balances of P&L Alc) Rs.20,OOO
Add: Transfer to Reserve Rs.20,OOO
Premium on redemption of debentures 2,500
Provision for tax for 1986 35.000
Proposed dividend for 1986 40.000
Goodwill written-off 10,000
Depreciatio~ 75.750
Written-down-value of machine discarded 3.000
Debenture discount written-off 3,000 1,89,250
2.09.250
Less: Profit on sale of machine 15,000
Profit on sale of investment 10.000 25.000
:. Funds from Operation 1.84.250
Management Accounting : 298
. W or k'mg C apl'ta I
I fCh anges m
Sched ueo
Amount (R~.) a~ on Changes in Working
Particular.. I December 31, ... Capital (Rs)
19H5 1986 Increa<;e Decrease
Current Assets
Prepaid expenses 4.000 5,7S0 1,750 01
I
Sundry debtors 1.60.000 1.10.000 I 0 50,OO~ I
Stock 50,000 80,000 30,000 0
Cash and Bank 10,000 20,000 10,000 0
(a) 2.24.000 2.1S.750
Current Liabilities: Trade Creditors 90,000 70,000 20.000 0
(b) 90.000 70,000
:. Working Capital (a - b) 1.34,000 1.45.750
i
I
Total Increase and Decrease in Workll1g Capital 61.750 SO,OOO
I
Net increase in Working Capital 11.750
61.750 61.750
I
e ecem ber 31, 1986
F un d s FIow Stat ement 0 fH usmun d'I & C0., Ltd ., ~or the year en ddD
Amount Amount
Sources of Funds Uses of Funds
Rs. Rs.
Funds from Operation 1,84,250 Redemption of Debentures 52.500
Issue of Shares 1,00,000 I Payment of Tax 45,000
Sale of Investments . 50.000 Payment of Dividend for 1985 30,000
Sale of Machines 35,000 Purchase of ~Iant and Machinery 2,30,000
Increase in Working Capital 11,750
3,69,250 3,69,250

Illustration: 4.29
The comparative Balance Sheets of Bharat Ltd .. are indicated in a condensed form a<; under.

31-3-11)1)2 31,.3-1991
Fixed Assets Rs. 5.20.000 Rs. 4,80,000
• Less: Depreciation to date 1.40,000 1,08,000
3,80,000 3,72,000
Funds Flow Statement: 299

Investment at cost 50,000 1,00,000


Stocks 90,500 55,600
Sundry Debtors 1,67,800 1,18,300
Cash and Bank Balance 47,500 49,800
Preliminary Expenses 7.200
7.35,800 7.02.900
Share Capital: Equity Shares of Rs. IOU l:i.u.:h l~~ued fur ~a~h 4.00,000 3,60,000
General Reserve 60,000 1.10,000
Surplus in Profit and Loss Account 53,450 20,450
Sundry Creditors 1,75,350 1,83,650
Proposed Dividend 15,000 28,800
Provision for Taxation 32.000
7.35,800 7,02,900

The net profit for the year 1991-92 (after providing for depreciation Rs. 40,000: writing
off preliminary expenses Rs. 7.200 and making provision for taxation Rs. 32,000) amounted to
Rs. 58,000. The company sold during the year 1991-92 old machinery costing Rs. 9,000 for Rs.
3,000. The accumulated depreciation on the said machinery was Rs. 8,000. A portion of the
company investments became worthless and was written off to General Reserve during 1991-92.
The cost of such investment was Rs. 50,000. During the year 1991-92, the company paid interim
dividend of Rs. IO,OQO and the Directurs have recommended a final dividend of Rs. 15,000 for the
year 1991-92. Prepare ( I ) a statement uf sources and application of funds for the year ended 31 st
March 1992, and (2) a schedule ufwurkll1g capilul changes. leA (Fill), May 1992J

Solution:

1. Acquisition cost of the machine sold Rs.9.000


Less: Depreciation (accumulated) 8,000
Written-down value of machine sold 1,000
Selling price 3,000
:. Profit on sale of machine 2.000
2. Funds from Operation:
Net Profit for the year 1991-92 Rs.58,000
Add: Depreciation Rs. 40,000
Preliminary expenses written off 7.200
Management Accounting : 300

Provision for taxation 32,000 79.200


1,37,200
Less: Profit on sale of machine 2,000
:. Funds from Operation 1,35,200

3. ·Fixed assets at cost on 31-3-1991 Rs.4,80,000


Less: Cost price of machine sold 9,000
Cost price of remaining assets 4,71,000
Cost price of assets on 31-3-1992 5,20,000
:. Acquisitions made during the year 49,000

Schedueo
I f Ch anees in Workine Capital
Amount (Rs.) as on Changes in Working
Particulars March 31, ... Capital (Rs)
1991 1992 Increase Decrease
Current Assets:
Stocks 55,600 90,500 34,900 0
- ---.
Sundry Debtors I, 18,300 1,67,800 49.500 0
Cash and Bank Balances 49,800 47,500 0 2,300
(a) 2.23,700 3,05,800
Current Liabilities:
Sundry Creditors 1,83,650 1,75,350 8,300 0
(b) 1,83,650 ' 1,75,350
:. Working Capital (a - b) 40,050 1,30,450
Total changes in Working Capital 92,700 2,300
Net increase in Working Capital 90,400
! 92,700 92,700
.
I
i I
Funds Flow Statement
Amount Amount
Source~ Uses
Rs. Rs.
Funds from Operation 1,35,200 Purchase of Fixed Assets 49,000
Sale of machine 3,000 Payment of dividends:
Funds Flow Statement: 301

I Issue of equity shares 40,000 I Interim (1991-92) 10,000 i


I ... ROO i!
,
1 -
F111<11(199091) "R
I i InLTea~t' in Working Capital 90.400
1.7R.200 1.78.200

Illustration: 4.30
Given below i~ the Balance Sheet of Excellent Ltd.
I
An at 31 st March A~ at 31 ~t March
Particu lars 1990 I 1991
R~. i Rs. I
Rs. Rs.
Fixed Assets at cost 62.000 I 70.000
Additions during year ~WOO 17.000

I 70.00(J I
I
'd7.000 I
I
I Depreciation 25.000 45.000 ! 36.000 51.000 i
I I 1

I Current Assets: I I
I

I
Investment 10.000 i I
I
15.000
I
i
Stock at cost I.X 1.500 II I 1.90,000 I
II i
I Trade Debtors I
1.31,500 Ii 1,38,700
!
I
: '.23.000 I 3.43.700
I
I Less:
I
Cun'ent Liabilitle-;: !
I
I
:
! BanJ... Overdraft I
I .16.0()() i
I
55.000
, 1

Trade Creditor~ and ProvIsions ~,g()() ! 1.19,200


Proposed DiVidend 16.()()O 24,000
2,31,ROO 91.200 1,98,200 1.45,500
1,36,200 1,96.500
Reprcscllled by:
Ordinary Share Capital II 75,000 1.00.000 I
General Reserve 26,000 38.000
Profit and Loss Account 35,200 48.500
Wif Debentures - 10,000
1,36,200 1,96,500
Management Accounting : 302

You are required to prepare Funds Flow Statement for the year ended 31 st March 1991.
Also highlight areas of major achievement of the management and mention the areas management
should re-examine from the view point of the financing pattern. .
[ICWA (Fin), December 1991J
Solution:

Funds from Operation:


Profit earned and retained (Rs. 48,500 - Rs. 35.'100) Rs . 13.300
. :Add: Transfer to General Reserve (R~. 38.000 - Rs.26.000) 12.000
Proposed Dividend for 1990-91 24.000
Depreciation (Rs. 36.000 - Rs. 25.000) 11.000
60,300

I fCh anges 10 W ork'109 C apia


Sch ed ueo 't I
Amount (Rs.) as on Changes in Working
Particulars March 31. ... Capital (Rs)
1990 1991 Increase Decrease
Current Assets:
Stocks 1.81.500 1.90.000 8.500 0
Trade Debtors 1.31.500 1.38.700 7.200 0
(a) 3.13.000 3.28.700
Current Liabilities:
Bank Overdraft 1.16.000 55.000 61.000 0
Trade Creditors 99.800 1.19.200 0 19,400
(b) 2.15,800 1.74.200 i
Working Capital 97.200 1.54.500
Total changes in Working Capital 76.700, .. 19,400
:.Net increase in Working Capital 57.390
76.700 76.700

Funds Flow Statement


Amount Amount
Sources Uses
Rs. Rs.
Funds from Operation .
60.300 Purchase of fixed assets 17.000
Issue ofordinary shares 25.000 Investments made 5,000
Funds Flow Statement. 303

Issue of gOk debentures I IO.O()() iI P,lyment of dividend (198l)-90) 16.000


;
I Increa~e in Working Capital .1 57.300
l
95.300 95.300
I
From the above. it is obvious that the company succeeded in paying hack a major portion
of its bank overdraft and improved its liquidity position. Return of bank overdraft helped the
company to lower Its interest burden. Further. the company 'proposed dividend for 1990-91 at
higher rate of 24~ when compared to 21.3Y'1c for the previous year 1989-90. This b more
important. because. the increased rate of dividend i~ un the increased equity share capital of R~.
1.00.000. It also transferred a sizeable amount of Rs. 12.000 of profit to General Reserve
Account and increased the credit halance in Profit and Loss Account by Rs. 13.30n. These are
some of the areas of major achievement.
However. an analysis of the capital structure of the company reveals that the deht
component forms a lower percentage. Hence. the company is not in a position tll <1\ ail of the tax
benefits on intere"t on debt capital. This assumes importance in the light or
the ... i.l.eable amount
of profit earned by the company. Hence. the company should reconsider or re-examine its capital
structure policy to mobilize additional loan capital instead of additional equity capital.
Illustration: 4.31
From the folluwin,g information. prepare a Funds Flow Statement I'or Excellent Ltd .. for
the year ended 31.3.1992.
c omparatlve Baance
I Sh eets 0 fE xce II ent Ld
t .,
As at 31 st March
II Particulars
.. I
1992 {Rs.) 1991 (Rs.)
I Liabilities and Capital: II
I
I Share Capital 50.00.000 I 40,00.000
Reserves and Surplus 15.00,000 5.00.000
Secured Loans 35.00.000 40,00,000
Current Liabilities 50,00.000 60,00,000
I 1.50,00.000 1,45,00.000
Assets:
Fixed Assets 31.00,000 30,00,000
Investments 1.50,000 -
Cash and Bank Balance 2.50.000 1,25,000
Stocks. Stores. Work-in-progress 75,00,000 78,75,000
Sundry Debtors 40,00,000 35,00,000
1.50,00,000 1,45,00,000
Management Accounting : 304

(a) The net profit for the year after adjustment in respect of provision for dividends and'
taxation was Rs. 10,00,000.

(b) There was addition to fixed assets during the year amounting to Rs. 4.00.000 and
depreciation for the year was Rs. 3,00,000. [ICWA (FiIlJ. December 1992J

Solution:
Funds from Operation =
Net profit. Rs. 10,00,000 + Depreciation, Rs. 3,00.000 = Rs. 13,00,000

. Work'ID2 Caplta
Sch ed u IeofCh an ~es In . I,
Amount (Rs.) as on Changes in Working
Particulars March 31. ... . Capital (Rs)
1991 1992 Increase Decrease
r
Current Assets: I
I
Stocks. Stores. WIP 7!U5.000 75.00.000 0 3,75,000
Sundry Debtors 35,00,000 I 40,00.000 5.00.000 I 0
Cash and Bank Balance 1,25.000 2,50,000 1.25,000 0
1,15,00,000 1,17.50.000
Current Liabilities: 60,00,000 50.00.000 10.00,000 0
:. Working Capital 55,00,000 67,50,000
Total changes in Working Capital 16.25,000 3.75,000
.'. Net increase in Working Capital 12,50,000
16,25,000 16,25,000

Funds Flow Statement


Amount Amount
Sources Uses
Rs. Rs.
Funds from Ope~ation 13.00.000 Purchase of Fixed Assets 4,00,000
Issue of shares 10,00.000 Repayment of Loan 5.00,000
Investments made 1,50,000
Net increase in Working Capital 12,50,000
23,00,000 23,00.000
Funds Flow Statement: 305

Illustration: 4.32

The following is the summarized Balance Sheet of Lotus and Company Ltd., at 1st
January 1986.

Capital and Liabilities Rs. Rs. Assets and Properties Rs.


Share Capital: Freehold Property at cost 77,000
Ordinary Shares 80,000 Plant and Machinery
(Less: Depreciation) 47,000
Redeemable 10%
Preference Shares 40,000 1.20.000
Furniture and Fittings
(Less: Depreciation) 2,000
Share Premium 8.000
Investment (Market value
Reserves and Surplus:
Rs.32,000) 25.000
Revenue Reserve 35.000
Stock 20,000
Profit & Loss Alc 12.000 47.000
Debtors 45.000
Bank Overdraft 3,000 I

Creditors 38.000 I I
I
2,16.000 2.16,000 I
On 31st December, 1986, the statement shown below was prepared where stock and debtors
amounted to Rs. 18.000 and Rs. 43.000 respectively and creditors amounted to Rs. 36.000.
Sources and Application of Working Capital
Revenue Receipts: Rs. Rs. Rs. I
Net Trading Profit for year
Add: Depreciation:
i
Plant and Machinery IO.OO() i I i
I
Furniture and Fittings 600 I 10.600 I 29.600 I
Capital Receipts:
Sale of investments 32.000 I I I
I I
Sale of Furniture and Fittings 300 I 32.3()O I
1--1 !
i
61.l)OO I
I

Capital payment: I
Redemption of Preference shares 44.000
Management Accounting : 306

Plant and Machinery 3,000 47,000


Dividend on ordinary shares' 8,000 55,000
Increac;;e in Working Capital 6,900
You are required to prepare the Balance Sheet at the end of the year, m similar form to
that shown above. Ignore taxation.
Solution:
I. There was no change in the Freehold Property during 19H6 ami it.., amuunt remained same at
Rs.77,OOO.

2. Plant and machinery on January I, 1986: Rs. 47,000


Add: Additions made during 1986 3,000
50,000
Less: Depreciation for 1986 10,000
:. Balance on December 31, 19X6 40,000

3. Furniture and Fittings on January I. 1986 Rs. 2.000


Less: Sale (assumed to be at book value) 300
1.700
Le..,s: Depreciation for 19X6 600
:. Balam:e on December31. IlJX6 I. 100
4. Investments (book value) stood at R..,. 25.000 on January I. Il)H6. These investments were
sold during IlJX6 for Rs. 32.()()(). I-knce. till..' ~ompany earned a profit of Rs. 7,000. And
therefore. the balance on December 31. IlJX6 = O.
5. ~here wa., no change in the equity share capital. However. tht.' preference shares were
redeemed at premium (Rs. 4,000 i.e .. Rs. 44.,900 - Rs. 40.000).

6. Profit and Loss Account balance on January I. IlJX6 R.,. 12,000


Add: Profit on sale of in\"estmems 7.000
Net Trading profit for 19X6 19,000
38.000
Les..,: Equity Dividend 8.000
Premium on Redemption of Preference Shares 4,000 12,000
:. Balance on December 31. 19X6 26,0(l()
Funds Flow Statement: 307

7. Computation of Cash and Bank Balance:


Working Capital items as on January 1, 1986:
Stock Rs.20,000
Debtors 45,000 Rs. 65,000
Less: Bank Overdraft 3,000
Creditors 38,000 41,000
:. Working Capital as on January I, 1986 24,000
Add: Increase in Working Capital 6,900

:. Working Capital as on December 31. 1986 30,900

It IS known that the Working Capital = [Cllm~nt Assets - Current Liabilities]. Therefore.
on December 31. 1986:

Working caPital] =[ Current - lrCurn:nt LiabilitIes J


[ Rs.30.900 Assets J Rs.36.UOU

Hence. CUlTent Assets = Rs. 30,<)00 + Rs. 36,000 = Rs. 66,900


Less: Stock Rs. 18.000
Dehtors 43.000 61,000

:.Cash and Bank Balance 5.l)O()


Balance Sheet of Lotus & Co., Ltd., as on December 31,1986
Amount Am01l11i
Liabilities Assets
Rs. Rs.
Proprietors Fund: Fixed Asse(s (at cost less I
1
deprecIation): II
Equity share capital 80,000
Share Premium 8,00U
Freehold Propaty (at cost) 77.000 I
Plam and Machinery 40.000 I
Revenue Reserve 35.000
I
rllrnltlln: und Fittings 1.\ no I
Profit and Loss Account ~6.000 I
Cllrrt'11l Asset'.: i
I Current Liabilities: i
Stock IS.OOO \
Creditors 36.0pO
Debtor... 43.000
I Cash and Bank BalallCl: 5.l)()0
j I
\.85.000 I I 1,85.000
Management Accounting: 308

Illustration: 4.33
The summarized Balance Sheet of FF Ltd., as on 31 st March 1990 and 31 st March 1991
were as follows:

Sources of Funds: 1990 (Rs.) 1991 (Rs.)


Share Capital 6,00,000 8,00,000
General Reserve 3.40,000 4.20,000
Profit and Loss Account 20,000
I
50,000 I
I
15% Debenture - A Series 4,00,000 I
- i
14lft- Debenture - B Series 5,00.000
Total 13,60,000 I 17,70,000
Application of Funds: II
Fixed Asset'i at cost 16,00,000 I 19.00,000
Less: Deprcciat ion to date 7.60.000
II lum,OOO
i Net Fixed Assets (a) i XAO,OOO ,I 10,20,000 i
I Investl11ent~ (b) I
I
I
- I 3,00.000
:
Sundry Debtors 2,00,000 2,00,000
Less: Pro\'ision for doubtful debts 20,000 50,000
I,SO,OOO I 1,50,000
I
I StockS 2.20.000 I 1.60.000 I
I Cash .lIld Bank Balances 1,80,000 2.20,000 I
I Other Cur, AS'iets
I
LIlt 72,000 96,000 !
I
Current Assets i 6.51.000 I 6,26.000 I
Less: Current Liabilities 1,32,000 1,76,000
Nl'\ Curn.'11\ :~w.,ets (c) 5.20.000 4.50,000
~--------~------~
Total (a) + (b) + (c) !! 13.60,000 17.70.000 I
! I

While going throu~h the aCl'llUnh. the following arc notIced:


(I) Fixed a~set~ of original cost of R~. 75.000 with book value of Rs. 10.000 were scrapped
and 'iold I'm Rs. 5,000.
(2) Included in current liahilities arc proposed dividend figures: 31-3-1990: Rs. 60,000;
31-3-1991: Rs. 80.000.
Funds Flow Statement: 309

(3) During the year. interim dividend for Rs. 45.000 was paid besides the outstanding as on
31-3-1990.
From the above, you are required to prepare a statement of funds tlow during 1990-91.
Solution:

Origmal cost of Fixed Assets sold Rs.75.000


Less: Accumulated depreciation on these assets 65.000
:. Book Value of Assets sold 10.000
Selling price 5,000

:. Loss on sale of fixed assets 5.000

Dr Depreciation ProvIsion Ale Cr


Amount Amount
Particulars Particulars
Rs. Rs.
To Fixed Assets Alc (accumulated By Balance bid 7.60,000
, depreciation on assets sold) 65.000
By Profit and Loss Account
To Balance cld 8.80.000 (Depreciation for 1990-91) \.85.000
9.45,000 9.45,000

Dr Fixed Assets Ale Cr


Amount Amount
Particulars Particulars
Rs. Rs.
To Balance bId 16.00,000 By Bank (Sale) 5,000
To Bank (balance, Purchases) 3.75,000 By Profit and Loss Alc (Loss on
tixed assets sold) 5,000
By Depreciation Provision Alc 65,000
By Balance cld 19,00,000
19.75.000 19,75,000

Funds from Operation:


, Profit earned during 1990-91 and retained (Rs. 50.000 - Rs.20.000) Rs.30,000
Add: Depreciation for 1990-91 1,85,000
Loss on Sale of fixed assets 5,000
Dividends - Interim 45,000
Proposed for 1990-91 80,000
Provision for doubtful debts (Rs. 50,000 - Rs. 20,000) 30,000
Management Accounting; 310

Profit transferred to General Reserve Alc 80,000


:. Funds from Operation 4,55.000

Sched u IeofCh an2es 10 Work'102 Capi'tal


Amount (Rs.) a'i on Changes in Working
Particulars March 31, ... Capital (Rs)
1990 1991 Increase Decrease
Current Assets:
Stocks 2,20,000 1,60,000 0 60,000
Cash and Bank Balance 1,80,000 2,20,000 40,000 0
Other Current Assets 72,000 96,000 24,000 0
Sundry Debtors 2,00,000 2,00,000 0 0
6.72.000 6,76.000
Less: Current Liabilities (excluding
proposed di vidends of Rs. 60,000 and Rs.
80,000) 72,000 96,000 0 24.000
:. Working Capital 6,00.000 5.80,000
Total changes in Working Capital 64,000 84,000

:.Net decrease in Working Capital 20.000


. 84,000 84,000

Funds Flow Statement


Amount Amount
Sources Uses
Rs. Rs.
Funds from Operation . 4.55,000 Purchase of Fixed Assets 3,75,000
Sale of Fixed Assets 5,000 Redemption of 15% Debentures
(A series) 4,00,000
Issue of Equity Shares 2,00,000 ,
Investments made 3.00.000
Issue of 14% Debentures (B series) 5.00;000
Payment of Interim Dividend 45,000
Decrease in Working Capital 20,000
Payment of 1989-90 Dividend 60,000
11.80.000 11.80,000

Illustration: 4,34
From the following Balance Sheet of X ltd .. as on 31st March. 1990 and other information
as set forth below, prepare projected Funds Flow Statement for the year ended 31 st March. 1991.
Funds Flow Statement: 311

Balance Sheet as on 31st March, 1990


Liabilities Rs. Assets Rs.
Equity Shares of Rs. 100 each ! Fixed Assets' 1
I I
fully paid 5,UU,00U I AI ~:o:-'l 6,00,000
10lk Redeemable preference
Le'i~: Depreciation 2,40,000
shares of Rs. 100 each I.OO,O()()
3,60,000
Reserve and Surplus:
Current Assets: i
Share Premium ),00,000
Stock 4,60,000
Profit and Loss Account 1,60,000
Debtors 4,80,000
Current Liabilities:
Bank 20,000
Sundry Creditors 3,60,000
Provision for Income Tax 1,00,000
13,20,000
13,20,000

Sales and expenses for the year ended 31st March, 1991 are budgeted for Rs. 40,00,000
and Rs., 4,00,000 respectively. G.P. percentage and stock velocity are estimated at 20% and 8
times respectively. Debtors and Creditors velocity are estimated at 73 days and 3 months
respectively. New machine worth Rs. 1,00,000 (W.O.V. Rs. 80,000) is to be sold and it is
expected to fetch Rs. 20,000. Depreciation is to be provided at 10% on cost, redeemable
preference shares are to be redeemed at \0% premium, by Issue of 2,000 equity shares of Rs. 100
each at 5% premium, 10% dividend is to be paid on equity capital as on 31-3-1991. Income tax
assessment of previous year is expected to be completed in the year ending 31 st March 1991, and
Rs. 20,000 additional payment is likely to be required. Advance income tax will be paid in the
year ending 31 st March 1991 Rs. 1,00,000. Income tax provisicn is to be made at 50% of net
profit. All workings should form part of your answer. leA (Fin), November 1990J
Solution:
1. Closing Dtebtors: Assuming that the entire sale was on credit basis, the closing balance of
Debtors can be computed as follows:

Debt COllection} = [ Debtors (Closing) J Debtors


:. 73 days = ( Rs. 40,00,000 )
Period Daily Credit Sale~
365 days

... Debtors -- t 3 days x Rs.165


40,00,OooJ --
_ d'ays
[RS. 40,00,000J
5 -- Rs. 8,00 ,000

2. Cash collected from customers (Debtors) = (Rs. 4,80,000 Opening Balance + Rs. 40,00,000
CUlTent Sales - Rs. 8,00,000 Closing Balance) = Rs. 36,80,000
Management Accounting: 312

3. Closing Stock: Sales Revenue Rs. 40,00,000


Less: Gross Profit (20%) 8,00,000
:.Cost of Sales 32,00,000

S toc k Vel OCI'ty = L Cost of Sales


A verage Inventory
J; 8 - [ Rs. 32,00,000
- A verage Inventory
J
:.Average Inventory =(Rs. 32,00,000 + 8) =Rs. 4,00,000
:. Opening Stock + Closing Stock = Rs. 4,00,000 x 2 = Rs. 8,00,000
Less: Opening Stock 4,60,000
:.Closing Stock 3,40,000
4. Purchase = Cost of Sales + Closing Stock - Opening Stock
= Rs. 32,00,000 + Rs. 3,40,000 - Rs. 4,60,000 = Rs. 30,80,000

5. Closing balance of Creditors (assuming all purchases on credit basis)

Creditors
Average 1_ Creditors (closing) 3 months =[ Rs. 30,80,000 J
Payment PeriodJ - Monthly Credit Purchases 12 months

... Creltors-
d' - ~x Rs.1230,80,OOOJ
month s
-- ( Rs. 30,80,000
4 J- R ."
-s 7 70 000

6. Cash paid to suppliers (creditors) = [Rs. 3,60,000 Opening Balance + Rs. 30,80,000 Current
Purchase - Rs. 7,70,000 Closing Balance] = Rs. 26,70,000.
7. Loss on Sale of Machine:

Acquisition cost of machine sold Rs. 1,00,000


Less: Accumulated ~epreciation 20,000
Written-down value 80,000
Sales Value 20,000
Loss 60,000
8. Computation of Profit:
Sales Revenue Rs. 40,00.000
Funds Flow Statement: 313

Less: Cost of Sales (80lk· as GP Ratio = 20~) 32,00.000


:. Gross Profit
Less: Expenses RIi. 4.00.000 8.00,000
Depreciation ( 10c,f. of Rs. 6,00,(00) 60.000
Loss on sale of machine 60,000
:. Profit before tax 5,20,000

Less: Provision for taxation 2,80,000

(50% of Rs. 2,80,000) 1.40,000


Tax arrears 20,000
Profit and Tax 1,60,000

Less: Dividend [10% of (Rs. 5.00.000 + Rs. 2.00.000)] 1,20,000

:. Retained Profit 70,000

9. Provision for Taxation: 50,000

Opening balance
Add: Provision for current year R~. 1040.000 Rs. 1.00.000

Arrears 20,000

Less: Payments made: 1,60,000

Previous year's tax Rs. 1,00,000 2,60,000·.

Arrears 20,000
Advance for 1990-91 1.00,000
:. Closing balance
2.20,000
10. Funds from Operation:
40,000
Profit after tax
Add: Depreciation
1,20,000
Loss on sale of machine
60.000
60.000
2,..J.0.UOO
Management Accounting: 314

Dr Bank Ale Cr
Amount Amount
Particulars Particulars
Rs. Rs.
To Balance bid 20,000 By Expenses 4,00,000
To Fixed Assets Alc (sale) 20,000 By Income Tax, 1989-90 1,20,000
To Debtors Alc .(collection) 36,80,000 By Income Tax, (90-91," Advance) 1,00,000
To Equity Share Capital Alc 2,00,000 By Creditors Alc (payments) 26,70,000
To Share Premium Alc 10,000 By Preference shareholders
(Rs. 1,00,000 + 10%) 1JO,OOO
By Dividend 70,000
By Fixed Assets Alc 1,00,000
By Balance bid 3,60,000
39,30,000 39,30,000
-

. W orki ne C api'ta I
Seh ed u1eofCh anges m
Amount (Rs.) as on Changes in Working
Particulars March 31, ... Capital (Rs)
1990 1991 Increase Decrease
Current Assets:
Stocks 4,60,000 3,40,000 0 1,20,000
Debtors 4,80,000 8,00,000 3.20,000
Bank 20,000 3,60,000 3,40.000
°
0
(a) 9,60,000 15,00,000
Current Liabilities:
Creditors 3,60,000 7,70,000 0 4,10,000
Provision for taxation 1,00,000 40,000 60,000 0
(b) 4,60,000 8,10,000
Working Capital (a - b) 5,00,000 6,90,000
Total changes in Working Capital 7,20,000 5,30,000
:. Net Increase in Working Capital - 1,90,000
7,20,000 7,20,000

.;.t-
Funds Flow Statement: 315

Funds Flow Statement (1990-91)


Amount Amount
Sources Uses
Rs. Rs.
Funds from Operation 2,40,000 Purchase of Fixed Assets 1,00,000
Sale of machine 20,000 Redemption of Preference Shares 1,10,000
Issue of equity shares 2,10,000 Dividend 70,000
Increase in Workin~ Capital 1,90,000
4,70,000 4,70,000

lllustration: 4.35
From the following summarized Balance Sheets of a company as at 31 st March, 1991 and
31st March, 1992 respectively, you are required to prepare:
a. a statement of changes in Working Capital; and
b. "tatement of sources and application of funds.
All workings should form part of your answers.
Liabilities 1991 1992 Assets 1991 1992
Equity share capital 75,000 1,20,000 Fixed Assets at Cost 2,40.070 2,53,730
10% Redeemable Less: Depreciation , ':)0,020 98,480
Preference share !
1,50,050 1,55,250
capital 1,00,000 80,000'
Investments 61,000 76,000
Reserve for replacement
of machinery 15,000 10,000 Stocks 98,000 1,04,000
Long term loans - 40,000 Trade Debtors 88,000 85.000
. Bank overdraft 22,000 - Bank 11,750 32,000
Trade creditors 84,450 75,550 ,',

Proposed Div. on equity


shares 12,000 24,000
Profit and Loss Account 1,00.350 1,02,700
4,08,800 4,52,250 -J..(lX,800 4,52.250
I

Additional Information:
1. During the year, additional equity capital was issued to the extent of Rs. 25,000 by way of
bonus shares fully paid up.
Management Accounting: 316

2. Final dividend on preference shares and an interim dividend of Rs. 4,000 on equity shares
were paid on 31 st March, 1992.
3. Proposed dividends for the year I!nded 31 sl March. IYLJ I were paid in October'IYY I.
4. Movement in Reserve for Replacement of Machinery Account represents transfer to
Profit and Loss Account.
5. During the year, one item of plant was up valued by Rs. 3,000 and credit for this was
taken in the Profit and Loss Account.
6. Rs. 1.700 being expenditure on fixed assets for the year ended 31st March, 1991 wrongly
debited to Sundry Debtors then, was corrected in the next year.
7. Fixed assets costing Rs. 6,000 (accumulated depreciation Rs. 4.800) were sold for Rs.
250. Loss arising therefrom was written off.
8. Preference shares redeemed in the year (June 1991) were out of a fresh issue of equity
shares. Premium paid on redemption wa., 10%.
leA (Fill), May 1993J
Solution:

I. Equity Share Capital:


Opening balance Rs.75.000
Add: Bonus issue 25.000
1.00.000
Closing balance 1.20.000
:.Issue of equity shares for cash 20,000
2. Preference Share Capital:
Opening balance Rs. 1,00,000
Closing balance 80.000
Redeemed Capital 20,000

Rs. 20.000 Preference Capital redeemed at 1-0% premium. Therefore, cash outflow on
this account = [Rs. 20.000 + 10% of Rs. 20,000J = [Rs. 20,000 + Rs. 2,000] = Rs. 22.000.

3. Fixed Assets:
At cost at the beginning of 1991-92 (i.e., 1-4-1991) Rs. 2,40,070
Add: Expenditure wrongly debited to Sundry Debtors 1.700
Increase in the value 3.000
2.44,770
Funds Flow Statement: 317

Less: Cost of Fixed Assets sold 6.0()O

2.JX.770
Fixed Assets at cost at the end of the year 2.5J.7.30

:. Purchases 14.%0

4. Cost of Fixed Assets sold: Rs.6.0()0

Less: Accumulated depreciation 4.XOO

Wriul!n dmvn valuL' 1.20()

Selling price 250

.". Lo ... .., lJ50

5. Dividend: On Preference Share Capital: I ()l!c tlf R~. :SO.OOO = R'i.H.OOO


On Equity Share Capital: Interim R~. -U)O()
6. Depreciation
At the beginning Rs. lJO.020

Less: Accumulated dcpreciation on aSSl!t sold 4.XOO


X5,22()

Depreciation at the end 9X,4XO

." Depreciation for 1991-92 13.260

7. l·lInds from Operation


Profit earned during 1991-92 and retained (Rs. 1,02.700 - Rs.\.OOJ50) Rs.2.350
Add: Depreciation for 1991-92 13,260
Prcmium on redemption uf prcferClll'l! 'ihare ... ( IO(i( of R..,. 2().OO{)) 2.000
Lo..,.., un .."I1l· of Fi "(cd :\ ..,..,ch 1)50

Bonus i.,suc (si nce there was no rC ... Lrve) 25,000


Dividend: Interim dividend (equity) 4.000
Final dividend on equity shares (\ 991-92) 24.000
Preference shares 8,000
Less: Increase in value credited to Profit and Loss Alc Rs.3,OOO 79,560
Transfer from Reserve for Replacement of Machinery 5,000 8.000
71.560
Management Accounting; 318

Sch ed ueo " W ork"109 C apl"tal


I fCh anges 10
Amount (Rs.) as on Changes in Working
Particulars March 31, ... Capital (Rs)
1991 1992 Increase Decrease
Current Assets:
Stock
.. 98.000 1.04.000 6.000 0
}

Trade Debtors (Rs. 88,000 - Rs. 1,700) 86.300 85.000 - 1,300


Bank 11.750 32.000 20.250 0
(a) 1.96.050 2.21.000
Current Liabilities:
Bank Overdraft 22.000 - 22.000 0
Trade Creditors 84.450 75.550 8.900 0
(b) 1.06.450 75.550
Hence. 1. Working Capital (a - b) 89.600 1.45.450
2. Changes in Working Capital i I I
57.150 I 1.300
I I I
I

3. Net increase in Working Capital 55.850 55.850 II


1.45.450 1.45.450 57.150 57.150 I
F un d s Flow Statement Iior t he year end e d Marc h 31 , 1992
Amount Amount
Sources Uses
Rs. Rs.
Funds from Operation 71,560 Redemption of Preference Shares 22.000
Issue of Equity Shares (I) 20.000 Purchase of Fixed Assets (3) 14.960 I
Sale of Fixed Assets i 250 Payment of Equity Dividend: i
I

Long term Loans -1-0.000 Interim '4.000 I


Final (for 1990-91) 12.000·1
Purchase of Investment 15.000
Payment of Preference Dividend 8.000
Increase in Working Capital 55.850

1.31.~~) i 1.31.8 to

Illustration: 4.36

The Balance Sheets for Hugo Products are gi yen below for the years 1986 and 1987.
Funds Flow Statement: 319

Liabilities: 1986 (Rs.) 1987 (Rs.)


Share Capital 12.00.000 14,00.000
General Reserve 4.00.000 5.00.000
Profit on sale of Investment 20,000
Profit and Loss Ale 2.00,000 4.00.000
7O/C Debentures 6.00.000 4.00.000
Creditors for expenses 20.000 24,000
Creditol's of supply of goods 3,20.000 5,00,000
Propose.d dividend 60,000 70.000
Provision for taxation 1,40,000 1,50,000
29,40,000 34,64,000
Assets:
Fixed Assets 20,00.000 24,00.000
Less: Related depreciation 4.00.000 5.00.000
16.00.000 19.00.000
Investment at cost 3.60.000 3,60.000
Stock (at cost) 4,00.000 5.40,000
Sundry debtors (less provision Rs.
40.000 and Rs. 50,000 respective\) ) 4.50.000 4.90,000
Bills Receivable ~O.O()O 1.30.000
Prepaid expenses 20,000 24,000
Miscellaneous expenditure 30.000 20,000
29,40,000 34,64,000

Other information:

I. During the year 1987. fixed assets (valued at Rs. 20,000, depreciation written off Rs.
60,000) was sold for Rs. 16,000.
2. The. proposed dividend of last year was paid in 1987.
3. During the year 1987, investments costing Rs. 1.60,000 were sold and later in the year,
investments of the same cost were purchased.
4. Debentures were redeemed at a premium of 10% in 1987.

"
Management Accounting : 320

5. Liability for taxation for 1986 came to Rs. 1.10.000.


6. During the year 1987. bad debts written off were Rs. 30.000 against the provision
amount.
You are required to prepare a Funds Flow Statement. [ICWA (Fill), JUlle 1988J
Solution:
Dr ( a ) F'Ixed A ssets AI'& Cr
Amount Amount
Particulars Particulars
Rs. Rs.
To Balance bId 20.00.000 By Provision for Depreciation Alc I 60.000
To Bank (balance. Purchase) 4.80,000 By Bank (sale) 16.000
I
By P and LAic (loss on sale) 4.000 I
By Balul1l;e c/d 24.00.000 I
24.80.000 24.80,000 I
(b) Provision for Depreciation at the beginning Rs. 4.00.000
Less: Depreciation on fixed asset sold 60.000
3.40,000
Provision for Depreciation at the end 5:00.000
:. Depreciation for the year 1,60,000

Dr (e) Provision for Taxation Ale Cr


Amount Amount
Particulars Particulars
Rs. Rs.
To Bank 1.10.000 By Balance bid 1.40.000
I
To Balance cld 1.50.000 By Profit and Loss Nc (bal) 1.20.000 I
2.60.000 2.60.000

Dr (d) Investment Ale Cr


Amount Amount
Particulars ParticulaI's
Rs. Rs.
To Balance bId 3,60.000 By Bank (Rs. 1.60.000 + Rs.20.000) 1.80.000
To Profit on Sale of Investment Nc :W,OOO B) Balance c/d 3.60,000
I
To Bank (Purchase) 1.60.000 I
i
5.40,000 5,40,000
Funds Flow Statement: 321

(e) Funds from Operation


Profit earned during 1987 and retained (R~. 4.0U.()()O - R~. 1.00,000) Rs. 2.00.00()
Add: Loss on sale of fixed assets R~. 4,000
Depreciation for the year 1,60.000
Proposed dividend for 1987 70,000
Transfer to (; 'L'ral Reserve Alc 1,00,000
Provision for Taxes (c) 1.20.000
Miscellaneous expenses written off 10,000
Premium on redemption of Debentures ( lOCk of Rs. 20.000
4.84,000
2.00.000)
6.1\4,000
Funds from Operation
Schedule of Changes in Working Capital
I Amount (Rs.) as on Change" in Working
I Particulars I December J I. ... Capital (Rs)
I t
Il)X6 I 19X7 Increase i Decrease
I
i--C--,----u-rr-en-t-A-ss-.e-ts-:- - - - - - - - - - - --1---- - - - - - --r- -,---=-:~-==:--=--+_=:....::...::c.:....:....:c::.....:.._l

I Stock ...J..()().()()() I
i
.'i ....J.().O()() I....J.O.()OO ()

Sundry Debtors 4.50.000 I


:
4.90.000 .
t
40.000 o
Bills Receivable 80.000
I
t
I 1.30.nOn I
I
SO.OOO ()

Prepaid Expenses 20.000 24.000 4.000 o


(a) 9,50.000 11.84.000
I
Current Liabilities: I
I
t
I
I
I
. Creditors for Expenses 20.000 !
I
14.000 O! 4.000 !
i
Creditors for Supply of Goods 3.20.000 5.00.000 0
1
1.80.000 I
(b) 3.40.000 I 5.24,000 I
I
Therefore, Working Capital (a - b) 6.10.000 6.60.000 I I
Changes in Working Capital I 2.J4.000 I 1.84.000
I 1
I i
:. Net increase in Working Capital 50.000 I I I 50.000

.6.60.000 ! 6.60.000 I 2.34.000 I


t
2.34.000
Management Accounting : 322

e ecem b er 31 , 1987
F und s Flow Statement 0 fH ugo Pro d ucts~or the year enddD
Amount Amount
Sources Uses
Rs. Rs.
Funds from Operation (e) 6,84.000 Purchase of Fixed Assets (a) 4,80,000
Sale of Fixed Assets 16,000 Payment of Taxes (c) 1.10,000
Sale of Investments (d) 1,80,000 Purchase of Investment 1,60.000
Issue of Shares 2,00,000 Redemption of Debentures
(including 10% premium) 2,20.000
Payment of Dividend 60.000
Increase in Working Capital 50.000
10,80.000 10.80,000

Illustration: 4.37

Two divisions of XYZ Ltd.:start the year 1988 with identical Balance- Sheets but the
position changed by the end of the year as shown below.

Division 'A' Division 'B'


Particulars 1-1-88 31-12-88 1-1-88 31-12-88
(Rs.) (Rs.) (Rs.) (Rs.)
Current Assets 6.25.000 6,25.000 6.25,000 6.25.000
Current Liabilities 3,75,000 3,75,000 3,75,000 5,00,000
Working Capital 2,50.000 2.50,000 2.50.000 1.25,000
Fixed Assets (net) 2.50.000 6.25,000 2.50,000 5.00,000
Capital Employed S.OO.OUO H.75.000 5.00.000 6.25.000
- .
Financed by:
Long term Debt - 2.50,000 - I- -
Equity Share Capital and Reserves 5,00.000 6,25.000 5.00.000 6.25,000
5.00.000 8.75.000 5.00.000 6.25.000

You are required to prepare Funds Flow Statement for each di\'i-;ion and comment on the
financial policy and practices adopted hy each as revealed by the Fund Flow Analysis. You have
the following additional information.
1. Both the divisions have identical earning power.
2. Each division earns a net profit of Rs. 60,000 after taxation at 50%.
3. Depreciation amounts to Rs. 40.000. /ICWA (Fill), JUlle 1989J
Funds Flow Statement: 323

Solution:

Division Division
'A' (Rs.) 'B' (Rs.)
a. Computation of Funds from OperatIon:
Net profit for the year ended December 31, 1988 60,000 60,000
Add: Depreciation written off 40,000 40,000
Taxes (equivalent to protit after taxes as tax rate is 50%) 60,000 60,000
:. Funds from Operation 1,60,000 1,60,000
b. Purchase of Fixed Assets:
Balance at the end of 1998 (net) 6,25,000 5,00,000
Add: Depreciation 40.000 40,000
6.65,000 5.40,000
Less: Balance at the beginning of 1998 2.50,000 2,50.000
----

:. Pun:hases 4,15,000 2,90.000


c. Changes in Working Capital: Working Capital at thL': End 2,50,000 1,25.000
Beginning 2.50.000 2.50.000

:. Increase in Working Capital 0 (-) 1,25,000

d. Issue of Equity Shares


Balance at the end 6.25.000 6.25,00

Less: Balance at the beginning Rs. 5.00,000 R.... ).lIO.O()O


60,000 60,000 5.60.000 5,60,000

:. Capital mobilized 6:".000 65,000


----

Funds Flow Statement for the year ended I>ecember 31.1988


Amount (Rs)
Particular~
____-+-DivislOn 'A' I Division 'B'
I Source" of Funds:
I
Funds from Operation (a) I 1.60.000 1.60,000
Long-term Loan 2.S0,O()(J -
Issue of Equity Shares (d) 65.000 65,000
Decrease in Working Capital (c) - 1.25,000
4,75.000 3,50,000
Management Accountfng : 324

Application of Funds:
Purchase of Fixed Assets (b) 4.15.000
I 2.9U.OOO
Payment of Taxes 60.000 I 60,000
4.75.000 3.50.000

Illustration: 4.38

The comparative Balance Sheets of XYZ Company are given below.


I
(R!>. lakh) End of
PaJ:ticulars
1990 1991
Share Capital 500 500
Reserves and Surplus 425 500
Long term Debt 300 330
Short term Bank borrowings 200 225
Trade Creditors 100
I,
\\.'>.95 I

Provisions i 75
I
I
',w

HO I
I I
1,600 1.730
I Fixed Asse" (net) 850 1.000
!Inventories 340 350
I, Debtors 360 330
I Cash 30 35
I
Other Assets 20 15
1.6()() 1.730

The Income Statement of XYZ Company for the year 1991 is given below.

Particulars (Rs. lakhs)

I Net Sales 1 2,040


I Co"t of !!oods ... old: Stock ...
I ~
1.0 IO I
\~'ages and Salaries 210 I

Other Manufm:turillg. E -.;penscs 140 \,360


Gross Profit 6RO
Operating Expenses: Depreciation 110
Funds Flow Statement: 325
,--------------------- I
Selling, administration and ~Jeneral 230 340
Operating Profit 340
Non-operating Surplus 25
Profit before Interest and Tax 365
Interest i
70
I

Profit before Tax 295


Tax 130
Profit after Tax I 165
I I
Dividends I 90
Relaineci Earnlllo~ 75

1. Prepare a sources and uses of funds statement on Working Capital basis.


2. Prepare a sources and uses of funds statement on total resources basis.

Solution:

Funds Flow Statement on Working Capital basis

(a) Funds from Operation: (b) Purchase of Fixed Assets:


Profit after tax Rs. 165 lakh Fixed Assets at the
beginning of 1991 850lakh
Add: Depreciation 1I0Iakh
Less: Depreciation for 1991 110 1akh
:. Funds from Operation 275lakh
740lakh
Closing balance of Fixed Assets 1,0001akh
Purchase (difference) 260lakh

. W ork'109 C aplta
(c) Sc hed u IeofCh anges 10 . I (Rs'
.10 Ia kh s )
Amount (Rs.) as Changes in Working
Particulars on December 31. Capital (Rs)
1990 1991 Increase Decrease
Current Assets: I

Inventories 340 350 I 10 0


!
Debtors 360 330 0 30
Cash 30 35 5 0
Management Accounting : 326

Other Assets 20 15 0 5
(a) 750 730
Current Liabilities:
Short term bank borrowings 200 225 0 25
Trade Creditors 100 95 5 0
Provisions 75 80 0 5
(b) 375 400
Hence. Working Capital (a - b) 375 330
<;hanges in Working Capital 20 65
:. Net decrease in Working Capital 45 45
375 375 65 65

F uncis FIow Statement 0 fXYZC ompany £or the year end ed M arch 31, 1991 (Rs. Iakh)
Amount Amount
Sources Uses
Rs. Rs.
Funds from Operation (a) 275 Purchase of Fixed Assets (b) 260
Long-term Debt 30 Payment of Dividends 90
Decrease in Working Capital 45
350 350
Funds Flow Statement on Total Resources basis
Funds Flow Statement (Total Resources Basis) of XYZ Company for the year ended
December 31, 1991 (Rs Lakh)
Amount Amount
Sources Uses
Rs. Rs.
Profit before Tax 295 Payment of taxes 130
Depreciation 110 Payment of dividend 90
Increase in Capital and Liabilities: Decrease in Capital and Liabilities:
Long term debt 30 Trade Creditors 5
Short term borrowings 25 Increase in Assets:
Provisions 5 Fixed Assets (purchases) 260
Decrease in Assets: Inventories 10
De~tors 30 Cash 5
Other assets 5
500 500
Funds Flow Statement: 327

Illustration: 4.39

A fire destroyed the books of account of Vikas '·.td .. on 31-3-95. The Chief Accountant
also noticed that the entire cash kept in the cash box wa:-. also destroyed. However. the following
information was available with the Chief Accountant.

Balance Sheet (Rs. in lakhs)


Liabilities 1994 1995 Assets 1994 1995
Share Capital Sundry Fixed Assets:
Equity Shares of Rs. 10 each 1,000 ? Gross Block 1,600 ?
General Reserve 900 ? Less: Depreciation 320 ?
Share Premium 20 ? Net Block 1,280 1,280
14% Debentures 400 450 Investment 600 700
Cash Credit 90 ? Stock 500 700
Sundry Creditors 180 220 Sundry Debtors :no I ')

.)
Provision for Taxation 10 Ca~h at Bank 45 120
.) ?
Proposed Dividend 150 Cash on hand 5
2.750 ? 2.750 ?

For the year ended 31 st March, 1995 the following transactions took place
I. The company issued 20 lakh equity shares of Rs. 10 each at a premium of Rs. 10%.
2. Fixed assets costing Rs. 403 lakhs was purchased during the year. An old asset (original
cost Rs. 3 lakh .. and accumulated depreciation Rs. I lakh) was sold for Rs. 1 lakh.
3. It paid advance tax of Rs. 70 lakhs for 1994-1)) and also balance tax liability of Rs. 8
lakhs for 1993-94. The excess provision for 1993-94 was transferred to general reserve.
The provision for taxation for the year 1994-95 was Rs. 90 lakhs.
4. Dividend for the year 1993-94 was fully paid. The company proposed a dividend of 20%
for the entire share capital standing as on 31st March, 1995.
5. The total increase in cash credit and sundry creditors at the year end was found to be Rs.
70 lakhs and the increase with respect to stock and debtors was found to be Rs. 330 lakhs.
6. The Chief Ac.countant remembered that the cash from operations for the year was Rs. 540
lakhs.
7. It was decided to write off the cash loss by fire. if any.
You are required to assist the Chief Accountant in completing the Balance Sheet as on 31 st
March, 1995 along with a Statement of Sources and Application of Funds. leA (Fin), May 1986J
Management Accounting : 328

Solution:

(a) Share Capital = [Rs. 1,000 lakh + (20 lakh shares x Rs. 10)]
=[Rs. 1,000 lakh + Rs. 200 lakh) =Rs. 1.200 lakh.
(b) Share Premium = [Rs. 20 lakh + (10% of Rs. 200 lakh)] = [Rs. 20 lakh + Rs. 20 lakh]
= Rs. 40 lakh

(c) Cash Credit = Rs. 90 lakh + [Rs. 70 lakh -- (Rs. 220 lakh - Rs. 180 lakh creditors)]
= IRs. 90 lakh + (Rs. 70 lakh - Rs. 40 lakh)] = [Rs. 90 lakh + Rs. 30 lakh] = Rs. 120
lakh.

(d) Provision for Taxation = lRs. YO lakh provision for IYY4-Y5 - Rs. 70 lakh advance for Y4-
95) =Rs. 20 lakh.

(e) Proposed Dividend for 1994-95 = 20% ofRs. 1,200 lakh = Rs. 240 lakh.
(f) Fixed Assets (Gross Block) on March 31, 1994 = Rs. 1,600 lakh
Less: Original cost of old assets sold 31akh
1,5971akh
Add: Purchases 4031akh
Gross value of fixed assets on 31-3-1995 2,000 lakh
Gross value of Fixed Assets on 31 ~3-1995 Rs. 2,000 lakh
Less: Net Fixed Assets on 31-3-1995 1,280 lakh
Accumulated depreciation till 31-3-1995 nOlakh
(g) Depreciation (till March 31,1994) Rs. 320lakh
Less:' Depreciation provided on the asset sold I lakh
3191akh
Add: Depreciation for 1994-95 (Rs. 720 lakh -
Rs.319Iakh) 401lakh
Accumulated Depreciation till March 31, 1995 no lakh
Original cost 'of old assets sold = Rs. 3,00,000
Less: Depreciation 1,00,000
WDV of the assets sold 2,00.000
Sales price 1,00,000
:. Loss on sale of old assets 1,00,000
Funds Flow Statement : 329

(h) Sundry Debtors = [Rs. 320 lakh + (Rs. 330 lakh increase in debtors and stock - Rs. 200
lakh increase in stock)) (Cash = 0 as it was burnt in the fire mishap).
= [Rs. 320 lakh + Rs. 130 lakh] = Rs. 450 lakh.

Balance Sheet ofVikas Ltd., as on March 31,1995


Amount Amount
Capital and Liabilities Assets and Properties
(Rs.lakh) (Rs.lakh)
Share Capital [Equity Shares of Sundry Fixed Assets (Gross) (f) 2,000
Rs. 10 each (a)] 1,200
Less: Accumulated Depreciation 720
Share Premium (b) 40
Net 1,280
General Reserve (balance) 960
Investments 700
14% Debentures 450
Stock 700
Cash Credit (c) 120
Sundry Debtors (h) 450
Sundry Creditors 220
Cash at bank 120
Provision for taxation (d) 20
Cash on hand 0
Proposed dividend (e) 240
3,250 3,250

(i) Opening balance of cash and bank balances Rs. 50 lakh


Add: Issue of shares at premium 220
Issue of debentures 50
Sale of Fixed Assets
Cash from operation 540 861

Less: Purchase of Fixed Assets 403


Investments 100
Payment of Dividend 150
Payment of advance tax for 1994-95 70
Payment of tax for 1993-94 8 731
.'. Closing balance of cash and bank 130
Less: Closing balance of bank 120
:. Cash balance at the end (destroyed) 10
Management Accounting: 330

G> Funds from Operation


Profit transferred to General Reserve Ale (Rs. 960 lakh -
Rs. 60 lakh
Rs. 900 crore)
240
Proposed Dividend (e)
90
Provision for taxes
401
Depreciation (g)
Loss on sale of fixed assets (0
10
Cash loss (i)
802
2
Less: Excess tax provision (Rs. 10 lakh - ~s. 8 lakh)
800
:. Funds from Operation

SchedoleofCh an2es W orki n2 C aplta


. I
Amount (Rs.) Changes in Working
Particulars as on March 31 Capital (Rs. lakh)
1994 1995 Increase Decrease
Current Assets:
Stocks 500 700 200 0
Sundry Debtors 320 450 130 0
Cash at Bank 45 120 75 0
Cash on hand 5 0 0 5
(a) 870 1,270
Current Liabilities:
Cash Credit 90 120 0 30
Sundry Clooitors 180 220 0 40
(b) 270 340
Hence, Working Capital (a - b) 600 930
Changes in Working Capital 405 75
:. Net intfr¢.iiSe in Working Capital 330 - 330
930 930 405 405
Funds Flow Statement: 331

I as Ltd., ~or th e year en d ed M arch31 , 1995


F und s FIow Statement 0 fV"k
Amount Amount
Sources Uses
(Rs.lakh) (Rs.lakh)
Funds from Operation 800 Purchase of Fixed Assets 403
Issue of Equity Shares (including Purchase of Investments 100
Premium) 220
Payment of Dividends 150
Issue of Debentures 50
Payment of Tax:
Sale of Fixed Assets 1
for 1993-94 8
Advance for 1994-95 70
Cash Loss 10
Increase in Working Capital 330
1,071 1,071

mustration: 4.40

ABC Limited gives you its Balance Sheet on March 31, 1995 and its Projected Profit and
Loss Account for the year ertded March 31, 1996.

Balance Sheet as on March 31,1995


Liabilities Rs. Assets Rs.
Share Capital: Fixed Assets:
Equity Shares of Rs. 100 each Machinery at cost 7,00,000
fully paid 6,00,000
Less: Depreciation 1140 1000 5,60,000
Reserve and Surplus:
Motor Car at cost 80,000
Share Premium 20,000
Less: Depreciation 30,000 50,000
General Reserve 1,30,000
Current Assets:
Profit and Loss Account 65,000
Stock 5,60,000
Secured Loans:
Book Debts 2,20,000
8% Debentures 3,00,000
Bank Balance 1,20,000
Current Liabilities:
Loans and Advances: .
Sundry Creditors 2,85,000
Advance Income Tax 1,00,000
Provision for Taxation 1,40,00ll::, Miscellaneous Expenditure: .
Proposed Dividend (Equity) 90,000
Preliminary Expenses 20,000
16,30,000

- 16,30,000
Management Accounting : 332

Pr'oJectdPr
e I andLoss Accountfior the year end ed31 stMare,
ofit h 1996
Particulars Rs. Particulars Rs.
To Opening Stocks 5,60,000 By Sales: Cash 3,70,000
To Purchases 14,40,000 Credit 18,00,000
To Wages 80,000 By Stock 4,20,000
To Manufacturing Expenses 40,000 By Profit on Sale of Machinery 10,000
To Office and Administration
Expenses 50,000
To Selling and Distribution
Expenses 30,000
To Interest 24,000
To Depreciation:
Machinery 56,000
Car 14,000 70,000
To Preliminary Expenses 10,000
To Provision for Taxation 1,36,000
To Proposed Dividend on Equity 1,00,000
To Balance 60,000
26,00,000 26,00.000

The company proposes to issue one equity share for two equity shares with a nominal
value of Rs. 3,00,000 at a premium of 10%. Machinery will be acquired for Rs. 1,00,000. The
cost of machinery to be sold in the year ended 31st March, 1996 is Rs. 80,000 with a depreciation
provision of Rs. 45,000. It is expected that:
a. Tax liability up to 31st March, 1995 will be settled for Rs. 1,20,000 within 31st March,
1996.
b. Advance Income Tax amounting to Rs. 1,30,000 is proposed to be paid in 1995-96.

c. Book Debts will be 10% more than warranted by the credit period of two months.
d. Creditors for goods will continue to extend one and half months credit and manufacturing
expenses outstanding at the end of March 1996 will be Rs. 5,000.
Funds Flow Statement: 333

You are required to:

(a) Draft the Company's Projected Balance Sheet as on 31st March, 1996.
(b) Draft the statement showing sources and application of funds during the year ended 31st
March, 1996. lCA (Fin), November 1995J
Solution:

(a) Share Capital and Premium:


On 31-3-1995 (6,000 shares of Rs. 100 each) Rs. 6,00,000
Add: Bonus issue [(1/2 x 6,000) x Rs. 100] 3,00,000
9,00,000
Share Premium on 31-3-1995 Rs.20,000
Add: Premium on Bonus Issue [10% of (Rs. 100 x 3,000)] 30,000
50,000
(b) Machinery and Depreciation:
Cost of Machinery on 31-3-1995 Rs. 7,00,000
Less: Cost of Machinery to be sold 80,000
6,20,000
Add: Cost of Machinery to be acquired 1,00,000
.o. Cost of Machinery on 31-3-1996 7,20,000
Less: Depreciation up to 31-3-11995 Rs. 1,40,000
Less: Depreciation on Machinery to be sold 45,000
95,000
Add: Depreciation for 1995-96 56,000
:. Depreciation up to 31-3-1996 1,51,000

:. Written Down Value on 31-3-1996 5,69,000

Cost of Machine to be sold (net of Depreciation) (Rs.


80,000 - Rs. 45,000) 35,000

Add: Profit on the sale of Machinery 10,000

.o. Selling Price of the Machine to be sold 45,000


Management Accounting : 334

(c) Tax Provision for 1994-95 Rs. 1,40,000


Tax Settled 1,20,000
:. Excess Provision made Rs. 20,000
(d) Book Debts on 31-3-1996
Credit Sales: for the year =Rs. 18,00,000
Permonth = 1,50,000
.'. Book Debts =2 month's credit sales = (Rs. 1,50,000 x 2) Rs. 3,00,000

Add: 10% 30,000


3,30,000
Debtors on 31-3-1995 Rs. 2,20,000 (e) Creditors:
Add: Credit Sales 18,00,000 Opening balance Rs. 2,85,000
20,20,000 Add: Purchases 14,40,000
Less: Closing balance 3,30,000 17,25,000
:, Cash collected 16,90,000 Less: Closing balance
[(14,40,000 x (1.5/12)] 1,80,000
:. Creditors paid 15.45,000

p rOJectedBaance
0 I Sheet 0fABCL t d 0' as on M arch31 , 1996
Amount Amount
Capital and Liabilities Assets and Properties
Rs. Rs.
Equity Share Capital Fixed Assets:
(Rs. 100 each) 9,00,000
Machinery: Cost 7,20,000
Reserves and Surplus ~

Less: Depreciation 1,51,000


Share Premium 50,000

5,69,000
General Reserve 1,30,000
Motor Car: Cost 80,000
Add: Excess Tax
Less: Depreciation
Provision 20,000 1,50,000
(30,000 + 14,000) 44,000 36,000
P and L Alc Balance 65,000
Current Assets:
Add: Current year
Stock 4,20,000
Profit 60,000 1,25.000
Book Debts 3,30,000
Secured Loans: 8% Debentures 3,00,000
Bank Balance (bal) 4.51.000
Current Liabilities:
Sundry Creditors 1,80,000 Loans and Advances:
Funds Flow Statement: 335

Tax Provision 1,36,000 Income Tax 1,30,000


Proposed Dividend 1,00,000 Miscellaneous:
Creditors for Expenses 5,000 Preliminary Expenses 20,000
Less: Written-off 10,000 10,000
19,46,000 19,46,000

(0 Funds from Operation:


Profit for the year (as per Projected P & LAIc) Rs.60,OOO
Add: Depreciation 70,000
Preliminary Expenses 10,000
Provision for taxation 1,36,000
Proposed Dividend 1,00,000
3,76,000
Less: Profit on sale of Machinery 10,000
3,66,000
'Workine: C apita
ScheduIeofCb ani es lD . I
Amount (Rs.) as on Changes in Working
Particulars March 31, ... Capital (Rs)
199~ 1996 Increase Decrease
Current Assets:
Stocks 5,60,000 4,20,000 0 1,40,000
Book Debts 2,20,000 3,30,000 1,10,000 0
Bank Balance 1,20,000 4.51,000 3,31,000 0
(a) 9,00,000 12,01,000
Current Liabilities:
Sundry Creditors 2,85,000 1,80,000 1,05,000 0
Expenses Creditors 0 5,000 0 5,000
(b) 2,85,000 1,85,000
Hence, Working Capital (a - b) 6,15,000 10,16,000
Changes in Working Capital 5,46,000 1,45,000
:. Net increase in Working Capital 4,01,000 4,01,000
10,16,000 10,16,000 5,46,000 5,46,000
Management Accounting : 336

oJectedFundsFlow StatementofABeLtd'0 Iior th e year end·109 March31, 1996


Pr·
Amount Amount
Sources Uses
Rs. Rs.
Funds from Operation 3,66,000 Purchase of Machinery 1,00,000
Sale of Machinery 45,000 Advance Tax Payment for 1995-96 1,30,000
Increase in Capital and Premium 3,30,000 Payment of Dividend for 1994-95 90,000
Tax (dit) for 1994-95 20,000
Increase in Working Capital 4,01,000
7,41,000 7,41,000

mustration: 4.41
Given below are the Balance Sheets of A Limited for a period of three years as at 31st
March each year (Rs. in Iakhs).

1991 1992 1993


Liabilities:
Share Capital in equity shares of Rs. 10 each 30 35 35
General Reserve 10 15 18
Surplus 5 8 9
13% Debentures 10 5 10
Bank Credit 5 10 15
Trade Creditors 10 12 15

Income Tax provision for the current year 8 11 14
Proposed Dividends 6 10.5 14
84 106.5 130
Assets:
Plant and Machinery 45 55 70
Investments 10 15 20
Stocks 12 15 15
Debtors 14 15 12
Cash and Bank 3 6.5 13
84 106.5 130
Funds Flow Statement : 337

Other details:

a. Depreciation provided in the books: 1990-91: Rs. 6 lakhs


1991-92: Rs. 8 lakhs
1992-93: Rs. 10 lakhs
b. A part of the Debentures was converted into equity at par in September 1991.
c. There was no sale <;>f fixed assets during the period.
The management seeks your advice on the liquidity position of the company. You are to
use the Funds Flow Statement for the purpose. lCA (Fin), November 1993]
Solution:
(a) Funds from Operation (Rs. lakh): 1991-92 1992-93
Increase in Surplus 3.0 1.0
Add: Transfer to General Reserve 5.0 3.0
Depreciation 8.0 10.0
Income Tax provision 11.0 14.0
Proposed Dividend 10.5 14.5
37.5 42.0

'Wki
(b) Sch ed u1eofCh anges lD or ng C 'I
aplta
Amount (Rs. lakh) as on
Particulars March 31.. ..
1991 1992 1993
Current Assets:
Stock 12.0 15.0 15.0
Debtors 14.0 15.0 12.0
Cash and Bank 3.0 6.5 13.0
(a) 29.0 36.5 40.0
Current Liabilities:
Bank Credit 5.0 10.0 15.0
Creditors 10.0 12.0 15.0
(b) 15.0 22.0 30.0
:. Working Capital (a - b) 14.0 14.5 10.0
In~rease (Decrease) in Working Capital - 0.5 (4.5)
Management Accounting : 338

(c) Plant (Rs. lakh): 1990-91 1991-92 1992-93


Opening balance (*Closing + Depreciation) *51 45 55
Less: Depreciation 6 8 10
45 37 45
Closing balance 45 55 70
:. Purchases 0 18 25

Funds Flow Statement of A Ltd., for the Periods ended March 31L, 1992 and 1993
Amount Amount
Sources (Rs.lakh) Uses (Rs.lakh)
91-92 92-93 91-92 92-93
Funds from Operation (a) 37.5 42.0 Purchase of Plant 18.0 25.0
Issue of Share Capital 5.0 0.0 Redemption of Debentures 5.0 0.0
Issue of Debenture 0.0 5.0 Payment of Tax 8.0 11.0
Decrease in Working Capital 0.0 4.5 Payment of Dividend 6.0 10.5
Purchase of Investment 5.0 5.0
Increase in Working Capital 0.5 0.0
42.5 51.5 42.5 51.5

(d) Calculation of Liquidity Ratios (March 31):


1991 1992 1993

current'L
Ratio f
r
Current Assets
= ~urrent Liabilitie~
1 RS. 29 lakh
= ( Rs.15Iakh)
I rlRs.22lakh
Rs. 36.5 lakhJ Rs. 40 lakh
( Rs. 30 lakh)
I
= 1.93 : 1 = 1.66: 1 = 1.33 : 1
LiqUid} =LLiguid Assets J = (RS.17Iakhl RS. 21.5 lakh I Rs. 25 lakh I
Ratio I!-iquid Liabilitie~) Rs. 10 lakh) ( Rs. 121akh) ( Rs.15Iakh)

= 1.7: 1 = 1.79: 1 = 1.67: 1


From the above. it is obvious that the amounts of funds mobilized through the issue of
shares and debentures during 1991-92 and 1992-93 of R~. 5 lakh and Rs. 5 lakh respectively, were
much lower than the amounts of funds used for the purchase of plant. investments and for
redemption of debentures of Rs. 28 lakh and Rs. 30 lakh respectively. 'That means, the company
has used a major part of Funds from Operations to finance its capital assets, long-term
investments and redemption of debentures. This has resulted in the reduction in the amount
available for Working Capital. This is evident from a marginal increase in Working Capital by
Funds Flow Statement: 339
the end of 1991-92 and a substantial reduction in Working Capital by the end of 1992-93.
Further, the Current Ratio has registered a continuous decline and moreover, it has been lower
than the standard of 2 : 1. Though the Liquid Ratio is higher than the standard of 1 : I, its
dependence more on bank credit is not desirable. Further, higher Dividend Pay-out has resulted
in lower Retention Ratio.
Summary of the Chapter
The concepts of Fund. Flow of Funds and the Funds Flow Statement are introduced and
anlysed at the beginning of this chapter followed by the importance and managerial uses of Funds
Flow Statement. Different sources and uses of Fund are also discussed in detail preceeded by
how the business transactions cause the flow of Funds. Then, a detailed procedure of preparing
the Funds Flow Statement is presented followed by its limitations. A large number of problems
on Funds Flow Statement are solved at the end.
Key-Terms to Remember
Fund Flow of Fund
Current Assets Current LIabilities
Working Capital Funds from Operation
Net Working Capital Funds Flow Statement
!'
Retained Profit Non-cash Expenses
Non-operating Expenses Non-operating Income
Book Costs Non-current Account Items
Current Account Items
Questions for Self - Study
01. What is Fund? What are the sources and application of Fund?
02. What do you mean by Funds from Operation? How is it computed?
03. Briefly describe the procedure of preparing a Funds Flow Statement.
04. What are the steps involved in the preparation of Fund Flow Statement?
[Bangalore Uni, B.Com, May 2001J
05. Define Funds Flow Statement and explain how it differs from the conventional Financial
Statements?
06. Briefly explain the Working Capital concept of funds flow.
07. Briefly describe the procedure for converting the net income into Funds from Operation.
08. Illustrate three examples of financial transactions that would be omitted if a Working Capital
concept of Fund is used as the basis for preparing a statement of changes in financial
position.
Management Accounting: 340
09. In analysing the funds flow, business transactions may be classified into three categories, viz.,
(a) transactions which affect only Current Assets or Current Liabilities, (b) transactions which
affect both Current and Non-current Accounts, and (c) transactions which affect only the
Non-current Accounts. Explain the effects of each category on Working Capital. Use
appropriate examples to support your answer.
10. Analyse the managerial uses of Fund Flow Analysis. [Bangalore Un;' B.ComJ
11. "Fund Flow Statement is a better substitute for an 'Income Statement' . Comment.
[Bangalore Un;' B.Com, April2004J
12. What is 'Fund'? What are the different meanings of the term Fund in the preparation of
statement of changes in financial position? Which meaning of Fund is most appropriate for
long-run financial planning and why?
13. What do you mean by the Fund Flow Statement? What are its managerial uses?
14. What are the uses of Funds Flow Statement?
15. Define Fund and Funds Flow Statement. What type o~ transactions will not be reflected in the
Funds Flow Statement?
16. How would you treat the following for Funds Flow Statement?
a. Payment of Dividend d. Declared Dividend
b. Proposed Dividend e. Unclaimed Dividend
c. Interim Dividend f. Unpaid Dividend
17. How do you treat each of the following items in the Funds Flow Statement?
a. Depreciation on Building
b. Purchase of Building by the issue of Shares
c. Purchase of Machine on cash basis
d. Purchase of Machine on three-month credit basis
e. Sale of Land
f. Conversion of Long-term Loan into Debentures
18. A few Short-Answer Questions:
a. How do you treat 'Provision for Taxation' while preparing Fund Flow Statement?
[Bangalore Uni., B.Com., May 2000J
b. Do all transactions cause flow of funds? Give two exemptions.
[Bangalore Uni, B.Com, November 2001J
c. What is 'Source from Operation'? [Bangalore Un;' B.Com, May 2001J
Funds Flow Statement: 341
• d. What is a 'Schedule of Changes in Working Capital'?
[Bangalore Uni, B.Com, May 2001]
e. How do you treat the 'Proposed Dividends' while preparing a Fund Flow Statement?
[Bangalore Uni, B.Com, November 2002]
f. Mention any two uses of Fund Flow Analysis. [Bangalore Uni, B.Com, May 2003]
g. Define a Fund Flow Statement. [Bangalore Uni, B.Com, November 2003]
h. What are the non-operating incomes and expenses?
[Bangalore Uni, B.Com, November 2003]
i. State any two limitations of Fund Flow Statement.
[Bangalore Uni., B.Com., April 2004]
19. From the figures given below, prepare a statement showing application and source of funds
for the year 1974.

Liabilities and 31st Dec. 31st Dec. 31st Dec. 31st Dec.
Assets
Capital 1973 (Rs.) 1974 (Rs.) 1973 (Rs.) 1974 (Rs.)
Share capital (equity) 3,00,000 3,50,000 Fixed assets (net) 5,10,000 6,20,000
Share capital Investments 30,000 80,000
(8% preference) 2,00,000 1,00,000
Current assets 2,40,000 3,75,000
Debentures 1,00,000 2,00,000
Discount on 10,000 5,000
Reserves and surplus 1,10,000 2,70,000 debentures
Current liabilities 80,000 1,60,000
7,90,000 10,80,000 7,90,000 10,80,000

You are informed that during the year: (a) a machine with a book value of Rs. 40,000' was sold for
Rs. 2,500; (b) Preference share redemption was done at a premium of 15% on 31st December,
1974; (c) Dividend at 15% was paid on equity shares for 1973; and (d) Depreciation charged
during the year Rs. 60,000. [M.Com., U.D.M., June 1983]

20. The balance sheets ofT Ltd., as on 31st December for years 1 and 2 were as follows.

Rs. in lakhs Rs. in lakhs


Liabilities Assets
Year 1 Year 2 Year 1 Year 2
Share Capital 300.00 300.00 Freehold Property (at cost) 225.00 240.00
Reserves 225.00 240.00 Plant and Machinery (at cost
less depreciation) 135.00 165.00
Debentures (unsecured) 75.00 75.00
Investments in shares of
Mortgage on Freehold
Companies under the same
Property 27.00 14.25
management (unquoted) 150.00 150.00
Management Accounting : 342

Creditors 45.00 45.00 Investment in shares of other


companies (market value:
Proposed Dividend (subject
year-2: 120 lakh; year-
to deduction of Tax) 22.50 23.25
1:150Iakh) 112.50 112.50
Provision for Taxation 21.00 37.50
Stock 52.50 75.00
Secured overdraft (by a . Debtors 45.00 75.00
floating charges on Assets) 15.00 82.50
Bank 10.50 -
730.50 817.50
730.50 !H7.50

The following additional information for the year 2 is relevant (amount in lakhs of rupees)

(a) Credit sales 675.00 (d) Depreciation on plant and machinery 17.50
(b) Credit purchases 520.00 (e) Dividend for year 1 paid in full
(c) Overheads 83.75 (f) Amount paid towards taxation for the year-I: 21.50

In view of credit squeeze, the company has been asked by the Bank to reduce the overdraft
substantially within six months, if possible by 50%.
Prepare a cash flow statement. Briefly comment on the financial position of the company and
suggest remedial measures to overcome the financial crisis, if any. [CA., May 1997J

21. From the information as contained in the statement of income and the Balance Sheet of
G.C. Ltd., you are required to prepare Funds Flow Statement, and to describe the
significant developments revealed by the statement. Your are also required to prepare a
statement of worIqng capital showing increase and decrease in each component thereof.
(a) Statement of Income and Reconciliation of Earnings for 1988
Particulars Rs. Rs.
Net Sales 25,20,000
Less: Cost of sales 19,80,000
Depreciation 60,000
Salaries and Wages 2,40,000
Operating Expenses 80,000
Provision for Taxation 88,000 24,48,000
Net Operating Profit 72,000
Non-recurring Income:
Profit on sale of an item of equipment 12,000
Funds Flow Statement : 343

84,000
Retained earning (Balance in P & L Alc brought forward) 1,51,800
2,35,800
Dividend declared and paid during the year 72,000
P & L Ale balance on 31-12-1988 1,63,800

(b) C omparatIve Baance


I Sheet
As at As at
Particulars
31-12-87 31-12-88
Fixed Assets:
Land Rs.48,000 Rs.96,000
Building and Equipment 3,60,000 5,76,000
Current Assets:
Cash 60,000 72,000
Debtors 1,68,000 1,86,000
Stock 2,64,000 96,000
Advances 7,800 9,000
9,07,800 10,35,000
Capital 3,60,000 4,44,000
Surplus in Profit and Loss Ale 1,51,800 1,63,800
Sundry Creditors 2,40,000 2,34,000
Outstanding Expenses 24,000 48,000
Income Tax Payable . 12,000 13,200
Accumulated Depreciation on Building and Equipment 1,20,000 1,32,000
9,07,800 10,35,000
Cost of equipment sold was Rs. 72,000.

22. The following are the summarised balance sheets of Ashok Ltd., as on 31st December for
years 1 and 2.

Liabilities: Year 1 Year 2


Equity share capital of Rs. 10 each RsA,OO,OOO Rs.4,80,000
Share premium Alc - 20,000
Management Accounting : 344

General reserves 60,000 1,00,000


Profit and Loss Alc 96,000 1,36,000
Debentures 1,00,000 -
Creditors 2.60,000 2,80,000
Proposed dividend 40,000 48,000
Total 9,56,000 10,64,000
Assets:
Freehold land and building 2,10,000 2,80,000
Plant & Machinery (Cost) 5,80,000 6,40,000
Less: Depreciation 2,80,000 3,00,000 3,00,000 3,40,000
Equipment:
Cost 18,000 20,000
Less: Depreciation 12,000 8,000 6,000 12,000
Inventories 2,60,000 2,10,000
Debtors 1,50,000 1,70,000
Cash 30,000 52,000
Total 9,56,000 10,64,000

Note:
(a) The plant and machinery, which cost Rs. 40,000 and in respect of which Rs. 26,000 had
been written off as depreciation, was sold during the year 2 for Rs. 6,000.
(b) Equipment which cost Rs. 10,000, and in respect of which Rs. 8,000 had been written off
as depreciation, was sold for Rs. 4,000 during year 2.
(c) The dividend which was declared in year 1 was paid during year 2.
1. Prepare a statement showing the change in working capital during year 2, and
2. Prepare a statement showing the sources and ~pplication of working capital (Funds
Flow Statement) during year 2. lCA (Fin), November 1979J
23. The summarized Balance Sheets of a company which has a good record of expansion are as
under (Rs. in lakhs).
Funds Flow Statement: 345-

1979 1978
Assets:
Land 33.60 36.00
Building 194.40 134.40
Machinery 79.20 48.00
Tools 9.60 16.80
Investments in Associated Company 4.32 3.60
Stock 50.40 52.32
Prepaid expenses 0.29 0.34
Debtors 48.24 28.56
Cash and Bank Balance 2.32 2.16
Discount on Debentures 0.50 0.60
Goodwill 48.00
422.87 370.78
Liabilities:
Share capital 168.00 96.00
Reserves 140.48 188.12
Provision for Depreciation 65.08 43.44
Debentures 36.00 24.00
Bank overdraft 2.28 1.63
Trade creditors 6.27 14.88
Interest accrued but not
, due 2.40 1.44
Provision for doubtful debts 1.08 0.55
Provision for Taxation 1.28 0.72
422.87 370.78

Summarised P and L Account for the year ended 31-12-1979 is as follows (Rs. lakh).

Sales 3f1.00
Manufacturing and othet expenses 164.96
Selling and Distribution expenses 70.80
Management Accounting : 346

General Expenses 37.70


Interest on Debentures 1.80
Depreciation on Building, Machinery and Tools 29.64 304.90
Net profit before tax 6.10
Provision for taxation 2.00
Net profit after tax 4.10

The chairman of the company is unable', to determine the reasons for the changes in working
capital. You are requested to assist him and prepare a statement of sources and applications
of funds for the year 1979 together with supporting schedules. On a scrutiny of records, the
following information relating to 1979 accounts is found relevant for the purposes of your
work.

a. Machinery costing Rs. 1.08 lakh was scrapped and written off. Accumulated
depreciation on this machinery was Rs. 80,000.
b. There was an issue of capital during August 1979 at par.
c. An interim dividend at 3.5% was paid in June 1979.
d. There were no purchases or sale of tools during 1979.
e. A small piece of land was sold for Rs. 2.40 lakh. , lCA (Fin), May 1980J

24. The directors of Chintamani Ltd., are alarmed at the deterioration of the financial
operation of the company. They find that the bank, overdraft is already at the limit
allowed by the bank, and they have not sufficient funds to pay their creditors on the due
dates. Not being trained in accounting and financial management, they are at a loss to
understand why. When the trade was bright, their audited accounts revealed satisfactory
profits, additional capital has also been introduced since and borrowings made. Why,
then, should there be shortage of funds?
,They present you with the balance sheets as on 31st March for two years, and ask you to
prepare, a statement which will show them what has happened to the money which has come
into the business during the year.
Previous Year Current Year
Authorised Capital, 15,000 shares of Rs. 100 each Rs. 15,00,000 Rs. 15,00,000
Paid-up share capital 13,00,000 14,00,000
General reserves 60,000 40,000
Profit and Loss Appropriation Nc 36,000 38,000
Loan on mortgage 5,60,000
Bank Overdraft 69,260 1,29,780
Bills payable, 40,000 38,000
Funds Flow Statement: 347
Sundry trade creditors 76,000 1,12,000
Proposed final dividend 78,000 72,000
16,59,260 23,89,780
Goodwill 2,40,000 2,20,000
Freehold building 8,00,000 11,76,000
Machinery and plant 1,44,000 3,94,000
Furniture 6,000 5,500
Shares in other companies 80,000 2,34,000
Stock 2,44,000 2,38,000
Sundry debtors 1,25,600 1,04,400
Cash 1,560 1,280
Bills receivable 7,600 6,400
Prepaid expenses 4,500 6,200
Preliminary expenses 6!000 4,000
16,59,260 23,89,780

You are given the following additional information.


(a) Depreciation has been charged on freehold building at 2.5% p.a on the cost of Rs.
10,00,000; on machinery and plant at 8% p.a on the cost of Rs. 4,00,000; and on furniture
at 5% p.a on the cost of Rs. 10,000. No depreciation has been written off on newly
acquired buildings, and plant and machinery.
(b) Shares in other companies were purchased for Rs. 1,60,000 cash, and dividends
amounting to Rs. 6,000 declared out of profits made prior to purchase have been received
and used to write down the investment (shares).
(c) The proposed dividend for the previous year was duly paid and, in addition, an int.erim
dividend of Rs. 52,000 was paid in the current year. lCA (Fin), November 1965J
25. The Balance Sheets of AB and Co., Ltd., at the end of 1968 and 1969 are given below.
Assets: 1968 1969 (Rs.)
Freehold land Rs. 1,00,000 Rs. 1,00,000
. Plant at cost 1,04,000 1,00,000
Furniture at cost 7,000 9,000
Investments at cost 60,000 80,000
Management Accounting : 348
Debtors 30,000 70,000
Stock 60,000 65,000
Cash 30,000 45,000
3,91,000 4,69,000
Capital and Liabilities:
Share Capital 1,00,000 1,50,000
Share Premium 5,000
General Reserve 50,000 60,000
Profit and Loss Account 10,000 17,000
6% Debentures 70,000 50,000
Provision for Depreciation of plant 50,000 56,000
Provision for depreciation of furniture 5,000 6,000
Provision for taxation 20,000 30,000
Sundry Creditors 86,000 95,000
3,91,000 4,69,000

A plant purchased for Rs. 4,000 (Depreciation Rs. 2,000) was sold for cash Rs. 800 on 30th
September, 1969. On 30th June, 1969 an item of furniture was purchased for Rs. 2,000.
These were the only transactions concerning fixed assets during 1969. Depreciation on plant
was provided at 8% on cost (the sold out item is not taken into consideration) and on furniture
at 12.5% on average cost. A dividend of 22.5% on original shares was paid. You are
required to prepare a statement showing sources and destination of funds during 1969.
leA (Fin), November 1970J
26. A and B are carrying on business in partnership since last two years sharing profit and loss in
the ratio of 3 : 2. Not all the necessary books and records have been maintained. However,
the following flgures have been ascertained from the available records:

YearEnd 1 YearEnd 2
(a) Drawings for the year: A Rs.5,OOO Rs.15,000
B 10,000 10,000
(b) Sundry creditors 24,000 27,000
(c) ·Stock 30,000 50,000
(d) Sundry debtors 28,000 48,000
(e) Profit for the year 15,000 28,425
Funds Flow Statement: 349
(t) Purchase of machineries 25,000 15,000
(g) Purchase of furniture 10,000
(h) Bills payable 12,000 7,000
(i) Bills receivable 8.000 6,000
U) Sale of furniture 2,000
(k) Bank loan obtained 50,000
(1) Bank loan discharged 10,000
(m) Salaries outstanding 2,000
(n) Prepaid insurance 500

Cash in hand and in bank amounts to Rs. 7,000 on 31st December, year 2. Partners have been
regularly charging depreciation on machineries @ 10%, and on furniture @ 5%, excluding
sold items. In year I, A and B introduced further capital of Rs. 10,000 and Rs. 15,000
respectively (other than their initial capital). On 31 st December, year I, the assets of business
included cash and bank balances. The book value of furniture sold was Rs. 3,000. Prepare a
statement showing sources and application of funds for the year 2 ended December 31. Show
your working properly. lCA (Fin), May 1981J
27. The summarised Balance Sheets of Alpha Ltd., as on 31st March 1973, 1974 and 1975 are
given below (Rs. in lakhs).
As on 31 st March
1973 1974 1975
Liabilities:
Paid up capital 194 194 194
Borrowings (Long-term): 1. From banks 68 97 124
2. From others 281 343 379
Current liabilities 52 54 99
Total 595 688 796
Assets:
Gross Block 355 356 361
Less: Depreciation 69 95 122
Net Block 286 261 239
Current Assets 143 199 234
Profit and Loss Account 166 228 323
595 688 796
Management Accounting: 350

Prepare a statement of Net Sources and Uses of Funds for the years ended on 31st March,
1974 and 1975 and give your comments on the same. rCA (Fin), May 1976J

28. The following is the balance sheet of a concern as on 30th June, Year 1.
Capital Rs.12,00,000 Fixed assets (at cost less depreciation) Rs. 5,00,000
Trade creditors 2,50,000 Stock 4,50,000
P&LNc 80,000 Debtors 2,50,000
Cash and Bank balance 3,30,000
15,30,000 15,30,000

The management makes the following estimates for the year ending 30th June, Year 2:

Up to 31st May, Year 2 June, Year,2


Purchases Rs. 15.20,000 Rs. 1,05,000
Sales 22,40,000 2,50,000

Additional information:

(a) It has been decided to invest Rs. 1,50,000 in purchase of fixed assets which are
depreciated @ 10%.
(b) The time-lag for payment of creditors and receipt from debtors is one month.
(c) The business earns a gross profit of 33 113% on turnover.
(d) Sundry expenses against profit amount to 12% of the turnover (excluding depreciation on
fixed assets).
You are required to prepare a projected Funds Flow Statement for the year 2 ending 30th
June. rCA (Fin), November 1974J
29. The following is the Balance Sheet of Spbinixs Limited (Rs. lakh).

As on As on As on As on
Liabilities Assets
30.6.90 30.6.91 30.6.90 30.6.91
Share capital:
10.00 Equity Shares of Rs. 100 each 20.00 13.00 Plant 18.00
7.50 Preference Shares of Rs. 100 2.50 8.00 'Stock 9.50
0.50 Share Premium 0.25 15.00 Debtors 14.50
0 Capital Redemption Reserve 5.00 3.00 Bank balance 2.50
Miscellaneous
Funds Flow Statement: 351

8.00 General Reserve 4.50 1.00 Expenditure 1.00


3.00 Profit & Loss Alc 5.00
5.00 Provision for Taxation 6.00
6.00 Current Liabilities 2.25
40.00 45.50 40.00 45.50

The following further information is furnished.


a. The company declared a dividend of 20% for the year ended 30.6.90 to equity
shareholders on 30.6.90. Dividend on preference share capital for the year ended
30.6.1990 was paid on 30.6.1990.
b. The company issued notice to preference shareholders holding preference shares of the
face value of Rs. 5 lakhs for redemption at a premium of 5% on 1.12.90 and the entire
proceedings were completed before 31.12.1990 in accordance with the Law.
c. The company provided depreciation at 10% on closing balance of plant. During the year,
one plant, whose book value was Rs. 2,00,000 was sold at a loss of Rs. 30,000.
d. Miscellaneous expenditure incurred during the year ended 30.6.1991 Rs. 25,000 for share
and other expenses.
e. A sum of Rs. 4 lakhs has been provided for taxation for the year.
Prepare a statement of sources and application of funds for the year ended 30.6.1991.
leA (Fin), November 1992J
30. The following are the summarized trial balance of Honesty Ltd., as at 31 st December 1971
and 31 st December 1972.

Particulars 31st December 1971 (Rs.) 31st December 1972 (Rs.)


Dr. Cr. Dr. Cr.
Isslled Share Capital 8,00,000 8,00,000
Capital Reserve 2,00,000
8% Debentures 2,50,000
Discount on Issue of Debentures 5,000
Land and Buildings 5,00,000 4,00,000
Plant and Machinery 10,00,000 12,50,000
Provision for depreciation of fixed
assets 4,00,000 5,00,000
Profit and Loss Alc balance on 1st
Management Accounting: 352

January
Net profit for the year
I 2,30,000
3,00,000 4,50,000
3,00,000
Dividend for the previous year 80,000
80,000
Provision for doubtful debts 20,000 35,000
Trade Investments (at cost) 1,00,000
3,00,000
Current assets 7,00,000
8,50,000
Current liabilities 3,50,000 2,50,000
Bank overdraft 2,80,000 1,00,000
23,80,000 23,80,000 28,85,000 28,85,000

The following additional information is available.


a. During the year, Land and Building having an original cost of Rs. 1,00,000 and written
down value of Rs. 75,000 have been sold for Rs. 3,00,000. The capital profit has been
transferred to Capital Reserve and the profit equivalent to the depreciation written off. in
the past has been included in the profit for the year.
b. On 1st January 1972, the company issued debentures of a face value of Rs. 2,50,000 at a
discount of 5%. Part of the discount has been written off out of profits.
Prepare a statement of sources and applications of funds during .the year.
lCA (Fin), May 1973J
31. Light and Dark are equal partners since the beginning of year 1. Their books and records
showed the following balances as on 31 st December, Year 1.

Building account Rs.50,000


Due from customers 1,20,000
Bills receivable account 15,000
Advance for machinery (machinery delivered and installed in early July, year 2) 20,000
Due to suppliers (including Rs. 2,000 for purchase of furniture in November, year 2) 90,000
Bank overdraft 15,000
Stock in trade at cost 1,00,000
Bills payable account 16,000
Machinery account 75,000
Furniture and fixtures 24,000
Funds Flow Statement: 353
A fire occurred towards the end of December, year 2, which destroyed a portion of the
building (book value Rs. 15,000) used as godown. Some of the books and records were
saved, from which the following information was gathered.
(a) The cost of machinery delivered and installed in July, year 2 was Rs. 50,000 (the balance
amount was paid in October).
(b) Sales for the year 2 were approximately Rs. 80,000 per month, of which 10% were cash sales.
The firm maintained a steady gross profit rate of 30% on turnover; cash purchases amounted
to Rs. 30,000.
(c) Collection from debtors Rs. 8,40,000
P~yment to creditors (including the payment of liability for furniture) 5,20,000
Discount allowed to debtors 5,000
Discount received from creditors 3,000
(d) Re: Bills receivable: Drawn and accepted 30,000
BIR collected 15,000
Dishonoured 3,000
Endorsed to creditors 4,000
Endorsed BIR dishonoured 2,000
(e) Re: Bills payable: Accepted 20,000
Discharged 15,000
(f) Partners' drawings: Light 18,000
Dark 15,000
(g) Stock in trade as on 31st December, year 2 was valued at Rs. 80,000.
(h) Cash and bank balances as on 31st December, Year 2 amounted to Rs. 1,94,000, and there was
no bank overdraft as on that date. The figures of cash and bank balance on January I, year 2
were not readily available.
(i) Net profits for the year may be assumed at Rs. 80,000 after providing for depreciation on
machinery: Rs. 7,500 and furniture: Rs. 2,500.
Prepare a statement showing sources and application of Funds for the year ended December
31, Year 2 showing separately statement of changes in working capital.
lCA (Fin), November 1981]
32. Robinson Crusoe Ltd., present the following financial statements for 1990 and 1991.
Prepare a sources and applications of funds statement and evaluate your findings.
Management Accounting : 354
Assets:
Cash Rs. 1,06,000 Rs.62,OOO
Investments 1,74,000
Sundry Debtors 6,92,000 10,56,000
Stock-in-trade 8,64,000 13,66,000
Net Fixed Assets 22,26,000 27,96,000
40,62,000 52,80,000
Liabilities:
Sundry Creditors 8,26,000 12,54,000
Bill payable 4,52,000 6,28,000
Loan from bank 2,00,000 4,70,000
Reserves and Surplus 13,84,000 17,28,000
.
Share Capital 12,00,000 12,00,000
40,62,000 52,80,000
Depreciation ofRs. 3,78,000 was written off for 1991 on fixed assets.
leA (Fin), May 1992]

33. A, B and C were partners in a publishing house. During 1969, B withdrew from the
business. The balance sheet as on 31 st December, 1968 and 31 st December, 1969 and
some other particulars are gi ven below.
As on 31st December
1969 (Rs.) 1968 (Rs.)
Assets:
Cash 8,500 3,300
Sundry Debtors 12,800 8,900
Cash surrender value of life insurance 4,050 5,700
Prepaid expenses 950 400
Goodwill 3,900
Land 4,200 4,200
Building and Equipment 40,000 30,000
74,400 52,500
Funl;Js Flow Statement: 355
Liabilities:
Reserve for doubtful debts. 930 1,130
Reserve for depreciation 12,800 13,370
Sundry creditors 1,650 2,950
Amount due to B including interest 10,250
Bank Loan 3,300
Hire-purchase instalment due for equipment 7,800
Accrued Expenses 1,690 2,480
A's Capital 16,660 9,800
B's Capital 10,600
C's Capital 19,320 12,170
74,400 52,500

Additional information is as follows.


a) A, B and C shared profits and losses equally. On July 1, 1969, B resigned and made an
agreement according to which he would accept Rs. 2,000 in cash as initial instalment and
leave Rs. 10,000 with the firm to be repayable in five equal instalments of Rs. 2,000 on
July 1st of each year with interest at the rate of 5%, the repayment of first instalment to
commence with effect from 1st July, 1969. At the time of his withdrawal, B's drawings
were Rs.100 less than his Rs. 5,700 share of the income up to June 30, 1969. It was
agreed that Rs.3,900 of goodwill would be recognised in the accounts prior to B's
resignation and that A and C would share income equally in the new partnership. The
total withdraw Is of A and C for the year were Rs. 14,640 and Rs. 14,350 respectively.
b) After B's resignation, the insurance policy on his life was cancelled and cash value of
Rs.3,000 was received by the partnership firm.
c) The firm purchased equipment on 1st November, 1969 on hire-purchase agreement
paying cash Rs. 6,100 and incurring an instalment obligation for payment of Rs. 300 per
month (plus interest) for 30 months. In the later half of 1969, equipment which cost Rs.
4,500 was sold for Rs. 1,200 and a gain of Rs. 680 was included in income for the second
half of the year.
You are required to prepare a statement of change in working capital and a statement of
sources and applications of funds for the period 1st January, 1969 to 31st December, 1969.
For the purpose of the preparation of the statement, the business carried on by A, B and C for
the period ending 30th June and by A and C for the period ending 31st December, 1969
should be treated as a running one. leA (Fin), November 1971J
l
Management Accountin : 356

34. The following are the balance sheets of Bharat Indu~ries as on 31 st December, years 1 and 2.

Year 1 (Rs.) Year 2 (Rs.)


Share capital (face value Rs. 100 per share) 1,00,000 92,500
Reserve and surplus 80,231 80,753
Reserve for contingencies 37,500 63,600
Allowance for inventory loss 2,000 8,500
Accumulated depreciation 96,618 81,633
Allowances for loss on Sundry Debtors Nc 4,630 3,815
14% Mortgage debentures 82,000 68,500
Accrued interest, taxes, etc 12,307 21,263
Sundry creditors 79,081 81,314
4,94,367 5,01,878
Building and machinery 1,72.778 2,07,782
Investments 1,17,500 90,000
Stock 83,164 95,438
Debtors 67,186 65,638
Advances 9,025 9,816
Cash and bank balances 40,409 30,337
Unamortised debenture discount 4,305 2,867
4,94,367 5,01,878

The following information concerning the transactions is available.


(a) Net profit for year 2 was shown by profit and loss statement as Rs. 48,097.
(b) The share capital account as on 31st December, year 1 included 15% redeemable
preference shares. These were redeemed during the year at Rs. III per share.
Subsequent to the redemption, a 10% cash dividend was paid.
(c) New machinery was purchased for Rs. 31,365, and machinery costing Rs. 32,625 was
scrapped. The scrapped machinery had accumulated depreciation of Rs. 29,105 on the
date of scrapping. The scrap was sold for Rs. 1,000, which was credited to the Profit and
Loss Account on scrapping of the assets. The remaining increase in fixed assets resulted
from the construction of the building.
(d) The debentures mature at the rate of Rs. 5,000 per year. In addition to the redemption of
Rs. 5,000 debentures due in year 2, the company purchased and redeemed Rs. 8,500 of
Funds Flow Statement: 357

the debentures at Rs. 103. Both the premium on redemption and the applicable discount
were charged as expenses.
(e) The allowance for inventory loss was created by a charge to expense in each year. It was
set up to reduce the inventory values of obsolete items to estimated market value.
(0 A sum of Rs. 11,400 was debited to reserve for contingencies account during the year. It
represented the amount of income-tax liability of an earlier year in dispute.
You are required to prepare a statement showing the sources and application of funds for the
year 2. ICA (Fin), May 1961J
35. The summarised Balance Sheets of Mis Shanti Products Ltd., for the years ended 31-3-1995
and 31-3-1996 are given below.

(Rs.OOO) (Rs.OOO)
Capital and Liabilities Assets
1995 1996 1995 1996
Share Capital 500 500 Land and Buildings 180 200
General Reserve 200 220 Plant and Machinery 210 276
Profit and Loss Account , 40 32 Other Fixed Assets 30 45
Bank Loan (long term) 100 Investments 50 50
Creditors 158 172 Stock 200 190
Provision for Taxation 45 30 Debtors 170 195
Cash 103 98
Total 943 1,054 Total 943 1,054

Prepare a statement of sources and application of fund given the following additional information
relating to the year ended 31-3-1996.
(a) Dividend amounting to Rs 30,000 was paid during the year.
(b) Provision for taxation made Rs. 12,000.
(c) Machinery worth Rs. 15,000 (book value) was sold at a cost of Rs. 3,000.
(d) Investment costing Rs. 10.000 was sold for Rs. 12,000.
(e) Depreciation provided on assets: Land and Buildings Rs. 5,000
Plant and Machinery Rs. 20,000
IICWA (Fin), June 1997]
Management Accounting : 358
36. Globe Traders Ltd., has furnished its Balance Sheet as at 30-6-1994 and 30-6-1995 as per the
details given below:

(in lakh of Rs.) (in lakh of Rs.)


Capital and Liabilities 30-6- 30-6- Assets 30-6- 30-6-
1994 1995 1994 1995
Equity Capital 200 250 Land 10 35
General Reserves 30 40 Buildings (net after Dep.) 15 17
Revaluation Reserve - 25 Machinery (net after Dep.) 140 220
15% Debentures - 50 Furniture (net after Dep.) 20 23
Dividends 10 12 Stock 15 20
Current Liabilities 10 13 Debtors 30 40
Bank and cash 20 35
Total 250 390 Total 250 390

The following additional particulars are also given:


(a) During the year ended 30-6-1995, land was revalued.
(b) The dividend for the year ended 30-6-94 was paid during the next year.
(c) An old machine (original cost Rs. 10 lakhs, written down value Rs. 2 lakhs) was sold for
Rs. 3 lakhs; and '
(d) Depreciation on assets for 1994-95 was adjusted as follows: Buildings Rs. 60,000
Machinery Rs.12,00,000
Furniture Rs. 2,00,000
Prepare the Funds Flow Statement of the company for the year ended 30-6-1995.
[ICWA (Fin), December 1995J
37. Mr. A is contemplating to set up a new company to manufacture automotive components and
has engaged you as consultant. The total cost of the project would be Rs. 270 lakh to be
financed as under.
Financing pattern:
Equity Share Capital Rs. 100lakh
Term loan: From Financial institution @ 13% rate of Interest 100
From Bank @ 14% rate of Interest 70
270
Funds Flow Statement: 359

There will be further borrowing from Bank for working capital attracting interest @ 20% as
under:
1st year: average outstanding loan throughout Rs. 50 lakh.
2nd year: average outstanding loan throughout the year, average 60 lakh.
3rd year onwards: average outstanding loan throughout the year Rs. 72 lakh.
The sales and costs are projected as under:
a) Sale valUe of production (SVP) Rs. 200 lakhs in the 1st year, thereafter to increase by
20% in value every year for next two years, and to stabilise at Rs. 320 lakh from 4th year
onwards till 7th year.
b) Raw materials and consumables 50% of SVP.
c) Other variables like Power and Fuel, Freight, Selling Overhead, 10% of SVP.
d) Salaries, Wages and other employee-related Costs Rs. 15 lakhs in 1st year and thereafter
to increase @ 10% every year till the 7th year.
e) Other Administrative Expenses Rs. 10 lakhs in 1st year and thereafter to increase @ 5%
every year till the 7th year.
The term loans are repayable as under:
a. Loan from Financial Institution:
(a) No repayment in first two years.
(b) Thereafter @ Rs. 20 lakh per annum for next 5 years.
b. Loan from Bank:
(a) No repayment in first three years.
(b) Thereafter 4th year: Rs. 10 lakhs, 5th year onwards: Rs. 20 lakhs per annum.
The loans are to be repaid in one instalment at the end of the year when due. The net Current
Assets will increase by Rs. 20 lakhs in 2nd year over 1st year and will continue to increase in
the same rate during the next two years. The company is required to incur capital expenditure
of Rs. 10 lakh every year from 2nd year onwards. You have been asked to make financial
projection for 7 years showing Profitability and Cash Flow showing utilisation of internally
generated Fund only. You are also required to give comments about the results. Effect of
Depreciation and Income Tax to be ignored. You may make assumption as considered
relevant. [ICWA (Fin), December 1992J
38. The latest and summarised financial statements of Indogrowth Ltd., are given below.
Balance Sheet as at 31st March, 1991
. Rs. in thousands
Sources of Funds:
Share Capital 8,000
Reserve and Surplus 30,000
15% Term Loans 30,000
Management Accounting: 360
68,000
Applications of Funds:
Fixed Assets: Gross Block 40,500
Less: Depreciation 13,500 27,000
Current Assets:
Inventories 24,000
Sundry Debtors 30,000
Cash at Bank 800
Other Current Assets 1,800
56,600
Less: Current Liabilities:
Sundry Creditors 11,400
Provision for Tax 2,600
Proposed Dividends 1,600
15,600
Net Current Assets 41,000
68,000

Income Statement for the year ended 31st March 1991 (in Rs. thousands)

Sales 1,44,000
Materials consumed 57,600
Labour 52,400 1,10,000
Gross Profit 34,000
Operating expenses 8,250
Depreciation 4,050
Interest 4,500
Provision for Tax 8,600 25,400
Net profit in 1991-92 8,600

a) Sales in terms of quantity is targeted to increase by 30%.


b) Additions to fixed assets at the beginning of the year will be Rs. 2,500 thousands.
Funds Flow Statement: 361

c) 115 of term loans will be repaid at the beginning of the year.


d) Raw materials prices and wage rates are expected to increase by 10%. No increase in
selling price is contemplated. Operating expenses are expected to be maintained in tenns
of percentage of turnover.
e) 10% of the year's provision for tax will be outstanding at the end of the year.
t) Depreciation will be 10% of the gross block.
g) Sundry Creditors will increase by 20%. No change in their Current Assets are anticipated.
h) Previous year's dividend will be paid; current year's provision will be 25%.
Assuming cash credit accommodations available as required (interest on which may be
ignored) and after calculating appropriate ratios in respect of inventory holding and sundry
debtors: prepare.
(a) Projected income statement for the year 1991-92; and
(b) Projected Balance Sheet as at 31-3-1992 leA (Fin), May 1991J

39. The Balance Sheet of TJ Ltd., as at 31 st March, 1994 stands summarised as below. (Rs.
in thousands)

Sources of Funds: Uses of Funds:


Capital: Fixed Assets 1,500
Equity Shares of Rs. 10 each 800 Investments (9% Tax-Free Bonds) 500
12% Preference Shares of Rs. 10 500 Net Current Assets:
Reserves 1,000 Cash at Bank 500
Loans: Other Current Assets .3,500
15% Convertible Debentures of Rs. 4,000
100 each 1,200
Less: Current Liabilities 2,000 2,000,
14 % Fixed Deposits 500
4,000 4,000 !

For preparing the projected financial statements for 1994-95. the following information is
available:
a) Profit from operations, before considering depreciation, interest and income from
investments. will be Rs. 10 lakhs.
b) , Depreciation may be assumed at 10%.
Management Accounting : 362
c) The convertible portion of Rs. 50 of each debenture, is due for conversion into equity
shares at a price of Rs. 25 per share, on 1-4-1994. Interest is payable on debentures
annually on 31st March.
d) Income-tax will be paid at 50% of taxable income after an estimated provision for
disallowances of Rs. 10,000.
e) No change is expected in other current assets and current liabilities.
f) Besides preference dividend, interim dividend on equity at 20% will be paid during the
year.
You are required to prepare the following projected statements of the company for the year
1994-95.
(a) Balance Sheet as at the end of the year; and
(b) Statement of changes in financial position, supported by necessary workings.
lCA (Fin), May 1990]
Answers

19. Increase in Working Capital = Rs. 55,000; Funds from Operation = Rs. 3,22,500; Loss on
Sale of Fixed Assets =Rs. 37,500; Total of sources =Rs. 4,75,000 =Total of applications.
20. Cash from Operation = Rs. 87.25 lakh; Net decrease in Cash = Rs. 10.5 lakh; (Solve this
problem after going through the theoretical aspects of Cash Flow Statement chapter).
21. Decrease in Working Capital = Rs. Rs. 1.56,000; Total of Funds Flow Statement = Rs.
4,08,000.
22. Decrease in Working Capital =Rs. 28,000; Total of Funds Flow Statement =Rs. 3,22,000.
23. Increase in Working Capital = Rs. 23.78 lakh; Total of Funds Flow Statement = Rs. 120.14
lakh.
24. Cash from Operation = Rs. 3,06,720; Total of Cash Flow Statement = Rs. 9,73,000; Decrease
in Cash = Rs. 200 (Solve this problem after reading theoretical aspects of Cash Flow
Statement).
25. Increase in Working Capital = Rs. 41,000; Funds from Operation =Rs. 49,700; total of Funds
Flow Statement = Rs. 1.05.500.
26. Funds from Operation = Rs. 33,500; Increase in Working Capital = Rs. 35,500; Total of
Funds Flow Statement =Rs. 85,500.
27. Increase in Working Capital = Rs. 54 lakh (1974) and Decrease in Working Capital = Rs. 10
lakh (1975};'Funds depleted by Operation = Rs. 361akh (974) and Rs. 681akh (1975); Total
of Funds Flow Statement = Rs. 911akh (1974) and Rs. 731akh (1975).
28. Funds from Operation = Rs. 5,31,200; Increase in Working Capital = Rs. 3,81,200; Total of
Funds Flow Statement =Rs. 5,31,200.
29. Increase in Working Capital = Rs. 4,25,000; Funds from Operation = Rs. 12,55,000.
Funds Flow Statement: 363

30. Funds from Operation = Rs. 4,07,500; Increase in Working Capital = Rs. 4,15,000; Total of
Funds Flow Statement = Rs. 9.45.000.
31. Funds from Operation = Rs. 1,05,000; Increase in Working Capital = Rs. 40,000; Total of
Funds Flow Statement = Rs. 1,05,000.
32. Increase in Working Capital = Rs. 44,000; Total of Funds Flow Statement = Rs. 9,92,000.
33. Increase in Working Capital = Rs. 6,090: Cash from Operation = Rs. 48,830; Total of Funds
Flow Statement = Rs. 60,530.
34. Funds from Operation = Rs. 72,115; Decrease in Working Capital = Rs. 9,744; Total of Funds
Flow Statement = Rs. 1,10,359.
35. Funds from Operation = Rs. 80,000; Decrease in Working Capital = Rs. 4,000; Total of Funds
Flow Statement = Rs. 2,08,000.
36. Increase in Working Capital Rs. 27 lakh; and total of Fund Flow Statement Rs. 138.61akh.
37. Cash Profit from year 1 to 7 is Rs. 22.2 lakh, Rs. 34.2 lakh, Rs. 48.9 lakh, Rs. 62 lakh, Rs.
63.3 lakh, Rs. 66.1 lakh and Rs. 68.4 lakh respectively; and cash flow (internal generation and
its utilization) cumulative surplus Rs. 92.3 lakh.
38. Current Assets Rs. 75,120 (000); and Current Liabilities Rs. 34,682.5 (000).
39. Balance Sheet total Rs. 41,17,000; and Increase in Working Capital Rs. 2,67,000.
Chapter- V

CASH FLOW STATEMENT

Objectives: Objectives of this chapter are:


• To understand the meaning of Cash Flow Statement,
• To know the difference between the Funds Flow Statement and the Cash Flow Statement,
• To study the importance, utility and limitations of Cash Flow Statement,
• To know the procedure of preparing the Cash Flow Statement, and
• To analyse and solve some problems on the Cash Flow Statement.
Structure
• Introduction - Meaning of Cash Flow Statement
• Similarities and Differences between Funds Flow Statement and Cash Flow Statement
• Importance or Utility of Cash Flow Statement
• Sources and Applications of Cash
o Cash from Operation
Income and Expenditure Approach
Net Profit Approach
o Decrease in any Asset except Cash
o Increase in any Liability
o Increase in any Asset except Cash
o Decrease in any Liability
Preparation of Cash Flow Statement
• Illustrations
• Limitations of Cash Flow Statement
• Summary of the Chapter
• Key-Terms to Remember
• Questions for Self-study
Cash Flow Statement : 365
Introduction
When cash is used to denote fund, the analysis relates to the movement of cash.
Cash Flow, therefore, refers to the movement of cash in and out of an organization. When
the cash flows into the organization, it is called Cash Inflow or Positive Cash Flow. In the
same way, when cash flows out of the organization, it is called Cash Outflow or Negative
Cash Flows. The difference between cash inflows and cash outflows is called Net Cash
Flow.
Cash Flow Statement is prepared to project the manner in which the cash received has
been utilized during an accounting year. [t considers only the cash transactions. It intends to
depict the impact of various transactions on the cash position of an organization. It is a
statement which shows the sources of cash receipts and also the purposes for which payments
are made. It, therefore, explains the reasons for the changes in cash balance of an
organization between the two Balance Sheet dates. Thus, the Cash Flow Statement analyses
the changes in both the Current and the Non-current Accounts to determine the flow of cash.
Hence, there is no need for separate statement for the changes in Working Capital. Further, it
may be noted here that this Statement is prepared on the basis of both actual and notional cash
concepts.
An analysis of transactions from the view point of their impact on the cash balance
or position reveals two kinds of cash flows. They are (a) Actual Cash Flows, and (b)
Notional Cash Flows. Actual flow or movement of cash into or out of the business
organizations is referred to as actual cash flows. Purchase of a machine on cash basis results
in the actual outflow of cash. Similarly, sale of fixed assets, loans obtained from the financial
institutions, issue and redemption of financial instruments like debentures, redeemable
preference shares, etc., payment of cash dividend, taxes, etc., cause the actual flow or
movement of cash. On the other hand, the notional cash flow results in the indirect flow of
cash into or out of the business undertakings. Most of these notional cash flows arise due to
the transactions which cause the change in the Current Account Items viz., items of Current
Assets and Current Liabilities. For example, increase in the closing balance of 'Frade Debtors,
when compared to opening balance, is considered as causing outflow of cash indirectly
(notional). Because, increase in Trade Debtors is not due to payment of cash to the customers
(Debtors) but only due to sale of goods and services on credit basis. In this case, the increase
in Trade Debtors causes an indirect cash outflow (i.e., notional) as some amount of cash flows
out of the organization in the form of material cost, labour cost, overhead expenses, etc.,
which are incurred for the production and sale of goods sold to customers (i.e., Debtors) on
credit basis. In the same way, the changes in the amounts of other items of Current Accounts
(i.e., Current Assets and Current Liabilities) are analysed and treated. And both the actual
cash flows and notional cash flows are considered and taken into account for preparing the
Cash Flow Statement.
Cash Flow Statement and Funds Flow Statement - Similarities
Though there are a number of differences between the Funds Flow Statement and
the Cash Flow Statement. they possess two similarities. They are presented below.
Management Accounting: 366
. 1. Both the Statements are prepared on the basis of the figures in the Balance Sheets,
Profit ,and Loss Account of the current year and other supplements. That means, the
sources of information for both the Statements are same, and
2. Both the Statements aim at presenting the causes of changes in the financial position of
. an organization during a particular period.
Cash Flow Statement and Funds Flow Statement - Differences
In spite of the above similarities between the Cash Flow Statement and the Funds
How Statement, there are differences between the two and the important are presented herein
under.
1. The Cash Flow Statement is based on the narrower concept of Fund viz., Cash which is
an item of Current Assets and therefore, shows the causes for the changes in the cash
position;
The Funds Flow Statement is based on a broader concept of fund viz., Working
Capital and therefore, it records all the changes in the Current Assets and Current
Liabilities of an organization during the period of study.
2. Funds Flow Statement presents the changes in the Working Capital of an organization
between two Balance Sheet dates.
However, the Cash Flow Statement shows only the changes in the cash balance or
position of the organization between two Balance Sheet dates.
3. Cash Flow Statement highlights the changes or movements in the cash which is an item
of Current Assets. As is known, Current Assets influence the Working Capital which is
used. to denote 'Fund' in the Funds Flow Statement. He~ce, any improvement in the
cash position improves the Working Capital position. However, any improvement in
the Working Capital position need not necessarily improves the cash p~sition of the
organization.
4. As the name itself reflects, the Cash Flow Statement ignores the accrual concept. What
is important is the time of receiving cash and the time of effecting payment. So, only
cash receipts and payments are recorded;
The Funds Flow Statement is based on accrual concept and all the changes in the
Working Capital are recorded here.
5. Cash Flow Statement as a tool of analysing financial position is more useful for short-
term.analysis and cash planning;
The Funds Flow Statement is useful for planning intermediate and long-term financing.
6. In order to prepare the Funds Flow Statement, a separate statement called 'Schedule of
Changes in Working Capital' is to be prepared.
But it [Schedule of Changes in Working Capital] is not necessary for preparing the
Cash Flow Statement.
7. For the purpose of preparing Funds Flow Statement, classification of Balance Sheet
items into Current and Non-current items is necessary.
But this type of classification is not required for preparing the Cash Flow Statement.
Cash Flow Statement: 367

8. In the Funds Flow Statement, sources and application of funds are qtatched and
reconciled.
But in the Cash Flow Statement, it begins with the opening balance of cash and ends
with the closing balance of cash.
Utility or Importance or Uses or Advantages of Cash Flow Statement
Cash Flow Statement is an Important tool for analysing the different sources and
application of cash. Hence, it extends its helping hand to the management to review the
decisions pertaining to the mobilization and utilization of cash. The Cash Flow Statement can
also be utilized to project the future cash flows and based on this. appropriate decisions can be
taken about the required cash levels. In this background, the specific uses or lJtility or
advantages of Cash Flow Statement are presented below. These uses, etc., of Cash Flow
Statement also help to understand and appreciate the importance of Cash Flow Statement.
1. Cash Flow Statement enables the company to prepare the projected Cash Flow
Statement for the future period. This facilitates proper cash planning for the future by
the company and/or management.
2. As the Cash Flow Statement gives the information about the amount of cash generated
by the operations, it enables the management to formulate proper internal financial
policies including the possible repayment of loans, payment of dividend, etc.
3. Cash Flow Statement shows the reasons as to why the cash position is not in
commensurate with profit. In other words, it reveals the reasons for lower cash position
in spite of higher profit and vice-versa. Hence, the management is able to take
corrective steps or remedial measures.
4. Cash Flow Statement sheds light on the short-term solvency or liquidity of the firm.
Therefore, it also sheds light on the capability of the organization to discharge its
immediate and/or short-term obligations as and when they fall due for payment.
5. Cash Flow Statement also helps to evaluate the efficiency and effectiveness with which
the management managed the cash.
6. Cash Flow Statement supplements the work of Funds Flow Statement as cash (used as
Fund in Cash Flow Statement) is a part of Working Capital (used as the Fund in the
Funds Flow Statement).
Sources and Application of Cash
From the above analysis, it is clear that the cash inflow and the cash outflow are
explainep and presented in a statement called Statement of Changes in Financial Position
on Cash Basis which is popularly known as Cash Flow Statement. It shows:
1. All sources from which cash moves or flows into the organization, and
2. The various uses to which cash is put in and therefore, moves or flows out of the
organization.
The net effect of such cash movements is called net cash flow which is to be added to
(or deducted from, as the case may be) the opening balance of cash to arrive at the closing
balance of cash. That means,
Management Accounting : 368

rOpening balance] + (Net CaSh] = (ClOSing balanCe]


L of Cash Inflow of Cash

Alternatively,

rOpening balancel + rCash J (CaSh] =


(
Closing balance ]
L of Cash J ~nflows Outflows of Cash

Hence, the different sources and applications (uses) of cash require some explanation.
The summary of the same is presented herein under.
Sources and Application of Cash
Sources of Cash or Cash Inflow or AppH~ation of Cash or Cash Outflows or
Derivation of Cash Uses of Cash or Desposition of Cash
Cash Trading (Operating) Profit Cash Trading (Operating) Loss
Decrease in any Asset Increase in any Asset
except Cash ------.~I C except Cash
A
Increase in any Liability----~.I-_ ___,~ Decrease in any Liability
S
Issue of Shares -------.~
H Redemption of Redeemable
Rent, Interest and Dividend Preference Shares
received Cash Dividend

The various sources of cash as presented above can be classified into two broad
categories as (1) Internal Sources, and (2) External Sources. Internal Sources of Cash refer
to the Cash from Operations. Other sources of cash such as (a) Sale of Fixed Assets,
Investments, Intangible Assets, etc., (b) Issue of Finartcial Instruments like Equity Shares,
Preference Shares, Debentures, etc., (c) Receipt of Interest, Dividend, etc., represent the
External Sources of Cash.
(1) Cash from Operation or Cash Trading (Operating) Profit or Loss
The major source of cash is the amount of profit earned during the year from its
business operations. But this profit is influenced even by the items which do not affect (i.e.,
decrease or increase) cash. Such items are to be adjusted to arrive at the Cash Trading Profit
or Loss. It may be noted here that we are interested in computing cash trading profit. That
means to say, only the operating activities are considered. The costs and the revenues
associated with the non-operating and non-cash items are to be excluded. The cash from
operation is, therefore, nothing but the Cash Trading or Cash Operating Profit. It should be
noted here that in case of cash trading loss, it represents the negative cash flows (i.e., cash
outflows). In order to arrive at the amount of cash from operation, two important approaches
are available. They are (a) Income and Expenditure Approach also known as, Cash Sales
Method and (b) Net Profit Approach.
Cash Flow Statement : 369

(a) Income and Expenditure Approach (Cash Sales Method)


It is known that all business activities for a certain period of an organization result in
either the profit or loss or 'no-profit, no-loss.' It is also a known fact that the profit usually
results in the inflow of cash (i.e., net) as the total inflows in the form of revenue or income
exceed the total outflow in the form of expense. In the l:ase of loss, the reverse will be the
effect. That is, since the outflow of cash exceeds the inflow of cash, the loss situation will
result in the outflow of cash (i.e., net). In case of 'no protit, no loss' situation, there is no net
inflow or net outflow of cash. That means,
Profit situation when, Revenue> Expenses;
Loss situation when, Revenue < Expenses; and
No-profit, No-loss situation when, Revenue =Expenses.
But this profit or loss is not exactly equal to the amount of cash from operation.
Because, the net profit or loss is arrived at after considering even non-operating or non-
trading items. The amount of profit is, therefore, not equal to the cash from operation or cash
depleted in operation. Hence, some adjustments are to be made to the profit or loss figure to
arrive at the cash from operation or cash depleted in operation.
Under this approach, cash from operation (it may be positive cash flows which is
also known as cash inflows is case of profit; or it may· be negative cash flows which is also
known as cash outflows in case of loss) is arrived at by subtracting 'the aggregate of cash
purchases and cash operating expenses including conversion costs' from the amount of cash
sales revenue. That means, the credit sales (it can be computed from the Total Debtors
Account) must be deducted from the total sales to arrive at the cash sales revenue. In the
absence of any specific information, the entire sale is normally reckoned as on cash basis. In
the same way, all the items (except the Book Costs and Non-trading Items) appearing in the
Income Statement are assumed to be cash items. Book Costs are excluded as they do to
either increase or decrease cash (e.g., depreciation, amortization of various deferred revenue
expenditures, etc). Further, Non-trading items are excluded with the object of ascertaining the
cash from trading activities. Of course, they are considered separately in the Cash Flow
Statement. On the basis of these hints, Profit and Loss Account may be re-stated on cash
basis to compute the Cash Trading Profit as shown in the following illustration.
Dlustration: 5.1
From the following Profit and Loss Account of Pondi Company for the year ended
December 31, 2005, compute the amount of Cash from Operation.
Particulars Rs. Particulars Rs.
To Opening Stock of Raw Materials 2,00,000 By Sales (cash) 30,00,000
To Purchase of Materials 10,00,000 By Closing Stock of Materials 8,00,000
To Wages and Salaries 6,00,000 By Dividend Received 20,000
Add: Outstanding 60,000 6,60,000 By Interest Received 15,000
To Other Conversion Cost 5,00,000
To Operating Expenses 2,00,000
Management Accounting : 370

Add: Outstanding 5,000 2,05,000


To Goodwill written off 10,000
To Preliminary expenses written off 15,000
To Net Profit 12,45,000
38,35,000 38,35,000
Note: 1. Conversion costs include depreciation on building, machmery, etc., of Rs. 3 lakh; and
2. All purchases were on cash basis.
Solution:
Dr Profit and Loss Alc ofPondi Co., for the year ended December 31, 2005 (Cash basis) Cr
Amount Amount
Particulars Particulars
Rs. Rs.
To Cash Purchases 10,00,000 By Cash Sales 30,00,000
To Wages and Salaries Paid 6,00,000
To Other Conversion Costs
(excluding Depreciation) 2,00,000
To Operating Expenses 2,00,000
To Cash Trading Profit (Le., Cash
from Operation; balancing figure) 10,00,000
30,00,000 30,00,000

Cash Trading Profit may also be computed by preparing a simple Income Statement
on cash basis as presented below.
Income Statement of Pondi Co., for the year ended December 31, 2005
Amount
Particulars Rs.
Rs.
Sales Revenue 30,00,000
Less: Cash Operating Costs:
Purchase of Raw-materials 10,00,000
Wages and Salaries Paid 6,00,000
Other Conversion Costs excluding Depreciation 2,00,000
Operating Expenses paid 2,00;000 20,00,000
:. Cash Trading Profit 10,00,000

(b) Net Profit Approach


Under this approach, cash from trading operations is ascertained with the help of net
profit as shown in the Profit and Loss Account. Non-cash items (such as amortization of
fictitious assets, depreciation, credit purchases, outstanding expenses) and non-trading items
I
Cash Flow Statement: 371

(such as provisions and reserves, loss on sale of fixed assets, etc.,) which are debited to the
Profit and Loss Account are added back to the amount of net profit. It is because of the
reason that these items do not result in cash outflow. In the same way, non-trading receipts
(such as profit on sale of fixed assets, dividend and commission received, etc.,) and non-cash
revenues (such as interest accrued, commission accrued, etc .• ) which are credited to the Profit
and Loss Account are to be deducted from the net profit. Further, decrease in accrued income
is added to the net profit and increase in accrued income is deducted from the net profit. A
pro-forma statement showing the computation of cash trading profit is presented below.
Statement Showing Cash from Operation
Particulars Rs. Rs.
Trading Profit as per Profit and Loss Account A
Add: Non-cash and Non-trading items (which are debited to Profit and
Loss Account):
Depreciation on Fixed Assets b
Amounts of Intangible Assets (such as Goodwill, Patents, Trade
Marks. etc.,) written off c
Amounts of Fictitious Assets (like, Preliminary Expenses.
Underwriting Commission. Discount on Issue of Shares and
Debentures) written off d
Reserve for Bad Debts and Reserve for Discount on Debtors e
Donations made, Compensation paid, Fines and Penalties paid, etc f
Loss on sale of Fixed Assets and Investments g
Appropriation of Profits (like Profit transferred to Reserve Account,
Dividend Proposed/Declared, etc) h
Decrease in Accrued Income i
Increase in Outstanding Expenses j
Increase in Tax Liability k L
Less: Non-trading Receipts (which are credited to Profit and Loss A+L
Account):
Profit on Sale of Fixed Assets and Investments m
Interest, Dividend, etc., Received n
Compensation, Donation, etc., Received p
Increase in Accrued Income q
Decrease in Outstanding Expenses r
Decrease in Tax Liability s T
:. Cash from Operation A+L-T
j

Management Accounting : 372

An illustration may be taken to show how the Cash Trading Profit can be arrived at
under the Net Profit Approach. The same illustration [Illustration: 5.1] which was used under
the 'Income and Expenditure Approach' is used here also.
Statement Showing the Amount of Cash from Operation
Amount
~articulars Rs.
Rs.
Net Profit as per the Profit and Loss Account 12,45,000
Add: Non-cash and Non-trading Items:
Outstanding Wages and Salaries 60,000
Depreciation 3,00,000
Outstanding Operating Expenses 5,000
Goodwill written-off 10,000
Preliminary Expenses written-off 15,000
Opening Stock of Raw-materials 2,00.000 5,90,000
18,35,000
Less: Non-cash and Non-trading Receipts:
Dividend 20,000
Interest 15,000
Closing Stock of Raw-materials 800000 835000
:. Cash from Operation 10.00,000
The amount of Operating Cash Profit can also be computed by preparing an Adjusted
Profit and Loss Account as presented below. 'c

Dr Adjusted Profit and Loss Account of A Company for the year ended ... Cr
Amount Amount
Particulars Particulars
Rs. Rs.
To Non-cash Expenses * By Balance bId (i.e., opening
To Non-operating Expenses * credit balance of P & L
To Appropriations * Appropriation Alc)
To Balance dd (closing credit balance By Non-operating Incomes *
ofP & L Appropriation Alc) By Non-cash Incomes *
By Operating Cash Profit
(balancing figure)

Note: * Details of items of non-cash expenses, etc., are as presented in the earlier statement.
Cash Flow Statement : 373

Whatever the approach followed to compute the amount of Cash from Operation, it
should be noted that the amount of Cash Trading Profit is to be added to the cash balance at
the beginning of the accounting period while preparing the Cash Flow Statement. In the same
way, if the business activities result in cash trading loss, it should be deducted from the
opening balance of cash while preparing the Cash Flow Statement.
(2) Decrease in any Asset (except Cash)
Decrease in any Asset (except Cash) on account of either depreciation or writing-off
of assets or credit sales of assets will not bring in any cash. But cash sales of assets will cause
an inflow of cash. Unless and otherwise stated specifically the reasons for the decrease in the
book value of the assets, it is reckoned as due to the sales and that too on cash basis. For
instance, assume that the following bits of information are extracted from the Balance Sheet
of a company as on March 31, 2005. The company sold a machine whose written-down value
is Rs. 2.5 lakh for a price of Rs. 1 lakh. The amount of depreciation provided on this machine ..
up to March 31, 2005 come to Rs. 1.751akh. The company used this machine for a period of
7 years and at the time of acquisition, it was estimated that the machine will have a useful life
of 17 years with no residual value. The company has been following Straight Line Method
for depreciation. From the above, it can be seen that the written-down value of the machine
goes on decreasing every year at the rate of Rs. 25,000 (i.e., Rs. 1,75,000 + 7 years) which is
the amount of depreciation. But this decrease in the value of the machine will not cause any
inflow of cash. But when the company sold this machine, its account will be closed and
therefore, the written-down-value of the machine gets reduced from Rs. 2.5 lakh to zero by
March 31, 2005. This decrease is due to the sale and this sale will cause the inflow of cash.
But the amount of cash inflow caused by this '~'pe of transaction is not necessarily equal to
the decrease in the amount of fixed assets. So, one has to consider the actual cash generated
by this type of activities. In this case, though the reduction in the fixed asset comes to Rs. 2.5
lakh, the cash inflow generated by this transaction is equivalent to only Rs. 1 lakh. Of course,
it is necessary to account for the difference of Rs. 1.5 lakh which is nothing but the loss. The
calculation is shown below.

Acquisition cost of machine [Rs. 25,000 x 17 years] Rs. 4,25,000


Less: Depreciation for the last seven years (Rs. 25,000 per annum) 1,75,000
Undepreciated book value of the machine 2,50,000
Sales price 1,00,000
:. Loss 1,50,000

From the above, it is obvious ,that the sale of assets would cause an inflow of cash. In
the same way, decrease in Trade Debtors represents the amount of cash collected from the
customers to whom the company had sold the goods and services on credit basis. That
means, the decrease in Trade Debtors, and in the same analogy, decrease in Bills Receivable
result in the inflow of cash. The decrease in Inventories implies that the cost of goods sold is
more than the cost of goods purchased or manufactured and therefore, decrease in Inventories
results in the inflow of cash. Even the decrease in Pre-paid Expenses represents the fact that
Management Accounting: 374

the company has paid less for services than the services actually used currently. The balance
in the Pre-paid Expenses Account (say, Insurance Premium) at the beginning of the
accounting year implies that cash has been paid in one of the previous years but a part or
whole or more of the service is being used during the current year. The actual cash outflow,
therefore, on the insurance expenses for the current year would be less by the amount of
balance in the Pre-paid Expenses Account at the beginning of the year. For instance, the
insurance premium for the year 2005 comes to Rs. ,10,000 and the company had an opening
balance of Rs. 2.000 in Pre-paid Insurance Premium Account. In this case, though the
expense for the year comes to Rs. 10,000, it has to pay only Rs. 8,000. That means, a
decrease in an asset is, unless and otherwise stated or implied, considered and recorded as a
source of cash. The total amount of cash generated through all these activities is to be added
to the opening balance of cash while preparing the Cash Flow Statement.
(3) Increase in any Liability
Whenever there is an increase in any existing liability or when a new lIability
(including the Share Capital) is created, cash flows into the organization and therefore, it
should be added to the opening balance of cash while preparing the Cash Flow Statement.
The liability so increased or created may be either long-term liability or fixed liability or
current liability. These transactions will result in the inflow of cash. But the increase in the
'liability or the creation of a new liability, on account of purchase of assets or on account of
redemption of debentures, etc., will not cause the inflow of actual cash. However, it generates
only the notional cash inflows.
(4) Increase in any Asset (except Cash)
When an organization purchases an asset (either fixed or current), it would result in
an outflow of cash and therefore, it reduces the cash balance. If a company purchases an asset
on (long-term) credit basis or if the company purchases an asset by issuing shares and
debentures, it will not cause outflow of actual cash. Hence, the increase in any asset may be
considered as causing an outflow of cash, only when the increase is on account of cash
purchases.
(5) Decrease in any Liability
When a liability (whether long-term or short-term) is discharged, either fully or
partly, there will be an outflow of cash.
(6) Increase or Decrease in Income-in-Advance
Increase in income received in advance (Le., when the closing balance exceeds the
opening balance) indicates the greater cash inflow and therefore, it should be considered as a
source of cash while preparing Cash Flow Statement. In the same way, decrease in income
received in advance (Le., when the closing' balance is less the opening balance) is to be
considered as an outflow of cash.
Preparation of Cash Flow Statement
Once the information about various sources and uses of cash are ascertained and
gathered, one can easily prepare the Cash Flow Statement. Though there is no set of format
for a Cash Flow Statement, it is normally prepared in the form of a report (Le., reconciliation
type) wherein the Statement begins with the opening balance of cash. To this opening
Cash Flow Statement : 375

balance, total of different sources of cash is added, and total of different applications of cash
is subtracted. The resultant figure represents the cash balance at the end of the accounting
period. A proforma of this type of Cash Flow Statement is presented below.
Cash Flow Statement ••• Company for the year •••
Amount
Particulars Rs.
Rs.
Opening balance of Cash M
Add: Cash Inflows:
Cash T~ading Profit n
Sale of Assets p
Loan raised q
Issue of Shares r
Interest, etc., Received s T
M+T
Less: Cash Outflows:
Cash Trading Loss a
Purchase of Assets b
Loan Repaid c
Preference Shares Redeemed d
Dividend Paid e F
:. Closing balance <:>f Cash M+T-F

A slightly variant approach to the above is the preparation of a statement to find out
the net increase or decrease in cash balance. As is known, the difference between the totals of
sources and applications of cash represents the net increase or decrease in cash. Another
statement is prepared which begins with the opening balance of cash. To this opening
balance, net increase is added or net decrease is deducted from the opening balance of cash.
The resultant figure represents the closing cash balance. A proforma of Cash Flow Statement
under this approach is presented below.

Statement Showing the Computation of Net Cash Inflows


Amount
Particulars Rs.
Rs.
Cash Inflows:
Cash Trading Profit a
Sale of Assets b
Loan Raised c
Management Accounting : 376

Issue of Shares d
Interest, etc., Received e T
Less: Cash Outflows:
Cash Trading Loss f
Purchase of Assets g
Loan Repaid h
Redemption of Preference Shares i F
Net Cash Inflows T-F=G

Cash Flow Statement of ••• Company for the year •••


Amount
Particulars
Rs.
Opening balance of Cash M
Add: Net Cash Inflows * G
:. Closing balance of Cash M+G
Note: * In case of N~t Cash Outflows, it should be subtracted from the opening balance of
cash.
The Cash Flow Statement is also prepared in an account form wherein the statement
is divided into two principal sections - Sources of Cash (plus opening balance) and
Applications of Cash. The difference between the two sections represents the closing balance
of cash. A model of Cash Flow Statement under this method is given below.
Cash Flow Statement of ... Company for the year ••.
Amount Amount
Sources Applications
Rs. Rs.
Opening balance of Cash Various Uses of Cash
Various Sources of Cash Closing balaiIce of Cash (hal)

Dlustration: 5.2
The Profit and Loss Nc of an enterprise for the year ended 31.12.1995 was as follows:
To Opening stock Rs.I0,000 By Sales Rs.l,40,OOO
To Purchases . 98,000 By Closin!5stock 12,000
To Wages paid 15,000
Add: Outstanding 3,000 18,000
To Gross profit 26,000
Cash Flow Statement : 377
1,52,000 1,52,000
To Salaries paid 10,000 By Gross profit 26,000
Add: Outstanding 2,000 12,000 By Interest received 2,000
To Insurance 1,200 Add: Accrued 500 2,500
Less: Pre-paid 200 1,000 By Net loss 8,500
To Selling expenses 8,000
To Depreciation 10,000
To Goodwill written off 5,000
To Preliminary
1,000
Expenses written off
37,000 37,000
From the above Profit and Loss Alc, ascertain cash profit from operations.
[Kuvempu Un;' B.B.M, November1999}
Solution:
Statement of Cash Profit from Operation
Amount
Particulars Rs.
(Rs)
Sales Revenue 1,40,000
Less: Purchases 98,000
Wages paid 15,000
Salaries paid 10,000
Insurance Premium paid 1,200
Selling expenses 8,000 1,32,200
.'. Cash Profit from Operation 7,800

mustration: 5.3
After taking into consideration the following items, Jain Ltd., made a net profit of Rs.
1,00,000 for the year ended 31st December 2003.
Loss on sale of Machinery Rs. 10,000
Depreciation on Building 4,000
Depreciation on Machinery 5,000
Preliminary Expenses written off 5,000
Provision for Taxation 10,000
Management Accounting : 378

Goodwill written off 5,000


Gain on sale of Building 8,000
Calculate cash from operations.
[Kuvempu Uni, B.Com, October 2004J
Solution:
Statement of Cash Profit from Operations
Amount
Particulars Rs.
(Rs)
Profit earned during 2003 1,00,000
Add back: Loss on sale of Machinery 10,000
Depreciation on: Building 4,000
Machinery 5,000
Preliminary Expenses written-off 5,000
Provision for Taxation 10,000
Goodwill written-off 5,000 39,000
1,39,000
Less: Gain on sale of Buildings 8,000
.'. Cash Profit from Operations 1,31,000

mustr&tion: 5.4
Calculate cash frOI.D trading operations.
Net Loss as per PIL Alc Rs.25,Ooo
OIS Wages and Salaries 80,000
Depreciation 2,00,000
Goodwill written off 85,000
Dividend received 10,000
Commission accrued 15,000
[Kuvempu Uni, B.C~m, October 2000 and May 2001J
Cash Flow Statement : 379
Solution:
Statement of Cash from Trading Operations
Amount
Particulars Rs.
(Rs)
Net Loss as per Profit and Loss Nc 25,000
Add back: Outstanding Wages and Salaries 80,000
Depreciation 2,00,000
Goodwill written off 85,000 3,65,000
3,90,000
Less: Dividend received 10,000
Commission accrued 15,000 25,000
:. Cash Trading Profit 3,65,000

mustration: SOS
X Ltd., made a profit of Rs.l,85,OOO after considering the following:
a. Depreciation on fixed assets Rs.5,OOO
b. Profit on sale of buildings 10,000
c. Loss on sale of machinery 4,000
d. Provision for taxation 30,000
e. Provision for doubtful debts 500
f. Transfer to general reserve 12,000
g. Goodwill written off 2,000
The following additional information is also given:
31.12.1997 31.12.1998
Debtors Rs.18,OOO Rs.17,000
Creditors 12,000 9,000
Bills Receivable 7,000 4,000
Bills Payable 3,000 3,500
Outstanding Expenses 4,000 2,000
Prepaid Expenses 2,000 2,500
You are required to ascertain the cash flow from operations.
[K14vempu Uni, B.Com, October 2003J
Management Accounting : 380

Solution:
Statement of Cash Flow from Operations
.------ -
Amount
Particulars Rs.
(Rs)
t----
Prolit earned during 1998 1,85,000
Add back: Depreciation on Fixed Assets 5,000
Loss on sale of Machinery 4,000
Provision for: Taxation 30,000
Doubtful Debts 500
Transfer to General Reserve 12,000
Goodwill written off 2,000 53,500

-.
2,38,500
Less: Profit on sale of Buildings lO,OOO
:. Cash Profit from Operation 2,28.500
Add: Decrease in Debtors 1,000
Decrease in Bills Receivable 3,000
Increase in Bills Payable 500 4,500
2,33,000
Less: Decrease in Creditors 3,000
Decrease in Outstanding Expenses 2,000
Increase in Pre-paid Expenses 500 5,500
:. Cash Flow from Operation 2,27,500
Illustration: -
~.6

From the following balances, you are required to calculate cash flow from operations.
31.12.90 31.12.91
Debtors Rs.50,000 Rs.47,000
Creditors 20,000 25,000
Bills Receivable 10,000 12,500
Bills Payable 8,000 6,000
Outstanding Expenses 1,000 5,000
Prepaid Expenses 800 700
Accrued Incomes 600 750
Cash Flow Statement : 381
Incomes received in advance 300 250
Profits made during the year 1,30,000
[Kuvempu Uni, B.Com, October 1996 and Mangalore Uni, B.Com, May 2000J
Solution:
Statement of Cash Flow from Operations
Amount
Particulars Rs.
(Rs)
Profit earned during 1991 1,30,000
Add: Increase in Outstanding Expenses 4,000
1,34,000
Less: Increase in Accrued IncQmes 150
~

:. Cash Pt9fit from Operation 1,33,850


Add: Decrease in Debtors' 3,000
Increase in Creditors 5,000
Decrease in Pre-paid Expenses 100 8,100
1,41,950
Less: Increase in Bills Receivable 2,500
Decrease in Bills Payable 2,000
Decrease in Income received in
50 4,550
Advance
:. Cash Flow from Operation 1,37,400

Illustration: 5.7
Compute cash from operation from the following information. Given: Profit for the
year 2002 is Rs.I 0,000 after providing depreciation of Rs.2,000.
31-12-2001 31-12-2002
Sundry Debtors Rs.I0,000 Rs.12,000
Provision for bad debts 1,000 1,200
Bills receivable 4,000 3,000
Bills payable 5,000 6,000
Sundry creditors 8,000 9,000
Inventories 5,000 8,000
Short-term investments 10,000 12,000
Management Accounting : 382

Outstanding expenses 1,000 1,500


Prepaid expenses 2,000 1,000
Accrued income 3,000 4,000
Income received in advance 2,000 1,000
{Bangalore Un;' B.Com, April 2003J
Solution:
Statement of Cash Profit from Operation
Amount
Particulars
(Rs)
Profit for 2001 10,000
Add back: Increase in Provision for Bad Debts 200
Depreciation 2,000
Increase in Outstanding Expenses 500
12,700
Less: Increase in Accrued Income 1,000
:. Cash Profit from Operation 11,700

Statement of Cash from Operation


Amount
Particulars
(Rs)
Cash Profit from Operation 11,700
Add: Decrease in Pre-paid Expenses Rs.l,OOO
Decrease in Bills Receivable 1,000
Increase in Bills Payable 1,000
Increase in Creditors 1,000 4,000
15,700
Less: Increase in Debtors 2,000
Increase in Inventories 3,000
Increase in Short-term Investments 2,000
Decrease in Income Received in
1,000 8,000
Advance
:. Cash from Operation 7,700
Cash Flow Statement : 383
Illustrations: 5.8
From the following particulars, prepare Cash Flow Statement of Kumar:
Asstes: 1-1-1995 31-12-1995
Cash Rs.I0,000 Rs.8,000
Debtors 80,000 90,000
Stock 60,000 50,000
Land 60,000 80,000
Building 1,00,000 1,10,000
Machinery 1,40,000 1,60,000
4,50,000 4,98,000
Liabilities:
Current Liability 70,000 80,000
Loan from Ravi 50,000
Bank Loan 80,000 60,000
Capital 3,00,000 3,08,000
4,50,000 4,98,000
During the year 1995, Kumar brought in an additional capital of Rs.20,000 and his
drawings during the year were Rs.62,000. Allow provision for depreciation on machinery:
opening balance Rs.60,000 and closing balance Rs.88,000.
[Kuvempu Uni, B.B.M, May 2000J
Solution:
1. Capital Alc balance on 1.1.1995 Rs.3,00,000
Add: Additional Capital 20,000
3,20,000
Less: Withdrawals 62,000
2,58,000
Closing balance 3,08,000
:. Profit for 1995 50,000
2. Closing balance of Depreciation Provision Rs.88,000
Less: Opening balance 60,000
.', Depreciation for 1995 28,000
3, Fixed Assets purchased during 1995:
Land Rs.20,000
Buildings 10;000
Management Accounting : 384

Machines 48,000*
*[Rs.l,60,OOO+Rs.88,OOO]-[Rs.1,40,OOO+Rs.60,OOO]
== [Rs.2,48,000 - Rs~2,00,000] = Rs.48,OOO
4. Determination of Cash Profit from Operation:
Profit earned during 1995 Rs.50,000
Add back: Depreciation 28,000
:. Cash Profit from Operation 78,000

Cash Flow Statement of Kumar for the Year ended December 31,1995
Amount Amount
Sources Applications
(Rs) (Rs)
Opening balance of Cash 10,000 Amount Withdrawn 62,000
Cash Profit from Operation 78,000 Purchase of:
Additional Capital brought 20,000 Land • 20,000
Decrease in Stock LO,OOO Buildings 10,000
Increase in Current -Liability 10,000 Machines 48,000
Loan from Ravi 50,000 Increase in Debtors 10,000
Repayment of Bank Loan 20,000
Closing balance of Cash 8,000
1,78,000 1,78,000

mustration: 5.9
Balance sheets of Devashri Ltd., on 1-1-1999 and 31-12-1999 were as follows:
Liabilities 1-1-1999 31-12-1999 Assets 1-1-1999 31-12-1999
Creditors Rs.40,000 Rs.44,OOO Cash
, Rs.1O,OOO Rs.7,000
Mrs. Deva's Loan 25,000 Debtors 30,000 50,000
Loan from Bank 40,000 50,000 Stock 35,000 25,000
Capital 1,25,000 1,53,000 Machinery 80,000 55,000
Land 40,000 50,000
Building 35,000 60,000
2,30,000 2,47,000 2,30,000 2,47,000
Cash Flow Statel'flent : 385
During the year, a machine costing Rs.lO,OOO (accumulated depredation Rs.3,000)
was sold for Rs.5,OOO. The provision for depreciation against machinery as on 1.1.1999 was
Rs.25,000 and un 31.12.1999 Rs.40,000. Net profit for the year 1999 amounted Rs.45,OOO.
You are required to prepare cash flow statement.
[Mangalore Uni., B.Com., October 2001, Kuvempu Uni., B.Com., November 2000J
Solution:
1. Cost of the Machine sold Rs.IO,OOO
Less: Accumulated Depreciation 3,000
:. Written down-value of the Machine sold 7,000
Selling Price 5,000
:. Loss on the Machine sold 2,000
2. Provision for Depreciation on Machinery on 1.1. 1999 Rs.25,000
Less: Accumulated Depreciation on the Machine sold 3,000
22,000
Provision for Depreciation on Machinery on 31.12.1999 40,000
:. Depreciation on Machinery for 1999 18,000
3. Net Profit for 1999 Rs.45,000
Add back: Depreciation on Machinery 18,000
Loss on Machine sold 2,000
:. Cash Trading Profit 65,000

Dr. Machinery Alc Cr


Amount Amount
Particulars Particulars
(Rs) (Rs)
To Balance b/d* 1,05,000 By Provision for Depreciation Alc
(accumulated Depreciation on
Machine sold 3,000
By Cash Alc (selling price) 5,000
By Profit and Loss Alc (loss on sale) 2,000
By Balance c/d* 95,000

1,05,000 1,05,000
* Including accumulated depreciation.
Management Accounting : 386

Dr. Capital Alc Cr.


Amount Amount
Particulars Particulars
(Rs) (Rs)
To Drawings (balance) 17,000 By Balance bId 1,25,000
By Profit and Loss Alc (Profit of
the year] 45,000
To Balance cld 1,53,000
1,70,000 1.70,000

Cash Flow Statement of Devashri Ltd., for the Year ended December 31, 1999
Amount Amount
Sources Applications
(Rs) (Rs)
Opening balance of Cash 10,000 Repayment of Mrs.Deva's Loan 25,000
Cash Trading Profit 65,000 Increase in Debtors 20,000
Sale of Machine 5,000 Drawings 17,000
Increase in Creditors 4,000 Purchase of Land lO,OOO
Increase in Loan from Bank lO,OOO Purchase of Buildings* 25,000
Decrease in Stock 10,000 Closing balance of Cash 7,000
1,04,000 1,04,000
*Increase IS assumed to be on account of purchase
mustration: 5.10
Following are the Balance Sheets of A Company Ltd., as on 31st December:'
2002 2003
Liabilities:
Share Capital Rs.70,000 Rs.74,000
Profit and Loss Alc 10,040 lO,560
Debentures ·12,000 6,000
Trade Creditors 10,360 11,840
Outstanding Expenses 700 800
Total 1,03,lOO 1,03,200
Assets:
J
Cash Flow Statement : 387

Goodwill 10,000 5,000


Land 20,000 30,000
Investments 14,900 17,700
Stock 49,200 42,700
Cash 9,000· 7,800
Total 1,03,100 1,03,200

Additional Infonnation:
a. Dividend paid Rs.3,500 c. Goodwill written off Rs.5,OOO
b. Land was purchased for Rs.lO,OOO d. Debentures paid off Rs.6,000
Prepare Cash Flow Statement.
IKuvempu Uni., B.B.M., December 1996, Mangalore Uni, B.Com., May 2003 and May and
November 2004]
Solution:
Profit and Loss Alc balance:
31.12.2003 Rs.lO,560
31.12.2002 10,040
:. Profit earned during 2003 and retained 520
Add back: Goodwill written off 5,000
Increase in Outstanding Expenses 100
Dividend 3,500
:. Cash Profit from Operation 9,120

Cash Flow Statement of A Company Ltd., for the Year ended December 31, 2003
Amount Amount
Sources Applications
(Rs) (Rs)

Opening balance of Cash 9,000 Redemption of Debentures 6,000

Cash Profit from Operation 9,120 Purchase of: Land 10,000

Increase in Share Capital 4,000 Investments 2,800

Increase in Creditors 1,480 Dividend 3,500

Decrease in Stock 6,500 Closing balance of Cash 7,800


30,100 30,100
Dlustration: S.l1
Management Accountin~
l 388

From the following Balance Sheets of M Ltd., P '!pare Cash Flow Statement for the
period ended 31st December, 1990.
Liabilities 1989 1990 Assets 1989 1990
Equity share capital ·3,00,000 4,00,000 Goodwill 1,15,000 90,000

Preference share Land and building 2,00,000 1,70,000


capital 1,50,000 1,00,000 Machinery 80,000 2,00,000
General reserve 40,000 70,000 Stock 77,000 1,09,000
Profit and Loss Nc 30,000 48,000 Debtors 1,60,000 2,00,000
Proposed dividend 42,000 50,000 Bills receivable 20,000 30,000
Creditors 55,000 83,000 Cash and Bank 25,000 18,000
Bills payable 20,000 16,000
Provision for taxation 40,000 50,000
6,77,000 8,17,000 6,77,000 8,17,000

Additional information:
a. Depreciation Rs.l 0,000 and Rs.20,000 have been charged on machinery, and land and
buildings respectively in 1990.
b. An interim dividend of Rs.20,000 has been paid in 1990.
c. Rs.35,000"income tax was paid during the year 1990.
[Kuvempu Uni., B.Com., May 1996J
Solution:
1. Machinery Nc - Opening balance Rs.80,000
Less: Depreciation for 1990 10,000
70,000
Closing balance of Machinery Nc 2,00.000
:. Purchases during 1990 1,30,000
2. Opening balance of Land and Buildings Rs.2,00,000
Less: Depreciation for 1990 20,000
1,80,000
Closing balance of Land and Buildings 1,70,000
:. Sale of Land and Buildings 10,000
Cash Flow Statement : 389

3. Proposed dividend of Rs.50,000 for 1990 and interim dividenc,! of


Rs.20,000 are to be added back to profit to arrive at the Cash Trading
Profit. Proposed dividend of Rs.42.000 for 1989 and interim dividend of
Rs.20.000 are to be taken to Cash Flow Statement.
4. Opening balance of Tax Provision Rs.40,000
Less: Tax paid 35,000
5,000
Closing Balance 50,000
:. Provision made for 1990 45,000
5. Computation of Cash Operating Profit:
Profit and Loss Alc balance on: 31.12.1990 Rs.48,000
31.12.1989 30,000
:. Profit earned during 1990 and retained 18,000
Add back: Depreciation on: Machine 10,000
Land and Buildings 20,000
Proposed Dividend for 1990 50,000
Interim Dividend for 1990 20,000
Provision for Taxation 45,000
Transfer to General Reserve 30,000
Goodwill written-off 25,000
:. Cash Profit from Operation 2,18,000

Cash Flow Statement of M Ltd., for the year ended December 31, 1990

Sources Amount(Rs) Applications Amount (Rs)

Opening balance of Cash and Bank 25,000 Purchase of Machines 1,30,000


Cash Protit from Operation 2,18,000 Payment of Dividend
Sale of Land and Buildings 10,000 Interim (1990) 20,000
Increase in Shares Capital 1,00,000 Fmal(1989) 42,000
Increase in Creditors 28,000 Payment of Tax 35,000
Redemption of Pref. Shares 50,000
Decrease in Bills Payable 4,000
Increase in Stock 32,000
Increase in Debtors 40,000
Increase in Bills Receivable 10,000
Management Accounting : 390
Closing balance of Cash &
Bank 18,000
3,81,000 3,81,000
mustration: S.12
The following are the summarised Balance Sheets of a company as on 31 st December
2001 and 2002. .
31-12-2001 31-12-2002
Liabilities:
Share capital Rs.l,OO,OOO Rs. 1,25,000
General Reserve 25,000 30,000
Profit and Loss Alc 15,250 15,300
Bank loan (long-term) 35,000
Sundry creditors 75,000 67,600
Provision for taxation 15,000 17,500
2,65,250 2,55,400
Assets:
Land and Buildings 1,00,000 95,000
Machinery 75,000 85,500
Stock 50,000 37,000
Sundry Debtors 40,000 32,100
Cash 250 300
Bank 4,000
Goodwill (at cost) 1,500
2,65,250 2,55,400
Additional information: During the year ended 31 st December 2002:
a. Dividend of Rs.ll,500 was paid,
b. Depreciation charged on Land and Buildings Rs.5,000,
c. Machinery was further purchased for Rs.19,000,
d. Depreciation written off on Machinery Rs.6,000,
e. Income tax provided during the year Rs.16,500, and
f. Loss on sale of machinery Rs.I00 was written off to General Reserve.
You are required to prepare a cash flow statement.
[Bangalore Uni, B.Com, November 2001, May 2002, and April 2003]
Cash Flow Statement: 391

Solution:
1. Computation of Cash Trading Profit:
Profit and loss Alc balance on: 31.12.2002 Rs.15,300
31.12.2001 15,250
:. Profit earned during 2002 and retained 50
Add back: Dividend 11,500
Depreciation on: Land and Buildings 5,000
Machinery 6,000
Income Tax Provision 16,500
Transfer to General Reserve (Note 2) 5,100 44,100
:. Cash Trading Profit 44,150

2. General Reserve: Opening balance 25,000


Less: Loss on sale of Machine 100
24,900
Closing balance of General Reserve 30,000
:. Profittransferred to General Reserve 5,100
----
3. Provision for Taxation Alc: Opening balance Rs.15,000
Add: Provision for 2002 16,500
31,500
Closing balance of Provision for Taxation Alc 17,500
:.Amount of Tax paid during 2002 14,000
4. Opening balance of Machinery Alc Rs.75,OOO
Add: Machines Purchased 19,000
94,000
Less: Depreciation for 2002 6,000
88,000
Closing balance of Machinery Alc 85,500
:. Written-down-value of Machine sold 2,500
Less: Loss on sale of the Machine 100
:. Selling Price of the Machine sold 2,400
Management Accounting : 392

Cash Flow Statement of •••••Company for the Year ended December 31, 2002
Amount Amount
Sources Applications
(Rs) (Rs)
Opening balance of Cash Tax paid 14,000
,j

and Bank (balance) 250 Purchase of Machines 19,000


Cash Operating Profit 44.150 Repayment of Bank Loan 35,000
Sale of Machine 2,400 Decrease in Sundry Creditors 7,400
Increase in Share Capital 25,000 Goodwill (cost) 1,500
Decrease in Stock 13,000 Dividend paid 11,500
Decrease in Sundry Debtors 7,900 Closing balance of Cash and
Bank (balance) 4,300
92,700 92,700

Illustration: 5.13
The following schedule shows the Balance Sheets in condensed form of Swaraj Co.,
Ltd. You are required to prepare a Cash Flow Statement.
1-1-2000 31-12-2000
Assets:
Cash and Bank balances Rs.45,000 Rs.45,000
Sundry Debtors 33,500 21,500
Temporary Investments 55,000 37,000
Prepaid Expenses 500- 1,000
Stock-in-trade 41,000 53,000
Land and Buildings 75,000 75,000
Machinery 26,000 35,000
2,76,000 2,67,500
Liabilities:
Sundry creditors 51,500 48,000
Outstanding Expenses 6,500 6,000
8% Debentures 45,000 35,00.0
Depreci~t!0n Fund 20,000 22,000
Cash Flow Statement : 393

Reserve for Contingencies 30,000 30,000


Profit and Loss Account 8,000 11,500
Capital 1,15,000 1,15,000
2,76,000 2,67,500
The following information is also available:
a. 10% dividend was paid in cash.
b. New machinery for Rs.15,000 was purchased but old machinery costing Rs.6,000 was
sold for Rs.2,000; accumulated depreciation was Rs.3,000.
c. Rs.lO,OOO 8% Debentures were redeemed by purchase from open market at Rs.96 for
a debenture of Rs.1 00.
d. Rs.18,000 investments were sold at book value
{Manga/ore Uni, B.Com, April 2001 and Bangalore Uni, B.Com, April 2004]
Solution:
1. Machinery Alc: Opening balance Rs.26,000
Add: Purchases 15,000 41,000

Less:' Cost of Machine sold 6,000

Closing balance 35,000

2. Cost of Machine sold Rs.6,000


Less: Accumulated Depreciation 3,000

:. Written-down-value of Machine sold 3,000

Selling Price 2,000

:.Loss on Machine sold 1,000

3. Depreciation Fund: Opening balance 20,000

Less: Accumulated Depreciation on Machine sold 3,000

17,000

Closing balance of Depreciation Fund 22,000


Management Accounting : 394

:. Depreciation for 2000 5,000


4. Face value of 8% Debentures redeemed Rs.I0,OOO
Less: Redemption value
[(Rs. 96 + Rs.100) x Rs. 10,000] 9,600
Discount on redemption 400
5. Cost and sales value of investments Rs. 18,000
6. Calculation of Cash Operating Profit:
Profit and Loss Alc balance on 31-12-2000 11,500
01-01-2000 8,000
:. Profit earned during 2000 and retained 3,500
Add back: Dividend (10% of Rs.l,15,000) 11,500
Loss on Machine sold 1,000
Depreciation on Machinery for 2000 5,000
21,000
Less: Discount on Redemption of Debentures 400
Decrease in Outstanding Expenses 500
Increase in Prepaid Expenses 500 1,400
:. Cash Operating Profit 19,600
-----
Cash Flow Statement of Swaraj Co., Ltd., for the Year ended December"31, 2000
Sources Amount Applications Amount
.Opening balance of Cash and Purchase of Machine 15,000
Bank Alc 45,000
Redemption of Debentures 9,600
Cash Operating. Profit 19,600 Dividend paid 11,500
Sale of Machine 2,000 Increase in Stock in trade 12,000
Sale of Investments 18,000 Decrease in Sundry Creditors 3,500
Decrease in Sundry Debtors
. 12,000 Closing balance of Cash and
BanKAlc 45,000
0

·96,600 96,600
Cash Flow Statement: 395

Illustration: 5.14
From the following Balance Sheets of NDA Co., Ltd., for the years 1999 and 2000,
prepare a cash flow statement:
Assets: 1999 2000
Plant and Machinery Rs.6,00,00 Rs.7,25,000
0
Less: Cumulative Depreciation 1,20,000 1,45,000
4,80,000 5,80.000
Land 1,83,000 1,98,000
Loan to subsidiary company 25,000 Nil
Shares in subsidiary company 30,000 40,000
Stock 1,60,000 1,48,000
Sundry Debtors 1,20,000 1,62,000
Bank balance 67,000 98,000
10,65,000 12,26,000
Liabilities:
Equity shares of Rs.l 00 each 4,50,000 6,00,000
Share premium Nil 15,000
P & L Appropriation Alc 60,000 60,000
Profit for the year Nil 50,000
8% Debentures 2,50,000 2,00,000
Profit on Redemption of Debentures Nil 1,000
Sundry Creditors 2,20,000 1,90,000
Provision for Taxation 40,000 50,000
Proposed Dividend 45,000 60,000
10,65,000 12,26,000
During the year, plant costing Rs. 40,000 was sold for Rs. 15,000. Accumulated
depreciation on plant was Rs. 20,,000. Loss on sale of plant was charged to Profit and Loss
Account. Tax paid during the year was Rs. 55,000. {Bangalore Uni., B.Com., April
2001J
Solution:
1. Plant and Machinery Alc: Opening balance Rs.6,00,000
Less: Cost of Plant sold 40,000
5,60,000
Closing balance 7,25,000
Management Accounting : 396

.
... Purchases made during 2000 1,65,000
2. Cost of Plant sold Rs.40,000
Less: Accumulated Depreciation 20,000
:. Written-down-value 20,000
Selling Price 15,000
Loss on Plant sold 5,000
----
3. Cumulative Depreciation up to 31.12.1999 Rs.l,20,000
Less: Accumulated Depreciation on Plant sold 20,000
1,00,000
Closing balance on 31.12.2000 1,45,000
:. Depreciation for 2000 45,000
4. Provision for Taxation (opening balance) Rs.40,000
Less: Tax paid 55,000
-15,000
Closing balance 50,000
:. Tax Provision for 2000 65,000
5. Land purchased Rs.15,000
6 .. Computation of Cash Profit from Operation:
Profit for the year 2000 Rs.50,000
Add back: Loss on Plant sold 5,000
Depreciation for 2000 45,000
Tax Provision for 2000 65,000
Proposed Dividend 60,000
:. Cash Profit from Operation 2,25,000

Cash Flow Statement of NDA Co., Ltd., for the Year ended December 31, 2000
Amount Amount
Sources Applications
(Rs) (Rs)
Opening Bank balance 67,000 Purchase of Plant 1,65,000
Cash Profit form Operation 2,25,000 Tax paid 55,000
Sale of Plant 15,000 Purchase of Land 15,000
Loan recovery from subsidiary 25,000 Dividend paid (1999) 45,000
Issue of Shares (+ Premium) 1,65,000 Purchase of Shares of Subsidiary 10,000
Decrease in Stock 12,000 Redemption of Debentures (- Profit) 49,000
Cash Flow Statement : 397
Increase in Debtors . 42,000
Closing Bank balance 98,000
Decrease in Creditors 30,000
5,09,000 5,09,000

lllustration: 5.15
A company finds on January 1, 1983 that it is facing short of funds with which to
implement its expansion programme. On January 1, 1982, it had a balance of Rs. 1,80,00Q.
From the following information, prepare statement for the Board of Directors- to show how
the overdraft of Rs. 68,750 as at December 31, 1982 has arisen. Figures as per Balance Sheet
as on December 31:
1981 (Rs.) 1982 (Rs.)
Fixed assets 7,50,000 11,20,000
Stock and stores 1,90,000 3,30,000
Debtors 3,80,000 3,35,000
Bank balance 1,80.000 (Cr) 68,750 (Dr)
Trade creditors 2,70,000 3,50,000
Share capital in shares of Rs. 10 each 2,50,000 3,00,000
Bills receivable 87,500 95,000
The profit for the year ended December 31, 1982 before charging depreciation and
taxation amounted to Rs. 2,40,000. 5,000 shares are issued on January 1, 19~2 at a premium
of Rs. 5 per share. Rs. 1,37,500 was paid in March 1982 by way of income tax. Dividend
was paid as follows: (a) 1981 (Final) on capital at 31-12-1981 10% less tax (25%); and (b)
1982 interim 5% free of tax. [M.Com., University of Mysore, 1984J
Solution:
Cash Flow Statement of ••• for the Year ended December 31, 1982
Amount
Particulars Rs.
Rs.
Bank balance on January I, 1982 1,80,000
Add: Cash Inflows:
Issue of shares [5,000 shares x (Rs. 10 + Rs.5)] 75,000
Increase in Trade Creditors - 80,000
Decrease in Debtors 45,000
Cash flow from Operation 2,40,000 4,40,000
Less: Cash Outflow: 6,20,000
Dividend (10% of Rs. 2,50,000) 25,000
Management Accounting : 398

Less: 25% Tax 6,250 18,750


Interim Dividend (5% of Rs. 3 lakh) 15,000
Tax paid 1,37,500
Increase in Bills Receivable 7,500
Purchase of Fixed Assets 3,70,000
;

Increase in Stocks and Stores 1,40,000 6,88,750


Bank balance on December 31, 1982 - 68,750*
Note: *Overdraft
illustration: 5.16
The financial position of ABC Ltd., on 1st January, 1985 and 31st December, 1985
was as follows.
1st January 31st December
Particulars
Rs. Rs.
Cash 8,000 7,200
Debtors 70,000 76,800
Stock 50,000 44,000
Land 40,000 60,000
Buildings > 1,00,000 l,to,OOO
Machinery 1,60,000 1,72,000
4,28,000 4,70,000
Current Liabilities 72,000 82,000
Loan from associate company 0 40,000
Loan from Bank 60,000 50,000
Capital and Reserves 2,96,000 ~
2,98,000
4,28,000 4,70,000

During the year, Rs. 52,000 were paid as dividends. The provision for depreciation
against machinery as on 1st January, 1985 was Rs. 54,000 and on 31st December, 1985, Rs.
72,000. You are required to prepare the Cash Flow Statement as well as the Funds Flow
Statement. [ICWA (Fin), December 1986J
Solution:
1. Depreciation-for 1985 =Rs. 18,000 (Le., Rs. 72,000 - Rs. 54,000)
2. Calculation of Fund from Operation:
Profit earned and retained [Rs. 2,98,000 - Rs. 2,96,000] Rs. 2,000
Add: Dividend paid Rs. 52,000
Depreciation 18,000 70,000
... Funds/Cash from Operation 72,000
Cash Flow Statement : 399

3. Opening balance of Machine Rs. 1,60,000


Less: Depreciation 18,000
1,42,000
Closing Balance 1,72,000
:. Purchases 30,000
Cash Flow Statement of ABC Limited for the Year ended December 31, 1985
Amount
Particulars Rs.
Rs.
Opening balance 8,000
Add: Sources of Cash:
Cash from Operation 72,000
Decrease in Stock 6,000
Increase in <;::urrent Liabilities 10,000
Loan from Associate Company 40,000 1,28,000
1,36,000
Less: Application of Cash
Increase in Debtors 6,800
-
Purchase of Land 20,000
Purchase of Buildings 10,000
Purchase of Machinery 30,000
Part Re-payment of Bank Loan 10,000
Payment of Dividends 52,000 1,28,800
Closing balance of Cash 7,200

Schedule of Changes in Working Capital


Changes in Working
Amount (Rs.) as on ...
Particulars Capital (Rs.)
1.1.1985 31.12.1985 Increase Decrease
Current Assets:
Cash 8,000 7,200 0 800
Debtors 70,000 76,800 6,800 0
Stock 50,000 44,000 0 6,000
(a) 1,28,000 1,28,000
Current Liabilities 72,000 82,000 - 0 10,000
Management Accounting : 400
(b) 72,000 82,000
Therefore, Working Capital (a- b) 56,000 46,000
Total increase and decrease in Working Capital 6,800 16,800
Therefore, Net Decrease in Working Capital 10,000
16,800 16,800

Funds Flow Statement of ABC Ltd., for the Year ended December 31,1985
Amount Amount
Sources Uses
Rs. Rs.
Funds from Operation 72,000 Purchase of Machinery 30,000
Loan from Associate Company 40,000 Payment of Dividend 52,000
Net decreaSe in Working Capital 10,000 Acquisition of Land 20,000
Purchase of Buildings 10,000
Part Re-payment of loan 10,000
1,22,000 1,22,000

mustration: 5.17
The summarised balance sheets ofXYZ Ltd., as at 31-12-79 and 1980 are given below.
Liabilities 1979 1980 Assets 1979 1980
Share Capital 4,50,000 4,50.000 Fixed Assets 4,00,000 3,20,000
General Reserve 3,00,000 3,10,000 In~estments 50,000 60,000
P&LNc 56,000 68,000 Stock 2,40,000 2,10,000
Creditors 1,68,000 1,34,000 Debtors 2,10,000 4,55,000
Provision for Tax 75,000 10,000 Bank . 1,49,000 1,97,000
Mortgage Loan - 2,70,000
10,49,000 12,42,000 10,49,000 12,42,000

Additional Information:
1. Investments costing Rs. 8,000 were sold during the year 1980 for Rs. 8,500;
2. Provision for tax made during the year was Rs. 9,000;
3. During the year, part of the fixed assets costing Rs. 10,000 was sold for Rs. 12,000
and the profit was included in P & L Nc; and
4. Dividend paid during the year amounted to Rs. 40,000.
You are required to prepare a statement of sources and uses of cash.
res (Fin), June 1981]
Cash Flow Statement: 401
Solution:
Calculation of Cash from Operation:
Profit earned during the year and retained (difference between
closing and opening balances of P&L Account) Rs.12,000
Add: Transfer to General Reserve Rs.IO,OOO
Provision for Taxation 9,000
Dividend 40,000
Depreciation (4,00,000 -10,000 - 3,20,000) 70,000 1,29,000
1,41,000
Less: Profit on Sale of Investments 500
Profit on Sale of Fixed Assets 2,000 2,500
1,38,500

Dr Provision for Taxation Ale Cr


Particulars Rs. Particulars Rs.
To Bank (balance, paid) 74,000 By Balance bId (opening) 75,000
To Balance cld (closing) 10,000 By P&L Ale (Provision for 1980) 9,000
84,000 84,000

Opening balance of Investments Rs.50,000


Less: Cost Price of Investments sold 8,000
42,000
Closing balance 60,000
:. Purchases 18,000

Cash Flow Statement of XYZ Ltd., for the Year ended December 31,1980
Amount Amount
Sources Uses
Rs. Rs.
Opening balance of Bank 1,49,000 Payment of Tax 74.000
Cash from Operation 1,38,500 Payment of Dividend 40,000
Sale of Investments 8.500 Purchase of Investments 18,000
Sale of Fixed Assets 12,000 Increase in Debtors 2,45,000
Decrease in Stock 30,000 Decrease in Creditors 34,000
Mortgage loan 2,70,000 Closing balance of Bank 1,97,000
6,08,000 6,08,000
Management Accounting : 402

Dlustration: 5.18
XYZ Company Limited: Summary Balance Sheet
End of End of End of End of <
Liabilities Period - Period - Assets Period - Period -
I (Rs.) II (Rs.) I (Rs.) II (Rs.)
<

Share Capital: Fixed Assets:


Equity shares 65,500 65,500 Plant & Machinery 75,760 94,040
Preference Shares 41,100 38,000 Land & Building 32,000 31,000
Reserves: Loose Tools 840 760
General 6,000 8,000~ Goodwill 12,000 9,000
P&LAlc 1,21,000 1,24,799 Trade Investments 9,400 10,600
Loans 20,000 30,000 Current Assets:
Current Liabilities: Stock 1,72,789 1,50,372
Taxation 27,200 31,594 Investments 1,050 7,602
Dividends 4,900 7,111 Debtors 30,250 48,147
Creditors 34,600 40,987 Bills Receivable 1,206 3,079
Bank Overdraft < 15,500 18,770 Cash 505 10,161

3,35,800 3,64,761 3,35,800 3,64,761

Summary Profit and Loss Account for Period II


Particulars Rs. Rs. Rs.
Trading Profit (Prior to deducting the under noted items) 64,684
Depreciation, Buildings 1,000
Depreciation', Plant , 15,120
Depreciation, Loose Tools 360 16,480
Audit Fees 1,500
Directors: Remuneration 1,200
Other Remuneration 9,400 10,600
Loan interest 1,600 30,180
Profit before taxation 34,504
Income Tax 13,600
Profit after tax 20,904
Appropriation:
Amount written off, Goodwill 3,000
Transfer to General Reserve 2,000
Proposed Preference Dividend (Net) 1,339
Proposed Ordinary Dividend (Net) 5,772
Income tax payable< in respect of dividends 4,994 17,105
Cash Flow Statement: 403

3,799
Unappropriated profit brought forward 1,21,000
Unappropriated profit carried forward 1,24,799

Fixed Assets Schedule


Depreciation, Depreciation,
Book Value Cost Book
Assets Cost end of Period end of Period
(Rs.) (Rs) Value (Rs.)
- I (Rs.) - II (Rs.)
Plant 1,01,560 25,800 75,760 1,26,560 32,520 94,040
Land & Bldg. 35,000 3,000 32,000 35,000 4,000 3J.000
1,36,560 28,800 1,07,760 1,61,560 - 36,520 1,25,040
Loose Tools 840 760
Goodwill 12,000 9,000
1.20,600 1,34,800

Note that during period-II, plant was purchased at a cost of Rs. 35,200. Some of old plant
was also sold for Rs. 2,000. Any gain/loss is included in the trading profit of Rs. 64,684.
From the above information, prepare a statement accounting for the increase of Rs. 9,656 in
the cash position which has taken place during period-II. lCA (Final), November 1988J
Solution:
1. Goodwill written-off =Rs. 3.000
2. Opening balance of Loose Tools Rs.840
Less: Depreciation 360
480
Closing balance 760
:. Purchase of Loose Tools 280

3. Opening balance of Plant & Machinery Rs.75,760


Less: Depreciation for year-II [Rs. 32,520 - Rs. 25,800] 6,720
69,040
Add: Purchases 35,200
1,04,240

Closing balance 94,040


:. Cost of Plant Sold 10,200
Management Accounting : 404
4. Depreciation on Plant and Machinery up to the end of year - I: Rs. 25,800
J

Add: Depreciation on Plant (P & L Nc) 15,120


40,920
Less: Depreciation on remaining Plant & Machinery
up to the end of year - II: 32,520
:. Depreciation provided on Plant sold 8,400

5. Cost of Plant Sold Rs.IO,200 6. Calculation of Cash from Operation:


Less: Depreciation 8,400 Profit before Tax Rs. 34,504
Written-down-value 1,800 Add: Depreciatio~ 16,480
SeIling Price 2,000
50,984
:. Profit 200
Less: Profit on sale of old Plant 200
Cash from Operation 50,784

Cash Flow Statement of XYZ Company Ltd., Showing the Reasons for the Increase in
Cash Balance
Amount Amount
Sources Uses
Rs. Rs.
Cash from Operation 50,784 Purchase of Plant 35,200
Sale of Plant 2,000 Purchase of Loose Tools 280
Decrease in Stock 22,417 Purchase of Trade Investment 1,200
Increase in Loans 10,000 Purchase of Investment 6,552
Increase in Creditors 6,387 Increase in Debtors 17,897
Increase in Bank Overdraft 3,270 Increase in Bills Receivable 1,873
Redemption of Preference Shares 3,100
Payment of Tax 14,200
Payment of Dividend 4,900
85,202
,
Excess of Cash inflow over cash
outflow (Net increase in Cash
balance)
9,656
94,858 94,858
Cash Flow Statement : 405
Cash Flow Statement
Amount
Particulars
Rs.
Opening balance of Cash 505
Add: Net Cash Inflow during period - II 9,656
:. Closing balance of Cash 10,161

D1ustration: 5.19
The financial position of XYZ Co., as it stood on the 31st December, 1984 is set out
below.
Amount Amount
Liabilities Assets
Rs. Rs.
Accounts Payable 8,000 Cash 500
Loan from Bankers 12,500 Accounts Receivable 1,500
Capital 5,500 'Inventories 18,000
Equipments 6,000
26,000 26,000
The company adopted the following goals and forecasts as a basis for planning its
activities for the year ending 31st December, 1985.
1. An end of the year current ratio of 2: 1.
2. An end of the year cash balance of Rs. 1,500.
3. An end of the year Receivable balance equivalent to a 30-day collection period on
average sales assuming 360 selling days in a year.
4. The reduction of the Bank Loan consistent with the other plans and the liquidation in
full of the Accounts Payable.
5. Drawing or additional investment to account for the resulting fund balance.
6. A sale forecast of Rs. 20,000 with a gross margin of 50%.
7. Expenses estimated at Rs. 5,000 excluding depreciation. No other purchase or
manufacture during the year will occur. .
a. Draw up a Proforma Balance Sheet as at the end of 31-12-85 based on the
above targets.
,b. Prepare a Proforma Cash Flow Statement using the Actual Balance Sheet in
comparison with the Projected Balance Sheet to determine how much outside
finance it needs and the source from which it could secure this finance.
[ICWA (Fin), December 1985J
Management Accounting : 406

Solution:
1. Estimated Sales Revenue for 1985 = Rs.20,OOO
Gross Profit =50% = Rs.IO,OOO This lis. 5,000 Profit may
Less: Other Expenses (excluding Depreciation) 5,000 also be considered as Cash
Net Profit (to Balance Sheet) 5,000 from Operation.

2. Accounts Payable =0 (as this is to be liquidated in full)


3. Year-end Cash balance = Rs. 1,500

4. Year-end Accounts Receivable = [RS. 20,000 x 301 = Rs. 1,667


'.
5. Current RatIo = 2: I
360 days j
Current Assets: Current Liabilities:
Cash Rs.l,500 Loan from Bankers 6,584**
Accounts Receivable 1,667 Accounts Payable o
Inventories 10,000* 6,584
13,167

Note: * Comparison of Inventory with the estimated Sales Revenue reveals that the
Inventories are much higher and they may be reduced to 50% of Sales
Revenue, i.e., to Rs. 10,000.
** Current Assets come to Rs. 13,167 and to have 2 : 1 Current Ratio,
Currents Liabilities which include only Loan from Bankers should be
equal to Rs. 6,584 (i.e., 50% of Current Assets).
Comparative Balance Sheet
31.12.84 31.12.85 31.12.84 31.12.85-
Liabilities Assets
Rs. Rs. Rs. Rs.
Capital 5,500 7,583* Equipment 6,000 6,000
Profit & Loss Account 0 5,000 Current Assets:
Current Liabilities: Inventories 18,000 10,000
Accounts Payable 8,000 0Accounts 1,500 1,667
Loan from 12,500 6,584 Receivable 500 1,500
Bankers 26,000 19,167 Cash 26,000 19,167
..
(*balance, assumed to have rmsed Rs. 2,083 of additIonal capItal)
Cash Flow Statement : 407

Cash Flow Statement of XYZ Co., for the Year ended December 31,1985
Amount Amount
Sources Uses
Rs. Rs.
Opening Balance of Cash 500 Increase in Accounts Receivable 167
Cash from Operation 5,000 Decrease in Accounts Payable 8,000
Decrease in Inventories 8,000 Decrease in Loan from Bankers 5,916
Issue of Shares 2,083 Closing Balance of Cash 1,500
15,583 15,583

Illustration: 5.20
A and B are equal partners since 1984. Their books and records showed the
following balances as on 1st January, 1989:
Building Account: Rs. 1,20,000 Stock at cost: Rs. 2,40,000
Due from customers: Rs.2,70,000 Bills payable: Rs. 32,000
Bank overdraft: Rs. 1,15,000 Machinery Account: Rs. 1,80,000
Furniture Account: Rs. 50,000
Advance for machinery (Machinery installed in July 1989) Rs. 40,000
Due to suppliers (including Rs. 10,000 for purchase of Furniture in. December 1988):
Rs.l,96,OOO.
The following further information is furnished:
(a) Cost of machinery delivered and installed in July 1989 was Rs. 1,20,000.
(b) Sales for the year 1989 were Rs. 2,00,000 per month and of which 15% were cash
sales. The firm maintains a steady gross profit rate of 25% on sales. Cash
purchases amounted to Rs. 50,000.
(c) Collection from Debtors: Rs. 22,00,000
Payments to Creditors (including liability for Furniture): Rs. 15,40,000
Discount allowed to Debtors: Rs. 10,000
Discount received from Creditors: Rs. 12,000
(d) Bills payable accepted: Rs. 1,00,000
Bills payable discharged: Rs. 90,000
Partners' Drawings: A: Rs. 50,000
B: Rs. 50,000
(e) Stock as on 3tst December, 1989: Rs. 2,00,000
(0 Cash and Bank Balance as on 31-12-1989 amounted to Rs. 4,20,000 and there
was no Bank Overdraft. The figures of Cash and Bank on 1st January, 1989 are
not available.
(g) Net profit for the year may be assumed Rs.l,80,000 after providing for
depreciation on machinery Rs.30,000 and Furniture Rs. 5,000.
Management Accounting : 408

Prepare: (1) The statement showing sources and application of funds, for the year ended 31 st
December, 1989, showing separately statement of changes in Working Capital.
(2) Cash flow statement for the year ending 31st December, 1989. Show your
workings.
lCA (Fin), May 1990J
Solution:
a. Net Profit Rs. 1,80,000
Add: Depreciation on: Machinery 30,000
Furniture 5,000 35,000
:. Cash/Fund from Operation 2,15,000

b. Balance of Debtors at the beginning of the year (1-1-1989) = Rs.2,70,000


Add: Credit Sales made during tl).e year (Rs. 2,00,000 x 12 x 85%) 20,40,000
23,10,000
Less: Discount allowed to Debtors 10,000
23.00,000
Less: Collection from Debtors 22,00,000
:. Closing balance of Debtors 1,00,000
c. Cost of goods sold [(Rs. 2,00,000 x 12) x (1- 25%)] Rs. 18,00,000
Add: Cost of Stock on 31-12-1989 2,00,000
20,00,000
Less: Opening Stock 2,40,000
Total Purchases 17,60,000
Less: Cash Purchases 50,000
:. Credit Purchases 17,10,000

d. Opening Balance of Bills Payable Rs. 32,000


Add: Bills accepted (creditors) 1.00,000 1,32,000
Less: Bills discharged 90,000
Closing balance of Bills Payable 42,000
Cash Flow Statement: 409

e. Opening balance of Creditors (excluding Rs. 10,000


for purchase of Furniture) Rs.l,86,000
Add: Credit Purchases 17,10,000
18,96,000
Less: Discount Received 12,000
18,84,000
Less: Payments to Creditors (excluding for Furniture ofRs. 10,(00) 15,30,000
3,54,000
Less: Bills accepted 1,00,000
:. Closing balance of Creditors 2,54,000
Cash Flow Statement for the year ended December 31, 1989
Amount Amount
Sources Uses
Rs. Rs.
Opening Balance of cash Decrease in Bank Overdraft 1,15,000
(balancing figure) 2,22,000 Decrease in Liabilities (Furniture) 10,000
Cash from Operations 2,15,000 Drawings by Partners:
Decrease in Debtors 1,70,000 A: 50,000
Increase in Creditors 68,000 B: 50,000 1,00,000
Increase in Bill Payable 10,000 Purchase of Machinery
Decrease in Stock 40,000 (balance) 80,000
Closing Balance (of Cash) 4,20,000
7,25,000 7,25,000

Schedule of Changes in Working Capital


Amount (Rs.) Changes in Working
Particulars as on ... Capital (Rs.)
1.1.1989 31.12.1989
. Increase Decrease
Current Assets:
Debtors 2,70,000 1,00,000 0 1,70,000
Stock 2,40,000 2,00,000 0 40,000
Cash 2,22,000 4,20,000 1,98,000 0
(a) 7,32,000 7,20,000
Current Liabilities:
Creditors (excluding for Furniture) 1,96,000 2,54,000 0 58,000
Management Accounting : 410

Bank Overdraft 1,15,000 0 1,15,000 '0


i Bills Payable 32,000 42,000 0 10,000
(b) 3,43,000 2,96,000
Therefore, Working Capital (a - b) 3,89,000 4,24,000
Changes in Working Capital 3,13,000 2,78,000
Therefore, Net increase in Working Capital 35,000 35,000
4,24,000 4,24,000 3,13,000 3,13,000

Projected Funds Flow Statement of •••.•. for the Year ended December 31,1989
Amount Amount
Sources Uses
Rs. Rs.
Funds from Operation 2,15,000 Purchase of Machinery 80,000
Drawings by Partners 1,00,000
Increase in Working Capital 35,000
2,15,000 2,15,000

Limitations of Cash Flow Statement


Though Cash Flow Statement is an important managerial tool for evaluating short-
term financial position, it suffers from a few limitations. Hence, the analysts must have an
idea about these limitations so that they can interpret the results in the right perspective.
1. Cash Flow Statement fails to reveal certain practices pursued by the management to
manipulate the firm's cash position. The practice of post-poning the payment and/or
purchases helps the company to show a healthy liquidity position. Hence, it is said
that the Cash Flow Statement may not be able to reveal the short-term liquidity
position.
2. Cash Flow Statement cannot be used to replace either the Profit and Loss Account or
the Funds Flow Statement as each is expected to perform a separate set of functions.
In spite of the above limitations, the Cash Flow Statement serves as an important tool
in understanding the short-term financial position of the companies and to manage the cash
more efficiently and effectively.
Summary of the Chapter
The Statements - Cash Flow Statement and Funds Flow Statement are very useful
tools in the hands of both the internal and the external parties, particularly to the financial
analysts and the financial institutions which are extending loan facility to the corporate sector.
These statements provide figures of cash and fund generated by the operation during the year
and these figures are more reliable than the figures of profit (as revealed by the Profit and
Loss Account) as an index of the relative efficiency of operation. Because, the amount of
profit can be manipulated through the process of increasing or decreasing the amount of
vmous assets (particularly, fictitious assets) to be written off.
Cash Flow Statement: 411

In the light of the above, different aspects of Cash Flow Statement are discussed in
this chapter. The chapter begins with the introduction to Cash Flow Statement explaining the
meaning of Cash Flow Statement followed the similarities and differences between Funds
Flow Statement and Cash Flow Statement. Thereafter, the importance of Cash Flow
Statement is discussed. An exhaustive analysis of different sources and uses of cash is made
before discussing the procedure of preparing the Cash Flow Statement. A number of
problems are discussed followed by the identification of limitations of Cash Flow Statement.
Key-Terms to Remember
Fund Cash Flow
Cash Inflow Cash Outflow
Cash Flow Statement Actual Cash Flow
Notional Cash Flow Cash from Operation
Questions for SeIf;.Study
01. Define Fund and explain the cash concept of Fund.
02. What are the sources and uses of Cash?
03. Define Cash Flow Statement and distinguish it from Funds Flow Statement.
04. Distinguish between Funds Flow Statement and Cash Flow Statement.
[Bangalore Uni, B.Com, November 2001 and 2003, and Kuvempu Uni, B.Com, May 2002J
05. What is Cash Flow Statement? How it is different from Fund Flow Statement.
[Kuvempu Uni, B.Com, May and November 2001J
06. State the significance of Cash Flow Analysis. [Kuvempu Uni, B.Com, November 2002J
07. Define Cash Flow Statement and analyse its uses. [Kuvempu Uni, B.Com, May 2000J
08. List out the objectives of Cash Flow Statement. [Kuvempu Uni, B.Com, May 1999J
09. What is Cash Flow Statement? State its objects. [Kuvempu Uni, B.Com, November 2000J
10. Give the following format:
a. Statement showing Operating Cash Profit.
b. Cash Flow Statement. [Kuvempu Uni, B.Com, May 1999J
11. How do you compute the Cash from Operation?
12. What is Cash from Operation and how does it differ from Funds from Operation?
13. Explain the main differences between Cash Flow Statement and Funds Flow Statement.
14. How do you treat each of the following items in the Cash Flow Statement?
a. Purchase of Material on cash basis
b. Issue of Bonus Shares
c. Purchase of Material on credit basis
d. Changes in Bills Payable
e. Changes in Bills Receivable
Management Accounting : 412

15. Directors of Z Ltd., are facing the problem of working capital. They are not in a position
to co-ordinate the inflow and outflow of cash. Examine the existing management of
working capital and submit a report to the management including your findings and
recommendations to correct the situation. [Bangalore Uni., B.Com., November 2003]
16. "Cash Flow Statement is a Managerial Devise". Discuss and explain the objectives and
limitations of this statement. [Kuvempu Uni, B.Com, May 1999]
17. A Few Short-answer Questions:
a. State the objectives of preparing the Cash Flow Analysis.
[Bangalore Uni., B.Com., May 2000 and 2003]
b. Mention any four objectives of Cash Flow Analysis.
[Bangalore Uni., B.Com., November 2001]
c. State any two uses of Cash Flow Statement. [Bangalore Uni., B.Com., May 2001]
d. What is Cash Flow Statement? [Bangalore Uni., B.Com., May and November 2002]
18. Moon and Co., gives you its Balance Sheet 31st December 1984 and its projected Profit
Loss Account for 1985 as indicated below.
Balance Sheet as on 31st December, 1984
Liabilities Rs. Assets Rs.
Share Capital: Equity shares ofRs. 100 Goodwill 40,000
Each, fully paid 5,00,000 Machinery at Cost 6,00,000
Reserve and Surplus: Less: Depreciation 1,50,000 4,50,000
General reserve 1,00,000 Current Assets:
Profit and Loss Nc 40,000 Stock 2,00,000
Unsecured Loan: 12% debentures 1,00,000 Debtors 2,10,000
Current Liabilities and Provisions: Cash at Bank 90,000
ForGoods 80,000 Miscellaneous Expenditure:
For Expenses 20,000 Preliminary Expenses 10,000
For Taxation 1,00,000
Proposed Dividend on Equity Shares 60,000
to,OO,OOO to,OO,OOO

Projected Profit and Loss Account for the Year ending 31st December, 1985
Particulars Rs. Particulars Rs.
Opening Stock 2,00,000 Sales: Cash 4,00,000
Purchases 9,00,000 Credit 12,00,000
Wages 1,30,000 Stock 1,70,000
Manufacturing Expenses· 70,000 Miscellaneous Income 30,000
Cash Flow Statement: 413

Selling and Distribution Expenses 1,00,000 Profit on Sale of Machinery 7,000


Office and Admn. Expenses 60,000
Interest 12,000
Depreciation 75,000
Provision for Taxation 1,00,000
Goodwill written off 10,000
Proposed Dividend on Equity
Shares 70,000
Profit 80,000
18,07,000 18,07,000

Debentures are due for redemption on 31st December, 1985. The company proposes
to issue equity shares of the nominal value of Rs. 2,00,000 at a premium of 10%. Machinery
will be acquired for Rs. 75,000. The cost of machinery to be sold in 1985 is Rs. 60,000 with
depreciation provision of Rs. 30,000. It is expected that:
1. Sundry Debtors will be 5% more than that warranted by the period of one month.
2. Creditors for purchase will continue to extend two months' credit and manufacturing
expenses outstanding will be Rs. 10,000.
3. Tax liability up to 31-12-84 will be settled at Rs. 90,000.
You are required to:
(a) Prepare the company's projected balance sheet as on 31st December, 1985.
(b) Prepare the statement showing sources and application of funds, for 1985,
showing separately statement of changes in Working Capital.
(c) Cash flow statement for the year ending 31st December, 1985. Show your
workings. leA (Fin), November 1985J
19. From the following Balance Sheet of Anand Ltd., prepare a Cash Flow Statement.
1982 1983 1982 1983
Liabilities Assets
Rs. Rs. Rs. Rs.
Equity Share Capital 3,00,000 4,00,000 Goodwill 1,00,000 80,000
8% Redeemable Land and Buildings 2,00,000 1,70,000
Pref. Share Capital 1,50,000 1,00,000 Plant 80,000 2,00,000
Capital Reserve - 20,000 Investment 20,000 30,000
General Reserve 40,000 50,000 Sundry Debtors 1,40,000 1,70,000
P&LAlc 30,000 48,000 Stock 77,000 1,09,000
Proposed Dividend 42,000 50,000 Bills Receivable 20,000 30,000
Sundry Creditors 25,000 47,000 Cash in Hand 15,000 10,000
Management Accounting : 414

Bills Payable 20,000 16,000 Cash in Bank 10,000 8,000


Liability for Expenses 30,000 36.000 Preliminary
Provision for Taxation 40,000 50,000 Expenses 15,000 10,000
6,77,000 8,17.000 6.77,000 8,17,000

Notes:
1. A piece of land has been sold out in 1983 and the profit on sale has been credited to
capital reserve.
2. A machine has been sold for Rs. 10,000. The written down value of the machine was
Rs.12,000. Depreciation ofRs. 10,000 is charged on plant account in 1983.
3. The investments are trade investments. Rs. 3,000 by way of dividend is received
including Rs. 1,000 from pre-acquisition profit which has been credited to investment
account.
4. An interim dividend of Rs. 20,000 has been paid in 1983. lCA (Fin)]
Answers
18. Balance Sheet total = Rs. 12,80,000; Increase in Working Capital = Rs. 2,60,000; Total
of Funds Flow Statement =Rs. 4,95,000; Total of Cash Flow Statement =Rs. 7,90,000.
19. Cash Trading Profit =Rs. 1,79,000; Total of Cash Flow Statement =Rs. 3,89,000.
Chapter- VI

BUDGETARY CONTROL
Objectives: The important objectives of this chapter are:
• To introduce the Budgetary Control and to explain the terms Budget, Budgetin!:. 'md Budgetary
Control,
• To explain the basic aspects of Budgetary Control,
• To discuss the Objectives, Merits or Advantages and Demerits or Limitations of Budgetary Control,
• To explain the Importance and Preparational aspects of different kinds of Budgets, and
• To solve some practical problems.

Structure
Introduction
Meaning ana Definition of Budget, Budgeting and Budgetary Control
Organisation for Budgetary Control
o Budget Centres
o Organization Chart
o Budget Committee
o Budget Manual
o Budget Period
o Principal Budget Factor
• Objectives of Budgeting or Budgetary Control
• Classification of Budgets - Functional
o Sales Budget
o Production Budget
o Purchase Budget
o Production Cost Budget
o Labour Budget
o Other Budgets
o Cash Budget
Receipts and Payments Method
Adjusted Profit and Loss Account Method
Balance Sheet Method
o Master Budget
• Classification Budgets - Flexibility
o Fixed Budget
o Flexible Budget
Advantages or Merits of Budgetary Control System
• Demerits or Limitations of Budgetary Control System
• Illustrations
• Summary of the Chapter
• Key-Terms to Remember
• Notes and References
• Questions for Self-study
Management Accounting: 416
Introduction
There has been continuous and revolutionary changes in the industrial society. This has
been creating a number of problems to the business enterprises. Consequently, if the companies
fail to anticipate and recognize these changes, they will not be able to survive in the competitive
era which has been transforming rapidly from healthy and constructive competition to unhealthy
and destructive type of competition. Further, the companies must aim at achieving some degree of
growth on consistent and continuous basis. This growth rate is influenced, to a greater extent, by
the profit and profitability. It is, therefore, necessary for the companies to plan and work properly.
This has resulted in the companies planning for future and try their best to achieve this planned
result. This planning for future and trying to achieve the target result is necessary for both
survival and growth.
Meaning of Budget, Budgeting, Budgetary Control, etc
The word Budget is derived from a French word Bougette representing a leather pouch
into which funds were appropriated to meet the anticipated expenses. The word 'Budget'
therefore refers, in the case of business enterprises, to a plan in the form of a quantitative and
financial statement, of the firm about the work to be done by the executives and their officials.
The 'Chartered Institute of Management Accountants (CIMA), London has, therefore, defined
Budget as a rmancial and/or quantitative statement prepared and approved prior to a
dermed period of time, of the policy to be pursued during that period for the purpose of
attaining a given objective. It may include income, expenditure and the employment of
capital. I This is a comprehensive definition of J;ludget which spells out some of the important
features of a budget. Anyhow, it is a guide for the future. In order to plan for 'the future activities,
it is necessary to undertake an exhaustive survey of past events, present happenings and the future
things. Because, past experience of the company provides a good and sound base for the future ,
plan of action. Dr. Gupta has, therefore, rightly pointed out, past is the father of present and to
a greater extent, present is the guide of future. 2 On the basis of these, a forecast can be made
about the future. John R. Bertizel has, therefore, viewed a Budget as ••• a forecast, in detail, of
the results of an officially recognized rrogramme of operations based on the highest
reasonable expected operating efficiency. That means, the Budgets incorporate the forecasts
about the sales revenue, expenses, purchases, etc. Gordon and Shillinglaw have gone a step
further to state Budget ••• as a ••• basis for the subsequent evaluation of performance.4 In
brief, Budget is a plan of operations to be carried out during a specified future period and this plan
comprises of all aspects of business activities and summarizes the result of carrying out this plan.
Analysis of the above definition and meaning reveals four important and essential features of a
Budget. They are:
1. Budget is a comprehensive and co-ordinated plan of action, and it is based on the
objectives which the enterprises aim to achieve during the plan or Budget Period;
2. Budget is expressed in quantitative and financial terms;
3. Budget is a plan for the operations and resources of the company; and
4. Budget is always for a future specified period.
Budgetary Control: 417
Budgets are prepared on the basis of the forecast. A forecast is a statement of probable
events.s That means, it is a mere estimate of what is likely to happen. It is, therefore, an
assessment of probables. Budgeting starts after the completion of Forecast. Therefore, Budget is
broader than Forecast. Because, it (Le., Budget) involves even an element of control. Further,
Budgets are prepared after co-ordinating Budgets for various divisions of the company. For
instance, the Sales Department forecasts the demand for the product at 2,000 units. Sales Budget
is prepared on the basis of this forecast after taking into account the production capacity.
Budgeting refers to the procedure followed to prepare the Budgets. Budgeting, therefore,
involves an analysis carefully and systematically of different. business operations .with the sole
objective of preparing specific plans (viz., Budgets) for the future. William 1. Vatter has,
therefore, said, Budgeting is a kind of future tense accounting in which the problems of
future are met on paper before the transactions actually occur.6 However, both Budget and
Budgeting are, sometimes, used synonymously to denote budget. For instance, George R Terry
has said, Budgeting is the principal tool of planning and control offered to management by
accounting functions. 7
But Budgetary Control is a broader one as it includes Budgets and also an additional
step of comparing the actual results accomplished with the budgeted results. The word control in
Budgetary Control refers to a systematic and an organized effort to keep the costs under control
(Le., at the minimum level) and the revenue at the higher level. It, therefore, aims at ensuring
mobilization and utilization of input factors and resources more efficiently, productively and
effectively to accomplish the targets set for the period. This control is exercised through a process
called comparison - comparing the actual with the budgeted. Budgetary Control therefore refers
to a process of rmding out what is being done and comparing actual results with the
corresponding budget data in order to approve accomplishments or to remedy differences
by either adjusting the budget estimates or correcting the cause of differen~es. The CIMA,
London has defined the Budgetary Control as the establishment of a policy and the continuous
comparison of actual with budgeted results either to secure by individual actioil the
objective of that policy or provide a basis for its reunion. The following steps are, therefore,
involved in a Budgetary Control system: (1) Preparation of Budgets, (2) Measurement of actual
performance at the end of the budget period, (3) Comparison of actual performance with the
budgeted performance to find out whether the company (or a part of it Le., department, section,
etc.,) has achieved the target set in the Budget, and (4) Analysis of the reasons for not achieving
the target so that corrective measures can be taken.
Organisation for Budgetary Control
In order to introduce Budgetary Control System, it is necessary for the organizations to
consider and decide about the following. These can, therefore, be called pre-requisites for the
introduction of an effective Budgetary Control System. The important aspects to be considered
are identified below followed by a brief analysis of the same.
1. Budget Centres 4. Budget Manual
2. Organization Chart 5. Budget Period
3. Budget Committee 6. Principal Budget Factor
Management Accounting: 418
Budget Centres
Budget Centre represents a part of the organization for which Budget is to be prepared. It
may be in the form of a division or department or section or process or any other convenient form.
CIMA, London has defined Budget Centre as a section of the organization of an undertaking
dermed for the purpose of budgetary control. That means, in an undertaking one can find a
number of Budget Centres and it will facilitate the preparation of various Budgets for various
Budget Centres with the help of the heads of these centres. For instance, Production Department
may be reckoned as a Budget Centre. The Budget for the Production Department (viz.,
Production Budget) may be prepared in consultation with the Production Manager. In the same
way, Sales Department may be recognized as a Budget Centre and the Sales Budget may be
prepared and finalised after consulting sales manager as it is this manager who has to achieve this
sales target. It is, therefore, necessary to determine the number of Budget Centres clearly
demarcating their area of operation and coverage.
Organization Chart
In order to introduce a Budgetary Control System, the company should have a clear
organizational structure explaining c;learly the position of each member in the organizational
hierarchy and his relationship with other members of the Budget Committee. Of course, authority
and responsibility of members differ from one company to another. Anyhow, a simple
Organizational Chart is presented below to show clearly the type of Budgets to be prepared by them.
Organization Chart

Chairman (Chairman of the company and also the

1
Budget Officer
Chairman of the Budget Committee)

(Since he is the person entrusted with the


responsibility of administering the Budgets
and since he is required to deal with
voluminous figures, he should be a person
with Accounting knowledge)

Purchase "
Personnel Production SaIes Accounts} Members

MTeT M all ager

"
M
T e
, l
M age
,
Offi~er

,
of Budget
Committee

Purchase
Budget,
Material
Budget
Labour
Budget
Production
Budget,
Plant Utilization
Budget, etc
Sales Budget,
Advertising
Budget,
Sand D Cost
Budget, etc
Cost
Budgets,
MaSll'
, Budget
} Budgets
to be
Prepared
Budgetary Control; 419

While specifying the duties and responsibilities of the members, it is necessary to ensure
that there is a clear cut division of responsibility and authority, and there is no overlapping. This
will create an environment wherein all the persons know their responsibility and work as a team.
and try to achieve the desired target result.
Budget Committee
This is the Committee comprising of all executives in charge of major functions and
entrusted with the preparation and finalisation of Budgets for various Centres and also for the
whole company called Master Budget. Usually, the Chief Executive of the organization will be
the Chairman of the Committee and the Accounts Officer will function as Budget Officer. All the
functional managers and important executives will be the members of the Committee. But in case
of small scale organizations, the entire work will be entrusted to an individual (normally, chief of
the organization) and he will be assisted by the accounts officer. The Committee will request the
functional managers to prepare, and submit to it, Budgets for their respective departments. Mter
the receipt of these Budgets, the Committee will make a detailed analysis of the same and make
necessary adjustments in consultation with the functional managers. Then it will co-ordinate all
the Budgets and prepare the Master Budget. Once these Budgets are approved by the Committee,
they will be implemented. Further, it is also entrusted with the responsibility of monitoring the
work of different Budget Centres to ensure that they are working in accordance with the Budgets.
They also receive the reports of actual performance from various Centres, compare them with the
budgeted performance and find out how far the Centres have achieved their targets. On the basis
of this, the Committee will advise the Budget Centres about the remedial measures, if necessary.
Budget Manual
Budget Manual is a written document which guides and helps the executives in the
preparation and execution of various Budgets. Further. it spells out the duties and responsibilities
of various executives and also the inter-relationships among themselves. ICW A. has therefore
defined Budget Manual as ••• a document which sets out the responsibilities of the persons
engaged in the routine of, and the forms and records required for, budgetary control. The
following aspects are normally covered in the Budget Manual.
1. A brief explanation about the objectives, benefits and the principles of Budgetary Control
System;
2. A clear and unambiguous explanation of the procedure to be followed not only for the
preparation of Budgets but also throughout the system.
3. An Organization Chart clearly spelling out the duties and responsibilities of each member of
the management team and relationships with each other. This is very essential for a manager
to know precisely whorp he should obey and whom he can command, and also his position in
the organization;
4. Purpose, format, number of copies and frequency of each report and statement;
5. Spelling out of accounts code in use, etc.
Management Accounting : 420

The Budg~t Manual is normally prepared in the form of loose-leaf so that alterations,
modifications, etc., can easily be made whenever the changes are necessary. It also facilitate the
handing over of the relevant portion to the executives whenever they ask for the same.
Budget Period
One of the pre-requisites for the preparation of Budgets is the decision about the Budget
Period. This Budget Period represents the length of the period for which the Budget is to be
prepared. Though it is influenced by a number of factors, the nature of business and the degree of
control which the company wishes to exercise playa decisive role in deciding the Budget Period.
Normally, the business enterprises prepare the Budgets for a period of more than one year, for one
year and for less-than one year (say, quarterly, or monthly or weekly). A company may have all
the three tYPL'" of Budgets (which is normally observed in the corporate sector) and another may
have only one or two. There is no hard and fast rule. It depends upon the nature of activity for
which the business organization is intending to prepare the Budget. However, the companies
normally prepare the following three types of Budgets.
1. Weekly, Monthly and/or Quarterly Budgets which fall into the category of Short-term
Budgets are normally prepared and adopted by the companies for the purpose of
exercising control. These Budgets are necessary for watching progress in the actual
performance against the budgeted performance. This helps the companies to ensure that
the actual performance is progressing as bUQgeted and to take early corrective measure if
there is any deviation;
2. Annual Budgets which coincide with the Financial Accounting year are prepared for
operating activities such as sales, purchase, production, etc. When Annual Budgets are
adopted, the Budget Period is usually the fiscal year, i.e., the financial year commencing
from April 1 in a year and ending with March 31 of the following year (e.g., Aprill, 2004
- March 31, 2005);
3. Long-term Budgets for three to five years are prepared for capital expenditure
programmes like expansion and modernization of the company, introduction of new
products. undertaking of new projects, etc.
Principal Budget Factor
Principal. Budget Factor which is also called limiting factor, key factor or governing
factor is a. factor which dominates the business operations and which acts as an obstacle or
impediment in accomplishing the desired result specified in the company's Budget. For instance,
assume that the company can easily sell 1,00,000 units of its product but the availability of a raw-
material is sufficient to produce only 65,000 units. In this case, raw-material acts as the Principal
Budget Factor as the Budget is to be prepared keeping the availability of raw-material in mind.
CIMA. o '.llIldon, has, therefore, defined limiting factor as the factor in the activities of an
undertaking which at a particular point in time or over a period will limit the volume of
output. Therefore, the Budget pertaining to the key factor should be prepared first and other
Budgets should be prepared in the light of influence of Principal Budget Factor. In the light of the
Principal Budget Factor, all Budgets are to be co-ordinated if a company plans to reap the full
benefits of Budgetary Control System. That means, the relevant Budgets are to be prepared and
integrated with others only after the identification of the Principal Budget Factor. Demand for the
Budgetary Control: 421

product, production capacity, skilled labour force, raw-material, etc., are examples to Key Factor.
The Limiting Factor, in majority of the cases, is of temporary nature. That means, this can be
overcome, over a period of time, by suitable managerial policies and actions. For instance, lack
of adequate demand is the key factor for majority' of the companies and this can be overcome, to a
greater extent, by taking appropriate steps to promote the sales. In the same way, production
capacity can be increased by expanding the plant. Another important point is the presence of two
or more Limiting Factors in a company. In this case, it is necessary to analyse them properly to
find out the most dominant Key Factor. The figures in the statement presented below shed light
on this aspect.
Identification of Principal Budget Factor
Principal Budget
Case Particulars Remarks
Factor
1 Demand for the Product: 2,000 units
>--
Production Capacity: 2,800 units Demand Produce 2,000 units
oJ

2 Demand for the Product: 2,000 units }


Production Capacity: 1,400 units Production Capacity Produce 1,400 units

3 Demand for the Product: 3,000 units .....,


Availability of Raw-material sufficient
to produce: 2,500 units >-
Employees able to produce: 4,000 units
Raw-material Produce 2,500.units
~

From the foregoing analysis, it is obvious that the Budgets, to be realistic, are to be
prepared after considering the Limiting Factors. Otherwise, the Budgets will not reflect the
reality.
This way, a systematic attempt is to be made to introduce a scheme of Budgetary Control
System. Further, successful introduction and implementation of Budgetary Control System
depends upon the following. Hence, the following are the essentials of a successful Budgetary
Control.
1. The top management has to extend its full support to all the concerned at each stage;
2. Budgets should be, as far as possible, realistic. lJIal means, they should b~ attainable.
The goals should not be set at a very high level which is practically not attainable;
3. There should be clear cut distribution of responsibility among the Budget Centres
supported by delegation of adequate power and authority;
Management Accounting : 422

4. The company should educate the personnel about their responsibility and authority, and
therefore, it is necessary to introduce Responsibility Accounting System;
5. There should be a full participation of all the personnel irrespective of their cadre,
designation. position, etc. Because, the result depends upon team work;
6. Proper communication system should be adopted;
7. The company has to educate all the personnel about the need for, and the importance of,
Budgetary Control System. Because, the degree of co-operation extended by the
employees and executives depends upon the realization of importance of, and need for,
Budgetary Control System; and
8. There should be a flexibility in the Budgets so that necessary revisions and modifications
can be made whenever the need arises.
Objectives or Purpose of Budgeting
The following are the important objectives or purposes of Budgets and Budgeting.
1. To state clearly and unambiguously the targets for different Budget Centres and also for
the whole company. That means, the Budgets have to state explicitly what the firm
expects to achieve during the Budget Period from different Budget Centres;
2. To communicate to different· heads of Budget Centres about what they are required to
achieve during the Budget Period. This is necessary to avoid confusion and to ensure the
accomplishment of the target results; .
3. To provide a comprehensive plan of action in the form of guidance to departmental heads
to achieve the budgeted results; and
4. To provide a means for evaluating the performance of different Budget Centres. This can
be done by comparing the actuals with the budgeted results. On the basis of this
comparison, necessary corrective measures may be suggested.
In brief, the Budgets aim at setting the target and to achieve this target by putting the
maximum effort, and utilizing the resources more productively, profitably and judiciously.
Classification of Budgets
There are a number of bases for classifying Budgets into two or more categories. But the
most important and widely used bases are functional classification and classification according to
flexibility. Hence, these are discussed in detail.
Functional Classification of Budgets
As is known, an organization has to undertake a number of activities to accomplish its
objective. Therefore, it has to perform a number of functions. For example, raw materials are to
be purchased, required human resource is to be recruited, production facilities are to be made
available, sales are to be made, etc. Therefore, functional budgets occupy important place.
On the basis of the functions that the Budgets or Budget Centres are meant to perform,
Budgets may be classified into a number of groups. A functional budget, therefore, presents the
details about the income and/or expenses appropriate to a particular function. Of course, the
Budgetary Control : 423

number of functional budgets differs from one organization to another depending upon the size,
number of products produced and sold, number of materials used, number of departments through
which the production is carried out, sales territories, etc. However, the following are considered
as the minimum Functional Budgets to be prepared by any company.
1. Sales Budget, 2. Production Budget,
3. Production Cost Budget. 4. Materials andlor Purchase Budget,
5. Labour Budget, 6. Overhead Budget,
7. R&D Budget, 8. Capital Expenditure Budget,
9. Cash Budget, and 10. Master Budget.
Sales Budget
Sales Budget is one of the important Functional Budgets. This is considered as an
important one as all other Functional Budgets such as Purchase Budget, Production Budget,
Personnel Budget, etc., are affected by, and depending upon, this Budget (viz., Sales Budget).
Sales Budget is primarily concerned with the forecasting 6f what the company can expect to sell
during the Budget Period. This expected sale is usually expressed in terms of both the physical
units and the monetary value. While preparing the Sales Budget, the sales manager has to
consider a number of relevant and influencing factors. Though it is very difficult to identify all
the relevant factors, the sales manager has to take into account the following factors. .
1. Analysis of past sales, 7. Changes in the market environment,
2. Production capacity, 8. Opinion expressed by the executives,
3. Budget factor, 9. Pricing Policy,
4. Analysis of market trends, 10. Corporate objectives,
5. Reports and the opinion of salesmen, 11. Seasonal variations,
6. Results of market research studies, 12. Sales promotion, etc.
After considering the above and a number of other factors, a Sales Budget may be
prepared. This Budget may be prepared on the basis of either the products (if the company is a
multi-product concern) or the sales territories (if it is selling its productls in more than one
territory) or the salesmen or the customers or the time factor (such as, monthly, weekly, quarterly
or six-monthly budgets).
Production Budget
After the preparation of Sales Budget, Production Budget is prepared. Production Budget
shows the number of units of each of the products of the company that must be produced during
the Budget Period to meet the demand for the products and to keep the units of finished goods at
the desired level at the end of the Budget Period. In order to find out the number of units to be
produced, it is necessary to take into account the opening and closing stocks of finished goods and
the sales volume. Because,
Management Accounting : 424

~
Units to be} _ Udgetej [DeSired ClOSing~ [Opening StOCk]
Produced - • Sales + Stock of Finished - of Finished
Volume Goods Goods
It is, therefore, said that the Production Budget is essentially a Sales Budget adjusted for
changes in inventory levels. While preparing the Production Budget, the production manager has
to consider the following.
1. Sales Budget (Le., estimated sales),
2. Key or Budget Factor,
3. Opening and Closing Stocks of Finished Goods,
4. Production Capacity of the Company,
5. Management policy about purchase or production of components, etc.
IDustration: 6.1
A manufacturing company submits the following figures for the first quarter of 1980.
Product X Product Y Product Z
1. Sales (in units): January 25,000 30,000 10,000
February 20,000 25,000 10,000
March 30,000 35,000 10,000
2. Selling Price per unit (Rs.) 10 20 40
3. Targets for 1st Quarter of 1981:
Sales Quantity increase (%) 20 10 10
Sales Price increase (%) Nil 10 25
4. Stock Position, January I, 1981:
Percentage of January 1981 Sales 50 50 50
5. Stock Pos.ition, March 31, 1981: 20,000 25,000 5,000
6. Stock Position, end January and February:
Percentage of subsequent months Sales 50 50 50
You are'required to prepare the Sales and Production Budgets for the 1st Quarter of 1981.
Show working clearly. lCA (Fin), November 1980J
Budgetary Control : 425

Solution:
Sales Budget for the First Quarter of 1981
Product X Product Y ProductZ Total
Month
Rate Value Rate Value Rate Value Value
1981 Units Units Units Units
Rs. Rs. Rs. Rs. Rs. Rs. Rs.
January 30,000 JO 3,00,000 33,000 22 7.26.000 11.000 50. 5,50.000 74.000 15,76.000
February 24.000 JO 2,40,000 27,500 22 6,05,000 11,000 50 5.50.000 62.500 13.95.000
March 36,000 JO 3,60,000 38,500 22 8,47.000 11,000 50 5,50,000 85,500 17,57.000
Total 90,000 JO 9.00.000 99,000 22 21,78,000 33.000 50 16.50.000 2,22,000 47.28.000

Production Budget (units) for the First Quarter of 1981


Product Particulars January February March Total
X Expected Sales 30,000 24,000 36,000 90,000
Add: Closing Stock 12,000 18,000 20,000 20,000
Total requirement 42,000 42,000 56,000 1,10,000
Less: Opi:ming Stock 15,000 12,000 18,000 15,000
Budgeted Production (Le., to be produced) 27,000 30,000 38,000 95,000

Y Estimated Sales 33,000 27,500 38,500 99,000


Add: Closing Stock l3,750 19,250 25,000 25,000
Total requirement 46,750 46,750 63,500 1,24,000
Less: Opening Stock 16,500 l3,750 19,250 16,500
Budgeted Production (Le., to be produced) 30,250 33,000 44,250 1,07,500
Z Estimated Sales 11,000 11,000 11,000 33,000
Add: Closing Stock 5,500 5,500 5,000 5,000
Total requirement 16,500 16,500 16,000 38,000
Less: Opening Stock 5,500 5,500 5,500 5,500
Budgeted Production (Le., to be produced) 11,000 11,000 10,500 32,500

Production Cost Budget


After the preparation of Production Budget, this Budget is prepared. Because, only when
the company knows the number of units to be produced, it can compute the production cost.
Production Cost Budget shows the cost of the production determined in the Production Budget.
Management Accounting : 426
That means, it shows the amount of expenditure expected to be incurred to produce the units
estimated in the Production Budget. It is very well known that the production cost (also known as
manufacturing cost or works cost) includes the cost of materials, labour cost and factory
overhead expenses (also known as works-on-cost). Factory overhead expenses are classified into
variable and fixed, and they are shown separately.
D1ustration: 6.2
Prepare a Production Budget for each month and a summarized Production Cost Budget
for the six months period ending December 31. 2005 from the following data of product X.
a. The units to be sold for different months are as follows:
2005 July: 1,100 2005 November: 2,500
August: 1.100 December: 2,300
September: 1,700 2006 January: 2,000
October: 1,900
h. There will be no work-in-progress at the end of any month.
c. Finished units equal to half the sales for the next month will be in stock at the end of each
month (including June 2005).
d. Budgeted production and production cost for the year ending December 31, 2005 are as
follows:
Production (units): 22,000 Direct Wages per unit: Rs. 4.00
Direct Material per unit: Rs. 10.00 Total Factory Overhead apportioned to Product: Rs. 88,000
[ICWA (Inter)]
Solution:
Monthly Production Budget for Six-month Period Ending December 31, 2005
Particulars July August Sep Oct Nov Dec Total
Sales Volume (units expected to be sold) 1,100 1,100 1,700 1,900 2,500 2,300 10,600
Add: Closing Stock of Finished Goods
(half of next month's sales) 550 850 950 1,250 1,150 1,000 1,000
.'. Total numher of units required 1,650 1,950 2.650 3,150 3,650 3,300 11,600
Less: Opening Stock of Finished Goods 550+ 550 850 950 1,250 1,150 550
:. Production 1,100 1,400 I 1.800 2,200 2,400 2,150 11,050
L
Note: * [Opemng Stock + Production - Closing Stock] = Sales Volume
.'. Production =[Sales Volume + Closing Stock - Opening Stock]
+ This is the closing stock of June 1979 =50% of sales of July 2005.
Budgetary Control : 427

Summarized Production Cost Budget for Six-month Period ending December 31, 2005
Total Cost
Cost per Unit
Elements of Costs (for 11,050 units)
(Rs.)
(Rs.)
Direct Material Cost (at Rs. 10 a unit) 1.10,500 10
Direct Wages (Rs. 4) 44,200 4

( Rs. 88,000
Factory Overhead 22,000 units x 11,050 units
J 44,200$ 4
Total 1,98,900 18
$ Assumed to be variable. If it is fixed, 50% of Rs. 88,000 (viz., Rs. 44,000) is to be charged.
Purchase Budget (Raw-material)
Once the Production Budget is prepared, it is necessary to determine the different inputs
required to carry out the production activities. One such input factor is the raw-material. The
Purchase Budget shows the number of units of materials (both direct and indirect) and services to
be purchased during the Budget Period. It may also incorporate the monetary value of the units of
materials to be purchased for producing the goods and services as per its Production Budget. In
the case of a company which purchases finished goods for resale, Purchase Budget of the
company will include the information about the number of units of finished goods to be purchased
during the Budget Period. While preparing the Purchase Budget, the purchase manager or the
materials manager has to take into account the following factors. .
1. Sales and Production Budgets,
2. Expected changes in the prices of Raw-materials,
3. Inventory Levels, Economic Ordering Quantity, etc.,
4. Storage Facilities,
5. Nature and availability of Raw-materials (seasonal or otherwise),
6. In case the company is engaged in the sale of goods manufactured by other companies,
then the information about the units of finished goods expected to be sold, opening and
closing stocks, etc.
IDustration: 6.3
The Sales Director of a manufacturing company reports that in the new year he expects to
sell 54,000 units of a certain product. The production manager consults the store-keeper and casts
his figures as follows.
Two kinds of raw materials, A and B are required for manufacturing the product. Each
unit of the product requires 2 units of A and 3 units of B. The estimated opening balance at the
commencement of the next year are: Finished Product: 10,000 units, A: 12.000 units. and B:
15,000 units. The desirable closing balances at the end of the next year are: Finished Product:
Management Accounting : 428
14,000 units, A: 13,000 units, and B: 16,000 units. Draw up a quantitative chart showing the
Material Purchase Budget for the next year. [ICWAJ
Solution:
Production Budget (Finished Product)
Particulars Units
Expected Sales Volume 54,000
Add: Desired Closing Stock 14,000
Total requirements 68,000
Less: Opening Balance 10,000
:. Production (i.e., units to be produced during next year) 58,000
Production = [Sales Volume + Closing Stock - Opening Stock]
Purchase Budget (Raw Materials A and B)
Raw Materials
Particulars
A (units) B (units)
Raw Materials required for producing 58,000 units: (A: 58,000 x 2) 1.16,000
(B: 58,000 x 3) 1,74,000
Add: Desired Closing Stock 13,000 16,000
:. Total requirements 1,29,000 1,90,000
Less: Opening Balance 12,000 15,000
:. Purchases to be made, next year 1,17,000 1,75,000
Purchases =[for Production + C10smg Stock - Openmg Stock]
mustration: 6.4
Peko Electronics is manufacturing for sale, four models of Television Sets. The major
components viz., Cabinet, High Voltage Transformer and the Speaker are bought out by the
0

company. Picture Tubes for three, out of the four models, are purchased from other firms. Four
cabinet styles (A, B, C and D), two kinds of transformers (X and V); three kinds of speakers and
three types of picture tubes are assembled in the following ways in the final product.
Model Cabinet Transformer Speakers Picture tubes
Stimdard A @ Rs. 200 X @ Rs. 200 5" cone at OWN
Rs.300
Deluxe B @ Rs. 300 X @ Rs. 200 5" cone at BEL@
Rs.300 Rs.l,200
Aristocrat C @ Rs. 500 Y @ Rs. 300 6" cone at BEL@
Rs.400 Rs.l,200·
Royal D @ Rs.700 Y @ Rs. 300 12" cone at TELETUBE@
Rs.600 Rs.l,600
Budgetary Control : 429
The company expected the following inventories in had on 1st July 1980:
Standard 46; Deluxe 73; Aristocrat 64: Royal 69.
Sub-assemblies:
Cabinet: A - 30; B - 40: C - 20; D - 25
Transformers: X - 31; Y - 17
Speakers: 5" Cone 27; 6" Cone 47: 12" Cone 18
Picture Tubes: OWN 20; BEL 17: TELTUBE 34
The Sales Manager estimates that sales for the quarter, July-September 1980 will be:
Model: Standard Deluxe Aristocrat Royal
Quantity: 200 600 500 300
The fotlowing inventory quantities have been budgeted for 30th September, 1980.
Finished Sets: 25 in each model
Sub-assembles:
Cabinets - 15 (each Model)
Transformers: 20 (each type)
Speakers: 30 (each type)
Picture tube: OWN - 30: BEL - -W: TELTUBE - 20.
You are required to prepare the Production and Purchase Budgets for the various items
stated above for the quarter July-September 1980. lCA (Fin), May 1980J
Solution:
Production Budget for the Quarter ending September 30, 1980
Models (units)
Particulars
. Standard Deluxe Aristocrat Royal
Estimated Sales (during the quarter) 200 600 500 300
Add: Closing Stock at the end of the quarter 25 25 25 25
Total Requirements 225 625 525 325
Less: Opening Stock 46 73 64 69
:. Budgeted Production (units to be assembled) , 179 552 461 256
Purchase Budget (Components) for the Quarter ending September 30,1980
Require- Total
Closing Opening Total Rate per Total
Com- Model ment for Require-
Model Stock Stock Purchase Unit Amount
ponents Type Production ment
(units) (units) (units) (Rs.) (Rs.)
(units) (umts)

Standard Cabinets A 179 15 194 30 164 200 32,800


Management Accounting : 430
Deluxe B 552 15 567 40 527 300 1,58,100

Aristocrat C 461 15 476 20 456 500 2,28,000

Royal D 256 15 271 25 246 700 1,72,200


5,91,100

Standard & Trans-


Deluxe formers
X 731 20 751 31 720 200 1,44,000
Aristocrat
& Royal Y 717 20 737 17 720 300 2,16,000
3,60,000

Standard &
Deluxe Speakers 5" cone 731 30 761 27 734 300 2,20,200

Aristocrat 6" cone 461 30 491 47 444 400 1.77,600


Royal 12" cone 256 30 286 18 268 600 1.60,800
5,58,600

Standard Picture
Tubes OWN* - - - - - - -
Deluxe &
Aristocrat BEL 1013 40 1,053 17 1,036 1,200
12,43,200
Royal TELE- 256 20 276 34 242 1,600 3,87,200
TUBE
16,30,400
Grand Total 31,40,100

* For Standard Model, the company uses its own picture tube. Hence, It IS not consIdered.
However, the company has to produce 1~9 picture tubes [i.e., 179 + 30 - 20].
Labour Budget
This Budget shows the number of employees and/or the number of labour hours (skilled,
semi-skilled and unskilled for both production, administration, and selling and distribution)
required to produce and/or sell the budgeted output, and/or budgeted sales. While preparing this
Budget, it is necessary to consider the budgeted output and sales, capital expenditure programmes,
research and development activities, etc. This Budget may also incorporate the monetary value.
Therefore, appropriate remuneration rates are to be used. These rates should include the provision
for possible changes in the rates due to new wage agreement, enhance in the admissible
allowances, etc.
Illustration: 6.5
The direct labour requirements of three of the products manufactured in a factory, each
involving more than one labour operation, are estimated as follows.
Direct labour hours per unit (in minutes):
Budgetary Control: 431

Products
2 3
Operation: 18 42 30
2 12 24
3 9 6
The factory works 8 hours per day, 6 days in a week. The budget quarter is taken as 13
weeks and during a quarter. lost hours due to leave and holiday and other causes are estimated to
be 124 hours. The budgeted hourly rates for the workers manning the operations 1. 2 and 3 are
Rs. 2.00, Rs. 2.50 and Rs. 3.00 respectively. The budgeted 'iab of the products during the
quarter are: Product - 1: 9,000 units; Product - 2: 15,000 units; and Product - 3: 12.000 units.
There is a carry over of 5,000 units of Product 2 and 4.000 units of Product 3. and it is
proposed to build up a stock at the end of the budget quarter as follows: Product - 1: 1.000 units;
and Product - 3: 2,000 units. Prepare a Man-power Budget for the quarter showing for each
Operation, (i) direct labour hours (ii) direct labour cost and (iii) the number of workers.
[ICWA (Fill), JUlle 1980)
Solution:
Production Budget for a Quarter
Product (units)
Particulars
I 2 3
Budgeted Sales 9,000 15,000 12.000
Add: Closing Stock at the end of the quarter 1.000 - 2,000
Total Requirement 10,000 15,000 14,000
Less: Opening Stock - 5,000 4,000
:. Budget Output (i.e., units to be produced) 10,000 10,000 10,000

Direct-Labour Budget (for the Quarter)


Operation (Hours)
Particulars Total
1 2 3
Number of Labour Hours required to produce 10,000
units each of Product: 1 3,000 - 1,500 4,500
2 7,000 2,000 1,000 10,000
3 5,000 4,000 - 9,000
:. Total Labour Hours to be worked 15,000 6,000 2,500 23,500
Budgeted Hourly Rate (Rs.) 2 2.5 3
:. Direct Labour Cost (Le., Hourly Rate x Total Labour
Hours) (Rs.) 30,000 15,000 7,500 52,500
Management Accounting : 432
Number of Employees required per Quarter* 30 12 5 47
15,000; 6,000; 2,500; 23,500J
[
500 500 500 500

Note: * Number of EmPlOyees} _ Number of Labour Hours


Labour Hours per Employee
J
Required - [
per Quarter

Calculation of Labour Hours expected to work during the quarter by an employee:


Maximum Labour Hours per employee for the quarter [13 weeks x 6 days x 8 hours] 624
Less: Number of Labour Hours expected to lose during the quarter 124
:. Number of Labour Hours for which an employee will work during the quarter 500
In addition to the Budgets discussed above, a number of other Functional Budgets are also
to be prepared, These include:
(1) Production Overhead Expenses Budget It shows the amount of production
overhead expenses expected to be
incurred to produce the budget output.
(2) Administrative Overhead Expenses Budget It shows the probable expenses
pertammg to top managerial and
supervisory functions.
(3) Selling and Distribution Overhead
Expenses Budget It presents the information in detail
about the probable expenditure to be
incurred to promote the sale of goods
and services, and for distribution.
(4) Plant Utilization Budget This Budget estimates the number of
machine hours (or in any other
convenient form) required for producing
the budgeted output as set out in the
Production Budget.
(5) Research and Development Budget This Budget presents the details about
the limits within which the research and
development activities are to be carried
out during the Budget Period.
(6) Capital Expenditure Budget It shows the details about the future
capital expenditure programmes which
the company intends to undertake in
future. Further. it presents information
about the probable capital to be
Budgetary Control : 433

employed on the projects during the


Budget Period. This Budget is normally
prepared for a long period.
Cash Budget
Cash Budget forecasts both the inflow and outflow of cash during the Budget Period.
Since the corporate functions cause the flow of cash. this Budget is prepared after the preparation
of all the Functional Budgets. This Budget is necessary as the company has to know the sources
from which the cash is expected to flow into the company and also the activities for which this
cash is required. Because, cash iHequired for the purpose of meeting its current cash obligations.
If a company fails to meet its cash requirements, it will impair the reputation of the company.
Therefore, this Budget is prepared. It is prepared on the basis of detailed estimates of both cash
receipts and cash disbursements. Further, time-lag in credit transaction is also taken into
consideration while preparing the Cash Budget. Though this Budget is prepared for the Budget
Period, normally companies prefer to split the Budget Period so that Weekly. Monthly and
Quarterly Cash Budgets can be prepared. This is necessary for controlling purposes. Three
important methods are available for preparing the Cash Budgets. They are (1) Receipts and
Payments Method. (2) Adjusted Profit and Loss Account Method, and (3) Balance Sheet Method.
Receipts and Payments Method
Under this method, Cash Budget is prepared on the basis of all cash receipts (from
different sources) and all cash payments (to different parties and purposes) that are expected to
take place during the period. Therefore, cash requirements of all the Functional.Budgets are to be
considered. All expected cash receipts are added to the opening balance of cash and from this
aggregate. all expected cash payments are subtracted to arrive at the balance of cash at the end of
the Budget Period. Since this method considers only the anticipated cash receipts and payments,
it is implied that all accruals and adjustments are not considered for preparing the Cash Budget.
Special attention is to be given to the time lag in credit transactions.
Illustration: 6.6
Prepare Cash Budget for January-June from the following information.
(a) The estimated sales and expenses are as follows (Rs):
Nov Dec Jan Feb Mar Apr May June
Sales 2,00.000 2,20,000 1,20,000 1,00,000 1,50,000 2.40,000 2,00,000 2,00,000
Wages &
Salaries 30,000 30,000 24,000 24,000 24,000 30,000 27,000 27,000
Misc.
Expenses 27.000 27.000 21,000 30,000 24,000 27,000 27,000 27,000

(b) 20% of the sales are on cash and the balance on credit.
Management Accounting : 434
(c) The firm has a gross margin of 25% on sales.
(d) 50% of the credit sales are collected in the month following the sales, 30% in the second
month and 20% in the third month.
(e) Material for the sales of each month is purchased one month in advance on a credit for
two months. .
(t) The time lag in the payment of wages and salaries in one-third of a month and of
miscellaneous expenses one month.
(g) Debentures worth RsAO,OOO were sold in January.
(h) The firm maintains a minimum cash balance of RsAO,OOO. Funds can be borrowed at
12% per annum in the multiples of Rs.1,OOO, the interest being payable on monthly basis.
(i) Cash balance at the end of December is Rs.60,OOO.
leA (Fin), November 1989J
Solution:
Calculation of Collection from Customers (Rs)
Particulars January February March April May June
Cash Sales (20%) 24,000 20,000 30,000 48,000 40,000 40,000
Collection :
50% of Credit Sales in the following month 88,000 48,000 40,000 60,000 96,000 80,000
30% in the second month 48,000 52,800 28,800 24,000 36,000 57,600
20% in the third month 32.000 35,200 19,200 16,000 24,000

:. Total collection 1,60,000 1,52,800 1,34,000 1,51.200 1,88,000 2,01,600


Payments to Suppliers of Raw-
Materials: Purchases (75%
of Sales) are made one month in
advance on two-month credit
basis. (*2,20,000 x 75%) 1,65,000* 90,000 75,000 1,12,500 1,80,000 1,50,000
Wages and Salaries:
2/3 in the same month 16,000 16,000 16,000 20,000 18,000 18,000
1/3 in the following month 10,000 8,000 8,000 8,000 10,000 9,000

26,000 24,000 24,000 28,000 28,000 27,000

Six-Monthly Cash Budget (January to June)


Month (Rs)
Particulars
January February March April May June Total
Opening balance of Cash 60.000 42,000 59,800 64,800 51,500 40,500 60,000

Add: Receipts from


Customers 1,60,000 1,52,800 1,34,000 1.51,200 1,88,000 2,01,600 9,87,600

Sales of Debentures 40,000 --- --- --- --- --- 40,000


Bank Borrowings* --- --- --- --- 36,000 3,000 39,000
Budgetary Control : 435
Total (a) 2,60,000 1,94,800 1,93,800 2,16,000 2,75,500 2,45,100 11,26,600

Payments to Suppliers 1,65,000 90,000 75,000 1.12.500 1,80,000 1.50,000 7,72,500

Payment of Wages and


Salaries 26,000 24,000 24,000 28,000 28,000 27,000 1.57,000

Payment of
Miscellaneous Exps. 27,000 21,000 30,000 24,000 27,000 27,000 1.56,000
Interest on Bank Loan --- --- --- --- --- 360 360

Total (b) 2,18,000 1,35,000 1.29,000 1,64,500 2,35,000 2,04.360 10.85.860

:: Closing Balance 42,000 59,800 64,800 51,500 40,500 40,740 40,740

May June
* Total of Opening Balance and Cash
Receipts (excluding Bank Loan) 2,39,500 2,42,100

Total Cash Payments 2,35,000 2,04,360


Add: Minimum Cash Balance 40,000 2,75,000 40,000 2,44,360

:. Loan to be raised 35,500 2,260

Therefore, it has to raise Rs.36,OOO and Rs.3,000 of loan (Le., in multiples of Rs.l,OOO).
Adjusted Profit and Loss Account Method
This method of preparing the Cash Budget is similar to the preparation of Cash Flow
Statement. The procedure is, therefore, explained very briefly.
1. Preparation of Cash Budget begins with the opening balance of cash;
2. To the opening balance of cash, cash profit is to be added. This can be computed by
adding the following to the profit figure as reported in the Profit and Loss Account: (a)
Depreciation, (b) Outstanding Expenses, (c) Provision for Tax, Dividend, etc. These
items are to be added back as they have been debited to Profit and Loss Account to arrive
at the profit. But, these items do not cause outflow of cash.
3. To the opening balance of cash, capital receipts (e.g., issue of Shares, Debentures, etc.,
sale of Fixed Assets, raising of long-term loans) are also to be added. Further, decrease in
Current Assets except cash (as the reduction represents the sale of Current Assets and
therefore, the cash receipts) and increase in Current Liabilities (as this represents the
additional loans raised) are to be added to the opening balance of cash.
4. From the aggregate of the above, capital expenditure (such as, acquisition of Fixed
Assets), capital redemption (e.g., redemption of Redeemable Preference Shares,
Debentures, etc.,), repayment of long-term loans, increase in Current Assets except cash
(because, it represents the acquisition resulting in cash outflow) and reduction in Current
Management Ar counting : 436

Liabilities (ao:; it represents the discharge of the liability) are to be subtracted as they
involve the cao:;h outflow.
5. The balance [Le., (aggregate of first three) - (fourth item)] represents the closing balance.
Balance Sheet Method
Under this method. a Budgeted Balance Sheet will be prepared for the Budget Period
incorporating all the items of liabilities, capital and assets except cash. The balancing figure in
the Budgeted Balance Sheet denotes the Cash and Bank Balance or the Bank Overdraft depending
upon whether the capital and liabilities side exceeds the assets side or vice versa.
Master Budget
A budget which is prepared incorporating the summary of all the Functional Budgets is
called Master Budget. It normally comprises of Budgeted Profit and .Loss Account, and
Budgeted Balance Sheet. The Budget Committee prepares these on the basis of the Functional
Budgets. Once this Master Budget is prepared and approved by the Budget Committee, then it
will be reckoned as the target to be achieved and accomplished by the company during the Budget
Period.
llIustration: 6.7
SO Ltd., manufactures two products. A and B. The summarized Balance Sheet of the
company as at September 30th is as under.
Share Capital Rs. 12,00,000
Retained Income 96,000 Rs. 12,96,000
R~presented by Fixed Assets 12,00,000
Provision for Depreciation (3.00,000) 9,00,000
Inventories: Raw material 1,14.000
Finished goods 2,40,000 3,54,000
Debtors 90,000
Bank/Cash 60,000
14,04,000
Less: Creditors 48,000
Provision for Taxation 60,000 1,08,000
12,96,000
The following information is furnished to you for preparation of the budget for the
coming year ending September 30th.
(a) Sales Forecast: Product A, 24,000 units at Rs. 30 per unit.
Product B, 15,000 units at Rs. 40 per unit.
Budgetary Control : 437
(b) Raw Material: Product A Product B
Material X @ Rs. 3 per kg 2 kgs 4 kgs
Material Y @ Re. 1 per kg 1 kg 2 kgs
(c) Direct Labour: Department P: 2 hours @ Re. 1 per hour for A
1 hour @ Rs. 2 per hour for B
Department Q: 1 hour @ Rs. 3 per hour for A
1 hour @ Rs. 3 per hour for B
(d) Overheads: Fixed Overht:aJ~ per annum: Department P (Rs.) Department Q (Rs.)
Depreciation 48,000 12,000
Others 96,000 30,000
Variable Ov~rheads per hour 0.50 1.50
(e) Inventories (1) Raw Material: Opening Stock: X 36,000 kgs
Y 6,000kgs
Closing Stock: X 48,000 kgs
Y 12,000 kgs
(2) Finished Goods: Opening Stock: A 600 units
B 6,000 units
Closing Stock: A 6,600 units
B 3,000 units
(f) Selling, distribution and administration expenses are estimated at Rs. 1,80,900 p.a.
(g) The cost of raw material purchases, direct wages, factory overheads and selling,
distribution, and administration overheads of the year will be met in full in cash
during the year. The estimated position of debtors and creditors as on September
30th of the coming year is Rs. 1,50,000 and Rs. 48,000 respectively. Income tax
provision standing at the beginning of the year will be paid for during the year. The
rate of income tax is 50%. An equipment purchased at Rs. 1,20,000 will be paid for
during the year.
You are required to prepare the following for the coming year ending September 30th:
(a) Cost of Goods Sold Budget, (b) Cash Budget, and (c) Projected Balance Sheet as at
September 30th, in the same format as given in this question. lCA (Fin), November 1988J
Management Accounting : 438
Solution:
Production Budget of Products A and B (units)
Particulars A B
Sales Forecast 24,000 15,000
Add: Closing Stock 6,600 3,000
30,600 18,000
Less: Opening Stock 600 6,000
:. Production 30,000 12,000

Purchase Budget of Raw-materials X and Y


Material X (Rs. 3) Material Y (Re. 1)
Particulars Quantity Amount Quantity Amount
(Kgs) (Rs.) (Kgs) (Rs.)
For producing 30,000 units of A (2 kgs and 1 kg) 60,000 1,80,000 30,000 30,000
12,000 units of B (4 kgs and 2 kgs) 48,000 1,44,000 24,000 24,000
1,08,000 3,24,000 54,000 54,000
Add: Closing Stock 48,000 1,44,000 12,000 12,000
1,56,000 4,68,000 66,000 66,000
Less: Opening Stock 36,000 1,08,000 6,000 6,000
:. Purchases 1,20,000 3,60,000 60,000 60,000

Computation of Material and Labour Cost per Unit:


A (Rs.) B (R,s.)
Material Cost per unit: X (2 kgs x Rs. 3; 4 kgs x Rs. 3) 6 12
Y (1 kg x Re. 1; 2 kgs x Re. 1) 1 2
--
7 14
Labour Cost per unit: Department P: 2 hours x Re. 1 2
1 hour x Rs. 2 2
Q: 1 hour x Rs. 3 3
1 hour x Rs. 3 3
5 5
Budgetary Control : 439

Budgeted Material and Labour Cost Budget


Product (Rs.)
Total
Particulars A (30,000 B (12,000 (Rs.)
units) units)
Material Cost: X (@ Rs. 6 and Rs. 12) 1,80,000 1,44,000 3,24,000
Y (@ Re. 1 and Rs. 2) 30,000 24,000 54,000
2,10,000 1,68,000 3,78,000
Labour Cost: P (@ Rs. 2 and Rs.2) 60,000 24,000 84,000
Q (@ Rs. 3 and Rs. 3) 90,000 36,000 1,26,000'
1,50,000 60,000 2,10,000
:. Total of Material and Labour Cost 3,60,000 2,28,000 5,88,000

Direct Labour Hours Budget


Department (hours)
Particulars
P Q
Product: A (30,000 units at 2 hours and 1 hour) 60,000 30,000
B (12,000 units at 1 hour and 1 hour) 12,000 12,000
72,000 42,000

Computation of Departmental Overheads Absorption Rate


Departments Total
Particulars
P (Rs.) Q (Rs.) (Rs.)

Depreciation 48,000 12,000 60,000


Others 96,000 30,000 1,26,000
Total Fixed Costs 1,44,000 42,000 1,86,000
Variable Overheads (Hourly Rate x Number of Labour Hours)
(0.5 x 72,000; 1.5 x 42,000) 36,000 63,000 99,000
1,80,000 1,05,000 2,85,000
:. Departmental Overhead Absorption Rate per Hour:

. 1 Rs. 1,44,000
FIxed = 72,000 hours
l r~ Rs. 42,000 ~l
42,000 hours Rs.2 1 3
Management Accounting : 440

Variable 0.5 1.5 2


2.5 2.5 5

overh ead E xpenses Chargeable t 0 P roducts


Products (Rs.)
Particulars
A B
Department P: 60,000 and 12,000 hours @ Rs. 2.5 1,50,000 30,000
Q: 30,000 and 12,000 @ Rs. 2.5 75,000 30,000
2.25,000 60,000

Va Iueof Year-end I fF
nventory 0 'lOIS
. h ed G 00dS (Rs) .
A B
Particulars Total
(6,600 units) (3,000 units)
Material Cost (@ Rs. 7 and Rs. 14) 46,200 42,000 88,200
Labour Cost (@ Rs. 5 and Rs. 5) 33,000 15,000 48,000
Overhead Expenses: (@ Rs. 7.5 and Rs. 5) 49,500 15,000 64,500
1,28,700 72,000 2,00,700

.
Cost 0 f G00dS S0 Id BU d1get (Rs )
Products (Rs) Amount
Particulars
A B Rs.

Cost of Opening Stock of Finished Goods (given in Balance


Sheet) 2,40,000
Add: Cost of current production:
Material Cost 2,10,000 1,68,000 3,78,000
Labour Cost 1,50,000 60,000 2,10,000
Overhead Expenses 2,25,000 60,000 2,85,000
Manufacturing Cost of Goods available for sale 11,13,000
Less: Cost of Closing Stock of Finished Goods 2,00,700
:. Cost of Goods Sold 9,12,300
Budgetary Control: 441
Overhead Expenses per unit of Output:
A B
Number of Labour Hours per unit: Department P 2 1
Q 1 1
--
3 2
Overhead Absorption Rate per hour (Rs.) 2.5 2.5
:. Overhead Expenses per unit of output (Rs.) 7.5 5.0

Expected Collection from Debtors:


Opening balance of Debtors Rs.90,000
Add: Sales (assumed to be on credit):
24,000 units of Ax Rs. 30 = 7,20,000
15,000 units of B x Rs. 40 = 6,00,000 13,20,000
14,10,000
Less: Closing balance of Debtors 1,50,000
:. Amount expected to be collected from Debtors 12,60,000
Cash Budget
Particulars Amounts (Rs.)
Opening balance of Cash 60,000
Add: Cash collectible from Debtors 12,60,000
13,20,000
Less: Payment to Suppliers of Raw-materials 4,20,000
Direct Wages 2,10,000
Overhead Expenses excluding Depreciation:
A: 2,25,000
B: 60,000
(Rs. 2,85,000 - 60,000) = 2,25,000
Selling and Administration Expenses 1,80,900
Payment of Tax 60,000
Purchase of Equipment 1,20,000
12,15,900
:. Closing balance of Cash 1,04,100
Management Accounting : 442
Payment to Creditors:
Opening balance Rs.48,000
Add: Purchases: X Rs. 3,60,000
y 60,000 4,20,000
4,68,000
Less: Closing balance 48,000
:. Payment 4,20,000

Projected Income Statement and Retained Earnings


Particulars Rs. Amounts (Rs.)
Sales Revenue 13,20,000
Less: Cost of Goods Sold 9,12,300
Selling and Administration Expenses 1,80,900
10,93,200
Earnings before Tax 2,26,800
Less: Provision for Taxes (@ 50%) 1,13,400
Profit after Tax 1,13,400
Add: Opening balance of Retained Earnings 96,000
:. Closing balance of Retained Earnings 2,09,400

Projected Balance Sheet of ••. Company as at September 30, .•.


Particulars Rs. Amounts (Rs.)
Share Capital 12,00,000
Add: Retained Earnings 2,09,400 14,09,400

Represented by:Fixed Assets 12,00,000


Add: Additions 1,20,000
13,20,000
Provision for Depreciation (3,00,000 + 60,000) (3,60,000) 9,60,000
Inventories: Raw materials X 1,44,000
Y 12,000 1,56,000
Budgetary Control : 443

Finished goods A 1,28,700


B 72,000 2,00,700 3,56,700
Debtors 1,50,000
Cash 1,04,100
15,70,800
Less: Creditors 48,000
Provision for Taxation 1,13,400 1,61,400
14,09,400

Classification of Budgets according to Flexibility


On the basis of flexibility, the Budgets can be classified into two categories viz., Fixed
Budgets and Flexible Budgets.
Fixed Budgets
A Budget prepared for a particular level of activity is called Fixed Budget. It presents the
cost details for a specific level of activity. Consequently, budgeted costs for budgeted level of
activity are compared with the actual costs for actual level of activity. Therefore, Fixed Budget is
not going to highlight the cost variances due to the difference in the levels of activity. This type
of Budget is not of much use to the management as no adjustment is made to the costs for the
difference in the activity levels. The below presented statement clarifies this point.
2005
Particulars Budget Actual Variance
Rs. Rs. Rs.
Direct Material Cost 2,00,000 3,00,000 1,00,000
Direct Labour Cost 1,50,000 1,60,000 10,000
Pr~duction Overhead Expenses 1,00,000 2,00,000 1,00,000
Operating Expenses 50,000 40,000 -10,000
Total 5,00,000 7,00,000 2,00,000
Production (units) 2,00,000 2,50,000 50,000

From the above, one can come to a conclusion that the company has incurred Rs.
2,00,000 more expenses than budgeted. It is due to both the inefficiency and also due to the
increase in the output. But the Fixed Budget makes no such distinction. Therefore, it is of less
use to the managerial personnel.
Flexible Budget
A Budget prepared for a range of activities rather than for a single level of activity is
called Flexible Budget. It is capable of furnishing the budgeted cost at any level of activity. It
Management Accounting : 444
recognizes the behaviour of costs and classifies them into variable, fixed and semi-variable. On
the basis of this, the Budget is designed to change (i.e., flex) in relation to the level of activity
attained. Therefore, it is possible to compute and compare the budgeted costs for actual level of
activity attained. In order to prepare the Flexible Budgets, tabular method is normally used.
Under Tabular Appro~ch, Flexible Budget is prepared in the form of a table. This table
provides columns for different levels of activity and the expenses are computed and recorded
against the corresponding activity levels. The expenses are normally recorded under the heads
variable, fixed and semi-variable. The illustration given below clarifies the preparational
procedure.
mustration: 6.8
A factory is currently working at 50% capacity and produces 10,000 units at a cost of Rs.
180 per unit as detailed below.
Material Rs.100
Labour 30
Factory Overhead 30 (Rs. 12 fixed)
Administrative Overhead 20 (Rs. 10 fixed)
Total 180
The current selling price is Rs. 200 per unit. At 60% working, material cost per unit
increases by 2% and selling price per unit falls by 2%. At 80% working, material cost per unit
increases by 5% and selling price per unit falls by 5%. Estimate profits of the factory at 60% and
80% working and offer your comments. [SAS Commer., 1979J
Solution:
Fixed,Costs: 1. Factory Overhead = (Rs. 12 per unit x 10,000 units) = Rs. 1,20,000
2. Administration Overhead =(Rs. 10 per unit x 10,000 units) = 1,00,000
2,20,000
Flexible Budget Showing Profit at 50%, 60% and 80% Levels of Activity
50% (10,000) 60% (12,000) 80% (16,000)
Particulars
(Rs.) (Rs.) (Rs.)
Variable Costs:
Material Cost (Rs. 100, Rs. 102 and Rs. 105) 10,00,000 12,24,000 16,80,000
Labour Cost (R~. 30) 3,00,000 3,60,000 4,80,000
Factory Overhead (Rs. 18) 1,80,000 2,16,000 2,88,000
Administration Overhead (Rs. 10) 1,00,000 1,20,000 1,60,000
Total Variable Cost 15,80,000. 19,20,000 26,08,000
Budgetary Control : 445

Sales Revenue (Rs. 200, Rs. 196 and Rs. 190) 20,00,000 23,52,000 30,40,000
Contribution 4,20,000 4,32,000 4,32,000
Less: Fixed Costs: Factory 1,20,000
Administration 1,00,000 2,20,000 2,20,000 2,20,000
:. Estimated Profit 2,00,000 2,12.000 2,12,000

Comments:
1. Out of the three alternatives. operating at 50% is less profitable when compared to other two;
2. Profit is identical at both 60% and 80% levels of activity;
3. If the company intends to earn more by taking less risk, then operating at 60% level is
advisable; and
4. If the company aims at improving its market share, then operating at 80% level is
advisable.
Another method used for preparing the Flexible Budgets is based on the budget for
normal level of activity. This Budget estimates the different cost items at that level. Each item of
variable cost is then expressed for one unit of output. In the same way, variable portion of each
item of semi-variable costs is also expressed per unit of output. Since the fixed costs (including
the fixed portion of semi-variable costs) are expected to be constant for different levels of activity,
they do not pose any difficulty. On the basis of this, the Flexible Budgets are prepared. The
illustration discussed below clarifies this point.
mustration: 6.9
The cost of an article at capacity level of 5,000 units is given under 'A' below. For a
variation of 20% in capacity above or below this level, the individual expenses vary as indicated
under 'B' below:
A B
Material Rs.25,000 100% varying
Labour Cost 15,000 100% varying
Power 1.250 80% varying
Repairs and Maintenance 2,000 75% varying
Stores 1,000 100% varying
Inspection 500 20% varying
Administration Overheads 5,000 25% varying
Depreciation 10,000 100% fixed
Selling Overheads 3,000 50% varying
62,750
Cost per unit 12.55
Management Accounting : 446

Find the unit cost of the product under each individual expenses at production levels of
4,000 units and 6,000 units. {ICWA (Int)]
Solution:
Flexible Budget Showing the Unit Cost at Production Levels of 4,000 and 6,000 units
at 4,000 units at 6,000 units
Costs Total Unit Total Unit
Cost (Rs.) Cost (Rs.) Cost (Rs.) Cost (Rs.)
Variable Costs 1:
Material Cost (Rs. 5) 20,000 5.00 30,000 5.00
Labour Cost (Rs. 3) 12,000 3.00 18,000 3.00
Stores (Re. 0.2) 800 0.20 1,200 0.20
(a) 32,800 8.20 49,200 8.20
Fixed Costs2 :
Depreciation (Rs. 10,000 p.a) 10,000 2.50 10,000 1.67
(b) 10,000 2.50 10,000 1.67
Semi-variable Costs3:
Power (0.2x + 250) 1,050 0.26 1,450 0.24
Repairs & Maint (0.3x + 500) 1,700 0.43 2,300 0.38
Inspection (0.02x + 400) 480 0.12 520 0.09
Administration (0.25x + 3750) 4,750 1.19 5,250 0.88
Selling Overheads (O.3x + 1500) 2.700 0.68 3.300 0.55
(c) 10,680 2.67 12.820 2.14
Total Cost (a + b + c) 53,480 13.37 72,020 12.00
Note: 1. Total costs at different levels are computed on the basis of unit cost (e.g .• matenal cost
at 5,000 units = Rs. 25,000. Therefore, unit material cost = Rs. 5. Using this,
material costs at different levels are computed);
2. On the basis of total fixed cost, unit fixed costs ,.It different levels are computed (by
dividing the total fixed cost by units); and
3. Power expense at 5,000 units comes to Rs. 1.250. This includes variable (80%)
portion of Rs. 1,000 (therefore, per unit variable portion = Rs. 0.2) and fixed
portion of Rs. 250 (i.e., Rs. 1250 - Rs. 1,000). Therefore, power expense at different
levels can be computed by using Y = mx + c (where, Y = Total power expense, m =
Budgetary Control: 447

Variable portion of power expense per unit, x = Output and c = Total fixed portion of
power expense). Therefore, at 4,000 units, Y = (0.2 x 4,000) + 250 = Rs. 1,050.
This way, other items of semi-variable costs are computed.
IDustration: 6.10
The following data are available in a manufacturing company for a yearly period (Rs.
lakhs):
Fixed Expenses: Wages and Salaries 9.5
Rent, Rates and Taxes 6.6
Depreciation 7.4
Sundry Administrative Expenses 6.5
Semi-variable Expenses (at 50% of capacity):
Maintenance and Repairs 3.5
Indirect Labour 7.9
Sales Department, Salaries, etc 3.8
Sundry Administrative Expenses 2.8
Variable Expenses (at 50% of capacity): Materials 21.7
Labour 20.4
Other Expenses 7.9
98.0
Assume that the fixed expenses remain constant for all levels of production; seJ¢-variable
expenses remain constant between 45% and 65% of capacity increasing by 10 per cent between
65 per cent and 80 per cent capacity, and by 20 per cent between 80 per cent and 100 per cent
capacity. Sales at various levels are (Rs. lakhs):
Capacity (%): 50 60 75 90 100
Sales (Rs. lakh): 100 120 150 180 200

Prepare a Flexible Budget for the year and forecast the profit at 60 per cent, 75 per cent,
90 per cent and 100 per cent of capacity. [ICWA (lnt), June 1974J
Solution:
Flexible Budget Showing Profit at 50%,60%,75%,90% and at 100% (Rs. in lalm)
Particulars 50% 60% 75% 90% 100%
Fixed Expenses:
Wages and Salaries 9.50 9.50 9.50 9.50 9.50
Management Accounting : 448

Rent, Rates and Taxes 6.60 6.60 6.60 6.60 6.60


Depreciation 7.40 7.40 7.40 7.40 7.40
Sundry Administrative Expenses 6.50 6.50 6.50 6.50 6.50
(a) 30.00 30.00 30.00 30.00 30.00
Semi Variable Expenses:
Repairs and Maintenance 3.50 3.50 3.85 4.20 4.20
Indirect Labour 7.90 7.90 8.69 9.48 9.48
Salary of Sales Department 3.80 3.80 4.18 4.56 4.56
Sundry Administrative Expenses 2.80 2.80 3.08 3.36 3.36
(b) 18.00 18.00 19.80 21.60 21.60
Variable Expenses:
Materials 21.70 26.04 32.55 39.06 43.40
Labour 20.40 24.48 30.60 36.72 40.80
Other Expenses 7.90 9.48 11.85 14.22 15.80
(c) 50.00 60.00 75.00 90.00 100.00
,
Total Cost (a + b + c) 98.00 108.00 124.80 141.60 151.60
Sales Revenue 100.00 120.00 150.00 180.00 200.00
:. Profit 2.00 12.00 25.20 38.40 48.40

mustration: 6.11
A company has a capacity of producing 1,00,000 units of a certain product in a month. The
sales department reports that the following schedule of sale prices is possible.
Volume of Production 60% 70% 80% 90% 100%
Selling Price per unit (Re) : 0.90 0.80 0.75 0.67 0.61

The variable cost of manufacture between these levels is Re. 0.15 per unit and fixed cost
Rs. 40,000. (a) Prepare a statement showing incremental revenue and differential cost at each
stage. At which volume of production will the profit be maximum? (b) If there is a bulk offer at
Re. 0.50 per unit for the balance capacity over the maximum profit volume for export and price
quoted will not affect the internal sale, will you advise to accept this bid and why?
[ICWA (lnt), July 1964J
Budgetary Control : 449

Solution:
(a) Statement of Incremental Revenue and Differential Cost
Unit Variable Incre- Incremental
Level of Sales Fixed Total Different-
Output Selling Cost mental Profit
Activity Revenue Cost Cost tial Cost
(units) Price Revenue
(%) (Rs OO) (Rs 00) (Rs 00) (Rs 00) (Rs OO) (Rs 00)
(Re.) (Rs OO)

60 60,000 0.90 540 90 400 490 - - -


70 70,000 0.80 560 105 400 505 20 15 5
80 80,000 0.75 600 120 400 520 40 15 25
90 90,000 0.67 603 135 400 535 3 • 15 -12
100 1,00,000 0.61 610 150 400 550 7 15 -8

It can be seen from the above that the incremental revenue exceeds the differential cost up
to 80% capacity. Increase in the sales volume up to a maximum of 80% increases the ~fit and if
it exceeds 80% (say, 90% or above), the differential costs will be more than the incremental
revenue resulting in the reduction in total profit. From the available information, it can, therefore,
be said that the company earns maximum profit by setting its plant at 80% capacity.
(b) Comparative Income Statement (Rs)
Reject the Accept the Export Offer
Particulars Export Offer Home Market Export Total
(80,000) (80,000) (20,000) (1,00,000)
Sales Revenue 60,000 60,000 10,000 70,000
Less: Variable Cost 12,000 12.000 3,000 15,000
:. Contribution 48,000 48,000 7,000 55,000
Less: Fixed Costs 40,000 40,000
:. Profit 8,000 15,000

Since the acceptance of the export order increases the profit by Rs. 7,000, it should be
accepted. Alternatively, the proposal can be examined by considering only the export offer as
shown below.
Revenue from export sales (20,000 units x Re. 0.5) Rs.10,000
Less: Incremental Cost (at Re. 0.15 per unit) 3,000
:. Incremental Profit (Le., Contribution) 7,000
Management Accounting : 450
Illustration: 6.12
Demand for the output of a certain company is very elastic and a modem plant recently
installed is capable of increased production. Output at present is 80,000 units per year. and half a
million units annually are estimated to be within the capacity of the new plant. The present
selling price per unit is Rs. 150. The need for flexible budgeting is recommended and six
alternative projected output levels, up to a maximum of 5,00,000 units, which involve
corresponding reductions of Rs. 10 each in the unit price for a different output of 70,000 units.
The price per unit at the maximum output will be Rs. 90. The present variable costs amount to
Rs. 40.00,000. Fixed costs which at present amount to Rs. 20,00,000 are not expected to increase
for any of the six alternative output levels contemplated. Semi-fixed costs are expected to vary
from the present annual figures of Rs. 23,00,000 to Rs. 32,00,000 the upward steps being Rs.
26,00,000 at 2,20,000 units, Rs. 28,00,000 at 3,60,000 units and Rs. 32,00,000 at 5,00,000 units.
The costs classified as variable at the six projected levels of output are calculated to be as follows:
Rs. 75,00,000; Rs. 1,10,00,000; Rs. 1,50,00,000;
Rs. 1,75,00,000; Rs. 2,05,00,000; Rs. 2,50,00.000;
(a) Tabulate the above data and show total costs, incremental costs, total and incremental
sales at the various levels of output.
(b) Which volume would be set for the budgeted output?
(c) What is the selling price at that volume? [ICWA (Fin), June 1985]
Solution:
(a) Table Showing Total Costs, Incremental Costs, Total and Incremental Sales at Various
Levels
Cost (Rs. lakh) Differ- Incre-
Sales Total Incre-
Selling Profit ential mental
Production Revenue Cost mental
Price Semi- (Rs. Cost Profit
(units,OOO) (Rs. Variable (Rs. Revenue
(Rs.) variable lakh) (Rs. (Rs.
lakh) lakh) (Rs.lakh)
lakh) lakh)

80 150 120 40 23 83 37 - - -
150 140 210 75 23 118 92 90 35 55
220 130 286 110 26 156 130 76 38 38
290 120 348 150 26 196 152 62 40 22
360 110 396 175 28 223 173 48 27 21
430 100 430 205 28 253 177 34 30 4
500 90 450 250 32 302 148 20 49 -29

(b) Annual output is to be set at 4.30,000 units; and


(c) At 4,30,000 units, the selling price comes to Rs. 100 per unit.
,
Budgetary Control: 451

mustration: 6.13
A manufacturing co., is operating at 75% of normal capacity. It is proposed to offer a
price reduction of 5% to 10% depending upon the sales volume desired. Given below are the
relevant data.
Capacity (%) 75 85 100
Output (units): 75,000 85,000 1,00,000
Selling Price/unit (Rs.): 96 5% off 10% off
Materials Cost/unit (Rs.): 40 10% less 15% less
Wages Cost/unit (Rs.): 10 10 10
Fixed Overhead: Production Rs. 14,00,000
Selling and Administration Rs. 5,00,000
Variable Overhead: Production Rs. 14,00,000 at normal capacity
Selling and Administration Rs. 4,40,000
(a) Prepare a simple statement to show profitfloss at each level of output.
(b) Compute unit variable cost, unit fixed cost, and unit total cost at different levels of output.
(c) Indicate which of the three levels is most profitable. IM.Com (Fin), 1975J
Solution:
(a) Statement Showing the Profit at Different Levels of Activities
Levels of Activity (Rs. 000)
Particulars
75% 85% 100%
Variable Cost of Sales:
Materials 3,000 3,060 3,400
Wages 750 850 1,000
Production Overheads 1,050 1,190 1,400
Selling and Administration 330 374 440
Total Variable Cost 5,130 5,474 6,240
Sales Revenue 7,200 7,752 8,640
:. Contribution 2,070 2,278 2,400
Less: Fixed Costs: Production 14,00,000
Selling and Administration 5,00,000 1,900 1,900 1,900
:. Profit 170 378 500
1
Management Accounting : 452

Additional Information:
Sales Volume (units) 75,000 85,000 1,00,000.
Selling Price per unit (Rs.) 96 91.2 86.4
Material Cost per unit (Rs.) 40 36 34
Wages per unit (Rs.) 10 10 10
Variable Overheads:
Production =(Rs. 14,00,000 + 1,00,000 units) 14 14 14
Selling and Admn(Rs. 4,40,000 + 1,00,000 units) 4.4 4.4 4.4
Fixed Overheads:
Production 14,00,000
Selling and Administration 5,00,000
Rs. 19,00,000

(b) Statement Showing Unit Variable Cost, Unit Fixed Cost and Total Unit Cost
Levels of Activity (Rs. 000)
Particulars
75% 85% 100%
Unit Variable Cost:
Material 40.00 36.00 34.00
Wages 10.00 10.00 10.00
Production Overhead 14.00 14.00 14.00
Selling and Administration 4.40 4.40 4.40
Total Unit Variable Cost (a) 68.40 64.40 62.40
Unit Fixed Cost: (Total + Units)
Production Overhead 18.67 16.47 14.00
Selling and Administration 6.67 5.88 5.00
Total Fixed Cost per unit (b) 25.34 22.35 19.00
:. Total Unit Cost (a + b) 93.74 86.75 81.40

(c) Since the profit is maximum at 100% capacity, it is profitable to increase the sales from the
present 75,000 units to 1,00,000 units. Of course, it is necessary to reduce the selling price
by 10%. Still it is profitable to set the plant at 100% capacity.
Advantages of Budgetary Control System
Budgetary Control System is being viewed by the corporate society as an effective
management tool for both minimizing cost, and maximizing revenue and profits. As opined by
Dr. S.N. Maheshwari, it acts as a friend, philosopher and guide to the management. It begins
Budgetary Control : 453
with the establishment of Budgets for various departments and functions, and ends with the
comparison of actuals with the budgeted identifying the variations and suggesting remedial
measures. In the light of this, the important advantages of Budgetary Control System to the
company and to the management are presented below.
1. Ensures Economy in Working: Since Budgetary Control System clearly spells out the
responsibilities of heads of various Budget Centres, the entire company will conduct its
operation in the most.economical manner. Because, each one aims at utilizing the resources
at his disposal more effectively and productively. This will result in both the minimization of
waste, loss, etc., and maximization of productivity. Van Sickle has therefore opined, even
though a monetary reward is not offered, the budget becomes a game - a goal to achieve
or a target to shoot at - and hence it is more likely to be achieved or hit than if there was
no pre-determined goal or target. The. budget is an impersonal policeman that
maintains ordered effort and brings about efficiency in results.
2. Avoids Buck-Passing: Budgetary Control clearly spells out the responsibilities of each of
the executives and it will be supported by the delegation of authority commensurate with the
responsibility. Therefore, everyone working in the organization knows very well what he has
to do and achieve. This fixation of divisional responsibility, of course in consultation with the
heads of the divisions or Budget Centres, prevents buck-passing when the targets or goals are
not achieved.
3. Establishes Co-ordination among Divisions: Since the success of the scheme (viz.,
Budgetary Control System) depends upon the team work, Budgetary Control System co-
ordinates the various divisions, departments, functions, etc., of a company such as production,
purchase, personnel, sales, finance, etc. Because, the work of one division depends upon the
other and vice versa. Further, the Budgets are prepared and finalized after consulting
different levels of management and therefore, the approved Budget represents the collective
opinion. It (i.e., Budgetary Control) establishes proper co-ordination among various divisions
of the company.
4. Management by Exception (MBE): When the actuals are compared with the budgeted,
some deviations or variations can be found. Comparison also reveals the root of inefficiency
and therefore, the management can concentrate only on the divisions which are not working
according to plan and leaving the others. This will enable the management to devote more of
its time and effort to improve the performance of the divisions which have failed to achieve
their targets.
5. Optimum Utilization of Resources: Since the Budgetary Control System creates a healthy
competition among divisional managers, it ensures the optimum utilization of available
resources to achieve the goals.
6. Continuous Review of Performance: Budgetary Control System helps the management to
review the performance of different Budget Centres - both continuously and periodically.
Consequently, it is possible for the management to. exercise proper control over the activities
on time as the deviations are revealed in time. This helps to achieve the overall objective set
in the Budget.
Management Accounting : 454

The other benefits which accrue to the company from Budgetary Control System are as
follows.
1. It helps to adopt the principles of Standard Costing.
2. It helps to receive greater favour from the credit agencies.
3. It facilitates the avoidance of both under- and over-capitalization.
4. Budgetary Control System facilitates the adoption of uniform policy.
Limitations of Budgetary Control System
Budgetary Control System suffers from a few limitations. All the parties - both the Budget
Committee and the executing people must have a complete and clear idea about these limitations
to reap the full benefits. The limitations may, therefore, be considered as precautions to be taken
by all the concerned to introduce and implement Budgetary Control System successfully. The
important limitations are identified below.
1. Budget Plan is based on Estimates: As is known, Budgets are prepared on the basis of
forecasts and estimates about the future. Since they are based on estimates, successful
accomplishment of the targets depends, to a greater extent, upon the degree of accuracy with
which the estimates have been made. If there is any lapse in the estimates. the entire Budget
exercise will be futile.
2. Budgets are not Substitutes for Management: Mere preparation of Budgets wi,lI not ensure
the desired result. Because, the successful introduction and implementation of Budgetary
Control System depends upon the effort put in by all the concerned. Budgeting is only a
means to achieve the goal. Therefore, every individual (both the employees and management)
has to work hard to achieve the budgeted result. Further, Budgeting is normally an impersonal
approach and therefore, the Budgets are to be supported by the systematic management.
Because, it is the management which prepares the Budgets and which strives to achieve the
targets. Therefore, management's effort - starting from the preparation to the execution
cannot be disregarded.
3. Budgets do not ensure Result: Budgets clearly specify the targets and ways through which
the targets can be achieved. Mere preparation of Budgets will not ensure the desired result. It
is, therefore, necessary on the part of all the departmental heads to work sincerely to achieve
the result. They have to extend full co-operation to others and they must obtain co-operation
from others. Each employee must work for reaching the target set for him in the Budget.
Because, it is a group effort. For instance, assume that the Production Department of a
company has succeeded to achieve its target. If the Marketing Department of the company is
not able to sell the target volume at the budgeted price, the company will not be able to
achieve its overall target. Therefore, all the departments must strive for the accomplishment
of the target.
4. Budgeting is a Costly Exercise: In order to introduce Budgetary Control System, a huge
amount of expenditure is to be incurred. Because, the introduction of Budgetary Control
System involves the preparation of Budgets, execution, obtaining the actual, comparison of
actuals with the Budgets and finding out the deviations. In order to perform all these, a huge
Budgetary Control : 455

amount of expenditure is to be incurred. If the employees are not serious about their
responsibilities, no benefit can be obtained from the system. Consequently, only the costs
(without backed by substantial benefit) will have to be incurred. Further, small scale
organizations do not afford to spend this much for the introduction of Budgetary Control
System. Therefore, one must compare the costs with the benefits expected to be generated by
the system.
5. Rigidity: As the Budgets express quantitatively all the relevant facts and figures, an element
of rigidity is attached to the Budgetary Control System. Because, once the Budgets are
finalised and approved, the next step will be to achieve the target result by all the concerned
following the guidelines suggested in the Budgets. Therefore, an element of rigidity or finality
or static can be observed in the Budgets. But, the Budgets are revised periodically in the light
of the changes that are taking place. They are revised and improved in the light of the
changed conditions in the business .
., These are some of the important limitations of Budgetary Control System and the
Budget Committee and execution agency must have complete knowledge about these limitations
so that realistic Budgets are prepared and implemented successfully.
mustration: 6.14
The sales budget for the first five months of 1981 is given for a particular product line
manufactured by Prabhakar & Co., as below.
Months: January February March April May
Sales Budget (units): 10,800 15,600 12,200 10,400 9,800
The inventory of finished product at the end of each month is to be equal to 25% of the
sales estimated for the next month. On January I, 19"81, there were 2,700 units of product on
hand. No work is in process at the end of any month. Each product unit requires two types of
materials in the following quantities: Material A: 4 units; and Material B: 5 units. Materials
equal to one-half of the next month's production are to be in hand at the end of each month. This
requirement was met on January 1, 1981. Prepare a budget showing quantities of each type of
material to be purchased each month for the first quarter of 1981.
[MBA, Karnatak University, April 1981]
Solution:
Monthly Production Budget of Prabhakar and Co for 4-month Period ending April 30, 1981
January February March April
Particulars
(units) (units) (units) (units)
Sales (budgeted) 10,800 15,600 12,200 10,400
Add: Closing Stock (25% of next month sales) 3,900 3,050 2,600 2,450
Total Requirements 14.70,0 18,650 14,800 12,850
Less: Opening Stock 2.700 1 3,900 3,050 2,600
:. Production (budgeted) 12,000 14,750 11,750 10,250
Management Accounting : 456
Monthly Purchase Budget for the first Quarter of 1981
January (units) February (units) March (units)
Particulars
A B A B A B
Production (requirements)
(units to be produced X material
in units per unit of output)
48,000 60,000 59,000 73,750 47,000 58,750
Add: Desired end inventory (50%
of next month production
requirements) 29,500 36,875 23,500 29,375 20,5002 25,625 3

Total requirements 77,500 96,875 82,500 1,03,125 67,500 ,84,375


4 4
Less: Opening Stock 24,000 30,000 29,500 36,815 23,500 29,375

:. Quantity (in units) to be Purchased 53,500 66,875 53,000 66,250 44,000 55,000

Note: 1. Given in the problem; 2. ~ x (10,250 x 4); 3. ~ x (10,250 x 5)


4. Given in the problem that this requirement was met on January 1, 1981.
mustration: 6.15
Mysore Manufacturer operates three sales divisions (X, Y, Z) which sell three branded
products A, B and C. The budget committee needs a sales budget for the next year from the
following information. Budget sales units for current year are as follows.
Product X Y Z
A 8000 12,000 12,000
B 6,000 16,000 8,000
C 4,000 24,000 10,000

Actual sales units for the current year based on actual sales to the date and estimated sales
for the balance of the year are:
Product X Y Z
A 10,000 16,000 14,000
B 4,000 20,000 10,000
C 2,000 20,000 8,000
Budgetary Control : 457
The selling prices per unit of A, B and C are Rs. 5, Rs. 10 and Rs. 20 respectively
applicable for all the divisions. The discussions with divisional sales managers have resulted in
the following suggestions and estimates. Product A is oversold and if the price is increased by
10%, even then it finds a ready market; Product C is overpriced and the price of it can be reduced
by 5%. By incorporating these changes, the sales will be as follows:
Product X y. Z
A +30 +40 +20
B -10 +30 -10
C +10 +20 +10

In view of these estimates, an intensive advertising campaign is decided on to boost sales


of B in divisions X and Z. It was thought that it leads to an increase of B as follows: Division X =
10%; Division Y = 5%; and Division Z = 20%. It is the practice of the company to prepar~ sales
budgets which show the budgets, the budgeted sales, and actual sales for the current period.
1M. Com, UoM., May 1977J
Solution:
Product-wise Sales Potential Statement
Budget for the Current
Divisions Budget for Next Year Actual for Current Period
Period
and Workings
Rate Total Rate Total Rate Total
Products Units Units Units
Rs. Rs. Rs. Rs. Rs. Rs.
X A 10,400 5.50 57,200 8,000 5.00 40,000 10,000 5.00 50,000 8000+30%
Rs.5+10%
B 5.940 10.00 59,400 6.000 10.00 60.000 4.000 10.00 40,000 (6000-
10%)+10%
C 4,400 19.00 83,600 4,000 20.00 80,000 2,000 20.00 40,000 4000+10%
Rs.20-5%

Total X 20,740 2,00,200 18.000 1.80,000 16.000 1,30,000

Y A 16,800 5.50 92,400 12,000 5.00 60,000 16,000 5.00 80,000 12000+40%
Rs.5+10%
B 21.840 10.00 2,18,400 16,000 10.00 1,60,000 20,000 10.00 2,00,000 (16000+30%)
+5%
C 28.800 19.00 5,47,200 24,000 20.00 4.80,000 20,000 20.00 4,00,000 24000+20%
Rs.2~5%

TotalY 67,440 8,58,000 52,000 7,00,000 56,000 6,80,000

Z A 14,400 5.50 79,200 12,000 5.00 60,000 14.000 5.00 70,000 12000+20%
5+10%
8,640 86,400 8,000 80,000 10,000 10.00 1,00,000 (8~10%)
B 10.00 10.00
Management Accounting : 458
+20%
C 11,000 19.00 2,09,000 10,000 20.00 2,00,000 8,000 20.00 1,60,000 10000+10%
20-5%

TotalZ 34,040 3,74,600 30,000 3,40,000 32,000 3,30,000

Total
Produc-
tion
41,600 5.50 2,28,800 32,000 5.00 1,60,000 40,000 5.00 2.00.000
A
36,420 10.00 3,64,200 30,000 10.00 3,00,000 34,000 10.00 3,40,000
B
44,200 . 19.00 8,39,800 38,000 20.00 7,60,000 30,000 20.00 6,00,000
C

Grand 1,22,220 14,32,800 1,00,000 12,20,000 1,04,000 11,40,000


Total

IDustration: 6.16
SM Ltd., furnishes the following forecast for the quarter ending 31st March, 1981.
Sales: January: Rs. 12,00,000; February: Rs. 11,00,000; 'and March: Rs. 14,00,000.
During the month of December last, the company made a sale of Rs. 10,00,000 and computed the
cost of sales as under: .
Raw Materials: Rs. 3,50,000; Wages (variable): Rs. 1,75,000; Overheads, variable: Rs.
1,75,000; and Overheads, fixed: Rs. 1,50,000. The fixed overheads include depreciation of Rs.
40,0000. One-fifth of sales is for cash on which a cash discount of 1.5% is allowed. Of the
remaining portion, 50% is collected in the same month and the balance in the next month. Raw
material suppliers allow a credit of one month. Wages are paid on the last working day of the
month to which they relate. While variable overheads are paid in the next month, fixed overhead
expenses are met in the same month.
The percentage of contribution to sales as obtained in December last is expected to be
maintained during the forthcoming quarter also. The cash balance on 1st January, 1981 is Rs.
50,000. The company has to pay a sum of Rs. 60,000 as an instalment of arrears of wages in
March 1981 in terms of a contract with the Union. All production will be sold in the same month.
Term loan interest of Rs. 3,50,000 is payable in January 1981 and the Bank will debit quarterly
interest of Rs. 4,00,000 on drawings in March 1981. Prepare a cash budget showing the cash
position for each of the three months of the quarter ending March 1981.
[ICWA (Fin), December 1980J
Budgetary Control : 459

Solution:
Monthly Cash Budget of SM Ltd., for the first Quarter of 1981 (Rs)
Particulars January February March
Opening balance 50,000 (28,600) 1,75,600
1
Add: Receipts: Sales Realization: Cash Sales 2,36,400 2,16,700 2,75,800
Customers2 8,80,000 9,20,000 10,00,000
(a) 11,66,400 11,08,100 14,51,400
Less: Payments: Suppliers (35% of Sales, I-month credit) 3,50,000 4,20,000 3,85,000
Wages (17.5% of Sales, same month) 2,10,000 1,92,500 2,45,000
. Variable Overheads (17.5% of Sales paid in next month) 1,75,000 2,10,000 1,92;500
Fixed Overheads (less Depreciation) 1,10,000 1,10,000 1,10,000
Instalment of arrears of Wages - - 60,000
Term Loan Interest 3,50,000 - -
Bank Interest - - 4,00,000
(b) 11,95,000 9,32,500 13,92,500
:. Closing balance (a - b) (28,600) 1,75,600 58,900
1. 1/5 of Sales 2,40,000 2,20,000 2,80,000
Less: Discount, 1.5% 3,600 3,300 4,200
2,36,400 2,16,700 2,75,800
2. ·50% of 80% of Sales (in the same month) 4,80,000 4,40,000 5,60,000
50% of 80% Sales (previous month) 4,00,000 4,80,000 4,40,000
8,80,000 9,20,000 10,00,000

mustration: 6.17
From the following data, prepare a forecast balance sheet as on 31.12.1978
1. Position as on 1.1.1978: Rs.lakhs 2. Budget for 1978: Rs.lakhs
Share capital 5.0 Sales 15.0
Reserves 10.0 Production at sale value 20.0
Debentures 3.0 Direct cost of production 12.0
Public deposit 2.0 Fixed overheads 1.0
Debtors 5.0 Variable selling and distribution costs 2.0
Management Accounting: 460
Stocks and stores at cost 3.0 Collection of Debtors and sale proceeds 17.0
Net fixed assets 13.0 Payment of dividends 0.5
Cash and bank balance 1.0 Refund of public deposit 1.0
Current liability 2.0 Net increase in current liabilities 0.5
Additional stocks at closing at cost 3.0
[ICWA (Fin), December 1977J
Solution:
Profit and Loss Ale of ••• Co., for the Year ending December 31, 1978
Amount Amount
Particulars Particulars
Rs. Rs.
To Opening Stock 3,00,000 By Sales 15,00,000
To Direct Cost of Production 12,00,000 By Closing Stock
To Fixed Overheads (Rs.3,OO,OOO + Rs.3,OO,OOO
(assumed to be Depreciation) 1,00,000 additional) 6,00,000
To Variable S & D Costs 2,00,000
To Balance cld 3,00,000
21,00,000 21,00,000
To Dividends 50,000 By Balance bId 3,00,000
To Balance to Balance Sheet: Profit 2,50,000
3,00,000 3,00,000

Balance Sheet of .•• Co., as at December 31, 1978 (Forecast)


Amount Amount Amount
Liabilities Assets
Rs. Rs. Rs.
Share Capital 5,00,000 Fixed Assets 13,00,000
,Reserves 10,00,000 Less: Depreciation 1,00,000 12,00,000
P & L Alc balance 2,50,000 Debtors: Opening 5,00,000
Debentures 3,00,000 + Sales 15,00,000
Public Deposits 20,00,000
(2,00,000 - 1,00,000) 1,00,000 Less: Collection 17,00,000 3,00,000
Current Liability Stock (3,00,000 + 3,00,000) 6,00,000
(2,00,000 + 50,000) 2,50,000 Cash and Bank Balance 3,00,000
or
Opening 1,00,000
Budgetary Control: 461

Cash Profit 4,00,000


Increase in Current Liabilities 50,000
Decrease in Debtors 2,00,000
7,50,000
Dividends -50,000
Public Deposit Refund -1,00,000
Increase in Stock -3,00,000
3,00,000
24,00,000 24,00,000

mustration: 6.18
Chandra Manufacturers wishes to arrange overdraft facilities with its bankers during the
period April to June 1972 when it will be producing mostly for stock purpose. The following data
is available from the relevant books.

(a) Months Sales Purchase Wages


February Rs.3,60,000 Rs.2,49,600 Rs.24,000
March 3,84,000 2,88,000 28,000
April 2,16,000 2,86,000 22,000
May 3,4~,000 4,92,000 20,000
June 2.52,000 5,36,000 30,000
July 2,86,000 4,98,000 28,000

(b) 50% of credit sales are realized in the month following the sales and the remaining 50%
in the second month following. The creditors are paid in the month following the month
of purchase. Payment of wages and supplies are made in the following month.
(c) Cash at bank on 1.4.1972 (estimated) was Rs. 50,000.
From the information, prepare a cash budget for the above period indicating the extent of
the bank facilities the company will require at the end of the each month.
EM. Com, UoM., May 1974]
Management Accounting : 462

Solution:
Monthly Cash Budget of Chandra Manufacturer~(Aprii to June 1972)
April May June
Particulars
Rs. Rs. Rs.
Opening balance 50,000 1,06,000 98,000
Add: Amount received from Customers* 3,72,000 3,00,000 2,82,000
Total (a) 4,22,000 4,06,000 3,80,000
Less: Payment to Suppliers 2,88,000 2,86,000 4,92,000
Payment of wages 28,000 22,000 20,000
(b) 3,16,000 3,08,000 5,12,000
:. Closing balance (a - b) 1,06,000 98,000 (1,32,000)
*Cash Collected from Customers 1,80,000 1,92,000 1,08,000
1,92,000 1,08,000 1,74,000
3,72,000 3,00,000 2,82,000

U1ustration: 6.19
PAC, a progressive enterprise manufacturing only two products and selling them under
the brand names, Resina and Pipto, prepares every month, forecast of profit (or loss) and a
budgeted cash flow statement for presentation to the managing director. Each of the products
requires only two types of raw materials in the following proportions:
Resina Pipto
(per unit) (per unit)
Material (1) 2kgs 4kgs
Material (2) 4kgs 2kgs
The direct labour hours for R and P are 4 and 6 per unit respectively. For the month of
November, sales forecasts were as follows:
Product Units Price per Unit
Resina 3,000 Rs.60
Pipto 6,000 Rs.80
The opening inventory on 1st November and the proposed closing inventory on 30th
November were:
Opening Stock Budgeted Closing Stock
Material (1) 4,000 kgs 5,000 kgs
Material (2) 2,500 kgs 4,000 kgs
Resina 200 units 250 units
Pipto 400 units 500 units
Budgetary Control : 463
. Standard cost data for the month of November were:
Material (1): Rs. 2 per kg; Material (2): Rs. 4 per kg: Direct Labour: Rs. 4 per
hour; Manufacturing Overhead (Application Rate): Rs. 2 per direct labour hour;
Administration Overhead: Rs. 20,000; and Selling and Distribution Overhead: Rs.
40,000.
Sales are collected 50% in the month of sale and 25% in each of the next two months.
Material (I-) is purchased in cash but for material (2), the suppliers allow a credit of one month,
Le., all payments are cleared in the following month. Wage calculations are ready by the first
week of the next month and payment is made on 9th and 10th. Relevant figures for the three
months are extracted below:
September (Rs.) October (Rs.) November (Rs.)
Sales 8,00,000 6,00,000
Purchase: Material (2) 1,00,000
Wages for the month 2,00,000
Other net cash expenses 2,00,000 (Budgeted)
Opening balance of cash 20,000 (on 1st November)
In addition to the above, advance tax estimated at 60% of the net profit in November was
required to be paid in the month. You are required to prepare a production budget, material and
direct labour cost budgets, budgeted profit and loss statement and budgeted cash flow statement
for November. [ICWA (Fin), December 1979J
Solution:
Production Budget of PAC for the Month of November
Particulars Resina (units) Pipto (units)
Sales Forecast 3,000 6,000
Add: Closing Stock of Finished Goods 250 500
3,250 6,500
Less: Opening Stock of Finished Goods 200 400
:. Budgeted Production 3.050 6,100

Calculation of Material Requirements


Material (1) Material (2)
Particulars
(kgs) (kgs)
Material for Budgeted Production:
Resina: (3,050 x 2); (3,050 x 4) 6,100 12,200
Pipto: (6,100 x 4); (6.100 x 2) 24,400 12,200
Add: Closing Stock 5,000 4,000
Management Accounting : 464

35,500 28,400
Less: Opening Stock 4,000 2,500
:. Material to be Purchased 31,500 25,900

Material Cost Budget of PAC for the Month of November


Material (1) Material (2) Total
Particulars Rate Amount Rate Amount Amount
Kgs Kgs Kgs ~
Rs. Rs. Rs. Rs. Rs.
For Budgeted Production
Resina 6,100 2 12,200 12,200 4 48,800 18,300 61,000
Pipto 24,400 2 48,800 12,200 4 48,800 36,600 97,600
Total Consumption 30,500 2 61,000 24,400 4 97,600 54,900 1,58,600
Add: Closing Stock 5,000 2 10,000 4,000 4 16,000 9,000 26,000
35,500 2 71,000 28,400 4 1,13,600 63,900 1,84,600
Less: Opening Stock 4,000 2 8,000 2,500 4 10,000 6,500 18,000
.·.Purchases 31,500 2 63,000 25,900 4 1,03,600 57,400 ·1,66,600

Direct Labour Cost Budget of PAC for the Month of November


Particulars Resina Pipto Total
Production, Budgeted (units) 3,050 6,100 9,150
Direct Labour Hours per unit 4 6
:. Total Number of Direct Labour Hours 12,200 36,600 48,800
Wages Rate (per hour, in Rs.) 4 4 4
:. Total Labour Cost (Rs.) 48,800 1,46,400 1,95,200

Budgeted Profit and Loss Statement of PAC for the Period ending 30th November
Particulars Resina Pipto Total
Rs. Rs. Rs.
Sales Revenue (3,000 x 60; 6,000 x 80) 180,000 480,000 660,000
(a) 180,000 480,000 660,000
Less: Total Manufacturing Cost of Sales, Material Cost 61,000 97,600 158,600
Direct Labour Cost 48,800 146,400 195,200
Production Overhead (Rs. 2 per Direct Labour Hour) 24,400 73,200 97,600
Cost of Production 134,200 317,200 451,400
l
Add: Opening Stock 8,800 20,800 29,600
Budgetary Control : 465

143,000 338,000 481,000


Less: Closing Stock2 11,000 26,000 37,000
(b) 132,000 312,000 444,000
:. Gross Profit (a - b) 48,000 168,000 216,000

Less: Non-manufacturing Expenses: Administrative Rs. 20,000


Selling and Distribution 40,000 60,000
Profit (before Tax) 156,000

Note: 1. [(1,34,200 + 3,050) x 200] [(3,17,200 + 6,100) x 400]


2. [(1,34,200 + 3,050) x 250] [(3,17,200 + 6,100) x 500]
Budgeted Cash Flow Statement
Particulars Rs. Amount
(R~,.)
Opening balance of Cash (as on 1st November) 20,000
Add: Receipt of Cash: Sales Revenue:
25% of September Sales 2,00,000
25% of October Sales 1,50,000
50% of November Sales 3,30,000 6,80,000
Total 7,00,000
Less: Payments: Material (1), Purchases for November 63,000
Material (2), Purchases for October 1,00,000
Wages (October, paid on 9th and 10th of November) 2,00,000
Other net cash expenditure for November 2,00,000
Advance payment of Tax (60% of 1,56,000) 93,600 6,56,600
:. Closing balance of Cash as on 30th November 43,400

IDustration: 6.20
A company expects to have Rs. 37,500 cash in hand on 1st April, 1983 and required you
to estimate the cash position during the three months. April to June, by preparing a cash budget.
The following information is supplied to you.
Months SR Purchase Wages Fact. OH Off. Exps. Sell. Exps.
1983, February 75,000 45,000 9,000 7,500 6,000 4,500
March 84,000 48,000 9,750 8,250 6,000 4,500
Management Accounting : 466

April 90,000 52,500 10,500 9,000 6,500 5,250


May 1,20,000 60,000 13,500 11,250 6,000 6,570
June 1,35,000 60,000 14,250 14,000 7,000 7,000

Additional Information:
1. Period of credit allowed by suppliers is two months.
2. 20% of the sales is for cash and the period of credit allowed to customers for credit sales
is one month.
3. Lag in payment of all expenses: one month.
4. Income tax Rs. 57,000 is due in June.
5. Dividends and Bonus are Rs. 15,000 and Rs.22,500 respectively to be paid in April.
6. An amount of Rs. 1,20,000 is to be paid in May for plant.
[M.Com., Karnatak University, 1984J
Solution:
Cash Budget of ••• Co., for Three Months (April to June 1983) (Rs.)
Particulars April May June
Opening Balance 37,500 49,200 65,950
Add: Receipts:
Cash Sales (20% of sales) 18,000 24.000 27.000
Debtors (for Credit Sales, one month) 67,200 72,000 96,000
(a) 1,22,700 1,45,200 1,88,950
Less: Payments: Sundry Creditors 45.000 48,000 52,500
Wages 9,750 10,500 13,500
Factory Expenses 8,250 9,000 11,250
Office Expenses 6.000 6,500 6,000
Selling Expenses 4,500 5,250 6,570
(b) 73,500 79,250 89,820
Closing Balance (a - b) 49,200 65,950 99,130

lllustration: 6.21
The management of Gemini Ltd., has called for a statement showing the working capital
needed to finance a level of activity of 3,00,000 units of output for the year. The cost structure for
the company's product, for the above-mentioned activity level, is detailed below.
Budgetary Control: 467
Cost per Unit (Rs.)
Raw Materials 20
Direct Labour 5
Overheads 15
Total Cost 40
Profit 10
Selling Price 50

Past trends indicate that raw materials are held in stock, on an average, for two months.
Work-in-progress will approximate to half-a-month's production. Finished goods remain in
warehouse, on an average, for a month. Suppliers of materials extend a month's credit. Two
months credit is normally allowed to debtors. A minimum cash balance of Rs. 25,000 is expected
to be maintained. The production pattern is assumed to be even during the year. Prepare the
statement of working capital determination. [ICWA (Fin), June 1980J
Solution:
Statement of Working Capital Determination
particulars Amount (Rs.)
Current Assets:
Raw Materials (3,00,000 units x Rs. 20 x 2 + 12) 10,00.000
Work-in-progress:
Materials (300,000 x 20 x 0.5112) = 250,000
Labour (300.000 x 05 x 0.5112) = 62,500
Overheads (300,000 x 15 x 0.5/12) = 187.500 5,00,000
Finished Goods:
Materials (3,00,000 x 20 x 1 + 12) = 5,00,000
Labour (3,00,000 x 05 x 1 + 12) = 1.25,000
Overheads (3,00,000 x 15 x 1 + 12) = 3.75.000 10,00,000
Debtors (3,00,000 x 40 x 2 + 12) 20,00,000
Cash 25,000
45,25,000

Less: Current Liability:


Creditors (3.00,000 x 20 x 1 + 12) 5,00,000
40,25,000
:. Working Capital requirement
Management Accounting : 468
IDustration: 6.22
A company making for a stock in the first quarter of the year is assisted by its bankers
with overdraft accommodation. The f)I1owing are the relevant budget figures.
Months Sales (Rs.) Purchase (Rs.) Wages (Rs.)
November 60,000 41,500 4,900
December 64,000 48,000 5,000
January 36,000 81,000 4,000
February 58,000 82,000 2,000
March 42,000 89,500 5,200

Budgeted cash at bank, 1st January 1981 = Rs. 8,600. Credit terms of sale on payment by the
end of the month following the month of supply. On an average, one-half of sales are paid on the
due date whilst the other half are paid during the next month. Creditors are paid during the month
following the month of supply. You are required to prepare a cash budget for the quarter, 1st
January - 31st March 1981 showing the budgeted amount of bank facilities required at each
month. [M.Com., UoM, May 1983J
Solution:
Cash Budget of •.• Co., for Three Months from January 1 to March 31, 1981
January February March
Particulars .,
Rs. Rs. Rs .
Opening Balance 8,600 17,600 (17,400)
Add: Receipts: Sales Realization
50% of Sales of the month following the month of sales 32,000 18,000 29,000
50% of Sales of the month in the second month following 30,000 32,000 18,000
(a) 70,600 67,600 29,600
Less: Payments for:
Purchases 48,000 81,000 82,000
Wages (assumed that wages are paid on 1st of the
following month) 5,000 4,000 2,000
(b) 53,000 85,000 84,000
:. Closing balance (a - b) 17,600 (17,400) (54,400)

At the end of February 1981, the company will require the overdraft facilities to the
extent of Rs. 17,400 and Rs. 54,400 at the end of March 1981.
Budgetary Control : 469
IDustration: 6.23
The salesmen of UP Fertilizers Ltd., have prepared a sales forecast for 1982-83 (for first
six months only) of its two products for the two regions of Western and Eastern UP on the basis
of previous year's results. The forecast is as follows.
Eastern UP Western UP
Product: A (tons) 9,900 5,940
B (tons) 14,850 8,910
The above estimates were made by adding simple 10% to the previous year (6 months)
actual figures. The previous year actuals (6 months) were down by 10% from their corresponding
budget figures. In building up the correct forecast (for 1982-83, first six months), however,
instead of the 10% increases from 1981-82 (6 months) actuals, an increase of 15% in product 'A'
and 20% in product 'B' is considered as reasonable. .
In the past, when the industry was 20% below normal, the company's business was 30%
below normal; when the industry was above normal, the reverse was true. In 1982-83, the
industry was expected to be below normal by 10%. according to reliable estimates. This factor
should be adjusted after giving effect to the provisions of the previous paragraph. You are
required to prepare a sales budget for the first six months of 1982-83.
[M.Com., UoM., May 1982]
Solution:
Sales Budget of UP Fertilizers Ltd., for the first six months of 1982-83 (in tons)
First 6 months of 1982-83
Previous Year
(Budget)
Product Budget Actual
Eastern Western
Eastern Western Eastern Western Total
Total Total UP UP
UP UP UP UP
A 10,000 6,000 16,000 9,000 5,400 14,400 8,797 5,279 14,076
B 15,000 9,000 24,000 13,500 8,100 21,600 13,770 8,262 22,032
Total 25,000 15,000 40,000 22,500 13,500 36,000 22,567 13,541 36,108

Computation of Budgeted Sales of Product A


Eastern Western
Particulars Total
UP UP
Estimates as given in the problem (tons) 9,900 5,940 15,840
Less: 101110 (i.e., added to the previous period's actuals to
arrive at the above estimates) 900 540 1,440
Management Accounting: 470

:. Actual Sales (previous year, 6 months) 9,000 5,400 14,400


Add: 10/90 (previous year actuals were 10% less than the -

previous year budget) 1,000 600. 1,600


:. Budgeted Sales (previous year, 6 months) 10,000 6,000 16,000
Previous year Actuals (as calculated above) 9,000 5,400 14,400
Add: 15% (reasonable estimate) 1,350 810 2,160
:. Normal Sales (if industry is normal) 10,350 6,210 16,560
Less: 15% Decrease due to industrial decline [Le., (30/20) x 10] 1,553 931 2,484

:. Budgeted Sales for the first six months of 1982-83 8,797 5,279 14,076

Computation of Budgeted Sales of Product B


Eastern Western
Particulars Total
UP UP
Estimates as given in the problem (tons) 14,850 8,910 23,760
Less: 101110 added to previous period's actuals 1,350 810 2,160
:. Actual Sales (previous year, six months) 13,500 8,100 21,600
Add: 10/90 (as previous actuals were 10% lower than budgeted) 1,500 900 2,400
:. Budget (previous year, 6 months) 15,000 9,000 24,000
Previous year actuals, six months 13,500 8,100 21,600
Add: 20% (reasonable estimate of increase) 2,700 1,620 4,320
Normal Sales 16,200 9,720 25,920
Less: 15% due to industrial decline 2,430 1,458 3,888
Therefore, Budget for the first 6 months of 1982-83 13,770 8,262 22,032

IDustration: 6.24
Your company manufactures two products A and B. A forecast of the units to be sold in
the first seven months of the year is given below.
Month: January February March April May June July
Product A: 1,000 1,200 1,600 2,000 2,400 2,400 2,000
Product B: 2,800 2,800 2,400 2,000 1,600 1,600 1,800
Budgetary Control: 471
It is anticipated that (i) there will be no work-in-progress at the end of the month; and (ii)
finished units equal to half the sale for the next month will be in stock at the end of each month
(including the previous December). Budgeted production and production costs for the whole year
are as follows.
Product A Product B
Production (units) 22,000 24,000
Per unit direct material (Rs) 12.50 19.00
Per unit direct labour (Rs) 4.50 7.00
Total factory overhead, apportioned (Rs) 66,000 96,000

Prepare for the six months period ending June 30, a production budget for each month
and a summarized production cost budget. [ICWA (Fin)]
Solution:
Production Budget forSix Months ending June 30
Particulars Jan Feb March Apr May June Total
Product A (units):
Sales 1,000 1,200 1,600 2,000 2,400 2,400 10,600
Add: Closing Stock 600 800 1,000 1,200 1,200 1,000 1,000
Total Requirements 1,600 2,000 2,600 3,200 3,600 3,400 11,600
Less: Opening Stock 500 600 800 1,000 1,200 1,200 500
Therefore, Production 1,100 1,400 1,800 2,200 2,400 2,200 11,100
Product B (units):
Sales 2,800 2,800 2,400 2,000 1,600 1,600 13,200
Add: Closing Stock 1,400 1,200 1,000 800 800 900 900
Total requirements 4,200 4,000 3.400 2,800 2,400 2,500 14,100
Less: Opening Stock 1,400 1,400 1,200 1,000 800 800 1,400
Therefore, Production 2,800 2,600 2,200 1,800 1,600 1,700 12,700

Summarized Production Cost Budget for Six Months ending June 30


Product

Particulars A B Total
(11,100 (12,700
units) Rs. units) Rs. Rs.

Direct Material Cost (at Rs. 12.5 and Rs. 19) 1,38,750 2,41,300 3,80,050
Management Accounting: 472

Direct Labour Cost (at Rs. 4.5 and Rs. 7) 49,950 88,900 1,38,850
Prime Cost 1,88,700 3,30,200 5,18,900
. Add: Factory Overhead (assuming it as . ;.rriable)
[(Rs. 66,000 + 22,000 units) x 11,100 units]; 33,300
.
[(Rs. 96,000 + 24,000 units) x 12,700 units] 50,800 84,100
Total Production Cost 2,22,000 3,81,000 6,03,000
If Factory Overhead is fixed, the Production Cost will be:
Prime Cost 1,88,700 3,30,200 5,18,900
Factory Overhead
[(Rs. 66,000 + 12) x 6; (96,000 + 12) x 6] 33,000 48,000 81,000
Total Production Cost 2;21,700 3,78,200 5,99,900

lliustration: 6.25
The following are the details of the budgeted and the actual cost in a factory for six
months from January to June, 1980. From the figures give below, you are required to prepare the
production cost budget for the period from January to June 1981.
January to June 1980: Budget Actual
Production (units) 20,000* 18,000**
Material cost Rs. 40,00,000 39,90,000
[@ Rs. 20 per hour] [@ Rs. 22 per hour]
., Labour cost Rs. 8,00,000 Rs. 7,99,920
Variable overheads Rs. 2,40,080 Rs.2,16,000
Fixed overheads 4,00,000 4,20,000

In the first half of 1981, production is budgeted for 25,000 units. Material cost per tonne
will increase from last year's actual by Rs. 100, but it is proposed to maintain the consumption
efficiency of 1980 as budgeted. Labour efficiency will be lower by another 1% and labour rates
will be Rs. 22 per hour, variable aI,1~ fixed overheads will go up by 20% over 1980 actual. You
are required to prepare the production cost budget for the period January-June 1981 giving all the
working. [Note: * (2,000 M.T @ Rs. 2,000); and ** (Rs. 39,90,000 @ Rs. 2,100)].
leA (Fin), November 1980J
Solution:
1. Material Consumption, budgeted for 1980 per unit:
[2,000 MT + 20,000 units] = 0.1 MT
Budgetary Control: 473

Price per MT of material during the first six months of .1981 =


[1980 actual + 100] = [Rs. 2,100 + Rs. 100] = Rs. 2,200 per MT.
Therefore, Material Cost for 25,000 units = (25,000 units x 0.1 MT per unit) x Rs. 2,200
per MT = [2,500 MT x Rs. 2,200 per MT] = Rs. 55,00,000
2. Labour Hours per unit, 1980:
Budgeted: [(Rs. 8,00,000 + Rs. 20) + 20,000 units] =[40,000 +20,000] =2 hours
Actual: [Rs. 7,99,920 + Rs. 22) + 18,000 units] = [36,360 + 18,000] = 2.02 hours
That means, the actual efficiency has reduced by 1% (i.e., 0.02 + 2). It is given in the
problem that the labour efficiency will be lower by another 1%. That means, the
employees will lake an additional 1% time (i.e., 1% of 2 = 0.02 hour) during 1981 to
produce a unit Therefore, the number of hours per unit = (2 + 0.02) + 0.02 = 2.04 hours.
Therefore, Total number of labour hours required for 25,000 units = [25,000 x 2.04] =
51,000 hours.
Total Labour Cost =[51,000 hours x Rs. 22] =Rs. 11,22,000.
3. Actual Variable Overheads per unit during 1980 =
[Rs. 2,16,000 + 18,000 units] Rs.12.00
Add: 20% increase 2.40
Therefore, Budgeted Variable Overhead Expenses per unit during
the first six months of 1981 14.40
Therefore, budgeted total variable overheads for first six months
of 1981 =[25,000 units x Rs. 14.4] =Rs. 3,60,000
4. Actual Fixed Overheads for January-June 1980 = Rs.4,20,000
Add: 20% anticipated increase 84,000
Budgeted Fixed Overheads for January June 1981 5,04,000
Production Cost Budget, January-June 1981 (25,000 units)
Particulars AmountRs.
Direct Material Cost 55,00,000
Direct Labour Cost 11,22,000
Variable Overheads 3,60,000
Total Variable Cost (budgeted) 69,82,000
Fixed Overheads 5,04,000
Budgeted Cost of Productio~ 74,86,000
Management Accounting : 474
U1ustration: 6.26
A company manufactures Product A and Product B during the year ending 31st December
1992. It is expected to sell 15,000 kgs of Product A and 75,000 kgs of Product B at Rs. 30 and
Rs. 16 per kg respectively. The direct materials P, Q and R are mixed in the proportion of 3: 5 : 2
in the manufacture of Product A. Materials Q and R are mixed in the proportion of 1 : 2 in the
manufacture of Product B. The actual and budget inventories for the year are given below.
Expected Anticipated
Opening Stock
Closing Stock Cost per Kg
Material P 4,500 3,000 Rs.12
Material Q 3,000 6,000 10
Material R 30,000 9,000 8
Product A 3,000 1,500
ProductB 4,000 4,500

Prepare the production budget and the materials budget showing the expenditure on
purchase of materials for the year ending 31st December, 1992.
[ICWA (Fin), December 1982}
Solution:
Production Budget for the Year ending December 31,1992
Product (kgs)
Particulars
A B
Expected Sales 15,000 75.000
Add: Expected Closing Stock 1,500 4,500
Total Requirements 16,500 79,500
Less: Opening Stock 3,000 4,000
Therefore, Production 13,500 75,500

Materials Budget for the Year ending December 31,1992


Materials (kgs) Total
Particulars
P Q R (kgs)

Materials required for production of product:


.
. A: 13,500 kgs in 3: 5 : 2 ofP, Q and R 4,050 6,750 2,700 13,500
B: 75,500 kgs in 1 : 2 of Q and R 0 25,167 50,333 75,500
Budgetary Control: 475

Add: Closing Stock 3,000 6,000 9,000 18,000


Therefore, Total Requirements 7,050 37,917 62,033 1,07,000
Less: Opening Stock 4,500 3,000 30,000 37,500
Therefore, ~rchases 2,550 34,917 32,033 69,500
Price per kg (Rs.) 12 10 8
Therefore, Total purchase price (Rs.) 30,600 3,49,170 2,56,264 6,36,034

mustration: 6.27
Soloproducts Ltd., manufactures and sells a single product and has estimated a sales
revenue Rs. 126 lakh this year based on a 20% profit on selling price. Each unit of the product
requires 3 lb of material P and 1.5 lb of material Q for manufacture as well as a processing time of
7 hours in the Machine Shop and 2.5 hours in the Assembly Section. Overheads are absorbed at a
blanket rate of 33 113% on direct labour. The factory works 5 days of 8 hours a week in a normal
52 weeks a year. On an average, statutory holidays, leave and absenteeism and idle time amount
to 96 hours, 80 hours and 64 hours respectively in a year. The other details are as under.
Purchase Price: Material P - Rs. 6 per lb; and Material Q - Rs. 4 per lb.
Comprehensive Labour Rate: Machine Shop - Rs. 4 per hour; and
Assembly - Rs. 3.20 per hour.
Number of Employees: Machine Shop - 600; and Assembly -180.
Finished Goods Material P Material Q
Opening Stock 20,000 units 54,0001b 33,0001b
Closing Stock (estimates) 25,000 units 30,0001b 66,0001b

Your are required to calculate:


(a) The number of units of the product proposed to be sold.
(b) ~rchases to be made of materials P and Q during the year in rupees.
(c) Capacity utilization of Machine Shop and Assembly Section, along with your comments.
lCA (lnt), November 1985J
Solution:
Income Statement (per unit basis)
Particulars Amount (Rs.)
Material Cost: P: (3 lb x Rs. 6) 18
Q: (1.51b x Rs. 4) 24
-6
Add: Labour Cost
Management Accounting: 476

Machine Shop: (7 hours x Rs. 4) 28


Assembly: (2.5 hours x Rs. 3.2) 8 36
Prime cost 60
Add: Overheads (at 33 113% of Labour Cost) 12
Unit cost 72
Add: Desired Profit (20% of Selling Price = 25% of Cost Price) 18
Therefore, Selling Price per unit 90

-(a) Estimated Sales Revenue: Rs. 1.,26,00,000


Unit Selling Price: Rs. 90
:. Units proposed to be sold = 1,40,000
(b) Units to be Sold 1,40,000
Add: Closing Stock 25,000
Total Requirement 1,65,000
Less: Opening Stock 20,000
:. Budgeted Production 1,45;000
Purchase Budget
Materials (Ib)
Particulars
p Q
Material required for Production
(1,45,000 @ 31b; @ 1.51b) 4,35,000 2,17,500
Add: Closing Stock 30,000 66,000
Total Requirements 4,65,000 2,83,500
Less: Opening Stock 54,000 33,000
:. Purchases to be made 4,11,000 2,50,500
Purchase Price per lb (Rs.) 6 4
:. Total PuJ'chase Bill (Budgeted) (Rs.) 24,66,000 10,02,000
Budgetary Control: 477

Capacity Utilization of Machine Shop and Assembly


Machine
Particulars Assembly
Shop
1. Hours required.to manufacture 1,45,000 units at 7 hours and
at 2.5 hours, a unit
10,15,000 3,62,500
2. Hours available per year [(52 weeks x 5 days x 8 hours)
-(96 + 80 + 64)] =[2,080 - 240] =1,840 hours per employee)
Therefore, (1,840 x 600; 1,840 x 180)
11,04,000 3,31,200
:. Capacity Utilization = (Required + Available) (%) 91.94 109.45

mustration: 6.28
Ace Ltd manufactures three products A, C and E in two production departments F and G,
in each of which are employed two grades of labour. The cost accountant is preparing the annual
budgets for the next year and he has asked you to prepare, using the data given below: (a) the
production budget in units for products A, C and E; (b) the direct wages budget for departments F
and G with the labour costs of products A, C and E and total shown separately.
Product A Product C Product E
Finished stocks (Rs. 000)
Budgeted stocks are: 1st Jan, next year 720 540 1,800
31 st Dec, next 600 570 1,000
All stocks are valued at standard cost per unit Rs.24 Rs.15 Rs.20
Standard profit: calculated as percentage of selling price 20% 25% 16213%
Budgeted sales are (Rs. 000): Total Product A Product C ProductE
South 6,600 1,200 1,800 3,600
West 5,100 1,500 1,200 2,400
North 6,380 1,500 800 4,080
18,080 4,200 3.800 10,080
NormaIloss in production (%) 10 20 5
Standard labour time per unit and standard rates per hour
Rate Product A ProductC ProductE
Rs. Hours per unit Hours per unit Hours per unit
Department F: Grade 1 1.80 2.0 3.0 1.0
Grade 2 1.60 1.5 2.0 1.5
Management Accounting: 478
Department G: Grade 1 2.00 3.0 1.0 1.0
Grade 2 1.80 2.0 1.5 2.5
lCA (Fin), May 1987J
Solution:
Production Budget of Products A, C and E (units)
Products
Particulars
A C E
Budgeted Sales Volume (units) (Working Note 1) 1,40,000 1,90,000 4,20,000
Add: Closing Stock [Value + Unit Standard Cost) 25,000 38,000 50,000
Total Requirements 1,65,000 2,28,000 4,70,000

Less: Opening Stock (Value + Unit Standard Cost) 30,000 36,000 90,000

Budgeted Production (Net) 1,35,000 1,92,000 3,80,000

Working Notes:
1. Standard Cost per unit (Rs.) A C E
Add: Standard Profit (20%, 25% and 162/3% of 24 15 20
Selling Price or 25%, 33 1/3% and 20% of Cost Price) 6 5 4
:. Standard Selling Price (Rs.) 30 20 24
Budgeted Sales Revenue (in all the three zones) (Rs. 000) 4,200 3,800 10,080
:. Budgeted Sales Volume (units) 1,40,000 1,90,000 4,20,000
2. Input to Production (units) =Budgeted Output + Normal Loss
=
A: [1,35,000 +'(1/9 x 1,35,000)] 1,50,000 units
=
C: [1,92,000 + (1/4 x 1,92,000)] 2,40,000 units
E: [3,80,000 + (1/19 x 3,80,000)] =4,00,000 units

Labour Budget

Particulars 1,50,000 Product A 2,40,000 Product C 4,00,000 Product E Total


SHrs Rs. S Hrs Rs. SHrs Rs. Rs.

Department F:
Grade 1: (@ 2, 3 and 1 3.00,000 7.20.000 4,00,000
hours a unit; at Rs. 1.8
an hour) 5,40,000 12,96,000 7,20,000 25,56,000

Grade 2: (1.5, 2 and 1.5 2,25,000 4,80,000 6,00,000


hours a unit @ Rs. 1.6) 3,60,000 7,68,000 9,60,000 20,88,000

Total (a) 9,00,000 20,64,000 16,80,000 46,44,000

Department G:
Grade 1: (@ 3, 1 and 1 hours 4,50,000 2,40,000 4,00,000
Budgetary Control: 479
8 unit @ Rs. 2 an hour) 9,00,000 4,80,000 8,00,000 21,80,000

Grade 2: (@ 2, 1.5 and 2.5 3,00,000 3,60,000 10,00,000


hours 8 unit @ Rs. 1.8
an hour) 5,40,000 6,48,000 18,00,000 29,88,000

Total (b) 14,40,000 11,28,000 26,00,000 51,68,000

Grand Total (8 + b) 23,40,000 31,92,000 42,80,000 98,12,000

mustration: 6.29
A proforma cost sheet of a company provides the following particulars:
Raw materials cost per unit Rs. 80
Direct labour cost per unit 30
Overhead expenses per unit 60
Unit total cost 170
Unit profit 30
Selling price 200
The following further particulars are available.
(a) Raw materials are in stock on average one month; materials are in process on average,
half a month; finished goods are in stock average one month.
(b) Credit allowed by suppliers is one month; credit allowed to debtors is two months. Time
lag in payment of wages is 1.5 weeks, lag in payment of overhead expenses is one month.
(c) One-fourth of the output is sold against cash. Cash on hand and at a bank is expected to
be Rs. 25,000.
You are required to prepare a statement showing the working capital needed to finance a
level of activity of 1,04,000 units of production. You may assume that production is carried on
evenly through the year, wages and overheads accrue similarly and a time period of 4 weeks is
equivalent to a month. [M.Com, UoM., May 1975J

Solution:
Working Capital Budget (Level of Activity =1,04,000 units)
Particulars Rs. Amount (Rs.)
Current Assets to be maintained:
(a) Stock of Raw materials [(1,04,000 units x Rs. 80) + 52] x 4 6,40,000
(b) Work in Progress:
Material [(1,04,000 units x Rs. 80) + 52] x 2 3,20,000
Labour [(1,04,000 units x Rs. 30) + 52] x 2 1,20,000
Overhead (1,04,000 units x Rs. 60) + 52] x 2 2,40,000 6,80,000
(c) Finished Goods:
Material [(1,04,000 units x Rs. 80) x 4] + 52 6,40,000.
Management Accounting: 480

Labour [(1,04,000 units X Rs. 30) X 4] + 52 2,40,000


Overhead (1,04,000 units X Rs. 60 x 4) + 52 4,80,000 13,60,000
(d) Debtors:
Material [(1,04,000 units x Rs. 80 x (3/4) x 8) + 52 9,60,000
Labour [(1,04,000 units x Rs. 30 x (3/4) x 8) + 52 3,60,000
Overheads [(1,04,000 units x Rs. 60 x (3/4) x 8) + 52 7,20,000 20,40,000
(e) Cash 25,000
Less: Credits allowed by Supplier (1,04,000 units x Rs. 80 x 4) + 52 47,45,000
6,40,000
Lag in-payment of Wages (1,04,000 units x Rs. 30 x 1.5) + 52
90,000
Lag in payment of Overheads (1,04,000 units x Rs.60 x 4) + 52
4,80,000 12,10,000
:. Working Capital required (estimated)
35,35,000

Illustration: 6.30
The following is the operation budget of your company phased by quarters for a calendar
year. From this and the additional information given, prepare a cash flow forecast by quarter
(figures in Rs. lakhs).
1st Qr. 2ndQr. 3rdQr. 4thQr.
Sales: Credit 13.50 12.60 8.20 13.20
Cash 0.50 0.60 0.40 0.80
14.00 13.20 8.60 14.00
Materials consumed 9.40 8.80 5.60 9.20
Operating expenses (variable) 0.60 0.60 0.50 0.60
Fixed expenses (include Rs. 80,000 per quarter, as
depreciation) 1.30 1.30 1.30 1.30
11.30 10.70 7.40 11.10
Operating profit 2.70 2.50 1.20 2.90
End of the quarter balances:
Debtors 7.70 7.70 6.50 7.70
Stock, raw materials 6.50 5.00 6.50 6.50
Stock, fmished goods 3.45 3.75 5.35 3.75
Creditors 2.50 2.50 2.50 2.50
Budgetary Control: 481
Opening balance at beginning of 1st Quarter:
Debtors 7.00 Additional plant on order 4.50 (to be paid in Sept)
Stock, Raw materials 6.00 Anticipated loan for new plant 3.00 (available in Sept)
Finished goods 3.75 Sale of old car 0.08 (in August)
Creditors 2.40 Loan instalment due each quarter 1.50
Bank borrowing 4.70 Advance tax payable each quarter 0.80
[ICWA (Fin), June 1978]
Solution:
Quarterly Cash Budget (Rs. in lakh)
Particulars IQ IIQ llQ IVQ
Opening Balance -4.7 -4.6 - 2.10 -4.12
Add: Receipts: Cash Sales 0.5 0.6 0.40 0.8
Collection from Customers I 12.8 12.6 9.40 12.0
Additional Loan - - 3.00 -
Sale of Old Car - - 0.08 -
(a) 8.6 8.6 10.78 8.68
Less: Payments: Payments to Suppliers2 9.8 7.3 7.1 9.2
Operating expenses
Variable 0.6 0.6 0.5 0.6
Fixed (excluding depreciation) 0.5 0.5 0.5 0.5
Purchase of plant - - 4.5 -
Loan instalment 1.5 1.5 1.5 1.5
Advance tax 0.8 0.8 0.8 0.8
(b) 13.2 10.7 14.9 12.6
Closing Balance (a - b) - 4.6 - 2.1 - 4.12 - 3.92
Working Notes:
Particulars IQ lIQ llQ NQ
1. Collection from Customers:
Opening Balance 7.0 7.7 7.7 6.5
Add: Credit Sales 13.5 12.6 8.2 13.2
Total 20.5 20.3 15.9 19.7
Less: Closing Balance 7.7 7.7 6.5 7.7
Therefore, Collection 12.8 12.6 9.4 12.0
2. Purchases and Payments
Consumption 9.4 8.8 5.6 9.2
Add: Closing Balance 6.5 5.0 6.5 6.5
Management Accounting: 482

Total 15.9 13.8 12.1 15.7


Less: Opening Balance 6.0 6.5 5.0 6.5
llherefore, ~rchases 9.9 7.3 7.1 9.2
Opening Balance of Creditors 2.4 2.5 2.5 2.5
Add: ~chases 9.9 7.3 7.1 9.2
12.3 9.8 9.6 11.7
Less: Closing Balance Total 2.5 2.5 2.5 2.5
llherefore, Payments 9.8 7.3 7.1 9.2
Dlustration: 6.31
Summarized below are the income and expenditure forecasts for the months of March to
August, 2006 (Rs):
Sales
~rchases Manufacturing Office Selling
Months (all Wages
(all credit) Expenses Expenses Expenses
credit)
March 60,000 36,000 9,000 4,000 2,000 4,000
April 62,000 38,000 8,000 3,000 1,500 5,000
May 64,000 33,000 10,000 4,500 2,500 4,500
June 58,000 35,000 8,500 3,500 2,000 3,500
July 56,000 39,000 9,500 4,000 1,000 4,500
August 60,000 34,000 8,000 3,000 1,500 4,500

You are given the following further information.


(a) Plant costing Rs. 16,000 is due for delivery in July payable 10% on delivery and the
balance after three months.
(b) Advance tax of Rs. 8,000 is payable in March and June each.
(c) Period of credit allowed (i) by suppliers two m~nths and (ii) to customers one month.
(d) Lag in payment of manufacturing expenses: half a month.
(e) Lag in payment of all other expenses: one month.
You are required to prepare a cash budget for three months starting 1st May, 2006 when
there was a cash balance of Rs. 8,000.
Solution:
Monthly Cash Budget for three Months, May to July 2006

Particulars May June July


(Rs.) (Rs.) (Rs.)
Opening balance of Cash 8,000 15,750 12,750
Add: Receipts from Customers 62,000 64,000 58,000
(a) 70,000 79,750 70,750
Less: Payments: To Suppliers
J Budgetary Control : 483

36,000 38,000 33,000


Purchase of Plant - - 1,600
Advance Payment of Tax - 8,000 -
Manufacturing Expenses:
50% of previous month
50% of current month
1,500
2,250
2,250
1,750
.2,000
1,750

Wages 8,000 10,000 8,500


Office Expenses 1,500 2,500 2,000
Selling Expenses 5,000 4,500 3,500
(b) 54,250 67,000 52,350
Closing balance (a - b) 15,750 12,750 18,400

mustration: 6.32
In May 1978, Shri Ananth got Rs. 10,000 in a Delhi Lottery and started a business to
manufacture a certain component for BHEL. During the same month, he purchased machineries
for Rs. 5,00,000 out of a gift of Rs. 1,00,000 from his father-in-law and a loan of Rs. 4,00,000
from a bank. Interest at 9% per annum is payable quarterly in arrears from the month of
borrowing. The principal is repayable at Rs. 20,000 every half year. He commenced manufacture
on 1.6.1978 and his production and delivery schedule is as under:
30.06.1978: 1,000 units 30.09.1978: 2,500 units
31.07.1978: 1,500 units 31.10.1978: 3,000 units
31.08.1978: 2,000 units And thereafter, 3,000 units every month.
He gets Rs. 10 per unit from BHEL. His variable cost is Rs. 6 per unit. He has fixed
expenses to the extent of Rs. 1,000 per month. He also wants to draw Rs. 1,000 per month for his
maintenance. His bills are settled after 30 days from the date of supply. His variable cost is to be
met by actual payment in the very month. The fixed cost and his drawings are met on the 1st day
of the next month. He desires always to carry a minimum cash balance of Rs. 2,000 and a
maximum one of Rs. 10,000.
You are required to prepare a cash budget for each of the nine months from May 1978 to
January 1979. Assume a temporary overdraft from the Bank, whenever required within the above
mentioned minimum and maximum cash balance requirements. Ignore interest on the overdraft.
leA (Fin), May 1979]
Solution:
Monthly Cash Budget for 9 months, May 1978 - January 1979 (Rs)
1978 1979
Particulars June July Aug Sep Oct Nov Dec
May Jan
Opening Balance - 10,000 3,000 2,000 2,000 2,000 2,000 2,000 2,000
Cash receipts from
Management Accounting :. ,84
l
Lottery 10,000 - - - - - - - -
Gift from Father-in-law 1,00,000 - - - - - - - -
Bank Loan 4,00,000 - - - - - - - -
Receipt from Customer,
BHEL - - 10,000 15,000 20,000 25,000 30,000 30,000 30,000
Bank Overdraft - - - 8,000 - - 19,000 - -
(a) 5,10,000 10,000 13,000 25,000 22,000 27,000 51,000 32,000 32,000
Cash Payments:
Purchase of Machineries 5,00,000 - - - - - - - -
Interest - - - 9,000 - - 9,000 - -
Repayment of Principal - - - - - - 20,000 - -
Variable Cost - 6,000 9,000 12,000 15,000 18,000 18,000 18,000 ' 18,000
Fixed Cost - - 1,000 1,000 1,000 1,000 1,001) 1,000 1,000
Drawings - 1,000 1,000 1,000 1,000 1,000 1,000 1,000 1,000
Repayment of Bank
Overdraft - - - - 3,000 5,000 - 10,000 9,000
(b) 5,00,000 7,000 11,000 23,000 20,000 25,000 49,000 30,000 29,000
Closing Balance (a - b) 10,000 3,000 2,000 2,000 2,000 2,000 2,000 2,000 3,000

Illustration: 6.33
Prepare a cash budget for the six months ending December 31, 1982 from the monthly
budgeted operating results of the company and other additional information given below (Rs.
lakh).
Material Overheads
purchased
Months Sales Wages Admini- R&D
and Production Selling Distribution
consumed stration
March 8,.00 3.60 0.80 0.48 0.40 0.20 0.10 0.11
April 12.00 6.00 1.28 0.64 0.56 0.29 0.14 0.16
May 9.60 5.20 1.20 0.62 0.48 0.25 0.10 0.12
June 6.40 3.36 0.56 0.30 0.20 0.11 0.06 0.06
July 8.00 3.84 0.80 0.44 0.32 0.16 0.08 0.10
August 8.80 4.00 0.96 0.49 0.40 0.21 0.10 0.12
September 11.20 4.96 1.20 0.62 0.52 0.26 0.12 0.13
October 12.80 6.00 1.04 0.54 0.40 0.20 0.10 0.12
November 14.40 6.40 1.36 0.72 0.56 0.29 0.15 0.16
December 16.00 8.00 1.52 0.74 0.58 0.30 0.16 0.17
Budgetary Control : 485
New machinery which was installed in April at a cost of Rs. 1.2 lakh is to be paid for on
August 1. Extension to the Research and Development Department amounting to Rs. 8 lakh in
total was contemplated from September at the rate of Rs. 1.6 lakh per month. Rs. 2.4 lakh per
month is to be paid under a hire purchase scheme agreement. The sales commission of 4% on
sales, not included in selling overheads, is to be paid within the month following actual sales. The
period of credit allowed by suppliers is 4 months and that allowed to customers is 3 months. The
delay in the payment of overheads is 2 months and that in payment of wages is one-fourth of a
month.
. Preference share dividend of 8 per cent on the capital of Rs. 160 lakh is payable on
December 1; calls on equity shares at the rate of Rs. 9.60 lakh is due on July 1, September 1, and
November 1. Taxation of Rs. 8 lakh is payable on November 1. Dividends on in,:estment
amounting to Rs. 2.40 lakh is expected on July 1, and December 1. Cash sales of Rs. 0.80 lakh
per month are expected on which no commission is payable. This cash sales is not included in the
details for sales given in the table above. Cash balance on July 1 was expected to be Rs. 2 lakh.
[ICWA (Fin), December 1982J
Solution:
Cash Budget for six Months from July 1 to December 31, 1982 (Rs.)
Particulars July August September October November December
Opening Balance 2,00,000 18,23,400 17,06,400 22,07,200 20,66,400 20,58,200
Add: Receipts:
From Customers 12,00,000 9,60,000 6,40,000 8,00,000 8,80,000 11,20,000
Cash Sales 80,000 80,000 80,000 80,000 80,000 80,000
Calls on Capital 9,60,000 - 9,60,000 - 9,60,000 -
Dividend 2,40,000 - - - - 2,40,000
(a) 26,80,000 28,63,400 33,86,400 30,87,200 39,86,400 34,98,200
'Less: Payments:
To Suppliers 3,60,000 6,00,000 5,20,000 3,36,000 3,84,000 4,00,000
Wages:
Previous month (~) 14,000 20,000 24,000 30,000 26,000 34,000
Current month (%) 60,000 72,000 90,000 78,000 1,02,000 1,14,000
Overhead Expenses:
Production 62,000 30,000 44,000 49,000 62,000 54,000
Administration 48,000 20,000 32,000 40,000 52,000 40,000
Selling 25,000 11,000 16,000 21,000 26,000 20,000
Distribution 10,000 6,000 8,000 10,000 12,000 • 10,000
Research & Development 12,000 6,000 10,000 12,000 13,000 12,000
Management Accounting : 486
Sales Commission 25,600 32,000 35,200 44,800 51,200 57,600
Purchase Price of new
machinery - 1,20,000 - - - -
Extension of R&D Dept - - 1,60,000 1,60,000 1,60,000. 1,60,000
Hire Charges 2,40,000 2,40,000 2,40,000 2,40,000 2,40,000 2,40,000
Preference Dividend - - - - - 12,80,000
Taxation - - - - 8,00,000 -
(b) 8,56,600 11,57,000 11,79,200 10,20,800 19,28,200 24,21,600
:. Closing Balance (a - b) 18,23,400 17,06,400 22,07,200 20,66,400 20,58,200 10,76,600

mustration: 6.34
Prepare a cash budget for MIs Alpha Manufacturing Company on the basis of the
following information for the first six months of 1981.
1. Cost and price remain unchanged.
2. Cash sales are 25% of the total sales and 75% credit sales.
3. 60% of credit sales are collected in the month after sales, 30% in the second month and 10%
in the third, no bad debts are anticipated.
4. Sales forecasts are as follows (Rs):
1980 October: 12,00,000 1981 March: 8,00,000
November: 14,00,000 April: 12,00,000
December: 16,00,000 May: 10,00,000
1981 January: 6,00,000 June: 8,00,000
February: 8,00,000 July: 10,00,000
5. Gross profit margin 20%
6. Anticipated purchases (Rs.)
1981 January: 6,40,000 1981 April: 8,00,000
February: 6,40,000 May: 6,40,000
March: 9,60,000 June: 9,60,000
7. Wages and salaries to be paid (Rs.)
January: 1,20,000 April: 2,00,000
February: 1,60,000 May: 1,60,000
March: 2,00,000 June: 1,40,000
8. Interest on Rs. 20,00,000 at 6% on debentures is due by end March and June.
Bu'dgetary Control : 487
9. Excise deposit due in April Rs. 2,00,000.
10. Capital expenditure on plant and machinery planned for June Rs. 1,20,000.
11. Company has a cash balance of Rs. 4,00,000 at 31.12.1980.
12. Company can borrow on monthly basis.
13. Rent is Rs. 8,000 per month. lCA (Fin), November 1980J
Solution:
Monthly C~h Budget of MIs Alpha Manufacturing Company for Six Months ending June
30,1981
Particulars January February March April May June
Rs. Rs. Rs. Rs. Rs. Rs.
Opening balance of Cash 4,00,000 9,07,000 10,34,000 6,51,000 4,00,000 5,50,000
Add: Receipts:
Cash Sales 1,50,000 2,00,000 2,00,000 3,00,000 2,50,000 2,00,000
60% of Credit Sales (0f75%) in
the month after sales 7,20.000 2,70,000 3,60.000 3.60,000 5,40.000 4,50,000·
30% of Credit Sales (of 75%) in
the second month 3,15,000 3,60,000 1,35.000 1,80,000 1,80,000 2.70,000
10% of Credit Sales (of 75%) in
the third month 90.000 1,05,000 1,20,000 45,000 60,000 60,000
Bank Borrowings· - - - 72,000 - 1,28,000
(a) 16,75,000 18,42,000 18,49,000 16,08,000 14,30.000 16,58,000
Payments:
To Suppliers 6,40,000 6,40,000 9,60,000 8,00,000 6,40,000 9,60,000
Wages and Salaries 1,20,000 1,60.000 2,00.000 2,00,000 1,60.000 1,40,000
,
Interest
[20,00,000 x 6% x (3112)] - - 30,000 - - 30,000
Capital Expenditure on Plant and
Machinery - - - - - 1,20,000
Rent 8,000 8,000 8,000 8,000 8,000 8,000
2
Payment of Bank Loan - - - - 72,000 -
Excise Deposit - - - 2,00.000 - -
(b) 7.68,000 8,08,000 11,98,000 12.08.000 8,80,000 12,58,000
:. Closing Balance (a - b) 9.07,000 10,34,000 6,51,000 4,00,000 5,50,000 4.00,000
Management Accounting : 488
Note: 1. Assumed that the- company wants to keep a cash balance of at least Rs. 4,00,000.
Hence, whenever the difference between the cash inflows and the cash outflows falls
short of Rs. 4,00,000, it will borrow from banks ·(e.g., for the month of April,
Rs. 15,36,000 ~ Rs. 12,08,000 = Rs. 3,28,000. Hence, Rs. 72,000 is to be borrowed
from banks). Same arrangement is expected to be made in the month of June 1981 to
borrow Rs. 1,28,000.
2. When cash is above minimum, it is assumed that the bank borrowings will be paid
back. (e.g., in May, closing cash balance is above Rs. 4,00,000 and hence, Rs. 72,000
borrowed during April will be paid during May). '
IDustration: 6.35
A glass manufacturing company requires you to calculate and present the budget for the
year from the following information.
Sales: Toughened Glass: Rs. 3,00,000; and Bent Toughened Glass: Rs. 5,00,000.
Direct material cost: 60% of sales
Direct wages: 20 workers at Rs. 150 per month.
Factory overheads:
Works manager: Rs. 500 per month Light and power Rs.5,000
Foreman: Rs. 400 per month Repairs and maintenance Rs. 8,000
Stores and spares 2.5% on Sales Othe~ sundries: 10% on the direct wages
Depreciation on
machinery Rs.12,600
Administration, selling and distribution expenses: Rs. 14,000 per year.
rCA (Int), 1970J
Solution:
Budgeted Income Statement
Particulars Amount (Rs.)
Sales Revenue: Toughened Glass 3,00,000
. .'
Bent Toughened Glass 5,00,000

.. ," (a) 8,00,000


Less: Cost of Sales:
Direct Material Cost (60% of Sales) 4,80,000
Direct Wages (20 workers x Rs. 150 per month x 12 months) 36,000
Factory Overheads:
Works Manager (Rs. 500 x 12 months) 6,000
Budgetary Control : 489
Foreman (Rs. 400 x 12 months) 4,800
Stores and Spares (2.5% of Sales) 20,000
Depreciation 12,600
Light and Power 5,000
Repairs and Maintenance 8,000
Others (10% of Direct Wages) 3,600 60,000
Administration and Selling and Distribution Expenses 14,000
(b) 5,90J)OO
Therefore, Budgeted Profit (a- b) 2,10,000

mustration: 6.36
A limited company is to be incorporated to take over a running business. It is proposed to
raise Rs. 50 lakh by issuing equity shares and balance of capital required in first six months is to
be financed by a financial institution. The latter is to be given 8% debentures to the nearest Rs. 1
lakh above the'amount required, secured on fixed assets. Initial investment consists of: Freehold
premises: Rs. 20 lakhs; Plant: Rs. 8 lakhs; Stock: Rs. 5 lakhs; and Motor vans and 'other items Rs.
6 lakhs. Payments on account of above items are to be made in the month of incorporation.
Estimates of transactions for the first six months commencing from July are given below:
Sales: July: Rs. 12.50 lakhs; August: Rs. 15.00 lakhs; September: Rs. 18.00 lakhs; October: Rs.
22.00 lakhs; November: Rs. 23.00 lakhs and for each of the next three months, Rs. 24.00
lakhs.
Gross profits on sales to be at the rate of 20%
Debtors: To get two months credit.
Creditors: To be paid at the end of the month following the month of purchase.
Expenses:
a. Preliminary expenses Rs. 50,000 to be paid in August.
b. General expenses Rs. 40,000 per month payable at the end of each month.
c. Wages and salaries payable on the first day of next month, Rs. 70,000 for each offIrst
three months and Rs. 85,000 per month thereafter.
Assume that shares and debentures are issued on 1st July. Draw up a cash budget and
budgeted final accounts up to 31 st December, and balance sheet as on that date.
lCA (Fin), May 1971J
Management Accounting : 490

Solution:
Monthly Cash Budget for Six Monfhs, July-December (Rs.)
Particulars July Aug Sep Oct Nov Dec Total

Opening Balance - 11.60.000 7(}'oOO 80,000 1.00,000 1.00,000 -


Add: Receipts:
Issue of Equity Shares 50,00,000 - - - - 50,00,000
Issue of 8% Debentures 2
1,00,000 - - - - 1.00,000
Collection from
Customers - - 12,50,000 15,00,000 18,00,000 22,00,000 67,50,000
(a) 51.00,000 11,60,000 l3,20,OOO 15,80,000 19,00,000 23,00,000 1,18,50,000
Less: Payments:
Purchase of Freehold
Premises, etc
39,00,000 - - - - - 39,00,000

Payments to Suppliers
of Materials' - 9,30,000 11,30,000 l3,70,OOO 16,75,000 17,55,000 68,60,000

Preliminary Expenses - 50.000 - - - - 50,000


General Expenses 40,000 40,000 40,000 40,000 40.000 40,000 2,40,000
Wages and Salaries - 70,000 70,000 70,000 85,000 85,000 3,80,000
(b) 39,40.000 10,90,000 12.40,000 14.80,000 18,00,000 18,80,000 1,14,30,000
Closing Balance (a - b) 11.60,000 70.000 80,000 1.00,000 1,00,000 4,20,000 4,20,000

1. Payments to Suppliers (Rs)


Particulars July Aug Sep Oct Nov Dec Total
Sales Revenue 12,50,000 15,00,000 18,00,000 22,00,000 23.00.000 24,00,000 1,14,50,000
Less: Gross Profit 20% 2,50,000 3,00,000 3,60,000 4,40,000 4,60,000 4,80,000 22,90,000
Cost of Sales 10,00,000 12,00,000 14,40,000 17,60,000 18,40,000 19,20,000 91,60,000
Less: Wages & Salaries 70,000 70.000 70,000 85,000 85,000 85,000 4,65,000
Therefore. Purchases 9,30,000 11,30,000 13,70,000 . 16,75,000 17,55,000 18,35.000 86,95,000

2. Since the available resources fall short during August, Rs. 1 lakh Debentures were assumed to
have been issued during July.
Budgetary Control: 491

Dr Budgeted Profit and Loss Ale for Six-Month Period ending December 31, Cr
Amount Amount
Particulars Particulars
Rs. Rs.
To Stock 5,00,000 By Sales (12.5+15+18+22+23+24) 1,14,50,000
To Purchases 86,95,000 By Closing Stock l 5,00,000
To Wages and Salaries
[(70,000 x 3) + (85,000 x 3)]
4,65,000
To Gross Profit cld
22,90,000
1,19,50,000 1,19,50,000
To General Expenses (40,000x6)
2,40,000 By Gross Profit bId 22,90,000
To Depreciation on Freehold
Premises, etc
(assumed at 10%)
[34 lakh x 10% x (6/12)] 1,70,000
To Debenture Interest
[1,00,000 x 8% x (6/12)] 4,000
To Net Profit cld 18,76,000
22,90,000 22,90,000
Note: 1. Assumed to be maintaming the stock level at Rs. 5lakh throughout.
Balance Sheet as on December 31,
Amount Amount
Capital and Liabilities Assets and Properties
Rs. Rs.
Equity Share Capital 50,00,000 Freehold Premises 20,00,000
8% Debenture Capital 1,00,000 Less: Depre @ 10% 1.00,000 19,00,000
P & L Nc Balance 18,76,000 Plant 8,00,000
Creditors (December Purchases) 18,35,000 Less: Depre @ 10% 40,000 7,60,000
Outstanding Expenses: Motor Van 6,00,000
Debenture Interest 4,000 Less: Depre @ 10% 30,000 5,70,000
Wages and Salaries (December) 85,000 Preliminary Expenses 50,000
Stock 5,00,000
Debtors (November and
December Sales) 47,00,000
Cash 4,20,000
89,00,000 89,00,000
Management Accounting : 492 •

Dlustration: 6.37
From the following information relating to 1963 and conditions expected to prevail in
1964, prepare a budget for 1964. Also state the assumptions you have made.
1963 Acutals: 1964 Expected:
Sales (40,000 units) Rs. 1,00,000 Sales (60,000 units): Rs. 1,50,000
Raw materials 53,000 Raw materials: 5% price increase
Wages 11,000 Wages: 10% increase in wage rates
Variable overheads 16,000 5% increase in productivity
Fixed overheads 10,000 Additional plant: One lathe Rs. 25,000
One drill Rs. 12,000
[ICWA (lnt), January 19641
Solution:
Budgeted Income Statement for 1964
Particulars Actuals for Budget for
1963 (Rs.) 1964 (Rs.)
Sales Quantity (units) 40,000 60,000
Cost of Materials I 53,000 83,475
Wages2 11,000 17,286
3
Variable Overheads 16,000 24,000
Variable Cost 80,000 1,24,761
Sales Revenue 1,00,000 1,50,000
Therefore, Contribution 20,000 25,239
Less: Fixed Cost4 10,000 13,700
Therefore, Profit 10,000 11,539

Note: 1. Unit Material Cost (1963) = [Rs. 53,000 + 40,000 units] = Rs. 1.32500
Add: 5% increase due to price increase =0.06625
:. Unit Material Cost for 1964 = 1.39125
Total Material Cost = [Rs. 1.39125 x 60,000 units] = Rs. 83,475
2. Unit Labour Cost (1963) = [Rs. 11,000 + 40,000 units] =0.2750
Add: 10% increase due to wage rate jncrease =0.0275
= 0.3025
Budgetary Control : 493

Unit Labour Cost for 1964 =[(0.3025 x 100) + 105] (i.e., 5%


increase in labour productivity) : 0.2881
Therefore, for 60,000 units = (Re. 0.2881 x 60,000 units) = Rs. 17,286
3. Variable Overheads = [(Rs. 16,000 + 40,000 units) x 60,000 units I = Rs. 24,000
4. Fixed Overheads =Rs. 10,000 + [(Rs. 25,000 + Rs. 12,000) + 10 years]
= [Rs. 10,000 + Rs. 3,700] = Rs. 13,700. (assumed 10 years useful life
for the new fixed assets with no scrap value. Further, it is
assumed that the company uses the Straight Line Method.

mustration: 6.38
Jamuna Printing Company Private Limited enqed with the following profitlloss during the
.year 1976 (all figures in Rs.lakhs).
Sales 35.58
Less: Expenses: Raw material 7.42
Stores 4.88
Expenses 20.40
Interest 2.00
Depreciation 2.00 36.70
:. Loss for the year 1.12

The Press had been working at 60% capacity during 1976. Of the expenses of Rs. 20.40
lakhs, 25% is variable. In 1977, production/sale volume at 80% of capacity is expected to be
achieved. Fixed cost is however expected to. increase by 1.20 lakh. Draw the 1977 Budget.
[ICWA (Int), December 1976J
Solution:
Budgeted Income Statement of Jamuna Printing Company for 1977
Particulars Amount (Rs.)
Sales Revenue l 47,44,000
(a) 47,44,000
Less: Total Cost: Raw materials2 9,89,333
Stores3 6,50,667
Expenses: Variable (25%t 6,80,000
Fixed (75%i 16,50,000
Interest 2,00,000
Management Accounting : 494

Depreciation 2,00,000
(b) 43,70,000
Therefore, Profit (a - b) 3,74,000

Working Notes:
1. [(Rs. 35.58 + 60) x 80] 4. 25% of 20,40,000 =5,10,000 is variable at 60%
2. [(Rs. 7.42 + 60 x 80] :. at 80% = [5,10,000 + 60) x 80]
3. [(Rs. 4.88 + 60) x 80] 5. 75% of 20,40,000 = 15,30,000
Add: increase 1,20,000
16,50,000
Illustration: 6.39
Carryon Ltd., manufactures two products Band T. It is going to prepare its budget for the
year ending 31 st December 1988. Expectations for 1988 include the following.
a. Opening balance: Fixed assets: Land Rs.20,OOO
Buildings and plant 1,50,000
Cumulati ve depreciation (60,000) Rs.l,IO,OOO
Current assets: Stock: Raw materials 3,000
B 3,400
T 7,200
l3,600
Debtors 45,000
Cash/Bank 10,000
Less: Current liabilities: Creditors 29,000 68,600
Taxation 28,000 57,000 11,600
1,21,600
Represented by: Share capital 71.600
Retentions 50,000 1,21,600
b. Finished products: B T
Budgeted sales (units) 5,000 1,000
Budgeted selling price (per unit) Rs.30 Rs.50
Opening stock of finished goods (in units) 200 300
Budgeted closing stock (in units) 1,200 200
Budgetary Control : 495

c. 1. Direct material per unit of production: P Q


Raw material per unit of production: B 5 kg 3 kg
T 2 kg 4kg
Opening stock of raw materials 2,000 kg 2,000 kg
Budgeted closing stocks 2,500 kg 1,500 kg
Cost per kg of material purchased Re.0.50 Re.l.00
2. Direct labour: Labour is paid at the rate of Rs. 2 per hour, 3 direct labour hours are
required to produce one unit of Band 5 labour hours are required for one unit of T.
3. Factory overheads: It has been estimated that factory overheads will be Rs. 33,750
including Rs. 11,750 for depreciation. Factory overheads are absorbed on a direct
labour hour basis.
4. Work in progress: This is negligible and can be ignored (i.e .. no opening or closing
stocks)
d. Administration overheads: These are estimated to be Rs. 11,625. They are charged to
goods leaving work-in-progress and entering finished goods stocks and are absorbed as a
percentage of factory cost.
e. Selling and distribution overheads: These are estimated to be Rs. 20,000. They are
charged to the cost of sales on the basis of a percentage of the selling price.
f. Stock pricing: Goods are priced on FIFO basis. Opening stock of raw materials are
2,000 kgs of Pat Re. 0.50 (Rs. 1,000) and 2,000 kgs of Qat Re. 1 (Rs. 2,000)
g. Taxation in 1988 on profit is estimated at Rs. 30,000. Overdraft interest in 1988 is
expected to be Rs. 595, Rs. 500 of which will be paid during the third quarter of the year.
You are required to prepare the following budgets for 1988 for Carryon Ltd., using
Absorption Costing method:
a. Sales budget b. Production budget (Quantities only)
c. Direct materials budget (usage and purcha<;e) d. Direct labour budget
e. Overhead absorption rate f. Closing stock budget
g. Cost of goods sold budget h. Budgeted profit and loss account
[ICWA (Fin), December 1987]
Management Accounting : 496

Solution:
a. Sales Budget .
Sales Unit Price Budgeted
Product
Units Rs. Sales Revenue (Rs.)
B 5,000 30 1,50,000
T 1,000 50 50,000
2,00,000

b. Production Budget (units)


Particulars Product B ProductT
Budgeted Sales 5,000 1,000
Add: Closing Stock 1,200 200
6,200 1,200
Less: Opening Stock 200 300
Therefore, Budgeted Production 6,000 900

c. Material Budget
Particulars Material P Material Q
Number of kgs of Raw materials required for budgeted
production of (kgs): -
B: 6,000 units @ 5 kgs and 3 kgs 30,000 18,000
T: 900 units @ 2 kgs and 4 kgs 1,800 3,600
Add: Closing Stock 2,500 1,500
34,300 23,100
Less: Opening Stock 2,000 -, 2,000
:. Purchase (kgs) to be made 32,300 21,100
Purchase Price per kg (Re.) O.S 1
Therefore, Total Purchase Bill (Rs.) 16,150 21,ioo

d. Direct Labour-,Budget
.

Particulars ProductB Product T


Output (units) 6,000 900
'(
Direct Labour Hours per unit 3 5
Budgetary Control : 497

Therefore, Total number of Direct Labour Hours required i8,000 4,500


Therefore, Direct Labour Cost (at Ri;. 2 an hour) (Rs.) . 36,000 9,000

e. Overhead Ab} _ [ Rs.33,750


sorption Rate - 22,500 hours
J-- Rs. 15.
Calculation of Factory Cost and other Overhead Expenses
Particulars ~ (Rs.) T (Rs.) Total (Rs.)
Material Cost:
P: (6,000 x 5 x 0.5; 900 x 2 x 0.5) 15,000 .900 15,900
Q: (6,000 x 3 x 1; 900 x 4 xl) 18,000 3,600 21,600
33,000 4,500 37,500
Direct Labour Cost: (6,000 x 3 x 2: 900 x 5 x 2) 36.000 9.000 45.000
Factory Overheads: (18,000 x 1.5; 4,500 x 1.5) 27,000 6,750 33,750
Therefore, Factory Cost 96.000 20,250 1,16,250
Add: Administration OREs (10% of Factory Cost)l 9,600 2,025 11,625
Office Cost 1,05,600 22,275 1,27,875
S & DOHEs (10% of Sales Revenue)2 15,000 5,000 20,000
Note: 1. (Rs. 11,625 AOREs + Rs. 1,16.250 Factory Cost) = 10%
2. (Rs. 20,000 S & DOREs + Rs. 2,00,000 Revenue) =10%
f. Closing Stock Budget
Rate per Value of Closing
Particulars Units/kgs
unit/kg (Rs.) Stock (Rs.)
1. Raw Material: P 2,500 0.50 1,250
Q 1,500 1.00 1,500
2,750
2. Finished Goods: B 1,200 17.601 21,120
T 200 24.75 4,950
Grand total (Rs. 28,820) 26,070
Note: 1. B: (1,05,600 + 6,000) =Rs. 17.6; T: (22,275 + 900) =Rs. 24.75
Management Accounting : 498
g. Cost of Goods Sold Budget
Prod. B Prod. T Total
Particulars
(Rs.) (Rs.) (Rs.)
Cost of Opening Stock of Finished Goods (200 units & 300 units) 3,400 7,200 10,600
Add: Office Cost of units produced during the current period
(6.000 and 900 units) 1,05,600 22,275 1,27.875
1,09,000 29,475 1.38,475
Less: Cost of Closing Stock (1200 and 200 units) 21,120 4.950 26,070
87,880 24,525 1,12,405
Add: Selling and Distribution Overheads 15,000 5.000 20,000
Therefore. Cost of Sales 1,02,880 29,525 1,32,405

h. Budgeted Profit and Loss Statement


Particulars Amount (Rs.)
Sales Revenue: B: Rs. 1,50,000
T: 50,000 2,00,000
Less: Cost of Sales: B: Rs. 1,02,800
T: 29,525 1,32,405
Profit before Interest and Tax 67,595
Less: Interest 595
Earning before Tax 67,000
Less: Tax 30,000
Therefore, Budgeted Profit after Tax 37,000

Illustration: 6.40
The following details have been extracted from the books of a company for the year
ended 31st March, 1989.
Sales Rs. 13,60,800 Factory overheads Rs. 1,40,000
Direct materials 4,00,000 Administration overheads 1,68,000
Direct wages 2,80,000 Selling overheads 1,26,000
Direct expenses 20,000 Profit 2,26,800
50% of factory overheads and 60% of selling overheads are analysed as variable. The
forecasts for the next year are as under (Le., year ending 311311990).
a. Sales volume will increase by 30% but the selling price will be reduced by 5%.
Budgetary Control : 499

b. The raw material prices will remain unchanged but because of increased purchases, a
quantity discount of 5% will be obtained.
c. Variable overheads (selling and factory), direct wages and expenses will increase in
proportion to sales volume.
d. Wages and factory overheads (variable only) will further go up by 10% for increase in
rates.
e. Administration overheads will decrease by 2%.
f. There will be no stocks of work-in-progress and finished goods in the beginning or end of
the year.
Required:
a. Prepare a budget for the next year Le., year ending 31 st March, 1990.
b. Establish rates of recovery offactory, administration and sellin~ overheads.
c. The company is required to quote for an export order. The direct materials, direct wages
and direct expenses -relating to this order are Rs. 50,000, Rs. 40,000 and Rs. 10,000
respectively. Find the minimum sales value of the export order.
[ICWA (Fill), June 1989J
Solution:
Budgeted Income Statement for the next year, March 31, 1990
Amount
Particulars
Rs.
Sales Revenue [(13.60,800 x (1301100) x (95/100)] 16,80,588
Less: Variable Costs:
Direct Material cost [4,00,000 x (1301100) x (951100)] Rs. 4,94,000
Direct Wages [2,80,000 x (1301100) x (1101100)] 4.00.400
Direct Expenses [20,000 x (1301100)] 26,000
Prime Cost 9.20.400
Variabie Overheads:
Factory [1,40,000 x (50/100) x (130/100) x (110/100) 1,00,100
Selling [1,26,000 x (601100) x (130/100)] 98,280 11,18,780
Therefore, Contribution 5,61,808
Less: Fixed Costs:
Factory [1,40,000 x (501100)] 70,000
Administration [1,68,000 x (98/100)] 1,64,640
Selling [1,26,000 x (40/100)] 50.400 2,85,040
Therefore, Budgeted Profit 2,76,768
Management Accounting : 500

Overhead Recovery Rates:


a. Factory Overhead Expenses as a percentage of Direct Wages =
Variable = (Rs. 1,00,100 + Rs. 4,00,400) =25.00%
Fixed = (Rs. 70,000 + Rs. 4,00,400) =17.48%
Total = (Rs. 1,70,100 + Rs. 4,00,400) =42.48%
b. Administration Overhead Expenses as a percentage of Factory Cost =
[Rs. 1,64,640 + Rs. 10,90,500] = 15.1 %
c. Selling and Distribution Overhead Expenses as a percentage of Factory Cost:
Variable = (Rs. 98,280 + Rs. 10,90,500) = 09.01 %
Fixed = (Rs. 50,400 + Rs. 10,90,500) = 04.62%
Total = (Rs. 1,48,680 + Rs. 10,90,500) = 13.63%
Minimum Sales Value of the Export Order:
Direct Material Cost Rs.50,000
Direct Labour Cost 40,000
Direct Expenses 10,000
Prime Cost 1,00,000
Variable Overhead Expenses:
Factory (25% of wages) 10,000
Selling and Distribution [9.01 % of (1,10,000 + 6,992*)] 10,541
1,20,541
Note: * 17.48% of Direct wages Rs. 40,000
Dlustration: 6.41
SV Ltd., engaged in manufacturing four products, has prepared the following budget for
the current year.
Products
A B C D
Production (units) 20,000 5,000 25,000 15,000
Selling price (Rs/unit) 21.75 36.75 44.25 64.00
Direct material (Rslunit) 6.00 13.50 10.50 24.00
Direct wages (Rs/unit) 7.50 10.00 18.00 24.00
Variable overheads (Rs/unit) 2.25 5.00 6.00 6.50
Fixed overheads (Rs. per annum) 75,000 25.000 2,25,000 1,80,000
Budgetary Control : 501

When the budget was discussed, it was proposed that the production should be increased
by 10,000 ~nits for which the capacity already existed. It was also decided that for the next year,
the production capacity ·should be further increased by 25,000 units over and above the increase of
10,000 units envisaged in the current year. The additional production capacity of 25,000 units
should be used for the manufacture of product B for which new production facilities were to be
created at an annual fixed overhead cost of Rs. 35,000. The direct material costs of all the four
products were expected to increase by 10% in the next year, while the other costs and selling
prices would remain the same.
a. Find the profit of current year on the assumption that the existing capacity of 10,000 units
is utilised to maximize the profit.
b. Prepare a statement of profit for the next year.
c. Assuming that the increase in the output of product B may not fully materialize in the
next year, ,find the number of units of product B to be sold next year to earn the same
overall profit as in the current year. rCA (Fin), May 1989J
Solution:
Comparative Unit Contribution Statement (Current Year)
Particulars A (Rs.) B (Rs.) C (Rs.) D (Rs.)
Direct Material Cost 6.00 13.50 10.50 24.00
Direct Wages 7.50 10.00 18.00 24.00
Variable Overheads 2.25 5.00 6.00 6.50
Unit Variable Cost 15.75 28.50 34.50 54.50
Unit Selling Price 21.75 36.75 44.25 64.00
Unit Contribution 6.00 8.25 9.75 9.50

From the above, it is obvious that product C is more profitable than any other product.
Hence, idle capacity which exists now is to be utilized to produce an additional 10,000 units of
product C (Le., 25,000 units + 10,000 units =35,000 units of C).
a. Computation of Profit of Current Year
Sales Volume Unit Contri- Total Contri- Fixed Cost Profit
Product
(units) bution (Rs.) bution (Rs.) (Rs.) (Rs.)
A 20,000 6.00 1,20,000 75,000 45,000
B 5,000 8.25 41,250 25,000 16,250
C 35,000 9.75 3,41,250 2,25,000 1,16,250
D 15,000 9.50 1,42,500 1,80,000 (-) 37,500
6,45,000 5,05,000 1,40,000
Management Accounting : 502

Product: A B C D
Sales units (next year): 20,000 30,000 35,000 15,000
Unit Material Cost (110%): 6.60 14.85 1l.55 26.40
b. Budgeted Income Statement for the Next Year
Particulars A (Rs.) B (Rs.) C (Rs.) D (Rs.) Total Rs.
Direct Material Cost 1,32,000 4,45,500 4,04,250 3,96,000 13,77,750
Direct Wages 1,50.000 3,00,000 6,30,000 3,60,000 14,40,000
Variable Overheads 45,000 1,50,000 2,10,000 97,500 5,02,500
Total Variable Cost 3,27,000 8,95,500 12,44,250 8,53,500 33,20,250
Sales Revenue 4,35,000 11,02,500 15,48,750 9,60.000 40,46,250
Contribution 1,08,000 2,07,000 3,04,500 1,06,500 7,26,000
Less: Fixed Cost (B: increased by
Rs.35,OOO) 75,000 60,000 2,25,000 1,80,000 5,40,000
Profit 33,000 1,47,000 79,500 -73,500 1,86,000

c. Desired Profit for next year = Profit for current year = Rs. 1,40,000
Less: Contribution towards profit from products: A: 33,000
C: 79,500
D: -73,500 39,000
:. Expected Profit from Product B 1,01,000
Therefore, contribution from B = Rs. 1,01,000 + Rs. 60,000 (fixed cost) = Re. 1,61,000;
B's unit contribution = Rs. 6.9 (Le., Rs. 2,07,000 + 30,000 units). Therefore, number of units to
be sold, next year = 1,61,000 + 6.9 = 23,334 units. Therefore, 23,334 units (approxim~tely) of B
are to be sold to earn the same overall profit as in the current year.
Illustration: 6.42
ABC firm manufactures cakes in three varieties viz., A. B and C each requiring similar
material, labour and production facilities. The trading results of the firm for current year ending
March are as under.
Particulars A (Rs.) B (Rs.) C (Rs.) Total (Rs.)
Sales 21,44,000 17,20,000 10,40,000 49,04,000
Variable Cost: Material 5,36,000 5,16,000 4,16,000 14,68,000
Budgetary Control : 503

Wages 4,28,800 2,58,000 3,36,000 10,22,800


Overheads 4,28,800 2,58,000 3,36,000 10,22.800
Total 13,93,600 10,32,000 10,88,000 35,13,600
:. Contribution 7,50,400 6,88,000 (48,000) 13.90,400
Fixed Overheads 8,70,400
Profit 5,20,000

The cake of variety C, despite best efforts, does not yield the desirable margin and thus
the firm has decided to discontinue its production. However, the other two varieties, A and B,
have good potential to grow and the market can easily absorb the increased production. The firm
has, therefore, decided to raise the production of these varieties by diverting the labour and
production facilities engaged in production of variety C. In accordance with this, it is decided,
effective 1st April, to transfer two-thirds of the labour engaged in variety C to variety A and the
remaining one-third to variety B, thus totally discontinuing production of variety C. The
following data for the next year beginning April is also available.
a. Total direct wage bill for the next year would be at the same level as for the last year
ending March.
b. Similarly, the variable costs per unit of production and the selling price are to be assumed
unchanged in the forthcoming year beginning April.
c. Fixed overheads would increase by Rs. 55,200.
Your are required:
a. to prepare the budget for nexryear beginning April in the same format as given above,
b. to analyse and compare the budget for the year beginning April with that of the previous
year and highlight main features, and
c. to advise the management on comparative profitability if two-thirds of the workers
engaged in C are transferred to B instead of A and the remaining one-third are engaged in
A instead of B. Give detailed reasoning for your advice. [ICWA (Fin), June 1989]
Solution:
Based on the cost structure for the current period, Variable Cost Ratios can be calculated
as below (Le., ratio of each item of variable cost to sales revenue):
Particulars A (0/0) B (0/0)
Material Cost Ratio 25 30
Wages Ratio 20 15
Overhead Ratio 20 15
65 60
Management Accounting : 504

Therefore, Plv Ratio (100 - 65; 100 - 60) 35 40


Sales Ratio 100 100

It is given in the problem that the unit variable costs and the selling prices are to be
assumed unchanged in the forthcoming year. Hence, the sales revenue for the forthcoming year
can be worked out as under: Wages of Product:
A: Rs. 4,28,800 + [(2/3) x 3,36,000] =Rs. 6,52,800. This is 20% of Sales Revenue.
Therefore, Sales Revenue = [Rs. 6,52,800 + 20%] = Rs. 32,64,000.
B: Rs. 2,58,000 + [(113) x 3,36,000] = Rs. 3,70,000. This is 15% of Sales Revenue.
Therefore, Sales Revenue =[Rs. 3,70,000 + 15%] =Rs. 24,66,667.
Based on the above Sales Revenue and the Variable Cost Ratios, Budgeted Income
Statement for the forthcoming year can be prepared as below.
a. Budgeted Income Statement for the Next Year beginning April
Product Total
Particulars
A (Rs.) B (Rs.) (Rs.)
Sales Revenue 32,64,000 24,66,667 57,30,667
(a) 32,64,000 24,66,667 57,30,667
Less: Variable Costs:
Material Cost (25% and 30%) 8,16,000 7,40,000 15,56,000
Wages (20% and 15%) 6,52,800 3,70,000 10,22,800
Overheads (20% and 15%) 6,52,800 3,70,000 10,22,800
(b) 21,21,600 14,80,000 36,01,600
Therefore, Contribution (a - b) 11,42,400 9,86,667 21,29,067
Less: Fixed Costs (Rs. 8,70,400 + Rs. 55,200) 9,25,600
Therefore, Profit 12,03,467

b. A few yardsticks listed below highlight,the overall improvement that the company expects to
achieve in the next year beginning April when compared to the current year ending March.
Current year Next year Change (%)
Total Sales Revenue Rs.49,04,OOO Rs.57,30,667 16.86
Total Contribution Rs. 13,90,400 Rs.21,29,067 53.13
Total Profit Rs. 5,20,000 Rs. 12,03,467 131.44
Composite Plv Ratio 28.35% 37.15%
Overall Profit Ratio 10.60% 21.00%
Budgetary Control : 505

c. If 2/3 of the employees engaged in C are transferred to B and the remaining 1I3rd to A, the
performance of the company in the forthcoming year will be much better as B is more
profitable (with 40% Plv Ratio) than A (with the Plv Ratio of only 35%). This is evident
from the following calculations and the income statement. Wages of Product:
A = Rs. 4,28,800 + [(1/3) x 3,36,000] = Rs. 5,40,800.
Therefore, Sales Revenue from A =[5,40,800 + 20%] =Rs. 27,04,000.
B: Rs. 2,58,000 + [(2/3) x 3,36,000] = Rs. 4,82,000
Therefore. Sales Revenue from B: [Rs. 4,82,000 + 15%] = Rs. 32,13,333.
Budgeted Income Statement for the Next Year beginning April
Product Total
Particulars
A (Rs.) B (Rs.) (Rs.)
Sales Revenue (a) 27,04,000 32,13,333 59,17,333
Less: Variable Costs:
Material Cost (25% and 30%) 6,76,000 9,64,000 16,40,000
Wages (20% and 15%) 5,40,800 4,82,000 10,22,800
Overheads (20% and 15%) 5,40,800 4,82,000 10,22,800
(b) 17,57,600 19,28,000 36,85,600
Therefore, Contribution (a - b) 9,46,400 12,85,333 22,31,733
Less: Fixed Costs (Rs. 8,70,400 + Rs. 55,200) 9,25,600
Therefore, Profit 13,06,133

IDustration: 6.43
The selling prices are fixed under control but raw materials have to be procured from the
open market. The material prices are stable but principal material is not available freely. Prepare
a budget for 1977 from the following data for 1976 with the aim of optimizing the operating
profit.
Product A Product B
Particulars Territory Territory Territory Territory Territory Territory
-1 (Rs.) -2 (Rs.) -3 (Rs.) -1 (Rs.) -2 (Rs.) -3 (Rs.)
Units sold 10,000 15,000 8,000 15,000 12,000 5,000
Sales value 1,00,000 1,65,000 96,000 1,80,000 1,44,000 65,000
Contribution 30,000 49,500 24,000 45,000 43,200 13,000
Profit 10,000 12,500 6,000 12,500 11,520 3,250

Other data:
Management Accounting: 506
1. Consumption of principal material: 1 kg in Product A and 1.25 kgs in Product B (per
unit).
2. Availability of principal material: same as 1976.
3. Scope for changing the sales mix: Revised volume not to fall below 50% for either
product or exceed 125% of the current volume.
Ignore the impact of revised sales mix on the revised costs. [ICWA (Fin), June 1977J
Solution:
1. Calculation of availability of Principal Material:
Sales during 1976 of:
Product A: [33,000 units x 1 kg] = 33,000 kgs
Product B: [32,000 units x 1.25 kgs] =40,000kgs
,
:. Total availability = 73,000 kgs
2. Calculation of products' profitability (in the light of Principal Material):

Contribution per kg of Principal Material: (Rs.)

Territory
2 3
Product: A (1 kg per unit) 3/1 = 3.0 3.311 = 3.30 311 = 3.00
Product: B (1.25 kgs per unit) 3/1.25 = 2.4 3.6/1.25 = 2.88 2.6/1.25 = 2.08
3. Resource Allocation: It is obvious from the above that Product A is more profitable than B.
Hence, its sale is to be maximized. In case of B, its sale is to be maximized in Territory - 2
followed by in Territories - 1 and 3 depending upon the availability of Principal Material and
min-max sales mix.
(
. Number of kgs of Principal Material available 73,000
Less: Number of kgs of Principal Material required to produce 125% of
1976's sales of Product A:
Territory 2: [(15,000 + 25%) x 1 kg] = (18,750 x 1) 18,750
54,250
1: [(10,000 + 25%) x 1 kg] = (12,500 x 1) 12,500
41,750
3: [(8,000 + 25%) x 1 kg] = (10.000 x 1) 10,000
31,750
Number of kgs of Principal Material required to produce B:
Territory 2: [(12,000 + 25%) x 1.25 kg] = (15,000 x 1.25) 18,750
Budgetary Control : 507
13,000*
3: [(5,000 - 50%) x 1.25] = (2,500 x 1.25) 3,125
With this, 9,875 kgs of Material, it can produce 7,900 units of B for
Territory - 1 9,875

Note: * With this 13,000 kgs, it is not possible to produce either 15,000 units (it needs 18,750
kgs) or 18,750 units (it requires 23,437.5 kgs of Material) of B for Territory-I. Hence,
minimum of B for Territory - 3 is calculated and remaining can be utilized to produce B
for Territory - 1.

Budgeted Income Statement for 1~77

Product A ProductB
Particulars Total
Terr. 1 Terr.2 Terr.3 Total Terr. 1 Terr.2 Terr.3 Total
Sales Volume
(units) 12.500 18.750 10.000 41.250 7.900 15.000 2.500 25.400 66.650

Selling Price (Rs.) 10 11 12 12 12 13

Unit Variable Cost


(Rs.)
7 7.7 9 9 8.4 10.4

Fixed Cost (Rs.)


20.000 37,000 18.000 75.000 32,500 31.680 9.750 73.930 1,48.930

Sales Revenue
1.25.000 2.06.250 1,20.000 4.51.250 94.800 1.80.000 32.500 3.07.300 7,58.550

Less: Variable
Cost
87.500 1.44.375 90.000 3.21.875 71.100 1.26.000 ·26.000 2,23.100 5.44.975

Contribution 37.500 61.875 30.000 1.29.375 23.700 54.000 6.500 84.200 2.13.575

Less: Fixed Cost 20.000 37.000 18.000 75.000 32.500 31.680 9.750 73.930 1,48.930

Budgeted Profit for


1977 17.500 24.875 12.000 54.375 -8.800 22.320 -3.250 10.270 64.645

Profit earned during


1976
10.000 12.500 6.000 28.500 12.500 11.520 3.250 27.270 55.770

Impact on Profit 8.875


7.500 12.375 6.000 25.875 -21.300 10.800 -6.500 -17.000

Unit Variable Cost =[(Sales Revenue - Contribution) + Sales Quantity]


Fixed Cost =(Contribution - Profit)
mustration: 6.44
Manufactures Ltd., produces three products from three basic raw materials in three
departments. The company operates a budgetary control system and values its stock of finished
Management Accounting : 508

goods on a total cost basis. From the following data, you are required to produce the following
budgets for the month of July: (a) Production, (b) Material Usage, (c) Purchases, and (d) Profit
& Loss Account for each product and in total. Budgeted data for July are as follows.
Products
Particulars
A B C
Sales (Rs.) 15,00,000 10,80,000 16,80,000
Stock of finished products at July 1. (in units) 3,000 2,000 2,500
Departments
I II III
Production overheads (Rs) 2,39,000 2,01,300 3,91,200
Directlabour hours 47,800 67,100 65,200
Direct Material
Ml M2 M3
Stock at July 1, (in units) 24,500 20,500 17,500

The company is introducing a new system of inventory control which would reduce
stocks. The forecast is that stocks as at July 31st will be reduced as follows: raw materials by
10% and finished products by 20%. Fixed production overhead is absorbed on a direct labour-
hour basis. It is expected that there will be no work-in-progress at the beginning or end of the
month. Administration costs are absorbed by products at a rate of 20% of production cost, and
selling and distribution costs are absorbed by products at a rate of 40% of production cost. Profit
is budgeted as a percentage of total costs as follows: Product A 25%, Product B 12.5% and
Product C 162/3%. Standard cost data per unit of product is as follows.
Price per unit Products
(Rs.) A (units) B (units) C (units)
Direct Material: M 1 2.00 5 12
M2 4.00 10 9
M3 1.00 5 5
Rate per
hour (Rs.) Hours Hours Hours
Direct Wages: Department I 2.50 4 2 2
Department II 2.00 6 2 3
Department III 1.50 2 4 6
Other Variable Costs (Rs.) 10 20 15
lCA (Fin), May 1986J
Budgetary Control: 509

Solution:
Budgeted Income Statement (per unit basis)
Product (Rs.)
Particulars
A B C
Direct Material Cost: Ml: 5, 0,12 @ Rs. 2 10 0 24
M2:0, 10,9@Rs.4 0 40 36
M3:5,5,0@Re. 1 5 5 0
Total Material Cost per unit (a) 15 45 60
Add: Direct Wages: Department: I: 4, 2, 2 @ Rs. 2.5 10 5 5
II: 6, 2, 3 @ Rs. 2 12 4 6
III: 2,4,6 @ Rs. 1.5 3 6 9
Total Direct Wages per unit (b) 25 15 20
Therefore, Prime Cost (a + b) 40 60 80
Add: Other Variable Costs 10 20 15
Fixed Production Overheads I
Department I: 4, 2, 2 @ Rs. 5 20 10 10
II: 6, 2, 3 @ Rs. 3 18 6 9
III: 2, 4, 6 @ Rs. 6 12 24 36
Therefore, Unit Production Cost 100 120 150
Add: Administration Costs (20% of Production Cost) 20 24 30
Selling and Distribution Costs (40% of Production Cost) 40 48 60
Therefore, Unit Cost 160 192 240
Add: Desired Profit (25%, 12.5%, 162/3% of Total Cost) 40 24 40
Unit Selling Price 200 216 280
Estimated Sales Revenue (Total) (Rs. 00,000) 15 10.8 16.8
Therefore, Sales Volume (units) (i.e., Revenue + Price) 7,500 5,000 6,000
Management Accounting: 510
Note: 1. Fixed Production Overhead Expenses Absorption Rates:
Direct Absorption
Deptart- Overheads
Labour
ment (Rs.) Rate (Rs.)
Hours
I 2,39,000 47,800 5
II 2,01,300 67,100 3
ill 3,91,200 65,200 6

a. Production Budget, July


Product (units)
Particulars
A B C
Estimated Sales 7,500 5,000 6,000
Add: Closing Stock (20% lower than Opening Stock) 2,400 1,600 2,000
Therefore. Total Requirements 9,900 6,600 8,000
Less: Opening Stock 3,000 2,000 2,500
Therefore, Budgeted Production 6,900 4,600 5,500

b. Material Usage Budget


Material (units)
Particulars
Ml M2 M3
Material requirements for budgeted production:
Product A (6,900 x 5, 0, 5) 34,500 0 34,500
B (4,600 x 0, 10,5) 0 46,000 23,000
C (5,500 x 12, 9, 0) 66,000 49,500 0
Total Material Usage 1,00,500 95,500 57,500

c. Purchase Budget
Material (units)
Particulars
Ml M2 M3
Material requirement for budgeted production 1,00,500 95,500 57,500
Add: Closing Stock (10% lower than Opening Stock) 22,050 18,450 15,750
Total requirements 1,22,550 1,13,950 73,250
Less: Opening Stock 24,500 20,500 17,500
Purchases 98,050 93,450 55,750
Budgetary Control: 511

Purchase Price per unit (Rs.) 2 4 1


Therefore, Total Purchase Bill (Rs.) 1,96,100 3,73,800 55,750

d. Budgeted Product-wise and Total Income Statement


Products
Particulars Total (Rs.)
A (Rs.) B (Rs.) C (Rs.)
Sales Revenue 15,00,000 10,80,000 16,80,000 42,60,000
Less: Cost of Sales
A: (7,500 x 160) 12,00,000
B: (5,000 x 192) 9,60,000
C: (6,000 x 240) 14,40,000 36,00,000
Profit 3,00,000 1,20,000 2,40,000 6,60,000

mustration: 6.45
Prepare a production budget of Anoop Ltd., from the following information.
Stocks for the budgeted Period

Product As on 1st Jan As on 31st Dec


x 6,000 units 8,000 units
y 10,000 units 9,000 units
Z 14,000 units 15,000 units

Budgeted Sales:
x 50,000 units
Y 70,000 units
Z 1,00,000 units

Normal Loss in Production:


x 3%
Y 4%
Z 5%

[Kuvempu Uni, B.Com, October 2003 and May 2004]


Management Accounting: 512

Solution:
P t 'Ion Bu dIget
rodUC
Products (units)
Particulars
X y Z
Budgeted Sales Quantity 50,000 70,000 1,00,000
Add: Desired Closing Stock 8,000 9,000 15,000
Total Requirements 58,000 79,000 1,15,000
Less: Opening Stock 6,000 10,000 14,000
:. Production (net) 52,000 69,000 1,01.000
Add: Normal Loss in Production* 1,608 2,875 5,316
:. Budgeted Production 53,608 71,875 1,06,316

* Assumed to be on Input. Hence, Loss to Output = J ~ ~


97 96 95
mustration: 6.46
MAT and ZAT are the two brands of Ecom LTD., which makes following sales estimates
for April 2001:
MAT: 15,000 units, Selling price Rs.200 per unit
ZAT: 45,000 units, Selling price Rs.150 per unit.
Raw materials A and B are used in the production of MAT in equal proportions. While
for ZAT, Raw materials B and C are used in the 2:3 ratio. The actual and estimated stock details
and their relevant prices are as below:

Existing Desired Price


Materials Opening Stock Closing Stock (Rs.) per
(units) (units) unit
A 1,000 2,000 5.50
B 5,000 4,000 8.00
C 26,000 6,000 4.50
Finished Stock
MAT 500 1,000
ZAT 10,000 15,000
Draft: (a) Material Purchase and (b) Production Budget with details of units and value,
wherever possible. [Mangalore Un;' B.Com. October 2001J
Budgetary Control: 513

Solution:
roduct 'IOn B u d1getfior the M onth 0 fALpn'1 2001
P
Product (units)
Particulars
MAT ZAT
Estimated Sales 15,000 45,000
Add: Desired Closing Stock 1,000 15,000
Total Requirements 16,000 60,000
Less: Existing Opening Stock 500 10,000
:. Budgeted Production 15,500 50,000

. Is Po rch ase B u d1get ~or t he M onth 0 fALpn'1 2001


M atena
Materials (units)
Particulars Total
A B C
Materials required for production of Product:
MAT: 15,500 in equal proportions [A and B] 7,750 7,750
ZAT: 50,000 in 2:3 [B and C] 20,000 30,000
Add: Desired Closing Stock 2,000 4,000 6,000
:. Total Requirements 9,750 31,750 36,000
Less: Opening Stock 1,000 5,000 26,000
:. Purchases 8,750 26,750 10,000
Price per unit (Rs.) 5.5 8 4.5
:.Total Purchase Price (Rs.) 48,125 2,14,000 45,000 3,07,125

mustration: 6.47
M.V.K Company has three sales divisions at Bangalore, Davangere and Belgaum. It sells two
products S and M. The budgeted sales for the year ending 31-12-1996 at each place are given below:
Bangalore: Product S, 1,20,000 units at Rs.9 each
Product M, 80,000 units at Rs.6 each
Davangere: Product M, 1,30,000 units at Rs.8 each
Belgaum: Product S, 1,60,000 units at RS.I0 each
The actual sales during the same period were as follows:
Bangalore: Product S, 1,30,000 units at Rs.9 each
Product M, 90,000 units at Rs.6 each
Davangere: Product M, 1,60,000 units at Rs.8 each
Belgaum: Product S, 1,80,000 units at Rs.l 0 each
Management Accounting: 514
From the reports of the sales personnel it was considered that the sales budget for the year
ending 31-12-1997 would be higher than 1996 budget on the following aspects.
Bangalore: Product S, 10,000 units
Product M, 20,000 units
Davangere: Product M, 20,000 units
Belgaum: Product S, 40,000 units
Intensive sales campaign in Bangalore and Belgaum is expected to result in additional
sales of 30,000 units of product S in Bangalore and 20,000 units of product M in Belgaum.
Prepare sales budget from the above particulars. [Kuvempu Un;' B.Com, October 1997J

Solution:
..
Sales Bu dIgetofMVKC ompany £or th e Year endin2 Decemb er 31, 1997
1996 Budget for
Budget Actuals 1997

Division Product S ProductM Product S ProductM Product (unit)

Units Rs. Units Rs. Units Rs. Units Rs. S M


(000) (000) (000) (000) (000) (000) (000) (000) (000) (000)

Bangalore 120 1,080 80 480 130 1,170 90 540 160· 1002


1,28
Davangere 130 1,040 160 15cY
0
4
Belgaum 160 1,600 180 1,800 200 2lf
1,82
280 2,680 210 1,520 310 2.970 250 360 270
0

Notes:
1. [1,20,000 units + 10,000 units] = 1,30,000 units
[1,30,000 units + 30,000 units] = 1,60,000 units
2. [80,000 units + 20,000 units] = 1,00,000 units
3. [1,30,000 units + 20,000 units] = 1,50,000 units
4. [1,60,000 units + 40,000 units] = 2,00,000 units
5. Due to inensive sales eampaingn.
IDustration: 6.48
India Sales Ltd., request you to construct a Sales-overhead Budget from the following
particulars. Sales volume estimated for the month of October 2001:

At the Counter By the Travelling Salesmen


Rs.80,000 Rs.l,OO,OOO
1,20,000 1,50,000
1,60,000 2,00,000
Budgetary Control: SlS

Details of Expenses:
Payment to advertising agencies Rs.25,000
Salaries of the sales department 50,000
Other expenses of the sales department 15,000
Salaries of counter-salesmen 60,000
Commission to counter-salesmen: 2% on their sales.
Commission to travelling salesmen: 5% on their sales:
Out-of-pocket expenses to salesmen travelling: 3% on their sales.
[Mangalore Uni, B.Com, April 2001 and October 2003J
Solution:
Sales 0 verheadBu d g~t at n'ffi
I erent Saes
I Leve s
Sales Revenue
Particulars
@Rs.l ,80,000 @Rs.2,70,000 @Rs.3,60,OOO
Payment to Advertising Agencies 25,000 25,000 25,000
Sales Department Salaries 50,000 50,000 50,000
Other Expenses of the Sales Department 15,000 15,000 15,000
Salaries of Counter-salesmen 60,000 60,000 60,000
Commission [2% and 5%]1 6,600 9,900 13,200
Out-of-pocket Expenses [3% f 3,000 4,500 6,000
Total Budgeted Sales Overhead 1,59,600 1,64,400 1,69,200
Note:
1. 2% and 5% of sales at the Counter and
by the Travelling Salesmen 1,600 2,400 3,200
5,000 7,500 10,000
2. 3% of Sales by the Travelling Salesmen 3,000 4,~00 6,000

IDustration: 6,49
Reliance Ltd., produces two products, ALpha and Beta. There are two sales divisions,
North and South. Budget sales for the year ending 31st December 2000 were as follows:
Price per
Divisions Products Units
unit (Rs.)
North: Alpha ·25,000 10
Management Accounting : 516

Beta 15,000 5
South: Alph 24,000 10
Beta 30,000 5

Actual sales for the above said period were as follws:


Products North South
Alpha 28,000 units at Rs.1O each 25,000 units at Rs.1O each
Beta 18,000 units at Rs.5 each 33,000 units at Rs.5 each
On basis of the assessment of the salesmen, the following sales estimates were made for
the next budget period 2001.
North Zone: Alpha, Budget Sales will increase by 40% over 2000 budget.
Beta, Budget Sales will increase by 10% over 2000 budget.
South Zone: Alpha, Budget Sales will increase by 12% over 2000 budget.
Beta, Budget Sales will increase by 15% over 2000 budget.
It was 'further decided that an intensive sales campaign in the North will bring an
additional sales of 5,000 units of each of the products. You are required to prepare a sales budget
for the year 2001 incorporating the above estimates and also show the budgeted and actual sales
of the current year. [Mangalore Uni., B.Com., April 2003J
Solution:
aes ud12et 0 fRerIance Ltd ., f,or theyear 2001
SIB
Budget for
Budget for 2000 ActuaIs for 2000
2001 (units)
Product
North (000) South (000) North (000) South (000)
North South
Units Rs. Units Rs. Units Rs. Units Rs.
Alpha 25 250 24 240 28 280 25 250 40,000 26,880
Beta 15 75 30 150 18 90 33 165 21,500 34,500
-
325 390 370 415

North Alpha: [25,000 units + 40% of 25,000 units] = 35,000 units


[35,000 units + 5,000 units due to sales campaign] =40,000 units.
Beta: [15,000 units + 10% of 15,000 units] = 16,500 units
[16,500 units + 5,000 units due to sales campaign] = 21,500 units
South, Alpha: [24,000 units + 12% of 24,000 units] = 26,880 units
Beta : [30,000 units + 15% of 30,000 untis] =34,500 units
Budgetary Control: 517

Dlustration: 6.50
A company is expecting to have Rs.25,000 cash in hand on April 1, 2000 and it requires
you to prepare an estimate of cash position during the three months, April to June 2000. The
following information is supplied to you.
Sales Purchases Wages Expenses
February Rs.70,000 Rs.40,000 Rs.8,OOO Rs.6,000
March 80,000 50,000 8,000 7,000
April 92,000 52,000 9,000 7,000
May 1,00,000 60,000 tO,OOO 8,000
June 1,20,000 55,000 12,000 9,000

Other information:
a. Period of credit allowed by suppliers: two months
b. 25% of sales is for cash and the period of credit allowed to customers for credit sales: one
month
c. Delay in payment of wages and expenses: one month
d. Income tax Rs.25,000 is to be paid in June 2000.
[Kuvempu Uni., B.Com., October 2003J
Solution:
Three-monthiIy Cash B udIget [A.prl'1 - J une 2000]
Month (Rs.)
Particulars
April May June
Opening Balance of Cash 25,000 53,000 81,000
Add: Receipts :
Cash Sales [25% of current month's
sales] 23,000 25,000 30,000
Collection from Customers [75% of
previous month's sales] 60,000 69,000 75,000
Total (a) 108,000 147,000 186,000
Payments:
Suppliers [two-month credit] 40,000 50,000 52,000
Wages [one month] 8,000 9,000 10,000
Expenses [one month] 7,000 7,000 8,000
Management Accounting: 518

Income Tax - - 25,000

Total (b) 55,000 66,000 95,000

:. Closing Balance (a - b) 53,000 81,000 91,000

mustration: 6.51
The details given below relate to 60% activity, when the production was 600 units.
Materials: Rs.120 per unit
Labour: Rs.50 per unit
Expenses: Rs. 15 per unit
Factory expenses: Rs.50,000 (40% fixed)
Administration expenses: Rs.35,000 (60% fixed)
Prepare a flexiable budget showing marginal cost and total cost, for 60%, 80% and 100%
activity.
[Bangalore Uni., B.Com., May 2000, Mangalore Uni., B.B.M., April and October 2000, and
Kuvempu Uni., B.Com., October 1997, May and October 2002, and B.B.M., December 1996
and 1999J
Solution:
Flexible Budget showing Marginal and Total Costs at 60%, 80% and 100%
Levels of Activity
Cost at 60% (Rs.) Cost at 80% (Rs.) Cost at 100% (Rs.)
Particulars (600 units) (800 units) (1,000 units)
Per Unit Total Per Unit Total Per Unit Total
Marginal Cost:
Materials 120.00 72,000 120.00 96,000 120.00 1,20,000
Labour 50.00 30,000 50.00 40,000 50.00 50,000
Expenses 15.00 9,000 15.00 12,000 15.00 15,000
Variable Portion of:
Factory Expenses· 50.00 30,000 50.00 40,000 50.00 50,000
Administration
Expenses2 23.33 14,000 23.33 18,667 23.33 23,333
:.Marginal Cost (a) 258.33 1,55,000 258.33 2,06,667 258.33 2,58,333
Fixed Portion of 3:
Factory Expenses· 33.33 20,000 25.00 20,000 20.00 20,000
2
Administration Expenses 35.00 21,000 26.25 21,000 21.00 21,000
Budgetary Control: 519

:. Fixed Cost (b) 68.33 41,000 51.25 41,000 41.00 41,000


Total Cost (a + b) 326.66 1,96,000 309.58 2,47,667 299.33 2,99,333

Working Notes:

. 1. At 60% level of activity, Factory Expenses = Rs.50,000

Less: Fixed portion (40%) 20,000

:. Variable Factory Expenses 30,000

Hence, Variable Factory Expenses per unit = Rs.50 [Le., Rs.30,000 + 600 units]

2. Administration Expenses at 60% level of activity = Rs.35,000


Less: Fixed portion (60%) 21,000
:. Variable Administration Expenses 14,000
-----
:. Variable Administration Expenses per unit =Rs.23.33 [Le., Rs.14,000 + 600 units]

[Total Overheads (fixed portionU


3. [ Number of Units J=Unit Fixed Expenses
mustration: 6.52
The expenses for budgeted production of 10,000 units in a factory are given below (per
unit).

Materials Rs.70
Labour 25
Variable overheads 20
Fixed overheads (Rs. 1,00,000) 10
Variable expenses (direct) 5
Selling expenses (10% fixed) 13
Distribution expenses (20% fixed) 7
Administration expenses (Rs.50,OOO) 5
Total Cost per unit (to make and sell) 155

Prepare a budget for production of (a) 8,000 units, (b) 6,000 units and (c) indicate cost
per unit at both the levels.
[Bangalore Uni., B.Com., November 2000, and 2001, and May 2002, and Kuvempu Uni.,
B.Com., May 1991J
Management Accounting : 520

Solution:
Flex.'ble Bu dIget Showmg T0 tal and U01't Costs at 6,000, 8,000 and 10,000 um'ts
Cost at 6,000 units Cost at 8,000 units Cost at 10,000 units
Particulars (Rs.) (Rs.) Rs.
Per Unit Total Per Unit Total Per Unit Total
Variable Costs:
Materials , 70.00 4,20,000 70.00 5,60,000 70.00 7,00,000
Labour 25.00 1,50,000 25,00 2,00,000 25.00 2,50,000
Variable Oveheads 20.00 1,20,000 20.00 1,60,000 20.00 2,00,000
Variable Expenses 5.00 30,000 5.00 40,000 5.00 50,000
Variable Portion of :
Selling Expenses I 11.70 70,200 11.70 93,600 11.70 1,17,000
Distribution Expenses2 .5.60 33,600 5.60 44,800 5.60 56,000
(a) 137.30 8,23,800 137.30 10,98,400 137.30 13,73,000
Fixed Costs:
Fixed Overheads3 16.67 1,00,000 12.50 1,00,000 10.00 1,00,000
Fixed Portion of:
Selling Expenses I 2.17 13,000 1.63 13,000 1.30 13,000
2
Distribution Expenses 2.33 14,000 1.75 14,000 1.40 14,000
Administration Expenses 8.33 50,000 6.25 50,000 5.00 50,000
(b) 29.50 1,77,000 22.13 1,77,000 17.70 1,77,000
Total Cost (a + b) 166.80 10,00,800 159.43 12,75,400 155.00 15,50,000

Working Notes:
1. At 10,000 units, Selling Expenses per unit come to Rs.l3.
Hence, for 10,000 units, Rs.l,30,000
Less: Fixed (10%) 13,000
:, Variable Selling Expenses 1,17,000

Therefore, Variable Selling Expenses per unit =Rs.11.7


Budgetary Control : 511
2. Distribution Expenses per unit, at 10,000 units = Rs.7
:.For 10,000 units = Rs.70,000
Less: Fixed Portion (20%) = 14,000
:. Variable Distribution Expenses 56,000
:. Variable Distribution Expenses per unit = Rs.5.6
3. Fixed Overheads are Rs.1,OO,OOO and these remain constant irrespective of the units. Hence,
Fixed Overhead Expenses per unit are computed by dividing Rs.1,00,OOO by the number of
units. For instance, at 8,000 units, it comes to Rs.12.5 [i.e., Rs.l ,00,000 + 8,000 units]
illustration: 6.53
The following information at 50% capacity is given. Prepare a flexible budget and
forecast the profit or loss at 60%, 70% and 90% capacity.
Fixed Expenses:
Salaries Rs.50,000
Rent and Taxes 40,000
Depreciation 60,000
Administration Expenses 70,000
Variable Expenses:
Materials 2,00,000
Labour 2,50,000
Others 40,000
Semi-Variable:
Repairs 1,00,000
Indirect Labour 1,50,000
Others 90,000
It is estimated that fixed expenses will remain constant at all capacities. Semi-variable
expenses will not change between 45% and 60% capacity, will raise by 10% between 60% and
75% capacity, a further increase of 5% when capacity crosses 75%. Estimated slaes at various
levels of capacity are:
Capacity Sales (Rs.)
·60% 11,00,000
70% 13,00,000
90% 15,00,000
[Kuvempu Uni, B.Com, May 2004 and October 2001 and 2004, and B.B.M, May 2000;
Manga/ore Uni, B.Com, April 2003 and November 2004J
Management Accounting : 522

Solution:
It is assumed that the Semi-variable Expenses will remain constant to 60% capacity
(including at 60%) and increase by 10% if the capacity utilization exceeds 60% but does not
exceed 75%. "A further increase of 5% when capacity crosses 75%" is assumed to mean a total
15% increase in Semi-variable Expenses if the capacity utilization exceeds 75%. And the variable
expenses are assumed to be increasing with the increase in the capacity utilization
proportionately.
FIeXI'ble BUdIget at 60~11, 70~11,and 90~11 L eveIs 0 fA' ,
ctlvlty
Amount (Rs.) at ....... % Level of Activity
Particulars
50 60 70 90
Variable Expenses:
Materials 2,00,000 2,40,000 2,80,000 3,60,000
Labour 2,50,000 3,00,000 3,50,000 4,50,000
Others 40,000 48,000 56,000 72,000
(a) 4,90,000 5,88,000 6,86,000 8,82,000
Semi-variable Expenses:
Repairs 1,00,000 1,00,000 1,10,000 1,15,000
Indirect Labour 1,50,000 1,50,000 1,65,000 1,72,500
Others 90,000 90,000 . 99,000 1,03,500
(b) 3,40,000 3,40,000 3,74,000 3,91,000
Fixed Expenses:
Salaries 50,000 50,000 50,000 50,000
Rent and Taxes 40,000 40,000 40,000 40,000
Depreciation 60,000 60,000 60,000 60,000
Administration Expenses 70,000 70,000 70,000 70,000
(c) 2,20,000 2,20,000 2,20,000 2,20,000
Total Cost (a + b + c) 10,50,000 11,48,000 12,80,000 14,93,000
Sales Revenue 11,00,000 13,00,000 15,00,000
Profit (or Loss] (48,000) 20,000 7,000
Dlustration: 6.54
Jaya Printers furnish the following Revenue Statement for the year 2000-2001:
Revenue from sales· Rs. 3,55,800
Less: Expenses:
Raw Materials Rs.74,200

,
,-
Budgetary Control : 523

Stores 48,800
Others expenses (25% variable) 2,04,000
Interest on Loans 10,000
Depreciation 20,000
Insurance 10,000 3,67,000
... Loss (Rs.) 11,200

The firm had been operating at 60% of its capacity; for the forthcoming year, it intends to operate
at 80% capacity. Fixed expenses are likely to rise by Rs.12,000 and material cost by 2%. Draw
budget for 2001-2002. [Mangalore Uni., B.Com.. April 2001]
Solution:
Bud12etedlncome Statemen t ~or 200102 -
Amount (Rs.)
Particulars Actual for Budget for
2000-01(60%) 2001-02 (80%)
Sales Revenue 1 3,55,800 4,74,400
(a) 3,55,800 4,74,400
Expenses:
Raw-materials2 74,200 1,00,912
3
Stores 48,800 65,067
Other expenses : Variable 4 51,000 68,000
Fixed 1,53,000 1,53,000
5
Interest on Loan 10,000 1~000
5
Depreciation 20,000 20,000
5
Insurance 10,000 10,000
Increase (expected] in fixed costs - 12,000
(b) 3,67,000. 4,38,979
... Profit (a - b) -11,200 35,421

Note:

1. LRS.3,55,800 'I
l 60% x 800/, = RsA,74,400
Management Accounting : 524

2. rtRs.74,200
60~ X
I
800/~ = Rs.98,933 [at 2000-01 price level]
At 2001-02 price level, Rs.98,933 + 2% of Rs.98,933 increase = [Rs.98,933 + Rs.l,979]
= Rs.l,00,912.

3. rt Rs.48,800
60% x
I
80o/~ = Rs.65,067
4. Rs.2,04,000 = [Variable 25%, Rs.51,000 + Fixed 75%, Rs.l,53,OOO]

r- Rs.51,000 "')
Variable Portion at 80% =l 60% x 800/:) =Rs.68,OOO
Fixed Portion remains same at Rs. 1,53,000 even at 80%
5. Remains constant
IDustration: 6.55
X Co., attains a sale of Rs.6,OO,OOO at 80% of its normal capacity. The production
expenses are given below:

Material cost per unit: Rs.60 Selling on cost:


Labour cost per unit: Rs. 40 Salaries: 8% of sales
Administration cost: Travelling expenses: 2% of sales
Office salary: Rs.90,OOO Sales office expenses: 2% of sales
General expenses: 2% of sales Distribution cost:
Depreciation: Rs.7,500 Wages (fixed): Rs.15,OOO
-Rent, rates and taxes: Rs.8,750 Rent: 2% of Sales
Other ~xpenses: 3% of sales
Draw up Flexible Budget to ascertain the cost at 90% and 100% normal capacity. The
selling price is Rs. 200 per !lnit. [Kuvempu Uni, B.Com, May 1999}
Solution:

. r§ales Revenue, Rs.6,OO,OOQl


Sales Quantity =[ Selling Price, Rs.200.J =3,000 units at 80% capacity
Therefore, Sales Quantity at 90% = 3,375 units [(3,0004- 80) x 90]
100% = 3,750 units [(3,0004- 80) x 100]
Budgetary Control : 525

Flexible Bud2et at 80%, 90% and 100% Levels of Activity


Levels of Activity
Particulars
80% (Rs.) 90% (Rs.) 100 % (Rs.)
Sales Quantity (units) 3,000 3,375 3,750
Sales Revenue (Rs) 6,00,000 6,75,000 7,50,000
Variable Cost:
Material [at Rs.60 a unit] 1.80.000 2.02,500 2,25,000
Labour [Rs.40 a unit) 1,20,000 1,35,000 1,50,000
General Expenses, Administration [2%
of Sales] 12,000 13,500 15,000
Selling Expenses:
Salaries [8% of Sales] 48,000 54,000 60,000
Travelling Expenses [2% of Sales] 12,000 13,500 15,000
Sales Office Expenses [2% of Sales] 12,000 13,500 15,000
Distribution Expenses:
Rent [2% of Sales] 12,000 13.500 15,000
Others [3% of Sales] 18,000 20,250 22,500
(a) 4,14,000 4,65,750 5,17,500
Fixed Cost:
Administration:
Office Salary 90,000 90,000 90,000
Depreciation 7,500 7,500 7,500
Rent, Rates, etc., 8,750 8,750 8,750
Distribution Costs: Wages 15,000 15,000 15,000
r_------~--------r_------~

(b) 1,21,250 1,21,250 1,21,250


Total Cost (a + b) 5,35,250 5,87,000 6,38,750
~------~--------~------~
:.Profit 64,750 88,000 1,11,250

mustration: 6.56
ABC Ltd manufactures a single product which is in great demand in the market. The
present sales of Rs. 60,000 per month utilises only 60% capacity of the plant. The sales manager
anticipates that with a reduction of 10% in the price, the sales would go up by 25% to 30%. The
following data is available
Management Accounting : 526

Selling price: Rs. 10 per unit; Variable cost: Rs. 3 per unit
Semi-variable cost: Rs. 6,000 fixed plus Re. 0.50 per unit
Fixed cost: Rs. 20,000 at present level, estimated to be Rs. 24,000 at 80% output.
You are required to submit statements to the Board showing: (a) the operating profits at
60%, 70% and 80% levels at current selling price and at proposed selling price, and (b) the
percentage increase in the present output which will be required to maintain the present profit at
the proposed selling price. [ICWA (Int), 1989]
Solution:
(a) Budgeted Income Statement at Different Levels (Rs.)

Particulars 60% 70% 80%


(6,000) (7,000) (8,000)
Variable Cost (at Rs. 3) 18,000 21,000 24,000
Variable portion of Semi-variable Cost (Re. 0.5) 3,000 3,500 4,000
Total Variable Cost (a) 21,000 24,500 28,000
Fixed Cost 20,000 20,000 24,000
Fixed portion of Semi-variable Cost 6,000 6,000 6,000
Total Fixed Cost (b) 26,000 26,000 30,000
Therefore, Total Cost (a + b) 47,000 50,500 58,000
Sales Revenue: at Current Price (of Rs. 10) 60,000 70,000 80,000
at Reduced Price (of Rs. 9) 54,000 63,000 72,000
Therefore, Profit at: Current Price 13,000 19,500 22,000
Reduced Price 7,000 12,500 14,000

b. Desired Profit = Present Profit = Rs. 13,000 (at 60% and at the current Price)
Fixed Cost (up to 80%) = 26,000
_ Therefore, Desired Contribution = 39,000
Unit Contributio~ at reduced Price = [Rs. 9 - (Rs. 3 + Re. 0.5) = [Rs. 9 - Rs. 3.5] = Rs. 5.5
Desired Sales Volume = (Rs. 39,000 + Rs. 5.5) = 7,091 units
Therefore, Percentage increase in Output = [(7,091- 6,000) + 6,000] x 100
=[(1,091 +6,000) x 100] = 18.18%
mustration: 6.57
The following is a summary of output and production costs of a manufacn!ring
organisation for different levels of activity.
Budgetary Control : 527

Level of Activity
Particulars
60% 70% 80%
Output (units) 1,200 1,400 1,600
Cost (Rs.): Direct materials 24,000 28,000 32,000
Direct labour 7,200 8,400 9,600
Production overheads 12,800 13,600 14,400
Total Production Cost 44,000 50,000 56,000
Management is considering an increase of production to 90% level of activity. It is
expected that fixed overheads will remain unchanged at this level. You have to generate the
following information for use by management.
a. The total prime cost at 90% level,
b. The average marginal cost per unit of producing the additional output,
c. The total marginal cost at 90% level,
d. The total production cost at 90% level, and
e. The break-even point at 90% level. {MBA., Karnatak Univ., March 1984J
Solution:
Budgeted Cost Statement at 90% Level of Activity
Amounts
Particulars
Rs.
Direct Material Cost (Rs. 20 per unit x 1,800 units) 36,000
Direct Labour Cost (Rs. 6 per unit) 10,800
Prime Cost 46,800
Add: Production Overheads:

V . bl ( . ) - [RS. 13,6pD - Rs. 12,800


ana e per umt - 1,400 _ 1,200 units) -
J-[ Rs. 800J- R 4
200 units - s. per umt
. 7,200
Variable Cost 54,000
Fixed (Total- Variable), [at 60%, Rs. 12,800 - (Rs. 4 x 1,200 units)] 8,000
Total Cost 62,000

Therefore,
a. At 90% level of activity, Prime Cost = Rs. 46,800
b. Assume that the present level of activity is 80%. Then,
at 80%, Output = 1,600 units; Cost =Rs. 56,000; per unit cost =Rs. 35.00
Management Accounting : 528

at 90%, Output = 1,800 units; Cost = Rs. 62,000; per unit cost = Rs. 34.44
Difference = 200 6,000
Therefore. Average Cost of additional 200 units = Rs. 6.000
Therefore, per unit cost = [Rs. 6,000 + 200] = Rs. 30 or (20 + 6 + 4) = Rs. 30
c. Total Marginal Cost at 90% = Rs. 54,000 (assuming that Marginal Cost represents only
Variable Cost)
d. Total Production Cost at 90% = Rs. 62.000
e. Break-even Point (BEP):
Let 'P' be the Selling Price per unit.
At, break-even level, total contribution from sales is equal to the fixed cost.

. [ Fixed Cost l r.
Rs. 8,000
Therefore, BEP (umts) = ~nit Contributiol!J = UP - (20 + 6 + 4)~=
l r
l
Rs. 8,000
P - 30
J
IDustration: 6.58 .
Paints Private Ltd Company, manufacturing a single product, is facing severe competition in
selling it at Rs. 50 per unit. The company is operating to 60% level of capacity at which level the
sales are Rs. 12,00,000 and variable costs are Rs. 30 per unit. Semi-variable costs may be
considered as fixed at Rs. 90,000 when output is nil and the variable element is Rs. 250 for each
additional 1% level of activity. Fixed costs are Rs. 1,50,000 at the present level of activity, but at,
80% level of activity or above, these costs are expected to increase by Rs. 50,000. To cope with
the competition, the management of the company is considering a proposal to reduce the selling
price by 5%. You are required to prepare a statement showing the operating profit at levels of
activity of 60%, 70%, 80% and 90% assuming that: (a) the selling price remains at Rs. 50; and
(b)"the selling price is reduced by 5%. {M.Com., UoM, June 1984J
Solution:
Comparative Income Statement
Levels of Activity
Particulars
60% 70% 80% 90%
Sales Volume (units) 24,000 28,000 32,000 36,000
Rs. Rs. Rs. Rs.
Variable Costs (at Rs. 30 a unit) 7,20,000 8,40,000 9,60,000 10,80,000
Semi-variable Costs =[(250 x Level of
Activity) + 90,000] 1,05,000 1,07,500 1,10,000 1,12,500
Fixed Cost 1,50,000 1,50,000 2,00,000 2,00,000
Total Cost of Sales 9,75,000 10,97,500 12,70,000 13,92,500
Budgetary Control : 529

Sales Revenue (at Rs. 50 Selling Price) 12,00,000 14,00,000 16,00,000 18,00,000
Therefore, Profit 2,25,000 3,02,500 3,30,000 4,07,500
Sales Revenue (at Rs. 47.5 Selling Price) 11,40,OUO 13,30,000 15,20,000 17,10,000
Less: Total Cost 9,75,000 10,97,500 12,70,000 13,92,500
Therefore, Profit 1,65,000 2,32,500 2,50,000 3,17,500

Illustration: 6.59
A manufacturing business has drawn up a budget for the current year, the proportion of
the expected Gosts being as follows.
Direct material 34.0%
Direct labour 22.0
Variable factory overhead 16.5
Fixed factory overhead 12.0
Other variable costs 5.5
Other fixed costs 4.0
Profit 6.0
Total 100.0
After six months' working. it becomes apparent that the volume of business anticipated
will not be obtained, and that a figure of 75% of the sales will be obtained. The cost accountant
also observes the following:
a The variable factory overhead had been over-estimated by 10%
b. Purchase of a new equipment increases the fixed factory overhead by 15%
If the sales amount to Rs. 3,30,000 at 75% of the budgeted figures, .prepare a revised
budget giving the actual figures of costs and revenue considering the above changes.
1M. Com., UoM, June 1985}
SolutiQD:
Revised Budgeted Income Statement
Original (100%) Revised (75%)
Particulars
Amount Amount
% %
Rs. Rs.
Sales Revenue l 100.0 4,40,000 100 3,30,000
(a) 100.0 4,40,000 100 3,30,000
Total Costs:
Management Accounting : 530

Direct Material Cost 34.0 1,49,600 34.0 1,12,200


Direct Labour Costs 22.0 96,800 22.0 72,600
Factory Overhead:
Variable2 16.5 72,600 15.00 49,500
3
Fixed 12.0 52,800 18.40 60,720
Other Costs:
Variable 5.5 24,200 5.50 18.150
Fixed 4.0 17,600 5.33 1/3 17,600
(b) 94.0 4,13,600 100.23 1/3 3,30,770
Profit (or Loss) (a - b) 6.0 26,400 (0.23 1/3) (770)
Note: 1. [3,30,000 + 75%]; 2. [(16.5 + 110) x 100]; 3. [52,800 + 15%]
Dlustration: 6.60
Better Budgets Ltd., is preparing their budget for next year. They are keen on leaving
nothing to chance and so would like to envisage all sorts of possibilities and incorporate them in
the budget. Their considered estimates are as under.
a. In the worst possible scenario, sales will be 8,000 units at a price of Rs. 19 per unit; the
material costs will be Rs. 9 per unit; direct labour Rs. 2 per unit and the variable
overheads will be Rs. 1.50 per unit. The fixed costs will be Rs. 60,000 per annum.
b. In the best possible scenario, sales will be 15,000 units at a price of Rs. 20 per unit. The
material costs will be Rs. 7 per unit; direct labour Rs. 3 per unit and the variable
overheads will be Re. 1 per unit. The fixed costs will be Rs. 48,000 per annum.
c. It is most likely, however, that the sales will be 2,000 units above the worst possible level
at a price of Rs. 20 per unit. The material costs will be Rs. 8 per unit, direct labour Rs. 3
per unit and variable overheads will be Re. 1 per unit. The fixed cost will be Rs. 50,000
per annum.
d. There is a 20% probability that the worst will happen, a 10% probability that the best will
happen and a 70% probability that the most likely outcome will occur.
What would be the expected value of profit as per the budget for the coming year?
[ICWA (Fin), December 1988J
Solution:
Budgeted Income Statement of Better Budgets Ltd
Worst Best Most
Particulars
Possible Possible Likely
Sales Volume (units) 8,000 15,000 10,000
Rs. Rs. Rs.
Selling Price 19 20 20
Budgetary Control: 531

Unit Material Cost 9 7 8


Unit Labour Cost 2 3 3
Variable Overheads per unit 1.5 1 1
Annual Fixed Overheads 60,000 48,000 50,000
Direct Material Cost 72,000 1,05,000 80,000
Direct Labour Cost 16,000 45,000 30,000
Variable Overheads 12,000 15,000 10,000
Total Variable Cost 1,00,000 1,65,000 1,20,000
Sales Revenue 1,52,000 3,00,000 2,00,000
Therefore, Contribution 52,000 1,35,000 80,000
Less: Fixed Cost 60,000 48,000 50,000
Therefore, Profit - 8,000 87,000 30,000
Therefore, expected amount of profit:
Expected Value
Possible Scenario: Profit (Rs.). Probability
(Rs.)
Worst - 8,000 0.2 - 1,600
Best 87,000 0.1 8,700
Most Likely 30,000 0.7 21,000
28,100
Summary of the Chapter
Budgetary Control is an important technique in the hands of management which enables it
to discharge its duties relating to Planning and Control of business activities more efficiently and
effectively. In this background, fundamentals of Budgetary Contr~l such as the meaning of
Budget. Budgeting and Budgetary Control are discussed at the beginning of this chapter followed
by the pre-requisites for the successful introduction and implementation of Budgetary Control
System. A detailed discussion of different kinds of functional Budgets viz., Sales Budget,
Production Budget, Purch~e Budget, Labour Budget, Cash Budget and Master Budget is made
with relevant practical problems. Besides, the preparation of both the Fixed Budget and Flexible
Budgets are explained with a number of examples. A large number of problems are discussed and
solved at the end.
Key-Terms to Remember
Budget Budgeting
Budgetary Control Forecast
Budget Manual Budget Centre
Key Factor Fixed Budget
Flexible Budget Cash Budget
Production Budget Sales Budget
Management Accounting : 532
Notes and References
1. CIMA .. London ,as quoted in N.K. Kulshreshta. Theory and Practice of Management
Accounting, Aligarh, Navman Prakashan. 2nd Ed, 1985-86, p. 544.
2. S.P. Gupta, Management Accounting, Sahitya Bhawan, p. 720.
3. John R, Bertizel as quoted in S.P. Gupta, Ibid, p. 721.
4. Gordon and Shillinglaw, Accounting - A Management Approach.
5. G.A. Welsch, Budgeting - Profit Planning and Control.
6. William 1. Vattar as quoted in N.K. Kulshreshta, Op. cit., p. 544.
7. George R. Terry as quoted in N. Vinayakam and LB. Sinha, Management Accounting -
Tools and Techniques, Mumbai, Himalaya Publishing House, 1st Ed., 1988, pp. 15-6.
Questions for Self-study
01. Defme Budget and distinguish it from Budgeting and Budgetary Control.
02. Define Budget and Budgeting. How does Budgetary Control differ from Standard Costing?
- [Kuvempu Un~ B.Com, May 1999J
03. Explain the different types of Budgets.
[Kuvempu Uni., B.Com., October 1997 and Mangalore Un~ B.Com., May 2002J
04. "Budgetary Control is an aid to Management and not substitute for Managemet". Do you
agree? Why?
05. Define Budgetary Control and explain the pre-requisites for its successful introduction and
implementation.
06. Explain the importance of Budgetary Control System as a cost reduction mechanism.
07. Critically evaluate the limitations underlying the Budgetary Control System.
08. Define Budget and Budgetary Control. Explain the objectives of Budgeting and state the
advantages and limitations of Budgetary Control. [Kuvempu Uni., B.Com., May 2000J
09. What do you understand by the terms Budget and Budgetary Control? State the objects of
Budgetary Control. How does Budgetary Control differ from Standard Costing?
[Kuvempu Un~ B.Com, November 2000J
10. Critically examine the statement that 'Budgets are not merely accounting statements, they are
blue prints for managerial action during a Budget Period' .
11. What do you understand by 'Master Budget'? How is it prepared?
12. How do you prepare Production Budget? [Kuvempu Un~ B.Com, May 1991J
13. Write a: note on Sales Budget. [Kuvempu Un~ B.Com, May 1997J
14. Define 'Flexible Budget' and distinguish it from Fixed Budget. Under what circumstances
would you recommend Flexible Budgeting?
Budgetary Control : 533
15. What is Master Budget? Explain the procedure you would follow to prepare the Master
Budget.
16. What do you mean by Budgetary Control? What are its" objectives?
17. Critically evaluate the limitations of Budgetary Control System.
18. What is Sales Budget? How is it prepared?
[Kuvempu Uni, B.Com, May and November 2001]
19. Write a note on Sales Budget. [Kuvempu Uni, B.Com, May and October 1999]
20. Discuss the role of Budgetary Control System in planning for profit of an organization.
21. Define Budgetary Control and explain its advantages.
22. What is Principal Budget Factor? How does it influence the target results?
23. Write a note on Limiting Factors. [Kuvempu Uni, B.Com., October 1998]
24. Write short notes on: (a) Sales Budget, (b) Master Budget (c) Variance Analysis (d) Key
Factors. [Kuvempu Uni, B.Com, October 1998]
25. What do you mean by Budget Centre?
26. Write a note on Budget Committee?
27. Explain the importance of Budget Manual in the effective introduction and implementation of
Budgetary Control System.
28. Define Budget Period.
29. Define Flexible Budget and distinguish it from Fixed Budget.
30. Define Cash Budget and explain in brief the different approaches to the preparation of Cash
Budget.
31. A Few Short-answer Questions:
a. What is Flexible Budget? [Bangalore Uni, B.Com, May 2000 and 2003, November
2001 and 2002, and Kuvempu Uni., B.Com, November 2002]
b. Give the meaning of Budget [Bangalore Uni, B.Com, November 2000]
c. What is Budgetary Control? [Bangalore Uni, B.Com, May 2001]
d. What is a Production Budget? [Kuvempu Uni, B.Com, May 2002]
32. Modern Manufactures Ltd., make an automobile component using a special kind of alloy steel
as raw material. Each component requires 3 Ib of raw material costing Rs. 90 per Ib and 8
ounces of pai"nt at Rs. 3.50 per ounce. It takes half an hour for fabrication and assembly, and"
quarter of an hour to paint the unit. Fabrication and assembly labour is paid at Rs. 6 per hour
while painters are paid Rs. 3.20 per hour. The variable overheads per component work out to
Rs. 5.50 while the fixed overheads assuming a normal annual production of 17,500
components, amount to Rs. 3,50,000.
The manager - materials management is worried about the procurement of the special kind of
alloy steel which is in scarce supply. Considering this aspect, the management envisages a
maximum production of 15,000 components only for next year.
Management Accounting : 534
a. In the first instance, you are required to work out the total budget costs for next year,
based on the present working.
b. Even to make 15,000 components, the manager - materials management feels that only
two thirds of the requirements of raw material could be pro.cured locally, that too at a 10%
price increase; for the balance requirement, imports have to be made in the form of semi-
finished components at a cost of Rs. 380 per piece. As the components would have been
painted, there would be no painting operations. Besides, the fabrication and assembly
time would be reduced by half and the variable overheads would be only Rs. 4 per
component. You are required to take note of this eventuality also and prepare the total
budget costs for next year on this basis also. [lCWA (Fin), June 1982]
33. Chemicals Ltd., manufacture two products AB and CD by mixing the following raw materials
in the proportion shown.
Raw material Product AB Product CD
A 80%
B 20%
C 50%
D 50%
The finished weight of products AB and CD are equal to the weight of their ingredients.
During the month of June, it is expected that 60 tons of AB and 200 tons of CD will be sold.
Actual and budgeted inventories for the month of June are as follow.
Material Actual Inventory (1st June) Budgeted Inventory (30th June)
Quantity (tons) Quantity (tons)
A 15 20
B 10 40
C 200 300
D 250 200
ProductAB 10 5
Product CD 50 60
The purchase price of materials for June is expected to be as follows.
Material: ABC D
Cost per ton: Rs. 500 400 100 200
All materials will be purchased on 3rd of June. Prepare:
e. Production budget for the month of June.
f. The material requirement budget for June.
Budgetary Control : 535

g. The material purchase budget indicating the expenditure for materials for the month of
June. IICWA (Fin), June 1986J
34. You are required to prepare, for the Board of Directors of Varimall Co., Ltd., a statement
showing the working capital needed to finance a level of activity of 5,200 units of output.
You are given the following information.
Raw materials cost per unit Rs.8
Labour cost per unit 2
Overhead per unit 6
Unit total cost 16
Profit per unit 4
Selling price 20
Raw materials are in stock on average one month. Materials are in process, on average half a
month. Finished goods are in stock on average six weeks. Credit allowed by creditors is one
month. Credit allowed to debtors is two months. Lag in payment of wages is 1.5 weeks.
Cash on hand and cash at bank is expected to be Rs. 7,300. You are informed that production
is carried on evenly during the year and wages and overheads accrue similarly.
1M. Com., UoM, May 1983J
35. Prepare a profit statement and a cash budget for the first quarter April-June for AB Industries
Ltd., from the following information for the coming year.
a. The company produces two products and their unit sales prices and material contents are
as under:
Sales Price (Rs.) Material Content
Product A 75,000 60% of Sales Price
Product B 25,000 60% of Sales Price
b. The production target has been fixed as under:
Product A Product B
April 50 50
May 60 60
June 70 50
Production for January, February and March was at 80% level of April production.
c. The monthly expenses are as under:
1. Salaries and wages Rs. 7,50,000 payable in the following month.
2. Variable overheads 5% of sales value payable in the following month.
Management Accounting : 536

3. _ Fixed overheads Rs. 2,00,000 payable 50% in the current month and 50% in the
following month.
d. Payment for material is made in the third month from the month of procurement.
e. The company maintains a constant level of inventory. No stock of finished goods is kept
and the entire production is invoiced the same month. The company gives 30 days credit
to its customers.
f. Company's products attract excise duty @ 15%. Sales tax @ 2% is payable to the
authorities in the following month. These are to be borne by the buyers. The selling price
of products is exclusive of these levies.
g. The company enjoys a cash credit facility from its banker to the extent of Rs. 35 lakhs,
which is fully drawn. The interest payable is @ 17% which is charged every quarter, i.e.,
June, September,. December and March. The company carries its banking operation
presently through a current account. IICWA (Fin), June 1991J
36. The month by month forecast of profitability of a company for the 5 months of May to
September is given below (Rs. 000).
May June July Aug Sep
Material consumed 60 70 80 102 90
Wages 32 32 32 40 32
Depreciation 7 7 7 7 7
Factory expenses 5 5 5 5 5
Rent 3 3 3 3 3
Salaries and office expenses 32 32 32 32 32
Advertising and publicity 12 14 10 16 20
Sales commission 8 9 10 13 11
159 172 179 218 200
Sales 160 180 200 260 220
Profit 1 8 21 42 20
Raw material stock (end of month) 70 80 90 70 60
The following additional information is given:
a. On average, payment is made to suppliers one month after delivery.
b. The lag in payment of wages is one-eighth of month.
c. Factory expenses are paid during the month incurred.
d. Rent is paid quarterly on the last day of March, June, September and December.
e. Salaries and office expenses are paid in the month in which -they arise.
Budgetary Control : 537

f. Advertising and publicity expenditure is paid monthly but two months' credit is taken.
g. Sales commission is paid one month in arrear.
h. On average, debtors take two months' credit.
1. Cash balance at July 1 is Rs. 52,000.
j. In September, Rs. 30,000 will be paid for machinery. A dividend and tax thereon
amounting to Rs. 6,000 will be paid ttl August. Investment grants of Rs. 20,000 would be
received in September.
You are required to prepare a cash budget for each of the three months to September 30.
[M.Com., UoM, June 1985J
37. The projected sales and anticipated purchases of ABC Limited for the months of July to
November 1983 are:
Month Sales (Rs.) Purchases (Rs.)
1983 July 6.20,000 3.80,000
August 6,40,000 3.33,000
September 5,80,000 3,50,000
October 5,60,000 3,90,000
November 6,00,000 3,40,000
The wages are expected to be Rs. 1,00,000 per month, the management is expected to pay two
months' wages as bonus during October 1983. The company is expected to pay an advance
tax for income tax Rs. 90,000 before September 1983. The company has ordered in June
1983 for a machine costing Rs. 16 lakh which is expected to be delivered in January 1984.
The company pays 5% as advance on June 30, 1983 and another 10% advance after 3 months.
The baiance on the delivery of the machine. The company extends two months' credit for the
customers and the company enjoys one month credit from the suppliers. The general
expenses for the company is Rs. 60,000 per month payable at the end of each month. The
company anticipates to receive interim dividend of 10% for the investment of 90,000 shares
of Rs. 10 each during October 1983. The company anticipates to have an overdraft of Rs.
40,000 on lst September 1983 (limit sanctioned is Rs. 55,000). Draw a cash budget for
September to November 1983 for approaching your bankers for a short term further credit.
[C.SJ
38. Prepare a monthly cash budget for six months period ending December 31 from the following
information (Rs.):
Month Sales Material Wages POHEs AOHEs SOHEs DOHEs R&D
April 40,000 15,000 4,000 2,000 1,500 700 300 400
May 50,000 25,000 4,600 2,200 1,400 750 350 400
Management Accounting : 538

June 30,000 15,000 3,000 2,300 1,420 850 300 500


July 40,000 25,000 3,200 2,100 1,380 750 400 500
August 50,000 30,000 3,600 2,400 1,410 950 500 600
September 60,000 35,000 4,000 2,500 1,440 1,000 600 600
October 70,000 40,000 4,200 2,700 1,460 1,050 630 700

November 80,000 45,000 4,400 2,800 1,470 1,100 675 700
December 90,000 50,000 4,800 3,000 1,500 1,200 750 700

a. Cash balance on July 1 was expected to be Rs. 82,500.


b. Expected capital expenditure:
1. Plant and machinery to be installed in August at a cost of Rs. 15,000 will be payable
on September 1;
2. Extension to research and development department amounting to Rs. 5,000 will be
completed on August 1. It would be payable at Rs. 1,000 per month from completion
date.
c. Under a hire purchase agreement, Rs. 2,000 is to be paid each month.
d. A sales commission of 5% on sales is to be paid within the month following the actual
sales.
e. . Period of credit allowed by suppliers 3 months.
Period of credit allowed to customers 2 months.
Delay in payment of overheads 1 month.
Delay in payment of wages 114 month.
f. Income tax of Rs. 35,000 is due to be paid on October 1.
g. Preference shares dividend of 10% on capital of Rs. 10,00,000 is to be paid on November 1.
h. 10% calls on ordinary share capital of Rs. 2,00,000 is due on July 1 and September 1.
i. Dividend from investment amounting to Rs. 15,000 expected on November 1.
j. Cash sales of Rs. 3,000 per month are expected; no commission payable.
[M.Com., UoM, May 1983J
39. Adamjee and Burjorjee doing business in partnership have developed a new technique to
manufacture a certain product. The cost of the factory is estimated at Rs. 1,50,000 and that of
plant and machinery at Rs. 2,50,000. A limited company is to be formed from January 1966
when production will begin. The partners will subcribe Rs. 3,00,000 towards the share capital
of the company. The following is the other relevant information.
Budgetary Control : 539
a. Costs: Prime cost is estimated at Rs. 20 per unit, being Rs. 10 direct labour and Rs. 10
direct material. Factory and administration overhead cost is estimated for each month to
be Rs. 10,000. A publicity campaign is to be launched from February. It is to cost Rs.
10,000 for the month and then Rs. 2,000 for each subsequent month. Selling expenses are
expected to amount Rs. 15,000 per month commencing from 1st February. Preliminary
expenses have been estimated at.Rs. 20,000 and initial contingencies at Rs. 25,000.
b. Sales: These have been estimated as follows: January: 2,000 units; February: 3,000 units;
and March to December: 4,000 units per month. The selling price has been fixed at Rs.
50 per unit.
c. Stocks: Raw material costing Rs. 60,000 are to be purchased for the initial stocks and this
is to be maintained at this level. A stock of finished goods is also to be maintained at 500
units.
d. Terms of purchase and sales: Materials purchased are to be paid for within one month.
Twenty per cent of sales will be for cash and the balance on credit receivable within one
month. On an average, credit sales will be allowed a cash discount of 5%. All overh~ads
are payable within the month in which they are incurred.
You are required to prepare initial Cash Forecast, for four months up to April 1966 divided to
show working capital and permanent capital. lCA (Fin), November 1966J
40. Vinak Ltd., a company engaged in the manufacture of electrical appliances, has set the
following budget for 1984.
Immersion Table Bread Room
Particulars
Heaters Lamps Toasters Heaters
Production (units) 40,000 10,000 50,000 30,000
Selling Price per unit (Rs.) 30.00 50.00 60.00 80.00
Cost per unit:
Direct materials 6.00 13.50 10.50 24.00
Direct labour 7.50 10.00 18.00 24.00
Variable overheads 4.50 10.00 12.00 13.00
Fixed overheads 7.50 10.00 18.00 24.00
ProfitILoss 4.50 6.50 1.50 - 5.00

When the budget was placed before the Budget Committee, the Marketing Manager put up a
proposal to increase the sales by 20,000 additional units for which capacity existed. The
additional 20,000 units could be one product or any combination of products. The proposal
was accepted by the committee. The co.mmittee also decided that the production capacity for
the next year, namely 1985 could be set in such a way that there would be a further increase in
the output by 50,000 units over and above the increase of 20,000 units envisaged for 1984.
The additional production of 50,000 units would be of Table Lamps only for whicJt a new
plant would be acquired. The additional fixed expenses of the new plant were estimated at
Management Accounting : 540
Rs. 70,000 per annum. During 1985, raw material and labour costs were expected to increase
by 10% but the other costs and selling expenses would remain the same.
Required: (a) Set a budget for 1984 in such a way that the additional capacity of 20,000
units is utilised to maximize the profits.
(b) Set a budget for 1985.
(c) Assuming that the increased output may not fully materialize, calculate the
number of units of Table Lamps required to be sold in 1985 at the given price
in order to ensure that profitability, at least at 1984 level, is maintained.
[ICWA (Fin), December 1983J
4l. In the year ended December 31 st (year 1), the actual costs, output and sales of a company
manufacturing a range of products are as under.
Product: A B C 0
Per unit Selling Price (Rs.) 20 40 50 30
Variable costs: Direct material 4 9 10 3
Direct wages 3 5 10 4
Units manufactured and sold 7,500 5,000 3,000 6,000
Variable overheads are incurred at a rate of 200% of direct wages. Fixed overheads are Rs.
2,00.000 for the year. The comp~ny's summarized budgeted results for the year 1 ended
December 31st are:
Sales Rs. 7,00,000
Variable cost of sales 4,55,000
Contribution 2,45,000
Fixed overheads 1,90,000
Budgeted Profit 55,000
In preparing its budget for the year 2 ending December 31st, the company has made the
following allowances for inflation over the actual figures for year 1: .
a. An increase of 10% in all the selling prices; these increases are not expected to alter the
quantities of each product sold as compared with year 1.
b. An increase in unit product cost of: Direct material 10%
Direct wages 20%
Variable overheads 10%
c. An increase of 2% in fixed overheads.
In addition to these allowances for inflation, the company proposes the following changes
in its cost, sales volume, and selling price structure.
Budgetary Control: 541
Product A: Increase the price by 10% yielding a reduction of 5% in the volume sold.
Product B: Use different materials which will reduce direct material cost by Rs. 2 per
unit and reduce volume sold by 4%.
Product C: (a) Incur advertising cost of Rs. 10,000 for the year which is expected to
increase sales by 20%.
(b) Buy a machine costing Rs. 8,000
Product D: Reduce the selling price by 10%, giving a 15% increase in the sales volume.
Increase the stocks held by an average of Rs. 40,000 over the whole year; this would be
financed by bank overdraft at an interest rate of 12% per annum.
Increase the size of the delivery van fleet at an outlay of Rs. 9,000 and an increase in
annual fixed costs of Rs. 2,000 (excluding depreciation).
The company calculates its depreciation on a straight line basis with a standard life of five
years for production equipment and three years for non-production equipment.
You are required:
a. To show, in a format helpful to management, a summary statement of the budgeted and
actual results for the year 1 ended December 31st with an analysis of the difference
between the two profits.
b. To compile a budgeted profit and loss account for the company for the year 2 ending
December 31 st after taking account of allowances for inflation and the additional changes
proposed. [ICWA (Fin), December 1989J
42. Amicable Relations Ltd., is facing a trade union demand for an increase of 15% on the hourly
wage rates, in response to a management offer of 5%. The management is reluctant to agree
to such a demand but is willing to consider an increased offer provided it is linked with
productivity. The suggestion iSlO offer 5% on basic hourly rates plus Re. 0.15 for every
standard hour of output produced. If this is agreed to, then it is expected that production
would increase by 10% within the budgeted hours (normal factory capacity). One standard
hour is required to produce one unit of output. In order to sell the increased output, it would
become necessary to effect a reduction of 2.5% in the selling price. The draft budget for the
forthcoming year, excluding the wages and sales increase, is as follows (Rs. in lakhs).
Sales (15,00,000 units) 60
Direct material 12
Direct wages 18
Variable production overheads 3
Fixed production overheads 10
Variable sales overheads (5% of turnover) 3
Fixed sales overheads 6
Management Accounting: 542
Variable distribution overheads 1
Fixed distribution overheads 1
Fixed administration overheads 2 56
Profit 4
You are required to work out alterations in the budget based on the following:
a. If the trade union demand is accepted by management.
b. If the management's proposal of wage increase linked to productivity is accepted by the
trade union. -
lCA (Fin), November 1988]
43. Lookahead Ltd., produces and sells a single product. Sales budget for the current calendar
year by quarter is as under:
Quarter: I II III IV
Units to be sold: 12,000 15,000 16,500 18,000
The year is expected to open with an inventory of 4,000 units of finished product and close
with an inventory of 6,500 units. Production is customarily scheduled to provide for two-
thirds of the current quarter's sales demand plus one-third of the following quarter's demand.
The standard cost details for one unit of the product is as follows.
Direct material, 10 lb at 50 paise per lb
Direct labour, 1 hour 30 minutes at Rs. 4 per hour
Variable overheads, 1 hour 30 min,utes at Re. 1 per hour
Fixed overheads, 1 hour 30 minutes at Rs. 2 per hour based on a budgeted production
volume of 90,000 direct labour-hours for the year.
a. Prepare a production budget, by quarters, showing the number of units to be produced and
the total costs of direct material, direct labour, variable overheads, and fixed overheads.
b. If the budgeted selling price per unit is Rs. 17, what would be the budgeted profit for the
year as a whole?
c. In which quarter of the year is the company expected to break-even?
lCA (Int), May 1986]
44. Sterling Enterprises has prepared a draft budget for the next year as follows (10,000 units):
Sale price per unit Rs.30
Variable costs per unit:
Direct material Rs.8
Direct labour (2 hours x Rs. 3) 6
Budgetary Control : 543
Variable overheads (2 hours x Re. 0.50) 1 15
Contribution per unit 15
Budgeted contribution 1,50,000
Budgeted fixed costs 1,40,000
Budgeted profit 10,000

The board of directors is dissatisfied l with this budget and asks a working party to come up
with an alternative budget with higher profit figures. The working party reports back with
some suggestions which will lead to a budgeted profit of Rs. 25,000. The company should
spend Rs. 28,500 on advertising and Iput the sales price up to Rs. 32 per unit. It is expected
that sales volume w9uld also rise, in spite of the price increase, to 12,000 units. In order to
achieve the extra production capacity, however. the work force must be able to reduce the
time taken to make each unit of the product. It is proposed to offer a pay and productivity
deal in which the wage rate per ho\.ir is increased to Rs. 4. The hourly rate for variable
overheads will be unaffected. Preparb a revised budget giving effect to the above suggestions.
leA (Fin), May 1991J
45. ABC Ltd., makes two types of polisJi, one for floors and the other for cars. It sells both these
types to industrial users only in one litre containers. The specifications for the two products
per batch of 100 litres are as follows.
Floor Polish Car Polish
Material: Delta 120litres 100 litres
Gamma 20kgs 10 kgs
Containers (cost per 100) Rs.lOO Rs.100
Manufacturing 12 man-hours 16 man-hours
Primary packing 5 man-hours 5 man-hours

During the six months to end of September 30th, the company expects to sell 15,000 litres of
Floor Polish at Rs. 9 per litre and 25,000 litres of Car Polish at Rs. 7 per litre. Material is
expected to cost Re. 1 a litre for Delta and Rs. 8 a kg for gamma. Manufacturing wages in the
industry are stable at Rs. 6 per hour and packing wages at Rs. 4 per hour, throughout the
period. Flexible overhead expense. budgets are operated for manufacturing and packing
departments based on the number of man-hours worked. These budgets for six months up to
the end of September are as follows.
Manufacturing Department Primary Packing Department
5,000 (man-hours) Rs.40,000 1,700 (man-hours) Rs.26,000
6,000 50,000 1,900 28,000
7,000 . 60,000 2,100 30,000
8,000 80,000 2,300 32,000
Management Accounting: 544
General administrative overheads are budgeted at Rs. 37,000. At the beginning of the period
(1st April), packed stocks wOuld be: Floor Polish 2,000 litres; Car Polish 3.000 litres. By the
end of the period (30th September), it is desired to maintain the packed stocks of the products
at 3,000 litres and 4.000 litres, respectively. The following is required:
a. A statement of the standard prime cost per 100 litres of each product.
b. A sales and production budget (in quantities) for the six months up to September 30th.
c. A profit statement for the period. Show separate GP for the two products but do not
allocate overheads between them. No overheads are included in stock valuations.
[ICWA (lnt), June 1987J
46. Shri Girish is starting a new business on 1st January 1977. He is able to provide Rs.
30,000 on that date. He has approached the New Rural Bank for financial assistance. He
has been asked by the Bank to produce a monthly cash budget for the first year of trading.
You, as a consultant to Shri Girish, ascertain the following facts and estimates on the
basis of information and other relevant data furnished by Shri Girish.
A. a. Premises would cost Rs. 1,20,000. 10% deposit will be required to be paid on 1st
January 1977. Interest on balance amount to be paid will be 10% per annum till
the property is finally purchased, and paid for, on 1st April 1977.
b. A mortgage of 50% of purchase price has been arranged one month after
completion of purchase of premises. The mortgage amount is repayable over ten
years by equal quarterly instalments payable in arrears. Interest at 8% per annum
is payable with each instalment.
c. A sub letting of premises is arranged at a rent of Rs. 9,000 per annum payable
quarterly in advance commencing from 1st April 1977.
B. a. Stock will be purchased on credit in December 1976 at a cost of Rs: 54,000 and
will be maintained at this level up to' August 1977, but will be permanently
increased in September 1977 by 75% to meet the higher anticipated sales in 1978.
C. b. All goods purchased are paid at the end of the second month after the month in
which they are purchased.
c. Shri Girish will mark up all goods by 33 113% on cost price.
a. Credit sales are expected at:
Rs. 9,000 per month for the first quarter.
Rs. 12,000 per month for the second quarter.
Rs. 15,000 per month thereafter.
b. Debtors are expected to pay by the end of the month following the month in
which the sales were made.
c. Cash sales are estimated to be:
Rs. 30,000 per month for the first quarter.
Budgetary Control : 545

Rs. 33.000 per month for the second quarter.


Rs. 36,300 per month for the third quarter.
Rs. 39,930 per month for the last quarter.
D. Expenses will be paid as under:
Rs. 2,700 per annum for insurance payable in advance on 1st January 1977.
Rs. 6,300 per annum for rates and taxes payable in advance in two equal instalments in
April 1977 and October 1977.
Rs. 5,250 monthly overheads payable at the end of the same month in which they are
incurred.
E. Shri Girish expects his drawings to be Rs. 1,800 per month, all months, except June and
December when he intends to wIthdraw a larger sum of Rs. 3,000 during each of these
two months.
You are required to prepare:
a. Monthly cash budget for the first year of trading, and
b. Projected profit and loss account and balance sheet for the year (ignoring
depreciation)
rCA (Fin), November 76J
47. The following is the sales budget of a company engaged in manufacturing and marketing
certain consumer products (in 3 lines), their market being Eastern and Western Zones (Rs.
lakhs):
Eastern Western Total
Product
Units Rs. Units Rs. Units Rs.
A 30,OqO 15.00 16,000 8.00 46,000 23.00
B 10,000 6.00 15,000 9.00 25,000 15.00
C 4,000 3.20 6,000 4.80 10,000 8.00
24.20 21.80 46.00

No changes are expected in the inventory levels. The following are the unit standard cost
details for the three products.
A (Rs.) B (Rs.) C (Rs.)
Direct materials: Material x at Rs. 6 per kg 12 24 18
Material Y at Rs. 4 per kg 8 4
Direct labour: Rs. 4 per hour 12 16 20
Factory overheads: Variable at Re. 1 per std. hr 3 4 5
Fixed at Rs. 2 per std. hr 6 8 10
Management Accounting : 546
Variable overheads comprise of indirect material, indirect labour and indirect expenses in the
ratio of 50 : 25 : 25. Fixed factory overheads stated above are based on the following product
mix: Product A: 20,000 units; Product B: 15,000 units; and Product C: 10,000 units. The mix
of fixed factory overheads consists of indirect material, indirect labour and indirect expenses
in the ratio of 30 : 30 : 40. Price of material X is expected to increase by Re. 0.20 per kg in
the budget period. There will be 2% inefficiency (i.e., 2% wastage allowance) in case of
direct materials. A 3% increase in productivity of direct labour is expected. No other
variances in direct costs are expected. These variances and any other variances in direct items
have to be built into the budgets. The selling and distribution cost budget for the two zones is
as follows.
Eastern Western
Zonal Manager's Control: Commission 10% on std. Gross Profit 10% on std. GP
Travelling Rs.40,OOO 35,000
Advertising 15,000 12,000
Office expenses 10,000 8,000
Other Fixed Expenses: Salary 20,000 20,000
Perquisites 2,000 2,000
Depreciation 5,000 4,000
Insurance 1,000 1,000

The head office selling and distribution expenses are: Advertising and Sales Promotion Rs.
70,000; Salaries Rs. 42,000; Stationery, Postage, etc., Rs. 50,000; Depreciation Rs. 5,000;
Insurance Rs. 1,000. Head office administration expenses are Rs. 2,00,000 and this should be
met out of gross profit. The average rate of tax is 40%. You are required to prepare the
budgeted income statement for the company. lCA (Fin), November 1981J'
48. A company manufacturing thee labour intensive products A, B and C prepared its budget for a
year based on proposed annual production of 12,000. 6,000 and 8,000 units of A, B and C
respectively. The input for each of the products passes through three highly specialized producing
departments X, Y and Z. The standard data per unit in respect of the three products are:
A B C
Direct material (Rs.) 10.00 25.00 17.00
Direct labour (hours): Department X 35 18 40
Department Y 12 8 26
Department Z 44 10 12
Variable overheads (Rs.) 4.00 3.50 6.00
SeIling 'price (Rs.) 89.00 89.00 137.00
Total fixed cost: Rs. 2,00,000.
Budgetary Control: 547

Departmental labour rates (per hour): Department X: Re. 1.00; Y: Rs. 1.50; and
Z: Rs. 1.25
No sooner the budget was ready, it was revealed that potential sales could be increased to
15,000, 8,000 and 10,000 units respectively of A, B and C and that the demand could be
satisfied with the existing resources of the company without incurring any additional fixed
expenditure. The constraint, however, was that workers suitable for Department Z being in
short supply, no addition to the existing labour strength of that department was possible. It
was not also possible to inter-change workers between the departments. For the departments
X and y, although fresh workers from outside were available, the new recruits had to be put
on a three months' training programmes before they could be considered suitable for the jobs.
With a view to increasing profit, the management proposed to revise the budget, basing it on
the most profitable sales mix. You are required to:
1. Prepare a statement showing the net profit as originally planned.
2. Determine the revised profit based on the best revised sales mix.
3. Calculate the deficient or surplus labour hours in Departments X and Y, consequent upon
the implementation of the proposal to revise the sales mix.
4. Give your comments on the proposal indicating whether or not you agree to it.
[ICWA (Fin), December 1980J
49. The following data relates to the working of a factory at Wardha for the year 1976 (capacity
worked 50%).
Fixed cost: Salaries Rs.84,000
Rent and rates 56,000
Depreciation 70,000
Other administrative expenses 80,000 Rs. 2,90,000
Variable costs: Materials 2,40,000
Labour 2,56,000
Other expenses 38,000 5,34,000
Possible sales at various levels of working are:
Capacity: 60% 75% 90% 100%
Sales (Rs.): 9,50,000 11,50,000 13,75,000 15,25,000

Prepare a flexible budget and show the forecast of profit at 60%, 75%, 90% and 100%
capacity operations. [M.Com., Nagpur, 1977J
Management Accounting : 548

50. For production of 10,000 electrical automatic irons, the following are budgeted expenses.
per unit
Direct materials Rs.60
Direct labour 30
Variable overheads 25
Fixed overheads (Rs. 1,50,000) 15
Variable expenses (direct) 5
Selling eX}Jenses (10% fixed) 15
Administration expenses (Rs. 50,000 rigid for all levels of production) 5
Distribution expenses (20% fixed) 5
Total cost of sale per unit 160
Prepare a budget for production of 6,000, 7,000 and 8,000 irons showing distinctly marginal
cost and total cost. ICA (Int), M03 1977J
51. The monthly budgets for the manufacturing overhead of a concern for two levels of activity
were as follows.
Capacity: 60% 100%
Budget production (units) 600 1,000
Indirect wages Rs.l,200 Rs.2,000
Consumable stores 900 1,500
Maintenance 1,100 1,500
Power and fuel 1,600 2,000
Depreciation 4,000 4,000
Insurance 1,000 1,000
a. Indicate which of the items are fixed, semi-fixed and variable
b. Prepare a flexible budget for 80% of the activity. IICWA (Int)J
52. Draw up a flexible budget for overhead expenses on the basis of the following data and
determine the overhead rate of 70%, 80% and 90% plan capacity.
Capacity Levels
Particulars
70% (Rs.) 80% (Rs.) 90% (Rs.)
Variable overheads:
Indirect labour - 12,000 -
-.-
Stores including spares - 4,000 l
.. -
Budgetary Control : 549

Semi-variable overheads:
Power (30% fixed, 70% variable) - 20,000 -
Repairs and maintenance (60%F, 40% V) - 2,000 -
Fixed overheads:
Depreciation - 11,000 -
Insurance - 3,000 -
Salaries - 10,000 -
Total overheads - 62,000 -
Estimated direct labour hours - 1,24,000 -
[ICWA (Int), June 1980J
53. ABC Ltd., have prepared the budget for the production of one lakh units of the only
commodity manufactured by them for a costing period as under (Rs.lakhs).
Raw materials 2.52
Direct labour 0.75
Direct expenses 0.10
Works overhead (60% fixed) 2.25
Administrative overheads (80% fixed) 0.40
Selling overheads (50% fixed) 0.20
The actual production during the period was only 60,000 units. Calculate the revised
budgeted cost per unit. [ICWA (Int), January 1962J
54. The budgeted output of a factory, specializing in the production of a single product, at the
optimum capacity, is 6,400 units per annum. The total cost at this level amounts to Rs.
1,76,048 which is arrived at as follows.
Variable costs: Power Rs.l,440
Repairs 1,700
Miscellaneous 540
Direct material 49,280
Direct labour 1,02,400
1,55,360
Fixed cost 20,688
Total Cost 1,76,048
Having regard to possible impact on sales turnover by market trends, the company decides to
have a flexible budget with a production target of 3,200 units and 4,800 units (the actual
quantity proposed to be produced being left to a later date before commencement of the
Management Accounting : sso
budget period). Prepare a flexible budget for production levels at 50% and 75%. Assuming
the sales price per unit is maintained at Rs. 40 as at present, .indicate the effect on net profit.
Administration, selling and distribution expenses continue at Rs. 3,600 per annum.
[M.Com., UoM, May 1975J
55. A company, producing electronic watches, estimates the following factory overhead cost for
producing 5,000 units.
Indirect materials Rs. 16,000 Expendable tools Rs. 8,000
Indirect labour 30,000 Supervision costs 8,000
Inspection cost 16,000 Equipment depreciation 4,000
Heat, light and power 8,000 Factory rent 4,000
Indirect labour, indirect material and expendable tools are entirely variable; heat, light and
power, and inspection costs are variable to the extent of 50% and 40% respectively. Other
costs are fixed costs for a month. Prepare a flexible budget for overheads for production of
4,000 and 6,000 units per month. Also find the average factory overheads per unit for these
two production levels. [CSJ
56. The total costs at two capacity levels have been estimated as follows:
Capacity (%): 50 80
Total costs (Rs.): 1,50,000 1,98,000
You are required to prepare a flexible budget showing the revenue, variable and fixed cost
components of the firm at 40%, 60% and 100% capacity using the following pieces of
additional information.
a. The product price has to be determined by considering the total costs at 70% capacity and
adding profit margin of 25% on selling prices.
b. 70% of fixed costs represent production costs and the remaining are administrative
overheads.
c. 40%, 30%, 15% and 15% of the variable costs represent material, labour, production
overheads, and selling and distribution overhead respectively.
[M.Com., UoM, June 1984J
57. AB Ltd., has prepared the following budget for 1979-80.
Raw materials 40%
Direct wages 25
Factory overheads: Variable 10
Fixed 5
Administration and selling overheads: Variable 6
Fixed 12
Profit 2
Total sales value 100
Budgetary Control: 551

After considering the half yearly performance, it was felt that budgeted volume of sales would
not be obtained but the company expected to achieve 80% equivalent to a sales value of Rs.
1601akhs. At this stage, the company receives an export order for its usual line of products.
The prime cost of the export order is Rs. 13 lakhs and special export expenses are estimated at
Rs. 40.000. you are required to:
a. Present the original budget and the revised budget based on 80% achievement showing
the quantum of profit or loss.
b. Prepare a statement of budgeted cost for establishing percentages of overhead recovery
rates.
c. Calculate the lowest quotation for the export order and offer your comments.
lCA (Int), November 1979J
Answers
32. (a) Rs. 49,59.500; (b) Rs. 53,40,500.
33. Budgeted Production (tons): 55 (AB) and 210 (CO), Materials to be purchased (tons): 49
(A), 41 (B), 205 (C) and 55 (0) and Total Purchase Bill: Rs. 24,500 (A), Rs. 16,400 (B),
Rs. 20,500 (C) and Rs. 11,000 (0).
34. Rs. 32,600
35. Profit: April Rs. 7,50,417; May Rs. 11,00,417; June Rs. 12,75,416; and Total Rs.
31,26,250; Month-end Cash balance: April Rs. 4,50,000; May Rs. 18,50,000; June Rs.
34,51,250; and Total Rs. 34,51,250.
36. Purchase Bill: June Rs. 80,000; July Rs. 90,000; August Rs. 82,000; September Rs. 80,000.
Month-end Cash balance: July Rs. 42,000; August Rs. 26,000; and September Rs. 32,000.
37. Month-end Cash balance: September -Rs. 3,000; October -Rs. 1,43,000 and November-
--Rs.l,13,OOO.
38. Month-end Cash balance: July Rs. 1,28,480; August Rs. 1,22,850; September Rs. 1,40,590;
October Rs. 1,17,300; November Rs. 47,910; and December Rs. 67,465.
39. Month-end Cash balance: January -Rs. 10,000; February Rs. 11,000; March Rs. 68,000;
and April Rs. 1,53,000.
40. (a) Since the Bread Toaster is the most profitable product, among the four, the additional
capacity of 20,000 units is to be utilized to produce an additional 20,000 units of Bread
Toaster, Profit for 1984 = Rs. 5,60,000;
(b) Profit =Rs. 7,76,500; (c) 34,700 units.
41. (a) Profit for year - 1 = Rs. 52,500; Variances, Sales Revenue Rs. 20,000 (A); Contribution
Rs. 7,500 (F); Fixed Overheads Rs. 10,000 (A); Profit Rs. 2,500 (A);
(b) Budgeted Profit for year - 2 Rs. 59,000 (for this purpose, compute the Material Cost,
Direct Wages, Variable Overheads, Selling Price per unit; Budgeted Sales Volume,
Management Accounting : 552
Total Fixed Overheads, etc., in the light of the anticipated changes. This will facilitate
the preparation of Budgeted Income Statement for year - 2).
42. (a) If the union's demand is accepted, revised budgeted profit = Rs. 1,30,000;
(b) If management proposal is implemented: revised budgeted profit =Rs. 3,15,170.
43. Budgeted Production (units): 13,000 (I Quarter), 15,500 (11), 17,000 (III), 18.500 (IV) and
64,000 (Total); Budgeted Costs: Rs. 2,07,500 (I), Rs. 2,38,750 (11), Rs. 2,75,500 (111),
Rs.2,76,250 (IV) and Rs. 9,98,000 (Total), Budgeted Profit for the year = Rs. 96,750. BEQ
= 40,000 units, expected to break-even in the third quarter.
44. Budgeted Profit =Rs. 25,000
45. Prime Cost per 100 litres : Rs. 472 (Floor); Rs. 396 (Car);
Budgeted Production (litres): 16,000 (Floor), 26,000 (Car);
Required Man-hours; 6,080 (Floor), 2,100 (Car);
Budgeted Profit Rs. 22,400.
46. Month-end Cash balance: Rs. 38,250 (January), Rs. 16,200 (February), Rs.18,900 (March).-
Rs. 87,000 (April), -Rs. 18,300 (May), -Rs. 15,300 (June), -Rs. 5,500 (July), Rs. 2,250
(August), Rs. 8,025 (September), Rs. 16,530 (October), -R,s. 17,235 (November), -Rs,
11,753 (December), -Rs.1I,753 (Total);
Purchases (January to December and Total): Rs. 29,250; Rs. 29,250; Rs. 29,250; Rs.
33,750; Rs. 33,750; Rs. 33,750; Rs. 38,475; Rs. 38,475; Rs. 78,975; Rs. 41,198; Rs. 41,198;
Rs. 41,198; Total Rs. 4,68,519.
Profit Rs. 71,591:
Balance Sheet total Rs. 2,29,500.
47. Material Cost: Rs. 14,03,928 (X); Rs. 4,16,160 (Y); Labour Cost Rs.ll,17,440; Profit after
Tax: Rs.2,71,243.
48. (1) Profit Rs. 2,12,000 (Original);
(2) Best Revised Mix : 6,500 units (A) + 8,000 units (8) + 10,000 units (C). Profit Rs.
2,22,000.
(3) Surplus in X: 76,500 hours; Deficient in Y: 2,000 hours;
49. Profit: Rs. 19,200 (60%); Rs. 59,000 (75%); Rs. 1,23,800 (90%); Rs.l,67,000 (100%)
50. Total Costs = Rs. 10,50,000 (at 6,000 units); Rs. 11,87,500 (at 7,000 units); Rs. 13,25,000
(at 8,000 units).
51. Fixed: Depreciation and Insurance;
Semi-fixed: Maintenance, and Power and Fuel;
Variable: Indirect Wages and Consumable Stores;
Total Overheads at 80% =Rs. 10,900.
Budgetary Control : 553

52. Total Overheads: Rs. 58,150 (70%); Rs. 62,000 (80%); and Rs. 65,850 (90%); Overhead
Expenses per Direct Labour Hour = Re. 0.536 (70%); Re. 0.5 (80%) and Re. 0.472 (90%).
53. Revised Budgeted Cost per unit = Re. 7.4
54. Profit = Rs. 26,032 (50%); Rs. 51,192 (75%)
55. Overhead Expenses per unit: Rs. 22.78 (at 4,000 units); Rs. 17.81 (at 6,000 units)
56. Sales Revenue: Rs. 1,38,667 (40%), Rs. 2,08,000 (60%), and Rs. 3,46,667 (100%);
Variable Cost: Rs. 64,000 (40%), Rs. 96,000 (60%) and Rs. 1,60,000 (100%);
Fixed Cost: Rs. 70,000
57. (a) Profit: Rs. 4,00,000 (original); -Rs. 3,60,000 (Revised);
(b) Factory Overhead Expenses: Variable: 40% of direct wages
Fixed: 20% of direct wages
Administration and S&D Overhead Expenses: Variable: 7 ..5% of works cost or 8% of
variable work cost or 6% of sales;
Fixed: 15% of works cost or 12% of sales.
(c) Rs. 16,60,000.
Chapter - VII

STANDARD COSTING
Objectives: The important objectives of this chapter are:
• To understand the Meaning of Standard Costs, Standard Costing, and types of Standards.
• To know the Objectives of Standard Costing and also to identify the Differences that exist
between Budgetary Control and Standard Costing.
• To learn both the Advantages and Umitations of Standard Costing.
• To understand the Meaning of Variance and Variance Analysis.
• To discuss the different Kinds of Variances •
• To solve the problems relating to different types of Variances.
Structure
• Introduction
• Meaning and Definition of Standard Costing
• Standard Costs
• Types of Standards
• Objectives of Standard Costing
• Standard Cost and Estimated Cost
• Standard Costing and Budgetary Control
• Advantages and Umitations of Standard Costing
• Analysis of Variances
o Cost Variances
Material Cost Variances
+ Material Price Variance
+ Material Usage Variance
= Material Mix Variance; = Material Yield Variance
Labour Cost Variances
+ Labour Rate Variance
+ Labour Efficiency Variance
= Labour Mix Variance; = Labour Yield Variance
+ Idle Time Variance
Overhead Cost Variances
+ Variable Overhead Variance
= Variable Overhead Expenditure Variance
= Variable Overhead Efficiency Variance
+ Fixed Overhead Variance
= Fixed Overhead Expenditure Variance
= Fixed Overhead Volume Varinace
* FOH Efficiency Variance; * FOH Capacity Variance
* FOH Calender Variance
o Sales Variances - Value Method - Sales Value Variance
Sales Price Variance
Sales Volume Variance
+ Sales Mix Variance
+ Sales Volume Variance
= Quantity Variance; = Mix Variance
• Disposal of Variances
• Illustrations
• Summary of the Chapter
• Key Terms to Remember
• Questions for Self-study
Standard Costing : 555

Introduction
The success or otherwise of an industrial enterprise depends, to a greater extent, upon
how effectively it has controlled its costs. In order to exercise proper control over the costs,
. historical approach to Costing provides a very little scope. Because, historical approach to
costing collects and records the costs in the books of accounts after they have been incurred.
Since the management obtains the information about costs after they are being incurred, it will not
be in a position to take any action as the costs have already been incurred. Further, none of either
the present or the future managerial decisions is in a position to alter these costs. Therefore,
Prospective Approach to Costing (viz, Standard Costing) has been suggested. Because, under
this Costing System, various elements of costs are estimated in advance and used to compare with
the actual costs so that the efficiency of personnel involved can be measured. Because, Standard
Costing specifies the standard costs within which a given level of output is to be achieved. For
instance, assume that a company has set the target of producing its product by incurring Rs. 4 of
material cost per unit of output. That means, the company has given a target to its Production
Department to produce the product by incurring not more than Rs. 4 of material cost per unit of
output. On the basis of comparison of this standard material cost with that of the actual,
management will be able to evaluate the efficiency of its Production Department and also that of
the Purchase Department as the material cost is influenced by the price at which the materials
(used in the production) were acquired by the Purchase Department. This helps initially to
evaluate the performance. Further, exact reason for failure to achieve the target can be identified
and the responsibility be fixed. On the basis of the identification of reasons, suitable measures
can be suggested and taken to correct the deficiencies.
Meaning and Definition of Standard Costing
Realization of deficiencies of Historical Costing (i.e., Historical Approach to Costing)
led to the evolvement of Standard Costing which has a prospective approach to Costing.
However, it should be noted at this stage that Standard Costing is only a control device and not a
distinct method of Product Costing. That means, it is not a substitute for any method of Product
Costing. It only suggests to establish standards for each element of cost and see that the activities
are performed by incurring not more than the pre-determined or standard costs. Therefore, the
prospective approach can be introduced into any of the methods of Product Costing. That means,
it can be used with any method of Costing, viz., Process Costing and Job Costing. In this
background, Standard Costing has been defined by the Chartered Institute of Management
Accountants (CIMA), London as the preparation and use of standard costs, their comparison
with actual cost and the analysis of variances to their causes and points of incidence. Brown
and Howard have defined Standard Costing as ••• a technique of cost accounting which
compares the standard cost of each product or service with actual cost to determine the
efficiency of the operation, so that any remedial action may be taken immediately. w. W.
Big has opined that standard costing discloses the cost deviations from standard and classifies
these as to their causes, so that management is immediately informed of the sphere of
operations in which remedial action is necessary. This way, a number of institutions and
individuals have defined Standard Costing meaning, more or less, the same as presented above. A
careful analysis of the above definitions reveals some of the important steps involved in
introducing Standard Costing System. They are summarized below.
Management Accounting : 556
1. Determination of standards for each element of costs;
2. Collection of information abou.t actuals pertaining to each element of cost on a continual
basis;
3. Comparison of actual cost with standard cost to find out the 'deviations (known as
variances);
4. Analysis of variances to find out the areas where the company h~ fared well and the
areas in which it has failed to achieve the target result;
5. Identification of probable reasons for such variances; and
6. Suggesting and taking suitable measures.
Standard Costs
The successful introduction of Standard Costing depends primarily on the accuracy with
which the standards are set for each element of costs. This is one of the pre-requisites for
introducing Standard Costing. Therefore, it is necessary to know the meaning of Standard Costs.
Standard Cost, in simple, represents the cost computed prior to the undertaking of production
activities on the basis of specification of all the influencing factors of cost. That means, in order
to compute the Standard Cost, it is necessary to find out the factors which influence the costs such
as material specification, labour requirements, requirements of other production facilities, etc. On
the basis of this, standards are to be set for each of the elements of costs and for all the elements
put together. This is to be completed in advance (i.e., before the commencement of production)
so that the personnel engaged in the production activities will be given a target to achieve. Brown
and Howard have, therefore, defined Standard Cost as ••. a pre-determined cost which
determines what each produce or service should cost under given circumstances. This
definition states the Standard Cost as representing a specified amount of expenditure within which
the product is to be manufactured. Blocker and Weltmer have defined Standard Cost as ••• a pre-
determined cost based upon engineering specifications and representing highly efficient
production for quantity standards and forecasts of future market trends for price
standards, with a fIXed amount expressed in rupees for materials, labour and overhead for
an estimated quantity of production. The Chartered Institute of Management Accountants,
London has defined Standard Cost as a pre-determined cost which is calculated from
management's standards of efficient operation and the relevant necessary expenditure.
An analysis of these definitions reveals the factors which should to be kept in mind while
fixing standards. The Standard Costs are pre-determined costs, i.e.,' the costs which are
determined before undertaking the production activities. These Standard Costs are to be
determined on the basis of product requirement and specifications. That means, it is necessary to
study what' amount of material, labour and other manufacturing facilities are required to produce
the product. Further, product features are also to be taken into account. Because, requirement of
, input factors depends upon the features which the product is expected to possess. Besides, the
physical requirements of input factors are to be determined considering the normal level of
efficiency. Once the physical quantities are determined, they are to be converted into monetary
figures by multiplying the physical quantities by the prices of respective input factors. For this
purpose, prices of various input factors prevailed in the past, current prevailing prices and a
Standard Costing: 557

forecast about future prices are to be considered. This is a very important task and it is to be
noted here that the success of Standard Costing depends greatly upon the accuracy with which the
standards are set. It is, therefore. necessary to set the standards for each element of costs very
carefully.
Types of Standards
The term 'Standard' is a relative one and it refers to a 'criterion' in the form of 'pre-
determined rate or amount' against which actual figure can be measured and compared to find out
by how much actual figure differs from the standard figure. For example, standard material cost
per unit of a product is fixed at Rs. 5. That means, the company wants to produce a unit of its
product by incurring not more than Rs. 5 of material cost. Against this standard material cost of
Rs. 5, actual material cost (say, Rs. 5.2) is compared to find out the deviation (Re. 0.2). Once the
deviations are computed, the management will analyse the variances to unearth the reasons for the
variance - whether it was due to the increase in material prices or due to fall in material
productivity. On the basis of this, suitable action will be initiated by the management. Since the
changes take place in the influencing factors, no standard is normally a permanent one. Revision,
in the light of changed environment and conditions, is to be made to make the standard more
effective and useful.
Though a number of bases are available for classifying the standards into two or more
categories, degree of attainability is the most widely used base for classifying the standards. On
the basis of degree of attainability of goals set in the standards, the standards can be classified
into two categories viz., ideal standards and real standards. Ideal Standards represent the pre-
Qetermined goals set assuming ideal conditions with highest degree of efficiency. That means,
these standards set targets which can be achieved only in ideal situation, viz, if there is no loss of
labour time, if there is no machine breakdown, if there is no power failure, if there is no loss of
material in the process of production, etc. But, in reality, it is very difficult to achieve the targets
set in the ideal standards. Since they are set without considering the reality, the company cannot
held the concerned manager responsible for not achieving the target. Therefore, the Real
Standards which are also called normal, practical or attainable standards are widely being
used in the corporate sector. These are the standards which are established considering the
nonnallevel of efficiency and also the practicality of achieving the targets. In this case, standards
are established after making provision for normal wastage and inefficiency which is normally
unavoidable. That means, standards are set on the basis of real and practical conditions prevailed .
in the company.
Objectives of Standard Costing
The important objectives of Standard Costing are as follows.
1. To exercise control over all the items of costs pertaining to production, administration,
and selling and distribution;
2. To prepare and submit the reports promptly to the managerial personnel regularly about
the progress and also how the costs to-date compare with the corresponding standards.
This is done with the objective of enabling the management to take necessary and timely
corrective measures;
Management Accounting : 558
3. To create cost-consciousness among the employees of the company;
4. To create team spirit among the human resources of the company;
5. To reap the benefit of management by exception so that the precious time and effort of
management are devoted only for the areas wherein the progress is not in accordance with
the standards, and
6. To guide the management and the staff about the possible ways for improving the
performance of the organization in future.
However, it must be noted here that the primary objective of Standard Costing is to
exercise greater control over the co~ts by setting standards and requiring the personnel to achieve
the targets set in the standards.
Difference between Standard Cost and Estimated Cost
Both the Estimated Cost and the Standard Cost have atleast one similarity. That is, both
pertain to the future period. However, a close look at these two terminologies reveals a few
important differences as identified below.
1. Standard Cost specifies clearly about what should be the cost of a product or service.
On the other hand, Estimated Cost states about what will be the cost of a product or
service;
2. Standard Cost specifies the permissible amount of expenditure to produce a unit, or a
given number of units, of a product. On the other hand, Estimated Cost states the amount
of cost that the company is likely to incur to produce a unit, or a given number of units,
of a product;
3. Standard Cost is used as a criterion to evaluate the cost economies. Because, the very
objective of setting up of standards is to ensure that the costs do not exceed certain limits.
Actuals will be compared with the standards and on the basis of the variances, necessary
action will. be taken. On the other hand, control aspect is absent in Estimated Cost.
Because, no further action will be initiated in the case of Estimated Costing. And in most
of the cases, Estimated Costs are used for preparing and submitting tenders and for price
fixation.
4. Standard Costs are normally computed on the basis of scientific and technical analysis.
However, the Estimated Costs are determined only on the basis of the past cost data and
the anticipated changes in future. .
This way, Estimated Cost differs from the Standard Cost and the differences between
these two stem mainly from the manner in which the costs are pre-determined.
Standard Costing and Budgetary Control
Both Budgetary Control and Standard Costing have a common objective. That is, in both
the systems, pre-determined targets are fixed, actual results are measured and compared with the
targets to find the deviations. This will be followed by the identification of reasons for deviations
and initiation of suitable action to correct these deficiencies. However, some differences can be
found between these two systems of Costing. They are summarized below.
Standard Costing : 559

l. The scope of Budgetary Control extends to cover the operation of a department or the
whole organization. For instance, Budgets are prepared for Production Department,
Selling and Distribution Department, etc., and also for the whole company. Therefore,
Budgetary Control is more extensive in its scope. On the other hand, Standard Costing
lays emphasis on setting standards for various elements of costs and sales with the
objective of controlling expenses. It is, therefore, more intensive;
2. Budgetary Control can be applied even to a part of the business. But this is not possible
in the case of Standard Costing;
3. Budgetary Control System can be operated without Standard Costing. But Standard
Costing cannot be introduced without a proper Budgetary Control System.
4. The scope of Standard Costing is usually confined to production activities and production
cost. On the other hand, the scope of Budgetary Control extends to cover wider area as
budgets can be, and/or are, prepared for all functional areas and also for the whole
organization.
However, both the Budgetary Control and Standard Costing systems should be introduced
to reap the full benefits of the same by the companies.
Advantages of Standard Costing
The important advantages.of Standard Costing system are enumerated below.
1. Standard 'Costing sets the standards for each of the elements of costs. Consequently, the
concerned employees will be asked to produce the commodity by incurring not more than
the costs set in the standards. Since these standards are determined before the
commencement of production, they act as the goals to be reached;
2. It helps to exercise control over the costs as the variances can regularly be ascertained and
corrective measures can be initiated at the right time;
3. It helps to evolve proper incentive schemes to the employees. Because. incentive is
necessary to encourage the employees to achieve the desired result. Since the standards
act as the yardsticks against which the actuals may be compared, suitable incentive policy
may be evolved;
4. Standard Costing provides valuable help in the form of guidance to the managerial
personnel in all spheres of their functions;
5. Since it is possible to identify the cost centres or the elements of costs in which the
progress is not in accordance with the planned, Standard Costing helps to apply
management by exception principle;
6. It helps to promote the labour efficiency and productivity;
7. Since Standard Costing provides the standard rates for valuation of period-end or year-
end inventories, distortion of profit figure by the frequent fluctuation in the volume of
output can be avoided; and
8. Since the cost centres are identified, it is possible to assign specific responsibilities to the
heads of the cost centres. As a result, identification of responsibility becomes easier.
Management Accounting : 560
This way, a number of benefits accrue to the company if it adopts proper Standard
Costing system.
Limitations of Standard Costing
In order to reap the full benefits of Standard Costing, the companies should keep the
following points in their mind. Because, these can be considered as precautions to be taken while
adopting Standard Costing system. If a company fails to take note of these, the same may act as
limitations. ' The important limitations, therefore, are as follows.
1. Since the setting up of standards require high degree of technical proficiency and skill, it
is alleged that the adoption of Standard Costing is a costly exercise. And therefore, many
of the small scale organizations may find it very difficult to adopt this system. But what
is to be noted here is that the benefits from Standard Costing are many a times higher than
the costs involved;
2. Another important difficulty is in classifying the variances into ~ontrollable and non-
controllable. Because, heads of the cost centres can be held responsible only for the
controllable variances but not for both controllable and non-controllable variances.
Because, the executives have virtually no control over the non-controllable variances.
Therefore, proper care is to be taken to classify the variances into controllable and non-
controllable; and
3. Since the standards are influenced by a number of variables and since these variables are
subject to continuous or frequent changes, it is alleged that it is very difficult to adopt
Standard Costing. .
Of course, all these allegations are true. It is also true that these limitations can be
overcome, if the management realizes the benefits of Standard Costing. What is required is a
serious and a sincere attempt to introduce and implement the system. Once the system is evolved
and introduced, the system will continue to work with minimum cost and effort generating high
results.
Analysis of Variances
When a comparison between the actual and the standard is made, some difference is
normally found. The difference between the actual and the standard is called variance. If the
actual cost is lower than the standard cost or if the actual result is better than the standard result,
the variance is called favourable variance. In the same analogy, if the actual cost exceeds the
standard cost or if the actual result is lower than the standard result, the variance is called
unfavourable or adverse variance. The favourable and unfavourable variances are also called
credit and debit variances respectively.
Standard Costing, through Variance Analysis, lays emphasis on cost control and cost
reduction. In order to exercise proper control over the costs, the management need not devote its
time and effort on all the items. Rather, it is sufficient if it concentrates on the areas wherein the
performance is not in accordance with the plan. That means, concentration of control effort is
necessary only on the points of exceptions. Any how, analysis of variance helps the management
to identify the person who is responsible for this unfavourable variance. For instance, for adverse
Standard Costing: 561

material price variance, purchase manager can be held responsible. This way, it helps to exercise
control over costs. It, therefore, involves three important steps;
1. Calculation of Variances,
2. Identification of reasons for these Variances, and
3. Disposition of Variances.
Classification of Variances
Variances can be classified into two or more groups on a number of bases such as
controllability, impact, nature, element, etc., as detailed below.
Classification of Variances
Controllable Variances
Controllability
Uncontrollable Variances

'-------'
1----. C Favourable Variances
Unfavourable Variances

C
On the
Basis of ...
1--_ _• Basic Variances
~------' Sub-Variances
Material Cost Variances
Elements
of Cost
1---....
E Labour Cost Variances
Overhead Variances

If the management is able to identify the individual who is responsible for a variance and
if reasons for the variance seem to be within the control of the individual, then it is a controllable
variance. Otherwise, it is an uncontrollable variance. That means, if it is not possible for the
management to pin-point the exact person who is responsible for the variance and if the causes
for the variance are not within the control of that person, then it is called uncontrollabl~
variance. (Meaning of favourable and unfavourable variances have already been explained).
Basic variance denotes the variance which arises due to changes in monetary rates (e.g., price of
material) and non-monetary factors (e.g., physical units in quantity). This basic variance due to
non-monetary factors will be analysed into sub-variances considering the factors responsible for
that variance (e.g., Material Usage Variance is sub-divided into Material Mix Variance and
Material Yield Variance).
Of the four bases, element-wise classification of cost variances is both popular and
useful. On the basis of the elements of costs, cost variances can be classified into a number of
groups as presented below.
Management Accounting : 562
Element-wise Classification of Variances

Material Cost
r--t Variances

Cost Labour Cost


Variances r----- Variances

Variances ~ Overhead Cost


Variances

Sales
Variances

Material Cost Variance (MCV)


A diagrammatic view or-Material Cost Variance and its sub-variances is presented below
for easy and clear understanding.
Material Cost Variances

Material Price
----+ Variance

Material Cost
Material Mix
Variances ----+ Variance

----+ Material Usage ----+


Variance
Material Yield
~
Variance

Material Cost Variance (MCV) represents the difference between the standard cost of
materials that should have been incurred in producing the actual output and the actual cost of
materials used. Hence,
Material Cost Variance =[Standard Material Cost - Actual Material Cost]

~
Mate~al CosQ
f -_ ~Stan~ard Standard] Actual
Vanance QuantIty of x Pr· - Quantity of ActUal]
.al Ice x Price
Maten s Material
MCV =[(SQ x AO) x SP] - [(AQ x AO) x AP] =AO [(SQ x SP) - (AQ x AP»
andard Costing: 563
i.

where, SQ =Standard usage of material per unit of output


AQ =Actual usage of material per unit of output
AO =Actual output in units
SP = Standard price of material per unit
AP = Actual price of material per unit
If the actual material cost is lower than the standard material cost, the variance will be
favourable. If reverse is the case (Le., if actual material cost exceeds the standard material cost),
the variance is adverse or unfavourable. The actual material cost will be lower (a) if the actual
unit price of material is lower than the standard price and/or (b) if the actual quantity of material
used per unit of output is lower than the standard quantity. This lower actual material cost than
the standard material cost results in favourable variance. On the other hand, (a) if the actual unit
price of material is higher than the standard price and/or (b) if the actual quantity of material used
per unit of output is more than the standard quantity, the actual material cost will be higher than
the standard material cost reSUlting in adverse variance.
It is obvious from the above that the Material Cost Variance depends upon two important
variables, viz., the price paid and quantity of materials used in the actual production. Therefore,
Material Cost Variance is sub-divided into two, viz, Material Price Variance (MPV) and Material .
Usage or Quantity Variance (MUV). That means,
Mate~al Costl = [Mate~al Price] (Material Usagel
Vanance f Vanance +l Variance J
MCV =[MPV + MUV]
Material Price Variance (MPV)
Material Price Variance is a part of Material Cost Variance and this arises due to the
difference between the actual price paid for the materials purchased and the standard price
specified. It, therefore, depends upon two important factors - one, the difference between the
standard price and the actual price per unit of material, and two, the actual number of units of
materials purchased. Therefore,

Material } Standard Price for] Actual Price for ]


Price = actual quantity of - actual quantity of
Variance [ material purchased [ material purchased

MPV =[(SP x AQP) - (AP x AQP)] =AQP (SP - AP)


where AQP =Actual quantity of material purchased:
It may be noted here that this variance is computed assuming that the quantity of material
purchased is equivalent to the quantity of material consumed. If the actual price of material is
lower than the standard price, the variance will be favourable. Otherwise (Le., if AP > SP), the
variance will be adverse.
Management Accounting 564

Material Usage or Quantity Variance (MUV)


Another portion of Material Cost Variance is the Material Usage Variance (MUV). This
variance sheds light on how well materials are used in the production. It arises due to the
difference between standard. quantity of materials specified for actual production and the actual
quantity of materials consumed. Since the Material Price Variance takes care of price differences,
the same is ignored while computing the Material Usage Variance. Therefore, Material Usage
Variance is computed by multiplying the difference in quantities (i.e., between standard quantity
specified and actual quantity used) by the standard price per unit. Therefore,

Material} Ltandard Quantity of Actual Quantity Of]


Usage =. material for actual - material for actual x
Variance production production

MUV =[(SQ x AO) - (AQ x AO)] x SP =[AO (SQ - AQ)] x SP


If, AQ < SQ, the variance will be favourable. Otherwise (i.e., if AQ > SQ), the variance
will be adverse.
IDustration: 7.1
A manufacturing concern which has adopted Standard Costing furnishes the following
information.
Standard: Material for 70 kgs of finished products 100 kgs
Price of material Re. 1 per kg
Actual: Output 2,10,000 kgs
Material used 2,80,000 kgs
Cost of material Rs. 2,52,000
Calculate: (a) the Material Usage Variance, (b) the Material Price Variance, and (c) ·the
Material Cost Variance. lCA (Int), 1975J
Solution:
Standru;d Price (SP) =Re. 1 per kg
Actual Price (AP) =(Rs. 2,52,000 + 2,80,000 kgs) =Re. 0.9

. rt
00 kgs of Materiall
Standard Quantity (SQ) = L70 kgs of Output J =(10/7) kgs per umt. of output
. (2,80,000 kgs of Materiall
Actual Quantity (AQ) = 2,10,000 kgs of Output J =(4/3) kgs per umt. of output
Actual Quantity of Material Purchased (AQP) = 2,80,000 kgs
Actual Output (AO) =2,10,000 kgs
Standard Costing : 565

Material Cost } = rstandard Q~antity Stan.dard!_ [ActUal Qu~ntity


Variance, (MCV) 0 l
of Matenal x Price Jof Matenal x
Actual ]
Price

= [(SQ x AO) x SP] - [(AQ x AO) x AP]


=AO [(SQ x SP) - (AQ x AP)]

=2,10,000 kgs ((~O kgs x Re. 1) - (j kgs x Re. 0.9 ) J


= 2,10,000 kgs (Rs. ~O - Rs. 1.2 ) = 2,10,000 kgs pO ~ 8.4 )
= (2,10,000 kgs x RS 1.6 ) =Rs. 48,000 (F)
7
Material Price } Standard Price forJ ,[ Actual Price for oJ
actual quantity of - actual quantity of
Variance, (MPV) = [ material purchased material purchased

=[(SP x AQP) - (AP x AQP)] =AQP (SP - AP)


= 2,80,000 kgs (Re.l- Re. 0.9) = (2,80,000 kgs x Re. 0.1)
= Rs. 28,000 (F)

Material Usage 1 _ ~tand~d Quantity of Actual Quantity Of]


Variance, (MUV) J - matenal fo~ actual - material for actual x
production production

= [(SQ x AO) - (AQ x AO)] x SP = [AO (SQ - AQ) x SP]

=2,10,000 kgs (~O kgs - j kgs) x Re. 1


= (2,10,000 kgs x (30;; 28 ) J x Re. 1

= (2,10,000 kgs x ..L


21
J x Re. 1 = 20,000 kgs x Re. 1

=Rs. 20,000 (F)


Verification: MCV =MPV + MUV
Rs. 48,000 (F) =Rs. 28,000 (F) + Rs. 20,000 (F) =Rs. 48,000 (F)
Management Accounting : 566
The Material Usage Variance can be further sub-divided into Material Mix Variance and
Material Yield Variance.
Material Mix Variance (MMV)
A product may require the use of more than one material for its production. A
combination of materials used in the process of producing a product is called material mix. In
this type of situation, one can find some difference between the actual mix of materials used and
the pre-<ietermined standard mix (ratio). This is due to a number of reasons including non-
availability, or inadequate supply, of one or more types of materials; availability of a new
substitute material for one or more types of materials currently being used; availability of similar
type of material for lower price; etc. Material Mix Variance has, therefore, been defined by
Chartered Institute of Management Accountants, England as the difference between standard
and the actual composition of mixture. This variance arises only when two or more types of
material are required to be consumed to produce a commodity. As the procedure of computing
the Material Mix Variance differs from one situation to another depending upon whether the
actual weight of mix differs from the standard weight of mix or not, the same is discussed under
two heads viz., (a) when the actual weight of mix does not differ from the standard weight of mix,
and (b) when the (ratio of mix and) actual weight of mix differs from the standard weight of mix.
(a) When the Actual Weight of Mix does not differ from Standard Weight of Mix
Material Mix} (Standard
Variance l
= Quantity -
Actual
Quantity
J (l
x
Standard
Unit Price
J
MMV = [(SQ - AQ) x SP]
This formula can be used even when the standard is revised due to shortage of a specific
type of material. But the actual weight of mix should be equal to the standard weight of mix. If
the actual quantity is lower than the standard quantity, the variance is favourable. Otherwise [Le.,
if actual quantity is higher than the standard quantity], the variance will be adverse.
mustration: 7.2
From the following, compute Material Mix Variance.
Standard Mix Actual Mix
Material Quantity Rate Amount Quantity Rate Amount
(kgs) (Rs.) (Rs.) (kgs) (Rs.) • (Rs.)
p 225 10 2,250 275 10 2,750
Q 275 12 3,300 225 12 2,700
500 5,550 500 5,450

Solution:
Mate~al MiX} _ (Stand~d
Vanance - Quantity
Actual
Quantity
J[x
Standard
Unit Price
J
MMV= (SQ-AQ) x SP
Standard Costing: 567

For Material P, MMV = (225 kgs - 275 kgs) x Rs. 10 = (- 50 kgs x Rs. 10) = Rs. 500 (A)
Q, MMV = (275 kgs - 225 kgs) x Rs. 12 = ( 50 kgs x Rs. 12) = 600 (F)
:. Total MMV Rs. 100 (F)
(b) When the (Ratio of Mix) and Actual Weight of Mix differs from the Standard
Weight of Mix, Revised Standard Quantity (RSQ) for each material is to be computed
. by using,

Revised Standard 1_ Total Weight of Actual Mix


x
Standard Quantity
Quantity, (RSQ) f- Total Weight of Standard Mix of a Material

Based on this, the Material Mix Variance is calculated by altering the formula slightly as
shown below.

Mate~al MiX} = [:ev~~ _ Actu~J


Vanance Qtan ~
(St~d~d J
QuantIty x Umt Price
uantIty

MMV = [(RSQ - AQ) x SP]


Using this formula, MMV is computed for each type of raw material. If actual quantity is
lower than revised standard quantity (RSQ), the variance is favourable. Otherwise (Le., if AQ >
RSQ), it is adverse.
IDustration: 7.3
From the information presented below, compute the Material Mix Variance.
Standard Mix Actual Mix
Material Quantity Rate Amount Quantity Rate Amount
(kgs) (Rs.) (Rs.) (kgs) (Rs.) (Rs.)
M 250 10 2,500 200 10 2,000
N 150 15 2,250 225 15 3,375
400 4,750 425 5,375

Solution:
Revise~ Standardl=
Quantity, (RSQ~
r Total Weight of Actual Mix
[TOtal Weight of Standard Mix
x Standard Quantity]
of a Material
Management Accounting : 568
Therefore, RSQ of Material:
M = [425 x 250 1= 265.625 kgs
. L400)

N ;:: (:~~ x 150J = 159.375 kgs


425 kgs

Mate~al MiX} = [
Vanance
Revised Actual) [ Standard
Standard - Qua~ti) x Unit Price
J
Quantity
MMV of Material:
M = [(265.625 kgs - 200 kgs) x Rs. 10] = (65.625 kgs x Rs. 10) = Rs. 656.250 (F)
N = [(159.375 kgs - 225 kgs) x Rs. 15] = (65.625 kgs x Rs. 15) = 984.375 (A)
:. Total Material Mix Variance = 328.125 (A)
Material Yield Variance (MYV)
This is another sub-variance of Material Usage Variance. As is known, loss in the process of
producing a commodity is inevitable. Considering this normal loss, standard mix of material is fixed
for normal expected yield or output. Further, actual output normally differs from the standard output
and this necessitates the calculation of Yield Variance. The calculation procedure depends upon
whether the standard mix differs from actual mix or not Therefore, the procedure of computing the
Material Yield Variance differs from one situation to another as discussed below.
(a) When the Actual Mix does not differ from the Standard Mix, Material Yield Variance can
be computed by using the following formula.

Material Yield
Variance, (MYVU
l =[A~tual _ Standar~
.YIeld Yield j x
[ Standard Material
Cost per unit
J
where,
Actual Yield =Actual Output
Standard Yield =Standard Output for Actual Mix
Standard Materiall _ [Total Cost of Standard Mix at Standard Pricel
Cost per unit f - Net Standard Output )
Standard Costing : 569

mustration: 7.4
From the following information, compute the Material Yield Variance.
Standard Mix Actual Mix
Material Quantity Rate Amount Quantity Rate Amount
(kgs) (Rs.) (Rs.) (kgs) (Rs.) (Rs.)
p
Q
. 60
40
10
20
600
800
56
44
10
20
560
880
100 1,400 100 1,440
Less: Loss 30 (30%) - 25 (25%) -
70 1,400 75 1,440
Solution:
0ta! Cost of Standard]
Standard Materiall_
Cost per unit J-
L
Mix at Standard Price
Net Standard Output

Material YieldL _ [Actual _ Standard I


_ rR~. 1,40~
- l70 kgs j

[Standard Material
_ R 20
- s.

J
Variance J- Yield Yield) x Cost per unit

=[(75 kgs - 70 kgs) x Rs. 20] =(5 kgs x Rs. 20) =Rs. 100 (F)
(b) Even when the Actual Mix differs from the Standard Mix, the same formula (as given
above) may be used to compute Material Yield Variance. However, Revised Standard Mix is
to be computed as the weight of actual mix differs from the weight of standard mix. Standard
material cost per unit is to be computed on the basis of the Revised Standard Mix.
mustration: 7.S
Standard Mix Actual Mix
Material Quantity Rate Amount Quantity Rate Amount
(kgs) (Rs.) (Rs.) (kgs) (Rs.) (Rs.)
P 60 10 600 80 10 800
Q 40 20 800 70 II 20 1,400
100 , 1,400 150 2,200
Less: Loss 30(30%) - 37.5 (25%) -
70 1,400 112.5 2,200
From the above, compute the Matenal YIeld Vanance.
Management Accounting : 570
Solution:

Total Weight of Actual Mix Standard Quantity


Revised Standard}
Quantity, (RSQ) = -------------------- x of a Material
Total Weight of Standard Mix

Computation of RSQ and Standard Material Cost


RSQ Rate Amount
Material
(kgs) (Rs.) (Rs.)
p = [(150 + 100) x 60] = 90 10 900
p= [(150 + 100) x 40 = 60 20 1,200
150 2,100
Less: Loss (30%) 45 -
105 2,100

Standard Material} _ fRs. 2,100)


Cost per unit -l105 kgsj = Rs. 20

Material Yield} _ [Actual


Variance - Yield-
ReViSed~
Standard
Yield
x
[
Standard
Cost per umt
Mat~al J
, = '[(.112.5 kgs - 105 kgs) x Rs. 20]
=(7.5 kgs x Rs. 20) =Rs. 150 (F)
Standard Costing : 571

A summary of Material Cost Variances is presented below.


Material Cost Variances

Mat.erial Price 1.. = [AQP (SP _ AP)] When the Actual Weight of Mix does
Vanance, MPVJ not differ from Standard Weight of Mix,
MMV =[(SQ - AQ) x SP]
Material Mix
Variance, MMV
Material Cost Variance, MCV =
. AO [(SQ x SP) - (AQ x AP)] When the (Ratio of Mix and) Actual

1 Weight of Mix differ from the Standard


Weight of Mix, RSQI is computed. Then,
-
MMV - [(RSQ - AQ) x SP]
Material Usage or Quantity Variance,
MUV =[AO (SQ - AQ)] x SP
I When the Actual Weight of Mix does not
differ from the Standard Weight of Mix,
r--I MYV = [(A Y - SY) x Standard Material
Cost per unit]
Material Yield
Variance, MYV
~

When Weight of Actual mix differs from


~ the Weight of Standard Mix, Revised
Standard Mix I is to be computed.
Standard Material Cost per unir is to be
computed based on Revised Standard Mix.
Then, MYV = [(AY - RSy) x Standard
Material Cost per unit of Output]

Note:

Total Weight of Actual Mix Standard Quantity


1. Revised Standard} _ x
Quantity, (RSQ) - of a Material
Total Weight of Standard Mix

2. Standard Material} _
Cost per unit -
~eVi;:~~~~J MiX]
at Standard Price
Net Standard Output
mustration: 7.6
The standard material cost for a normal mix of one tonne of Chemical P is based on:
Chemical A B C
Usage (kg) 240 400 640
Price per kg (Rs.) 6 12 c 10
Management Accounting : 572
.During a month, 6.25 tonnes of P were produced from:
Cherrricru A B C
Consumption (tonnes) 1.6 2.4 4.5
Cost (Rs.) 11,200 30,000 47,250
Cruculate the Materiru Variances. lCA (Fin), November 1978J
Solution:
Standard Mix Acturu Mix
A 1,500 kgs @ Rs. 6 = 9,000 1,600 kgs Rs.11,200 (@ Rs.7)
B 2,500 kgs @ Rs. 12 = 30,000 2,400 kgs Rs.30,000 (@ Rs. 12.5)
C 4,000 kgs @ Rs. 10 = 40,000 4,500 kgs Rs.47,250 (@ Rs. 10.5)
- -
8,000 79,000
--
8,500 88,450

Materiru cost} ( Standard Actual Materiru] = [Rs. 79,000 - Rs. 88,450]


Variance, MCV =-lMateriru Cost Cost
= Rs. 9,450 (A)

Materiru Price} A.cturu Qu~tity rtandard _ ActuruJ


·
Vanance, MPV = of Matenal Pri Pn'ce
Purchased ce

:. MPVof A = 1,600 kgs (Rs. 6 - Rs. 7) = [1,600 kgs x Re. 1] = Rs. 1,600 (A)
B = 2,400 kgs (Rs. 12 - Rs. 12.5) = [2,400 kgs x Re. 0.5] = Rs. 1,200 (A)
C = 4,500 kgs (Rs. 10 - Rs. 10.5) = [4,500 kgs x Re. 0.5] = Rs. 2,250 (A)
Rs. 5,050 (A)

Materiru Usa el
Variance, MJv = f
Ltandard Quantity of
Materiru for Acturu -
Acturu Quantity o~.
Materiru for.Acturu x
Production Production

:. MUV of A = [(1,500 kgs - 1,600 kgs),?< Rs. 6] = (100 kgs x Rs. 6) = Rs. 600 (A)
B = [(2,500 kgs - 2,400 kgs) x Rs. 12] = (100 kgs x Rs. 12) = Rs. 1,200 (F)
C = [(4,000 kgs - 4,500 kgs) x Rs. 10] = (500 kgs x Rs. 10) = Rs. 5,000 (A)
Rs. 4,400 (A)
Standard Costing : 573

Material MiX} _ [:eVised _


Variance - tand~d Quantity
ActUal~
x
( Standard
Unit Price
J
Quantity ,

~(8,500
kgs x 1,500 kgs - 1,600 kgS
... MMV of A -- ~ 8,000 kgs J l
j x Rs. 6

=[(1,593.75 kgs - 1,600 kgs) x Rs. 6] =(6.25 kgs x Rs. 6) = Rs. 37.5 (A)

~(8,500
B -- ~ 8,000 kgs J
kgs x 2,500 kgs - 2,400 kgS l x Rs. 12
j
=[(2,656.25 kgs - 2,400 kgs) x Rs. 12] =(256.25 kgs x Rs. 12) =Rs. 3,075 (F)
~(8,500
kgs x 4,000 kgs - 4,500 kgS
C = ~ 8,000 kgs J l
j
x Rs. 10

=[(4,250 kgs - 4,500 kgs) x Rs. 10] =(250 kgs x Rs. 10) = Rs. 2,500 (A)
Rs. 537.5 (F)

Material YieldL = [A~tual _ Standar~ [ Standard Material ]


Variance J Yleld Yield j x Cost per unit

1,000 kgs
79'000~
=. 6,250 kgs - of output x 8,500kgs
1,280 kgs of
standard
input
of input x
tRS.
6,250 kgs of
output

=[(6,250 kgs - 6,640.625 kgs) x Rs. 12.64] =[390.625 kgs x Rs. 12.64] = Rs. 4,937.5 (A)

Verification:
1. MCV = MPV + MUV
Rs. 9,450 (A) = Rs. 5,050 (A) + Rs. 4,400 (A) = Rs. 9,450 (A)
2. MUV = MMV + MYV
Rs. 4,400 (A) = Rs. 537.5 (F) + Rs. 4.937.5 (A) = Rs. 4,400 (A)
Management Accounting : 574
Labour Cost (or Wages) Variance
The figure presented below gives a complete idea about the Labour Cost Variance and its
sub-variances.
Labour Cost Variances

_ .
r
Labour Rate
Variance
.
r
Labour Mix
Variance

Labour
Labour Cost
Variance
.. Efficiency
Variance
..
r Labour Yield
Variance
Idle Time
Variance

The Labour Cost Variance (LCV) which is also known as Labour Variance or Wages
Variance represents the difference between the standard labour costs specified for the output or
activity achieved and the actual labour cost of the period.

Labour Costl (Standard Labour cost] ( Actual ]


Variance f
= for the Actual Output -lLabour Cost
, _ (Standard Time for Standardl (Actual Time for Actual ]
-l Actual Output x Rate J- Actual Output x Rate

Actual
Standard
abour Hours Actual} Standard Labour Hours Actual
= er unit of x Ou~ut x Wage - Worked per x x Wage
{ p Output (umts) R~~!er unit of Rate per
Output Hour

=[(SH x AO) x SR] - [(AH x AO) x AR] =AO [(SH x SR) - (AH x AR)]
where, AH = Actual labour hours worked per unit,
AR =Actual wage rate per hour
SH = Standard labour hours per unit of output, and
=
SR Standard wage rate per hour.
Standard Costing : 575

If the actual labour cost is lower than the standard labour cost, the variance is favourable.
On the other hand, if the actual labour cost is higher than the standard labour cost, the variance
will be adverse.
D1ustration: 7.7
From the following details, determine the Labour Cost Variance. Actual number of units
produced during 2005 = 2,500.
Standard Actual
Number of labour hours per unit 8 9
Wage rate per hour (Rs.) 5 6
Solution:

La~our cost = AO [(SH x SR) - (AH x AR)]


1
Vanance, LCVJ
= 2,500 units [(8 hours x Rs. 5) - (9 hours x Rs. 6)
= [2,500 units (Rs. 40 - Rs. 54)] = [2,500 units x Rs. 14] = Rs. 35,000 (A)
The Labour Cost Variance can be sub-divided into three, viz, Labour Rate Variance,
Labour Efficiency Variance and Idle Time variance.
Labour (or Wage) Rate Variance (LRV)
This is a part of Labour Cost Variance and it arises due to the difference between the
actual wage rate and standard wage rate. As in the case of Material Price Variance, this variance
is computed by considering the difference between the standard and the actual wage rates, and the
actual number of hours worked in a period. Therefore,

Labour Cost for actual Actual Labour Cost for


number of hours actual number of hours
LabOur}
Rate = worked at standard worked at the actual
Variance wage rate rate

= ~ruw
Hours x
Standard ~
WageRate -
[ Actuw
Hours x Wage Rate
Actuw ]
orked per Hour Worked per Hour
Actual

[ ~m} S~J
Actual
Actual Labour
Wage
= Output
x Hours x Wage - x
Rate
(units) Worked Rate per
per unit per Hour
Hour

=[(AO x AH x SR) - (AO x AH x AR)] =[(AO x AH) x (SR - AR)]


Management Accounting : 576

~J
=
t Total Number of

for Actual Output


cStandard
Labour Hours Worked x Wage Rate -
per Hour
Actual
Wage Rate
per Hour

If the actual hourly wage rate is lower than the standard rate, the variance is favourable.
Otherwise (Le., if the actual hourly wage rage is higher than the standard hourly wage rate), the
variance is adverse.
Dlustration: 7.8
You are required to compute the Labour Rate Variance from the information presented
below. Actual output: 200 units.
Standard Actual
Number of hours per unit of output 2 2.5
Wage rate per hour (Rs) 5 6

r
Solution:

~..:::} = m:~t
Actual
tandard
AcruruJ]
x Hours
J
per unit
x
Rate - Rate

=[(200 units x 2.5 hours per unit) x (Rs. 5 - Rs. 6)]


=[500 hours x Re. 1] = Rs. 500 (A)
Labour Efficiency (Time) Variance (LEV)
Labour Efficiency Variance is similar to Material Usage Variance. It is a portion of
Labour cost Variance and it is due to the difference between the standard labour hours specified
for the actual output or activity achieved and the actual labour hours worked or expended.
Chartered Institute of Management Accountants, London defines Labour Efficiency Variance as
that portion of labour cost variance which is due to the difference between standard labour
hours specified for the activity achieved and the actual hours expended. As in the case of
Material Usage Variance, here also only the standard wage rate is considered. Therefore,
Labo~r Efficiencyl = [Standard Time for _ Actual Time for] x (Standard]
Vanance, LEV f Actual Output Actual Output Rate

= [f~~:~
l(units)
Standard t
{Actual
x Labour Hours - Output
per unit (units)
x
Actual
Labour H~urs
perumt
Standardj
~
~ x Wage Rate
per Hour

=[(AO x SH) - (AO x AH)] x SR =[(SR x AO) x (SH - AH)]


Standard Costing : 577

If the actual labour hours worked are lower than the standard labour hours, the variance is
said to be favourable. On the other hand, the variance is said to be adverse if the actual number of
labour hours worked are higher than the standard labour hours. At this stage, it is necessary to
note that 'actual labour hours' used in the above fonnula should be the actual number of labour
hours used for production. That means, it should not include the idle time forced by factors such
as strikes, lockouts. machine break-down. power shortage, etc.
mustration: 7.9
From the following details, compute the Labour Efficiency Variance.
Standard Actual
Wage Rate per Hour (Rs.) 1.50 1.75
Number of Labour Hours per unit 4.00 3.75
Actual Output: 200 units.

Solution:
LabOur} Standard Actual] [Standard Actual ~
Efficiency = [
Wage Rate x Output x Hours per - Hours per Unit
Variance per Hour (units) Unit
=[(Rs. 1.5 x 200 units) x (4 hours - 3.75 hours)]
=(Rs. 300 x 0.25 hour) =Rs. 75 (F)
Labour Mix (Gang Composition) Variance (LMV)
This variance is similar to Material Mix Variance and it arises due to the deviation in the
grade of labour employed from the standard labour mix. When the actual composition of labour
force is not in accordance with the standard mix, this variance arises. The impact of this change on
the labour cost is measured by finding out the Labour Mix Variance. The computational procedure
for this variance is similar to Material Mix Variance which is evident from the following.
a. When both the Total Standard Labour Hours (or Weeks) and Total Actual Labour
Hours are same but the Standard Mix Ratio differs from the Actual Mix Ratio, the
Labour Mix Variance is computed by using the following fonnula.

Ubou, }
Mix =
{S~ooro
Hours for
Acturu
Hours
} Standard
Wage Rate
- for Actual x
Variance Actual
per Hour
Output Output

wru [StMd~dj
~x
{Acwru
[t S~dard} Actual
= Output x Hours - Output x Hours Wage Rate
(units) per unit (units) per unit per Hour

=[(AO x SH) - (AO x AH)] x SR =[(AO x SR) x (SH - AH)]


Management Accounting : 578
b. When the Total Standard Labour Hours differ from the Total Actual Hours and also
when the Standard Mix Ratio differs from the Actual Mix Ratio, Labour Mix Variance
can be calculated by calculating Revised Standard Mix of Actual Labour Hours worked and
using the following formula. It may be not€?d here that this variance is computed for each
grade of labour.

~
Revised}
Standard =
Total Number of
Hours of Actual Mix Standard HOur~
of a particular
Total Number of Hours x Grade of Labour
Time, RST
of Standard Mix

Labour Mix
Variance, LMV }
=
[
Revised Standard
Mix or Time of
Actual Hours
Actual Mix or
Time of Actual
Hours Worked
Jx
Standard
Wage Rate
[ per Hour
J
Worked
= [RST - (AO x AH)] x [SR]
Labour Yield Variance (LYV)
This variance is similar to Material Yield Variance. It is computed on the basis of the
increase or decrease in the actual yield or output when compared to the relative standard. The
formula for computing Labour Yield Variance is as follows.

. Standard
. Yield ActuaI } Standard
Labour Yield} = 10 umts expected _ Yield x Labour Cost
Variance { from the Actual (. ) per Unit
umts
Hours Worked
Idle Time Variance (lTV)
Due to the non-availability of raw materials, break-down of machines, failure of power,
strikes, etc., the actual number of hours worked may be lower than the standard hours fixed or
paid hours. The Idle Time Variance is, therefore, segregated from the Labour Efficiency Variance
and this is always unfavourable or adverse variance. This variance is determined by using the
following formula.

IdleTime} = Abnormal Idle x Standard wage]


Variance ( Time in Hours Rate per Hour
Illustration: 7.10
The Wye Company Ltd., manufactures a particular product, the standard direct labour
cost of which is Rs. 120 and which is arrived as follows. .
r
Standard Costing : 579

Grade of Workers Hours per unit of Output Rate per Hour (Rs.) Amount (Rs.)
A 30 2 60
B 20 3 60
50 120

In a particular period, 100 units of the product were produced, the actual cost of which
was as follows.
Grade of Workers Hours Rate (Rs.) Amount (Rs.)
.,
A 3,200 1.50 4,800 .
B 1,900 4.00 7,600
5,100 12,400

You are required to calculate, (1) Total Labour Cost Variance, (2) Labour Rate Variance,
(3) Labour Mix Variance, and (4) Labour Efficiency Variance.
1M. Com., Madurai Kamraj Uni., 1983J
Solution:
Labour Cost Variance, LCV =

t
Standard Actual ActuaI Actual
Standard Actual}
bour Hours 0 x Wage Labour Hours x Output x Wage
. x utput
per umt of (.) Rate per Worked per . Rate per
Output umts . (umts)
Hour umt of Output Hour

LCV of A = [(30 hours x 100 units) x Rs.2] - [3,200 hours) x Rs. 1.5]
= [(3,000 hours x Rs. 2) - (3,200 hours x Rs. 1.5)]
= (Rs. 6,000 - Rs. 4,800) = Rs. 1,200 (F)
B = [(20 hours x 100 units) x Rs. 3] - [(1,900 hours) x Rs.4]
=[(2,000 hours x Rs.3) - (1,900 hours x Rs. 4)]
=(Rs. 6,000 - Rs. 7,600) = Rs. 1,600 (A)
:. Total LCA Rs. 400 (A)

~
TOtal Number of Standard Actual ~J
Labour Rate = Labour Hours x Wage Rate - Wage Rate
Variance, LRV} Worked for Actual
Output
C
per Hour per Hour
/ ~'I'--'
I I

Management Accounting' : :580

LRVof A =[3,200 hours x (Rs. 2 - Rs. 1.5)] =(3,200 hours x Re. 0.5) = Rs. 1,600 (F)
B =[1,900 hours x (Rs. 3 - Rs. 4)] =(1,900 hours x Re. 1) = Rs. 1,900 (A)
:. Total LRV Rs. 300 (A)

E~~~:CY} = [{~~;~ La~~:d;:urJJ {~~:~


Vanance, LEV (umts)
x
. per umt
-
(units)
x Lab~~tu~oursrD ~:d:a~J
per umt
x
Ger HOur)
LEV of A =[(100 units x 30 hours) - (3,200 hours)] x [Rs. 2]
=[(3,000 hours - 3,200 hours)] x Rs. 2] =(200 hours x Rs. 2) = Rs. 400 (A)
B =[(100 units x 20 hours) - (1,900 hours)] x [Rs. 3]
=[(2,000 hours - 1,900 hours) x Rs.3] =(100 hours x Rs. 3) = Rs. 300 (F)
:. Total LEV Rs. 100 (A)

Total Number of Standard Hours


ReVised} Hours of Actual Mix
Standard = x of a Particular
Total Number of Hours Grade of Labour
Time, RST
of Standard Mix

RST of A = ( 5,100 hours


(50 hours x 100 units)
x (30 hours x 100 units) J
5,100 hours
=( 5,000 hours x 3,000 hours
J= 3,060 hours

RST f
o B
-( 5,100 hours h .
- (50 hours x 100 units) x (20 ours x 100 umts)
J
=( 5,100 hours
5,000 hours x 2,000 hours
J= 2,040 hours
5,100 hours

L~bourMix } =
Vanance, LMV
[R~~~~~ ~~:::rd
Actual Hours
_ ~ctual Mix or J
Time of Actual
x [~~;~a~eJ
Per Hour
Worked Hours Worked
LMVof A =[(3,060 hours - 3,200 hours) x Rs.2] =(140 hours x Rs. 2) = Rs. 280 (A)
B =[(2,040 hours -1,900 hours) x Rs. 3] =(140 hours x Rs. 3)::: Rs. 420 (F)
:. Total LMV Rs. 140 (F)
A summary of Labour Cost Variances is presented in the next page.
Labour Cost Variances
Labour Rate
Variance, LRV = When both the Standard Labour Hours (or
[(AH x AO) x (SR Weeks) and the Total Actual Labour
-AR)] Hours are same but the Standard Mix
Ratio differs from Actual Mix Ratio:

Labour Cost
LMV = rS:C~~r
loutput
_ ~~::~~ ~:~~J
outputj
x
lper HOW]
Variance, LCV =
AO [(SH x SR) - When both the Standard Labour Hours
(AHxAR)] and Standard Mix Ratio differ from the ,..,.
(J)
III
:::J
Total Actual Labour Hours and Actual Co
III
Labour Efficiency Mix Ratio respectively, a.
Variance, LEV = n
o
[(SR x AO) x Revised Actual Mix [Standard] ,..,.
III
:;-
(SH - AH)] Standard of Actual Wage IC
LMV= Mix of Actual - Hours x Rate
VI
Labour Hours Worked per Hour
Worked ....00

LYV=
.
10
. exYield
Standard
urnts te
d
from the =ual -
A ctu
.
Yl~ld
~ ~ Standard
x Labour ~ost
J
[ (urnts) per urnt
Hours Worked

Idle Time Variance, lTV =


Abnormal Idle
( Time in Hours
x Standard J
Wage Rate per Hour
Management Accounting : 582
Overhead Variances
As is known very well, overhead expenses include indirect material cost, indirect labour .
cost and other indirect expenses. These expenses pertain to all the three major functions of the
company viz., production, administration, and selling and distribution. Overhead variances
therefore, relate to production overhead expenses, administration overhead expenses, and selling
and distribution overhead expenses. It may be noted here that these expenses include both
variable and fixed elements. For the purpose of computing overhead variances, overhead
expenses are classified into variable and fixed overhead expenses on the basis of their behaviour
or response to the changes in the levels of activity. Overhead Variance and its Sub-variances are
presented below.
Classification of Overhead Variances
Variable Overhead
Expenl
d'ture Vanance
.
Variable
r--t Overhead ~

Variance
~ Variable Overhead
Efficiency Variance
Overhead Cost
~
Variance
Fixed
Overhead
~
Expenditure Fixed Overhead
Fixed Variance ~ Efficiency Variance
4 Overhead Cost 1--1
Variance
Fixed Fixed
Overhead Overhead
4
Volume
f-t Capacity
Variance Variance

Fixed Overhead
---- Calendar Variance
Overhead Cost Variance or Overhead Expenses Variance or simply, Overhead Variance
represents the difference between the overhead expenses recovered (Le., at standard rate for actual
output) and the actual overhead expenses incurred. Therefore,

Overh~ad Costl = (
Vanance f
Overhead Expenses
Recovered
Actual Overhead Expenses
Incurred
J
I

= 0
Actual
( utpu
t X
Standard
Overhead
Recovery Rate
J -
(
Actual Over-
head Expenses
Incurred
J
Standard Costing : 583

Here, overhead expenses recovered is computed by applying standard overhead recovery


rate (SR) to the actual output (AO). Therefore,

Standard overhead}
Recovery R~te (SR) =
[ Standard (Budgeted) Overheads
Standard Output (units)
J
per umt

Standard Overhead Recovery Rate may also be computed on the basis of hours.
Therefore,

Standard overhead}
Recovery Rate =
[ Standard (Budgeted) Overheads
Standard Hours
J
(per hour)
If the Standard Recovery Rate based on hours is used, the number of hours for actual
output at the standard hours per unit is to be computed. This is known as standard hours for
actual output and it is calculated as follows.
Standard Hours fOr} _
Actual Output
r.
Standard Hours
- lStandard Output
J x
[ActUal Output)
(in units) J
mustration: 7.11
From the following information pertaining to the operation of P Company for the year
2004-05, compute the Overhead Cost Variance.
Standard Actual
Labour hours 1,000 1,200
Output (units) 500 400
Overhead expenses (Rs.): Variable 200 300
Fixed 300 300
Solution:
Standard Overheadl _ [Total Standard Overhead~ _ rRs. 200 (V) + Rs. 300 (F)") _ R 1
Recovery Rate J- L Standard Output J-l
500 units j - e.

Overhead Cost
Variance }=
Actual
Output
[ (units)
x
Standard
Overhead
Recovery Rate
J -
[Actual Overhead
Expenses
Incurred
J
= (400 units x Re. 1) - (Rs. 300 V + Rs. 300 F)
=(Rs. 400 - Rs. 600) =Rs. 200 (A)

Alternatively, Standard }
Overhead Standard over-J
head Expenses RS. 200 V +.Rs. 300 FJ _ R 05
Recovery Rate
(based on Hours)
-
bStandard Hours
= ( 1,000 hours - e. .
Management Accounting : 584

~
Standard Hours for Actual ] [Actual overhead]
Overhead }
Cost Variance = Overhead Recovery x Output - Expenses
Rate per Hour at Standard Rate ' Incurred

r.
=leo 0.5 x {1.000 hours .
500 units x 400 umts
}UU- k~s. 300 V + Rs. 300 ~~

=[(Re. 0.5 x 800 units) - (Rs. 600)] =(Rs. 400 - Rs. 600) =Rs. 200 (A)
As is known very well. if the actual overhead expenses incurred is lower than the,
overhead expenses recovered (i.e., recovered on actual output at standard recovery rate), the .
vanance is favourable. Otherwise (i.e., if the actual overhead expenses incurred is higher than the
overhead expenses recovered), it is adverse.
Overhead Cost Variance can be classified into two viz, Variable Overhead Variance and
Fixed Ov.erhead Cost Variance.
Variable Overhead Variance (VOV)
Variable Overhead Variance represents the difference between the variable overhead
expenses recovered for actual, output at the standard recovery rate and the actual variable
overhead expenses incurred for the same output. Therefore,

Variable OVerhead}
Variance VOV =
[ To~ Standard
Vanable Costs -
J [TOtal Actual vanableJ
Overhead Expenses
, for Actual Output Incurred

tandard Variable
Actual] (Actual Varia?le

L
= Cost per unit x Actuall
Output - Cost per umt x
of Output of Output outpu~
If the actual variable overhead expenses incurred is lower the standard variable overhead
expenses recovered for actual output, the variance is favourable. Otherwise (i.e., if the actual
variable overhead expenses incurred is higher than the standard variable overhead expenses
recovered for actual output), it (Le., variance) is adverse. Since the Total Variable Overhead
Variance arises due to the difference in either the hourly rate or the labour efficiency or both,
Variable Overhead Variance is classified into two viz, Variable Overhead Expenditure (or
Spending) Variance and Variable Overhead Efficiency Variance.
Variable Overhead Expenditure (or Spending or Controllable) Variance (VOEV)
This is computed by using the following formula.

J ~
Variable Over- }
head Expenditure
Variance, VOEV
=
ttandard Variable
Overhead Rate per x
Hour
Actual
Number of -
Hours Worked
[Actual Variable
Overhead Expenses
Incurred
Standard Costing : 585

Variable Overhead Efficiency Variance (VOEtv)


It j:-. a function of the difference between standard hours and actual hours, and the

J
standard variable overhead recovery rate per hour. Therefore, .

~[Actual
Variable
Overhead
Efficiency
Variance VOEtV
}
= Output x
Standard
Hours.
per Umt
] J[-
Actual
Hours x.
Standard
Variable
Overhead
Recovery Rate
Dlustration: 7.12
From the following details, compute the Variable Overhead Variances.
Standard Actual
Labour hours 1,000 1,200
Output (units) 500 400
Variable overheads (Rs.) 200 300
Solution:
Standard Recovery Rate:

Per hour = (1'~O ~~~rsJ= Re. 0.2 . = (RS.


Per umt 200
500'
umts
J= Re. 04-
.

Standard Hours for} _ (1,000 hours


Actual Output -
. l
500 units x 400 umts) = 800 hours
Actual Recovery Rate:

Rs.300 J
Per hour = ( 1,200 hours = Re. 0.25 Per unit = ( 4~~ !~~ J=Re. 0.75

Vana~le
. Overhead }
Vanance, VOH
= [Actual
Output
(units)
x Variabl~
Standard
Cost
per umt of
Output
J - [A
0
V~~bleal
h d
ver ea s
J
=[(400 units x Re. 0.4) - Rs. 300] =(Rs: 160 - Rs. 300) =Rs. 140 (A)
J J
~
Variable
Overhead
}
tandard Variable
= Overhead Rate per x
Actual
Number of -
[Actual Variable
0 verheadE xpenses
Expenditure Hour Hours Worked
Variance. VOEH

= [(Re. 0.2 x 1,200 hours) - Rs. 300] = (Rs. 240 - Rs. 300) = Rs. 60 (A)
Management Accounting : Sf!6

~[Actual J Standard J
J[
Variable
Overhead } = x Standard Actual Variable Over-
Efficiency Output Hours - Hours x head Recovery
Variance, VOEfV per Unit Rate per Hour

. x
= [{ 400 umts 1,000 J
hours} -1,200 x Re. 0.2
500 units

= [(400 units x 2 hours) - 1.200 hours] x Re. 0.2


=[(800 hours -1,200 hours) x Re. 0.2] =(400 hours x Re. 0.2) =Rs. 80 (A)
Verification: VOV = VOEV + VOEfV
Rs. 140 (A) = Rs. 60 (A) + Rs: 80 (A) = Rs. 140 (A)
Fixed Overhead Cost Variance (FOCV)
This represents the difference between the standard fixed overhead expenses for actual
output (at standard recovery rate) and the actual fixed overhead expenses incurred for the same
output. Therefore,

Fixed overhead}
Cost
[Standard Fixed [ Actual Fixed ~
= Overhead Expenses - Overhead Expenses
J
Variance, FOCV for Actual Output Incurred

=
Actual
Output
[ (units)
x
Standard Fixed
Overhead Recovery
Rate per unit
J -
Actual Fixed]
Overhead
( Expenses

In case the calculation is made on hourly basis, then,

Fixed overhead}
Cost =
~ Output
Actual
x
Standard }
Hours x
Standard Fixed J[
Overhead Expenses -
Actual Fixed
Overhead
. Expenses
J
perum .t Recovery
Variance, FOCV (units) Rate per hour Incurred

= [(AO x SH) x SRRf]- [AFO]


where,
SRRf =Standard Fixed Overhead Recovery Rate per unit or per hour
AO =Actual Output (units)
Standard Costing : 587

SH =Standard Hours per unit


APO = Actual Fixed Overhead Expenses incurred
IDustration: 7.13
From the information furnished below, determine the Fixed Overhead Cost Variance.
Standard Actual
Labour hours 1,000 1,200
Output (units) 500 400
Fixed overhead (Rs.) 300 300

Solution:

Fixed overhead} [ActUal Standard FiXed [Actual Fixed]


=
Cost
Variance, FOCV
Output
(units)
x Overhead
Recovery.
Rate per umtJ - Overhead
Expenses

=(400 units x 5~~ ~~~s J-Rs. 300


= [(400 units x Re. 0.6) - Rs. 300] = [Rs. 240 - Rs. 300] = Rs. 60 (A) OR

J J
~n{
Standard Fixed [Actual Fixed
Fixed Overhead { Actual Standard } Overhead Expenses _ Overhead
Cost
}
= Output
. ) x Hours. x Recovery Expenses
Variance, FOCV (umts per umt Rate per hour Incurred

= ~ 400 umts x
. 1,000 hOUrs} Rs.300 l
500 units x 1,000 hour~ - [Rs. 300]

=[(400 units x 2 hours) x Re. 0.3] - [Rs. 300] =[(800 hours x Re. 0.3) - Rs. 300]
= (Rs. 240 - Rs. 300) = Rs. 60 (A)
If the amount of fixed overhead expenses incurred is lower than the arpount of fixed
overhead expenses recovered, the variance is favourable. On the other hand. it is 'adverse if the
amount of fixed overhead expenses incurred is higher than that of recovered.
Fixed Overhead Cost Variance arises due to two important reasons. One, due to the
change in the overhead expenses (price) and two, due to the change in the output (volume) .. It is,
therefore, necessary to split the Fixed Overhead Cost Variance into its constituent parts viz.,
Fixed Overhead Expenditure Variance and Fixed Overhead Volume Variance.
Management Accounting : 588
Fixed Overhead Expenditure (or Spending) Variance (FOEV)
This variance is due to the difference between the budgeted fixed overhead expenses and
the actual fixed overhead expenses incurred, The budgeted fixed overhead expenses are
computed on the basis of the expected prices and consumption rates of various items of fixed
input factors, Consequently, whenever there is a change in these variables, which is of course a
reality, some change in the fixed overhead expenses can be observed. Hence, this variance is
~omputed, Therefore,

Fixed overhead} = 'rBudgeted Fixed Actual Fixe~


Expenditure
Variance, FOEV
l
Overheads overheadsJ

Standard Fixed BUdgetej' (Actual Fixed]


= Overhead,Recovery x Output - Overheads
[ Rate per uOlt of Output

Budgeted production may be computed by dividing budgeted or standard working hours


by the budgeted or standard hours per unit. That is,

BUdgeted}
'
[TOtal Budg~ted or Standardj
Working Hours
Producti on =
(um'ts) Budgeted or Standard
H '
ours per uOlt

mustration: 7.14
From the following information, compute the Fixed Overhead Expenditure Variance,
Budgeted Actual
Number of hours 1,000 1,200
Output (units) 500 400
Fixed overheads (Rs,) 300 300

Solution:

Fixed overhead}
Expenditure
Variance, FOEV
=
~BUdgeted
Output
(units)
x
Standard Fixed }
Overhead Recovery -
Rate per Unit
Actual
Fixed
J
Overheads

=[ {500 units x 5~~i~} - RS.300J =(Rs. 300-Rs. 300) =0


Standard Costing : 589

If the amount of fixed overheads Incurred is lower than the budgeted fixed overheads, the
variance is favourable. Otherwise (i.e., If the amount of fixed overhead expenses incurred is
higher than the amount of budgeted fixed overhead expenses), it is adverse.
Fixed Overhead Volume Variance (FOVV)
Fixed Overhead Volume Variance is a part of Fixed Overhead Cost Variance and it is due
to the difference between the budgeted level of output and the actual level of output achieved.
This difference results in, or causes, the under or over-recovery of fixed overhead expenses. It,
therefore, represents the difference between the fixed overhead expenses recovered or absorbed
on actual output and those on budgeted output.

BUdgeted Fixed] Fixed Overhead Expenses ]


Fixed ~verhead Volume
Vanance, ·FOVV
} = L Overhead
Expenses
_
[ Recovered on Actual
Output at Standard Rate

J J
~
Budgeted Standard Fixed Actual Standard Fixed
= Output x Overhead Recovery - Output x Overhead Recovery
[ (units) Rate per Unit (units) Rate per Unit

Standard Fixed Budgeted


= Overhead Recovery Output - Actual}]
Output
[ Rate per Unit { (unit,,) (UnIts)

Therefore, for the previous Illustration (Illustration: 7.14),

Fixed Overhead VOlume}


Variance, FOVV
[RS 300 {
= 500 units 500 units - 400 units
}J
=(Re. 0.6 x 100 units) =Rs. 60 (A)
Verification. FOCV =FOEV + FOVV
Rs. 60 (A) =0 + Rs. 60 (A) =Rs. 60 (A)
If the actual output is lower than the budgeted output. the variance is adverse. and it is
favourable if the actual output is higher than budgeted output.
Fixed Overhead Volume Variance can be further classified into three viz, Fixed Overhead
Efficiency Variance, Fixed Overhead Capacity Variance and Fixed Overhead Calendar Variance.
Fixed Overhead Efficiency Variance (FOEfV)
It is a part of Fixed Overhead Volume Variance and it helps to evaluate the efficiency or
otherwise in the performance. This is computed on the basis of the difference between the actual
hours taken to complete a work and the standard hours allowed to complete that work. This
difference is multiplied by the standard fixed overhead recovery rate to obtain Fixed Overhead
Efficiency Variance. Therefore,
Management Accounting : 590

Fixed Overhead } _ [~tandard Fixed OverheadI


Efficiency
Variance, FOEN
- Recovery Rate Jx [ forStandard Hours
Actual Output -
Actual ]
Hours

It may be noted here that if the actual hours worked exceed the standard hours allowed for
the actual output, the FOEN is unfavourable. On the other hand, if the time taken to complete a
work is lower than the time allowed, the variance is favourable. For the previous example
(Illustration 7.14), Fixed Overhead Efficiency Variance comes to Rs. 120 (A) as shown below.

Fixed ~verhead }
EffiCIency
Variance, FOEN
= Rs. 300 [(1,000 hours
1000 h
,ours
. J
500't x 400 umts - 0,200 hours)
um s
J
=[Re. 0.3 (800 hours - 1,200 hours)] = [Re.O.3 x 400 hours] =Rs. 120 (A)
Fixed Overhead Capacity Variance (FOCaV)
It is very well known that the actual capacity of the plant or machine normally differs
from the planned or expected capacity. It is due to a number of reasons including strikes,
lockouts, break-down, shortage of resources, etc. It may also be due to the work for more time
than the normal working hours, changes in the number of shifts of one or more machines, etc.
This variance, therefore, indicates the utilization of available capacity. In order to compute the
Fixed Overhead Capacity Variance, the following procedure is followed.

J
a. With Calendar Variance

Fixed overhead} ( Standard Fixed (Standard Standard outpuJ


Capacity = Overhead Recovery x Output for - for Actual
Variance, FOCaV Rate per Unit ,Actual Hours Working Days

J
OR

Fixed overhead} ~ Standard Fixed Standard Hours for _ ActUal]


Capacity . = Overhead Recovery x [ Actual Working Days Hours
Variance, FOCaV Rate per Unit
b. Without Calendar Variance

Fixed overhead}
Capacity
Variance, FOCaV
=
~ Standard Fixed
Overhead Recovery
Rate per Unit
J x
Standard Output
[ for Actual Hours
_ BUdgeted]
Output

J
OR

Fixed overhead} ~ Standard Fixed Budgeted _ Actual]


Capacity = Overhead Recovery x [
Hours Hours
Variance, FOCaV Rate per Hour
Standard Costing : 591

Fixed Overhead Calendar Variance (FOCIV)


This variance arises when there is a difference between the number of working days in the
budget period and the actual number of working days in that budget period. Fixed Overhead
Calendar Varinace is computed by using the following formula.

· dOh
Flxe ver ead} [ Standard Fixed ] Number of Excess]
Overhead Recovery or Deficit Hours
Calendar = x
Variance, FOCIV Ratele:\~Ia~ur or [ or Days Worked

Illustration: 7.1S
Calculate Overhead Variance from the following data.
Budget Actual
Number of working days 20 22
Man hours per day 8,000 8,400
Output per man hour in units 1.0 0.9
Fixed overhead cost (Rs.) 1,60,000 1,68,000 [CS]
Solution:

ActUal} [0.9 unit per 8,400 Man- 22 Work-I


Output = Man-hour x hours a Day x ing Days j = 1,66,320 units
BUdgeted} _ [1 unit per 8,000 Man- 20 work-I
Output - Man-hour x hours a Day x ing Daysj = 1,60,000 units

Standard Fixed overhead} _ rsudgeted Fixed Overhead


Recovery Rate per Unit - l
Budgeted Output
J_r Rs. 1,60,000 J-
R 1
- ll,60,000 uni~j - e.

Standard Fixed }
°RverheadMReCohvery =
[ Rs. 1 60 000
8,000 hou~s ~ 20 days
J= [ Rs. 1,60,000
1,60,000 man-hours
J= Re. 1
ate per an- our
Actual Man-hours =(8,400 Man-hours a day x 22 days) =1,84,800 hours.

Fixed Overhead }
. =
[ OActUal
utput x
Standard
Hours x
o::::a~~~~~~ej
R R
_ (Aoctuvealrh~aXdedJ
Cost Vanance, FOCV ( . )
umts .
per umt ecovery .t ate Expenses
perum
=[(1,66,320 units x 1 hour x Re. 1) - Rs. 1,68,000]
= (Rs. 1,66,320 - Rs. 1,68,000) = Rs. 1,680 (A)
Management Accountinq : 592

Fixed Overhead BUdgeted Standard Fixed } Actual ]


Expenditure = [{ Output x Overhead Recovery - Fixed
Variance, FOEV (units) Rate per Unit Overhead

= [(1,60.000 units x Re. 1) - Rs. 1,68,000]


= [Rs. 1,60,000 - Rs. 1,68.000] = Rs. 8,000 (A)

Fixed overhead} Standard Fixed {BUdgeted


Volume = Overhead Recovery Output- Actual}]
Output
Variance, FOVV [ Rate per Unit (units) (units)

= [Re. 1 0,60,000 units - 1,66,320 units)]


=(Re. 1 x 6,320 units) =Rs. 6,320 (F)
Verification, FOCV =FOEV + FOVV
Rs. 1,680 (A) = Rs. 8,000 (A) + Rs. 6,320 (F) = Rs. 1,680 (A)

Fixed Overhead }
Efficiency =
[
Standard Fixed
O verhead
J[
x
Standard Hours
for Actual Output
-
Actual
Hours
J
Variance, FOEfY Recovery Rate
= Re. 1 [(1 hour per unit x 1,66,320 units) - 1,84,800 hours]
=[Re. 1 (1.66.320 hours - 1.84.800 hours)] =(Re. 1 x 18.480 hours)
=Rs. 18,480 (A)

.
Fixed Overhead
Capaclty
} ~Standard F'lxed Standard Standard output}]
= Overhead Recover x Output for - for Actual
Variance, (with Calendar Ra t e per UOJ't Y {
,_ Actual Hours Working Days
Vanance), FOCaV

1.84.800 One unit}


of Output _
{l:;,ooo Work-
ing Hours per x
One unit
of Output x 22 Actual}
=Re.l Actual x per Hour Day (Stan- per Hour
Working
{ Hours Days
(Standard) dard) (Standard)

= [Re, I (1.84.800 Ul1Jts - 1.76.000 units)!


= (Re. I x 8,800 units) = Rs. S,800 (F)
Standard Costing: 593

Fixed Overhead } ' [


Calendar
Variance, FOClV
Standard Fixed
= Overhead Recovery
RatepperDHour or
J x
[Number of Excess
or Deficit Hours
or Days Worked
J
er ay

= (Rs. 8,000 x 2 days) = Rs. 16,000 (F)


Verification, FOVV =FOEfV + FOCaV + FOCIV
Rs. 6,320 (F) = Rs. 18,480 (A) + Rs. 8,800 (F) + Rs. 16,000 (F)
=Rs. 18,480 (A) + Rs. 24,800 (F) =6,320 (F)
Illustration: 7.16
Vivek Ltd., has furnished you the following information for the month of August 1979.
Budget Actual
Output (units) 30,000 32,500
Hours 30,000 33.000
Fixed overheads (Rs.) 45.000 50,000
Variable overhead (Rs.) 60,000 68,000
Working days 25 26
Calculate the Variance. leA (lilt), November 1979)
Solution:
Standard variable}_ r.Rs. 60,000
Cost per Unit -130,OOO units
J= Rs. 2

Standard variable} (RS. 60,000


Overhead Expenses = 30 000 hours
J = Rs. ~
Per Hour '

Standard Fixed Overheadl


Expenses per Unit
= r.Rs. 45,000
J 130 ,OOO units
J- -
R 15
s. .

Standard Hours }
for Actual Output
_ r~O,OOO hours
-13 0 ,OOO units x
32 500 .
, umts
J 32 500 h
=, ours

Standard output} _ r~O.OOO units x 33 000 hours]' .=Rs. 33.000 units


for Actual Hours -130.0oo hours' .
Management Accounting : 594

Standard O~tput for 1


= (30,000 units x 26 da sl = 31 200 units
Actual Workmg Daysf L 25 days Y)'

xed 0pverDheadl =r.R~54~,000


SRtandard FiR
ecovery ate er ay f L a y s
J= Rs. 1,800

Calculation of Variances

Vana~le
. Overhead }
Vanance, VOV
= [Actual
Output
(units)
x Variabl~
Standard J
Cost -
per umt of
[AV;:~ble
0 h d
al J
Output ver ea s

=[(32,500 units x Rs. 2) - Rs. 68,000]


=(Rs. 65,000 - Rs. 68,000) =Rs. 3,000 (A)
Variable Over- } tandard Variable Actual J
[ActUal Variable ~
head Expenditure
Variance, VOEV
=
L Overhead Rate
per Hour
x Number of -
Hours Worked
= [(Rs. 2 x 33,000 hours) - Rs. 68,000]
Overhead Expenses
Incurred

= (Rs. 66,000 - Rs. 68,000) = Rs. 2000 (A)

Variable overhead} (Standard Variable {St d d H


Efficiency Variance, = Overhead Recovery x for ~c=al O~:~t - Actual}]
Hours
VOEfV Rate per Hour
=[Rs. 2 (32,500 hours - 33,000 hours)]
=[Rs. 2 x 500 hours) =Rs. 1,000 (A)
Verification, VOV = VOEV + VOEfV

JL
Rs. 3,000 (A) = Rs. 2,000 (A) + Rs. 1.000 (A) = Rs. 3.000 (A)
ctual
Standard Fixed FiXed
Fixed Overhead cost} _ ~ctual

J
Overhead Expenses _ Overhead
Variance FOCV - utput x Recovery Expenses
• [ (umts) . Incurred
Rate per umt

=[(32.500 units x Rs. 1.5) - Rs. 50,000]


= (Rs. 48.750 - Rs. 50.000) = Rs. 1,250 (A)
Fixed Overhead Budgeted Standard Fixed } Actual J
Expenditure
Variance. FOEV
=
U Output x
(units)
Overhead Recovery
Rate per Unit
- Fixed
Overheads
tandato,"n. , 595

= [(30,000 units x Rs. 1.5) - Rs. 50.000]


= (Rs. 45,000 - Rs. 50,000) = Rs. 5,000 (A)

Fixed overhead}
Volume Variance, =
Standard Fixed
Overhead Recovery x
{BUdgeted
Output -
ActUal~
Output
FOVV [ Rate per Vnit (units) (units)

= [(Rs. 1.5 x (30,000 units - 32,500 units)]


= (Rs. 1.5 x 2.500 units) = Rs. 3,750 (F)
Verification, FOCV = FOEV + FOVV
Rs. 1,250 (A) = Rs. 5,000 (A) + Rs. 3,750 (F) = Rs. 1,250 (A)

Fix~~fio~erhead } [Standard Fixed Overhead {Standard Hours Actual}]


Varian~~:e;riEfV = Recovery Rate per Hour x for Actual Output - Hours

= ~~'~O;:h~o.:!. x (32,500 hours - 33,000 hours) ]

= (Rs. l.5 x 500 hours) = Rs. 750 (A)

·
·
.
Fixed Overhead
Capacity
(. h C I d
Vanance WIt a en ar
) FOC V
_
-
J ~Standard F·lxed
Overhead Recovery x
.
Rate per Vmt
{Standard
Output for -
Actual Hours
Standard output}]
for Actual
Working Days
Vanance, a

= [Re. 1.5 (33,000 units - 3l,200 units)]


=(Rs. 1.5 x 1,800 units) =Rs. 2,700 (F)

Fixed overhead} [ Standard Fixed ] [ Nurnber 0 f E xcess ]


Calendar = Overhead Recovery x D W k d
Variance, FOCIV Rate per Day ays or e
=[Rs. 1,800 x (26 days - 25 days)] =(Rs. 1,800 x 1) =Rs. 1,800 (F)
Verification, FOVV =FOEfV + FOCaV + FOCIV
Rs. 3,750 (F) =Rs. 750 (A) + Rs. 2,700 (F) + Rs. 1,800 (F)
=Rs. 750 (A) + Rs. 4.500 (F) =Rs. 3,750 (F)
Management Ai"ng . 6
Sales Variances ,
.,
"

Quantum of profit is influenced by both the cost and;' ·enue. Cost Variances influence the
amount of profit favourably or adversely depending upon whether the cost variance is favourable or
unfavourable. Further, one can expect a difference between the actual sales and the target sales. The
difference will have a direct effect on the profit and therefore, it is necessary to compute sales variances.
It is very well known that Sales Variances are due to either the change in quantity or price or both. It,
therefore, necessitates a detailed analysis. There are two ways to compute the Sales Variances viz, Value
Method and Profit Method. The break up of Sales Variances is presented below.
Classification of Sales Variances

,....... Price
Value Variance
Method
Quantity
~
Volume Variance
Sales
Variances
M ~
Variance
r---
Profit Mix
4, LJ.
Method Variance

Value Method
This is based on the sales value or revenue or turnover and therefore, this variance is
called value variance or sales revenue variance (SRV) or variance based on turnover. This
variance is. therefore, computed by finding out the difference between the budgeted sales value or
revenue (BSR) and the actual sales value or revenue (ASR). Therefore.
Sales Revenue} _ (Actual Sales
Variance. SRV - Revenue
Budgeted Sales
Revenue
J
If the actual sales revenue exceeds the budgeted revenue, the variance will be favourable.
Otherwise (i.e., if ASR < BSR). it will be unfavourable or adverse.
Illustration: 7.17
Arena Manufacturers operate Budgetary Control and Standard Costing Systems. The
following information is available for the month of March 2005 from their books.
Std. Cost of Std. Selling Budget Actual
Product sales per Price per Units to be Sales Value Sales Value
Units Sold
unit (Rs.) unit (Rs.) Sold (Rs.) (Rs.)
E 100 120 100 12,000 100 ,11,000
F 94 120 50 6.000 50 6,000
G 75 90 100 9,000 200 17,000
H 40 60 75 4.500 50 3.000
325 31,500 400 37.000
_ Standard Costing : 597

From the above data, calculate (a) Sales Variance, (b) Sales Volume Variance, and (c) Sales
Price Variance. [M.Com, UoMJ
Solution:
Sales Revenue} _ (Actual Sales
Variance, SRV - Revenue
Budgeted Sales
Revenue
J
Hence, SRV of E = (Rs. 11,000 - Rs. 12.000) = Rs. 1,000 (A)
F = (Rs. 6,000 - Rs. 6,000) = 0
G =(Rs. 17,000 - Rs. 9,000) =Rs. 8,000 (F)
H =(Rs. 3,000 - Rs. 4.500) =Rs. 1,500 (A)
=Rs. 5,500 (F)
This Sales Revenue Variance may be due to the difference between the actual and the
buageted selling prices or due to the difference between the actual and the budgeted sales volume
'or due to the difference between the actual and the standard sales mixes or a combination of these.
It may 'also be due to some other reasons. Therefore, Sales Revenue Variance is made up of Price
Variance and Volume Variance.
Sales Price Variance (SPV)
This variance which is a part of Sales Revenue Variance arises due to the difference
between the actual selling price (ASP) and the standard selling price (SSP). To compute the Price
Variance, the difference between the actual and the standard selling prices is to be multiplied by
the actual sales quantity (ASQ) as shown below.

Sales Price } _ [ActUal


Variance SPV - Sales x
{ Actual
Selling
Standard
Sel.ling
}J
. Quantity Price Price
If the actual selling price is higher than the standard selling price, the variance will be
favourable variance. Otherwise (Le., if ASP < SSP), it is adverse or unfavourable variance. With
this, let us compute the Sales Price Variance for the problem solved under Sales Revenue
Variance (Le., for l11ustration: 7.17).
Solution:
Actual Selling Prices are: E =Rs. 110; F =Rs. 120; G =Rs. 85, and H =Rs. 60

Sales Price} (Actual


Variance. SPV = Sales. x
{ Actual
Selling
Standard
Sel.ling
}J
Quantity Price Price
Management Accounting: 598
SPVof E =[100 units x (Rs. 110 - Rs. 120)] =(100 units x Rs. 10) = Rs. 1,000 (A)
F =[50 units x (Rs. 120 - Rs. 120)] = (50 units x Rs. 0) = 0
G = [200 units x (Rs. 85 - Rs. 90)] =(200 units x Rs. 5) =Rs. 1,000 (A)
H = [50 units) x Rs. 60 - Rs. 60)] = (50 units x Rs. 0) = 0
Rs. 2.000 (A)
Sales Volume Variance (SVV)
As already stated, this variance is a part of Sales Revenue Variance. This variance
represents the difference between the standard sales revenue and the budgeted sales revenue. It
arises due to the difference between the standard sales quantity (SSQ) and the actual sales
quantity (ASQ) as presented below.

Sales VOlume}
Variance, SVV =
(Standard
LSales Revenue -
Budgeted
Sales Revenue
J
Sales VOlume} _ [Actual Standard] [Standard]
Variance SVV - Sales Sales . x Selling
, Quantity Quantity Price

If the actual sales volume is higher than that of standard, the variance will be favourable.
Otherwise (Le., if ASQ < SSQ), it will be unfavourable variance. With this, let us compute the
Sales Volume Variance for the problem discussed under Sales Revenue Variance (Le., for
lliustration: 7.17).
Solution

Sales VOlume}_ [Actual Standard] [Standard]


Variance SVV - Sales Sale~ x Sel.ling
, Quantity Quantity Price

SVV of E =[(100 units - 100 units) x Rs. 120] =(0 units x Rs. 120) =Rs. o
F = [(50 units - 50 units) x Rs. 120] =(0 units x Rs. 120) =Rs. 0
G = [(200 units - 100 units) x Rs. 90] = (100 units x Rs. 90) = Rs. 9,000 (F)
H =[(50 units - 75 units) x Rs. 60] =(25 units x Rs. 60) =Rs. 1,500 (A)
Rs. 7,500 (F)
Verification, SRV =(SPV + SVV)
Rs. 5,500 (F) = [Rs. 2,000 (A) + Rs. 7,500 (F)] =Rs. 5,500 (F)
The Sales Volume Variance can be divided into two sub-variances viz., Sales Quantity
Variance and Sales Mix Variance. This break-up of Sales Volume Variance is necessary in the
case of multi-product concerns engaged in the sale of two or more products.
Standard Costing : 599

Sales Quantity Variance (SQV)


, This variance which is ,also called Sales Volume Sub-variance is a part of Sales Volume
Variance. This represents the difference between the budgeted sales and the revised standard
sales.
Sales Quantity} _ (Revised Standard Budgeted Sales]
Variance, SQV - LSales Revenue Revenue

Standard Standard]
Budgeted Salesl
Revenue, BSRJ
= [ Sales x Selling
Quantity Price

Revised Standard }
Sales Revenue, RSSR =
(Revised Standard
L
Sales Quantity' x
Standard
Selling Price
J
Revised Standard 1 [ Total Ouantity of Actual Mix Standard Sales Quantity]
Sales Quantity, RSSQf = Total Quantity of Standard Mix x of each Product

Hence, Sales Quantity 1_ ~R:::~ Standar~


Selling -
~tandard
Sales x
Standard]
Selling
Variance, SQvJ - Sales x
Price Quantity Price
Quantity

_ (Revised Standard _ Standard Salesl (Standard Selling]


- Sales Quantity Quantity J x L Price
If the revised standard sales quantity exceeds the standard or budgeted sales quantity,
variance will be favourable. Otherwise (Le., if RSSQ < SSQ), it will be unfavourable variance.
With this, let us compute the Sales Quantity Variance for the problem discussed under Sales
Revenue Variance (Le., for Illustration: 7.17).

Revised Standard} [TOtal Quanti.ty Standard Sale]


Sales Quantity. = of Actual Mlx x Quantity of
RSSQ Total Quantity a Product
of Standard Mix

RSSQof E= t400 units x 100 Units] = 123.08 !:::! 123 units


325 units

F= t400 units x 50 units ]


325 units = 61.54 !:::! 62 units

G= t400 un~ts x- 100 Units] = 123.08!:::! 123 units


325 umts
Management Accounting: 600

H- [400 units x 75 units] = 92.31 :::: 92 units


- L325 units
400 units

Sales Quantity} _ (ReVised Standard Standard Sales] rStandard Selling]


Variance, SQV - Sales Quantity - Quantity x l Price
SQVof E =[(123 units - 100 units) x Rs. 120] =(23 units x Rs. 120) =Rs. 2,760 (F)
F =[(62 units - 50 units) x Rs. 120] =(12 units x Rs. 120) =Rs. 1,440 (F)
"G =[(123 units - 100 units) xRs. 90] =(23 units x Rs. 90) =Rs. 2,070 (F)
H =[(92 units -75 units) x Rs. 60] =(17 units x Rs. 60) = Rs. 1,020 (F)
Rs. 7,290 (F)
Sales Mix Variance (SMV)
When the actual sales quantity of different products are not in the same proportion as
budgeted, this variance will arise. This represents the difference between the standard value of
standard mix of sales and the revised standard value of actual mix. Therefore,

Sales Mix 1 _ [ ~c:al Revised Standar~ rStandard Selling]


Variance, SMvf - Q e~
uantIty
- Sales Quantity x L Price

If the actual sales quantity exceeds the revised standard sales quantity, it will result in
favourable variance. Otherwise (Le., if ASQ < RSSQ), the variance will be adverse or
unfavourable. Using the same problem discussed under Sales Revenue Variance (i.e., lllustration:
7.17), the Sales Mix Variance can be computed as shown below.

Sales Mix 1
[Actual Revised ] [ Stan~ard ]
Variance SMVJ = Sales Standard x Selling
, Quantity Sales Quantity Price
SMVof E = [(100 units - 123 units) x Rs. 120] = (23 units x Rs. 120) = Rs. 2,760 (A)
F =[(50 units - 62 units) x Rs. 120] =(12 units x Rs. 120) = Rs. 1,440 (A)
G =[(200 units - 123 units) x Rs. 90] =(77 units x Rs. 90) = Rs. 6,930 (F)
H =[(50 units - 92 units) x Rs. 60] =(42 units x Rs. 60) = Rs. 2,520 (A)
Rs. 210 (F)
Verification, SVV = SQV + SMV
Rs. 7.500 (F) = Rs. 7,290 (F) + Rs. 210 (F)"= Rs. 7,500 (F)
A summary of Sales Variance is presented below.
Standard Costing: 601

Sales Variances

-+1 Sales Price Vllriance, SPV =ASQ (ASP - SSP)

Sales Revenue Sales Quantity


~
Variance, r+ Variance,
SRV =ASR - BSR SQV =(RSSQ - SSQ)
SSP
Sales Volume Variance,
-+ SVV =SSP (ASQ - SSQ) -I
=RSSR-BSR

Sales Mix Variance,


~
SMV =SSP (ASQ -
RSSQ)

Profit or Margin Method


Under this method, Sales Variances are ascertained and analysed based on profit or
margin. This method clearly shows the effect on profit. Classification and computation of
variances under this method are similar to those based on turnover. The important profit
variances are presented below.
Classification of Variances (Profit Method)

Sales Price } =(Standard Profit


-+ Variance - Actnal Profit)

Sales Quantity Variance =


Overall Profit -+
~Udgeted
-t
Variance =
(Budgeted Profit-
Standard Profit)
--. Sales Volume Variance
= (Budgeted Profit -
Standard Profit)
. Profit
- Standard]
Profit

Sales Mix Variance =


~
[ Standard -
Profit .
Revised
Standard
J
Profit
Management Accounting : 602
Since no difference is found between Sales Variances (based on turnover) and Profit
Variances, the profit variances are not discussed separately.
Disposal of Variances
As far as the disposal of variances are concerned, three methods are available as presented
below.
1. One of the methods suggests to transfer all the variances to Costing Profit and L~s
Account of the period in which these varianc~s occurred,
2. The second approach advises to distribute the variances among the inventories (work-in-
progress and finished goods) and the goods sold. That means, the variances are to be
distributed between the year-end inventories and units sold during the year in which these
variances occurred. This results in the presentation of both the inventories and units sold
at actual costs, and
3. The third alternative is to transfer all variances to a separate Reserve Account created for
this purpose. In this case, the balance in this account at the end of the period, may be
transferred to Costing Profit and Loss Account may be distributed between the
inventories and the units sold. The second method is- to (;arry forward the balance until
the variances are automatically offset.
mustration: 7.18
The standard materials required for producing 200 units is 250 kgs. Standard price is 60
paise per kg. 2,50,000 units were produced during the period. Actual materials purchased were
3,20,000 kgs at a cost of Rs.l,95,200. From the above, calculate: (a) Material Cost Variance, (b)
Material Price Variance, and (c) Material Usage Variance.
[Bangalore Uni., B.Com., May 2002J
Solution:
Standard Q.uantity } _
of Matenal for
Actual Production
-
t 250 kgs
200 units x2.50.000 units] = 3.12.500 kgs

kg of Matenal l3,20,OOO kg~


1
Actual Price 'per } = [ Rs.l,95, 200 = Re 0.61

. Material Cost } = (Standard Quantity x Standardl_ [Actual Quantity x Actuall


Variance, MCV l
of Material Price J
of Material Price J
= [3,12,500 kgs x Re 0.6] - [Rs.l,95,200]
=[Rs.l,87,500 - Rs.l,95,200] =Rs.7,700 (A)

Ma.terial Price} = [ActUal Quantity of


Vanance MPV Material Purcha~ed j
l x [Standard _ AC~alJ
Price Price
Standard Costing : 603

-
= 3,20,000 kgs X [Re 0.6 - Re 0.61]

r
=[3,20,000 kgs X Re 0.01] =Rs.3,200 (A)
Material usage}
tandard
Quantity Actual Quantity
Variance MUV = of Material for - Material for Actual
o~ X [Standard~
Pri
. Actual Production Production ce

= [3,12,500 kgs - 3,20,000 kgs] x Re 0.6


= [7,500 kgs x Re 0.6] = Rs.4,500 (A)
Verification: MCV = MPV + MUV
Rs.7,700 (A) = Rs.3,200 (A) + Rs.4,500 (A) = Rs.7,700 (A)
mustration: 7.19
The following information is given in respect of a material.
Standard Quantity: 1,000 units
Actual Quantity: 1,040 units
Standard Price: Rs.20 per unit
Actual Price: Rs.22 per unit
Calculate: a) Material Cost Variance, b) Material Price Variance, and c) Material Usage Variance.
[Mangalore Uni, B.B.M, May 1998 and 2000J
Solution:
Material cost} rStandard Quantity Standard I [Actual Quantity Actual)
Variance, MCV = l
of Material x Price J- of Material x Pricej
=[1,000 units x Rs.20] - [1,040 units x Rs.22]
=[Rs.20,000 - Rs.22,880] =Rs.2,880 (A)
Mattrial Price} _
Variance, MPV -
Actual Quantity Of]
[ Material Purchac;;ed x
[Standard _ Actuall
Price Price J
= 1,040 units x [Rs.20 - Rs.22] = [1,040 units x Re. 2] = Rs.2,080 (A)

Material usage}= [Standard quantity ~


Variance MUV of Matenal for
Actual Quantity
Material for Actual
Of] x
[Standard"1
Price J
, Actual Production Production

= [1,000 units - 1,040 units] x Rs.20 = [40 units x Rs.20] = Rs.800 (A)
Management Accounting : 604

Verification: MCV =MPV + MUV


Rs.2,880 (A) =Rs.2,080 (A) + Rs.800 (A) =Rs.2,880 (A)
Illustration: 7.20
For producing a commodity, the standard quantity of material fixed was at 10 kgs and
standard price was fixed at Rs.2 per kg. The actual quantity consumed was 12 kgs and the actual
price was Rs.l.90 per kg. Calculate Material Cost Variance.
[Kuvempu Un~ B.B.M, November 1998J
Solution:

Material Cost 1_ (Standard Quantity Standardl_ [Actual Quantity Actuall


Variance, MCV f- l of Material x Price Jof Material x Price J
= [10 kgs x Rs.2] "':-[12 Kgs x Rs.1.9]
= [Rs.20 - Rs.22.8] = Rs.2.8 (A)
Illustration: 7.21
From the following particulars, compute (a) Material Cost Variance (b) Material PriC(
Variance and(c) Material Usage Variance.
Quantity of material purchased: 3,000 units
Value of material purchased: Rs.9,000
Standard quantity of material required per tonne of output: 30 units
Standard rate of material: Rs.2.50 per unit
Opening stock of material: Nil
Closing stock of material: 500 units
Output during the period: 80 tonnes.
[Kuvempu Uni., B.Com., May 1997, 2002, and 2004 and October 1999J
Solution:
Output =80 tonnes
[Standard] material per tonne of output = 30 uni~
Hence, standard quantity of material for actual output =[80 tonnes x 30 units] =2,400 units
Actual material consumption = [3,000 units purchased - 500 units of closing stock]
=2,500 units
Actual Price per unit = [Rs.9,000 + 3,000 units] = Rs. 3

Material cost} _ (Standard Quantity Standard) [ActUal Quantity Actual)


Variance, MCV - lof Material x Price - of Material x Price
Standard Costing : 605

= [2,400 units x Rs.2.S] - [2.S00 units x Rs.3]


= [Rs.6.000 - Rs.7,SOO] = Rs.l,SOO (A)

Material Price} ( Actual Quantity Of] [Standard Actual!


Variance, MPV = Material Purchased x Price Price)

= 2,SOO units x [Rs.2.S - Rs.3]


=[2,SOO units x Re O.S] =Rs.l,2S0 (A)
Material usage} Ltandard quantity
Variance MUV =
Actual Q~antity
of Matenal for - of Matenal for x 'I'L!
J
[Standard]
, Actual Production Actual Production rnce

= [2,400 units - 2,SOO units] x Rs.2.S


=[100 units x Rs.2.S] =Rs.2S0 (A)
Verification: MCV = MPV + MUV
Rs.l.SOO (A) = Rs.l.2S0 (A) + Rs.2S0 (A) = Rs.t,SOO (A)
mustration: 7.22
The standard material and standard cost per kg of material required for the production of
one unit of product A is as follows.
Material: S kgs
Standard price: Rs.S per kg
Actual production: 400 units of A
Materials used: 2,200 kgs
Price of material: Rs.4.80 per kg
Calulate : a) Material Cost Variance, b) Material Usage Variance, and c) Material Price Variance.
IKuvempu Un;' B.Com, May 1991, 1994 and 2003, October 1996, 2000 and 2003, Mangalore
Un;' BBM, May 1999 and Bangalore Un;' B.Cmn, November 2003}
Solution:
Material Cost } _ rStandard Quantity Standard!_ [Actual Quantity Actual!
Variance. MCV - lof Material x Price J
of Material x Price J
=[(S kgs per unit x 400 units) x Rs.S] - [2.200 kgs x Rs.4.8]
= [2,000 kgs x Rs.S] - [Rs.l 0.S60] = [Rs.l O.OUO - Rs.l 0.560] =Rs.S60 (A)
Management Accounting : 606

Material Price} _
Variance, MPV -
rlActual Quantity ofl
Material Purcha~e'!J x [
Standard ActUal)
Price - Price

= 2,200 kgs x [Rs.5 - Rs.4.S] = [2,200 kgs x Re 0.2] = Rs.440 (F)

.
Material usage} L
tandard quantity Actual Q~antity
Variance MUV = of Matenal for x of Matenal for
' Actual Production Actual Production
x
J [Standard~
Price

= [2,000 kgs - 2,200 kgs] x Rs.5 = [200 kgs x Rs.5] = Rs.l ,000 (A)
Verification: MCV = MPV + MUV
Rs.560 (A) =Rs.440 (F) + Rs.l ,000 (A) = Rs.560 (A)
IUustration: 7.23
Standard Mix for the manufacture of chemical TIP is:
Material A =60 tons at Rs. 5 per ton
Material B = 40 tons at Rs. 10 per ton
Actual Mix being:
Material A =SO tons at Rs.4 per ton
Material B =70 tons at Rs.S per ton
Calculate Material Cost Variance and analyse into Sub-Variances.
[Mangalore Uni., B.Com., October 2001J
Solution:
Statement of Standard and Actual Material and Costs

Standard Actual
Material Quantity Rate Amount Quantity Rate Amount
(tons) (Rs.) (Rs.) (tons) (Rs.) (Rs.)
A 60 5 300 80 4 320
B 40 10 400 70 8 560
100 SSO

Material Cost } = (Standard Quantity x Standardl_ [ActUal Quantity x Actuall


Variance, MCV l of Material Price j of Material PriceJ

:.MCV of Material:
A = [60 tons x Rs.5] - [SO tons x Rs.4]
Standard Costing : 607

= [Rs.300 - Rs.320] Rs. 20 (A)


B = [40 tons x Rs.I0] - [70 tons x Rs.8]
= [Rs.400 - Rs.560] Rs. 160 (A)

Rs. 180 (A)

Ma~erial usage} (Actual Quantity ofl [Standard Actuall


Variance, MPV = l Material PurchasedJ x Price - Price j
:. MPV of Material:
A = RO lons ;x [R~.5 - R~.4] = [80 tons x Re.l] Rs. 80 (F)
B = 70 tons x [Rs.lO - Rs.8] = [70 tons x Rs.2] Rs. 140 (F)
Rs. 220 (F)

Material Price} _ [Standa~d q~antity A~tual Q~antitYJ


Variance MUV - of M,ltenal for - of Matenal for x
, Actual Production Actual Production

:. MUV of Material:
A = [60 tons -- 80 tons] x Rs.5 =[20 tons x Rs.5] =Rs. 100 (A)
B = [ 40 tons - 70 tons] x Rs.IO = [30 tons x Rs.I0] = Rs. 300 (A)
Rs. 400 (A)
Verification: MCV = MPV + MUV
A: Rs. 20 (A) =Rs. 80 (F) + Rs. 100 (A) = Rs. 20 (A)
B: Rs. 160 (A) =Rs. 140 (F) + Rs. 300 (A) = Rs. 160 (A)
Total: Rs. 180 (A) = Rs. 220 (F) + Rs. 400 (A) = Rs. 180 (A)
lliustration: 7.24
Suresh Chemicals submit the following information for the month of May 1999.
Standard material cost to produce 100 kgs of chemical Mis:
Material A: 30 kgs at Rs.200 per kg
Material B: 40 kgs at Rs.lOO per kg
Material C: 50 kgs at Rs.120 per kg
The actual production of M was 1,000 kgs
Materials consumed:
A: 350 kgs at Rs.l80 per kg
B: 420 kgs at Rs. 120 per kg
C: 530 kgs at Rs. 140 per kg
Management Accounting : 608
Calculate Material Cost Variance and Material Price Variance in respect of A, B and C
[Mangalore Uni., B.B.M., October 2000J
Solution:
Statement of Standard and Actual Material and Cost
Standard Actual
Material Quantity ·Rate Amount Quantity Rate Amount
(kgs) (Rs.) (Rs.) (kgs) (Rs.) (Rs.)
A 300* 200 60,000 350 180 63,000
B 400 100 40,000 420 120 50,400
C 500 120 60,000 530 140 74,200 I
1,60,000 1,87,60.0

* Standard quantity given is for 100 kgs of output. Hence, for 1,000 kgs, 10
. times the standard.

Material cost} _ (Standard Quantity Standardl fActual Quantity Actua~


Variance, MCV - l
of Material x Price J-l of Material x PriceJ

:.MCVofMaterial:
A = [300 kgs x Rs.200] - [350 kgs x Rs.180]
= [Rs. 60,000 - Rs.63,000] = Rs. 3,000 (A)
B = [400 kgs x Rs.100] - [420 kgs x Rs.120]
= [Rs. 40,000 - Rs.50,400] = Rs. 10,400 (A)
C = [500 kgs x Rs.120] - [530 kgs x Rs.140]
= [Rs. 60,000 - Rs. 74,200] = Rs. 14,200 (A)
Rs. 27,600 (A)

Material Price} _
Variance, MPV -
(ActUal Quantity Of]
Material Purchased x
(Standard Actuall
Price - Price J
:.MPVofMaterial:
A= 350 kgs x [Rs.200 - Rs.180] = [350 kgs x Rs.20] = Rs. ';.000 (F)
B = 420 kgs x [Rs.I00 - Rs.120] = [420 kgs x Rs.20] = Rs. 8,400 (A)
C = 530 kgs x [Rs.120 - Rs.I40] = [530 kgs x Rs.20] = Rs. 10,600 (A)
= Rs. 12,000 (A)
Standard Costing : 609

mDStration: 7.25
From the following information, calculate: (a) Labour Rate Variance, (b) Labour Cost
Variance, and (c) Labour Efficiency Variance.
Standard hours per unit : 3 hours
Standard wage rate : Rs.3 per hour
Actual units produced : 250 units
Actual wage rate : Rs.3.50 per hour
Hours worked : 800 hours.
[Mangalore Uni., BBM, May 1997 and 1998,' Kuvempu Uni., B.Com., October 1997}
Solution:

Standard Hours for} .


Actual Output = [250 umts x 3 hours] =750 hours
Labour Cost 1_ (Standard Time for Standardl (ActUal Time for Actuall
Variance, LCVf - L
Actual Output x Rate J- Actual Output x Time J
=[750 hours x Rs.3] - [800 hours x Rs.3.5]
= [Rs.2,250 - Rs.2,800] = Rs.550 (A)

Variance, LRV -
r
Labour Rate } _ Number of Hours Worked)
L
for Actual Output x
[Standard Actua~
J
Rate - Rate j
= 800 hours x [Rs.3 - Rs.3.5]
= [800 hours x Re 0.5] = Rs.400 (A)

Labour EfficienCY} _rstandard Time for Actual Time for] (Standardl


Variance, LEV -L Actual Output - Actual Output x Rate l J
= [750 hours - 800 hours] x Rs. 3
=[50 hours x Rs 3] =Rs.150 (A)
Verification: LCV = LRV + LEV
Rs.550 (A) =Rs.400 (A) + Rs.150 (A) =Rs. 550 (A)
Management Accounting: '610

IDustration: 7.26
The information as per Standard Cost Card is given below:
Labour rate: 25 paise per hour.
Time set per unit for production: 2 ~ hours
The actual production data. is given below:
Units produced: 210
Labour rate: 28 paise per hour
Hours worked: 580
Calculate the Labour Cost Variance [Kuvempu Uni., B.Com., May 1992J

Solution:

Standard Time for} = l210 umts


. x 2.5 hours] = 525 hours
ActuaI 0 utput

Labour Cost } _ rStandard Time for Standard! (Actual Time for Actual!
Variance, LCV - l
Actual Output x Rate J- Actual Output x Time J
= [525 hours x Re 0.25] - [580 hours x Re 0.28]
= [Rs.131.25 - Rs.162.4] = Rs.31.15 (A)

Labour Rate} = r Number of Hours Worked) x (Standard _ Actuaii


Variance, LRV Lfor Actual Output J
Rate Rate J
= 580 hours x [Re 0.25 - Re 0.28]
= [580 hours x Re 0.03] =Rs. 17.4 (A)
Labour Efficiency}_ (standard Time for Actual Time for! [Standard I
Variance, LEV - LActual Output - Actual Output x J
Rate)

= [525 hours - 580 hours] x Re 0.25


=[55 hours x Re 0.25] =Rs.13.75 (A)
Verification: LCV =LRV + LEV
Rs. 31.15 (A) =Rs.17.4 (A) + Rs.13.75 (A) =RS.31.15 (A)
Standard .(iosting : 611

mustration: 7.27
Using the following information, calculate Labour Variance.
Gross direct wages: Rs.3,OOO
Standard hours produced: 1,600 hours
Standard rate per hour: Rs.I.50
Actual hours given: 1,500 hours [Mangalore Uni, B.Com, May 2000J
Solution:

Labour cost} _ (Standard Time for Standardl (ActUal Time for Actuall
Variance. LCV - l
Actual Output x Rate J- Actual Output x TimeJ
=:= [1,600 hours x Rs.1.5] - [Rs.3.000]
=[Rs.2.400 - Rs.3,OOO] =Rs.600 (A)
Labour Rate} = (Total Num~er of Labour hour~ x rStandard _ Actua~
Variance, LRV l
Worked for Actual Output Rate J lRate J
= 1,500 hours x (RS.l.5 - Rs.3,OOO
1,500 hours
J
= 1,500 hours x [Rs.1.5 - Rs.2] = [1,500 hours x Re.0.5] = Rs.750 (A)

Labour EffiCiency} _ rStandard Time for Actual Time fori x rStandard I


Variance. LEV - L
Actual Output - Actual Output Rate J L J
= [(1,600 hours - 1,500 hours) x Rs.1.5]
= [100 hours x Rs.1.5] = Rs.150 (F)
Verification: LCV = LRV + LEV
Rs.600 (A) = Rs.750 (A) + Rs.ISO (F) =Rs.600 (A)
mustration: 7.28
The following information is obtained from a Standard Cost Card:
Labour rate: Rs.l.80 per hour
Hours: 4 hours per unit
Actual production data are:
Units produced: 400 units
Management Accounting- ; "612

Lobour rate: Rs.1.90 per hour


Hours worked: 1,500
Calculate: (a) Labour Cost Variance, (b) Labour Rate Variance, and (c) Labour Efficiency
Variance. [Bangalore Uni., B.Com., October 2002J
Solution:

Labour Cost } _ (Standard Time for x Standard!_[Actual Time for x Actual!


Variance. LCV - L
Actual Output Rate J
Actual Output Rate J
= [(4 hours per unit x 400 units) x Rs.1.8] - [1,500 hours x Rs.1.9]
=[1,600 hours x Rs.1.8] - [Rs.2,850] = [Rs.2,880 - Rs.2,85'l] =Rs.30 (F)
Labour Rate.1_ [ Labour Hours worked] [Standard Actual!
Variance, LRVf - for Actual Output x Rate - Rate J
= 1,500 hours x [Rs.1.8 - Rs.1.9] =[1,500 hours x Re 0.1] =Rs.150 (A)
Labo~r EfficienCY} _ (Standard Time for Actual Time fo~ (Standard!
Variance, LEV - L Actual Output - Actual Output J x L Rate J
= [1,600 hours - 1,500 hours] x Rs.1.8 = [100 hours x Rs.1.8] = Rs.180 (F)
Verification: LCV = LRV + LEV
Rs.30 (F) = Rs.150 (A) + Rs.180 (F) = Rs.30 (F)
mustration: 7.29
From the following data, calculate Overhead Variance.
Budgeted Actual
Number of working days 20 21
Man-hours in period 40,000 43,000
Output per man-hour in units 3.2 3
Total output in units 1,28,000 1,29,000
Overheads: Variable Rs. 1,02,400 Rs. 1,14,400
Fixed Rs. 25,600 Rs. 25,000
[Bangalore Uni, B.Com, November 2000J
Standard Costing: 613

Solution:
Budgeted Quantity (output) = 1,28,000 units
Standard Quantity =[43,000 hours x 3.2 units per hour] =1,37,600 units

Revised Budgetedl = 1,28,000 units x 21day J = 1,34,400 units


Quantity f 20 days J
Variable Overheadl_ rStandard Variable Cost Actuall (Actual Variabl~
Variance, VOV f-
l per unit of Output x Outpu~ - l Overhead j

Rs.1,02,400 . J 00
= ( 1.28.000 Units x 1,29,000 UDlt) - [Rs.l,14,4 ]

=[Rs.l,03,200 - Rs.l,14,400] =Rs.ll,200 (A)


Variable Overhead Expen} _ r Standard Variable ActUal] _ [Actual Variablel
-diture Variance, VOEV - lOverhead per Hour x Hours Overhead j

= r
L40,000
Rs.l,02,400 J -[1,14,400]
hours x 43,000 hOU
J
= [Rs.l,lO,080 - Rs.1.14,400] = Rs.4,320 (A)

Van.able Overhead
.
EffiCiency .
Vanance,
}
V0 E =
[ Standard hours for Actual]
Actual Output - Hours x
[ 0Standard
ver
h dR
R ea ecovery
H
J
Variable
ate per our

40,000 hours 1r Rs.l,02,400 l


= 1,28,000units
( x 1,29,000 units-43,OOO hoursj x l40,000hours)

= [40,312.5 hours - 43,000 hours] x [Rs.2.56]


=[2,687.5 hours x Rs.2.56] =Rs.6,880 (A)
Verificatioa: VOV = VOEV +VOEfV
Rs.ll,200 (A) =Rs.4,320 (A) + Rs.6.880 (A) =Rs.ll,200 (A)

Vanance. FOCV
r
Fixed .Overhead cost} = Actual x Standard Fixed Overhead
lOutppt Recovery Rate per Unit j
1_ [Actual Fixed]
Overhead
Management Accounting : 614

= (1,29,000 units x Rs.25,600


1,28,000 units
JJ - [Rs.25,OOO]
=[1,29,000 units x Re 0.2] - [Rs.25,OOO]
=[Rs.25,800 - Rs.25,000] =Rs.800 (F)
Fi~ed Ove~head EXpen} _ (BUdgeted Standard Fixed Overhe~,_[ActUal Fixedl
diture Vanance, FOEV - Output x Recovery Rate per Urnt J
Overhead J
• .
= [ 1,28,000 urnts x
Rs.25,600
1,28,000 units
JJ - [Rs.25,000]
=[1,28,000 units x Re 0.2] - [Rs.25,000]
= [Rs.25,600 - Rs.25,000] = Rs.600 (F)

Fixed Overhead Volume1. =[BUdgeted _ Actual! x [Standard Fixed Overhe.ad!


Variance, FOVV f Output Outpu0 Recovery Rate per umt J
=[1,28,000 units - 1,29,000 units] x [Re 0.2]
= [1,000 units x Re 0.2] = Rs.200 (F)

Fixed Overhead EfficienCY} rStandard Hours for Actuall rStandard Fixed Overhead!
Variance. FOEtV =l
Actual Output - Hour~ x LRecovery Rate per Hour J

=
~[
40'000 hours.
1,28,000 umts
J
x 1,29,000 units - 43,000 hours
}x
[Rs.25,600
40 000 h
, ours
J
= [40,312.5 hours - 43,000 hours] x [Re 0.64]
=[2,687.5 hours x Re 0.64] =Rs.l,720 (A)
Fixed ~verhead capacity} r Standard Hours for Actual] [Standard Fixed Overhead!
Vanance. FOCaV =LActua) Working Days - Hours x Recovery Rate per Hour J
= {L
r. 40,000 hours
20 days
J
x 21 day) - 43,000 hours
} xLr.40,000
Rs.25,600 l
hoursJ

=[(42,000 hours - 43,000 hours) x Re 0.64]


Standard Costing : 615

= [1,000 hours x Re 0.64] = Rs.640 (F)

Fixed Overhead Calendar} (Number of Excess Standard Fixed I


Variance. FOCIV = Days Worked x Overhead Rate per Dayj

= ~21 days - 20 days) x Rs.25,600 l


l 20 days j
=[Iday x Rs. 1,280] =Rs.l,280 (F)
Verification: FOCV =FOEV + FOVV
Rs.800 (F) =Rs.600 (F) + Rs.200 (F) =Rs.800 (F)
FOVV =FOEfV + FOCaV + FOClV
Rs.200 (F) =Rs.I,720 (A) + Rs.640 (F) + Rs.I,280 (F)
=Rs.I,720 (A) + Rs.I,920 (F) =Rs.200 (F)
IDustration: 7.30
One kg of product K requires two chemicals. A and B. The following are the details of
product K for the month of June.
(a) Standard mix of chemical A 50% and chemical B 50%
(b) Standard price per kg of chemical A Rs. 12 and chemical B Rs. 15
(c) Actual input of chemical B 70 kgs
(d) Actual price per kg of chemical A Rs. 15
(e) Standard normal loss, 10% of total input
(1) Material cost variance, total, Rs. 650 adverse
(g) Material yield variance, total, Rs. 135 adverse
(h) Actual output, 90 kgs
You are required to calculate: (1) Material Mix Variance (total), (2) Material Usage
Variance (total), (3) Material Price Variance (total), (4) actual loss of actual input, (5) actual input
of chemical A and (6) actual price per kg of chemical B. lCA (Int), May 1987J
Solution:
1. Materials required and their costs at standard rates for actual output of 90 kgs of product
K:
Chemical: A 50 kgs x Rs. 12 = Rs. 600
B 50 kgs x Rs. 15 =Rs. 750
Therefore, Total 109 kgs Rs. 1,350
Management Accounting : 616
L~ss: Normal Loss (10%) 10 kgs o
Actual Output 90 kgs Rs.l,350
2. Standard material cost per kg of product K = (Rs. 1,350 + 90 kgs) = Rs. 15

3. Material YielJ-= [ActUal -


Variance J Yield
Standar~
Yield j
x [StanCdard Mau:rial
ost per umt
J
:. Rs. 135 (A) = [(90 kgs - SY) x Rs. IS] = (Rs. 1,350 - 15 SY)
:. 15 SY = 1,350 + 135 = Rs. 1,485
:. SY = (Rs. 1,485 + Rs. 15) = 99 kgs
4. Actual input of chemicals A and B, total [(100 + 90) x 99 kgs] = 110 kgs
5. Since actual input of chemical 'B' is 70 kgs, actual input of chemical A works out to 40
kgs (Le., 110 kgs -70 kgs).

.. 6. Actual Cost of Chemical~ _ 0~~~~d


A and Bused J - lRS. 1,35G
Material Cost
+ Variance (A) of
J = Rs. 2,000
Rs.650

7. Total Cost of Chemicals A and B = Rs.2,000


Less: Actual Cost of Chemical A (40 kgs x Rs. 15) Rs. 600
Therefore, AC,tual Cost of Chemical B (70 kgs) Rs.l,400
:. Actual price per kg of Chemical B = Rs. 20 (Le., Rs. 1,400 + 70 kgs)

8. Material 1
(Standard
Mix VarianceJ = Quantityl
Actual
Qu,antity
J x
(Standard
Unit Price
J
Therefore, MMV, Chemical A: [(50% of 110 kgs - 40 kgs) x Rs. 12]
B: [(50% of 110 kgs -70 kgs) x Rs. 15]
= [(55 kgs - 40 kgs) x Rs. 12] + [(55 kgs - 70 kgs) x Rs. 15]
= [(15 kgs x Rs. 12) + (15 kgs x Rs. 15)]
= Rs. 180 (F) + Rs. 225 (A) = Rs. 45 (A)

9. Material Usage Variance. MUV = MMV + MYV


• = Rs. 45 (A) + Rs. 135 (A) = R\l180 (A)

10. Material Price Variance (MPV) = [AQP (SP - AP)]


Standard Costing: 617

Therefore, MPV of Chemical A: 4O.kgs (Rs. 12 - Rs. 15) = Rs. 120 (A)
B: 70 kgs (Rs. 15 - Rs. 20) = Rs. 350 (A)
Rs.470(A)
11. Actual Loss of Input = [110 kgs input - 90 kgs output] = 20 kgs.
mustration: 7.31
From the data given below, calculate:
(a) Individual material price variance for the two materials M and N assuming that price
variances are calculated at the time of purchase;
(b) Individual material usage variances for materials M and N assuming that there was no
work-in-progress either at the comme,ncement or at the end of the period.
Material M Material N
Quantity Value Quantity Value .
(kgs) (Rs.) (kgs) (Rs.)
Raw material purchases 2,000 4,000 5,000 6,250
Issued to works 2,150 3,950
Works stock of mate~al:
Opening 300 1,000
Closing 200 1,250
Standard Price:
Material M, Rs. 1.90 per kg
Material N, Rs. 1.30 per kg
Standard usage: Product A 1 kg 1 kg
ProductB 0.5 kg 1 kg
Output during the period:
Product A: 1,130 units
Product B: 2,s50 units •
/ICWA (Int), June 1989}
Solution:
1. Calculation of Actual Quantity of Materials Used (kgs):
Material M Material N
.Opening stock 300 1,000
Add: Materials issued 2,150 3,950
Management Accounting: 618

2,450 4,950
Less: Closing stock 200 1,250
Therefore, Actual consumption 2,250 3,700
2. Standard Usage of Materials (kgs):
Material M Material N
ProductA: 1,130 units @ 1 kg and 1 kg 1,130 1,130
B: 2,550 units @ 0.5 kg and 1 kg 1,275 2,550
2,405 3,680

{Sta~dard AC~al}J
3. Matena. 1 p.
nce } -- [Actual
f MQuantity
.aI _
Variance, MPV Pu~Chas~~AQP Pnce Pnce

MPV of M =[2,000 kgs x (Rs. 1.90 - Rs. 2)] =(2,000 kgs x Re. 0.1) = Rs. 200 (A)
N = [5,000 kgs x (Rs. 1.30 - Rs. 1.25)] = (5,000 kgs x Re. 0.05) =Rs. 250 (F)
Rs. 50 (F)

4. Material } [Standard Quantity of Actual Quantity Of]


Usage = Material for Actual Material for Actual x
Variance, MUV Production Production

MUV of M = [(2,405 kgs - 2,250 kgs) x Rs. 1.90] = (155 kgs x Rs. 1.90) = Rs. 294.5 (F)
N =[(3,680 kgs - 3,700 kgs) x Rs. 1.30] =(20 kgs x Rs. 1.30) =Rs. 26.0 (A)
Rs. 268.5 (F)
Dlustration: 7.32
The standard cost of a chemical mixture is 40% material A at Rs. 20 per kg; and 60%
material B at Rs. 30 per kg.' A standard loss of 10% is expected in production. During a period,
these are used as below.
90 kgs material A at a cost ofRs. 18 per kg;
110 kgs material B at a cost of Rs. 34 per kg;
The weight produced is 182 kgs of good product. Calculate: (a) Material Price Variance;
(b) Material Mix Variance; (c) Material Yield Variance; and (d) Material Cost Variance.
[ICWA (Int), June 1978J
Standard Costing: 619

Solution:

(a) Material price} _ [{Standard


Variance, MPV - Price -
ActUal}
P'
nce
Actual Qu~tity
of Matenal
J
Purchased, AQP

:. MPV of Material A = (Rs. 20 - Rs. 18) 90 kgs] = (Rs. 2 x 90 kgs) = Rs. 180 (F)
,.'

B = [(Rs. 30 - Rs. 34) 110 kgs) = (Rs. 4 x 110 kgs) = Rs. 440 (A)
Therefore, MPV = Rs. 260 (A)

(b) Ma.terial Mix 1=


Vanance, MMVJ
[::~~~
Q .
uantlty
- Actual J
Quantity
x [ Stan.dard]
Pnce

MMV of A = [t~o x 200} - 90 kgS] x Rs. 20 = (80 - 90) x Rs. 20] = Rs. 200 (A)

6
B = ({1 g0 x 200} - 110 kgs ]x Rs. 30 = (120 - 110) x Rs. 30] = Rs. 300 (F)
Therefore, MMV = Rs. 100 (F)

(c) Standard Yield = (200 kgs of Actual Input - 10% Standard Loss)
= (200 - 10% of 200) = (200 kgs - 20 kgs) = 180 kgs
Standard Material1_ [[(A: 80 kgs x Rs. 20) + (B: 120 kgs x Rs. 30)] "')
Cost per Unit J - 180 kgs of standard yield :J
-l 180 kgs J
r
_ rRs. 1,600 + Rs. 3,600) _ Rs. 5,200l_ R 28 89
-CI80 kgs J-
s. .

Material Yield } _ [Actual Standard


Variance, (MYV) - Yield - Yield) x
'I [ Standard Material ]
Cost per unit

= [(182 kgs - 180 kgs) x Rs. 28.89] = (2 kgs x Rs. 28.89) = Rs. 57.78 (F)
(d) Material Cost Variance, MCV = MPV + MMV + MYV
= [Rs. 260 (A) + Rs. 100 (F) + Rs. 57.78 (F))
= Rs. 260 (A) + Rs. 157.78 (F) = Rs. 102.22 (A)
Management Accounting : 620
Illustration: 7.33
The following information is available from the books of a manufacturing company
which uses three types of materials for production.
I Standard Actual
Material Quantity Price Total . Quantity Price Total
(kgs) (Rs.) (Rs.) (kgs) (Rs.) (Rs.)
X 2,500 6.00 15,oao 2,000 6.0 12,000
Y 2,000 3.75 7,500 2,500 3.6 9,000
i Z 1,500 3.00 4,500 2,000 2.8 5,600
6,000 6,500
Less: 10% Normal Loss 600 1,100 (Actual Loss)
5,400 27,000 5,400 26,600

Calculate: (a) Material Cost Variance, (b) Material Price Variance, (c) M~terial Mix Variance,
(d) Material Yield Variance, and (e) Total
. . Usage Variance. [M.Com., UoM, May
Material .
1974J
Solution:
Material cost} [Stan~ard
~ ~
Actual
St~dard _ Quantity of x Actual)
Vanance MCV = Quantity of x Price
, Material Price Material
,
"

. ','.
MCV;'of X =[(2,500 kgs x Rs. 6) - (2,000 kgs x Rs. 6)]
=(Rs. 15,000 - Rs.12,000) =Rs. 3,000 (F)
Y =[(2,000 kgs x Rs. 3.75) - (2,500 kgs x Rs. 3.6)]
=(Rs. 7,500 - Rs. 9,000) = Rs. 1,500 (A)
Z = [(1,500 kgs x Rs. 3) - (2,000 kgs x Rs. 2.8)]
=(Rs. 4,500 - Rs. 5,600) = Rs. 1,100 (A)
Rs. 400 (F)

Material Price} _ [Standard


Variance, MPV - Price
_ Actual
Price
J x
(Actual Quantity of
Material Purchased
J
MPV of X =[(Rs. 6 - _Rs. 6) 2,000 kgs] =(Rs. 0 x 2.000 kgs) =Rs. 0
Y =[(Rs. 3.75 - Rs. 3.6) 2,500 kgs] =(Re. 0.15 x 2,500 kgs) =Rs. 375 (F)
Z =[(Rs. 3 - Rs. 2.8) 2,000 kgs] =(Re. 0.2 x 2,000 kgs) =Rs. 400 (F)
Rs. 775 (F)
Standard Costing: 621

Material usage} _
Standard Quantity of
Material for Actual -
Actual Quantity of
Material for Actual
Jx
Variance, MUV -
L Production Production

MUV of x =[(2,500 kgs - 2,000 kgs) x Rs. 6] =(500 kgs x Rs. 6) = Rs. 3,000 (F)
Y = [(2,000 kgs - 2,500 kgs) x Rs. 3.75] = (500 kgs x Rs. 3.75) = Rs. 1,875 (A)
Z = [(1,500 kgs - 2,000 kgs) x Rs. 3] = (500 kgs x Rs. 3) = Rs. 1,500 (A)
Rs. 375 (A)
Verification, MeV = MPV +MUV
Total: Rs. 400 (F) =[Rs. 775 (F) + Rs. 375 (A)] = Rs. 400 (F)
X: Rs. 3,000 (F) = [0 + Rs. 3,000 (F)] = Rs. 3,000 (F)
Y: Rs. 1,500 (A) = [Rs. 375 (F) + Rs. 1,875 (A)] = Rs. 1,500 (A)
Z: Rs. 1,100 (A) =[Rs. 400 (F) + Rs. 1,500 (A)] = Rs. 1,100 (A)

Total Weight of
Revised Standard Actual Mix Standard Quantity
Quantity, RSQ } = Total Weight of x of a Material
Standard Mix

RSQ of X = ( 6,500 kgs J


6,000 kgs x 2,500 kgs = 2,708.33 kgs

6,500 kgs
Y = ( 6,000 kgs x 2,000 kgs
J= 2,166.67 kgs

- (6,500
Z -. 6,000 kgs J-
kgs x 1,500 kgs - 1,625.00 kgs
= 6,500.00 kgs

Material MiX} [ Revised


·
V anance, MMV = Standard
Q '
- Actu~ I
Quantit~
x (Stan.dard
Price
J
uantlty

MMVof x = [(2,708.33 kgs - 2,000 kgs) x Rs. 6] = (708.33 kgs x Rs. 6) = Rs. 4,250 (F)
Y = [(2,166.67 kgs - 2,500 kgs) x Rs. 3.75] = (333.33 kgs x Rs. 3.75) = Rs. 1,250 (A)
Z = [(1,625 kgs - 2,000 kgs) x Rs. 3] = (375 kgs x Rs. 3) = Rs: 1,125 (A)
Rs. 1,875 (F)
Management Accounting : 622

S~dardl = (5,400 kgs of Standard Output x 6,500 kgs of Actual Inpu~J = 5,850 kgs
YIeld J L 6,000 kgs of Input
Standard Material Cost per Unit = [Rs. 27,000 + 5,400 kgs] = Rs. 5

Standar~
Material Yield } _ [Actual _
Variance, MYV - Yield Yield J x
[Standard Material]'
Cost per unit

= [(5,400 kgs - 5,850 kgs) x Rs. 5] = (450 kgs x Rs. 5) = Rs. 2,250 (A)
mustration: 7.34
Space Manufacturers produce a standard product, the standard material cost of which is
Rs. 20 per unit. The following information is obtained from the costing records.
(a) Standard Mix:
Materials Quantity (units) Rate (Rs.) Amount (Rs.)
X 70 20 1,400
Y 30 10 300
100 1,700
Less: (15%) 15
85 1,700

(b) Actual Results for September 1974:


Materials Units Rs. Rs.
X' 400 22 8,800
Y 200 12 2,400
600 11.200
Less: (10%) 60
--
540 11.200
You are required to compute (a) Material Price Variance; (b) Material Mix Variance; (c)
Material Yield Variance; and (d) Material Usage Variance. [M.Com., UoMJ
Standard Costing: 623

Solution:
Standard Actual
Particulars Quantity Price Cost Quantity Price Cost
(Units) (Rs.) (Rs.) (Units) (Rs.) (Rs.)
Mat X [(70/85) x 540] = 445 20 8,900 400 22 8,800
Y [(30/85) x 540] = 190 10 1,900 200 12 2,400
Total 635 10.800 600 11,200
Less: 15% and 10% 95 - 60 -
Output 540 10,800 540
. 1 ],200

t
ActUal
Material cost} (Standard Standard .

J
ActUal]
Variance, MCV = QMuanti~al0f x Price - Quantl~ of x Price
aten Matenal

MCVof X = [(445 units x Rs. 20) - (400 units x Rs. 22)]


= (Rs. 8,900 - Rs. 8.800) = Rs. 100 (F)
Y = [(190 units x Rs. 10) - (200 units x Rs. 12)]
= (Rs. 1,900 - Rs. 2.400) =Rs. 500 (A)
Rs. 400 (A)

Material Price} _ (Standard


Variance, MPV - Price
_ Actual
Price
J x
(Actual Quantity of
Material Purchased
J
MPV of X =(Rs. 20 - Rs. 22) 400 units] =(Rs. 2 x 400 units) = Rs. 800 (A)
Y = (Rs. 10 - Rs. 12) 200 units] = (Rs. 2 x 200 units) = Rs. 400 (A)
Rs. 1,200 (A)

Material usage} _ ~tand~d Quantity of Actual Quantity oj


Material for Actual x
Variance MUV - Matenal for Actual -
, Production Production

MUVof X = [(445 units - 400 units) x Rs. 20] =(45 units x Rs. 20) = Rs. 900 (F)
Y = [(190 units - 200 units) x Rs. 10] = (10 units x Rs. 10) = Rs. 100 (A)
Rs. 800 (F)

Revised Standard}_
Total Weight of
Actual Mix Standard Quantit~
Quantity, RSQ - Total Weight of
x
of a Material J
Standard Mix
,i
(

Management Accounting : 624

RSQ 'of X = (~~~ :::: ~5 units J = 420 units


X

Y= (~~~ :~!: x 190 u~ts J = 180. units


600 units

Material MiX} [ Revised·


· MMV = Standard
_ Actu~ I x (Stan.dardJ
Vanance, Q ' QuantIt~ Price
uantlty

MMVof X =[(420 units - 400 units) x Rs. 20] =(20 units x Rs. 20) = Rs. 4,00 (F)
Y =[(180 units - 200 units) x Rs. 10] =(20 units x Rs. 10) .= Rs. 200,(A)
Rs. 200 (F)
Standard Material Cost per Unit =[Rs. 10,800 + 540 units] =Rs. 20

Standardl_ ( 85 units, of Output


Yield J- . J
100 units of Input x 600 umts of Input or (~~ x 600 J = 510 units

Material Yield} _ [Actual _ Standardl


Variaqce, MYV - Yield' Yield x
( Standard Material
J
Cost per unit
J
=[(540 units - 510 units) x Rs. 20] =(30 units x Rs. 20) =Rs. 600 (F)
IDustration: 7.35
From the following data for May 1981 of a factory, calculate (a) Material Cost Variance, ..
(b) Material Price Variance, (c) Material Usage Variance, (d) Material Mixture Variance and (e)
Material Yield Variance.
Standard Actual
Material
Kgs Rate (Rs.) Kgs Rate (Rs.)
X 8,000 1.05 7,500 1.20
Y 3,000 2.15 3,300 2.30
Z ·2,000 3.30 2,400 3.50
[ICWA (lnt), December 1981J
Standard Costing : 625

Solution:
Material Cost
Standard
Quantity of x
Stan.d ar d~ - ~Quantity
Actual of x Actual]
Variance, MCV [ Pnce M ' I Price
Material atena

MCVof X = [(8,000 kgs x Rs. 1.05) - (7,500 kgs x Rs. 1.2)]


= (Rs. 8,400 - Rs. 9,000) = Rs. 600 (A)
Y = [(3,000 kgs x Rs. 2.15) - (3,300 kgs x Rs. 2.30)]
=
(Rs. 6,450 - Rs. 7,590) =Rs. 1,140 (A)
Z = [(2,000 kgs x Rs. 3.30) - (2,400 kgs x Rs. 3.5)]
= (Rs. 6,600 - Rs. 8,4(0) = Rs. 1,800 (A)
Rs. 3,540 (A)

Material Price} _ [Standard


Variance, MPV - Price
_ Actual
Price
Jx
(Actual Quantity of
Material Purchased
J
:MPV of X =(Rs. 1.05 - Rs. 1.20) 7,500 kgs] =(Re. 0.15 x 7,500 kgs) = Rs. 1,125 (A)
Y =(Rs. 2.15 - Rs. 2.30) 3,300 kgs] =(Re. 0.15 x 3,300 kgs) = Rs. 495 (A)
Z = (Rs. 3.30 - Rs. 3.50) 2,400 kgs] = (Re. 0.2 x 2,400 kgs) =Rs. 480 (A)
Rs. 2,100 (A)

Material usage}....::.
· MUV ....-
Ltand~d Quantity of
Matenal for Actual -
. Actual Quantity Of]
Material for Actual x
V anance,· .
Production Production

MUVof X =[(8,000 kgs - 7,500 kgs) x Rs. 1.05] =(500 kgs x Rs. 1.05) =Rs. 525 (F)
Y =[(3,000 kgs - 3,300 kgs) x Rs. 2.15] =(300 kgs x Rs. 2.15) =Rs. 645 (A)
Z =[(2,000 kgs - 2,400 kgs) x Rs. 3.30] =(400 kgs x Rs. 3.30) =Rs. 1,320 (A)
Rs. i ,440 (A)

Total Weight of
Revised Standard Actual Mix Standard Quantity
Quantity, RSQ } = Total Weight of x of a Material
Standard Mix

_ (13,200 kgs
RSQ of X - 13,000 kgs x 8,000 kg) -
J_ 8,123 kgs

_ [13,200 kgs
Y- 13,000 kgs x 3,000 kg) -
J_ 3,047 kgs
Management Accounting : 626

z=
13,200 kgs J_
[ 13,000 kgs X 2,000 kgs) - 2,030 kgs
= 13,200 kgs

M aten'al M'IX }
· MMV =
( Revised
Standard
_ ActU~11 x [Stan.dard]
Vanance, Q' Quantlt~ Pnce
uantIty

MMVof X= [(8,123 kgs - 7,500 kgs) x Rs. l.05] = (623 kgs x Rs. 1.05) =Rs. 654 (F)
Y = [(3,047 kgs - 3,300 kgs) x Rs. 2.15] = (253 kgs x Rs. 2.15) =Rs. 544 (A)
Z =[(2,030 kgs - 2,400 kgs) x Rs. 3.30] =(370 kgs x Rs. 3.30) =Rs. 1,221 (A)
Rs. 1,111 (A)

Material Yield } _ [Actual _ Standardl


Variance, MYV - Yield Yield x
[Standard Material
J
Cost per unit
J OR

= [Standard _ Standard Quantit~ (Standardl


Quantity of Revised Mixj x lPrice j
MYVof X = [(8.000 kgs - 8,123 kgs) x Rs. 1.05] = (123 kgs x Rs. l.q5) = Rs. 129 (A)
Y = [(3.000 kgs - 3,047 kgs) x Rs. 2.15] = (47 kgs x Rs. 2.15) = Rs. 101 (A)
Z = [(2,000 kgs - 2,030 kgs) x Rs. 3.30] = (30 kgs x Rs. 3.3) = Rs. 99 (A)
Rs. 329 (A)
Illustration: 7.36
100 skilled workmen, 40 semi-skilled workmen and 60 unskilled workmen were to work
for 30 weeks to get a contract job completed. The standard weekly wages were Rs. 60, Rs. 36 and
Rs. 24 respectively. The job was actually completed in 32 weeks by 80 skilled, 50 semi-skilled
and 70 unskilled workmen who were paid Rs. 65, Rs. 40 and Rs. 20 respectively as weekly
wages. Find out the Labour Cost Valiance, Labour Rate Variance, Labour Mix Variance and
Labour Efficiency Variance. [ICWA (lnt), June 1992J
~nda", """'ng , 627

Solution:
Standard Actual

Type of Number Standard Number Standard


Total Total
employees of number Rate of number Rate
Cost Cost
employ- of man- (Rs.) employ- of man- (Rs.)
(Rs.) (Rs.)
yees weeks yees weeks
Skilled 100 3,000 60 1,80,000 80 2,560 65 1,66,400
Semi- 40 1,200 36 43,200 50 1,600 40 64,000
skilled
60 1,800 24 43,200 70 2,240 20 44,800
Unskilled
6,000 2,66,400 6,400 2,75,200
(Note: Time is expressed in weeks in this problem)

Labour Cost
Variance, LCV
}= Standard
Time for x
Actual
Standard
Wage Rate
per Man-
J _
Actual
Time for
Actual
x
Actual
V!age Rate
per Man-
J
[ [
Output week Output week

LCV of Skilled = [(3,000 man-weeks x Rs. 60) - (2,560 man-weeks x Rs. 65)]
= (Rs. 1,80,000 - Rs. 1,66, 400) = Rs. 13,600 (F)
Semi-skilled = [(1,200 man-weeks x Rs. 36) - (1,600 man-weeks x Rs. 40)]
= (Rs. 43,200 - Rs. 64,000) = Rs. 20,800 (A)
Unskilled = [(1,800 man-weeks x Rs. 24) - (2,240 man-weeks x Rs. 20)]
=(Rs. 43,200 - Rs. 44,800) = . Rs. 1,600 (A)
Rs. 8,800 (A)

Labour Rate l [ Standard Wage . I Wage


Actua
x
J
[Total Number Of]
Man-weeks for
Variance. LRVf = Rate per Man-week - Ra te per Man-week ActuaI 0 utput

LRV of Skilled =[(Rs. 60 - Rs. 65) x 2,560 man-weeks]


=(Rs. 5 x 2,560 man-weeks) =Rs. 12,800 (A)
Semi-skilled = [(Rs. 36 - Rs. 40) x 1,600 man-weeks]
=(Rs. 4 x 1,600 man-weeks) =Rs. 6,400 (A)
Unskilled = [(Rs. 24 - Rs. 20) x 2,240 man-weeks]
=(Rs. 4 x 2,240 man-week) = Rs. 8,960 (F)
Rs. 10,240 (A)
Management Accounting 628

Labour EfficiencYl_ (Standard Time for _ Actual Time fo~ x [Standard Wage Rate]
Variance, LEV J- L
Actual Output Actual Output J per Man-week

LEV of Skilled =[(3,000 man-weeks - 2,560 man-weeks) x Rs. 60]


=(440 man-weeks x Rs. 60) =Rs. 26,400 (F)
Semi-skilled = [(1,200 man-weeks - 1,600 man-weeks) x Rs. 36]
= (400 man-weeks x Rs. 36) = Rs. 14,400 (A)
Unskilled = [(1,800 man-weeks - 2,240 man-weeks) x Rs. 24]
= (440 man-weeks x Rs. 24) = Rs. 10,560 (A)
Rs. 1,440 (F)

Total Time of
~:~~~ = _..!..A~c::::tu::::a~l:..:.M~ix~_ x
Standard TimeJ
of a Particular
Time, RST} [ Total Time of Grade of Labour
Standard Mix

RST of Skilled = [ 6.400 man-weeks


6,000 man-weeks
x 3,000 man-weeks J = 3,200 man-weeks

Semi-Skilled = ( 6,400 man-weeks


6,000 man-weeks
x 1,200 man-weeks J= 1,280 man-weeks
Unskilled = [ 6,400 man-weeks
6,000 man-weeks
x 1,800 man-weeks J = 1,920 man-weeks
6,400 man-weeks

Lb· } [ReVised Stand.ard _ Actual] x [Standard Wage Ratel


a our Mlx = Time of Actual Time
Variance, LMV Time Worked per Man-week J
LMVof Skilled =[(3,200 man-weeks - 2,560 man-weeks) x Rs. 60]
=(640 man-weeks x Rs. 60) = Rs. 38,400 (F)
Semi-skilled = [(1,280 man-weeks - 1,600 man-weeks) x Rs. 36]
=(320 man-weeks x Rs. 36) = Rs. 11,520 (A)
Unskilled = [(1,920 man-weeks - 2,240 man-weeks) x Rs. 24]
=(320 man-weeks x Rs. 24) = Rs. 7,680 (A)
Rs. 19,200 (F)
Standard Costing : 629

Dlustration: 7.37
From the data given below, calculate three labour cost variances for each of the two
departments.
Department A Department B
Actual gross wages (direct) Rs.2,000 Rs.l,800
Standard hours produced 8,000 6,000
Standard rate per hour (paise) 30 35
Actual hours worked 8,200 5,800
[ICWA (Int), December 1975J
Solution:

Labour Cost
Variance. Lev
}= (Standard Hours
Produced
Standard"')
x Wage RateJ -
[Actual Hours
Worked x
Actual wage]
Rate
LCV of A =[(8,000 hours x Re. 0.3) - (Rs. 2,000)] =(Rs. 2,400 - Rs. 2,000) =Rs. 400 (F)
B =[(6,000 hours x Re. 0.35) - (Rs. 1,800)] =(Rs. 2,100 - Rs. 1,800) =Rs. 300 (F)
Rs. 700 (F)

ctUal Number Of~


La~our Rate } _ ( Standard Hourly _ Actual HOUrlY] x
Varj.ance, LRV - Wage Rate Wage Rate
tHours Worked for
Actual Output

LRV of A = (Re. 0.30 - Rs. 2,000


8,200 hours
J x 8,200 hours

=[(Re. O.~O - Re. 0.244) x 8,200 hours] =(Re. 0.056 x 8,200 hours) =Rs. 460 (F)

B = r Re . 0.35 -
l Rs. 1,800
5,800 hours
.J x 5,800 hours

=[(Re. 0.35 - Re. 0.31) x 5,800 hours] =(Re. 0.04 x 5,800 hours) =Rs. 230 (F)
Rs. 690 (F)
Labo~r EfficienCY} = rStandard Hours for _ Actua~ x rStandard Hourly~
Vanance, LEV L Actual Output Hours J L Wage Rate J
LEV of A =[(8,000 hours.,.. 8,200 hours) x Re. 0.3] =(200 hours x Re. 0.3) =Rs. 60 (A)
B =[(6,000 hours - 5,800 hours) x Re. 0.35] =(200 hours x Re. 0.35) = Rs. 70 (F)
Rs. 10 (F)
Management Accounting : 630

IDustration: 7.38
From the following comparative statement for the years 1961 to 1962; (a) find out
whether the year 1962 showed an average better performance or otherwise; and (b) possible
causes of difference in performance. (Assume production of only one quality and same machinery
conditions in both years).
1961 1962
Wages incurred (Rs.) 2,80,000 5,10,000
Units produced 14,000 25,000
Average number of workers 225 400
[ICWA (lnt), January 1964J
Solution:
c omparative PerIiormance E vaIuabon Statement
Increase
Particulars 1961 1962
(or Decrease)
I. (a) Wages (Rs.) 2,80,000 5,10,000 82.14%
(b) Unit Produced 14,000 25,000 78.57%
II. (a) Wages per Umt. (Rs.) =[ Wages
U· PIncurred
d d
mts ro uce
J 20 20.4 2%

(b) Average Wages per Worker (Rs.)


( Wages Incurred
= Average No. of Workers
J 1,244.44 1,275 2.46%

(c) OUGU' per Worker (units)


Units Produced
= Average No. of Workers
J .- 62.22 62.5 0.45%

From the above, it is obvious that the rate of increase in the output was lower than that of
wages. Consequently, average wages per employee increased at a higher rate than the output per
employee. Therefore, unit labour cost increased at a higher rate. It can, therefore, be concluded
that 1962 did not show an overall better performance. It is due to, as identified above, greater
than proportional increase in wages when compared to output.
IDustration: 7.39
The direct labour strength of a section of an Engineering Factory is 100 workers paid at
the rate of Rs. 6 per day of 8 hours each. The normal production is 1,000 pieces per week of 48
hours. During a particular week, an order for 1,500 pieces was completed expending, in all, 7,650
hours made up 6,300 hours at normal wages and, 1,350 hours at overtime wages at double rate.
Standard Costing: 631

The total wages came to Rs. 6,300. Calculate the average labour cost per piece during the week
and analyse the Labour Cost Variance for the week. [ICWA (Fin), December 1982J
Solution:
(a) Actual wages paid: Rs. 6,300
Number of pieces completed: 1,500

... Average Labour} Rs. 6,300 J


rL.t ,500
Cost per Piece = pieces = Rs. 4.2

Standard Labour CostL [100 workers x Rs. 6 per Day x 6 Days a Week
of Actual Output J= 1,000 Pieces of Normal Production
"I ,500 Pieces
x of Actual Output
J
=Rs. 5,400
Labour cost} [Standard Labour Cost
Variance, LCV = for the Actual Output -
Actual
Labour Cost
J
=(Rs. 5,400 - Rs. 6,300) =Rs. 900 (A)

Number of Labour Hour

J
Labour Rate } _ [Standard Hourly _ Actual HOUrlY] x Worked for Actual
Variance, LRV - Wage Rate Wage Rate [ Output

=L
rRs. 6 per8day of 8 hours
hours
Rs. 6,300
7,650 hours
J x
[7 650 h
, ours
]

= [(Re. 0.75 - Re. 0.8235) x 7,650 hours] = (Re. 0.0735 x 7,650 hours)
= Rs. 562 (A)

Labour EfficienCY} _ (Standard Hours for _ Actuai, x (Standard HOUrlY]


Variance, LEV - LActual Output Hours J L Wage Rate

= [{ 48 hours x 100 workers


1,000 pieces
.} J
x 1,500 pIeces - 7,650 hour~ x Re. 0.75

= [(7,200 hours -7,650 hours) x Re. 0.75] = (450 hours x Re. 0.75)
= Rs. 338 (A)
mustration: 7.40
The standard cost sheet for producing a job consisting of 100 articles for the NMC Ltd is
appended.
Management Accounting : 632
Material: 66 kgs of A at Rs. 10 per kg
50 kgs of B at Rs. 12 per kg
Direct Wages: 20 hours Operation 1 at Rs. 9 per hour
30 hours Operation 2 at Rs. 12 per hO':lr
40 hours Operation 3 at Rs. 16 per hour
Actual cost of the job ,were:
Materials: 70 kgs of A at Rs. 10.50 per kg
48 kgs of Bat Rs. 13 per kg
Direct Wages: 25 hours Operation 1 at Rs. 8 per hour
28 hours Operation 2 at Rs. 12 per hour
40 hours Operation 3 at Rs. 15.50 per hour
Prepare a table to show: (a) The standard and actual cost of the job; and (b) The variances
analysed as between quantity and price. [CS (Fin), December 1978J
Solution:
comPJlrative cost Statement
Standard Ac~al Difference
Particulars
Rs. Rs. Rs.
Direct Material:
A: [60 kgs x Rs. 10]; [70 kgs x Rs. 10.50] 600 735 135 (A)
B: [50 kgs x Rs. 12]; [48 kgs x Rs. 13] 600 624 24 (A)
(a) 1,200 1,359 159 (A)
Direct Wages: Operation: "
.
1: [20 hours x Rs. 9]; [25 hours x Rs. 8] 180 200 .20 (A)
2: [30 hours x Rs. 12]; [28 hours x Rs. 12] 360 336 24 (F)
3: [40 hours x Rs. 16]; [40 hours x Rs. 15.5] 640 620 20 (F)
(b) 1,180 1,156 24 (F)
Total Prime Cost 2,380 2,515 135 (A)

Material Cost }
Variance, MCV
(Standard Actual
= l!v1aterihl Cost ~ Material Cost .
J
=(Rs. 1,200 - Rs. 1,359) =Rs. 159 (A)
Standard Costing : 633

Labour Cost1 [Standard Labour cost] r Actual ]


f
Variance, LCV = for the Actual Output -lLabour Cost
= (Rs. 1,180 - Rs. 1.156) = Rs. 24 (F)

Material Price1_ (Standard _ ActUal] [ActUal Quantity of ]


Variance, MPvf - Price Price x Material Purchased

MPV of A = [(Rs. 10 - Rs. 10.5) 70 kgs] = (Re. 0.5 x 70 kgs) =Rs. 35 (A)
B = [(Rs. 12 - Rs. 13) 48 kgs] = (Re. 1 x 48 kgs) =Rs. 48 (A)
Rs. 83 (A)

Material usage} _ ~tand~d Quantity of Actual Quantity Of]


Material for Actual x
Variance MUV - Matenal for Actual -
, Production Production

MUV of A = [(60.kgs -70 kgs) x Rs. 10] = (10 kgs x Rs. 10) = Rs. 100 (A)
B = [(50 kgs - 48 kgs) x Rs. 12] = (2 kgs x Rs. 12) = Rs. 24 (F)
Rs. 76 (A)
Verification, MCV = MPV + MUV
Rs. 159 (A) = Rs. 83 (A) + Rs. 76 (A) = Rs. 159 (A)

Nuffiber of L8bour Hours


La?our Rate
Vanance, LRV } ( _
-
.
Standard Hourly _ Actual Hourly
Wage Rate Wage Rate J L x Worked for Actual
0 t t
u~
J
LRVof 1 = [(Rs. 9 - Rs. 8) x 25 hours] = (Re. 1 x 25 hours) = Rs. 25 (F)
2 = [(Rs. 12 - Rs. 12) x 28 hours] = (Rs. 0 x 28 hours) =Rs. 0
3 = [(Rs. 16 - Rs. 15.5) x 40 hours] = (Re. 0.5 x 40 hours) = Rs. 20 (F)
Rs.45 (F)
Labour EfficienCY} = rStandard Hours for _ Actuall x rStandard HOUrlY]
Variance, LEV l
Actual Output Hours J lWage Rate

LEV of 1 = [(20 hours - 25 hours) x Rs. 9] = (5 hours x Rs. 9) = Rs. 45 (A)


2 = [(30 hours - 28 hours) x Rs. 12] = (2 hours x Rs. 12) =Rs. 24 (F)
Management Accounting : 634
3 = [(40 hours - 40 hours) x Rs. 16] = (0 hours x Rs. 16)= Rs. 0
Rs. 21 (A)
Verification, LCV = LRV + LEV
Rs. 24 (F) = Rs. 45 (F) + Rs. 21 (A) = Rs. 24 (F)
lliustration: 7.41
The following information is available from the records of a factory.
Budget Actual
Fixed overheads for June (Rs.) 10,000 12,000
Production in June (units) 2,000 2,100
Standard time per unit (hours) 10
Actual hours worked in June 22,000
Compute: (a) Fixed Overheads Cost Variance, (b) Expenditure Variance, (c) Volume
Variance, (d) Capacity Variance, and (e) Efficiency Variance. lCA (lnt), November 1989]
Solution:
Standard Fixed Overheadl _ [RS. 10,000
Recovery Rate per Unit J- 2,000 units
J- -
R 5
s.
Standard Hours for Actual Output = (2,100 units x to hours) = 21,000 hours
Standard Fixed Overheadl_ [
Recovery Rate per Hour J-
Rs.10,OOO
(2,000 units x to hours)
J--
R 05
e. .

Fixed Overhead }
Cost Variance, FOCV
[Standard Hours
= for Actual Output x
Standard Fixed
Overhead Recovery
J -
[Actual Fixed ]
Overhead
Rate per unit Expenses
= [(2,100 hours x Rs. 5) - Rs. 12,000]
= (Rs. to,500 - Rs. 12,000) = Rs. 1,500 (A)
Fixed overhead} = rBudgeted Fixed ActualFixe~
Expenditure
Variance, FOEV lOverhead Overhead J
= [(2,000 units x Rs. 5) - Rs. 12,000]
= (Rs. 10,000 - Rs. 12,000) = Rs. 2,000 (A)

Fixed Overhead VOIUm} (BUdgeted


Variance, FOVV =
o tp t -
u. u
0Actual]
uf?u
t
x
[s tan d ard F·lxedOh
ver ead
Recovery Rate per unit
J
(umts) (umts)
Standard Costing: 635

=[(2,000 units - 2,100 units) x Rs. 5] =(100 units x Rs. 5) =Rs. 500 (F)
Verification, FOCV = FOEV + FOVV
Rs. 1,500 (A) =Rs. 2,000 (A) + Rs. 500 (F) = Rs. 1.500 (A)
Fixed Overhead }
Standard Hours ActUal] [ Standard Fixed overhead]
Efficiency
= [ for Actual Output - Hours x Recovery Rate per Hour
Variance, FOEfV
= [(21.000 hours - 22,000 hours) x Re. 0.5]
=(1,000 hours x Re. 0.5) =Rs. 500 (A)
Fixed ~verhead.Capacity
Vanance (WlthOUt
.
Calendar Vanance), FOCaV
} rBUd eted
= Hg
ours
-
;'"Actual J
H
ours
x
.
[ Standard Fixed
Overhead Recovery
Rate per Hour
J
= [(2,000 units x 10 hours) - 22.000 hours] x Re. 0.5
= [(20,000 hours - 22,900 hours) x Re. 0.5]
= (2,000 hours x Re. 0.5) = Rs. 1,000 (A)
mustration: 7.42
In a factory, the standard units of production for the year were fixed at 1,20,000 units and
estimated overhead expenditures were estimated to be: Fixed = Rs. 12,000; Variable = Rs. 6,000;
and Semi variable = Rs. 1,800.
Actual production during April of the year was 8,000 units. Each month has 20 working
days. During the month in question, there was one statutory holiday. The actual overheads
amounted to: Fixed = Rs. 1, 190; Variable = Rs. 480; and Semi variable =Rs. 192. Semi-variable
charges are considered to include 60 per cent expenses of fixed nature and 40 per cent of variable
character. Find out the expenditure, volume and calendar variances. leA (Int), May 1976)
Solution:
(a) Fixed Overheads per year Rs.12,000
Add: 60% of Rs. 1,800 1,080
13,080

Standard Fixed Overhead per Unit =[ 1,~g:dgo~~tSJ = Re. 0.109


(b) Variable Overheads Rs.6,000
Add: 40% of Rs. 1,800 720
6,720

Staridard Variable Overhead per Unit = [ 1.2~~O~~~~itsJ = Re. 0.056


Management Accounting: 636

(c) Overhead Expenses Recovered or Absorbed (April):


Fixed (8,000 units x Re. 0.109) = Rs.872
Variable (8,000 units x Re. 0.056) = 448
1,320
(d) Actual Overhead (April):
Fixed (Rs. 1,190 + 60% of Rs. 192) =(Rs. 1,190 + Rs. 115.2) = Rs.l,,305.2
Variable (Rs. 480 + 40% of 192) =(Rs. 480 + Rs. 76.8) = 556.8
1,862.0

Estimated monthly Fixed Overhead = ( Rs. 13,080 per year


12 months
J = Rs. 1'090
(a) Expenditure Variance:
1. Fixed, J

= [BUdgeted Fixed _
Overhead
Actual
Overhead
J= (Rs. r 090 -
'
Rs. 1 305.2) =Rs. 215.2 (A)
,
2. Variable,
r Standard Variable Actual Variabl~ = (Rs. 448 - Rs. 556.8)
= lOverhead Absorbed - Overhead) = Rs. 108.8 (A)

(b) Value}
Variance =
(Actual
Production -
Standard I
Production) x
rStandard Rate
Lof Absorption
J
= (8000 . _ 1,20,000 units per annum] R 0109
, umts 12 mont hs x e. .

= [(8,000 units - 10,000 units) x Re. 0.109]


= (2,000 units x Re. 0.109) = Rs. 218 (A)

(c) Calendar} _
Variance - l
rStandar~ Number of
Workmg Days
_ Actual Number Of] x
Days Worked
[s~:n~ar~v~~:~j
fD
o ays

- (20 d
- ays -
19 d ) ( Rs. 13,080 p.a
ays x 20 days x 12 months
J-
-
(1 da x Rs. 13,080
y 240 days
J
= (1 day x Rs. 54.5) = Rs. 54.5 (A)
Standard Costing : 637

lliustration: 7.43
The following figures are extracted from the books of a company.
Budget Actual
Output (in units) 6,000 6,500
Hours 3,000 3,300
Overhead costs (Rs.): Fixed 1,200 1,250
Variable 6,000 6,650
Number of days 25 27
Compute and analyse the Overhead Variances [ICWA (Int), December 1977J
Solution:
1. Standard Output per hour =(6,000 units + 3,000 hours) =2 units
Budgeted Quantity (BQ) = 6,000 units
Actual Quantity (AQ) = 6,500 units
Standard Quantity (SQ) =(3,300 hours x 2 units per hour) =6,600 units

Revised Budgeted Quantity =


(6,000 units
L 25 days
J
x 27 days = 6,480 umts
.
2. Budgeted Variable Overhead (BVOH) = Rs. 6,000
Actual Variable Overhead (A VOH) = Rs. 6,650

Variable OverheadL [Standard V~able


Variance, VOV =f Cost per umt of x
Actual]
Output
_ (Actual Variable
L Overhead
J
Output

= (6~;og'~~i~S x 6,500 unitsJ- [Rs. 6,650] = (Rs. 6,500 - Rs. 6,650) = Rs. 150 (A)

3. Variable Ov~rhead Expendi-}


ture VarIance, VOEV
=(Standard Variable
Overhead per Hour
x Actual
Hours
J _ (Actual Variablel
Overhead J

Rs. 6,000
= ( 3,000 hours x,3
3 00 l 66 0
hoursJ - [Rs. ,5 ]

=[(Rs. 2 per hour x 3,300 hours) - Rs. 6,650] =(Rs. 6,600 - Rs. 6,650) =Rs. 50 (A)

4. Vari;~~i~::;head} = rJ{Standar~ ~~urs x


Variance, VOEfV ~ per m
Actual} _ Actuall x
Output Hour~ lrg:~:~ ~::~~~J
Rate per Hour
Management Accounting : 638

J
~ 3,000 hours
= ~ 6,000 units X
'. }
6,500 umts - 3,300 hours
]
X
[ Rs.6,000
L3,000 hours
= [(3,250 hours - 3,300 hours) X Rs. 2] = (50 hours x Rs. 2) = Rs. 100 (A)

5. ~:~dv~:e:d}=
FOCV'
[ActUal
Output
x o!:e~~~~~:~ry
Rate per Unit
J - (Actual Fixed
Overhead
J
= (6,500 units x :~o~ '~~i~S J-[Rs. 1,250]
= [(6,500 units x Re. 0.2) - Rs. 1,250] = (Rs. 1,300 - Rs. 1,250) = Rs. 50 (F)

6. Fi~:p~:;;!~:d} = IsUdgeted x o~!::~di::;:~eryJ _ (ActUal Fixed J


Variance, FOEV loutput Rate per Unit Overhead

= (6,000 units x 6~~O~';~~s J-[Rs. 1,250]


= [(6,000 units x Re. 0.2) - Rs. 1,250] = (Rs. 1,200 - Rs. 1,250) = Rs. 50 (A)

7. Fixed Over~ead } [BUdgeted Actuall rStandard Fixed overhead]


VOlu~~~~ance = Output Outpu0 x l Recovery Rate per unit
= C(6000
~'
. - 6500
umts , .) x 6,000
umts Rs. 1,200
units = (500 umts J
. x Re. O·.2)
= Rs. 100 (F)

8. Fixed Overhead Effici-l


ency Variance, FOEfVJ
=r Standard Hours
L,ror Actual Output
_ Actuall x [Standard Fixed Overhead
HoursJ Recovery Rate per Hour J

=~ 3,000 hours
.
6,000 umts
6500 um't}
x, s - 3300
, hours ~ x [RS.
3,0 001,200
hours ]

= [(0.5 hour x 6,500 units) - 3,300 hours] x [Re. 0.4]


= [(3,250 hours - 3,300 hours) x Re. 0.4] = (50 hours x Re. 0.4) = Rs. 20 (A)
Standard Costing : 639

J
~
Standard Hours Standard Fixed
9. Fixed Overhead = 1 for Actual - Actual x
[
Overhead Recovery
Capacity Variance, FOCaVJ [ Working Days Hours Rate per Hour

=~ 3,000 hours x 27 days } - 3,300 hours Rs. 1,200 ]


Q 25 days ] x [ 3,000 hours

= [(120 hours x 27 days) - 3,300 hours] x [Re. 0.4]


= [(3,240 hours - 3,300 hours) x Re. 0.4] = (60 hours x Re. 0.4) = Rs. 24 (F)
10. Fixed Overhead Calendar 1 _ (Number of Excess x Standard Fixed Overhead)
Variance, FOCIV f - L Days Worked Recovery Rate per Day J
( 5d RS.l,200J
= L(27 days - 2 ays) x 25 days

=(2 days x Rs. 48) =Rs. 96 (F)


Verification, VOV = VOEV + VOEfV;
Rs, 150 (A) =Rs. 50 (A) + Rs. 100 (A) =Rs. 150 (A)
FOCV = FOEV + FOVV;
Rs. 50 (F) = Rs. 50 (A) + Rs. 100 (F) = Rs. 50 (F)
FOVV =FOEfV + FOCaV + FOCIV
Rs. 100 (F) = Rs. 20 (A) + Rs. 24 (F) + Rs. 96 (F)
= Rs. 20 (A) + Rs. 120 (F) = Rs. 100 (F)
lliustration: 7.44
The budgeted/actual sales of a concern manufacturing and marketing a single product are
furnished below.
Budgeted sales 10,000 units @ Rs. 4.00 per unit
Actual sales 5,000 units @ Rs. 3.50 per unit
8,000 units @ Rs. 4.00 per unit
Calculate: (1) The Sales Price Variance, and (2) The Sales Volume Variance. leA (Inl))
Solution:

Sales Price} (Actual Selling Standard "") ( Actual Sales J


Variance, SPY =L Price Selling PriceJ x Quantity
=[(Rs. 3.5 - Rs. 4) x 5,000 units] + [(Rs. 4 - Rs. 4) x 8,000 units]
= (Re. 0.5 x 5,000 units) + (Rs., 0 x 8,000 units) = Rs. 2,500 + 0 = Rs. 2,500 (A)
Management Accounting : 640

Sales VOlume} [Actual Standard] [Stan~ard ]


Variance SVV = Sales Sales x Selhng
, Quantity Quantity Price
=[(13,000 units -10,000 units) x Rs. 4] =(3,000 units x Rs. 4) =Rs. 12,000 (F)

Sales Revenue} _ (ActUal Sales


Variance, SRV - Revenue
Budgeted Sales
Revenue
J
= [(5,000 units x Rs. 3.5) + (8,000 units x Rs. 4)] - [10,000 units x Rs. 4]
=[(Rs. 17,500 + Rs. 32,000) - Rs. 40,000] =(Rs. 49,500 - Rs. 40,000)
=Rs. 9,500 (F)
Verification, SRV =SPV + SVV
Rs. 9,500 (F) =[Rs. 2,500 (A) + Rs. 12,000 (F)] =Rs. 9,500 (F)
mustration: 7.45
PH Ltd., furnishes the following information relating to budgeted sales and actual sales
for April.
Product Sales Quantity (units) Selling Price per Unit (Rs.)
Budgeted Sales: A 1,200 15
B 800 20
C 2,000 40
Actual Sales: A 880 18
B 880 20
C 2,640 38

Calculate the (i) Sales Quantity Variance, (ii) Sales-Mix Variance, (iii) Sales Price
Variance, and.(iv) Total Sales Variance. leA (lnt), May 1981}
Solution:
C omputarIon 0 fA ctual and Bud eteel Sales R evenue
Budgeted Actual

Product Sales Sales Sales Sales


Sales Price Sales Price
Quantity Revenue Quantity Revenue
(Rs.) (Rs.)
(units) (Rs.) (units) (Rs.)
A 1,200 15 18,000 880 18 15,840
B 800 20 16,000 880 20 17,600
C 2,000 40 80,000 2,640 38 1,00,320
4,000 1,14,000 4,400 1,33,760
Standard Costing : 641

Sales Revenue} _ (Actual Sales Budgeted Sale~


Variance, SRV - l Revenue Revenue )
= (Rs. 1,33,760 - Rs. 1,14,000) = Rs. 19,760 (F)

S~es Price }
Vanance, Spy
_ (Actual. Selling
- l Pnce
St~dar~
Selhng Price
J x [ Actual ~ales ]
QuantIty
SPVof A = [(Rs. 18 - Rs. 15) x 880 units] = (Rs. 3 x 880 units) = Rs. 2,640 (F)
B = [(Rs. 20 - Rs. 20) x 880 units] = (Rs. 0 x 880 units) = Rs. 0
C = [(Rs. 38 - Rs. 40) x 2,640 units] = (Rs. 2 x 2,640 units) = Rs. 5,280 (A)
Rs. 2,640 (A)
Sales VOlume} (Actual Sales
Variance, SVV = l Quantity
Standard
- Sales Quantity
J (x
Standard
lSelling Price
J
SVVof A = [(880 units - 1,200 units) x Rs. 15] =(380 units x Rs. 15) =Rs. 4,800 (A)
B = [(880 units - 800 units) x Rs. 20] = (80 units x Rs. 20) = Rs. 1.600 (F)
C =[(2,640 units - 2,000 units) x Rs. 40] =.(640 units x Rs. 40) = Rs. 25,600 (F)
Rs. 22,400 (F)

Revised Standard 1_ [TOtal Quantity of Actual Mix Standard Sales Quantity]


Sales Quantity, RSSqf- Total Quantity of Standard Mix x of a Product

RSSQ of A = (4,400 un~ts x 1,200 units] =1,320 units


l4,OOO umts

B = (4,400 units x 800 units


l4,OOO units
J= 880 units

c = (4,400 units
x 2 000 units] = 2 200 units
l4,000 units' -'--
4,400 units
Sales Quantity} = [Revised Stan~ard _ Standard Sales] (Standard Selling]
Variance, SQV Sales Quantity Quantity x l Price
Management Accounting : 642
SQVof A =[(1,320 units - 1,200 units) x Rs. 15] =(120 units x Rs. 15) =Rs. 1,800 (F)
B =[(880 units - 800 units) x Rs. 20] =(80 units x Rs. 20) =Rs. 1,600 (F)
C =[(2,200 units - 2,000 units) x Rs. 40] =(200 units x Rs. 40) =Rs. 8,000 (F)
Rs. 11,400 (F)

Sales Mix
Variance, SMVf
1 (Actual Sales
L Quantity
_ Revised Standardl
Sales Quantity
(Standard
J
x LSelling Price
J
SMVof A =[(880 units - 1,320 units) x Rs. 15] =(440 units x Rs. 15) =Rs. 6,600 (A)
B =[(880 units - 880 units) x Rs. 20] =(0 units x Rs. 20) =Rs. 0
C =[(2.640 units - 2,200 units) x Rs. 40] =(440 units x Rs. 40) =Rs.17,600 (F)
Rs. 11,000 (F)
mustration: 7.46
The following particulars are available in respect of the working of a company for a
particular period.
Budgeted Actual
Product Quantity Price Amount Quantity Price Amount
(units) (Rs.) (Rs.) (units) (Rs.) (Rs.)
,
A 1,000 2 2,000 1,800 2.50 4,500
B 3.000 3 9,000 4,200 2.75 11,550
4,000 11,000 6,000 16,050
You are reqUIred to calculate: (1) Total Sales Volume Vanance, (2) Sales Price Vanance,
(3) Sales Mix Variance, and (4) Sales Quantity Variance. [ICWA (lnt)]
Solution:
S~es Price }
Vanance. SPV
_ (Actual.Selling
- L Pnce
St:mdar~
SellIng Price
J x ( Actual ~ales
QuantIty
J
SPVof A =[(Rs. 2.5 - Rs. 2) x 1,800 units] =(Re. 0.5 x1,800 units) =Rs. 900 (F)
B =[(Rs. 2.75 - Rs. 3) x 4,200 units] =(Re. 0.25 x 4,200 units) =Rs. 1,050 (A)
Rs. 150 (A)

Sales VOlume} _ (Actual Sales


Variance, SVV - Quantity
Standard
- Sales Quantity
J ( x
Standard
LSelling Price
J
Standard Costing : 643

SVVof A = [(1,800 units - 1,000 units) x Rs. 2] = (800 units x Rs. 2) = Rs. 1,600 (F)
B = [(4,200 units - 3,000 units) x Rs. 3] = (1,200 units x Rs. 3) = Rs. 3,600 (F)
Rs. 5,200 (F)

Revised Standard 1 [ Total Quantity of Actual Mix Standard Sales Quantity]


Sales Quantity, RSSQ[= Total Quantity of Standard Mix x ofaProduct

RSSQ of A = r6,000 units x 1,000 units] = 1,500 units


L4,000 units

, B = r6,000 units x 3 000 units] = 4,500 units


L4,000 units '
6,000 units

Sales Quantity} _ (ReVised Standard _ Standard Sales] rStandard Selling]


Variance, SQV - Sales Quantity Quantity x L Price
SQVof A = [(1,500 units - 1,000 units) x Rs. 2] = (500 units x Rs. 2) = Rs. 1,000 (F)
B = [(4,500 units - 3,000 units) x Rs. 3] = (1,500 units x Rs. 3) = Rs. 4,500 (F)
Rs. 5,500 (F)

Sales Mix 1rActual Sales


Variance, SMvf = L Quantity -
Revised Standardl
Sales Quantity J
r Standard
x LSelling Price
J
SMVof A = [(1,800 units - 1,500 units) x Rs. 2] = (300 units x Rs. 2) = Rs. 600 (F)
B = [(4,200 units - 4,500 units) x Rs. 3] = (300 units x Rs. 3) = Rs. 900 (A)
Rs. 300 (A)
mustration: 7.47
Compute the missing data indicated by the question marks from the following.
ProductR ProductS
Sales quantity (standard) ? 400
Actual (units) 500 ?
Price/unit (Rs.): Standard 12 15
Actual 15 20
Sales price variance ? ?
Sales volume variance 1,200 F ?
Sales value variance ? ?
Sales mix variance for both products together is Rs. 450 (F)
lCA (Int), November 1988J
Management Accounting : 644
Solution:
1. Sales Price. }
Variance of R
= (Actual Selling
l Price
Standard J
Selling Price x
[ Actual Sales
Quantity
J
= [(Rs. 15 - Rs. 12) x 500 units] = (Rs. 3 x 500 units) = Rs. 1,500 (F)

2. Sales VOlume}
Variance of R =
(ActUal Sales
Quantity -
Standard Jx (
Sales Quantity
Standard l
lSelling Price)

Rs. 1,200 (F) =[(500 units - SSQ) x Rs. 12] =(6,000 units - 12 SSQ)
:. 12 SSQ =6,000 -1,200 =4,800
:. Standard Sales Quantity of Product R =[4,800 + 12] =400 units
3. Sales Revenue} _ (Actual Sales Budgeted Sale;)
Variance of R - l Revenue Revenue J
=[(5,00 units x Rs. 15) - (400 units x Rs. 12)] =(Rs. 7,500 - Rs. 4,800)
= Rs. 2,700 (F)

4. Revised Standard
Sales Quantity, RSSQJ=
1 [ Total Quantity of Actual Mix
Total Quantity of Standard Mix x
Standard Sales Quantity
ofaProduct
J
Let 'a' be the actual sales quantity of Product S.

RSSQ of R = [ 500 u~its + a uni~ x 400 units J= [500 units + ,a units x 400 units
400 UOltS + 400 UOlts 800 UOltS
J
= ( (500 +;) units J = (250 + 0.5a) units

S= [ 500 u~ts + a unit~ x 400 units J = [500 units + ,a units x 400 units J
400 UOlts + 400 UOlts 800 UOlts

= ( (500 +;) units J = (250 + 0.5a) units

5. Sales Mix }
Variance, SMV
(Actual Sales
l Quantity -
Revised Standardl
Sales Quantity
(Standard
J
x lSelling Price
J
Rs.450 (F) = [{ 500 units - (250 + 0.5a) units} x Rs.12] + [{ a units - (250 + 0.5a) units} x Rs.15]
= [(500 units - 250 units - 0.5a units) x Rs.12] + [(a units - 250 units - 0.5a units) x Rs.15]
Standard Costing : 645 .

=[(250 units - 0.5a units) x Rs. 12] + [(0.5a units - 250 units) x Rs. 15]
=Rs. 3,000 - Rs. 6a + Rs. 7.5a - Rs. 3,750 =Rs. 1.5a - Rs. 750
:. Rs. 450 (F) + Rs. 750 = Rs. 1.5a = Rs. 1,200

:. a = (~'s\~~O J =800 units =Actual Sales Quantity of Product S.

6. Syale.s Price S}
anance 0 f
=[(Rs. 20 - Rs. 15) x 800 units] =(Rs. 5 x 800 units) =Rs. 4,000 (F)
7. Syale.s YOIUmse}
ananceof
=[(800 units - 400 units) x Rs. 15] =(400 units x Rs. 15) =Rs. 6,000 (F)

8 Sales Revenue}. .
. Y' f S = [(800 umts x Rs. 20) - (400 umts x Rs. 15)]
anance 0
=(Rs. 16,000 - Rs. 6,000) =Rs. lO,OOO (F)
IDustration: 7.48
Budgeted income from sales on 500 tonnes was Rs. 5,00,000 per month. In November
1974, actual sales was 550 tonnes with a sales value of Rs. 4,95,000. Calculate variance.
[ICWA (Int), December 1974J
Solution:
Standard Actual
Sales Revenue (Rs.) 5,00,000 4,95,000
Sales Volume (tonnes) 500 550
Selling Price (in Rs. per tonne) 1,000 900
:. Standard Sales = [Actual Quantity x Standard Price] = (550 tonnes x Rs. 1,000)
= Rs. 5,50,000
Sales Revenue}
Variance = [Actual Sales Revenue - Budgeted Sales Revenue]

=[Rs. 4,95,000 - Rs. 5,00,000] =Rs. 5,000 (A)

S~les Price }
Yanance, Spy
_ (Actual. Selling
- l Pnce
St~ndar~
Selhng Price
J x ( Actual ~ales
Quantity
J
=[(Rs. 900 - Rs. 1,000) x 550 tonnes] =Rs. 55,000 (A)
Sales YOlume}
Variance, SYY
(Actual Sales
= Quantity
Standard
- Sales Quantity
J ( x
Standard I
lSelling PriceJ
=[(550 tonnes - 500 tonnes) x Rs. 1,000] =Rs. 50,000 (F)
Management Accounting : 646

IDustration: 7.49
The budgeted sales for one month and the actual results achieved are as under.
Budgeted Actual
Product Quantity Price Value Quantity Price Value
(Nos.) (Rs.) (Rs.) (Nos.) (Rs.) (Rs.)
A 1,000 100 1,00,000 1,200 125 1,50,000
B 700 200 1,40,000 800 150 1,20,000
C 500 300 1,50,000 600 300 1,80,000
0 300 500 1,50,000 400 600 2,40,000
Total 2,500 5,40,000 3,000 6,90,000
Calculate in respect of each product, the Sales Volume, Sales Mix and the Sales Quantity
Variances. [ICWA (Fin), December 1977J
Solution:
Sales VOlume} _ [Actual Sales
Variance, SVV - Quantity
Standard
- Sales Quantity
J x
Standard J
[ Selling Price

SVV of A = [(1,200 units - 1,000 units) x Rs. 100] =(200 units x Rs. 100) =Rs. 20,000 (F)
B =[(800 units -700 units) x Rs. 200] =(100 units x Rs. 200) =Rs. 20,000 (F)
C =[(600 units - 500 units) x Rs. 300] =(100 units x Rs. 300) =Rs. 30,000 (F)
0= [(400 units - 300 units) x Rs. 500] = (100 units x Rs. 500) = Rs. 50,000 (F)
Rs. 1,20,000 (F)

Revised Standard
Sales Quantity, RSSQf=
1 ( Total Quantity of Actual Mix
Total Quantity of Standard Mix x
Standard Sales Quantity
ofaProduct
J
RSSQ of A = (3,000
l2,500umts
u~ts
x 1.000 units] = 1,200 units

B = (3,000 units x 700 units


l2,500 units
J= 840 units

C = (3,000 units x 500 units


l2,500 units
J= 600 units

o = (3,000 units
l2,500 units
x 300 units J= 360 units
3,000 units
Standard Costing: 647

Sales Quantity} _ [ReVised Standard _ Standard Sales!- rStandard Selling]


Variance, SQV - Sales Quantity Quantity J
x l Price
SQV of A =[(1,200 units - 1,000 units) x Rs. 100] =(200 units x Rs. 100) =Rs.20,000 (F)
B = [(840 units -700 units) x Rs. 200] = (140 units x Rs. 200) = Rs.28,000 (F)
C = [(600 units - 500 units) x Rs. 300] = (100 units x Rs. 300) = Rs.30,000 (F)
D =[(360 units - 300 units) x Rs. 500] =(60 units x Rs. 500) =Rs.30,000 (F)
Rs.l,08,000 (F)

Sales Mix
Variance, SMvf
1 rActual Sales
l Quantity
_ Revised Standardl
Sales Quantity J
r Standard
x lSelling Price
J
SMVof A =[(1,200 units - 1,200 units) x Rs. 100] =(0 units x Rs. 100) =Rs. 0
B =[(800 units - 840 units) x Rs. 200] =(40 units x Rs. 200) = Rs. 8,000 (A)
C =[(600 units - 600 units) x Rs. 300] =(0 units x Rs. 300) = Rs. 0
D =[(400 units - 360 units) x Rs. 500] =(40 units x Rs. 500) = Rs. 20,000 (F)
Rs. 12,000 (F)
Summary of the Chapter
The need for, and unique features of, Standard Costing are discussed at the beginning of
the chapter followed by its meaning and definition. Other fundamentals of Standard Costing such
as Standard Costs, Types of Standard, Objectives of Standard Costing, differences between
Standard Cost and Estimated Cost, Standard Costing and Budgetary Control, and Advantages and
Limitations of Standard Costing are also discussed before the commencement of Variance
Analysis. Variance which denotes the deviation of actual from the standard is analysed in detail
followed by solving a large number of problems on different kinds of variances.
Key Terms to Remember
Standard Cost Standard Costing Estimated Cost
Real Standard Ideal Standard Variance
Cost Variances Material Cost Variance Material Price Variance
Material Usage Variance Material Mix Variance Material Yield Variance
Labour Cost Variance Labour Rate Variance Labour Efficiency Variance
Labour Mix Variance Labour Yield Variance Idle Time Variance
Overhead Cost Variance Variable Overhead Variance VOH Expenditure Variance
VOH Efficiency Variance Fixed Overhead Variance FOH Expenditure Variance
Management Accounting : 648
FOB Volume Variance FOB Efficiency Variance FOB Capacity Variance
FOB Calendar Variance Sales Variance, Sales Price Variance
Sales Quantity Variance Sales Mix Variance Sales Volume Variance

Questions for Self-study


01. Describe the meaning and objectives of Standard Costing. Bring out the differences between
Estimated Cost and Standard Cost. [Kuvempu Uni., B.Com., May 1999J
02. Bring out the differences between Standard Costing and Budgetary Control.
[Kuvempu Uni., B.Com., November 2001 and May 2004J
03. What is Standard Costing? What are its advantages?
[Kuvempu Uni., B.Com., November 2003J
04. Define Standard Cost and distinguish it from Estimated Cost.
05. What do you mean by Standard Costing? What are its advantages?
06. Critically evaluate the role of Standard Costing in cost control and reduction.
07. Define Standard Costing and distinguish it from Budgetary Control.
08. What is Standard Costing? What are its objectives? Explain the limitations of Standard
Costing.
09. Explain the significance of Standard Costing. [Mangalore Uni., B.Com., May 2002J
10. What do you mean by Variance? Explain the different categories of Cost Variances.
11. "Standard Costing is always accompanied by a system of Budgetary Control". Explain this
statement and bring out the advantages of Standard Costing.
[Kuvempu Uni., B.Com., November 2004J
12. What do you mean by Material Cost Variance? Explain the classification of Material Cost
Variances into different categories with suitable illustrations.
13. Distinguish between Material Price Variance and Material Usage Variance with suitable
examples.
14. Define Labour Cost Variance and explain the possible reasons for Labour Rate Variance,
Labour Efficiency Variance and Idle Time Variance.
15. What is Overhead Variance? What are the different types of Overhead Variances? Bow are
they calculated and dealt with?
16. Define Sales Variance and explain the different types of Sales Variances. Also describe the
computational procedure of Sales Variances.
17. Write a note on Idle Time Variance. [Kuvempu Uni., B.Com., November 2000J
18. What is Standard Costing? Explain in brief the steps involved in it.
[Bangalore Uni., B.Com., May 2001J
Standard Costing : 649

19. What are the advantages of Standard Costing? [Kuvempu Uni., B.Com., May 1997]
20. Define the term 'Variance' and explain the major types of Variances.
[Kuvempu Uni., B.Com., May 2000]
21. Briefly describe the significance of Variances in Standard Costing. ,
[Kuvempu Uni., B.Com., May 1991]
22. Write a note on Disposal of Variances
23. A Few Short-answer Questions:
a Define Standard Cost. -[Banga/ore Uni., B.Com., November 2000J
b. Define Standard Costing. [Banga/ore Uni., B.Com., November 2001 and 2003, April
2004, and Kuvempu Uni., B.Com., October 1997J
c. What do you mean by Variance?
[Bangalore Uni., B.Com., November 2001 and May 2002J
d. What is Labour Cost Variance? [Bangalore Uni., B:Com., November 2001J
e. Mention any four steps involved in Standard Costing.
[Banga/ore Uni.,
<>
B.Com., May 2001J
f. What is Material Cost Variance? [Bangalore Uni., B.Com., May 2001J
g. What do you mean by Standard Costing? [Kuvempu Uni., B.Com., May 2002]
h. What is Variance Analysis? [Kuvempu Uni., B.Com., November 2002J
24. From the following, you are required to calculate (a) Material Price Variance, (b) Material
Usage Variance, and (c) Material Cost Variance.
Quantity of material purchased: 3,000 units
Value of material purchased: Rs. 9,000
Standard quantity of material required for one tonne of finished product~ 25 units
Standard rate of material: Rs. 2 per unit
Opening stock of material: nil
Closing stock of material: 500 units
Finished production during the period: 80 tonnes. . [ICWA (Int)J
25. XY Ltd., manufacturers of Product P, uses a standard cost system. Standard product and cost
specifications for 1,000 kgs of product P are as follows:
Ingredients Quantity (kgs) Price per kg (Rs.) Cost (Rs.)
A 800 2.50 2,000
B 200 4.00 800
C 200 1.00 200
Input 1,200 3,000 (Rs. 2.50 per kg)
Output 1,000 3,000 (Rs. 3.00 per kg)
Management Accounting : 650

Material records indicate: Consumption in January:


A = 1,57,000 kgs @ Rs. 2.40
B= 38,000 kgs @ Rs. 4.20
C = 36,000 kgs @ Rs. 1.10
Actual finished production for the month of January is 2,00,000 kgs. Calculate: (a)
Material Price Variance, (b) Material Usage Variance, (c) Material Mix Variance, and (d)
Material Yield Variance. [ICWA (Int), December 1979J
26. In a brass foundry where standard costing is in operation, the standard mixture consists of
70% copper and 30% zinc. Standard loss in casting is 2% on input. Material usage for a
certain period is:
Copper: 60,000 lb
Zinc: 25,000 lb
Actual production is 82,500 lb
Submit a report on these facts to your management [M.Com., Karnatak Univ., 1980J
27. In a manufacturing concern, standard cost card shows the following details relating to
materials.
The standard material cost to. produce a tonne of chemical P is 300 kgs of material A at Rs. 10
per kg, 400 kgs of material B at Rs. 5 per kg and 500 kgs of material C at Rs. 6 per kg.
During a period, 100 tonnes of mixture X were produced from the usage of 35 tonnes of
material A at the cost of 9,000 per tonne; 42 tonnes of material B at a cost of Rs. 6,000 per
tonne; and 53 tonnes of material C at a cost of Rs. 7,000 per tonne.
Calculate (a) Material Cost Variance, (b) Material Price Variance, (c) Material Usage
Variance, (d) Material Mix Variance, and (e) Material Yield Variance.
[M.Com., UoM, April 1976J
28. Vinak Ltd., produces an article by blending two basic raw materials. It operates a standard
costing system and the following standards have been set for raw materials.
Material Standard mix Standard price/kg
A 40% Rs.4
60% Rs. 3
The standard loss in processing is 15%
During April 1980, the company produced 1,700 kgs of finished output.
The position of stocks and purchases for the month of April 1980 is as under:
Stock on Stock on Purchased during Cost
Material
1-4-80 (kgs) 30-4-80 (kgs) April 1980 (kgs) (Rs.)
A 35 5 800 3,400
B 40 50 1,200 3,000
Standard Costing: 651

Calculate the following variances: (a) Material Price Variance. (b) Material Usage
Variance, (c) Material Mix Variance, (d) Material Yield Variance, (e) Total Material Cost
Variance. lCA (lnt), May 1980J
29. Mixed Ltd., is engaged in producing a standard mix using 60 kgs of chemical M and 40 kgs
of chemical N. The standard loss of production is 30%. The standard price of Mis Rs. 5 per
kg and of N is Rs. 10 per kg. The actual mixture and yield were as follows.
M: 80 kgs Rs. 4.80 per kg
N: 70 kgs Rs. 8.00 per kg
Actual yield 115 kgs
Calculate Material Variances (Price, Usage, Yield, Mix) llCWA (Int)J
30. A standard set for material consumption was 100 kgs @ Rs. 2.25 per kg
In a cost period: Opening stock was 100 kgs @ Rs. 2.25 per kg
Purchases made 500 kgs @ Rs. 2.15 per kg
Consumption 110 kgs
(1) Calculate:
a. Usage Variance.
b. Price Variance when variance is accumulated at point of purchase.
c. Price Variance when variance is accumulated at point of issue on FIFO basis.
d. Price Variance when variance is accumulated at point of issue on LIFO basis.
(2) What is the effect on closing stock valuation when materials are charged out to cost on
bases (c) and (d) above? llCWA (lnt), June 1975J
31. A gang of workers normally consists of 10 men, 5 women and 5 boys, paid at standard hourly
rates of Rs. 4, Rs. 3 and Rs. 2 respectively. In a normal working week of 40 hours, the gang
is expected to produce 1,000 units of output. In a certain week, the gang consisted of 13 men,
4 women and 3 boys, 72 hours were worked, actual wages paid amounted to Rs. 2,500, and
1,000 units of output were produced. Present the information in respect of Labour Cost
Variance arising during this period. (Source not known)
32. The standard labour component and the actual labour component engaged in a week for a job
are as under.
Skilled Semi-skilled Unskilleq
workers workers workers
a. Standard number of workers in a gang 32 12 6
b. Standard wage rate per hour (Rs.) 3 2 1
c. Actual number of workers employed in
a gang during the week 28 18 4
d. Actual wage rate per hour (Rs.) 4 3 2
Management Accounting : 652
During the 40 hours working week, the gang produced 1,800 standard labour hours of work.
Calculate: (a) Labour Efficiency Variance, (b) Labour Mix Variance, (c) Rate of Wages
Variance, and (d) Total Labour Cost Variance. lCA (Int), November 1980J
33. Following details are available from the records of ABC Ltd., engaged in manufacturing
article 'A' for the week ended 28th September. The standard labour hour and rates of
payment per article •A' were as follows.
Hours Rate per hour Total
Skilled labour 10 Rs.3.00 Rs.30
Semi-skilled labour 8 1.50 12
Unskilled labour . 16 1.00 . 16
58
The actual production was 1,000 articles of 'A' for which the actual hours worked and rates
are given below.
Hours Rate per hour Total
Skilled labour 9,000 Rs.4.00 Rs.36,000
Semi-skilled labour 8,400 1.50 12,600
Unskilled labour 20,000 0.90 18,000
66,600
From the above set of data, you are asked to calculate: (a) Labour Cost Variance, (b) Labour
Rate Variance, (c) Labour Efficiency Variance, and (d) Labour Mix Variance.
llCWA (Int), December 1985J
34. A gang of workers normally consists of 30 men, 15 women and 10 boys. They are paid at
standard hourly rates as under.
Men Re.0.80
Women Re.0.60
Boys Re.0.40
In a normal working week of 40 hours, the gang is expected to produce 2,000 units of output.
During the week ended 31st December 1977, the gang consisted of 40 men, 10 women and 5
boys. The actual wages paid were @ Re. 0.70, Re. 0.65 and Re. 0.30 respectively. 4 hours
were lost due to abnormal idle time and 1,600 units were produced. Calculate (i) Wage
Variance, (ii) Wage Rate Variance, (iii) Labour Efficiency Variance, (iv) Gang Composition
Variance (i.e., Labour Mix Variance), and (v) Labour Time Variance. lCA (Fin), May 1978J
35. The cost figures relating to a company are as follows.
Actual overhead: Rs. 1,800;
Budgeted overhead: Rs. 2,000
Standard Costing : 653

Budgeted period: 4,000 labour hours


Standard hours per unit: 10 labour hours
Budgeted number of days: 20
Standard overhead per hour: Re. 0.50
Actual number of days: 22
Actual hours: 4,300
Actual production: 425 units
Calculate Expense Variance, Calendar Variance, Capacity Variance, Efficiency Variance nd
Volume Variance. EM. Com., Kamatak Univ., 1 ; '5j
36. A company's annual expenditure budget for 1977 was set at Rs. 4,80,000. Monthly bl ge.t
accordingly was taken at Rs. 40,000. Standard output for the year, on a 300-workin day
basis was 3,00,000 units and on this. standard overhead recovery rate was fixed.
In February 1977, there were 24 working days (28 days less 4 Sundays). During this I·mth,
production achieved was 23,000 units. Actual expenses in February 1977 totalled . Rs.
41,000. Calculate Total Overhead Variance, Efficiency Variance, and Calendar Varian
E1CWA (Int), Decembe 9:'7J
37. A manufacturing company operating a standard costing system had the following . Hd in
respect of February 1969.
Actual Number of working days: 22
Actual man hours worked during the month: 8,600
Units produced: 850
Actual OHEs incurred: Rs. 3,600
Further, the following information is obtained from the company's budget and standard cost
data: ' .
Budgeted number of working days per month: 20
Budgeted man hours per month: 8,000
Standard man hours per unit produced: 10
Standard flxed overhead per man hour: Re. 0.50
Calculate the Fixed Overhead Variances for the month. EM.Com., UoM, April 1970J
38. The budgeted and actual sales of a concern manufacturing a single product are given below:
Sales as budgeted: 10,000 units at Rs. 3 per unit: Rs. 30,000
Actual sales: 5,000 units at Rs. 3 per unit: Rs. 15,000
8,000 units at Rs. 2.50 per unit: Rs. 20,000
Ascertain Sales Price Varialice and Sales Volume Variance EM.Com., UoM, May 1975J
39. A company operates on standard costing and budgetary control systems. The budget for April
1981 and the actual for the same month are indicated below.
Management Accounting : 654

Budgeted Actual
Product
Sales kgs Price Rs/kg AmountRs. Sales kgs Price Rs/kg AmountRs.
TA 50,000 10.50 5,25,000 52,000 11.00 5,72,000
MB 20,000 12.00 2,40,000 16,000 11.75 1,88,000
CC 6,000 15.00 90,000 5,000 15.00 75,000
OC 4,000 16.00 64,000 5,000 16.00 80,000
Total 80,000 9,19,000 78,000 9,15,000
Calculate Sales Variances [ICWA (Fin), December 1981J
40. Budgeted and actual sales for the month of December 1984 of two products A and B of
Messers XY Ltd., were as follows.
Budgeted Actual
Product
Units Price/unitUnits Price/unit
A 6,000 5.00 5,000 5.00
1,500 4.75
B 10,000 2.00 7,500 2.00
1,750 1.90
Budgeted costs for products A and B were Rs. 4.00 and Rs. 1.50 per unit respectively. Work
out from the above data the following variances:
(a) Sales Value Variance (b) Sales Volume Variance
(c) Sales Price Variance (d) Sales Mixture Variance
(e) Sales Quantity Variance
[ICWA (lnt), June 1985J
41. From the following information about sales, calculate (a) Total Sales Variance, (b) Sales Price
Variance, (c) Sales Volume Variance, (d) Sales Mix Variance, and (e) Sales Quantity
Variance.
Budgeted Actual
Product Rate per unit Rate per unit
Nos. Total (Rs.) Nos. Total (Rs.)
(Rs.) (Rs.)
A 5,000 5 25,000 6,000 6 36,000
B 4,000 6 24,000 5,000 5 25,000
C 3,000 7 21,000 4,000 8 32,000
Total 12,000 70,000 15,000 93,000
Standard Costing : 655

Answers
24. (a) MPV =Rs. 2,500 (A); (b) MUV =Rs. 1,000 (A); (c) MCV =Rs. 3,500 (A)
25. (a) MPV = Rs. 4,500 (F); (b) MUV = 19,500 (F); (c) MMV = Rs. 3,000 (A); (d) MYV =
Rs. 22,500 (F)
26. Standard Usage (Ib) Actual Usage (Ib) Variation (lb)
Mixture 85,000 85,000 0
Copper (70%) 59,500 60,000 500 (excess use)
Zinc (30%) 25,500 25,000 500 (economy)
Casting loss 1,700 2,500 800 (excess loss)
27. (a) MCV = Rs. 1,38,000 (A); (b) MPV = Rs. 60,000 (A); (c) MUV = Rs. 78,000 (A); (d)
MMV =Rs. 11,333 (A); (e) MYV =Rs. 66,667 (A)
28. (a) MPV = Rs. 376.25 (F); (b) MUV = Rs. 90 (A); (c) MMV = Rs. 22 (A); (d) MYV = Rs.
68 (A); (e) MCV =Rs. 286.25 (F)
29. MPV =Rs. 180 (F); MUV =Rs. 50 (F); MMV =Rs. 50 (A); MYV = Rs. 100 (F);
30. (1) (a) MUV = Rs. 22.50 (A); (b) MPV = Rs. 50 (F); (c) MPV = Re. 1 (F); (d) MPV = Rs.
11 (F); (2) Value of Closing Stock will be lower by Rs. 49 under FIFO, and by Rs. 39 under
LIFO.
31. LCV =Rs. 100 (F); LEV =Rs. 2,440 (A); LMV =Rs. 360 (A); LYV = Rs. 2,080 (A)
32. (a) LEV == Rs. 504 (A); (b) LMV = Rs. 80 (F); (c) LRV = Rs. 2,000 (A); (d) LCV = Rs.
2,024 (A).
33. (a) LCV = Rs. 8,600 (A); (b) LRV = Rs. 7,000 (A); (c) LEV = Rs. 1,600 (A); (d) LMV =
Rs. 4,200 (F)
34. LCV = Rs. 256 (A); LRV = Rs. 160 (F); LEV = Rs. 268 (A); lTV = Rs. 148 (A); LMV =
Rs. 120 (A)
35. Ex. Var = Rs. 200 (F); Cal. Var = Rs. 200 (F); Cap. Var = Rs. 150 (F); Eff. Var = Rs. 25
(A); Vol. Var =Rs. 125 (F); FOCV =Rs. 325 (F)
36. FOCV =Rs. 4,200 (A); FOEfV =Rs. 1,600 (A); FOCIV =Rs. 1,600 (A)
37. FOCV = Rs. 650 (F); FOEV = Rs. 400 (F); FOVV = Rs. 250 (F); FOEfV = Rs. 50 (A);
FOCaV =Rs. 300 (F)
38. SRV =Rs. 5,000 (F); Spy =Rs. 4,000 (A); SVV =Rs. 9,000 (F)
39. SRV = Rs. 4,000 (A); Spy = Rs. 22,000 (F); SVV = Rs. 26,000 (A); SQV = Rs. 22,975
(A); SMV =Rs. 3,025 (A)
40. (a) SRV = Rs. 450 (F); (b) SVV = Rs. 1,000 (F); (c) Spy = Rs. 550 (A); (d) SMV = Rs.
1,781 (F); (e) SQV = 781 (A)
41. (a) SRV = Rs. 23,000 (F); (b) Spy = Rs. 5,000 (F); (c) SVV = Rs. 18,000 (F); (d) SMV =
Rs. 500 (F); (e) SQV = Rs. 17,500 (F).
Chapter - VIn

MARGINAL COSTING
Objectives: Objectives of this chapter are:
• 'To know the Meaning of Marginal Cost, Marginal Costing and other concepts such as Contribution,
Plv Ratio, Margin of Safety" Angle of Incidence, etc.
• To understand the Fundamentals of Break-Even Analysis.
• To learn the Effects of Changes In Cost, Price' and Volume on Break-Even Point, Profit, etc.
• To learn the Importan~ and application of Marginal Costing.
• To solve the problems,on Marginal Costing.
Structure ' '

• Introduction
• Meaning arid peflnltlon of Marginal Costing
• Meaning and Definition of Marginal Cost a!1d Ascertainment of Marginal Cost
• Concept' of Contribution and Plv Ratio
• Break-Even' Analysis
o Algebraic Approach }
o Graphical ApprOClch • Mono-Product Concerns
'. Break-even Chart • Multi-Product Concerns
• Plv Graph
~ Assumptions of Break-Even Analysis
• C-V-P Analysis
• Illustrations
• SllInmary of the Chapter,
• Key Terms to Remember
• References
• Questions for Self-study
Marginal Costing : 657

Introduction
Industrial undertakings aim at maximizing their values to their owners. The managerial
personnel at the helm of affairs of the undertakings have to direct all their resources and efforts
towards the accomplishment of this objective. The decisions which the management has to take
should ensure both the minimization of cost and the maximization of profit. This decision making
process is, therefore, reckoned as the most significant one as the very survival of the business
entity depends upon these managerial decisions and their implementation.
Decision making process has been defined as the process of selecting tbe best
alternative out of a number of possible alternatives available. The use of the phrase best
alternative in the definition implies that the alternative must ensure the best result to the
company. An alternative may be considered as the best one if, and only if, it ensures least cost
and/or maximum profit. Possible alternatives used in the definition refer to the alternatives
which are within the reach of the company. That means, a company is capable of implementing
anyone of these alternatives. It may be noted here that, if there is only one possible way, then
there is no scope for taking decision as the management has no other alternative except
implementing the one which is available. Since the decisions have a number of implications on
the determinants of profit, performance, etc., it is necessary to have a comprehensive evaluation
of each of the possible alternatives so that the mariagement selects the best alternative. In order to
evaluate the alternatives, it is necessary to consider all the influencing factors.
The influencing factors include both the quantitative and the qualitative factors.
Quantitative factors are those factors the effect of which can be measured in t~rms of monetary
units. They are, therefore, called monetary factors. For example, material cost, labour cost, etc.
On the other hand, Qualitative factors are those influencing factors whose effect cannot easily
and directly be measured in terms of monetary units. For example. labour relation. Of course,
these qualitative factors, also known as non-monetary factors, influence the quantitative factors
indirectly. For instance, labour relation has an imp~ct on the wages, production, idle time wages,
overtime costs, labour productivity, etc. Because of the difficulty in measuring the effects of
qualitative factors and also due to the fact that these qualitative factors influence the quantitative
influencing factors, the decisions are normally taken on the basis of only the quantitative factors
with due regard to qualitative factors.
From the above analysis, it is obvious that the management has to take a number of decisions.
To take decisions, it (Le., management) needs information about the influencing factors. Therefore,
there is a need for a mechanism which ensures the furnishing of information to the management after a
comprehensive evaluation of all thc:r possible alternatives to take the final decision.
One of the important contributions of Marginal Costing is its invaluable service to the
managerial personnel for the purpose of taking different types of managerial decisions. The
'service' is in the form· of analyzing the costs and revenue from the viewpoint of their relevancy to
the decision under consideration and also furnishing the necessary data to the management. No
doubt, Absorption Costing also furnishes the data to the management. But it (Le., Absorption
Costing) furnishes both the relevant and the irrelevant data Consequently, the management
which is responsible to take proper decisions will be influenced by both the relevant and the
irrelevant data. As a result, there is every possibility of management taking wrong decisions.
Due to this reason, Marginal Costing which is capable of furnishing only the relevant factors to
Management Accounting: 658
the managerial personnel after distinguishing them from innumerable irrelevant factors is gaining
wide popUlarity as a Technique of Costing and as an effective tool in the hands of management
for the purpose of taking various decisions.
Meaning of Marginal Costing
The Chartered Institute of Management Accountants (CIMA), London defines Marginal
Costing as the ascertainment of marginal cost and effect of changes in volume or type of
output on the company's profit, by segregating total costs into variable and fixed costs. This
definition lays emphasis on the ascertainment of Marginal Costs and also the effect of changes in
volume or type of output on the company's profit. However, this definition suffers from a few
limitations. Because, profit and profitability are influenced not only by the changes in the volume
and type of output, as stated in the definition, but also by the changes in price level, productivity,
prices of input factors, product mix, etc. Further, Marginal Costing primarily aims at furnishing
relevant data to the managerial personnel for the purpose of assisting them to take a number of
wise decisions. But the definition presented above fails to recognize this aspect. Considering
these aspects, and on the basis of the definition by the CIMA, London, Marginal Costing
definition may be modified and improved as a technique of Costing which aims at ascertaining
Marginal Costs, determining the effects of changes in costs, volume, price~ etc., on the
company's profitability, stability, etc., and furnishing the relevant data to the management
for enabling it to take various managerial decisions by segregating total costs into variable
and fixed costs. This is the subject matter of this Chapter.
Synonymous Terminologies
A look at the literature on Marginal Costing reveals that different terminologies such as
Differential Costing, Direct Costing, Incremental Costing, Marginal Costing, Relevant Costing,
Variable Costing are used synonymously and interchangeably to denote the same Marginal
Costing. But a close look at the terminologies reveals some differences among themselves. For
instance, Marginal Cost denotes the extra cost to be incurred per period to increase the volume of
output by one unit. On the other hand, Incremental Cost represents the increase in the total cost
due to either the increase in the volume of output by one or more units or due to the changes in
the quality of the product or due to the changes in the method of production. In the same way,
Direct Cost represents the cost which can specifically be identified with the product. The Direct
Costs, therefore, include direct material cost, direct labour cost and other direct expenses but
exclude the variable overhead expenses. But the Marginal Costs ~ay include even the fixed costs
if the production of an additional unit increases the fixed costs. Like this, some differences can be
found between the terminologies. In spite of this, all these are used interchangeably to denote the
same unless it is stated specifically.
Marginal Cost
Th.! CIMA, London has defined Marginal Cost as the cost for producing one additional
unit of product. It has also been defined as the amount changes in the aggregate costs due to
changes in the existing level of production by one unit. An analysis of these definitions reveals that
the Marginal Cost is the cost of producing an additional unit. That means, Marginal Costs (MCs)
refer to the extra costs for the production of an additional unit. Whenever there is a change in the
volume of output, certainly there will be some change in the total costs. The change in total costs is
Marginal Costing: 659
due to the changes in variable costs. To put it alternatively, any change in the level of activity changes
the variable costs which in turn changes the total costs. Economists concentrate on the changes in the
total costs to find out the Marginal Costs. Since the change in the total costs is due to the change in the
variable costs, the Accountants concentrate on variable costs to compute Marginal Costs. They (i.e.,
Accountants) define Marginal Costs as equivalent to the variable costs. Hence, it (i.e., Marginal Cost),
from Accountants' point of view, refers to the change in the total variable costs. Of course, even the
Accountants reckon the increase in the total costs to compute the Marginal Cost. It is only when the
increase in the output necessitates the increase in the fixed costs.
Ascertainment of Marginal Cost
As already stated, Marginal Costing aims at ascertaining the Marginal Cost by
segregating total costs into variable and fixed. That means, in order to compute Marginal Cost, it
is necessary to segregate total costs into variable costs and fixed costs. Because, Marginal Cost is
equal to variable cost. The Glossary of Management Accounting Terms states Marginal Cost as
the additional or escapable cost of making one more or less unit respectively of a given part
of product. This takes into account only the increase in variable cost. That means, Marginal
Cost is equivalent to the extent to which the total cost has increased or decreased when the
volume of output is increased of decreased by one unit. Since the increase or decrease in total
costs is normally due to the increase or decrease in variable costs, the Marginal Cost may be
computed by finding out the increase or decrease in the total variable cost when the quantum of
output increases or decreases by one unit.
At this stage, it is necessary to have a look at the behaviour-wise classification of costs.
The costs can be classified into three broad categories on the basis their response to changes in the
levels of activity. The three categories are: variable costs, fixed costs (fixed overhead expenses)
and semi-variable or semi-fixed costs (or semi-variable or semi-fixed overhead expenses). The
semi-variable costs (SVCs) are to be segregated into their variable and fixed portions. Further,
the variable portion of semi-variable costs is to be added to variable costs and the aggregate of
these represents the total variable cost which represents the Marginal Cost. The fixed portion of
semi-variable cost is to added to fixed costs to obtain the total fixed costs.
Behaviour-wise Classification of Total Costs
Variable Costs - - - - - - - - - - - - - ,
Variable Cost or
+ = Marginal Cost or
Variable Portion-n----' I Product cost

Total
Semi-variable---7 [
Costs Costs Fixed Portion -----,

Fixed Costs
_ _ ---J+I= .
Fixed Cost or
Period Cost

Hence, it is necessary to know the meaning and behaviour of variable costs, fixed costs,
semi-variable costs and also the segregation procedure.
Management Accounting : 660
Variable Costs
Variable costs which are also called product costs are those costs which vary with the
levels of activity in the same direction and more or less, in the same proportion. ICW A, India has
also defined VariabIe Cost as an operating expense, or a group of operating expenses that
vary directly and in proportion to the level of activity, viz., sales or production. Examples
are materials consumed, direct labour, power, sales commission, utilities, freight, packaging,
etc. This definition reveals two important features of variable costs. They are uni-directional
changes and proportional changes. Both the (total) variable costs and the quantum of work
move in the same direction. That means, if the output increases, there will be an increase in the
total variable costs and vice-versa. Another important feature of variable costs is the proportional
changes. That means, any change in the level of activity leads to a proportional change in the
variable costs. This proportional change is due to constancy in the unit variable cost. That
means, unit variable cost is assumed to be a constant one irrespective of the levels of activity.
The summary of this analysis is presented below graphically.
Variable Cost-Output Relationship
y

Total Variable Cost Line

Unit Variable Cost Line

o Y-----------------·X
Output (units)

Fixed Costs (FCs) or Fixed Overhead Expenses (FOBEs)


Fixed Costs which are also called Period Costs are defined as those costs which remain
constant irrespective of the levels of activity. That means, they are constant costs and are not
affected by the levels of activity. The Research and Publications Committee of ICW A, India has,
therefore, defined Fixed Costs as the costs which are not affected by temporary fluctuations in
activity of an enterprise, e.g., interest, insurance, administrative expenses, etc. What is
important is the fact that the major portion of these costs is normally inescapable and therefore,
their incurrence by the organizations is unavoidable. To put it alternatively, they go on
accumulating as the time passes irrespective of the levels of activity. Normally, the Period Costs,
in total, are unaltered costs and hence, these costs per unit move inversely with the levels of
activity. That means, both the Unit Fixed Cost (UFC) and the volume of output move in opposite
directions. The behaviour of both the Total and Unit Fixed Costs can lucidly be depicted
graphically as below.
Marginal Costing : 661

Fixed Cost-Output Relationship

,....,
~ 1----\:--------- Total Fixed Cost Line
~
o
U
]
u:

Unit Fixed Cost Curve


O~---------------
Output (units)
..... x
Semi-variable Costs (Semi-variable Overhead Expenses)
The third category of costs is Semi-variable Overhead Expenses (SVOHEs) or Semi-
Variable Costs (SVCs). These costs are also called Semi-Fixed, or Mixed, or Fixed Plus,
Overhead Expenses. Whatever terminology is used, it is important to note that these expenses
behave in the manner of neither the Variable Costs nor the Fixed Costs. But at the same time, one
can easily say that the Semi-variable Costs behave in the manner of both Variable Costs and
Fixed Costs. Since this category of costs comprises of the CQsts which consist of both variable
and fixed elements in some proportion, they are called, Semi-variable Costs or Semi-fixed Costs.
The fixed portion is the cost of making the service ready for use. It may also represent the
minimum charge for using the facility up to a particular capacity level. On the other hand, the
variable portion represents the cost of actual utilization of the facility in excess of minimum level.
It may be noted from this analysis that there are more than one interpretation to the meaning of
variable and fixed portion of Semi-variable Costs.
It is, therefore, apparent that the Semi-variable Costs comprise of variegated types of
costs which differ from each other from the viewpoint of their behaviour with respect to the levels
of activity. But the effectiveness and the reliability of Marginal Costing technique, to a greater
extent, depends upon the accuracy with which the Total Costs or the Semi-variable Costs are
segregated into Variable Costs and Fixed Costs. Due to this reason, it is advisable to segregate
each item of Semi-variable Costs or Total Costs into variable and fixed. Since the number of
items of Semi-variable Costs is very large and the amount involved with the Semi-variable Costs
is comparatively small, segregation of Semi-variable Costs is carried out on aggregate basis. A
"number of methods are available for segregation of Semi-variable Costs into variable and fixed
costs. The important approaches are identified and explained herein under very briefly.
01. Comparison Method 04. Average Method
02. Equation Method 05. Scatter-graph Method
03. Range or High and Low Method 06. Least-squares Method
Management Accounting : 662

Comparison Method
Under this approach, the number of units at any two levels of activity is compared with
the corresponding amounts of Semi-variable Costs. By comparison, differences in both the Semi-
variable Overhead Expenses and the output may be computed. It may be recalled here that the
Semi-variable Overhead Expenses include both variable and fixed elements. Fixed portion of
Semi-variable Overhead Expenses remains same at both the levels. Therefore, the change in the
Semi-variable Overhead Expenses (represented by the difference in the Semi-variable Overhead
Expenses at two levels of activity) is due to the change in the variable portion of Semi-variable
Overhead Expenses. Hence, the variable portion of Semi-variable Overhead Expenses per unit of
output may be computed by dividing the change in Semi-variable Overhead Expenses by the
change in the volume of output.

Variable Overhead Expenses } Difference in semi-Variable]


per unit (included in Semi-variable = Overhead Expenses
Overhead Expenses [ Difference in Output

:. Total Variable Overhead Expenses} (V· bi 0 h d~ x (Number of Unit~


. Iuded'm Serm-vana
mc . . bie = Eana ever ea.
Overhead Expenses xpenses per umt of Output J
Fixed Overhead Expenses}
included in Semi-variable = ( . .
Serm-Vanable
J [_
Total Variable
Overhead Expenses
~
O verhead Expenses Overhead Expenses included in Semi-variable
Overhead Expenses
Equation Method
When cost and output figures for only two periods are given or available, Equation
Method may also be used. Here, the Straight Line or Linear Equation is used. The basic linear
cost equation is [Y = mx + c) where, Y = Semi-variable Cost, m = Variable portion of Semi-
variable Overhead Expenses per unit, x = Number of Units, and c =Total Fixed portion of Semi-
variable Overhead Expenses. By developing two cost equations (one for each period) and solving
these equations for om' and 'c', the Semi-variable Overhead Expenses can be segregated.
Illustration: 8.1
The following figures pertain to repairs and maintenance costs (R&M costs) of a
company for two years. Using Comparison and Equation Methods, segregate R&M costs into
variable and fixed portions.
Year: 2004 2005
Machine Hours: 10,000 20,000
R&M Costs (Rs.): 15,000 25,000
Marginal Costing : 663

Comparison Method

Variable portion ofl_ ( Difference in R&M Costs J-


R&M ~ost per f- Difference in Machine
r
Rs. 25,000 - Rs. 15,000
- 120,000 hours - 10,000 hoursJ
I
Machine Hour Hours

= r. Rs. 10,000 l =Re 1


lJ 0,000 hoursJ .

Total Variable portio~ _


in R&M Cost f -
r
Number of
LMachine Hours x
Variable R&M cost]
per Machine Hour

For 2004, = (10,000 hours x Re. 1) = Rs. 10,000


2005, =(20,000 hours x Re. 1) =Rs. 20,000
:. Fixed Portion ofR&M Cost = (Total R&M _ Variable portion OfJ
L Costs R&M Cost
:. For 2004, =(Rs. 15,000 - Rs. 10,000) =Rs. 5,000
2005, =(Rs. 25,000 - Rs. 20,000) =Rs. 5,000
Equation Method
It is known that, Y = mx + c. Therefore, equation for:
2004; Rs. 15,000 =m 10,000 + c ..... (1)
2005; Rs. 25,000 =m 20,000 + c ..... (2)
subtract (2) from (1): - Rs. 10,000 = - m 10,000

- Rs. 10,000J
:. m = [__ 10,000 =Re. 1 =Variable portion of R&M Cost per Machine Hour
:. Total variable portion of R&M Cost for:
2004 =(10,000 hours x Re. 1) = Rs. 10,000
2005 =(20,000 hours x Re. 1) =Rs. 20,000
:. Fixed portion of R&M Cost:
2004 = (Rs. 15,000 - Rs. 10,000) = Rs. 5,000

.
2005 =(Rs. 25,000 - Rs. 20,000) = Rs. 5,000
Range or High and Low Method
This method is an improvement over the Comparison and the Equation Methods as it can
be used even when the data are given for more than two periods. When costs and output figures
are furnished, or available, for more than two periods, only two periods are selected for
Management Accounting: 664

calculation purposes. Out of the two periods.• one is the period during which the company has
produced highest number of units, and the another one is the period during which it has produced
the lowest number of units. Hence, the name high and low method. Once the selection of the
periods is made, then either the Comparison Method or the Equation Method can be used.
Because, after the selection of periods, the same procedure (as under Comparison or Equation
Method) is used.
Average Method
This method is certainly an improved one over both the Comparison and Range Methods.
Under this approach, the periods for which the cost and output details are given are classified into
two groups. ~s far as the grouping is concerned, the problems with 'even' periods do not pose
any difficulty. Because, when the data are given for 2, 4, 6, 8 ...... , periods, it can easily be
classified into two groups, each group comprising of data for 1, 2, 3, 4 ... , periods respectively.
But, when the data are given for 'odd' periods say 3, 5, 7, 9 ... , one period is to be left. The
period which is to be left for calculation purposes may be either the first period or the last period
or the middle period or any other most appropriate period. Then, the remaining periods are to be
classified into two groups as in the case of 'even' periods. Once the periods are classified into
two groups, the next step is to compute the average (i.e., both average cost and average .output) for
each group. At·the end of this exercise, cost and output data for only two periods (i.e., for two
groups) will be available. Then (either) Comparison Method (or the Equation Approach) may be
used to segregate the Semi-variable Costs into variable and fixed portions.
Scatter-Graph Method
In the graph, horizontal line represents the output or the levels of activity and the vertical
axis represents the costs. The Semi-variable Costs at different levels of activity are plotted on the
graph against the corresponding levels of activity. A line is then drawn passing through the
maximum number of plotted points and this line is called line of best fit or regression line. This
is the Semi-variable Cost Line (average) and the graph is called scatter-graph. The regression line
is extended to the left side in the graph till it touches the vertical axis. The point at which the
regression line touches the vertical, or the cost, axis reading at some amount represents the fixed
portion included in the Semi-variable Costs. If a line is drawn, from the point at which the
Regression Line touches the vertical axis, parallel to horizontal axis (it may be called Fixed Cost
Line), we will be able to get an idea about the composition of Semi-variable Costs. Because, the
gap between the horizontal axis and the Fixed Cost Line represento; the fixed element of Semi-
variable Costs at any level of activity. In the same way, the gap between the Fixed Cost Line and
the Semi-variable Cost Line represents the variable portion of Semi-variable Costs. The
following figures may be used to construct a scatter-graph and to segregate the Semi-variable
Costs. .

Output (units): 1,000 1,500 2,000 2,500 3,000


Semi-variable Costs (Rs.): 30,000 40,000 35,000 50,000 45,000
Marginal Costing : 665

Scatter-Graph
Y

50 Semi-variable Cost Line


Semi-
variable 40
Costs Variable portion
(Rs.OOO) 30
Fixed Cost (portion) Line
20
Fixed portion
10

o 500 1000 1500 2000 2500 3000


output (units)

From the above, it becomes obvious that by fi.tting a line and extending the same till it
touches the vertical axis, the fixed element included in the Semi-variable Cost (Rs. 22,500) can be
ascertained. The Semi-variable Cost at any level of activity minus the fixed element (as
computed above) represents the variable portion included in the Semi-variable Costs at that level
of activity.
Least-Squares Method
This method is an improvement over the Scatter-graph Method. Because, as already
identified, the regression line is fitted by a simple visual look. It may be recalled here that the
variable and fixed elements of Semi-variable Costs depend upon this regression line. Any error in
fitting the regression line leads to the wrong calculation of variable and fixed elements. Hence,
the Least-squares Method which fits the regression line by statistical analysis is considered as an
improvement over Scatter-graph Method.
The term least-squares, as opined by Garrison, represents the sum of the squares of the
deviations from the plotted points to the regression line is smaller than would be obtained
from any other line fitted to the data. This method also bases its calculations on an equation
for a straight line which is expressed in the form of Y = mx + c. In the equation, 'm' and 'c' are
assumed to be constant and therefore, whenever there is a change in 'x' (Le., output or activity
level), there will be a change in 'V' (Le., Total Semi-variable Cost). That means, 'V' is a
dependent factor and 'x' is an independent factor. Let us assume different units of output for
different periods, and compute 'Y'. Therefore, for the year 2001, when a company produced 'XI'
units, the Semi-variable Cost equation will be,

YI=mxl+c
For 2002, when x = X2 Y2 =mX2 + c
For 200N, when x = Xn Yn = mXn + c
For 2001 to 200N (i.e., by adding), ~Y=~x+nc ..... (1)
Management Accounting : 666

That means, total semi-variable cost for on' years (EY) is equivalent to sum of 'product of
unit variable cost (included in Semi-variable Cost) and number of units produced' (i.e., mLx) and
the product of ftxed cost (included in Semi-variable Cost) per period and number of periods' (i.e.,
nc). Once again, let us take the basic equation for a straight line viz., Y = mx + c and multiply the
equation by x. Therefore, [(Y = mx + c) xl :. xY = mx2 + cx.

For 2001, x,Y, = (mx 2, + cx,)


For 2002, X2Y2 = (mx22 + CX2)
200N, Xn Yo = (mx2n + cXn)
Adding, :ExY = (mLX2 + c:Ex) ..... (2)
Using equations (1) and (2), values for m and c segregating the Semi-variable Cost (Le.,
Y) into variable (Le., m) and ftxed cost (Le., c) can be computed.
mustration: 8.2
The following data are extracted from the records of P Company for the last ftve years.
From these, using Least-squares Method, ftnd out the unit variable cost and the yearly ftxed cost
included in the semi-variable cost. Also determine the cost formula for the semi-variable cost.

Year: 2001 2002 2003 2004 2005


Output (units) 10,000 20,000 15,000 25,000 5,000
Semi-variable Cost (Rs.): 30,000 50,000 40,000 60,000 20,000

Solutioo:
computat'1000f:E x, :EY,et c
Output Semi-variable Cost
Year xY x2
(000 units) (x) (Rs. 000) (Y)
2001 10 30 300 100
2002 20 50 1,000 400
2003 15 40 600 225
2004 25 60 1,500 625
2005 5 20 100 25
n=5 :Ex = 75 :EY = 200 :ExY = 3,500 :Ex2 = 1,375
Let us substitute the values computed and shown in the above table in equations :EY =
m1:x + nc and :ExY = mLX2 + c:Ex. Therefore,
200=75 m+ 5 c ... (1)
3,500 = 1,375 m + 75 c ... (2)
Marginal Costing: 667

Now, there are two equations (1 and 2) with two unknown variables (m and c) which are
to be computed. When there are two unknowns, it is necessary to eliminate one of the two
unknowns f.or the purpose of solving the equations. This is possible by equating the coefficient of
anyone of the two unknowns. Therefore, divide both the sides of equation (2) by 15 to equate the
co-efficient of 'c'. By dividing equation (2) by 15, we get,
233 1/ 3 = 912/3 m + 5c ... (3)
200 = 75 m + 5c ... (1)
Subtract equation (1) from (3), 33 Ih = 16 2/ 3 m + 0

Therefore, m = (33 ~b~ = Rs.2 (Variable portion of Semi-variable Cost per unit)
16 hJ
Particulars • 2001 2002 2003 2004 2005
Semi-variable Cost (Rs.) 30,000 50,000 40,000 60,000 20,000
Less: Variable Cost (@ Rs. 2 per unit) 20,000 40,000 30,000 50,000 10,000
:. Fixed Portion 10,000 10,000 10,000 10,000 10,000
Therefore, Semi-variable Cost formula, Y = 2x + 10,000.
By using anyone of the above methods, Semi-variable Costs can be segregated into variable
and fixed. Once the segregation is completed, variable portion of Semi-variable Cost is to be added to
Variable Overhead Expenses and that of fixed portion to Fixed Overhead Expenses. The aggregate of
Direct Costs and Variable Overhead Expenses (including the variable portion of Semi-variable
Overhead Expenses) represents the Variable Cost which is nothing but Marginal Cost
Composition of Total Costs
Direct
Costs
+
•= Marginal or
Variable Cost
Total
f-+
t
Costs Variable
.. Variable
r-+ f= Overheads
Overheads

Variable
-+ Indirect Costs ~ ...---. Portion
(Overheads)
Semi-variable
r+
---- Overh~s
Fixed
4
Portion

. *=
Fixed Fixed Cost
Overheads r + (Fixed
[n simple, Marginal Cost =Direct Costs + Variable Overhead Expenses Overheads)
M~nagement Accounting : 668

(Direct Material Cost + Direct Labour Cost + }


Other Direct Expenses) + (Variable Overhead
= {
Expenses + Variable portion of Semi-variable
Overhead Expenses)

= (Direct Costs + Variable Production Overhead Expenses + Variable AdministratiVe]


LOverhead Expenses + Variable Selling and Distribution Overhead Expenses
It may also be analysed as below.
Composition of Total Costs

VariableCost-----------~----+
Vari~ble or
Marginal Cost

Total
f -.........Semi-variable Cost---l."
Cost
Fixed Portion - - -•• 1 Fixed Cost
Fixed cOl>.lst_ _ _ _ _ _ _ _ _ _ _ _ _ _ _ _ _ _ _--LT

The information about the Marginal Costs can .be collected from internal records; of the
companies. For example, information about Direct Material. Cost may be collected from the
records such as, Material Requisition Note, Stores Ledger, etc. The consumption of different
types of raw materials can easily be obtained from the stores records and these figures can be
converted into monetary units with the help of the prices at which the materials have been issued
from the stores department to the production departmynts. Figures about Direct Labour Cost can
be obtained from the Wages Analysis Sheet. Information about Other Direct Expenses can be
collected from the concerned records maintained by the companies. This way, information about
various costs - both Marginal Cost and Fixed Cost - can be collected from the records kept, and
maintained, by the companies. Since the items of Marginal Cost have already been identified,
ascertainment of Marginal Cost is simply a process of adding up various elements of Marginal
Cost.
mustration: 8.3
A firm produces 200 units for a total cost of Rs.730 and 500 units for a total cost of.Rs.
970. It is known that the cost curve is a straight line. Derive the equation of the cost line and use
it to estimate the cost of producing 400 units. [eS]
Marginal Costing: 669

Solution:
Output Total Cost
(units) (Rs.)
200 730
500 Variable CostL_(Difference in Total Co~ _ [Rs. 240J
970
per Unit f -lDifference in Output) -l200
units
300 240 = Re. 0.8

Therefore, Fixed Cost (Total) = [Total Cost - Total Variable Cost]


When x = 200, Total Fixed Cost = [Rs. 730 - (Re. 0.8 x 200 units)] = [Rs. 730 - Rs. 160]
=Rs. 570.
Therefore, cost equation is Y = [0.8x + 570]. Using this equation, cost of producing 400 units
can be estimated and it comes to Rs. 890 [i.e., (Re. 0.8 x 400 units) + Rs. 570].
lllustration: 8.4
Kaku Ltd., produces on~ standard type of article. The results of the last four months of
the year 1981 are as follows.

Month: September October November December


Output (units): 200 300 400 600

Prime cost is Rs. 10 per unit, variable expenses are Rs. 2 per unit, fixed expenses are Rs.
36,000 per annum, Find out cost per unit of each month.
res (Fin), December 1982}
Solution:
Unit Variable" Cost = [Prime Cost + Variable Expenses] = [Rs. 10 + Rs. 2] = Rs. 12.
Monthly Fixed Cost = [Rs. 36,000 + 12 months] =Rs. 3,000
Total Variable Cost Fixed Cost
Month Output Total Cost Cost per
(Rs. 12 per unit) per Month
(1981) (units) (Rs.) Unit (Rs.)
(Rs.) (Rs.)
September 200 2,400 3,000 5,400 27.00
October 300 3,600 3,000 6,600 22.00
November 400 4,800 3,000 7,800 19.50
December 600 7,200 3,000 10,200 17.00

lllustration: 8.S
From the following data, segregate fixed cost and variable costs.
Management Accounting: 670

Level of Activity
Capacity (%): 80 100
Labour Hours: 400 500
Maintenance Expenses of Plant (Rs.): 2,600 2,750
[ICWA (Int), June 1984J
Solution:
Difference in: Labour Hours = 100, and Maintenance Cost =Rs. 150.
:. Variable element (of Maintenance Cost) =[Rs. 150 + 100 hours] =Rs. 1.5 per Labour Hour.
Fixed element (of Maintenance Cost) =[Total Maintenance Cost - Variable element]
@ 80% capacity, fixed element =[Rs. 2,600 - (Rs. 1.5 x 400 hours)] =Rs. 2,000
@ 100% capacity, fixed element =[Rs. 2,750 - (Rs. 1.5 x 500 hours)] =Rs. 2,000
:. Maintenance Cost =[1.5x + 2,000], where x = Number of Labour Hours.
Maintenance Cost
Capacity Labour· Total Variable
(%) Fixed =
Hours (x) Cost (Rs.) (1.5 x x)
(Total- Variable)
Rs.
80 400 2,600 600 2,000
100 500 2,750 750 2,000

Contribution
Under Marginal Costing, variable costs are consider~ as product costs. On the other
hand, the fixed costs are treated as period costs but not as product costs. Therefore, only the
variable costs are charged against the revenue and the result is the Contribution. That means, the
difference between the total sales revenue and the total variable cost of sales represents the
contribution. Unit contribution is equivalent to the difference between the selling price and the
unit variable cost. Total contribution may also be com:puted by multiplying the number of units
sold by the unit contribution.
Please note from the above that the fixed cost is not considered for the purpose of
computing contribution. This fixed cost is treated as period cost as it goes on accumulating as the
time passes and it is, therefore, charged against the total contribution of the period. That means,
contribution earned during the year is utilized, initially, to recover the fixed cost incurred during
that year and anything left in the contribution (after the recovery of fixed cost) is the profit as
presented below.
Marginal Costing: 671

Treatment of Variable Costs and Fixed Costs


Particulars Prod. A Prod.B Prod. C Total
Sales Revenue SRa SRb SR, • TSR
Less: Variable Cost VC a VCb VCe TVC
.. Contribution Za Zb Ze Total Contribution Fund =Z
Less: Fixed Costs C
Profit P

In brief,
Total Contribution =[Contribution from first unit + Contribution from second unit + ....
+ Contribution from nth unit]
Therefore,
Total Contribution =[Unit Contribution x Number of Units Sold]
And this Contribution is equivalent to the sum of fixed costs and profit. Therefore,
Total Contribution =[Total Fixed Cost + Total Profit]
Further,
[Total Sales Revenue - Total Variable Cost of Sales =Total Fixed Cost + Total Profit]
:. Total Profit =[Total Contribution - Total Fixed Cost]
From the above analysis, two things become very clear. One, company's contribution
comprises of contribution from the sale of all the units of all the products of a company. From
this contribution fund, company's fixed costs are subtracted. Second, if the company's
contribution exceeds its fixed costs, there will be a profit to the extent of the difference. On the
other hand, if the amount of fixed costs exceeds the contribution, the company incurs loss to the
extent of the difference between the two. If the contribution earned is just equal to the amount of
fixed costs incurred, there is no profit and no loss, and therefore, this level of activity is called
break-even-Ievel which is popularly known as break-even point.
Profit-Volume Ratio (P/v Ratio)

Profit-volume Ratio which is popularly known as P/v Ratio establishes the meaningful
relationship between contribution and sales revenue. Therefore, this ratio is also called Marginal
Income Ratio or Contribution to Sales Ratio. For the purpose of computing Plv .Ratio,
difference between the sales revenue and the total variable costs (or the difference between the
unit selling price and the unit variable cost) is used to represent the contribution irrespective of the
composition of fixed costs. Sales revenue includes both cash and credit sales revenue. The ratio
is, therefore, calculated by using the following formulae.
PI R tio = (TOtal Contribution x 1O~ OR
v a Sales Revenue )
Management Accounting: 672

. - (Unit ContributiOll
PI v Ratio 10~
m Se 11·109 Price x
- U·t

Since the Contribution is equivalent to the aggregate of Fixed Costs and Profit,
rFixed Cost + Profit J
P/v Ratio = l Sales Revenue x 1O~

In case where a company incurs loss, Contribution =[Fixed Cost - Loss]. Therefore,

P/v Ratio = (Fixed Cost - Loss x 100 1


Sales Revenue )

P/v Ratio = (I_Total Variable Cos~ x 100


Sales Revenue J
Plv Ratio =[(1 - Variable Cost Ratio) x 100]

Like this a few more formulae are used to determine the P/v Ratio depending upon the
availability of data. Besides, one can also use the following formulae for computing P/v Ratio
under specific situations.
1. As long as the unit selling price and the unit v:ui.able cost remain constant, P/v Ratio can
be computed by the following formula.

P/v Ratio = (ChangeS in .Total Contribution x 100]


Changes 10 Sales Revenue
2. As long as the unit selling price, unit variable cost and tre total fixed cost remain
constant, P/v Ratio can also be computed by dividing the 'changes in profit' by the
'changes in sales revenue' as shown below.

P/v Ratio = ( Changes in Profit x 100 ]


Changes in Sales Revenue
While dealing with Contribution and P/v Ratio, the following points are to be kept in
mind:
a. As long as the unit selling price and the unit variable cost remain constant, the unit
contribution can also be computed by dividing the changes in the total contribution by the
changes in the sales quantity as presented below.

Um·t C ontn·buti· on -_ (ChangeS in Total COntribUtiOn]


. Sales Quantity
Changes 10 .
Marginal Costing : 673

b. As long as the unit selling price, unit variable cost and the total fixed cost remain
constant, the unit contribution can also be computed by dividing the changes in the
amount of profit by the changes in the sales quantity.

Um·t Contrt·b· - [
ubon -
Changes in Total Profit
Changes 10
. Sales Quantity
.
J
3. As long as the unit selling price, unit variable cost and the total fixed cost remain
constant, any change in the amount of contribution changes the amount of profit by an
equivalent amount. That means, Change in Contribution = Change in Profit
4. If a company is operating above the break-even level (i.e., earning some profit), increase
in the contribution increases the profit by an equivalent amount provided the fixed cost
remains constant. That means, there may be some change in either the unit selling price
or unit variable cost or both.
5. When a company is operating below the break-even level (i.e., incurring loss), any
increase in the contribution either reduces the amount of loss or converts the loss situation
into 'no-loss, no-profit' situation or changes the loss into profit situation depending upon
the changes in the amount of contribution provided the fixed cost remains constant.
A Note on P/v Ratio of Multi-product Concerns
In case of multi-product companies, Plv Ratio is computed in the same manner as for
mono-product companies described earlier. The only difference is the consideration of
contribution of the company (i.e., the aggregate of contributions from all the products) in the
numerator of the formula. In the denominator also, total sales revenue earned by the company
from the sale of all its products is considered. Therefore, Plv Ratio (also called. Composite P/v
Ratio) is computed by using the following formula .
. Composite or
Company's Plv Ratio
}
=
rCompany's Total Contribution
l Company's Sales Revenue x tOO
J
Further, one can also compute the Plv Ratio for each product which can be used to assess
the products' profitabilities. For example.
Plv Ratio Of} = rContribution from Product Z x 100
Product Z l
Sales Revenue of Product Z
J
Improvement of P/v Ratio
Since the companies aim at improving their profitabilities, they usually aim at improving
their Plv Ratio. It may be remembered here that the determinants of Plv Ratio are two in number.
They are variable costs and selling price. It is, therefore, necessary to widen the gap between the
selling price and the variable costs. The important ways of improving Plv Ratio are summarized
below.
1. Increasing the selling price without allowing the unit variable cost to increase;
2. Reducing the unit variable cost without downward revision of selling price;
Management Accounting : 674
3. Increasing the selling price at a higher rate than the rate of increase in the unit variable
cost;
4. Reducing the unit variable cost at higher rate than the rate of reduction in the selling price
so that the reduction in the amount of unit variable cost exceeds the reduction in the
selling price;
5. Increasing the selling price and reducing the unit variable cost; and
6. Selecting the most profitable sales mix by increasing the sale of more profitable products
even by reducing the sale of less profitable products.
lliustration: 8.6
The summarized operating results of a company for 2005 are furnished below.
Sales Revenue (10,000 units @ Rs. 10) Rs. 1,00,000
Less: Variable Costs (@ Rs. 8) Rs.80,000
Contribution Rs.20,000
Less: Fixed Cost 15,000
Profit 5,000

A look at the abridged income statement made the chairman disappointed as the company
has realized only 20% P/v Ratio. Therefore, he is seriously thinking of improving the P/v ratio
during the ensuing year 2006. A number of alternatives which he has been considering are as
follows (Note: consider each alternative independently). .
a. Earmark another Rs. 10,000 for advertisement and increase the selling price by Rs. 2,
b. Reduce the variable cost by Re. 1 and the fixed cost by Rs. 5.000,
c. Increase the selling price by 10% and the unit variable cost by 8%,
d. Reduce the unit variable cost by 10% and the unit selling price by 8%,
e. Increase the selling price by 5% and reduce the unit variable cost by 2%
Required: Find out P/v ratio and suggest the alternative which increases the P/v ratio the
most.
Solution:
c omparative I ncome Statement
2005 2006 Alternatives (Rs.)
(10,000 @
Rs.1O;
Particulars (e) SP
Cost, 8x + (a) SP Rs.12 (b) VC Rs.7; (c) SP Rs.ll; (d) SP Rs.9.2;
15,000) Rs.1O.5;
FC Rs. 25,000 FC Rs. 10,000 VCRs. 8.64 VCRs. 7.2
(Rs.) VCRs.7.84

Sales Revenue 1,00,000 1,20,000 1,00,000 I,IO,OOO 92,000 1,05,000


Marginal Costing: 675
Less: Variable
Costs 80,000 80,000 70,000 86,400 72,000 78,400
:. Contribution 20.000 40,000 30,000 23.600 20,000 26,600
Less: Fixed Costs 15,000 25,000 10.000 15.000 15,000 15,000

:. Profit 5.000 15.000 20.000 8.600 5.000 11,600

:. P/v Ratio (%) 20 33 1/3 30 21.45 21.74 25113

From the above, it is evident that alternative (a) is the most profitable alternative.
D1ustration: 8.7
A company has been selling 10,000 units of A, 15,000 units of Band 5,000 units of C at
Rs. 10, Rs. 5 and Rs. 15 each respectively. The unit variable costs come to Rs. 5, Rs. 4 and Rs. 5
respectively. The annual fixed cost of the company comes to Rs. 50,000. With a view to achieve
an improvement in the Plv ratio, the company contemplates to change the present sales mix to
10,000 units of A, 5,000 units of B and 15,000 units of C. Find out whether the changed sales
mix improves the Plv ratio or not.
Solution:
Comparati ve.Tncome Statement
Product
Particulars Total
A B C
Selling Price (Rs.) 10 5 15
Less: Unit Variable Cost (Rs.) 5 4 5
Unit Contribution (Rs.) 5 1 10
Fixed Cost (Rs.) 50,000
Sales Mix: Present (units) 10,000 15,000 5,000 30,000
Proposed (units) 10,000 5,000 15,000 30,000
Present Mix:
Sales Revenue (Rs.) 1,00,000 75,000 75,000 2,50,000
Contribution (Rs.) 50,000 15,000 50,000 1,15,000
.. Plv Ratio (%) 50 20 662/3 46
Proposed Mix:
Sales Revenue (Rs.) 1,00,000 25,000 2,25,000 3,50,000
Contribution (Rs.) 50,000 5,000 1,50,000 2,05,000
.. Plv Ratio (%) 50 20 662/3 58.57
Management Accounting: 676
From the above, it is clear that by selling more units of more profitable product (i.e., C
with 66 2/3% Plv Ratio), it is possible to increase the overall Plv ratio from 46% to 58.57%.
Break-Even Analysis
The term break-even analysis is used in two ways. One is in the narrow sense as dealing
with the determination of break-even point (BEP). Break-even point represents the level of
activity at which the revenue from the sale of goods and services is just equivalent to the total
costs incurred to produce and sell the same. Since the revenue is equal to total cost, the business
entity earns no profit nor it incurs loss. If the sales volume exceeds the break-even volume even
by one unit, the company earns profit. The amount of profit is equal to the product of excess units
sold (over and above break-even volume) and the unit contribution. In the same way, if the sales
volume falls below the break-even volume even by one unit, the company incurs loss. Break-
even Analysis does not aim only at finding out the Break-even Point. Rather, it aims at analyzing
a number of other things (besides the computation of Break-even Point) such as Plv Ratio, Angle
of Incidence, Margin of Safety, Profit or Loss, Absorbed and Unabsorbed Fixed Costs, etc.
Therefore, Break-even Analysis, in its broader sense, refers to the analysis of impact of costs,
price and volume on profit. In other words, it establishes the relationship between cost, price,
volume and profits.
Approaches to Break-Even Analysis

Mono-Product
-+ Companies
f-

Algebraic
~ Approach
Break-even
Break-even -+ Charts
~
Analysis
Graphical
4
Approach

Plv Granh
Multi-Product
-+ Companies f-

Computation of Break-Even Point (Algebraic Approach)


As already explained, Break-Even Point represents the level of activity at which the sales
revenue is equal to total cost. It is, therefore, said that the break-even point is reached _when
the total revenue from sales is equal to total costs. Nonnally, sales revenue includes both cost
and profit. But, at break-even level, sales revenue includes only the total cost as the undertakings
neither earn profit nor they incur any loss. Therefore,
Break-Even 1 (Fixed Costs
Point (units)J = Unit Contribution
J
Marginal Costing : 677

Break-Even} = rFixed Cost~ [


Point (Rs.) L
Plv Ratio = J
Fixed Cost
rContribution
~ales Revenue
JJ= LrFixed Cost
Contribution x
Sales ")
Revenu~

=( Fixed Cost x Unit ]


Unit Contribution Selling Price
=[Break-Even Point (units) x Selling Price per unit]
When a company wants to determine the required sales to earn the desired profit, the
amount of desired profit is to be added to the fixed cost in the above formulae and the aggregate
(which represents the desired contribution) is to be divided by the unit contribution or Plv Ratio
depending upon whether the company wants to determine the required sales quantity or the
required sales revenue. Therefore,
Required SaleSL_ rFixed Costs + Desired PrOfit]
Volume (units~- L Unit Contribution

Required Sale~ _ r Fixed Costs + Desired PrOfit]


Revenue (Rs.)f- L Plv Ratio
In the case of multi-product concerns, the entire fixed cost and the contribution (it has
earned from the sale of all its goods and services) are considered for the purpose of computing Plv
Ratio and Break-Even Point. To put it alternatively, company's Break-Even Point (i.e.,
CGmposite Break-Even Point) is computed by considering the company's fixed costs and its
average unit contribution (i.e., considering the contribution from all its products and units) or its

companY's}
Average
Unit
t
Plv Ratio (also called, Composite Plv Ratio). For computing Plv Ratio, the company's
contribution and its sales revenue are considered. That means,

Total Contribution earned by the ~


company during an accounting period
= Total Number of units of all the products

j
Contribution . sold by the company during that period

~
Company's AVerage} Total Contribution earned by the
(or Composite) company during an accounting period
Plv Ratio = Total Sales Revenue earned by the x 100

l
company during that accounting period

Company's or }
Composite Break-
Total Fixe~ Costs incurre~ by th~
_ company dunng an accountmg penod
J
Even Quantity - Company's Average Unit Contribution
Company's or }
Composite Break-
Management Accounting: 678

Total Fixed Costs incurred by the


_ company during an accounting period
J
Even Sales Revenue -[ Composite Plv Ratio

While computing the Break-Even Point using the above formulae, it is assumed that the
sales rriix of the company will remain constant. Further, the answers obtained by using the above
formulae are correct only when the break-even sales comprise of the sales of all the products in
the proportion given in the problem.
To compute the required sales to earn predetermined profit (Le., desired profit), desired
profit is to be added to the company's fixed cost as shown below.
Required SaleSL _ (Fixed Costs + Desired PrOfitJ
Volume (units~- Average Unit Contribution

Required Sale~ _ [Fixed Costs + Desired PrOfitJ


Revenue (Rs.) J- Composite Plv Ratio
IDustration: 8.8
The following facts and figures are extracted from the cost books of accounts of Vinayaka
Company for the year 2003-04: Sales volume: 18 lakh units; Selling price per unit: Rs. 40; Unit
variable cost: Rs. 25; and Annual fixed costs: Rs. 2.4 crore. From these, compute Break-Even
Point (both volume and value).
Solution:
As an aid for the computation of Break-Even Point, a simple income statement is
prepared.
Sales Revenue (at Rs. 40) Rs. 720,00,000
Less: Variable Cost (at Rs. 25) 450,00,000
Contribution 270,00,000
Less: Fixed Cost 240,00,000
:.Profit 30,00,000

(a) Break-Even} [Annual Fixed Cost) (. Rs. 2,40,00,000 J .


Point (units) = Unit Contributionj =~s.15 (Le., Rs.40 _ Rs.25iJ = 16,00,000 umts

(b) Break-Even}_ (Fixed Cos~


_ [RS.2,40,00,000J _ (Rs.2,40,00,000 720J
Point (Rs.) - LP/v Ratio j - (270 + 720) - 270l x
= Rs. 6,40,00,00Q
Marginal Costing: 679

mustration: 8.9
A company with Rs. 5x + Rs. 60,000 cost structure and with the objective of earning a
profit of Rs. 2 per unit (of sales) sells its product at Rs. 10 a unit. From this, find out the number
of units that it has to sell to achieve the target profit.
Solution:
Let 'x' be the required sales volume to earn a profit of Rs. 2 on each unit sold. Therefore,

_ (Fixed Cost + Desired Profit


x- Unit Contribution
J_ r Rs. 6?,000 + (Rs. 2 x x) I
-lRs. 5 (I.e., [Rs. 10 - Rs. 5rtJ

5x =60,000 + 2x; :. 5x - 2x =60,000; 3x =60,000;


x =[60,000 + 3]; :. x =20,000 units
mustration: 8.10
The following data are made available by Bandu Company for the year 2005; Sales: 10
lakh units @ Rs. 25 a unit, yes: Rs. 150 lakh and FCs: Rs. 60 lakh. During 2006, the company
expects no change either in the prices of input factors or in the selling price of its product or
productivities of input factors. In the light of the actual for 2005 and estimates for 2006, compute
how many units that the company has to sell during 2006 to earn a pre-tax profit of Rs. 75 lakh.
Solution:
Required L_ rfixed Cost + Desired Profi~ = [ (Rs. 60,00,000 + Rs. 75,00,000) I
Sales (units)J - l Unit Contribution Jl Rs. 10 [i.e., 25 - (150 lakh + 10 lakh)]

= lrRs. 135,00,000)
Rs. 10 j = 13,50,000 umts
.

That means, the company has to increase its sales by 35%. (i.e., from 10,00,000 units to
13,50,000 units) during 2006 to earn the desired profit.
mustration: 8.11
A company's turnover in a year was Rs. 50,00,000. Its profit was Rs. 5,00,000 and its Plv
ratio was 40%. What is the break-even point? res (Fin)]
Solution:
1. Computation of Yearly Fixed Cost
Contribution =[Fixed Cost + Profit]
(Sales Revenue x Plv Ratio) = (Fixed Cost + Profit)
(Rs. 50,00,000 x 40%) = [Fixed Cost + Rs. 5,00,000]
Rs. 20,00,000 =[Fixed Cost + Rs. 5,00,000]
:. Fixed Cost = [Rs. 20,00,000 - Rs. 5,00,000] = Rs. 15,00,000
2. Calculation of Break-Even Sales Revenue
Management Accounting : 680

Point (Rs.) f-
Break-Evenl_ [ Fixed Cost J _ [RS. 15,00,000J
Plv Ratio - 40% =Rs. 37,50,000

mustration: 8.12
Sales of XYZ company were Rs. 30,000 producing a profit of Rs. 800 in a week. In the
next week, sales amounted to Rs. 38,000 producing a profit of Rs. 2,400. Find out the break-even
point. [M.Com., UoM, May 1986J
Solution:
Week Sales (Rs.) Profit (Rs.) Contribution* Fixed Cost (Rs.t
I 30,000 800 Rs. 6,000 5,200
II 38,000 2,400 Rs.7,600 5,200
Difference 8,000 1,600

*Plv RatIo
. =[ I Increase in Profit
. Sal J = (Rs. 1,600 x 100~ =20% of Sales Revenue
ncrease m es Revenue x 100 Rs. 8000
,
+ Fixed Cost =(Contribution - Profit)

Because, Contribution =(Fixed Cost + Profit)


Weekly Fixed Cost =Rs. 5,200; Monthly Fixed Cost =Rs. 20,800 and
Yearly Fixed Cost =Rs. 2,70,400

(a) ~::kl:o!~e;:~.} = ~~~dR~~~J = (Rs'2~~0~ =Rs. 26,000


(b) Monthly Break-l
Even Point (Rs.lf
rlRs. 20%
20,80QI
j = Rs. 1,04,000

(c) Yearly B~eak- 1


= [RS.2,70,400J = Rs. 13 52000
Even Pomt (Rs.)J 20% ' ,
mustration: 8.13
Kamaran Company Limited manufactures and sells four types of products under the
brand names P, Q, R and S. The sales mix in value comprises 33 113,41213, 16213 and 8 113 of
products P, Q, R and S respectively. The total budgeted sales (100%) are Rs. 60,000 per month.
Operating costs are:
Variable costs: Product P: 60% of the selling price
Q: 68% of the selling price
R: 80% of the selling price
S: 40% of the selling price
Marginal Costing : 681

Fixed cost: Rs. 14,700 per month.


Calculate the break-even point for the products.
[M.Com, UoM, May 1985J
Solution:
Sales
Sales Revenue Variable Contribution Composite
Product
(Rs.)
Plv Ratio Revenue
Cost (Rs.) (Rs.) Plv Ratio
Ratio(%)
P 20,000 12,000 8,000 40% 331/3 13 1/3
1
Q 25,000 17,000 8,000 32% 412h 13 h
162/3 1
R 10,000 8,000 2,000 20% 3 h
1
S 5,000 2,000 3,000 60% 8h 5
60,000 39,000 21,000 35% 35%

Composit~ rTotal Contribution


Plv Ratio f =lTotal Sales Revenue x 100 =
J (RS.21,000
Rs. 60,000 x 100
J= 35%
Note: Composite Plv Ratio can also be computed by finding out the sum of product of each
product's Plv Ratio and Sales (value) Ratio as shown in the table (see, last column).

Company's Monthly
Break-Even Point (Rs.if"
1 r Fixed Cost
=lComposite Plv Ratio = l
J r Rs. 14,7001
35% J =Rs. 42.000
Sales Revenue Plv Ratio Contribution

~
P(33113% Rs.14,000 40% Rs.5,600
Q (412/3%) 17,500 32% 5,600
Rs.42.000 .
'---------' R (16 2/3%) 7,000 20% 1,400
S (8 113%) 3,500 60% 2,100
42,000 14,700
Since the contribution from Rs. 42,000 sales revenue is equal to the company's Fixed
Cost. Rs. 42,000 is the composite Break-Even Sales Revenue.
Illustration: 8.14
The promoters of a company are interested in the introduction of Fully-Automatic Plant
(FAP) or Semi-Automated Plant (SAP) for the manufacture of quality bicycles. The details of
costs are given below.
Management Accounting : 682
FAP SAP
Fixed costs per annum (Rs.) 70,00,000 30,00,000
Variable cost per unit (Rs.) 300 500

It is estimated that sales would normally be 50,000 bicycles per annum. Due to high
quality maintenance, the promoters decide to have a selling price of Rs. 1,000 per bicycle.
Required:
1. Break-even sales for each plant,
2. Sales level where both the plants are equally profitable, and
3. The range of sales where one plant is more profitable than the other.
[M.Com, UoM, May 1983J
Solution:
FAP SAP
1. Break-Evenl_ ( Fixed Cost. ) = RS. 70,00,000) RS. 30,00,000)
Quantity J
-lUnit Contribution ( Rs.700 = ( Rs.500
= 10,000 bicycles = 6,000 bicycles
2. Let 'x' be the sales level at which both the alternatives are equally profitable.

x=
(
i~;~~;~:st
Difference in Variable Cost
J( =
(Rs. 70,00,000 - Rs. 30,00,000>
(Rs. 500 - Rs. 300) (or Rs. 700 - Rs. 500U
"I'
or Unit Contribution

= (RS. 40,00,000J = 20 000 b· I


Rs. 200 ,1CYC es

3. Up to 20,000 bicycles, SAP is more profitable because of lower fixed costs and above
20,000 bicycles, FAP is more profitable due to higher Plv Ratio. At 20,000 bicycles, both
are equally profitable. This is evident from the following comparative income statement.
c omparabve Income Statement
19,000 bicycles 20,000 bicycles 21,000 bicycles
Particulars (Rs. 00,000) (Rs. 00,000) (Rs. 00,000)
FAP SAP FAP SAP FAP SAP
Sales Revenue 190 190 200 200 210 210
Less: Variable Cost 57 95 60 100 63 105
:. Contribution 133 95 140 100 147 105
Marginal Costing : 683

Less: Fixed Cost 70 30 70 30 70 30


:. Profit 63 65 70 70 77 75

lliustration: 8.1S
X and Y are two similar plants under the same ownership and the owners wish to merge
these two for the purpose of better operation. The details are furnished below.
Plant X Plant Y
(Rs.OO,OOO) (Rs. 00,000)
Sales revenue 0 100 90
Variable cost 75 60
Fixed cost 15 20
Capacity utilization 80% 60%
Required:
a. Find out the Break-Even Point for each plant and for the merged plant;
b. Find out the capacity of the merged plant for break-even;
c. Compute the turnover that the merged plant has to earn to earn a profit of Rs. 60 lakh; and
d. Compute the profitability of the merged plant at 100% capacity utilization.
Solution:
Income'Statement at 100% Capacity Utilization
Plants (Rs. 000) Merged
Particulars Plant
X Y (Rs.OOO)
Sales Revenue (Rs.100 1akh/80%); (Rs.90 1akh/60%) 12,500 15,000 27,500

. [ Rs. 75 lakh
Less: Van able Cost Rs. 1001akh x Rs. 125lakh
~

[RS.60Iakh
Rs. 90 lakh x Rs. 150 lakh
~ 9,375 10,000 19,375

:. Contribution 3,125 5,000 8,125


Less~ Fixed Cost 1,500 2,000 3,500
Profit 1,625 3,000 4,625
Plv Ratio 25% 331/3% 29.545%
Management Accounting : 684

(a) Break-EvenL_ rFixed Cos~


Point (Rs.) f
-lP/v Ratio J
For Plant A, BEP (Rs.) = (RS. li5~~'OOO J = Rs. 60.00,000

Plant B, BEP (Rs.) = (Rs3;Oi~~~00 J = Rs. 60,00,000

Merged Plant, BEP (Rs.) = (R;~~5~~~00 J = Rs. 1,18,46,154


(b) Break-Even Capacity of the Merged Plant =

r Break-Even Sales Revenue. x 100


~ales Revenue at 100% CapacIty
1
:J = [RS. 1,18,46,154 x lOJ =43.08%
Rs.2,75,00,000 )
(c) Let 'R' be the sales revenue that the merged plant has to earn to earn a profit of Rs. 60
lakh. Therefore,
R _ rfixed Cost + Desired Profi~ = r Rs . 35,00,000 + Rs. 60,00,000J
-l P/v Ratio J l 29.545%
= Rs. 321.53.846
(d) Profitability of the Merged Plant at 100% Capacity Utilization =
. r Profit at 100% Capacity. x 100J = [RS. 46,25,000 x
~ales Revenue at 100% CapaCIty Rs.2,75,00,000
lO)J = 16.82%
Graphical Approaches to Break-Even Point
The inter-relationship between costs, revenue, volume, profit, etc., can effectively and
lucidly be presented graphically. For proper presentation, two types of graphs are available.
They are Break-Even Charts and P/v Graphs. Further, a Break-Even Chart (BEC) can be
constructed in more than one way. Of course, all these graphs show the important aspects such as
costs (both variable and fixed), revenue, profit or loss, break-even point, etc. But, each differs
from others by laying more emphasis on one or the other aspect. For instance, one type of chart
lays emphasis on absorption of fixed costs, another chart on the profit or loss and so on. Anyhow,
there are three ways to prepare a Break-Even Chart for mono-product companies depending upon
the variables used. They are as follows.
a. Using Variable Costs, Total Costs and Sales Revenue figures;
b. Using Fixed Costs, Total Costs and Sales Revenue figures; and
c. Using Fixed Costs, Contribution and Sales Revenue figures.
Marginal Costing : 685

Preparation of Break-Even Chart using Variable Costs, Total Costs and Sales Revenue
Figures
For the purpose of preparing a Break-Even Chart, the following procedure is normally
followed.
1. Draw a horizontal line, popularly known as X-axis or ordinate, and space it into equal
distances. This line is used to represent the levels of activity which may take the form of
output or sales volume or both or capacity utilization in terms of %age or labour hours or
machine hours, etc.
2. On the left extreme of the horizontal line, draw a vertical line, also called Y-axis, usually
above the horizontal line. Vertical line is also spaced into equal distances and is used to
represent costs, revenue and profit or loss;
3. Compute the total variable costs, total costs and sales revenue at zero level of activity and
at other levels of activity. Normally, these figures are presented in a table called cost-
volume-profit table.
4. Take the sales revenue figures and plot them on the graph against the corresponding sales
quantity. Join all the plotted points with the help of a straight line starting from zero
point in the left hand corner (i.e., the point of origin). The line is called sales revenue
line. Origination of the line from zero point indicates that the sales revenue is equal to
zero if sales volume is equal to lero. Since the selling price is assumed to be a constant
one, sales revenue is represented by straight line;
5. Similarly, plot the total variable cost figures against the corresponding levels of activity
and join these plotted points with the help of a straight line. This line also starts from the
point of origin and it is also a straight line implying the constant unit variable costs
irrespective of the levels of activity. As in the case of sales revenue line, origination of
total variable costs line from zero point implies zero rupees of total variable cost when
the number of units produced and sold is equal to zero;
6. Next step is to plot the total cost figures at different levels of activity starting from zero
level at which the total cost is equivalent to total fixed cost. Because, at zero level, total
variable cost is equal to zero. But the fixed cost is to be incurred whether there is
production or not. Hence, at zero level, total costs comprise of only the fixed costs.
Once the total cost figures are plotted, these plotted points are to be joined with a straight
line. This line starts at the total cost point on the Y-axis (i.e., total cost when output and
sales = 0) and moves to the right. Because, at '> 0' levels of activity, the total costs
include both variable and fixed costs;
7. Now, looking at the graph, intersection of total cost line and sales revenue line can be
found. The point at which both the total cost line and sales revenue line intersect or break
evenly is called break-even point (BEP). From this point, if a line perpendicular to X-
axis is drawn, it will read the X-axis at some volume and this represents the break-even
volume or break-even quantity (BEQ). In the same way, if another line, from break-even
point (and left to BEP), parallel to X-axis or perpendicular to Y-axis is drawn, it will read
the Y-axis at some value representing both the total cost and the sales revenue as
Management Accounting : 686

equalling. Therefore, this is the break-even revenue. Since the total cost is equal to total
sales revenue, there is no profit and no loss. Hence, this is the break-even point. The
area (between total cost and sales revenue line) left to break-even point is the loss area
and the area (between total cost line and sales revenue line) right to break even point is
the profit area. Loss is nothing but unabsorbed fixed costs. Because, contribution from
sales is not adequate to cover the entire fixed costs. Fixed cost equivalent to the
contribution is absorbed and the rest is left unabsorbed representing the loss. The
intersection of total cost line and sales revenue line creates an angle called angle of
incidence and it sheds light on the profit earning capacity of the company, once it crosses
the break-even point. The difference between the actual or anticipated sales and the
break-even sales (either the physical units or the monetary units) represents the margin
of safety (in units or rupees respectively).
The above procedure can be understood with the help of an example. Therefore, assume
that a company sells its product at Rs. 10 a unit and incurs variable costs of Rs. 7 a unit and fixed
cost of Rs. 15 per period. Further, the company has the capacity to produce 10 units per period
and whatever it produces can be sold at the existing price. From these data, a break-even chart
can be prepared as presented below.
Cost-Volume-Profit Table
Sales Sales Variable Fixed Total Contri-
Profit
Quantity Revenue Cost Cost Cost bution
(Rs.)
(units) (Rs.) (Rs.) (Rs.) (Rs.) (Rs.)
0 0 0 15 15 0 - 15
1 10 7 15 22 3 - 12
3 30 21 15 36 9 -6
-5 50 35 15 50 15 0
8 80 56 15 71 24 9
10 100 70 15 85 30 15

On the basis of the above figures and the procedure enumerated hitherto, the following
chart is prepared.
Marginal Costing: 687
Break-Even Chart - I

y SR Line
Profit
TC Line
-;;;100
~ Fixed Costs
oE 80
e VC Line
c..
"1:1
C
'"
U
::I
C
60 +R"
U
> 40
~ Costs
..:
In
0
u 20
. . - MIS (units) ~

4 6 8 10 x
Sales Volume (units)

In the chart, total cost line is placed above the sales revenue line indicating the fact that
the costs exceed the revenue at the initial stage due to the high incidence of fixed costs. For
instance, look at the cost-volume-profit table. When there is no output, the company has to incur
a fixed cost of Rs. 15 but there is no revenue and therefore, there is no contribution. So, the entire
fixed cost is left unabsorbed which is nothing but loss. If the company is able to sell one unit
which earns a revenue of Rs. 10 and a contribution of Rs. 3, it is able to reduce the loss by Rs. 3
(Le., equivalent to the contribution) leaving the rest Rs. 12 of fixed cost as unabsorbed and this
represents the loss. Like this, unabsorbed fixed cost or loss goes on decreasing as the volume of
sales increases. By selling 5 units, the company is able to earn a contribution which is adequate to
recover the entire fixed cost. Therefore, unabsorbed fixed cost or loss is equal to zero at 5 units.
This is the level at which both the total cost line and the sales revenue line break evenly. This
point (B) is called Break-Even Point (BEP) and it reads the Y-axis at Rs. 50 indicating the break-
even sales revenue as equal to Rs. 50. The same point also reads the total cost at Rs. 50 and
therefore, the difference between the sales revenue and total cost is equal to zero. That means,
loss or profit is equal to zero. Thus, the entire fixed cost is absorbed and therefore, absorbed fixed
cost comes to Rs. 15 (equivalent to total fixed cost) and unabsorbed fixed cost comes to zero
rupees. Up to this level (Le., break...even level), total cost line is placed above the sales revenue
line and therefore, up to this level, the company incurs loss. At point 'B', the company's sales
revenue is equal to its total cost and therefore, there is no loss and no profit. If the sales volume
increases even by one unit, beyond this break-even level, the company earns profit as the entire
fixed cost has already been recovered from the contribution of the first five units. This profit is
equivalent to the product of unit contribution and excess sales volume over the break-even
volume. Therefore, it can be seen from the graph that, beyond the break-even point 'B', sales
revenue line is placed above the total cost line which indicates earning more revenue than the total
cost incurred.
Management Accounting : 688
Angle of Incidence
Intersection of sales revenue line and total cost line creates an angle called Angle of
Incidence. It is the indicator of the rate at which the company is going to earn profit once it
crosses the break-even point. Though it is not possible to say, from the angle, the rate at which
the company is going to earn profit, it is possible to say whether it is at higher or lower rate
depending upon the degree of the angle. Company with larger angle is going to earn profit at a
higher rate once it crosses the break-even point than the company with smaller angle of incidence.
This point becomes very obvious from the following example.
Break-Even Unit
Selling Price Unit Variable Annual Fixed
Company Quantity Contribution
(Rs.) Cost (Rs:) Cost (Rs.)
(Units) (Rs.)
A 10 9 15 15 1
B 10 3 , 105 15 7
From the above figures, it is obvious that both the companies have to sell IS umts each to
break-even. Once they reach the break-even point, Company A starts earning contribution (and
therefore, profit) at the rate of Re. 1 on each unit sold over and above the break-even quantity. On
the other hand, Company B starts earning contribution (and therefore, profit) at the rate of Rs. 7
on each unit sold over and above the break-even quantity. Hence, the difference in the size of
angle of incidence which is evident from the below presented graphs. One noteworthy point is .
that company A is having a very small angle and company B. a very big angle.
Break-Even Charts - Angle of Incidence
y Company A y Company BpRL'ne
-;r
!:S 200 -;r200 ~ TC
~ e:.. B 4" Line
0.
-g
160 -------- --. \
III : A
~ 120 I
cQ) Q) I
::l I
i 80
C
Q)
>
Q)
I
I
iii II: I
8 40
~
I
U I
II I ..
4 8 12 16 20 X
Sales Quantity (units)

For the reasons cited above, the companies work to achieve larger angle of incidence.
Larger angle is an indication of higher Plv ratio which in tum is an indication of lower variable
cost ratio. On the other hand, smaller angle indicates the lower Plv ratio and therefore, higher
variable cost ratio.
There is also another face to it. This is relating to the angle left to the intersection point
(i.e., break-even point). This indicates the rate at which the company's profit is going to decline if
the demand falls below the break-even point. For instance~ in the case of A company, if the
Marginal Costing : 689

demand falls below the break-even point even by one unit, the company incurs loss at the rate of
Re. 1 a unit. On the other hand, in the case of B company, if the demand falls below the break-
even quantity by one unit, it loses Rs. 7 of contribution and therefore, the company has to incur
loss at the rate of Rs. 7 a unit. It is, therefore, obvious that the company with large angle of
incidence earn more profit if they are operating above the break-even point and if they are
expecting higher demand for products when compared to other companies with smaller angles of
incidence. On the other hand, in the case of the company operating below the break-even level
and expecting a further fall in the demand, a small angled company earns more because it loses
less.
Margin of Safety (MIs)
Margin of Safety (MIs) represents the excess of actual or estimated sales over the break-
even sales. Since it is assumed that the volume of output coincides with the volume of sales,
Margin of Safety may also be computed by finding out the excess production over the break-even
point. In the break-even chart, Margin of Safety is represented by the distance between the break-
even point and the present sales on X-axis or Y-axis. Mathematically,

Margin of Safety} _ rActual or Estimated! _ [Break-E.ven!


(Units) - l
Sales Quantity j QuantIty j
If the company is able to sell all that it produces and if there is no opening stock of
finished goods, Margin of Safety may also be computed by using the following formula.
Margin?f Safety} _ (Actual or Es.timated! _ rBreak-E.ven!
(Umts) - l
ProductIOn j lQuantIty j
Margin of Safety is also measured in terms of monetary units and it can be computed by
using the following formula.

Margin of Safety} _ rActual or Estimated! - [Break-EVen


(Rs.) - l
Sales Revenue j
Sales Revenue
J
It (Le., Margin of Safety) is usually expressed in terms of %age. Therefore,
Margin of } _ r
Margin of Safely (Units)
Safety Ratio - LActual or Estimated Sales (units) x
100J
OR

Margin of } _ r Margin of Safely (Rs.)


Safety Ratio - LActual or Estimated Sales Revenue x
100 J
Margin of Safety, therefore, indicates the ability of the company to continue its activities,
without incurring loss, even in case of fall in its sales. It is an indicator of the stability position of
the company. In other words, it indicates by how much sales may decrease before a company will
Management Accounting : 690
suffer .a loss. That means, Margin of Safety, as already stated, shows how much the company's
sales are ahead of its break-even sales.
Margin of Safety, also sheds light on (he profitability of the companies. Because, the
companies are able to earn profit only when they cross the break-even levels. Because, only the
sales over and above the break-even sales enable the companies to earn profit. Therefore, the
company with high margin of safety is usually able to earn more profit than other companies with
small margins of safety. Of course, the role of the other determinant of profit (viz., Plv Ratio or
Unit Contribution) should not be disregarded. Because, profit is equal to the product ofMargin of
Safety and Plv Ratio or Unit Contribution depending upon whether the Margin of Safety is
measured in monetary unit or physical units. In brief,
Profit = [Margin of Safety (Rs.) x Plv Ratio]
Profit = [Margin of Safety (units) x Unit Contribution]
Alternative Form of Break-Even Chart (using Fixed Costs, Total Costs and Sales Revenue)
In this form of Break-Even Chart, fixed cost line is drawn instead of variable cost line.
Since the fixed cost remains constant irrespective of the levels of activity, fixed cost is
represented by a straight line parallel to X-axis. All the useful information which are obtained
from the first type of Break-Even Chart (except showing of absorbed fixed cost in addition to
unabsorbed fixed cost) can be obtained even from this type of chart. Using the information in the
cost-volume-profit table prepared for the first form of Break-Even Chart, the second type of
Break-Even Chart is prepared, and the same is presented below.
Break-Even Chart - II

y SR Line

Profit

l
----=-100 TCLine
g
~
80 tR
£:
'0
"J
fa 60 Variable Cost
II)

2
II)
:>
40
~
lif
0 "':;"~"'----_t---- _ _ _ _ _ _~_ FC Line
u 20
Fixed Cost
0 2 4 6 8 10 X
Sales Volume (units)

Third Form of Break-Even Chart (using Fixed Costs, Contribution and Sales Revenue)
This type of Break-Even Chart combines the special features of the first two types of Break-
Even Charts. That means, as in the case of the first type of chart, this chart shows both absorbed
and unabsorbed fixed costs separately. Further, instead of variable cost line, contribution line is
Marginal Costing : 691

drawn. As in the case of second form of Break-Even Chart, here also fixed cost line is drawn.
Even with these changes, the third type of Break-Even Chart furnishes all the important
information which the first two types of charts furnish. With this background, the third type of
Break-Even Chart, using the figures in the cost-volume-profit table prepared for the first break-
even chart is prepared.
Break-Even Chart - III
y SR Line

"'100
g
~ 80 Variable Cost
c.
'0
Iii 60
g
c01)
Contribution Line
~ 40
'Ii
8 20 Line
I----,z~-----,:::::>"I'-==;:.....:::u.....-----=:::...........---FC

Fixed Cost
o 2 4 6 8 10 x
Sales Volume (units)

Break-Even Point, in this form of Break-Even Chart, is at the level at which both the
contribution line and the fixed cost line break evenly. That means, break-even point is identified
by looking at the inter-section point of both the contribution line and the fixed cost line. Hence, 5
units, which the point 'B' reads X-axis, is the break-even quantity. The second point which is a
special feature of this type of Break-Even Chart is that the point 'B' reads the Y-axis at Rs. 15.
But this Rs. 15 cannot be interpreted as the break-even sales revenue as in the first two types of
break-even charts. Because, it is only the contribution which the break-even sales produce.
Profit-Volume Graph (P/v Graph)
It may be remembered here that in the case of break-even charts, in order to know the
quantum of profit at a particular level of activity, one has to find the sales revenue from that level
of activity and the total costs to be incurred to operate at that level. The difference between the
sales revenue and the total cost is to be ascertained to find the profit. But in the case of Profit-
Volume Graph (P/v Graph) or simply Profit Graph, emphasis is on the establishment of direct
relationship between profit and the levels of activity. Therefore, it can be said that the Plv Graph
is a simplified Break-Even Chart furnishing only a few but important information. To prepare a
profit graph, fixed costs, profit at a given level of activity and sales figures are used. For
preparing a Plv Graph, the following procedure is followed.
1. Draw a horizontal line. called X-axis, dividing the graph into two equal parts. It
represents the sales;
Management Accounting : 692
2. On the left extreme of the X-axis, a vertical line is drawn, both upward and downward
called Y-axis and yl-axis respectively. Y-axis above the X-axis represents the profit and
the yl-axis below the X-axis represents the loss (i.e., unabsorbed fixed cost);
3. Plot the total fixed cost below X-axis on the left side vertical axis. This represents the
loss when there is no sales. Therefore, loss is equal to fixed cost. Because, the entire
fixed cost is left unabsorbed and this is nothing but loss;
4. Compute the amount of profit at a given volume of sales and mark this profit figure
against that sales volume above X-axis and right to the vertical line already drawn;
5. Joint both the fixed cost point and the profit point with the help of a diagonal line called
profit line;
6. Profit line intersects the X-axis and identify this intersection point by 'B' (representing
the Break-Even Point). This Break-Even Point reads X-axis at some sales denoting the
break-even sales. It also reads the Y-axis at 'zero' representing profit or loss as equal to
zero. The area between the X-axis and the profit line, left to the break-even point, is the
loss area and right to the break-even point is the profit area. And other things are same as
in the case of break-even charts.
With this, a Plv Graph is prepared below using the following forecast operating data of M
company for 2006: Selling Price: Rs. 10 per unit; Variable Cost: Rs. 7 per unit; Fixed Cost: Rs. 15
per year; and Sales Volume: 10 units. Therefore, profit at 10 units of sales =Rs. 15 = [(10 units x
Rs. 10) - (10 units x Rs. 7)] - [Rs. 15] = [(Rs. 100 - Rs. 70 - Rs. 15].
Profit-Volume Graph
Y Profit Line

-- 15
en

;E1O
e
c..
5

0
2 8 10 X

--en

5 +- MIS (Units)-+-

'i1O
0
..J

15

yl
Marginal Costing : 693
Illustration: 8.16
The following figures relate to a company manufacturing a varied range of products.
Year Ended Total Sales (Rs.) Total Cost (Rs.)
31.12.80 22,23,000 19,83,600
31.12.81 24,51,000 21,43,200
Assuming stability in prices, with variable costs carefully controlled to reflect
predetermined relationships and an unvarying figures for fixed costs, calculate: (a) The Plv ratio
to reflect the rates of growth for profit and sales; (b) Fixed cost; (c) Fixed cost %age to sales; (d)
BEP; and (e) Margin of safety for the year 1980 and year 1981.
leA (lnt), November 1982J
Solution:
1980 (Rs.) 1981 (Rs.) Difference (Rs.)
Sales 22,23,000 24,51,000 2,28,000
Total Cost 19,83,600 21,43,200 1,59,600
Profit 2,39,400 3,07,800 68,400

(a) Plv Ratio = [ . Differ~nce in Profit x 100J = [ Rs.68.400 x 100J = 30%


DIfference m Sales Revenue Rs. 2,28,000

(b) Fixed Costs 1980 (Rs.) 1981 (Rs.)


Contribution (30% of Sales) 6,66,900 7,35,300
Less: Profit 2,39,400 3,07,800
:. Fixed Costs 4,27,500 4,27,500
:. Annual Fixed Cost = Rs. 4,27,500
(c) Fixed Cost %age to Sales

For 1980, [~~'~~3~ggO x100J = 19.231%; For 1981, [~~'~~;i~ggox 100 J= 17.442%
. rFixed Cos~ rRs.4,27,5001
(d) Break-Even Pomt (Rs.) = L:P/v RatioJ = 30%L J = Rs. 14,25,000

(e) Margin of 1_ [Sales - Break-Even Sales x 100 J


Safety (%)J - Sales
Management Accounting : 694

For 1980, MIs Ratio =[ Rs. 22,23,000 - Rs. 14,25,000


Rs. 22,23,000 x 100 J= 35.897%

·
For 1981 , MI s Ratio =
[RS. 24,51,000 - Rs. 14,25,000
Rs. 24,51,000 x 100 J= 41.86%
Graphical App~oaches to Multi-Product Break-Even Analysis
Break-Even Chart and Plv Graph may be prepared even for multi-product concerns
showing the Cost-Volume-Profit (C-V-P) relationship besides the Break-Even Point, Margin of
Safety and Angle of Incidence. While dealing with the multi-product concerns, it is assumed that
the products are sold in the standard mix.
Multi-Product Break-Even Chart
The procedure for preparing a multi-product break-even chart is enumerated below.
1. Draw a horizontal1ine called X-axis, and space it into equal distances and it represents
the sales,;
2. On the left extreme of the horizontal line, draw a vertical line above the horizontal line,
called Y-axis. Vertical line is also spaced into equal parts and this line represents the
contribution, fixed costs and profit or loss;
3. Compute Plv Ratio for each of the company's products. Afterwards, arrange the products
in descending order on the basis of the Plv Ratio. Then, determine the cumulative sales
and cumulative contribution;
4. Plot the fixed cost figure and draw the fixed cost line which will be a parallel line to X-
axis;
5. Take the product having the highest Plv Ratio, among the products of the company, and
mark its (cumulative) contribution against its (cumulative) sales. Then take the product
having the second highest Plv Ratio and plot the cumulative contribution against the
cumulative sales. This process continues and ends with the plotting of the cumulative
contribution of the product having the lowest Plv Ratio against its cumulative sales. Join
all the plotted points and this is the individual products' contribution line or path;
6. Draw the average contribution line or path by joining the origin point and the point of
cumulative contribution of the product having the lowest Plv Ratio;
7. The point at which the average contribution line crosses or intersects the fixed cost line is
the Break-Even Point. That means, point 'B' reads the X-axis at the break-even sales;
'8. The area, between the fixed cost line and the average contribution line, left to the break-
even point 'B' is the loss area and the area right to the break-even point is the profit area;
and
Marginal Costing : 695
9. The gap between the current sales and the break-even sales represents the margin of
safety and the angle created by the intersection of fixed cost and average contribution
lines is the angle of incidence.
IDustration: 8.17
M Company is engaged in the production and sale of three products A, B and C. Costs
and revenue figures are given below.
Product Sales (Rs.) Variable Cost (Rs.)
A 1,00,000 60,000
B 1,20,000 96,000
C 80,000 24,000
Fixed costs: Rs. 60,000 per annum. From these facts and figures, construct a Break-even
Chart for M company and identify therein the Break-Even Point, Margin of Safety, etc.
Solution:
From the problem, it is clear that variable cost ratios come to 60%, 80% and 30% for A, B
and C respectively. Hence, the Plv Ratios (1 - Variable Cost Ratio) come to 40%, 20% and 70%
for A, B and C respectively. Based on these, the products are to be arranged in descending order,
and cumulative contribution and cumulative sales revenue are to be computed as shown below.
Cumulative Cumulative
Sales Revenue Contribution
Product Plv Ratio Sales Revenue . Contribution
(Rs.) (Rs.)
(Rs.) (Rs.)
C 80,000 70% 56,000 80,000 56,000
A 1,00,000 40% 40,000 1,80,000 96,000
B 1,20,000 20% 24,00.0 3,00,000 1,20,000
Based on the above, Break-Even Chart may be constructed as presented below.
Management Accounting : 696

Multi-Product Break-Even Chart


y
Individual Products Contribution Line

300
Average Contribution Line

"'0 8240
138
-
.§ c2
::s '-"
III

=5 ~180
c..J
° ...
U O B
~tE
fi £ 120
} Profit
Contribution
1----=:70<......-":7",.:::....-0..:....1------,... FC Line
60
Fixed Cost

o 60 120 180 240 300 x


Sales Revenue (Rs. 000)

From the above, it is clear that point 'B' (Le., the intersection of fixed cost line and
average contribution line) reads the X-axis at Rs. 1,50,000 and therefore, Rs. 1,50,000 is the
company's break-even sales revenue provided the three products are sold in the standard mix as
given in the problem. In other words, the break-even sales revenue of Rs. 1,50,000 comprises of
revenue from' A, B and C in the ratio of 10 : 12 : 8. Let us verify this answer.

BESR Plv Ratio Contribution

-E
A Rs.50,OOO 40% Rs.20,000
Rs. 1,50,000
B 60,000 20% 12,000
(10: 12: 8)
C 40,000 70% 28,000
1,50,000 60,000
Less: Fixed Cost 60,000
Profit or Loss 0

Area (between fixed cost line and average contribution line) left to the Break-Even Point
represents the loss as the company operates below the break-even level. This loss is equal to the
fixed cot at 'zero' level of activity as the entire fixed cost is left unabsorbed. Because, there is no
contribution. This loss goes on decreasing as the sales revenue increases. Because, sales revenue
brings in some contribution towards the recovery of fixed costs. This way, it continues to reduce
and reaches zero unabsorbed fixed costs or zero loss. Increase in the sales revenue over and
Marginal Costing: 697

above this, brings in some profit. Therefore. the area (between fixed cost line and. contribution
line) right to the Break-Even Point 'B' is the profit area. On the graph, Margin of Safety is also
identified and it comes to Rs. 1.50.000 (i.e.: Rs. 3.00.000 - Rs. 1.50.000).
Multi-product P/v Graph
As in the case of mono-product Plv graph, multi-product Plv graph also lays emphasis on
the effect of sales volume on the quantum of profit. Plv graph. therefore, shows the profit figure
at different levels of activity. Besides. it shows the Break-Even Point. Margin of Safety, etc. The
procedure to be followed for constructing the Plv Graph involves the following steps.
1. Draw a horizontal line dividing the graph into two parts and this x-axis or horizontal line
represents the levels of activity; .
2. On the left side of X-axis, draw a vertical line - both downward and upward. This Y-
axis, above X-axis. represents the profit and the yl-axis. below X-axis, represents the
loss;
3. Compute the Plv Ratio for each of the products and arrange them in descending order on
the basis of the Plv Ratio:
4. Compute the cumulative sales revenue and cumulative profit. Cumulative profit is equal
to the difference between the cumulati.ve contribution and the fixed costs [Le., Cumulative
Profit = Cumulative Contribution - Fixed Cost];
5. Mark the total fixed costs below X-axis on the (left side) vertical line;
6. Then, take the product having the highest Plv Ratio, among the products of the company,
and mark its cumulative profit against its cumulative sales revenue. Then take the
product having the next highest (i.e., second highest) Plv Ratio and plot its cumulative
profit against its cumulative sales. This process continues till the marking of the
cumulative profit of the product having lowest Plv Ratio, among the company's products,
against its cumulative sales revenue.
7. Join all the plotted points beginning with the tixed cost point and ending with the
cumulative profit point of the product having the lowest Plv Ratio. This line is called the
individual products' profit line;
8. Then. draw a line joining the fixed .cost point and the cumulative profit point of the
product having the lowest Plv Ratio. This line is called average profit line:
9. The inter-section of X-axis and the average profit line is the Break-Even Point. This
point reads the X-axis at some sales representing the break-even sales. The same point
reads the Y-axis at zero-protit.
10. The area between X-axis and the average profit line. left to the break-even point 'B'. is
the loss area and right to the Break-Even Point 'B' is the profit area. Intersection of
. average profit line and X-axis creates an angle called Angle of Incidence. The difference
between the current sales and the (average) break-even sales is the Margin of Safety.
Management Accounting : 698

Cumulative Cumulative Cumulative


Sales Revenue Contribution
Product Sales Contribution Profit
(Rs.) (Rs.)
Revenue (Rs.) (Rs.) (Rs.)
C 80,000 80,000 56,000 56,000 - 4,000
A 1,00,000 1,80,000 40,000 96,000 36,000
B 1,20,000 3,00,000 24,000 1,20,000 60,000

Multi-Product Plv Graph


y
80

,"",60 B
~
';:'40
£20 . - - - - Average Profit Line

o r---~--~-~--------'
240 ~oo x
Sales Revenue (Rs. 000)
~ MIS (Rs.)-+

Dlustration: 8.18
Evenkeel Ltd., manufactures and sells a single product X whose selling price is Rs. 40 per
unit and the variable cost is Rs. 16 pe~ unit.
(a) If the fixed costs for the year are Rs. 4,80,000 and the annual sales are at 60% margin of
safety, calculate the rate of net return on sales, assuming an income tax level of 40%.
(b) Forthe next year, it is proposed to add another product line Y whose selling price would
,he Rs. 50 per unit and the variable cost Rs. 10 per unit. The total fixed costs are
estimated at Rs. 6,66,~00. The sales mix of X and Y would be 7 : 3. At what level of
sales next year, would Evenkeel Ltd., break-even? Give separately, for both X and Y, the
break even sales in rupees and quantities.
leA (Fin), May 1982J
Marginal Costing : 699

Solution:
(a) Unit variable cost =Rs. 16; Selling price =Rs. 40; Unit contribution =Rs. 24 and
Plv Ratio =60%
Break-Even }
Sales Revenue
= (Fixed C?st
Plv Ratio
J= ~RS.... 4.80.0ooJ
60%
=Rs. 800000
' ,

Annual Sales =[Break-Even Sales + Margin of Safety] =[Rs. 8,00,000 + Rs. 12,00,000]
=Rs. 20,00,000 [Because, Margin of Safety =60% of Sales Revenue].
Therefore, Break-Even Sales Revenue = (1- 60%) = 40% of Sales Revenue
Sales 1. (Break-Even Sales Revenu~1 . (Rs.8,00,00Q1
RevenueJ = LBreak-Even Sales Ratio) = 40%) =Rs. 20,00,000
Sales Revenue Rs. 20,00,QOO
Less: Variable Costs (40%) 8,00,000 Return on Sale = (~~i~~o~~ox 10~ =21.6~
Contribution 12,00,000
Less: Fixed Cost 4,80,000
Earnings before Tax 7,20,000
Less: Tax (40%) 2,88,000
Profit after Tax 4,32,000

(b) Unit Contribution from X =Rs. 24 and from Y =Rs. 40


Let 'x' be the total Break-Even Quantity
At Break-Even Point, Fixed Cost =[Contribution from x + Contribution from y]

Rs. 6,66,600 =( 7X 10
units x Rs. 24 J + [3X 10
units x Rs. 40 J
=(Rs. 16.8x + Rs. 12x) =Rs. 28.8x

(Rs.6.66.6001 =23,146 units (approximately) =Break-Even Quantity


:. X=
l Rs.28.8 )
Management Accounting : 700

Break-Even Break-Even Sales


Quantity (units) Revenue (Rs.)
16.202 6.48.083

----"'-b[ ~31~x
23.146 (units) _ _ X :

Y ttel 0.944
13.146
3.47.187
9.95.270

If the company sells 10 units comprising llr 7 unit" or X and 3 units of Y. the revenue
composition will hc as follows.
7 units of X @ Rs. 40 = Rs. 2XO :. Sale:-. Revenue Ratio = 280: 150
.3 unib of Y (f/ Rs. 50 . = R:-.. ISO
R.,. -U{l
Let 'R' he the total break-even :-.ales reVCnl1l'. :'\nd II '" known that. at break-even level.
Fixed Cost = IContributhm from X + Cuntribution from YI
(Sales Revenue PI\' Ratil~ fSaks Revenue Plv Ratiol
:. Rs. 6.66.600 = L from X x of X j + L from Y x ofY j

f~80R
= L430 x
6 )C']
(If +
n SUR ] [ 168R 120R] [288R]
L-BO x SOlk = 430 + 430 = 430
:. (Rs. 6.66.600 x ·-1-30) = 288R
..
~
:. R = ( (Rs. 6.66.600 x 43e)'
288 ] = Rs. 9.95.271
.
(approxlI11Lllely) = Bre~k-even Sales Revenue
Illustration: 8.19
The Plv ratio of Hansa Ltd is 50% and margin of safety is 40c':;. You are required to work
out the net pwtlt and Break-Even Point if sales volume i~ R". 1O.00.0()O.
[e4 (Fill), May 1974J
Solution:
Net Profit = [Margin of Safety (Rs.) x Plv Rutio I
= IRs) 0.00,000 x 401ff I x 50% = (RsA.()O.OOO x 5091.l = Rs.2,OO,OOO
Break-Even Point (Rs.) = [Actual or Eslimuled Sales - Margin of Safety (Rs.)1
= [Rs. 10.00,000 - Rs. 4'()0.000) = Rs. 6.00,000
Marginal Costing: 701

Assumptions Underlying Break-Even Analysis

The important assumptions underlying the Break-Even Analysis are identified below.
I. Total cosls can accurately be classified II1to fixed cost and variable cost;
2. Linear relationship exists between output on the one hand and (each at) costs and revenue
on the other;
3. Assumptions abolit unit variable cost, lotal fixed cost and selling price hold good within
the relevant range:
4. Volume of output coincides with the volume of sales: and
5. Company produces a single product or if it produces two or more, its sales mix remains
constant even if the total output changes.
Cost-Volume-Profit Analysis
Protit maximization is one of the important objectives of maJonty of corporate
undertakings. This profit is influenced by a large number of factors. These include both the
internal and the external factors. The important factors or determinants are price, sales volume,
output. costs (both variable and fixed), etc. All these determinants influence each other. For
instance, cost influences the price, price influences the demand, demand influences both the
productIOn and the sales, production influences the costs and so on and so forth.
Further, a number of changes take place in costs (both variable and fixed), price. sales
volume. etc .• and these changes will have an impact on profit. Since the profit is one of the
important aspects which draws the attention of managerial personnel, the managerial personnel
would naturally like to know the effects of changes in costs, price. volume. etc., on the company's
profit and on a number of other aspects which shed light on profitability of the company. The
study of the effects of changes in volume, costs, price, etc .• on profit and other aspects is the
subject matter of cost-volume-profit analysis which is popularly and simply known as C-V-P
Analysis. Before proceeding to delineate into the effects of changes in cost, volume and price on
profit. it is better to have a look at the relationship between the Break-Even Analysis and the C-V-
P Analysis. The important opinions about the two are summarized here. Since the term break-
even analysis has the unfortunate implication that the object of the business is merely to
break-even, some experts suggest usage of the term C-V-P analysis.) If this o.pinion is put to a
detailed analysis, it helps to form an idea that the C-V-P analysis is a broader one when compared
to Break-Even Analysis. This fact has clearly been brought out by the opinion of Garrison which
is reproduced here: Cost-volume-profit analysis is sometimes referred to simple as break-
even analysis. This is unfortunate, because break-even analysis is just one part of the entire
cost-volume-profit concept.2 The same view has been expressed, of course in a different tone,
by Horngren which is reproduced here: The study of cost-volume-profit relationships is often
called break-even analysis. The latter is a misnomer because the break-even point - the
point of zero net income - is often only incidental to the planning decision at hand.3
Break-Even Analysis lays more emphasis on break-even point and all other calculations
are centered around this break-even point. Of course. Break-Even Analysis also determines the
effect of changes in the determinants (of profit) on profit but the emphasis is on the effect of
Management Accounting : 702
changes on the break-even point which alters the margin of safety. Therefore, the effect on profit
is studied. But, in the case of C-V-P Analysis, emphasis is on the profit. More specifically, on
the effects of changes in cost, price, volume, etc., and also the effects of alternative courses of
action on the company's profit. Further, Break-Even Analysis appears to be a static one. That
means, Break-Even Analysis considers the costs (both variable and fixed), price, etc., at a
particular level of activity. On the other hand, C-V-P Analysis incorporates, to the Static Break-
Even Analysis, the changes in the determinants of profit and studies the effects of these changes
on profit. To put it alternatively, C-V-P Analysis also studies the Break-Even Analysis as the
knowledge of Break-Even Analysis provides a greater insight into the pros and cons of alternative
courses of action.
Usually, the companies plan for the profit to be earned during a year. They spell out
clearly the detailed plan as to how the planned profit is to be achieved. When they end up the
year, they normally fmd some difference between the actual profit earned and the. budgeted profit
planned. Even if a few companies are able to achieve the profit they had planned, they will find
some deviations or changes in the activities than budgeted. Any change in the profit is primarily
due to the changes in four important factors. They are selling price, variable costs, fixed costs and
volume of activities.4 The C-V-P Analysis deals with assessing the effects of changes in these
four factors on the profit and other variables. In brief, this aims at quantifying the effects of
changes in fixed costs, variable costs. selling price and sales quantity on the profit and other
aspects of the companies.
Effects of Changes in Fixed Costs·
There are a number of mis-conceptions about fixed costs. The most important one is the
view that the fixed costs do not alter and they remain constant irrespective of the changes in the
levels of activity. This is not true. Because, fixed costs are also subject to changes depending
upon the determinants. The most pertinent question now is, what will be the effects of changes in
the amount of fixed costs on the company's break-even point. contribution, Plv ratio, margin of
safety, profit. etc. In order to study the effects of changes in fixed costs. it is assumed that both
the selling price and the unit variable cost will remain constant. A comprehensive analysis of
effects of changes in fixed cost reveals a number of things. Only the important are enumerated
herein under.
a. Any change in fixed cost changes the Break-Even Point by an equivalent %age. Further,
both the fixed costs and the Break-Even Point move in the same direction. That means,
increase in the fixed cost increases the Break-Even Point and vice-versa;
b. As the change in the fixed cost changes the Break-Even Point, it can be said that the
change in the fixed cost changes the Margin of Safety. Because, any change in the Break-
Even Point changes the Margin of Safety.
c. Amount of profit is influenced by both the Plv Ratio or Contribution and the Margin of
Safety. Further, any change in the amount of fixed cost changes the Margin of Safety. It
may therefore, be said that the change in the amount of fixed cost changes the amount of
Profit. Further, both the amount of profit and the Margin of Safety move in the same
direction and normally. in the same proportion; and
Marginal Costing : 703

d. Since Plv Ratio = [(Contribution + Sales Revenue) x 100], fixed cost is not a detenninant
of either the numerator (viz, Contribution) or the denominator (viz., Sales Revenue) of the
Plv Ratio formula and therefore, any change in the fixed cost has no effect on both the
Contribution and the Plv Ratio.
Effects of Changes in Unit Variable Costs
Changes take place in the unit variable costs due to the rise in prices and also due to
ochanges in the productivities of variable input factors. Now the question is, 'what are the effects
of these changes in unit variable cost on the company's Break-Even Point, Profit, Plv Ratio, etc?
In other words, managerial group is interested in finding out the effects of changes in the unit
variable cost on the company's cost structure, profit performance, break-even point, margin of
safety, etc. In order to study the effects of changes in unit variable cost, it is assumed that the
changes in unit variable cost are not followed by corresponding changes in other things such as
selling price, sales volume, total fixed cost, etc. The important effects of changes in the unit
variable cost, assuming other things as remaining same, are summarized below.
a. Whenever there is a change in unit variable cost, there will be a change in both total
variable costs and variable cost ratio;
b. Change in unit variable cost changes the unit contribution, total contribution and Plv ratio.
Because, change in unit variable cost changes both the total variable cost and the variable
cost ratio;
c. On the one hand, change in the unit variable cost changes one of the detenninants of
Break-Even Point viz, unit contribution or plv ratio. On the other hand, it is assumed that
no change in another detenninant of Break-Even Point viz., fixed cost will take place.
Consequently, any change in unit variable cost changes the Break-Even Point.
d. Since the change in the unit variable cost changes the Break-Even Point (without
corresponding change in the actual sales), Margin of Safety is bound to change whenever
there is a change in unit variable cost; and
e. Profit is the product of Margin of Safety and Unit Contribution or Plv Ratio or
contribution per % capacity utilization depending upon the unit in which the Margin of
Safety is expressed. It has also been proved that the change in unit variable cost changes
both the Margin of Safety and Plv Ratio and therefore, profit is bound to change.
Effects of Changes in Selling Price
Price of any product is influenced by a number of internal and external factors. Further,
once the price is fixed for a product, the same cannot be kept for ever. That means, the price so
fixed is to be revised in the light of competition, cost hikes, corporate objectives, etc. Prices are
also revised by the companies to achie~e the desired result. Hence, it is necessary to study the
effects of changes in the selling price on the company's profit, break-even point, margin of safety,
etc. In order to draw conclusions about the effects of changes in the selling price on profit, break-
even point, etc, it is assumed that the change takes place only in the selling price. When there is
no change in either the unit variable cost or the total fixed cost or the sales quantity, the change in
the selling price will have the following effects.
Management Accounting : 704
a. Any change in the selling price changes the amount of sales revenue.
b. As the change in the selling price alters the gap between selling price and unit variable
cost, there will be a change in the variable cost ratio, unit contribution, total contribution,
plv ratio, unit profit and the total profit. To put it differently; the increase in the selling
price increases the revenue, unit contribution, total contribution, unit profit, total profit
and plv ratio, and reduces the variable cost ratio. On the other hand. decrease in the
selling price reduces revenue., unit contribution, total contribution. unit profit, total profit
and plv ratio, and increases the variable cost ratio; and
c. Since the change in the selling price changes the variable cost ratio, there will be a change
in the contribution and plv ratio. Consequently. there will be a change in the break-even
point. This change in the break-even point alters the margin of safety and finally, the
quantum ,of profit.
Effects of Changes in Sales Volume
Sales volume is one of the important determinants of profit and there will be frequent
fluctuations or changes in the sales quantity. And, therefore. managerial personnel are required to
study the effects of changes in sales volume on the company's break-even point. plv ratio. protit,
etc. Once again. it is assumed no change in other variables viz., unit variable cost. selling price
and total fixed co·st. Assuming these, the effects of changes in sales volume on various aspects-
are presented below.
a. Since there is no change in the unit variable cost and total fixed cost, mere change in the
sales volume does not 9hange either the unit contribution or the variable cost ratio or the
plv ratio or the break-even point;
b. Increase in the sales volume increases the margin of safety by an equivalent amount or
units and vice-versa; and
c. Since any change in the sales volume changes the margin of safety. it may be said that
any change in the sales volume changes both the total contribution and the profit.
Because. profit is the product of margin of safety and unit contribution or Plv ratio.
In 'the above paragraphs, the effects of changes in anyone of the four important
determinants of protit are studied by assuming other three determinants a<; remaining constant. In
simple, while studying the effects of changes in selling price on the break-even point. profit.
margin of safety, etc, it is assumed thm no change takes place in other three important variables
viz .• unit variable cost. total tixed cost and sales quantity. But in reality, changes take place in
two or more determinants of protit at a time. Naturally. the management wishes to study the
effects of changes in the selling price or the unit variable cost or the total fixed cost or the sales
quantity or any combination of all these variables on the break-even point, profit. margin of
safety, etc. The extent to which the changes in these determinants influence the profit depends
upon the effects of each of the changes in the determinants.
mustration: 8.20
-a. Sales Rs.5,OO~OOO: Fixed Cost Rs.l,50.000; Profit: Rs.I.OO,OOO. Calculate the Margin of
Safety [Kuvelllpu UIli., B.Com., May 1992J
Marginal Costing : 705

b. Find out Break Even Point in units and value from the following:
Sales 10,000 units at Rs.20 per unit
Variable cost Rs.1O per unit
Fixed cost Rs.80,OOO [Kuvempll Uni, B.Com, May 1991}
c. From the following particulars. find out the seIling' price per unit if BEP is to be brought
down to 9,000 units:
Variable cost per unit Rs.75
Fixed expenses Rs.2,70.000
Selling price per unit Rs.1 00. [KlIvempu Ulli., B.Com., May 2002}
d. From the following, calculate Break even point:
Selling price per unit:. Rs.l 0
Direct materials per unit: Rs.3
Direct labour per unit: Rs.2 .
Variable overheads: 100% on labour
Fixed overheads: Rs.I 0,000
e. Variable cost Rs.50.000, Fixed cost Rs.30.000 and Profit R.... IO.O()O. Calculate the amount of
sales. IBallga/ore U,,;., B.Colll.• May 2002}
f. Sales Rs.50,OOO. Fixed cost Rs.IO,OOO and Profit Rs.5.(J()O. Calculate Variable Cost.
[Ball1:a/ore U,,;.• B.Com., October 2()02}
g. Given sales is Rs.12.50 lakshs. its variable costs arc Rs.HJ.03 lakhs and loss is R.,.I.96lakhs.
find its fixed costs. IBllll1:a/ore UIl;.• H.Colll .. May 20001
h. Sales Rs.I.50.000, Profit RsAO,OOO. and Fixed cost Rs.30.0()O. Calculate the amount of
variable cost. [Ballga/ore Ulli., B.COI1l., November 2000}

Solution:
a. Contribution = [Fixed Cost + Profit]
= [Rs.I.50,OOO + Rs.1 ,c)O.OOO] = Rs.2'sO.OOO

PlY Ratio = [~(~ntr~ution x 100] = [RS.2.50.000 x 1()(0) =50%


a es evenue Rs.5.00.000

Fixed CostJ [RS. 1.5().dO(~


BEP (Rs) = [ PlY Ratio j = Soc,t j = R.,.3.1 )0.000
Management Accounting : 706

Margin of SafetY (Rs.) = [Sales Revenue - BEP (Rs.)]


= [Rs.5,00,000 -:- Rs.3,00,000] = Rs.2,00,000
b. Unit Contribution =[Selling Price - Unit Variable Cost] = [Rs.20 - Rs.1O] =Rs.1O

PlY Ratio
rUnit Contribution
= LSelling Price
J
x 10~ =
Rs.lO
[ Rs.20 x 1O~
J
=50%

BEP (units) =(u:'lXc:'n~~:tio~ = (__R_~_~_~;_~_OO_J =8,000 units

BEP (Rs.) = ( F:~ ~::~ J= ( RS~~O~~OO J= Rs.l,60,000

c. BEP (units) = 9,000 units and Fixed Cost = Rs.2,70,000. Hence, each unit has to earn a
contribution of Rs.30 [i.e., Rs.2,70,000 + 9,000 units]
Hence, Selling Price =[Unit Variable Cost + Unit Contribution]
=[Rs.75 + Rs.30] = RS.I05
d. Unit Variable Cost = [Direct Material Cost per unit + Direct Labour Cost per unit + Variable
Overheads per unit]
=[Rs.3 + Rs.2 + 100% of Rs.2] =[Rs.3 + Rs.2 + Rs.2] =Rs.7
Unit Contribution =[Selling Price - Unit Variable Cost] =[Rs.1O - Rs.7] =Rs.3

( Fixed Cost
BEP (units) = lUnit ContributiogJ =
I ( Rs.1O,OOO
Rs. 3
J= 3,333.33 units
== 3,334 units.
e. Variable Cost Rs.50,000
Add: Fixed Cost 30,000
:. Total Cost 80,000
Add: Profit 10,000
:. Sales Revenue 90,000

f. Contribution= [Fixed Cost + Profit] = [Rs.10;000 + Rs.5,000] =Rs.15,000


=[Sales Revenue - Variable Cost]
:. Variable Cost =[Sales Revenue - Contribution]
= [Rs.50,000 - Rs.15,000] = Rs.35,000
I
Marginal Costing : 707

g. Contribution = [Sales Revenue - Variable Cost]


=[Rs.12.50 lakh - Rs.l 0.03 lakh] =Rs.2.47 lakh
Contribution =[Fixed Cost + Profit] =[Fixed Cost - Loss]
:.Fixed Cost = [Contribution + Loss] = Rs.2.47Iakh + Rs.1.961akh
= Rs.4.43 lakh = 4,43,000.

h. Contribution =[Fixed Cost + Profit]


=[Rs.30,000 + Rs.40,000] =Rs.70,000
[Sales - Variable Cost] =Contribution
:. Variable Cost = [Sales Revenue - Contribution]
=[Rs.l ,50,000 - Rs.70,000] =Rs.SO,OOO
mustration: 8.21
The cost of production of 5,000 units is given below:

Materials Rs.40,000
Labour 20,000
Overheads 30,000 (60% fixed)
Find out the Marginal Cost in total and per unit and test the equation TC = VC + FC
[Mangalore Uni., B.Com., November 2004}
Solution:
M ar2lDa
. ICost Statement
Cost (Rs.)
Particulars
Total Per unit
Cost of Materials 40,000 S.OO
Labour Cost 20,000 4.00
Variable Overheads (40%) 12,000 2.40
:. Marginal Cost 72,000 14.40
Fixed Overheads (60%) IS,OOO 3.60
:. Total Cost 90,000 IS.00

Hence, the cost equation, Y = mx + c [where Y =Total Cost, m = Marginal Cost per unit,
x = Volume of Output and c =Total Fixed Cost] comes to Y = Rs.14.4x + Rs.1S,000. Therefore,
when x = 5,000 units, Total Cost Y = [(Rs.14.4 Marginal Cost per unit x 5,000 units) +
Rs.lS,OOO] = Rs.72,OOO Marginal Cost + Rs.lS,OOO Fixed Cost] = Rs.90,000 Total Cost.
Management Accounting : 708
Illustration: 8.22
A company has fixed expenses of Rs.90.000. sales Rs.3.00.000 and a profit of Rs.60.000.
Calculate the Profit Volume Ratio. If in the next period. the company suffered a loss of
Rs.30.000. calculate the Sales Volume.
[Kllvempll Uni., B.Col1l., October 1997 and B.B.M., April 19981
Solution:
Current Period:
Contribution =I Fixed Cost + Profit) =I Rs.90.000 + Rs.60.000) = Rs.1.50.000
Contribution ~ Rs.I.50.000 "I
:.P/V Ratio = [ SIR
a es evenuL' x too = ( Rs.3.00.000 x I ()~I = 50%
Next Period:
Contribution = IFixed Cost - Loss I
= I Rs.90.000 - Rs.30'()OO) = Rs.60.000 [i.e. 50(K of Sales Revenue as PlY
Ratio = 50lh I

:.Sale" Re\L'nuL' = [
Contribution
P'\ Ratio
J rl
=
Rs.60.000
SOCk
J = R~.1.20.000

IIIustrution: H.B
Assuming thL' l:ost slnlt·turc and sL'lIing pri«.:L' remain the same in periods I and II. find out:
a. Profit \'olume Ratio
b. Fixed Cost

d. Profit \\ hL'1l SaIL's arlo' l)f RS.i.lI().OO()


l.'. :\1argill (It" Saki) al a profit .If R,.15.0()O
f. SaiL'S l"l'quin:d to earn profit ,'~ Rs.20.00()
g. Variable ("l\,t in Period II
Penllli Sales Profit
Rs.I.20.doo Rs.9,OO()
II I AO.nOO 13.000

IItJallga/ore VIIi., B.Col11., Mll)' 2000. a1ll1 (}(:tobel" 2{)02, 2003 lllld 2004, B.B.M. April and
O(·tober 2()()(), Km'elllplI VII;., B.COIll., May 2()1)J, and October 1998 and 1999, B.B.M., May
1999, allli Ballglt/ol"e Ulli., May alld October 2(J02/
Marginal Costing : 709
Solution:
x 100~ = 20%
" .
a. p,v RatIo =
[ Increase in Profit ~
x 100 =
Rs.4.000
Increase in Sales Revenue [ R~.20.000

b. Fixed Cost = [Contribution - Profit]


Period I Period II
:.Contribution [20% of Sales] Rs.14.000 Rs.2KOOO
Less: Profit 9.000 1.1.000
:. Fixed Cost 15.000 15.000

c. BEP (Rs.) =[
Fixed Cost
P''''
IV R'atIo
j=[ Rs.15.000
20%
J= Rs.75.000

d. Sales Revenue Rs.I,OO,OOO


Les-s: Variable Cost at 80% of Sales [I - 20% P1V Ratio] 80.000
:. Contribution 20.000
Less : Fixed Cost 15.000
:. Profit 5,000
e.
Rs.15.000
20%
J = Rs.75.000

f. Required SalesL = rFixed Cost + Desired Profit] = r Rs.l5.000 + Rs.20.000 I


Revenue (Rs')f l P1V Ratio [ 20% J
Rs.35,000
~I
_ 9('
?O
J = Rs.1.75.000

g. Variable cost} [sales Revenue Variable Cos~


in Period - II = in Period· II x Ratio (it J
= [Rs.I.40.000 x 80%] = Rs.I, 12.000
Illustration: 8.24
You are given the following information:
Management Accounting : 710

Period Sales Profit


March 2001 Rs. 90,000 -Rs.lO,OOO
June 2001 1,30,000 + 10,000
Calculate:
a. PNRatio d. Profit when sales are Rs.l,80,000
b. Fixed and Variable Expenses e. Margin of Safety
Sales Volume to earn Profit of Rs.25,000
C.
{Mangalore Un;', B.Com., April 2001 and 2003, and Bangalore Un;', B.Com., November 2003}
Solution:

Increase iIi Profit ~


a. PN Ratio - [ Increase in Sales Revenue x 100
J

= [
Rs.lO,OOO - (-Rs.lO,OOO) l
[RS.20,000
Rs.1,30,000 - Rs.90,000 x lOOJ = Rs.40,000 x 100
J = 50%
b.
March 2001 June 2001
Sales Revenue Rs. 90,000 Rs. 1,30,000
Less: Variable Expenses at 50% (1- PN
45,000 65,000
Ratio)
:. Contribution 45,000 65,000

Less: Profit (add Loss) -10,000 10,000

:.Fixed Expenses 55,000 55,000


c.
~~.~i;:=~si~l,~~;} = [Fixed Cost + Desired Profi~ = [DeSired Contribution"'1
Profit per Month . PN Ratio J PN Ratio J
Rs.55,000 + Rs.25,000 J (RS.80,QOO J
=[ 50% J = 50% =Rs. 1,60,000 (per month)
. d. 'Sales Revenue Rs. 1,80,000
Less: Variable Cost (50%) 90,000
:. Contribution (50%) 90,000
Less: Fixed Cost 55,000
:.Profit 35,000
Marginal Costing : 711

e. BEP(Rs.) = [ Fixed Cost


PN Ratio
J= [
J
Rs.55,000
50%
J = Rs.I,IO,OOO

Margin of Safety = [Sales Revenue - Break Even Sales]


:. Margin of Safety for the month of :
March 2001 = [Rs.90,000 - Rs.I,lO,OOO] = - Rs.20,000
June 2001 = IRs. 1,30,000 - Rs.l,lO,OOO] = Rs.20,000
mustration: 8.25
A company furnishes you the following data relating to 1999.

First Half Second Half


Sales Rs.45,000 Rs.50,000
Total Cost 40,000 43,000
Assuming that there is no change in prices and variable costs and that the fixed expenses
are incurred equally in second half, calculate;
a. PN Ratio
b. Fixed Expenses
c. BE Sales
d. Percentage of rYlargin of Safety
[Kuvempu Uni., B.Com., October 2001J
Solution:
First Half Second Half Whole Year
Sales Revenue Rs.45,000 Rs.50,000 Rs.95,000
40,000
Total Cost 43,000 83,000

:.Profit 5,000 7,000 12,000

Increase in the Amount of Profit ~


a. PN Ratio = ( Increase in the Amount of Sales Revenue x 10)
Rs.7,000 - Rs.5,000 1 [RS.2,000 l
= [ Rs.50,000 _ Rs.45,000 x 1O~ = Rs.5,000 x 1O~ =40%
:. Variable Cost Ratio = [1 - PN Ratio] = [1 - 0.4] = 0.6 = 60%
Management Accounting: 712

b. First Half Second Half Whole Year


Total Cost Rs.40.000 Rs.43,OOO Rs.83,OOO
Less: Variable Cost [60%
of Sales Revenu~] 27,000 30,000 57,000

:. Fixed Cost (b) 13,000 13,000 26,000


-- -- --
:. Fixed Costs for six-month period = Rs.13,OOO
one year = Rs.26,000

c. Break-Even} _ r. Fixed Cost J


Sales (Rs) -l PN Ratio J

J -lr Rs.13,000
.Six - monthly BreakL _
.. Even Sales (Rs.) 40%
J .
=Rs.32.5oo
Yearly Break } - r Rs.26,OOO J
-l 40%
Rven Sllle... (Rs.) = Rs.65,000

d. Per~ent~ge
of } = [Marg~nal of Safety x 100J
Margm of Safety Sales Revenue
:. MIS Ratio for:
. [RS.45,OOO - Rs.32,500 . J
Fist Half: x 100 =[ Rs.12,500
x 100
J
=27.78%
Rs.45.000 Rs.45,ooO

Second Half: [
Rs.50,000 - Rs.32,500
Rs.50,000 x 10)
J = [Rs.50,000
Rs.17,500
x lOOJ
l =35%

Whole year: rR~.95,OOO


L
- Rs.65,OOO x 100J
Rs.95,OOO
= r Rs.30,OOO
LRs.95,OOO
x 1O~ =31.58%
J
Illustration: 8.26
Tarana Musics have two factories. producing identical gadgets which is sold at uniform
price of Rs.45 per unit. Particulars about two factories are:
Marginal Costing : 713

, Factory A Factory B
Capacity (units) 10,000 15,000
Costs: Variable Rs. 2,00,000 Rs. 1,50,000
Fixed 2,00,000 3,50,000
If the demand for the gadgets is only 20,000 units, how much should be produced at each
factory? Calculate B.E.P (units) for each factory and company as a whole. Estimate sales to earn
a profit of Rs.70,000 in each factory. [Mangaiore Uni., B.Com., October 20011..
Solution:
Factory A Factory B Company
Sales Revenue at full capacity
[10,000 units and 15,000 units
at Rs.45] Rs. 4,50,000 Rs.6,75,000 Rs. 11,25,000
Less: Variable Cost [per unit Rs.20
and Rs.1O respectively] 2,00,000 1,50,000 3,50,000
:. Contribution 2,50,000 5,25,000 7,75,000
Less: Fixed Cost 2,00,000 3,50,000 5,50,000
:.Profit 50,000 1,75,000 2,25,000
Unit Variable Cost [Total Variable
Cost + Number of Units] Rs.20 Rs.1O Rs.14
Unit Contribution = [Selling Price -
Unit Variable Cost] . Rs.25 Rs.35 Rs.31
PN Ratio [Contribution + Sales

jt
Revenue] 55.56% 77.78% 68.89%

:.BEP (units)
~ Fixed Cost
= Unit Contribution =
Rs2,OO,OOO
Rs.25
J[Rs.3,50,OOO
Rs.35
J [ Rs.5,50,OOO
Rs.31
J
= 8,000 10,000 17,742*
[*in the ratio of 10: 15 of units of Factories A and B respectively =7,097 units of Factory A and
10,645 units of Factory B]

Required Sales Quantity} = [Fixed Cost + Desired PrOfit]


to earn Rs.70,000 Profit Unit Contribution
:. Required Sales Quantity of Factory:
Management Accounting : 714

A
=
lrRs.2,OO,OOORS.25+ Rs.70,OOO J -- lrRS.2,70,OOOJ
RS.25
- .
- 10,800 umts

B=
rl Rsj,50,OOO+ Rs.70,Ooo J
Rs.35 .
r
=l
RS.4,20,OOOJ
RS.25
.
= 12,000 umts ..

Ifth~ demand is only for a total of 20,000 units, it is better to sell the maximum of \5,000
units of Factory B and the remaining 5,000 units of Factory A. Because. in the case of Factory B,
the unit" variable cost is very low and therefore, both the unit contribution and PN ratio are high.
Of course, the fixed cost is also high. Still, it is more profitable. The following comparative
condensed income statement substantiates this point.
c omparatlve Cond ensedI ncome statement
Factory (Rs)
Particulars Company (Rs.)
A B
Alternative -I: 5,000 units of A. and
15,000 units of B:
Contribution [Units x Unit
Contribution 1,25,000 5,25,000 6,50,000
Less: Fixed Cost 2,00,000 3,50,000 5,50,000
:.Profit -75,000 1,75,000 1,00,000
Alternative -D: lO,Ooo.units each of
AandB:
Contribution 2,50,000 3,50,000 6,00,0'00
Less: Fixed Cost 2,00,000 3,50,000 5,50,000
:.Profit 50,000 0 50,000
IDustration: 8.27
You are given the following data for the costing year of a factory:
Budgeted Output: 1,00,000 units
Fixed Expenses: Rs.5,00,000 .
Variable Expenses per unit: Rs.1O
Selling Price per unit: Rs.20
." ~ I'

Draw a Break-even Chart showing Break-even Point. 'Verify your results by calculation also .
. [Kuvempu Uni., B.B.M, April 1998, B.Com., October 1996 and May ~9'$, Mangalore Uni.,
B.Com., May 2002, 2003 and 2004J
Marginal Costing : 715

Solution:

at 0 units at 1,00;000 units


Variable Cost Rs.O Rs.I0,00,OOO
Fixed Cost 5,00,000 5,00,000
Total Cost 5,00,000 15,00,000
Sales Revenue 0 20,00,000
Profit -5,00,000 5,00,000
Unit Contribution 10 10
PN Ratio (%) 50 50

~EP (Rs.) = lr. Fixed Cost J [RS.5,OO,000 J .


PN Ratio J 50% J =Rs.lO,OO,OOO 0; ,

BEP (units) = U·
Fixed Cost
C 'b'
[ nIt ontn utIOn
J = [
Rs.5,00,000
Rs.IO
JJ= 50,000 UnIts.
MIS (units) = [1,00,000 units - 50,000 units] = 50,000 units
MIS (Rs.) = [Rs.20,00,000 - Rs.lO,OO,OOO] = Rs.lO,OO,OOO
Break Even Chart
,
y

100 SR Line
TC Line

Revenue, 75
Cost,
Profit
(R.'\.OOO)
50

25

25 50 100
x
Sales (000 Units)
Management Accounting : 716

Illustration: 8.28
The following figures relate to one year work in a manufacturing organization:

Fixed overheads Rs.12,000


Variable overheads 20,000
Direct wages 15,000
Direct materials 41,000
Sales 1,00,000

Prepare a Break- Even-Chart on the basis of above information and verify your results by actual
calculations.
[Bangalore Uni., B.Com., November 2001 and Kuvempu Uni., B.Com., May 2002 and October 2004J
Solution:
y

200

160 M'S (Rs.)

TC Line

Revenue. 120
Cost.
Profit"
(Rs.OtJO) 10
80

1 " = - - - - - . , , , & - - - - ; . . . - - - - - - - - FC Line


40

-- MIS Profit - ~-- _.. -


L---.---.r--r-.-----.--,----+x
20 40 50 60 80 100
Sales (000 Units)

Variable Cost:
Material Rs.41,000
Wages 15.000
Variable Overheads 20,000
76,000
Sales Revenue 1,00,000
Marginal Costing : 717

Contribution 24,000
Less: Fixed Overheads 12,000
Profit 12,000

PN Ratio = lr- Sales


Contribution
Revenue
~
x 10) = lr. Rs.24,000 J
Rs.I,OO,OOO x 10) = 24%

Variable Cost Ratio = [1 - PV Ratio] = [1 - 24% = 76%


At Rs.O Sales at Rs.l ,00,000 Sales
Variable Cost 0 76,000
Fixed Cost 12,000 12,000
Total Cost 12,000 88,000
Sales Revenue 0 1,00,000
Profit -12,000 12,000

BEP (Rs.) = [~~~=t]= Rs~~~OOO ~ [ =Rs.50,OOO

mDStration: 8.29
Opel company reports the following information for the year 2001:
Sales Rs.I,OO,OOO
Overheads: Variable 60,000
Fixed 25,000
Profit 15,000
Draw a Profit - Volume Graph to show (a) B E P in Rupees, (b) Margin of Safety and
(c) Loss-Profit Zone.
[Mangalore Uni., B.Com., April 2000 and 2001 and November 2003}
Solution:
01. Mark the fixed cost of Rs.25,000 below the X - axis on the left side vertical axis
02. The amount of profit of Rs.15,000 is to be marked against the sales revenue of
Rs.l,OO,OOO above the X - axis and right to the vertical axis already drawn, and
03. Join both the fixed cost point and the profit point with the help of a diagonal line called
profit line or path.
Management Accounting : 718
Profit - Volume Graph
y

30
Profit
[Rs.OOO]
.20 Profit Line

10

o~----+-------+-------~~~L-+------r----- x
20

Loss 10 Loss +-- MIS (Rs.)--+


[Rs.0001

20

30
Y
a. BEP (Rs.) =B =Rs.62,500
b. MIS (Rs.) = [Rs.l,OO,OOO - Rs.62,500) = Rs.37,500
c. Loss Zone =AOB; Profit Zone =CBX
Verification :

,"SEP(as,) = [ ~~~~:t J=[ RS::O J =Rs,62,;OO

b. MIs (Rs.)
;', : ' -
=[Sales Revenue - BEP (Rs.)] =[Rs. 1,00,000 -
-',
Rs.62,500] =Rs.37,500
Dlustration: 8.30
Data extracted from the books of Zenith Mills for the month of October 200 I:
Fixed ~xpenses : Rs.40,0?0 and Break - Even Sales. Rs.l ,00,000. Calculate:
a. PN Ratio,
b. Profit when sales are estimated at Rs.2 lakshs.
c. Revised BEP (Rs.) if selling price is reduced by 20%
Marginal Costing: 719

d. Estimate for sales to earn a profit of Rs.40,000 after reduction in selling price by
20%. ' .
[Kuvempu Uni., B.B.M., November 1998 and Mangaiore Uni., B.Com., October 2001J
Solution:

a. BEP (Rs.) = lrFixed Cost J


PlY Ratio j :. (BEP (Rs.) x PlY Ratio] =Fixed Cost

:. PlY Ratio
r
=l
Fixed Cost
BEP (Rs)
Jj =C Rs.40,000 J
lRs.I,OO,OOOj =0.4~ 40%
,.'

b. Contribution [Rs.2,00,000 sales x 40% PlY Ratio] = Rs.80,OOO


Less: Fixed Cost Rs.40,000
:.Profit Rs.40,OOO
c. 20% reduction in the Selling Price lowers the PlY Ratio from the present 40% to 25%
as shown below:

At the Present At the Reduced


Selling Price Selling Price
Selling Price Rs.100 Rs.80
Less: Variable Cost 60 60
Contribution 40 20
:.PIY Ratio 40% 25%

:. B~~:~e:.)} =[ ~~~~'J =( RS;~OO J=Rs.l,60,OOO


d. Required sales}
RpvpnllP =l
r Fixed Cost + Desired PrOfit]
PlY Ratio

=[ Rs.40,OOO + RS.40,000]
25%
=
lr. Rs.80,000
25%
J
J= Rs.3,20,OOO
mustration: 8.31
With a view to incr.ease volume of sales, Jaya Limited has in mind a proposal to reduce
the price of its product by 20%. No change in fixed costs or variable costs per unit is estimated.
Management Accounting : 720
The directors however desire the present level of profit to be maintained. The following
information have been provided. Advise the management on the basis of various calculations
rD:ade from the data given.
Sales 50,000 units, Rs.5,00,000
Variable Cost Rs.5 per unit
Fixed Costs Rs.50,000.
[Mangalore Uni., B.Com., May 2000 and November 2004J
Solution:
Present Level of Profit:
Sales Revenue [50,000 units x Rs.lO] Rs.5,00,000
Less: Variable Cost [at Rs.5] 2,50,000
:.Contribution [Rs.5] 2,50,000
Less: Fixed Cost 50,000
:.Profit 2,00,000
Proposal:
Proposed Price =[Rs.I0 - 20% of Rs.lO] =[Rs.lO - Rs.2] =Rs.8
Unit Contribution =[Selling Price - Unit Variable Cost] =[Rs. 8 - Rs.5] =Rs. 3
:. Required Sales Quantity} ~Fixed Cost + Desired Profit ]
at reduced Price to earn =
Rs.2,00,OOO Profit Unit Contribution

RS.50,000 + RS.2,00,000j [RS.2,50,000]


=[ = == 83,334 units
Rs.3 Rs.3

From the above, it is obvious that it is necessary to increase the sales volume by 66.67%
or by 33,334 units to earn the present level of profit of Rs.2,00,000 at reduced Price of Rs.8.
IDustration: 8.32
The following figures are extracted from the books of a manufacturing company for the
year 1996:
Direct materials Rs.4,10,OOO
Direct labour 1,50,000
Fixed overheads 1,20,000
Variable overheads 2.00,000
Sales 10,00,000
Marginal Costing: 721

Find out the Break-even Sales from the above data. Also find out the new Break-evn
Point when there is increase of 10% in fixed overheads and variable overheads.
IKuvempu Uni., BBM, May 1997}
Solution:
Present Proposed
Direct material cost Rs.4,1O,000 Rs.4,IO,OOO
Direct labour cost 1,50,000 1,50,000
Variable overheads 2,00,000 2,20,000*
:. Variable Cost
--------------------------
7,60,000 7,80,000
Sales Revenue 10,00,000 10,00,000
--------------------~~--
:.Contribution 2,40,000 2,20,000
Less: Fixed Overheads 1,20,000 1,32,000
:.Profit
--------------------------
1,20,000 88,000

PN Ratio =
I-_C_on_tn_'_bu_t_io_n_ x 1O~
J
RS.2,40,000 r
~ Rs.2,20,000 ~
L Sales Revenue = [ Rs.lO,OO,OOOx lO
J
LRS.lO,OO,OOOX 10)

=24% 22%

Break -Even}_ r.
Fixed Cost
Sales (Rs.) - L PN Ratio
J_r. Rs.1,20,OOO J
j -L 24% j (
RS.l,32,000
22%
J
j

= Rs.5,00,000 Rs.6,OO,000
[Note: * 10% increase in both variable and fixed overheads]
mustration: 8.33
From the following data, calculate the break even point.

Selling price per unit Rs.20


Direct material per unit 8
Direct labour cost per unit 2
Direct expenses per unit 2
Overheads per unit 3
Fixed overheads (total) 20,000

If sales are 20% above the break even point, determine the net profit.
IMangalore Uni., BBM, May 1999)
Management Accounting : 722
Solution:
Note: Overheads [per unit of Rs.3] are assumed to be variable as there is no other information.
Variable Cost per unit: ~elling Price Rs. 20
Material Rs.8 Unit Variable Cost 15
Labour 2 :. Unit Contribution 5
Direct Expenses 2
Variable Overheads 3
-----
15

BEP (units) =
r Fixed Cost J [Rs.20,000
l Unit Contributionj = l Rs.5 j = 4,000 units
If the sales are 20% above BEP, the company can sell 4,800 units [Le., 4,000 units + 20%
of 4,000 units = 4,000 + 800]. Hence. the amount of Net Profit will be Rs.4,OOO as detailed
below:

Total Contribution [4,800 units x Rs.5] = Rs.24,000


Less: Fixed Cost 20.000
.. Profit 4,000
Illustration: 8.34
The following information is obtained from A Ltd., for the year 1998.

Sales Rs.60,OOO
Variable cost 30,000
Fixed cost 15,000
You are required to:
a. -Calculate the PN Ratio,. Break-even Point and Margin of Safety at this level.
b. Calculate the effect of 10% increase in Sale Price.
c. Calculate the effect of 10% decrease in Sale Price.
[Bangalore Uni., B.Com., May 2000J
Solution:
(a) (b) (c)
Sales Revenue Rs.60,000 66,000 54,000
Less: Variable Cost 30,000 30,000 30,000
Contribution 30,000 36,000 24,000 •
Ma inal Costing : 723

Less: Fixed Cost 15,000 15,000 15,000


Profit 15,000 21,000 9,000

PN Ratio =
Contribution
[ Sales Revenue
x lOj and BEP (Rs.) =[ Fixed Cost
PN Ratio ]
MIS (Rs.) = [Sales Revenue - BEP (Rs.)]

PN Ratio BEP (Rs.) MIS (Rs.)


a.

[ Rs.30,000
Rs.60,000 x lOOJ
[ Rs.1S.OOO ]
50%
[Rs.60,OOO - Rs.30,OOO]

=50% = Rs.30,OOO = Rs.30,000


b.

[ Rs.36,OOO
Rs.66,000 XlOOJ
[ Rs.1S.OOO ]
36166
[Rs.66,000 - Rs. 27,500]

=54.55% = Rs.27,500 = Rs.38,500


c.
I-
lO~
Rs.24,000 RS.15,000J
l Rs.54,OOO x [ 24/54
[Rs.54,OOO - Rs.33,750]

=44.44% = Rs.33,750 = Rs.20,250


Illustration: 8.35
A retail dealer in garment is currently selling 24,000 shirts annually. He supplies the
following details for the year ended December 31, 2091.

Selling price per shirt Rs.40


Variable cost per shirt 25
Fixed Cost: Salaries for the year 1,20,000
General office costs for the year 80,000
Advertising cost for the year 40,000

From the above details:


a. Calculate the break-even point and margin of safety in sales revenue and number of shirts
sold.
Management Accoun g: 724

b. Assume that 20,000 shirts were sold in a year and 1 ld out the net profit of the firm.
c. If it is decided to introduce selling commission of Rs.3 per shirt, how many shirts would
require to be sold in a year to earn a net income of Rs.15,OOO
d. Assuming that for the year 2002, an additional staff cost of Rs.33,OOO is anticipated and price
of a shirt is likely to be increased by 15%, what should be the break-eyen point in nu~ber of
shirts and sales volume? [Bangalore Uni., B.Com., April 2004]
Solution:
Selling Price Rs. 40 Salary Rs. 1,20,000
Variable Cost 25 Office Costs 80,000
:. Unit Contribution 15 Advertisement Costs 40,000
:. Total Fixed Cost 2,40,000
PN Ratio =[Rs. 15 + Rs. 40] = Rs. 37.5 %
a. BEP (units) =( Fixed Cost
Unit Contribution
J=( Rs. 2,40,000
Rs. 15
lj - 16,000 shirts

MIS (units) = [Current Sales - BEP [units])


=(24,000 shirts - 16,000 shirts) = 8,000 shirts

BEP (Rs.) = [:~"!~:t J= [ RS3~~000J= Rs. 6,40,000

MIS (Rs.) = [Current Sales Revenue - BEP (Rs.)]


= [(24,000 Shirts x Rs. 40) - Rs. 6,40,000)]
= [Rs. 9,60,000 - Rs. 6,40,000] = Rs. 3,20,000
b. Contribution from 20,000 shirts at Rs. 15 = Rs. 3,00,000
Less: Fixed Cost 2,40,000
:. Net Profit 60,000

c. New Va;.iable Cost per shirt = (Rs. 25 + Rs. 3 Commission) = Rs. 28;
Unit Selling Price = Rs. 40; and Desired Profit = Rs. 15,000.

:. Nb umb1edr of ShirtsRtO} [FiXed Cost + Desired PrOfitj


e so to earn s. =
15,000 Net Profit Unit Contribution
Marginal Costing : 725

= e Rs. 2,40,000 + Rs. 15,000 J= I-Rs. 2,55,00~ =21,250 shirts


[ Rs. 40 - Rs. 28 j l Rs. 12 J
d. New Fixed Cost = [Rs. 2,40,000 + Rs.33,000] = Rs. 2,73,000
New Price = [Rs. 40 ,
+ 15% of Rs. 40] = [Rs. 40 + Rs. 6] = Rs. 46.
Unit Contribution = (Rs.46 Selling Price - Rs.25 Unit Variable Cost) = Rs.21

PN Ratio = eRs.21 Unit Contribution x IOJ = 45.652%


l Rs. 46 Selling Price J
BEP ~umber} =r Cost ~ = [RS. 2,73,000~ = 13 000 shirts
of Shirts) l UnitFixed
ContributionJ Rs. 21 J '
BEP (Rs.) = [Fixed C~st ) = eRS. 2,73,000 "I = Rs.5,98,000
l PN Ratio J l ~~= 45.6 52O/~
mustration: 8.36
Siddhartha Ltd., has prepared the following budget estimates:
Sales Units = 15,000; Fixed Expenses =Rs.34,000; Sales Value = Rs.1,50,000; and Variable
Costs = Rs.6 per unit. You are required to:
1. Find the PN Ratio, Break Even Point and Margin of Safety.
2. Calculate the revised PN Ratio, Break-even Point and Margin of Safety in each of the
following cases:
a. Decrease of 10% in Selling Price
b. Increase of 10% in Variable Costs
c. Increase the Sales Volume by 2,000 units
d. Increase of Rs. 6,000 in Fixed Costs
[Kuvempu Uni, B.Com, October 2003J
Solution:
Basic Data: Estimates Case (a) Case (b) Case (c) Case (d)
Sales Quantity (units) 15,000 15,000 15,000 17,000 15,000
Selling Price (Rs.) 10 9 10 10 1(}
Management Accounting : 726

Unit Variable Cost (Rs.) 6 6 6.6 6 6


Unit Contribution (Rs.) 4 3 3.4 4 4
Fixed Cost (Rs.) 34.000 34.000 34,000 34,000 40.000

PN Ratio BEP (Rs.) MIS (Rs.)

1 [ Unit Contribution
Selling Price
Xl, ~ Fixed Cost
PN Ratio
J ( Sales
Revenue -
Break - even
Sales Revenue
J
= (RS.4 x 100J
Rs.lO
tRs.34 z000
40%
j [Rs.l,50,000 - Rs.85,000]

= 40% Rs.85,000 = Rs.65,000

2. a. t RS 3
. x
Rs.9 1O~ tRs.34!000
33.33%
j [Rs.l,35,000 - Rs.l,02,000]

I,
=33.33% Rs.I,02,000 = Rs.33,000

b. t RS 3 4
. .
Rs.lO
X [RS.34,OOO
34%
J [Rs. 1,50,000 - Rs.l ,00,000]

=34% Rs.l,OO,OOO = Rs.50,000

c. tRS~x
Rs.lO
100J tRs.34,OOO
40%
j [Rs.l,70,000 - Rs.85,000]

=40% Rs.85,000 =Rs.85,000

d. r
l
Rs.4 x
Rs.lO
10~
)
lRS.40 Z000]
L 40%
[Rs.l ,50,000 - Rs.l ,00,000]

=40% Rs.l,OO,OOO =Rs.50,000


D1ustration: 8.37
There are two plants manufacturing the same product under one corporate management
which decides to merge them. The following particulars are available regarding two plants:
Marginal Costing : 727

Plant A Plant B
Capacity Utilisation 100% 60%
Sales Rs. 6,00,000 Rs. 2,40.000
Variable Costs Rs. 4,40,000 Rs. 1,80,000
Fixed Costs Rs. 80.000 Rs. 40,000

You are required to calculate for the Board of Directors:


a. What would be the capacity of merged plant to be operated for the purpose of break
even?
b. What would be the profitability on working at 75% of the merged capacity?
[Manga/ore Uni, B.Com, May 2002J
Solution:
I ncome Statement [at 100~lJ . U tITlzatIon
Capaclty . ]
Plant (Rs.) Merged Plant
Particulars
A B (Rs.)
Sales Revenue 6,00,000 4,00,000 1 10,00,000
2
Less: Variable Cost 4,40,000 3,00,000 7,40,000
Contribution 1,60,000 1,00,000 2,60,000
Less: Fixed Cost 80,000 40,000 1,20,000
Profit 80.000 60,000 1,40,000
PN Ratio (%) 26.67 25 26

Note: 1. At 100%, Sales Revenue = [Rs.2,40,000 + 60%] = Rs.4,00,000

2. [ Rs.l ,80,000 R 4 00 OOO~ = R 3 00 000


[Rs.2,40,OOO x s. , , J
S."

a. BEP (Rs) Of} = [Fixed Cost] = J


[RS. 1,20,000 == Rs.4,61,539
Merged Plant l PN Ratio J 26%)

:. Break-Even capacity}
(%) of Merged Plant
=
lrSales
HEP (Rs.)
at 100%
x 1001
:1 = [RS.4,61~539
Rs.IO,OO,OOO
x IOJ == 46.16%
)

b. Profit of Merged }
Plant.at 75% = [(Sales Revenue at 75% x PN Ratio) - Fixed Cost]
Management Accounting : 728

= [(Rs.I 0,00,000 X 75% X 26%) - Rs.I,20,OOO]


= [(Rs.7,50,OOO X 26%) - Rs.1,20,000]
= [Rs.I,95,OOO - Rs.I,20,OOO] = Rs.75,OOO

.'. Profitability O:Ofit} = (Profit 1O~ ( Rs.75,OOO ~


to Sales Ratio) L
Sales
X
) = LRs.7,50,OOO X 100J =10%
mustration: 8.38
An analysis of costs of a company led to the following infonnation.

Variable Cost Fixed Cost


(% of Sales) (Rs.)
Direct materials 33.6
Direct labour 28.4
Factory overhead 11.6 1,66,700
Distribution overhead 3.3 63,400
General administration overhead 1.1 99,900
Budgeted sales for the next year Rs.20,OO,OOO
You are required to determine:
a. Break- Even Sales,
b. Profit at the Budgeted Sales Volume, and •
c. Sales to generate a Profit of Rs.2,20,OOO
[Bangalore Uni, B.Com, April 2003]
Solution:
Variable Cost Fixed Cost
(% to sales) (Rs.)
Direct material 33.6
Direct labour 28.4
Factory overhead 11.6 1,66,700
Distribution overhead 3.3 63,400
General overhead 1.1 99,900
----------------------
Total 78.0 3,30,000

:.PN Ratio = [I - Variable Cost Ratio] = [I - 78%] = 22%

LPN Ratto
r
a. BEP (Rs.) = [Fixed C~s~ = Rs.3,30,OOOI = Rs. 15,00,000
J l 22% J
Marginal Costing : 729

b. Budgeted Sales Rs. 20,00,000


Less: Variable Cost (78%) 15,60,000
Contribution 4,40,000
Less: Fixed Cost 3,30,000
:. Profit 1,10,000

c. Required Sales} = rFixed Cost + Desired Profi~rRs.3,30,000 + Rs.2,20,oooj


(Rs.) l PN Ratio Tl 22% J
=
r
l
Rs.5,50,000")
22% J =Rs.25,00,000
mustration: 8.39
A factory furnishes the following figures:
August 1984 September 1984
Output (units) 50,000 55,000
Totals Cost (Rs.) 6,70,000 7,10,000
What is the amount of fixed expenses per month? [CS}

Solution:

Variable Costl (ChangeS in Total cost]


(per unit) f = Changes in Output =
(RS. 7,10,000 - Rs. 6,70,000)
(55,000 - 50,000) units
J
=
Rs. 40.000
( 5,000 units
J =
Rs 8
.
August 1984 September 1984
Total Cost Rs.6,70,000 • Rs. 7,10,000
Less: Variable Cost [50,000 units x Rs. 8] 4,00,000
[55,000 units x Rs.8] 4,40,000
:. Monthly Fixed Cost 2,70,000 2,70,000
,
Management Accounting : 730
, mustration: 8.40
Firm M manufactures surgical goods, Its normal production is 2,600 units per month at a
total cost of Rs. 32,000. At full capacity, it can manufacture 3,400 units per month at a total cost
of Rs. 38,000. Calculate:
(a) Average cost per instrument under normal operating conditions,
(b) Average variable cost per instrument.
(c) Total fixed cost, and
(d) Average fixed cost under normal operating conditions. res]
Solution:
(a) Average Cost = [Total C~st J = (.RS.32,0?OJ = Rs. 12.31
1
per InstrumentJ ProductIOn 2,600 UOlts

(b) Average } _ RRs. 38,000 - Rs. 32,000~ _ [RS. 6,000J _ R 750


Variable Cost - L
(3,400 - 2,600) units J-
800 units - s. .
(c)

at 2,600 units a~ 3,400 units


Total Cost Rs. 32,000 Rs.38,000
Less: Variable Cost (at Rs. 7.5 per unit) 19,500 25,500
:. Fixed Cost 12,500 12,500
:.Monthly Fixed Cost =Rs. 12,500

(~) Average L (RS.12,500J


'f;jxed CostS = 2,600 units =Rs. 4.81
Illustration: 8.41
. Th,e following are the maintenance costs incurred in a machine shop for six months with
the corresponding machine-hours. Analyse the maintenance cost which is semi-variable into
fixed and variable elements.
Month Machine-hours Maintenance Costs
January 2,000 Rs.300
,February 2,200 320
March 1,700 270
April 2,400 340
May 1,800 280
June 1,9QO 290
Total 12,000 1,800
Marginal Costing : 731

lCA (In.t), May 1985J


Solution:
For segregation, Least Squares Method is used.

Machine Maintenance
x2 xY
Hours (x) Cost (Rs.) (Y)
2,000 300 40,00,000 6,00,000
2,200 320 48,40,000 7,04,000
1,700 270 28,90,000 4,59,000
2,400 . 340 57,60,000 8,16,000
1,800 280 32,40,000 5,04,000
1,900 290 36,10,000 5,51,000
'~= 12,000 l:Y = 1,800 l:x 2 = 2,43,40,000 ~Y = 36,34,000
We know that, l:Y = ml:x + nc and l:x Y = ml:x2 + cl:x. Putting the computed values into
the equations,
1,800 = 12,000 m + 6 c ... (1)
36,34,000 = 243,40,000 m + 12,000 c ... (2)
For simplification, divide equation (2) by 1,000. So, the result is,
3,634 = 24,340 m + 12 c ... (3)
Divide equation (3) by 2 and from the resultant equation subtract equation (1),
1,817 = 12,170 m + 6 c
1,800 = 12,000 m + 6 c
17 = 170m

1
:. m = (1 ;0 J= Re. 0.1 (Le., Vari~ble part of Maintenance Cost)
Substitute the value of 'm' in equation (1),
1,800 = (12,000 x 0.1) +'6 c
1,800 = 1,200 + 6 c
, 6 c = (1,800 - 1,200) = 600

:. c = (6~J =Rs. 100 (Le., Fixed portion of Maintenance Cost)


Maintenance Cost equation, Y =O.lx + 100, where
Management Accounting: 732
Y =Total Maintenance Cost per month, and
x =Machine Hours per month.
An allySIS
. 0 fM·
aIDt enance C OSt
Variable Fixed Total Maintenance
Machine Hours·
Month Maintenance Maintenance Cost (Var + Fix)
(x)
Cost (O.lx) Cost (Rs. 100) = (O.lx + 100)
January 2,000 200 100 300
February 2,200 220 100 320
March 1,700 170 100 270
April 2,400 240 100 340
May 1,800 180 100 280
June 1,900 190 100 290
12,000 1,200 600 1,800

mDStration: 8.42
Calculate Plv ratio and Break-Even Point from the following particulars.
Sales Rs. 5,00,000
Fixed expenses Rs. 1,00,000
Profit Rs. 1,50,000
[B.B.M., UoM, May 1986J
Solution:

(a) Plv Ratio = (COntribUtiOn x 100J = Fixed Cost + Profit x 100 J


Sales ( Sales Revenue

= (RS. 1,00,000 + Rs. 1,50,000


Rs. 5,00,000
x 100 J= rRs.2,50,000 x 100J
LRs. 5,00,000
=50%

(b) Br~-Even} = ( Fixed C?stJ = (RS. 1,00,000 J =R 200000


Pomt (Rs.) Plv Ratio 50% s. , ,
mustration: 8.43
Break down of cost per unit at an activity level of 10,000 units of Zenith Razors is as
follows. How many units must be sold to break even?

Raw materials Rs.1O


Direct expenses 8
Chargeable expenses 2
Marginal Costing : 733

Variable overheads 4
Fixed overheads 6
Total cost per unit 30
Selling price 32
Profit per unit 2
[ICWA (1m), December 1977J
Solution:
Unit Variable Cost: Fixed Costs = [10,000 units x Rs.6 per unit] = Rs.60,000
Materials Rs.1O Break-Even 1 ( Fixed Cost J
Point (unitsif" = LUnit Contribution
Direct expenses 8
Chargeable expenses 2 Rs. 60,000J - 7 500 .ts
Variable overheads 4
=( Rs.8 -, um

24
Unit Selling Price 32
Contribution 8
mustration: 8.44
Calculate the Break Even Point in units and in rupees and also arrive at the Margin of
Safety Ratio from the following information.
Estimated sales (1,00,000 units) Rs.20,00,000
Variable cost Rs. 12,00,000
Fixed cost 4,00,000 16,00,000
Net Profit 4,00,000
Solution:
Break-Even 1 (. Fixed Costs .1 U Rs. 400000
Quantity (unitsf= Ipnit Contributio~ = [RS.20,00,000 _ (RS. 12,00,000
1,00,000 units 1,00,000 units

_r. Rs. 4,00,000 ~ = (Rs. 4,00,0001_ 50 000 'ts


- ~Rs. 20-Rs. 12~ Rs.8 l um J- ,

Break-Even} r
Fixed Cost
Revenue (Rs.) = LUnit Contribution x
Unit sellingJ
Price
Management Accounting : 734

= (Rs. :~~000 X Rs. 20J = (~s. ~~~,OOOJ = Rs. 10,00,000

Margin of } _ (Present Sales - Break-Even Sales x 1O~


Safety Ratio - l Present Sales )

= (.Rs.20,00,000 - Rs.lO,OO;OOO x 100J = (Rs. 10.00.000 x 1001 = 50%


l Rs.20,00,OOO l Rs. 20,00,000 J
Illustration: 8.45
From the following information relating to Quick Standards Ltd., you are required to find
out (a) Contribution, (b) Break even point in units, (c) Margin of safety, and (d) Profit. Also
calculate the value of sales to earn a profit of Rs. 6,000.
Total fixed cost Rs.4,500
Total variable cost 7,500
Units sold 5,OUO
Sales revenue 15,000 [es]
Solution:
Sales reveJ;lue (5,000 units at Rs. 3) Rs.3.00 Rs.15,000
Less: Variable Cost(a Rs. 1.5) 1.50 7,500
Contribution 1.50 7,500
Less: Fixed Costs 4,500
Profit 3,000
(a) Contribution = Rs. 7,500

(b) BEP} _ ( Fixed Cost J - (RS. 4.500 J - 3' 000 ·ts


. (in units) - Unit Contribution - Rs. 1.5 -, um

(c) Margin of Safety = [Sales - Break-Even Sales]


1. Mis (units) = [5,000 units - 3,000 units] = 2,000 units
2. Mis ·(Rs.) =[Rs. 15,000 - (3,000 units x Rs. 3)] =(Rs. 15,000 - Rs. 9,000)
= Rs. 6,000

3. Mis (%) = (MIs (units) x 1001 =( ;'~g un~ts x 100 J= 40%


Lsales (units) :J ' umts
(d) Profit = Rs. 3,000
J
Marginal Costing : 735

Fixed Desired
(e) ReqUired} _ [ Cost + Profit _ (RS. 4,500 + Rs. 6,ooOJ
Sales Volume - Unit Contribution - Rs. 1.5

(Rs. 10,5001 •
= L Rs. 1.5 J = 7,000 units
llIustration: S.46
You are given the following information in respect of a company.
Fixed cost Rs.13,OOO
Variable cost 14,000
Total cost 27,000
Net profit 3,000
Net sales 30,000
a. Find out the break-even point.
b. Forecast the profit for sales volume of Rs. 50,000.
c. Estimate the volume of sales turnover to make a net profit of Rs. 10,000. [C.S)
Solution:
. = (COntribution 100 J - (RS. 30,000 - Rs. 14,000 100 J
P1v Rabo Sales x - Rs. 30,000 x

(a) Break Even} (Fixed Cost J [RS. 13,0001 .


Point (Rs.)· = P1v Ratio = 53 1/3 ) =Rs. 24,375
(b) Profit = (Contribution - Fixed Cost) = [(Rs. 50,000 x 53 113%) - Rs. 13,000]
= (Rs. 26,667 - Rs. 13,000) = Rs. 13,667
(c) Let 'R' be the required sales revenue to earn a net profit of Rs. 10,000. Therefore,

R- (FiXed Cost + Desired PrOfitJ_ (Rs.13,000 + RS.IO,OOOJ ~(RS.23,000J - R 43125


-. P1v Ratio - L 53 113% -( 53 113% - s. ,

mustration: SA7
From the following particulars of ABC Company Limited., find out through mathematical
formula: (a) break even point of sales in terms of rupees, (b) break even point in terms of units,
and (c) how 'many units have to be sold in order to get Rs. 700 as profit.
Management Accounting : 736

Fixed cost: Rs.500 Selling price: Rs. 2 per unit


Variable cost: Re. 1 per unit Normal output: 750 units [C.S]
Solution:
(a) Break-Evenl r.
Fixed Cost Unit sellingJ = r. Rs.500 x Rs. 2J
Point (Rs.) f
= LUnit Contribution x Price liRs.2 - Re.l)

= ( RS.5oo J
Re.l x Rs. 2 =Rs. 1,000

(b) Break-Even
Point (units)
= t Fixed Cost
Unit Contribution
j =( Rs. 500
(Rs. 2 - Re. 1)
J= r- - RS . 500J 500 .
Re. 1 -
- umts

(c) Let 'x' be the required sales volume. Therefore,

x =(FiXed C?st + D~sir~ Profit J =(Rs.500 + Rs.7001 = ( Rs.l ,200 J =Rs.l 200 units
Umt Contnbution (Rs. 2 - Re. 1) J t Re. 1 '
mustration: 8.48
Three firms X, Y and Z manufacture the same product. The selling price is Rs. 8 per unit of
the product (equal for all the firms). The fixed costs for the firms X, Y and Z respectively are Rs.
80,000, Rs. 2,00,000 and Rs. 3,30,000, while the variable costs per unit are Rs. 6, Rs. 4 and Rs. 3.
a. Determine the break even point for all the firms in units.
b. How much profits are earned by the firms if each of them sells 80,000 units.
c. What will be the impact, percentage wise, on profits if sales increase by 20%.
Solution:
Comparative Statement
Particulars FirrnX FinnY FirmZ
Unit Contribution =Unit Selling Price - Unit Variable
Cost (Rs.) 2 4 5
(a) Break-Even} _
Point (units) -
~ Fixed Cost
Unit Contributio
J
= (RS. 80zoo0}
Rs.2
( Rs. 2.00.000
Rs.4
J, (RS. 3,30,000
Rs.5
J 40,000 50,000 66,000

(b) Contribution (Rs.) =(Units Sold x Unit Contribution)


=[80,000 units x Rs. 2 1,60,000
80,000 units x Rs. 4 3,20,000
4,00,000
Marginal Costing : 737

80,000 units x Rs. 5]


Less: Fixed Cost (Rs. 80,000 2,00,000 3,30,000
:. Profit 80,000 1,20,000 70,000

Percentage of Profit to Sales = (Sal Pr;fit


es evenue
x 100
J
=( 80,000
Rs.80,000 ~ ~RS.l'20'000
x Rs.8 x 100; 80,000 x Rs.8 x 100;
~ ~80,000
Rs.70,000
x Rs.8 x 100
~ 12.50 18.75 10.9375

(c) Sales Quantity (units) = (80,000 units + 20%) 96,000 96,000 96,000
Sales Revenue (at Rs. 8 Selling Price) (Rs.) 7,68,000 7,68,000 7,68,000
Less: Variable Cost (at Rs. 6, Rs. 4, and Rs. 3) 5,76,000 3,84,000 2,88,000
Contribution 1,92,000 3,84,000 4,80,000
Less: Fixed Costs 80,000 2,00,000 3,30,000
Profit 1,12,000 1,84,000 1,50,000
:. Percentl!g~ of Profit to Sales 14.5833" 23.9583 19.53125

Comparing the profit ratio in (b) with as in (c), it can be concluded that the profit ratio
increased at different rates.
IDustration: 8.49
A company is manufacturing three products, details of which for the last year are given
below.
Variable cost % of total
Product Price Rs.
per unit Rs. sales value
A 20 10 40
B 25 15 35
C 20 12 25
Total fixed cost: Rs. 1,10,000 and Total sales: Rs. 5,00,000. You are required to work out the
break even point in rupees sales for each product assuming that the sales mix is to be retained.
The management has approved a proposal to substitute product C by product D in the
coming year. The latter product has a selling price of Rs. 25 with a variable cost of Rs. 12.50 per
unit. The new sales mix of A, B and D is expected to be 50 : 30 : 20. Next year, fixed costs are
expected to increase by Rs. 31,000. Total sales are expected to remain at Rs. 5,00,000. You are
required to work out the new break even point in rupees and sales units for each product.
[IeWA (Int)]
Solution:
A B C
Sales Revenue (40%, 35% and 25% of Rs.5,00,000) Rs.2,00,000 1,75,000 1,25,000
Management Accounting : 738
Selling Price (Rs.) 20 25 20
rSales Revenuel
l
:. Sales Volume (units) = Selling Price) = 10,000 7,000 6,250

ComputatIon 0 fB reak E ven Sales (Last Year)


Product (Rs.) Total
Particulars
A B C (Rs.)

Sales Revenue 2,00,000 1,75,000 1,25,000 5,00,000


Less: Variable Cost (at Rs.lO, Rs.15 and Rs.12) 1,00,000 1,05,000 75,000 2,80,000
Contribution 1,00,000 70,000 50,000 2,20,000
Less: Fixed Cost 1,10,000
Profit 1,10,000
Plv Ratio 50% 40% 40% 44%
BES (Rs.) =~ Fix~d Cost . 10,000 J
J= [RS. 1,44%
ComposIte Plv RatIo

=Rs. 2,50,000 (2: 1.75: 1.25) 1,00,000 87,500 62,500 2,50,000

Coming Year, Plv Ratio of Product D =[R~~.li55 x 100 J= 50%


J
~
company's Sales of each P/V J" .
Contribution = Total x Product as a x ofRatio
the
Sales Revenue " %age of total ( Product
Sales Revenue
:. Contribution from Product:
A =(Rs. 5,00,000 x 50% x 50%) = Rs. 1,25,000
B =(Rs. 5,00,000 x 30% x 40%) = 60,000
D = (Rs. 5,00,000 x 20% x 50%) = 50,QOO
2,35,000
Less: Fixed Cost (Rs. 1,10,000 + Rs. 31,000) • 1,41,000
:. Profit 94,000
Marginal Costing : 739

Br~-Even}
Pomt (Rs.)
= ( Fix?d Cost.
ComposIte Plv RatIo
J= LrRs. 47%
1,41,00~1 = Rs. 300000 (in 5: 3 : 2)
J "
Rs. SP (Rs.) BE units

~nn
BEP (Rs.) 3,00,~ :
_L A
1,50,000
90,000
20
25
7,500
3,600
60,000 25 2,400
3,00,000 13,500

The change is, therefore, not a profitable one.


mustration: 8.50
(a) Calcutta Company Ltd., manufactures and sells four types of product under br:and names
Ace, Utility, Luxury and Supreme. The sales mix in value comprises the following:
Brand Percentage
Ace 33113
Utility 412/3
Luxury 162/3
Supreme 8113
100
The total budgeted sales (100%) are Rs. 6,00,000 per month. The variable operating costs
are:
Ace 60% of selling price
Utility 68% of selling price
Luxury 80% of selling price
Supreme 40% of selling price
The' fixed costs are Rs. 1,59,000 per month. Calculate the break-even point for the
products on an overall basis.
(b) It has been proposed to change the sales mix as follows, the total sales per month
remaining Rs. 6,00,000.
Brand Percentage
Ace 25
Utility 40
Luxury 30
Supreme 5
100
Management Accounting : 740

Assuming that this proposal is implemented, calculate the new break-even point.
lCA (Fin), November 1986J
Solution:
(a) Sales Plv Ratio Product of Sales (b) Sales Product of Sales
Brand
Ratio (l-VC Ratio) Ratio and Plv Ratio Ratio Ratio and Plv Ratio
Ace 331/3% 40% 13 1/3% 25% 10.0%
Utility 412/3% 32% 131/3% 40% 12.8%
Luxury 162/3% 20% 3113% 30% 6.0%
Supreme 81/3% 60% 5% 5% 3.0%
Composite P/v Ratio =35% 31.8%
<

(a) Monthly }
Break-Even
Sales Revenue
=(. Fix~ Cost . ~
LComposlte Plv Rattoj 35%
J
= ( Rs. 1,59,000 =Rs. 4 54 286
' ,

(comprising of revenue from Ace, Utility, Luxury and Supreme in the ratio of 33 113%,
41 2/3%, 162/3% and 8 1/3% respectively).

(b) Monthly BESR (Rs.) = r~}i~:%oooJ =Rs. 5,00,000 (comprising of revenue from Ace,
Utility, Luxury and Supreme in the ratio of 25%, 40%, 30% and 5% respectively).

mustration: 8.51
The selling price of a product was Rs. 200 per unit as against its variable cost of Rs. 100
per unit. The total fixed costs were Rs. 2,00,000. Calculate the effect of a reduction in price by
Rs. 40 per unit on the Plv ratio, Break even point and Margin of safety, if 4,000 units were
produced and sold. llCWA (Int), June 1982J
Solution:
Existing Proposed
Unit Selling Price (Rs.) 200 160
Unit Variable Cost (Rs.) 100 100
Unit Contribution (Rs.) 100 60

a. Break-Even l ( Fixed Cost


Point (units)f = LUnit Contribution
J= l rRs. 2,00,000")
Rs. 100 j (
RS. 2,00,00Q")
Rs.60 J
= 2,000 3,334
Marginal Costing : 741

b. Plv Ratio = [Unit Contribution x 1001 = [Rs. 100 x 1001 [Rs. 60 x 1001
Selling Price :J
LRs. 200 :J LRs.160 )
=50% 37.5%
c. Margin of } = ( Sales at _ Break-Even J
Safety, MIs given'level Sales
MIs (units) = (4,000 - 2,000) units (4,000 - 3,334) units
= 2,000 units = 666 units

MIs Ratio = [ MIs (uni.ts) [2,000 units ~ 666 units x 100 J


Sales at gIven
Level
L4,000 units x 100
J [ 4,000 units

= 50% 16.65%
mustration: 8.52'
A company budgets for a production of 1,50,000 units. The variable cost per unit is Rs.
14 and fixed cost is Rs. 2 per unit. The company fixes its selling price to fetch a profit of 15% on
cost.
a. What is the break even point?
b. What is the profit volume ratio?
c. If it reduces its selling price by 5%, how does the revised selling price affect the break
even point and the profit volume ratio?
d. If a profit increase of 10% is desired (more than the budget), what should be the sale at
the reduced price? rCA (Int), May 1984J
Solution:
1. Unit Variable Cost Rs. 14 2. Total Fixed Cost = (l,50,000 units x Rs. 2)
Unit Fixed Cost 2 = Rs. 3,00,000
Total Unit Cost 16
Add: Profit (15<:,; ) 2.4
Selling Price 18.4

a. Break-Even l r.Fixed Cost.1


Point (unitsif' = l!Jnit Contributio~ =
[RS. 3,00,0001 .
Rs.4.4 j = 68,182 umts

b. Plv Ratio = [unit Contribution x 100J =


Unit Selling Price
(~. ~~ x 1001 = 23.91 %
)
Management Accounting: 742

c. Budgeted Price (current) Rs. 18.40


Less: 5% Reduction 0.92
Revised Price 17.48
Less: Variable Cost 14.00
:. Unit Contribution 3.48

Br~ak-Ev.en1 = (Rs. 3,00,0001 = 86,207 units


Pomt (umts) J l Rs. 3.48 J
I
Plv Ratio = (Rs. 3.48 x·lOO = 19.91%
LRs.17-.48 'J
d. Desired Profit:
Budgeted Profit = (1,50,000 units x Rs. 2.4) Rs. 3,60,000
Add: 10% increase desired 36,000
3,96,000
(Sales Volume x Selling Price) = (Variable Cost + Fixed Cost + Profit)
(Sales Volume x Rs. 17.48) = [(Sales Volume x Rs.14) + Rs.3,OO,OOO + Rs.3,96,000]
Let 'x' be the sales y.olume, Therefore, (x x Rs. 17.48) = (x x Rs. ~4Y + Rs. 6,96,000
17.48x = [14x + 6,96,000]
[17.48x ~ 14x] = Rs. 6,96,000
3.48x = Rs. 6,96,000

- (Rs. 6,96,000
x- l
Rs. 3.48 ) - "
I-
2 00 000 ·ts
um
,', Required Sales Volume = 2,00,600 units (or 2,00,000 units x Rs. 17.48.= Rs. 34,96,000)
mustration: 8.53
The profitability statement of 0 Co., Ltd., has been summarized as given below.
Sales Rs. 15,00,000
Direct material Rs. 4,50,000
Direct wages 3,00,000
Variable ovellheads 1,20,000
Fixed overheads 4,40,000 13,10,000
Marginal Costing : 743

Profit 1,90,000

The budgeted capacity of the company is Rs. 20,00,000 but the key factor is sales
demand. It is proposed that in order to utilise the existing capacity, the selling price of the only
product manufactured by the company should be reduced by 5%. You are requested to prepare a
forecast statement which should show the effect of the proposed reduction in selling price and
include any changes in costs expected during the coming year. The following additional
information is given.
(a) Sales forecast Rs. 19,00,000 (after reduction)
(b) Direct material prices are expected to increase by 2%
(c) Direct wage rates are expected to increase by 5% per unit
(d) Variable overheads are expected'to increase by 5% per unit
(e) Fixed overheads wiII increase by Rs. 20.000. [ICWA (Fin), December 1982J
Solution:
Income Statement (Forecast)
. Amount
Particulars
Rs. Rs.
Sales Revenue [Rs.20,00,000 - 5% of Rs. 20,00,000] 19,00,000
Less: Variable cost:
'Direct Material (~~. ~~OOo:O + 2%Jx [Rs.20.00,000] 6,12.000

Direct Wages (~~'135~go~ggo + 5%Jx [Rs.20,00.0~0] 4,20,000

Variable Overheads (~~'115~gO~ggo + 5% Jx [Rs.20,00,000] 1,68,000 12,00,000


Contribution 7,00,000
Less: Fixed Cost (Rs. 4,40,000 + Rs. 20,000) 4,60,000
Expected Profit 2,40,000

. Summary of the Chapter


The chapter starts with the introduction in the form of need for Marginal Costing
followed by the analysis of meaning and definition of Marginal Costing. Thereafter, meaning and
definition, and ascertainment of Marginal Cost are discussed in detail. This is followed by the
explanation of concepts of Contribution and Plv Ratio. A comprehensive discussion is made
about the Break-Even Analysis discussing both the A~gebraic and Gra~hical approaches and
Management Accounting : 744
discussing from the viewpoint of both the mono-product and multi-product concerns. While
discussing the Break-Even Analysis, other fundamental aspects of Marginal Costing andlor
Break-Even Analysis such as Angle of Incidence, Margin of Safety, etc., are also discussed.
Thereafter, the impact of changes in unit selling price, unit variable cost, total fixed cost and the
sales quantity" on the Break-Even Point, Margin of Safety, Profit, etc., are discussed. At the end, a
number of problems are discussed.
Key-Terms to Remember
Marginal Cost Marginal Costing
Variable Cost Fixed Cost
Semi-variable Cost Break-Even Analysis
Break-Even Point Contribution
Plv Ratio Margin of Safety
Angle of Incidence C-V-P Analysis
Notes and References
1. Source not known.
2. Ray H. Garrison, ''Managerial Accounting - Concepts for Planning, Control,
Decision Making," Texas, Business Publication Inc., 1979, P. 172.
3. Charles T. Horngren, ''Introduction to Managerial Accounting", London, Prentice-
Hall International, Inc., 1981, P. 32.
4. Though the difference is also due to a number of decisions taken by the companies, they
are not considered here for detailed analysis. Further, even the managerial decisions are
going to influence either the cost items or the demand or the selling price or a
combination of these. Hence, the changes in costs, selling price and output are reckoned
here.
5. Difference in the profit figures using different formulae is due to the difference between
the actual Break-Even Point and the Break-Even Point taken. For example, actual BEQ
comes to 571.42857 units (i.e., Rs.4,OOOlRs.7). Since fraction of a unit is not normally
saleable and since "any answer less than 571.42857 ensures less than Rs. 4,000
contribution, "fraction will be eliminated by adding one to the full number (i.e., 572).
Hence, the difference.
Questions for Self-study
01. What is Variable Cost? Distinguish it from Fixed Cost and Semi-variable Cost.
02. Define Marginal Costing and explain its fundamental principles.
03. Define Contribution and explain how it differs from profit.
04. Describe PN Ratio and explain the relationship between PN ratio and BEP.
05. Define Break-Even Analysis and explain its managerial uses.
Marginal Costing : 745

06. Explain the following terms in relation to Marginal Costing: (a) MIS (b) PlY Ratio (c) Key
Factors (d) Variable Cost (e) BEP. [KuvemplI Vni, B.Com, May 1997J
07. "The technique of Marginal Costing can be a valuable aid to management'·. Discuss.
[Bangalore Vni, B.Com,J
08. What is BEC? Explain with a diagram the components of BEe.
[Kuvempu Vni., B.Com., October 1998J
09. Write briefly about C-V-P Analysis.
[Kuvempu Vni., October 1997 alld Mangalore Vni., B.Com., May 2002J
10. Critically evaluate the assumptions underlying the Break-Even Analysis.
11. 'Marginal Costing is a valuable tool to the managerial personnel.' Elucidate.
12. Draw a Break-Even Chart with illustrative figures. What are. the uses of this Chart?
13. Define Margin of Safety and what does it indicate?
14. Explain briefly the utility of Marginal Costing. [Kuvel1lpll Vni, B.Com, May 1999 and 2003J
15. Describe Angle of Incidence and explain how it rdlcl:ts the profit earning capability of a
company.
16. What is Break-Even Point? Explain the different approaches to the computation of Break-
Even Point.
17. Explain the following terms: (a) Key Factors (b) P/V Ratio (c) MIS.
[KlIVel1lpll Vni., B.COIll., Jlay 1000J
18. What is a BEC? State its significance. [KlIvempll Vni.• B.Com .• N()vember 2001J
19. Define Marginal Costing and identify the important area" of managerial.dcdsions opened up
by the application of Marginal Costing.
20. Explain PlY Graph. What useful information doe~ the PlY Graph provide?
21. How is Marginal Costing useful in the tlecision making of a firm?
IKlIvempll Vni., B.Com., May 2002J
22. What is meant by Marginal Costing? How it is essential for making managerial decisions
. I KlIvel1lpu VIIi., B.Com., May 2004J
23. "The effect of a price reduction is always to redul:c PlY Ratio. to raise the Break-Even Point
and to shorten the MIS." Explain and illu<;trate your view with appropriatc illustrations.
24. "Marginal Costing reveals not only the lowest price at which a product can be sold during a
·trade depression. but it also revcals to the management the most profitable lines during a
period of intense trade activity." Amplify the statement with suitable illustrations.
25. Explain the importance of Contribution in Marginal Costing.
{Manga/ore VIIi, B.Com, May 2002J
Management Accounting: 746
26. A few Short-answer Questions:
a. What is Contribution?
[Bangalore Uni., B.Com., May 2000 and 2003, April 2004, and Kuvempu Uni., B.Com.,
May 1991}
b. Define Angle oflncidence?
[Bangalore Uni., B.Com., November 2000 and 2003, and May 2001]
c. What is BEP? [Bangalore Uni., B.Com., November 2000 and 2003, and May 2001]
d. What do you understand by Margin of Safety?
[Bangalore Uni., B.Com, November 2000 and 2003, and Kuvempu Uni., B.Com., May
1991]
e. What is Marginal Costing? [Bangalore Uni, November 2001 and,May 2003]
f. What do you mean by Semi-variable Expenses? [Kuvempu Uni, B.Com, May 1991]
g. What is Break-Even Analysis? [Kuvempu Uni., B.Com., October 1997]
h. Distinguish between Marginitl Costing and Absorption Costing.
[Kuvempu Uni, B.Com, November 2003]
27. Mysore Sugar Company, Mandya has observed that its repairs and maintenance costs (R&M
Costs) amount to Rs. 1.5 crore when the company operates its machines for 10 lakh hours per
year. It is found that the annual repairs and maintenance costs increase to Rs. 2.5 crore when
the company operates its machines for 20 lakh hours. From this, find out the variable and
fixed portions of R&M costs. Also estimate the R&M costs that the company has to incur if it
uses its machines for' 30 lakh hours in a year.
28. HSL company sells its sole product N through its 100 sales representatives who are rewarded
on the basis of fixed salary every month plus commission based on the number of units sold
during that month. Given below are the data about the remuneration paid to the sales
representatives during the last six months.
Month: January February March April May June
Sales (lakh units); 20 32 12 28 41 30
. Remuneration (Rs. 000): 500 620 420 580 710 600
Ftom the above: (a) determine monthly salary and commission included in the sales
representatives' remuneration and express the same in the form of Y = mx + c; and (b) find
out the remuneration that the company has to pay to its sales representatives if they sell 48
lakh units during July.
29. You are given the following facts relating to the overhead expenses incurred and the number
of labour hours worked. Using these, compute the variable overhead expenses per direct
labour hour and also the monthly fixed overhead expenses. Use the Average Method.
Marginal Costing: 747

Month: Jan Feb Mar Apr May Jun Jul Aug


Direct Labour Hours: 240 200 250 300 180 220 230 240
OHEs (Rs. 000): 648 600 660 720 576 624 636 648
30. The following data are extracted from the records of Vivek Company for the year 2005.
Month No. of Machine Total Mixed
(2005) Hours Operated Cost (Rs.)
January 2,400 5,000
February 2,000 4,800
March 2,500 5,100
April 2,200 4,900
May 2,400 5,100
June 2,100 4,900
July 2,300 4.900
August 1,900 4,600
September 2,000 4,850
October 2,200 4,850
November 2,500 5,500
December 2,100 5,000
Required: (a) Using the Least-Squares Method, find out the unit variable cost and
the monthly fixed cost included in the mixed cost;
(b) Also determine the cost formula for mixed cost.
31. A company sells, on an average, 8,000 units at Rs. 12 a unit by incurring variable costs of Rs.
38,400 and fixed costs of Rs. 52,000. The company wants to earn an after-tax profit of Rs.
15,000 and the applicable tax rate is 25%. Find out the required sales.
32. M Company manufactures one standard product. The standard costs and selling price are:
VCs: Rs. 2 per unit: FCs: Rs. 90,000 p.a.; SP: Rs. 5 per unit. Further, the company plans to
earn 15% profit on selling price. Find out the number of units to be sold and the amount of
sales revenue to be earned to earn the desired profit.
33. A company which produces and sells two products in the ratio of 3 : 5 furnishes the
following: unit SP: A Rs. 10 and B Rs. 15; unit VC: A Rs. 8 and B Rs. 10; and total FCs: Rs.
6,20,000 p.a. Using these data, compute the composite BEQ.
34. A company selling three products, P, Q and R is experiencing that every Rs. 100 sales
revenue it earns comprises of Rs. 20 from P, Rs. 50 from Q and the remaining Rs. 30 from R.
But the product 'P' is more profitable with 70% Plv ratio followed by R with 40% and lastly,
Management Accounting : 748

Q with 20%. On an average. the company incurs Rs. 18 lakh per annum of FCs. From these.
compute the composite BEP.
35. On the ba.~is of the following information. compute both the individual products' and the
company's break-even points. FCs of Rs. 65.000 are to be apportioned to A. Band C in the
ratio of 4 : 6 : 3.
Product Selling Price (Rs.) Sales Qll~lI1tity (units) Variable Cost (Rs.)
A 10 10.000 6
B 12 15.000 8
C X 5,000 2
36. A company manufactun.·s and markets three products M. Nand P. An analysis of the cost
reveals the following. .
MCRs.) N (Rs.) P (Rs.)
Direct Material Cost per unit 6 10 8
Direct Labour Cost per unit 3 7 5
Variable Overhead Expensc~ pcr unit 6 8 7
Fixed Cost: Specitk: 50.000 80.000 70.000
Common: Rs. 2.60.000
Selling Pri~c 25 50 30
From the above. ·compute thl' company's BEP assuming that the company sells its products
M. Nand Pin thl! ratio of 3: 2 : 5 re!>pcctively.
37. PH Ltd .. manufactured and sold two products during 1981 as per particulars given below.
Product A (Rli.) Product B (Rs.)
Quantity (units) 6.0n.OOn 3:00.000
Rs./unit R!l./unit
Selling price 6'<)0 10.0f>
Direct materials 1.00 2.00
Direct wages 1.20 2.60
Other overheads (5Wi(. variable)' l.UU 0.60
Variable factory overheads arc absorbed as a percentage of direct wages. The summarized
statement of prolitability for 19H I i~ as umk·r.
Rs. in lakhs
Sales 66.00
Direct materials 12.00
Direct wages 15.00
Factory Qverheads 13.50 [of this, Rs. 6lakh is fixed]
Marginal Costing: 749
Other overheads (50% fixed) 7.80
For the -year 1982. due to fall in demand. the production and sale~ of product 'A' wi II be
reduced by 20% and of product 'B' by 40%. It is therefore decided to introduce a new
product -C'. the cost particulars of which are as under.
Production and Sales: 2.00.000 units

Rs. per unit


Selling price 7.00
Direct materials 1.40
Direct wages 2.40
Other variable overheads: same as in Product A
The fixed overheads will remain the same and the variable overheads will continue to be
incurred at the same rate as in 1981. You are required to:
(i) Prepare a product-wise and overall budget for 19H2; '~tI1d
(ii) State what conclusion can be drawn from the budget for 1982 by using Marginal
Costing Technique~ [ICWA (Fill), December 1981}
38. The Himalayan House Ltd., is a large retailer in mountaineering equipment. The company'-;
Profit and Loss Account for the year ended 31st March 1983 is given below.
Sales Rs. 24,00,000
Less: Cost of goods sold 15,00,000
Gross margin 9,00,000
Less: Operating expenses:
Selling expenses Rs. 3.00.000
Administration expenses 1,50,000 4,50,000
Net profit 4,50,000
The selling expenses are 90% variable and the -administrative expenses are 20% variable.
You are required to rewrite the profit and loss account in the "contribution" format and also to
calculate the break even sales. [ICWA (Int), June 1983J
39. Merry Manufacturer Ltd., has supplied you the following information in respect of one of its
products.
Total fixed costs Rs.18,000
Total variable costs Rs.30,o00
Total ,sales -Rs.60,000
Sales vQ~yme (units) 20,,000
Management Accounting : 750
Find out (a) contribution per unit (b) break even point (c) margin of safety (d) profit ana (e)
volume of sales to earn a profit of Rs. 24,000. {ICWA ,(Int)]
40. Your company manufactures a single product. The selling price is Rs. 100 per unit.
C~ently the capacity utilization is 60% with the sales turnover of Rs. 6 lakhs. The company
proposes to reduce the selling price by 20% but desires to maintain the same profit position
by increasing the output. Assuming that the increased output could be made and sold,
determine the level at which the company should operate to achieve the desired objectives.
The following further data are available:
a. Variable cost per unit Rs. 20. ~
b. Senti-variable cost (including a variable element of Rs. 5 per unit) Rs. 60,000.
c. Fixed cost of Rs. 2,00,000 will remain constant up to 80% level. Beyond this, an
additional amount of Rs. 40,000 will be needed. 1M. Com., UoM, May 1986]
41. The following is a summary of the trading results of a company selling an electrical
appliances for the year ended 31st December 1983 during which 80,000 units were sold (Rs.
lakhs).
Sales 96
Costs: Material 36
Labour: Direct 15
Indirect 6
Other costs 18 75
Profit 21
Taking -into consideration the following matters, prepare a summary of the expected results
for the following year.
(a) The selling price is to be reduced by Rs. 7.50
(b) Sales volume is expected to increase by 40%
(c) Suppliers have agreed to give a discount of 5% on all purchases of materials.
(d) Direct workmen are to be paid an incentive bonus of 2.5% in order to stimulate
production. Indirect labour is not expected to increase the following year.
(e) Other costs vary directly with production except to the extent of Rs. 3 lakhs which is
considered 'fixed' and an additional expenses of Rs. 1 lakh will arise due to rent in
respect of an extension of the factory.
(f) You are to assume that there are no stock or work in progress as at 31st December.
ICA (lnt), May 1984]
42. Sales of a product amounts to 200 units per month at Rs. 10 per unit. Fixed overhead is Rs.
400 per month and variable cost Rs. 6 per unit. There is a propos~ to reduce prices by 10 per
Marginal Costing : 751

cent. Calculate present and future P/v ratio and how many units must be sold to maintain
total profit. EM. Com., Karn. Uni.,1980J
Answers
27. Rs. lOx + Rs. 50,00,000; Rs. 3.5 crore.
28. O.Ix + Rs. 3,00,000; Rs. 7.8 iakh.
29. Rs, 1,200; Rs. 3.6Iakh.
30. Rs. 0.87218x + Rs. 3,025.
31. Rs. 1,20,000 or 10,000 units.
32. 40,000 units; Rs. 2,00,000.
33. 1,60,000 units.
34. Rs. 50,00,000.
35. 5,000 (A); 7,500 (B); 2,500 (C); 15,000 (Total) units; and Rs. 50,000 (A); Rs. 90,000 (B); Rs.
20,000 (c); Rs. 1,60,000 (Total).
36. Company BEQ = 35,200 units comprising of 11,000 units ofM, 7,200 units ofN, and 17,000
units ofP.
37. Contribution from A =Rs. 12,96,000; B =Rs. 6,84,000; and from C = Rs. 3,00,000; Profit =
Rs. 12,90,000 (if diversified) and Rs. 9,90,000 (no diversification).
38. Rs. 6,00,000.
39. (a) Rs. 1.5; (b) 12,000 units or Rs. 36,000; (c) 8,000 units or Rs. 24,000 or 40%; (d) Rs.
12,000; (e) 28,000 units.
40. 8,910 units or 89.1 % capacity.
41. Estimated profit Rs. 25,59,500.
42. Plv Ratio: Present 40% and Future 33.13%; 267 units are to be sold ·to maintain the total
profit.
Chapter-IX

MANAGERIAL REPORTING

Objectives: Objectives of this chapter are:


• To know the Meaning, Need for and Importance of, Managerial Reporting.
• To understand the Fundamental Principles of Managerial Reports.
• To Know the Modes and Types of Reporting.
Structure
• Introduction
• Meaning of Management Reporting
• Fundamental Principles of Managerial Reports
• Types or Kinds of Reports
o Internal and External Reports
o Functional Reports
• Material Reports
• Labour Reports
• Overhead Cost Reports
Marketing Reports
Financial Reports
• Summary, of the Chapter
• Key-Terms to Remember
• References
• Questions for Self-study
Managerial Reporting : 753

Introduction
The success or otherwise of. any undertaking depends on the honesty. engrossment.
efficiency. etc .• of the pe9ple,' more particularly the people at the helm of affairs. working in the
undertaking. This is an irrefutable fact. Because. it ,is. the management which decides the direction
in which the undertaking has to move. The, management. therefore. decides the fate ,of the
company through its decisions and their implementFrtion. If the management fails to discharge its
duty satisfactorily at any stage and at any point of d.me. its repercussions on the .performance of
the unit is unfathomable. Because. th~ eff.ects of the decisions already .taken ""nd j.DJplemented
cannot be avoided without mu,ch' loss. The 'management with its prude,rice:- 'prospicie~ce and
precise decisions is capable of trapsforming even the 'enterprises on the Verge 'of bankruptcy: into
sound entities. For instance. Guest Keen Williams came close to ext;.~ctiori ,in tlie:la~ e.ighties,
but several measures taken ,by the management to improve the coQtpany's operations
brought about a dramatic'reversal of its'fortunes. 1 This clearly brings out the impQrtance of
managerial decisions. In order to take proper decisions. management needs reports comprising of
all the relevant data. Hence. the need for managerial or management reporting system.
Management Reporting - Meaning
The word report denotes a form of upward communication. Because. the word 'r~port' is
normally used for factual co~m)Jnication f~mishing the relevant 'inform'ation about the
performance of functional areas of th~ organiz<,ltion by a'lower level to ~. higher l~'vel'of authority.
On the other hand. the word communication ·r~fers to it form of downward communication (i.e ..
from higher level to lower level). For example. orders are communicated by the superior to the
subordinates. and the results achieved are communicated by the subordinates to .the superiors., The
word information represents the data processed or evaluated. for the specific'purpoSe.:!
In the light of the above. Management Reporting has been defined as a system of
communication, normally in the written form, of facts which should be brought to the
attention of various levels of management who use them to take suitable action. 3 Dr.
Maheshwari has also detined Management Reporting System as an organized method of
providing each manager with ... only those data which he needs for his decisions, when he
needs them and in a form which aids his understanding and stimulates his action.4
According to Stoner. Management Information System is a formal method of making available
to management the account and timely information necessary to facilitate the decision
making process and enable the organisation's planning, control and operational functions to
be carried out effectively. That means. Management Reporting aims at serving the managerial
personnel by providing relevant information so that they discharge their functions pertaining to
organisation. planning. q.ecision-making and controlling satisfactorily.
Managerial Reports which provide the necessary and relevant information to the
managerial personnel for the purpose of enabling them to take appropriate and timely decisions
play an important role. Because. they aim at furnishing and/or providing relevant information
with requisite analysis and evaluation in suitable form and at right time to the management.
Management takes various decisions based on these reports. Therefore. Managerial Reports are
very essential and in the absence of these Reports. management is certainly not able to take the
right decisions.
Management Accounting : 754

Fundamental Principles of Managerial Reports


A careful analysis of the definitions of Managerial Reporting helps to identify certain
essential features of Management Reporting System. These features may also be interpreted as
the principles to be observed while reporting to the managerial personnel. Observance of thes~
principles is necessary as the reports to management influence the decisions of the managerial
personnel. Because, management takes decisions on the basis of the reports submitted to it by the
subordinates. It is, therefore, necessary to observe certain well established principles at the time
of preparing and submitting reports to the managerial personnel.
1. Objectivity: The reports to the management should include, as far as possible, the accurate
facts and figures. Because, the management takes the decisions on the basis of the information
included in the reports. There is, therefore, every possibility of management taking a wrong
decision if the Management Reports contain inaccurate, biased, wrong and distorted
information. The Reports should, therefore, include only the facts and figures which are not
intentionally distorted and which are subject to verification. Nigam and Sharma have,
therefore, rightly said reports inform, people perform. s However, it should be noted at this
stage that complete objectivity is both expensive and difficult. Because, most of the
Management Reports include the predictions about the future. And the prediction about the
future is essential for the purpose of taking various managerial decisions. Therefore, it is
necessary to use the most scientific methods to predict the future and report the same to the
management.
2. Comparability: The Reports should provide the information about both the actual and the
budgeted performance for the budget period so that meaningful comparison can be made to
find out the deviations and to initiate appropriate action. Further, the figures may be furnished
not only for the current period but also for the last 4-5 periods so that the trends in different
performance indicators can be identified. This "type of comparison helps the management to
keep a_yigilant eye on the performance of different functional areas.
3. Consistency: In order to make a meaningful and useful comparison, it is necessary on the part
of the reporting agency to use the accounting principles and practices on consistent basis year
after year. Because, the reports are submitted with a view to assist the managerial personnel to
take various appropriate decisions. Further, there should be a consistency in the preparation
and submission of reports to various levels of management. The summarised reports
submitted to the top management should agree with the detailed reports submitted to the
departmental or functional managers.
4. Relevancy: The Reports meant for use by the managerial personnel should include only the
relevant factors which have bearing on the decisions under the consideration of management.
Because, inclusion of both the relevant and the irrelevant data in the Management Reports
may result in faulty decisions. Therefore, only those factors which have an effect on the
decisions under the consideration of management are to be included in the reports.
5. Simplicity: The Reports should be as far as possible in simple form. And they should convey
the same meaning to the management which the reporting authority intends to convey. The
reports should, therefore., be prepared in clear, simple and lucid form, and they should easily
Managerial Reporting : 755

be understandable in the right perspective by the management. Further, there should not be
any scope for misinterpretation of the terminologies used in the reports.
6. Promptness: Since the managerial actions are based on the reports, there should be a prompt
submission of the reports by the reporting authorities to the managerial personnel. Because, in
the absence of proper reports, the management will not be in a position to take any decision
and to initiate appropriate action. Therefore, promptness in the preparation and submission of
reports assumes importance.
7. Distinction between Controllable and Non-controllable Variances: The Reports should
clearly indicate the areas in which a particular Responsibility Centre has achieved the target
and the areas in which it has failed. Further, the adverse variances are to be analysed into
controllable and non-controllable. Because, the head of the Responsibility Centre can be held
responsible only for controllable variances but not for variances which are beyond his control.
Further, in order to assist the management to initiate remedial measures, probable reasons for
the variances should also be incorporated in the reports.
8. Principle of Exception: Since the time and effort of managerial personnel are precious and
therefore, to be conserved, the reporting authorities should draw the attention of the
managerial personnel to only those activities which are not moving as anticipated. Because,
when the departments operate according to the standards and if their performances match with
the anticipated, there is no need for the management to waste its time on these aspects. It is
necessary, therefore, to draw the attention of management, through reports, only towards
exceptional matters.
This way, certain well established principles are to be followed while reporting to the
management. All principles aim at increasing the utility of these reports to the management. The
reports should help the management to take the most economical and profitable decisions.
Modes and Types of Reporting
Basically, there are two ways to report to the management. They are oral and written.
The reporting authority may report orally to the managerial personnel either meeting them
personally or in group meetings. Written Reports can be classified into two or more categories on
a number of bases as presented below.
Management Accounting : 756

Types of Reports
;--7 Reports, to Production I?epartment
Reports to Sales Department
Routine
Reports Reports to Finance Department '
Cost Report~

~ Reports to Top Management


According '.-
to
Frequency
_ Pricing
-Make or Buy -
Special Replacement
Repotts :Product Diversification
Product Mix
Written , -On Other Similar
Reports Decision Making Areas

External Reports
Acc9rding to PU;1ies ;--7
to be Served
C,..'

.
, ,

,~~'t~rnal or Management Reports

Material Reports
Labour Reports

According to Production Reports


Functions Overhead Reports
Marketing Reports
Financial Reports
From the figure. it may be noted that there is no specific yardstick about the number,
. format, etc .• of Reports. For instance. on the basis of the parties to whom the Reports are to be
submitted, Reports can be classified into external and internal reports. External Reports T€present
those reports which are prepared for circulation among the parties or persons who are external to
the business 'organizations such as shareholders. debentureholders. bankers, financial institutions.
government and it~ agencies. regulatory bodies. etc. External Reports usually contain the
published annual reports. But the emphasis Qf this chapter is only on the internal reports. I~ternal
Reports are those which are prepared for circulation among the parties or persons who are
internal to the company. viz .• managerial personnel. The internal reports are, therefore, called
Managerial Reporting : 757

management reports. These reports include both the routine and the special reports which may
also be regrouped into a number of functional reports.
Routine Reports rep'resent the reports which are prepared and submitted to different levels
of manageme'nt according to the fixed time schedule. For instance. there may be a standing
instructi{)n by the managing director to the marketing manager to submit the reports about the
actual sales against the budgeted on weekly basis. In this case. the marketing manager submits the
~ales reports to the managing director every week. It is a routine work. Besides these routine
reports, management needs some special reports for specific purposes. For instance. managing
director asks the marketing manager to submit a report within a month's time about the
commercial viability of entering into a new market. Here. two things are to be observed. One, the
purpose for which the report is caIled for. and two. the time duration within which the report is to
be submitted. A special report is to serve a specific purpose and it.is to be submitted only once.
That means. it is not submitted frequently or regularly.
Management Reports can also be c1assitied into three mor~ categories. viz., reports to top
management. to middle level management and to lower level management. Contents. frequency.
etc .• of reports differ from one level of management to another level. For instance. Reports to
lower level management should be more descriptive and to the top management. they should be in
summarised form. Further. more frequent reporting is, necessary in the case of lower level
management and in the cac;e of top management. a less frequent reporting is advisable. Any how,
there is no specific rule as opined earlier about the number. type. etc., of reports and also for
classifying the reports. However. an attempt is made in the following paragraphs to describe a few
management reports with an emphasis on functional reports.
Material Reports: Material cost is an important element of total cost and therefore. it
provides a number of avenues to the management to control it~ costs. Because. each one of the
three aSpel.:h I)f material (viz .. purchase. storage. and usa~e) enables the management to control
and reduce the costs. Material Reports should. therefore. assist the management to take proper
decisions 'ill that the company reaps. the full benefits of cost reduction. Important aspects to be
included in the Reports are as follows.
I. Material requirements - items-wise and time-wise;
2. Comparison of actual material price with,that of standard:
3. Information about material usage - both estimated and actual; reasons for the
difference:
4. Details about material carrying cost, loss of materials in the storage, analysis of storage
loss into controllable and uncontrollable;
5. Material productivity. trends in material productivity over a period of time (say, five
years). analysis of reasons for changes in the material productivity; ,
6. Information pertaining to waste. loss. ,etc., of material;
7. Trends in the prices of various items of material;
8. Average inventory and capital locked up in average inventory; inventory turnover ratio;
etc.
Management Accounting : 758
Labour Reports: It is very well known that the labour cost forms a major part of total cost
of production and sales. Further, the labour force is capable of doing a miracle and therefore, it is
necessary to utilize the available human resources more productively and effectively. Besides,
the company must aim at keeping the labour cost at the minimum level by exercising rigorous
control on the same. Because, even a 10% reduction in labour cost ensures 5% reduction in total
cost (assuming labour cost to total cost ratio of 50%) which is not a small amount. The report on
labour must, therefore, include all the relevant data about the labour force and the labour cost to
enabl~ the management to achieve the above dual objectives, viz., minimization of labour cost
and maximization of labour utilization. However, as in the case of other reports, it is very
difficult to specify the contents of labour reports. Anyhow, the following details are useful and
therefore, they should be included in the labour reports.
1. Labour force -- availability, requirement and the need, if any, for addition, etc.,
2. Labour productivity, in terms of output per man-day or in terms of any other measuring
unit;
3. Comparison of actual and budgeted labour productivity, measurement of variances
and analysis of reasons for the same;
4. Idle time, and idle time cost - analysis of idle time into controllable and uncontrollable;
5. Overtime work and overtime premium - reasons for overtime work;
6. Labour cost - both budgeted and actuals - difference between the same, reasons for the
difference - whether it is due to the rate variance or efficiency variance, etc.;
7. Labour turnover, analysis of cost of labour turnover, reasons for labour turnover, etc.
Overhead Cost Reports: Overhead expenses represent the aggregate of indirect material
cost, indirect labour cost and other indirect expenses. They include both the production,
administrative, and selling and distribution overhead expenses. Further, they include both the
variable, fixed and semi-variable overhead expenses. Most importantly, they constitute a major
part of total cost. Hence, the reports on overhead expenses are to be prepared and submitted to
management to enable it to take necessary corrective measures whenever the need arises. These
reports should include the following information, amongst others:
1. Details about the budgeted and the actual overhead expenses and also about the variances;
2. Information about the overhead expenses absorbed, over- or under-absorbed overhead
expenses, reasons for over- or under-absorbed overhead expenses, and disposal of the same;
3. Overhead cost ratio, trends in overhead cost ratio, reasons for increase or decrease in the
ratio, etc;
4. Utilization of machine capacity, loss of machine hours, reasons for the same and its
influence on overhead expenses, etc.
Marketing Reports: In order to accomplish the corporate objectives, it is necessary for
the corporate entities to promote the sale of goods and services they manufacture. Marketing
Reports play a stupendous role by drawing the attention of the management to the areas which
need action. Marketing Reports should, therefore, furnish all the relevant information to the
management. However, it is very difficult to specify the contents of Marketing Reports.
Because, a Marketing Report may contain only the information about budgeted and actual sales.
Managerial Reporting : 759

Another report may include the information about the performans;e of sales representatives
against the targets fixed for them. The third report may furnish the information about the
product-wise sales. It all depends upon what information the management needs for taking
proper decisions to achieve the desired result. It also depends upon the aspects which the
reporting authorities wish to report to the management. However, the following aspects are
normally reported to the management.
1. Budgeted sales, actual sales and the difference between the two;
2. Whether the difference between the budgeted and the actual sales revenue is due to the
difference between the budgeted and the actual sales volume or due to the difference
between the standard and the actual selling price or due to both;
3. Analysis of variances into controllable and uncontrollable and identifying the
division (or the person) which (or who) is responsible for the controllable variances;
'4. Analysis of sales - both territory-wise, officer-wise, product-wise. etc;
5. Comparison of current sales with that of the previous year/s and finding out the
trends;
6. Analysis of sales into cash sales and credit sales, costs and benefits associated with
the credit sales (e.g" collection charges, bad debts, cost of additional working
capital, increase in sales due to credit facility offered, etc .• );
7. Information about the sales commission, discount. etc;
8. Presence of. and increase in, unsold stock. reasons for the same. etc.
Marketing reports are, therefore. to be prepared furnishing the above and other relevant
information so that the management takes proper decision at the right time.
Financial Reports: Financial Reports furnish the relevant information to the management
enabling it to form an idea about the present and future financial position of the company. These
reports should include the information about the sources from which the funds were mobilized
during a specified period, the proportion in which various forms of capital were raised, the
manner in which the funds so mobilized was utilized. etc. Further. the reports should enable the
management to judge whether the company's liabilities are adequately supported by assets.
Because, in order to achieve the primary corporate objective, it is necessary to mobilize only the
required fund from the most economical sources and to employ the same more judiciously,
intelligently and productively. The Financial Reports should include all the relevant data so that
the management takes appropriate decisions whenever the need arises.
'These reports may be prepared showing the financial position at the end of a particular
date (say, at the end of the year), The Balance Sheet of a business undertaking is an example to
this. Since the assets-liabilities status at the end of a particular day is shown here, it is called static
fmancial report. On the other hand. the Financial Report may also be prepared showing the
movement of funds during a specified period. Since the changes in the funds during a specified
period are included in this type of reports, they are called dynamic fmancial reports. Cash Flow
Statement, Funds Flow Statement, etc., are examples to this type of report. Further, the Financial
Reports may also include some ratios which shed light on liquidity, solvency, etc., of the
company.
Management Accounting : 760

Conclusion
In conclusion it may be said that the Managerial Reporting System is very essential for
the successful management of activities of any business undertaking. To make the reporting
system more effective, meaningful and useful. the reporting authorities must work very efficiently
and promptly. Because, in the absense of adequate and relevant information at the right time,
management will not be in a position to discharge its responsibility satisfactorily.
mustration: 9.1
A company, producing 40,000 units of product M working at 80% capacity, receives an
order from foreign dealer-for 10,000 units at Rs.50 per unit although the local sales price is Rs.90
perunit. The present cost sheet is given below:
Materials Rs.20
Labour-Skilled (fixed) 10
Unskilled (variable) 10
Variable overheads 10
Fixed overheads 20
Total Rs. 70 per unit

a. Advise the management whether lo accept the foreign order or not.


b. What will be your advise if the order had come from a local merchant?
[Kllvempll VIli., B.Com., May 1991 and Manga/ore Vlli., B.Com., October 2002J
Solution:
(a) Differential Cost Statement
Amount (Rs.)
Particulars At 80% At 100% Difference
(40,000) (50,000) 20% (10,000)
Material Cost [Rs.20 per unit) 8,00,000 10,00,000 2,00,000
Labour Cost [Unskilled, variable at Rs.I 0 a unit] 4,00.000 5.00,000 1,00,000
Variable Overheads [at Rs.1O a unit] 4,00,000 5,00,000 1,00,000
:. Total Variable Cost 16.00.000 20.00.000 4.00,000
Fixed Cost * 12.00.000 12.00,000 0
Total losl 2H.OO.OOO ! 32.00.000 4.00.000

• at 80%. it comes to Rs.12.00,OOO as shown below:


Managerial Reporting: 761

Wages of skilled work~rs, fixed, at Rs.10 x 40,000 units =Rs. 4,00,000


Fixed overheads at Rs.20 a unit'" 40,000 units =Rs. 8,00,000
Rs. 12,00,000
And this total fixed cost of Rs.12,00,000 is assumed to be constant even at 100% capacity
utilization.
From the above, it is obvious that the company has to incur an additional cost of
RsA,OO,Ooo to produce another 10,000 units by increasing the capacity utilization from 80% to
100%. Therefore, the cost per unit works out to RsAO [Le., RsA,OO,OOO + 10,000 units] which is
lower than the price of Rs.50 offered by the foreign dealer. It is a profitable proposition. Hence,
it is to be accepted.
(b) If this special offer is from a local merchant. it is not advisable to accept the same as it
results in the price discrimination. The regular customers certainly demand the product at the
reduced price of Rs.50 which results in loss to the company.
lliustration: 9.2
Following information has been made available from the cost records of United
Automobiles Ltd., for the manufacturing of Product X and Product Y:
Per unit
Direct materials for:
Product X Rs.8
Product Y Rs.6
Direct wages for:
Product X 24 hours at 25 paise per hour
Product Y 16 hours at 25 paise per hour
Variable overheads: 150o/q of wages
Total fixed overheads Rs.750 p.a. [for X and Y tQgether)
Selling Price:
Product X Rs.25
Product Y Rs.20

The directors want to have the following alternative sales mix in the next budget period:
a. 250 units of X and 250 units of Y.
b. 400 units of Y only
c. 400 units of X and 100 units of Y
d. 150 units of X and 350 units of Y
State which of the alternative sales mix you would recommend to the management and
why?
Management Accounting : 762

[Mangalore Uni., B.Com., October 2003 and May 2004J


Solution:
Unit Contribution Statement
Product
Particulars
X (Rs.) Y (Rs.)
Direct Material Cost 8.00 6.00
Direct Wages [24 x 0.25; 16 x 0.25] 6.00 4.00
Variable Overheads [150% of Wages] 9.00 6.00
:. Unit Variable Cost 23.00 16.00
Selling Price 25.00 20.00
:. Unit Contribution 2.00 4.00
In this problem. there are no constraints [i.e. limiting factors] and therefore, the decision
about the selection of profitable sales mix should be based on the unit contribution and the sales
quantity. It is obvious from the above Income Statement that product Y is more profitable [Rs.4]
than product X [Rs.2]. Hence, the mix with more number of units of product Y is more
profitable. Between the alternative sales mixes (b) and (d), mix (d) is more profitable. Because,
the contribution from 150 units of Product X (Rs.2 x 150 = Rs.300] is higher than from 50 extra
units of Product Y [alternative (b)]. Hence. alternative (d) is recommended. The following
calculations substantiate this recommendation.
Evaluation of Alternative Sales Mixes
Alternative Sales Mix
Particulars
(a) (b) (c) (d)
Number of units of Product: X 250 0 400 150
Y 250 400 100 350
Contribution from Product:
X (at Rs.2 a unit) (Rs.) 500 0 800 300
Y (at Rs.4 a unit) (Rs.) 1,000 1,600 400 L,400
Total Contribution 1,500 1,600 1,200 1,700
Less : Fixed Cost 750 750 750 750
:. Profit 750 850 450 950

Illustration: 9.3
A Radio Manufacturing Company finds that it costs Rs.6.25 each to make a component X
273. The same is available in the market at Rs.5.75 each, with an assurance of continued supply.
The break up of cost is:
Managerial Reporting : 763

Materials Rs.2.75
Labour 1.75
Other Variable Cost 0.50
Fixed Cost 1.25
Total Cost 6.25
a. Should you make or buy
b. What would be your decision if supplier offered the component at Rs.4.85 each.
[Mangalore Uni., B.Com., October 2000J
Solution:
a. Variable cost of manufacture of component X 273 per unit:
Materials Rs. 2.75
Labour l.75
Other Variable Cost 0.50
5.00
Purchase Price per unit of X 273 = Rs.5.75
Since the variable manufacturing cost per unit of component X 273 is lower than
its market price, it is economical to manufacture the component by the company
internally on its own as it saves 75 paise [i.e., Rs.5.75 Market Price - Rs.5 Variable
Manufacturing Cost] on each unit produced internally. Here, the fixed cost [of Rs.l.25]
is not considered as it is to be incurred by the company whether it makes or buys the
component.
b. However, if the supplier offers the component at Rs.4.85 a unit, it is better to buy the same
from the external market as it is lower than the variable manufacturing cost of Rs.5 per unit.
As a result, on the purchase of each unit, the company saves 15 paise [i.e .• Rs.5 - Rs.4.85]
when compared to the cost of manufacturing.
Illustration: 9.4
X Ltd., produces two products. From the following information, show which product is
more profitable, time being the key factor.
Per unit Per unit
of A ofB
Materials Rs.60.00 Rs.60.00
Labour (Rs.12 per hour) 48.00 36.00
Fixed overhead 36.00 36.00
Variable overheads (Rs.12 per hour) 48.00 36.00
Total Cost 192.00 168.00
Management Accounting: 764

Profit 28.00 24.00


Selling Price 220.00 192.00
[Mangalore Uni., BBM, May 1998J
Solution:
Statement of Profitability
Product
Particulars
A(Rs.) B(Rs.)
Variable Cost per unit:
Material Cost . 60.00 60.00
Labour Cost [4 hours and 3 hours at Rs.12 per hour] 48.00 36.00
Variable Overheads [4 hours and 3 hours at Rs.12 per hour] 48.00 36.00
156.00 132.00
Unit Selling Price 220.00 192.00
:.Contribution per unit 64.00 60.00

j
Number of Hours per unit 4 3

r Unit
Contribution
Profitability = Contribution per Hour = No.ofHours 16.00 20.00
per Unit
'-
Hence, Product B is more profitable when compared to Product A. Because, one hour of
time devoted for the production of Product A ensures a contribution of only Rs.16. However, if
the same one hour of time is devoted for the production of Product B, it promises a contribution
of Rs.20.
Dlustration: 9.5
The following figures relate to a manufacturer of electric fans for a period of three months
ending 31.12.1999.
Completed stock on 1.10.1999 Rs. Nil
Completed stock on 31.12.1999 40,500
Stock of materials on 1.10.1999 10,000
Stock of materials on 31.12.1999 7,000
Wages 1,50,000
Indirect charges 25,000
Materials purchased 65,000
Sales 2,25,000
Managerial Reporting : 765

The number of fans manufactured during three months was 5,000. Prepare a statement
showing the cost per fan and the price to be quoted for 750 fans to realise the same percentage of
profit as was realised during the three months.
[Mangalore Uni., BBM, April2000}
Solution:
Income Statement for the three-month period ended December 31, 1999
Amount
Particulars Per Unit (Rs.)
(Rs.)
Cost of Opening Stock of Material 10,000
Add: Purchases 65,000
75,000
Less: Closing Stock of Material 7,000

Cost of Material 68,000 13.60


Wages 1,50,000 30.00
Indirect Charges 25,000 5.00
2,43,000 48.60
Add: Cost of Opening Stock of Completed Stock 0
2,43,000
Less: Cost of Closing Stock of Completed Stock 40,500

:. Cost of Sales 2,02,500


Sales Revenue 2,25.000

:.Profit 22,500

Percentage Of} = lCRs.22,500


Profit to Sales Rs.2.25.000
x IOJ = 10% [or 119 of Cost of Sales]
)

Statement of Price Quotation for 750 Fans


Amount (Rs.)
Particulars
Per Fan I For 750 Fans
Management Accounting : 766

Material Cost 13.60 10,200


Wages 30.00 22,500
Prime Cost 43.60 32,700
Indirect Charges 5.00 3,750
Total Cost 48.60 36,450
Add: Profit [1/9 of Cost or 10% of Sales] 5.40 4,050
:. Price to be Quoted 54.00 40,500

Summary of the Chapter


The chapter starts with an introduction to Managerial Reporting in the form of need for,
and importance of, Managerial Reports followed by meaning and definition of Managerial
Reports. Then, the Fundamental Principles of Reports are discussed in detail. Finally, a brief
description about different types of Managerial Reports is given followed by the analysis of few
problems pertaining to different decision areas.
Key-Terms to Remember
Report Communication
Information Management Reports
Oral Reports Written Reports
External Reports Internal Reports
Routine Reports Special Reports
Material Reports Labour Reports
Overhead Cost Reports Marketing Reports
Financial Reports
Notes and References
1. Clifton Desilva in Business World, August 12-25, 1992, p. 43.
2. Dr. S.N. Maheshwari, Management Accounting and Financial Control, New Delhi.
Sultan Chand & Sons, 10th Ed., 1995. p.c. 294.
3. A slightly modified version of the definition of Cost Reports by Patrick A.W. in Robort I
Dickoy (Ed.) Accountants Cost Hand Book. 1960, p. 20.36.
4. Op Cit., Dr. S.N.Maheshwari, p.c. 294.
5. Nigam and Sharma, Advanced Cost A.ccoulltancy, Bombay. Himalaya Publishing House,
2 Ed., 1983, p.1093.
Questions for Self-study
01. Define Managerial Reporting and bring out a case for Management Reporting.
02. What do you understand by the term 'Management Reporting'? What are the guiding
principles of Management Reporting'?
Managerial Reporting : 767

03. What do you mean by reporting to management? Explain the basic requisites of a good
Report. [Kuvempll UnL, November 2001J
04. What do you mean by Reporting to management? Discuss their uses and explain the
requisites of a good report. [Kuvempu Uni, B.Com, October 1999J
05. What do you mean by Reporting to management'? What are its essentials?
[Kllvempu Uni, B.Com, May 1998J
06. Write a note on 'Reports inform, People perform.'
07. What are the bases on which the Reports are classified? Describe Routine Report!o. and
distinguish them from Special Reports.
08. Discuss the general principles to be observed while preparing Reports.
[Bangalore Uni., B.Com., November 2003J
09. What do you mean by 'Reporting to Management''? What are the essentials and requisites of
good Reporting'? [Kuvempll Ulli, B.COI1l, October 1997J
10. Explain the essentials of a 'Good Reporting System'! [Kllvempu Uni, B.Com, October 1997J
11. What do you understand by Internal Reports? Explain how they differ from External Reports ..
12. Write a note on Static and Dynamic Reports.
13. Explain the various kinds of Reports prepared by Management Accountant and for different
levels of Management. [Bangalore Uni, B.Com, May 2002J
14. Describe the usual contents of Material Reports.
15. ABC Company is facing the problem of Raw-material shortage. As a Management
Accountant. you are required to draft a report suggesting reformulation of purchase policy to
have unintenupted supply of Raw-materials. {Bangalore Un;, B.Com, November 2000J
16. Critically evaluate the statement that 'the regular tlow of information within a business entity
is essential for proper planning, co-ordinating and controlling of efforts.'
17. Profits of XL Co .. are declining year after year. As the Management Accountant of the
company. draft a Report to the management rep0l1ing the reasons for declining profit and
suggest the conective measures.
[Bangalore Uni, B.Com, May 2000, and November 2001 and 2002J
18. Explain the utility of Management Reporting. What are the essentials of a Good Reporting
System? [Klivempll Uni, B.Com, May 1999 and 2000, and November 2000J
19. "Accounting Reports are a matter of necessity for the management and not a matter of
conven~ence·'. Explain. [Bangalore Uni, B.ComJ
20. State the essentials of an Ideal Report. [Kllvempll Uni, B.Com, May 2002J
21. Your management is not satisfied with the meager profit that your company is making. You
have been asked a" a Management Accountant to prepare a Report stating areas you should
look into with the idea of improving profits. [Bangalore Uni, B.ComJ
22. Critically evaluate the relevance and utility of Functional Reports to the managerial
personnel.
Management Accounting : 768

23. What are the various types of Management Reports? Explain the Functional Reports very
briefly?
24. A Few Short-answer Questions:
a. Mention four objectives of Management Reporting.
[Bangalore Uni, B.Com, November 2001 and 2002, and April2004J
b. What is Management Reporting? [Bangalore Uni., B.Com., May 2001 and 2002J
c. Define a Report. [Bangalore Uni., B.Com., May 2003J
. d. What do you mean by Reporting to Management? [Kuvempu Uni, B.Com, May 2001J
Bibliography
1. Allan R. Dvenbin and Harold Bierman, Jr, "Managerial Accounting", Tokyo, Japan, W.B.
Saunders Co, 3rd Ed., 1978.
2. Anthony. Robert N, "Management Accounting Principles", Illinois, Richard D Irwin Home
Wood, 6th Ed .. 1989.
3. Anthony, R. and D. Young, "Management Control in Non-Profit Organisations", IIIionois,.
Richard D Irwin Home Wood, 4th Ed., 1988.
4. Anthony R. "The Management Control Function", Boston, Harvard Business School Press,
1988.
5. Anthon H.R and Firmrnin P.A. "Management Accounting", Illinois, Richard D Irwin Home
Wood.
6. Batty J. "Management Accounting", London, McDonald & Evans. 4th Ed .• 1974.
7. Bernard Brooks, "Principles of Management Accounting", London. Gee & Co Publishers
Ltd., 1st Ed., 1975.
8. Bhadur Murao, "Management Accounting", Bombay. Meenakshi Prakashan. 2nd Ed.
9. Bhattacharya S.K, and John Deardon. "Accounting for Management - Text and Cases", New
Delhi, Vikas Publishing House Pvt. Ltd., 1st Ed (Reprint), 1979-80.
10. Bierman. Harold Jr, "Financial and Managerial Accounting", New York, Me-Millan
Publishing Co.
11. Bierman, Harold Jr and Dyckman R Thomas. "Managerial Cost Accounting", New York,
McMillan Publishing Co.
12. Brown G.L. and Howard L.R, "Principles and Practice of Management Accounting",
London. The English Language Book Society & Evans Ltd., 3rd Ed., 1966.
13. Bruns W and Kaplan R, "Accounting and Management: Field Study Perspectives",
Boston, Harvard Business School Press, 1987.
14. Camman, Eric A, "Basic Standard Costs", New York, American Institute Publishing Co.
15. Charles T. Horngren, "Introduction to Management Accounting", New Delhi, Prentice
Hall of India Pvt. Ltd., 4th Ed., 1980.
16. Charles T. Horngren, "Management Accounting: Where We Are? Information for
Decision Making", New Jersey, Alfred Rappaport Englewood Cliffs Prentice Hall, 1982.
17. Clancy D. "Annotated Management Accounting Readings", American Accounting
Association, 1986.
18. Colin Drury, "Management and Cost Accounting", London, English Language Book
Society, 2nd Ed., 1988.
Management Accounting : 770

19. Cooper D, Scapens R and Arnold J, "Management Accounting Research and Practice",
London, CIMA, 1983.
20. Dearden J, "Cost and Budget Analysis", USA, Prentice Hall.
21. Dafess Baig, "Problems on Managerial Accounting ", New Delhi.
22. D. Decoster, Eldon L. Schater, "Management Accounting - Decision Emphasis", New
York, John Wiley and Sons, 2nd Ed., 1979.
23. Dopuch and Birnberg, "Cost Accounting - Accounting Data for Management Decision".
24. Drebin Biraman, "Managerial Accounting - An Introduction", London, W.B. Sanders Co,
3rd Ed., 1978.
25. Edey. "Business Budgets and Accoullts".
26. Edward L, "Practical Problems of Budgetary Control", College Publishers. Champaign III.
27. Evans Hemming D.F. "Flexible Budgetary Control and Standard Costs", London.
McDonald & Evans.
28. Garrison Ray H. "Management Accounting".
29. Ghosh and Gupta G.S. "Fundamentals of Management Accounting", New Delhi, National
Publishing House, 3rd Ed .• 1983.
30. Gillespie Cecil. "Standard and Direct Costing", India, Prentice Hall.
3l. Gray and lhonston. "Accollllting and Mallagement Action", New Delhi. Tata McGraw-Hill
Publishing Co., 2nd Ed .• 1979.
32. Guruprasad Murthy. "Management Accounting", Bombay. Himalaya Publishing House 1st '1
Ed .• 1982.
33. Harper W.M, "Mallagement Accounting", M & E Hand Books.
34. Heckert and Wilson. "Business Blldgeting and COlltrol".
35. Henricy. "Standard Costs for M(lIlufactllring".
36. Hingorani N.L, Ramanathan A.R and Grewal T.S. "Management Accounting". New Delhi.
Sultan Chand & Sons.
37. Homer A Black and John E Champion. "Accounting;1/ Business Decisions", USA. Prentice
Hall Inc.
38. Howard and Brown. "Management Ac.counting".
39. CIMA, "Cost Reduction", London. Gee & Co.
40. CIMA. "I11troduction to Budgetary COlltrol, Stalldard Costing Control alld Production
COlltrol", London, Gee & Co.
41. CIMA. "Presentatioll of Infonllatioll to Management", London. Gee & Co.
Bibliography : 771

42. CIMA, "Report on Marginal Costing, Standard Costing and Budgetary Control", London,
Gee & Co.
43. Jack Gray and Kenneth S Johnson, "Accounting and Management Action", New Delhi,
Tata McGraw-Hill Publishing Co., 2nd Ed., 1979.
44. Jagwant Sing and Rantej Paul, "Management Accounting", Allahabad. Kitab Mahal. 4th
Ed., 1985.
45. Jones, Reginald L and Trent H George. "Budgeting: Key to Planning and Control", New
York, McGraw-Hill.
46. Kaplan R and A Atkinson, "Advanced Management Accounting", New Jersey, Prentice
Hall,1989.
47. Kaplan R.S, "Management Accounting in Hospitals: A Case Study into Accounting for
Social Goals: Budgeting and Analysis for Non-market Projects", New York, Harper &
Row, 1981.
48. Keller. "ManagementAccountingfor Profit Control", New York. McGraw-Hill.
49. Khan M.Y and Jain P.K, "Management Accounts", New Delhi, Tata McGraw-Hill
Publishing Company Limited, 1990.
50. Khan M.Y and Jain P.K, "Theory and Problems of Management and Cost Accounting",
New Delhi. Tata McGraw-Hill Publishing Company Limited. 1994.
51. Klemstine C and Maher M. "Management Accounting Research", New York, Garland
Publishing, 1984.
52. Larry N. Killough and Wayne E. Leininger, "Cost Accounting for Managerial Decision
Making", California, Dickenson Publishing Co., 1977.
53. Lawrence F.C, "Marginal Costing", London. McDonald & Evans.
54. Lynch R.M, and Williamson Robert W, "Accounting for Management Planning and
Control", New York, McGraw-Hill.
55. Magee R, "Advanced Managerial Accounting", New York, Harper & Row, 1986.
56. Maheswari S.N .• "Management Accounting and Financial Control", New Delhi, Sultan
Chand & Sons, 10th Ed., 1995.
57. Aan Mohan and Goyal S.N. "Principles of Management Accounting", Agra. Sahitya
Bhavan. 6th Ed., 1992.
58. Moore L C and Jaedicke Robert. "Management Accounting", USA South Western
Publishing Co.
59. Nafees Baig, "Problems of Managerial Accounting", New Delhi. Sterling Publishing Pvt.
Ltd .• 2nd Ed .. 1981.
60. Nagarathnam S. "Management Accountillg, Financial Accounting and Holding Company
Accounting", New Delhi. Sultan Chand & Sons. 9th Ed .. 1980.
Management Accounting: 772
61. Nickerson Clarence W. "Managerial Cost Accounting and Analysis", New York, McGraw-
Hill.
62. Norman Thorton, "Management Accounting", Bombay, Allied Publishers Pvt. Ltd., 1st Ed.,
1975.
63. Norman Thornton, "Solutions Manual to Management Accounting", Bombay, Allied
Publishers Pvt., Ltd., 1978.
64. Pandey LM, "Management Accounting", New Delhi, Vikas Publishing House Pvt., Ltd.,
2nd Ed., 1992.
65. Ray Choudhary H, "Advanced Cost and Management Accounting", Calcutta, Progressive
Publishers Ltd., 2nd Ed., 1970.
66. Ronald J Thacker and Richard L. Smith. "Modem Management Accounting", New Delhi,
Prentice Hall of India Pvt. Ltd .. 1st Ed .. 1978.
67. Rosen L, "Topics in Management Accounting", Toronto, McGraw-Hill, 1984.
68. Sahaf M.A. "Management Accounting - Principles and Practice", New Delhi, Ashish
Publishing House, 1st Ed., 1991.
69. Schattke. Jenson and Bean. "Managerial Accounting", New York. John Wiley & Co.
70. Schalatter. Jenson and Bean. "Managerial Accounting", Boston, Allyn & Bacon Ind, 1st
Ed.
71. Sharma N.K. "Management Accounting", Jaipur, RBSA Publishers, 1993.
72. Srinivasa N.P, "Management Accou1lting", New Delhi. Sterling Publishers, 1st Ed. 1986.
73. Vinayakam N and Sinha LB, "Management Accounting", Bombay, Himalaya Publishing
House, 1st Ed .• 1988.
74. Wayne Keller and William L Ferrara, "Management Accounting for Profit Control", New
Delhi, Tata McGraw-Hill Publishing Co., 1st Ed .. 1979.
Index
Absolute Liquid Assets 106 Budgetary Control 417
Absolute Liquidity Ratio 106 Budgeting 417
Accounting 10 Capital Gearing 122
Accounting Ratios 76 Capital Gearing Ratio 122
Accounting Reports 8 Capital Ratio 120
Accounts Payable 115 Cash 235, 365, 366
Accounts Receivable 103 Cash Budget 433-6
Accrual Concept 366 Cash Flow 365
Acid-test Ratio 105 Cash Flow Statement 367-8
Activity Ratios 107-16 Cash Inflow 365, 369
Actual Cash Flow 365 Cash Outflow 365, 369
Adverse Variance 560 Cash Sales Method 369
All Financial Resources 235 Cash Trading Profit 368
Analysis 26 Causative Relationship 4
Analysis and Interpretation 25, 27 Cent Per Cent Statements 37
Angle of Incidence 686, 688-9 Common-size Balance Sheet 36
Annual Reports 11 Common-size Financial Statements 34-7
Attainable Standards 557 Common-size P&L Account 34
Average Collection Period 112 Common-size Statements 37
Average Method 664 Communication 753
Average Payment Period 115 Comparability 754
Balance Sheet 21 Comparative Common-size Financial
Balance Sheet Ratios 81 Statements Method 36
Base Year 37 Comparative Financial Statements 29-34
Basic Variance 561 Comparison 26, 417
Best Alternative 657 Comparison Method 662
Bills 115 Component %age Statements 37
Bills Payable 115 Composite Break-even Point 677
Bills Receivable 112 Composite PN Ratio 673, 677
Book Costs 369 Concepts 21
Book Debts 112 Consistency 754
Bougette 416 Contribution 671-2
Break-even Analysis 676 Contribution to Sales Ratio 671
Break-even Charts 685-91 Controllable Variance 561
Break-even Level 671 Cost Accounting 7
Break-even Point 671, 676-8 Cost Audit 14
Buck-passing 453 Cost-Volume-Profit Analysis 701-4
Budget 416 Credit Variance 560
Budget Centres 418 Creditors 115
Budget Committee 419 Creditors' Turnover Ratio 115-6
Budget Manual 419 Current Assets 103, 238
Budget Period 420 Current Liabilities 104-5, 238-9
Current Ratio 103-5
Management Accounting : 774

Debit Variances 560 Forecast 417


Debt 120 Fourth Level Ratios 81
Debt Service Ratio 123 Functional Classification 82-3
Debt-Equity Ratio 119-21 Fund 235, 365
Debtors 112 Funds Flow Statement 235-6, 240
Debt-Service Coverage Ratio 124 Funds from Operation 241-6
Decision Making 657 GAAPs23
Depreciation 242 Gang Composition Variance 577-8
Direct Cost 658 Governing Factor 420
Dividend Yield Ratio 125 Gross Margin 84
Du Pont Analysis 98-9 Gross Profit Ratio 84-5
Du Pont Chart 99
Dynamic Analysis 28, 81 High and Low Method 663
Dynamic Financial Report 759 Highly Geared 122
Historical Approach to Costing 555
Earning per Share 102 Historical Costing 555
Equation Method 662 Horizontal Analysis 28, 81
Equity 121 Hundies 115
Equity Ratio 120
Evenly Geared 122 Ideal Standards 557
External Analysis 28 Idle Time Variance 578
External Sources of Cash 368 Importance-wise Classification 81-2
Income Statement 21
Favourable Variance 560 Incremental Cost 658
Financial Ac.;ounting 7, 12 Information 753
Financial Analysis 26 Interest Coverage Ratio 123, 124
Financial Position Ratios 118-25 Internal Analysis 29
Financial Reports 21,759 Internal Reports 8, 24, 756
Financial Statements 21-2 Internal Sources of Cash 368
First Level Ratio 81 Interpretation 26
Fixed Assets Turnover Ratio 118 Inter-Statement Ratios 81
Fixed Budgets 443 Inventory Turnover Ratio 107-10
Fixed Charges Cover 123
Fixed Dividend Cover 124 Key Factor 420
Fixed Dividend Coverage Ratio 124
Fixed OH Calendar Variance 590 Labour Budget 430
Fixed OH Capacity Variance 590 Labour Cost Variance 574
Fixed OH Cost Variance 586 Labour Efficiency Variance 576
Fixed OH Efficiency Variance 589 Labour Mix Variance 577
Fixed OH Expenditure Variance 588 Labour Rate Variance 575
Fixed OH Spending Variance 588 Labour Reports 758
Fixed OH Volume Variance 589 Labour Time Variance 576
Fixed plus OH Expenses 661 Labour Variance 574
Flexible Budgets 443-4 Labour Yield Variance 578
Flow 235 Least-Squares Method 665
Flow of Funds 235 Leverage 122-3
I
Index: 775

Leverage Ratios 118-25 Net Profit 92


Limiting Factor 420 Net Profit Approach 370-1
Line of Best Fit 664 Net Profit Ratio 92-3
~ Liquid Assets 105 Net Worth to Total Assets Ratio 120
Liquid Liabilities 105 Non-current Items 239
Liquid Ratio 105 Non-monetary Factors 657
Liquidity 102 Normal Standards 557
Liquidity Ratios 102-17 Notes Payable 115
Long-term Solvency Ratios 118-25 Notes Receivable 103. 112
Loss Area 686 Notional Cash Flow 365
Low Geared 122
Objectivity 11. 754
Management Accountancy 3 Operating Expenses 34, 87
Management Accounting 2-3, 12-3 Operating Ratio 87-90
Management Audit 14 Overall Cost Reports 758
Management by Exception 9, 453, 558. 559 Overhead Variances 582
Management Reporting 7, 753
Management Reports 757 PlY Graph 691-2
Management Vs Cost Accounting 13-4 PlY Ratio 671
Management Vs Financial Accounting 10-2 Payables Turnover Ratio 115
Management-oriented Accounting 5,8 Percentage 79
Managerial Accounting 3 Period-wise Classification 82
Manufacturing Concerns 108 Perspective Approach to Costing 555
Manufacturing Cost 426 Positive Cash Flow 365, 369
Margin Method 601 Possible Alternatives 657
Margin of Safety 686, 689-90 Practical Standards 557
MargiQal Cost 658-9 Preference Share Capital 121
Marginal Costing 658 Price-Earning Ratio 125
Marginal Income Ratio 671 Primary Ratio 81
Marketing Reports 758-9 Principal Budget Factor 420-1
Master Budget 419.436 Principle of Exception 755
Material Cost Variance 562 Production Budget 423-4
Material Mix 566 Profit 83
Material Mix Variance 566-7 Profit and Loss Account 21
Material Price Variance 563 Profit and Loss Account Ratios 80
Material Quantity Variance 564 Profit Area 686
Material Reports 757 Profit Graph 691
Material Usage Variance 564 Profit Method 601
Material Yield Variance 568-9 Profit to Assets Ratio 95
Mixed Expenses 661 Profitability 83
Modus Operandi 27 Profitability Ratios 83, 95-100
Monetary Factors 657 Profit-Volume Ratio 671
Proportion 78
Near Money Ratio 105 Proposed Dividend 253
Negative Cash Flow 365, 368, 369 Proprietary Ratio 120
Net Cash Flow 365. 367 Provision for Taxation 254
~---:--

Management Accounting : 776

Purchase Budget 427 Standard Costing 555


Pure Ratio 78 Standard Costs 556
Purpose-wise Classification 82-3 Statement of Changes in Financial Position
235-6
Qualitative Factors 657 Static Analysis 28, 81
Quantitative Factors 657 Static Financial Report 759
Quick .Ratio 105-6 Stock Turnover Ratio 107
Range Method 663 Structural Ratios 81
Rate of Return 96 Sub-variances 561
Ratio 76 Super-Quick Ratio 106
Ratio Analysis 76 Supporting Ratios 81
Real Standards 557
Receivables Turnover Ratio 112 Tabular Approach 444
Regression Line 664 Third Level Ratios 81
Relevancy 754 Total Assets Turnover Ratio 118
Report 753 Trading Concerns 107
Restatement Method 245 Trading on Equity 122-3
Retained Profit Method 245 Trend Analysis 28, 37
Return on Capital Employed 96 Trend Percentages 37
Return on Investment 96 Trend Ratios 37, 81
Return on Net-worth Ratio 99
Return on Proprietors' Fund 99 Uncontrollable Variance 561
Return on Shareholders' Equity 99 Unfavourable Variance 560
Return on Total Assets 95 Unit Contribution 671
Routine Reports 757
Value Variance 596
Sales Budget 423 Value Variance 596
Sales Mix Variance 600 Value Variance 596
Sales Price Variance 597 Variable Costs 660-1
Sales Quantity Variance 599 Variable OH Controllable Variance 584
Sales Revenue Variance 596 Variable OH Efficiency Variance 585
Sales Variance 596 Variable OH Expenditure Variance 584
Sales Volume Variance 598 Variable OH Spending Variance 584
Scatter-Graph Method 664 Variable Overhead Variances 584
Semi-variable Costs 661 Variance 560
Semi-variable OH Expenses 661 Vertical Analysis 28
Short-term Solvency Ratios 102-17
Simple Ratio 78 Wage Rate Variance 575
Simplicity 754 Wages Variance 574
Simplified Break-even Charts 691 Window Dressing 25
. Sources and Applications of Fund 240-56 Working Capital 33, 235, 366
Source-wise Classification 81-2 Working Capital Ratio 103
Special Reports 757 Working Capital Turnover Ratio 118
Specific Explanatory Ratios 81 . Works Cost 426
Works-on-Cost 426

You might also like